Wochenaufgabe Mathe

Aufgabe der Woche

Serie 32

Serie 32

Aufgabe 1

373. Wertungsaufgabe

Logikaufgabe
Mike und Bernd waren in den Ferien in Wien. Sie trafen sich dort in der Jugendherberge mit fünf Geschwisterpaaren – immer ein Junge mit dessen Schwester. Das waren Heinrich, Arne, Linus, Marcus und Tobias. Die Mädchen hießen Elli, Mira, Line, Melanie und Isabell. Jedes Geschwisterpaar hatte ein Zimmer für sich. (Nummer 1, 2, ..., 5). Die Türen der Zimmer hatten alle eine andere Farbe (rot, blau, grün, gelb bzw. grau). Wer wohnte in welchem Zimmer und welche Farbe hatten die Türen? 6 blaue Punkte
1. Die Zimmernummer von Heinrich und Elli war um eins höher als die des Paares, dessen Tür grün ist.
2. Isabells Zimmer hatte eine gelbe Tür, aber nicht die Nummer 5.
3. Die Zimmertür mit der Nummer 4 war grau, aber da war nicht die Mira untergebracht.
4. Arne war im Zimmer 3.
5. Markus' Zimmernummer war um zwei größer als die Nummer auf der blauen Zimmertür.
6. Tobias war im roten Zimmer untergebracht. Der Name seiner Schwester ist kürzer als der Name der Schwester von Linus.
Die  Geschwisterpaare sind – wie Bernd und Mike auch – sehr sportlich.
Die Mädchen aber spielen alle auch je ein Instrument. (Geige, Harfe, Klavier, Flöte bzw. Oboe). Die Instrumente spielen sie recht unterschiedlich lange – 3 Monate, 9 Monate, 2 Jahre, 3 Jahre und in einem Fall sogar schon 5 Jahre. „Woher kamen die Paare denn?“, fragte Bernds Opa. Es sind die Städte Linz, Passau, Chemnitz, Bamberg und München.
Wer spielt welches Instrument, übt wie lange und wohnt in welcher Stadt? 6 rote Punkte
1. Melanie wohnt in Passau. Sie übt schon länger als die Harfenspielerin.
2. Isabell spielt Klavier und wohnt nicht in Linz. Sie übt noch nicht so lange wie das Mädchen, welches Oboe spielt, aber nicht Melanie heißt.
3. Das Mädchen, welches schon 3 Jahre spielt, wohnt weder in Passau noch in Chemnitz.
4. Mira übt seit 2 Jahren.
5. Das Mädchen aus München übt seit 5 Jahren.
6. Das Geige spielende Mädchen wohnt in Chemnitz.

Termin der Abgabe 22.11.2012 Deadline for solution is the 22th. november 2012.

logic puzzle
373
Mike and Bernd had spent their holidays in Vienna. In the youth hostel they met 5 pairs of siblings. Always a boy and his sister. There were Heinrich, Arne, Linus, Marcus and Tobias. The girls were Elli, Mira, Line, Melanie and Isabell. Each pair of siblings had a room for themselves (numbers 1 … 5). The dors to these rooms each had a different colour (red, blue, green, yellow and grey). Who lived in which room and what colours were the doors? - 6 blue points
1. The room number of Heinrich and Elli was one higher than the one of the pair whose door was green.
2. Isabell's room had a yellow door but didn’t have the number 5.
3. Door number 4 was grey, but didn't belong to Mira's room.
4. Number 3 was Arne's room.
5. Markus' room number was two higher than the one with the blue door.
6. Tobias stayed in the red room. The name of his sister is shorter than the one of Linus' sister.
The pairs of siblings are – as Bernd and Mike – very keen on sports. However, each of the girls also plays an instrument (violin, harp, piano, flute and oboe). They have been playing their instruments for different periods of time – 3 months, 9 months, 2 years, 3 years and in one case for even 5 years.
“Where do the pairs come from?”, Bernd's granddad asked. They are from different towns: Linz, Passau, Chemnitz, Bamberg and Munich.
Who plays which instrument for how long and where does she live?
1. Melanie is from Passau. She has been practising her instrument for longer than the harp player.
2. Isabell plays the piano and isn't from Linz. She hasn't been practising as long as the girl who plays the oboe who isn't Melanie.
3. The girl who has been playing her instrument for three years now does neither live in Passau nor in Chemnitz.
4. Mira has been practising for 2 years.
5. The girl from Munich has been practising for 5 years.
6. The violin player is from Chemnitz.

Lösung/solution:
Blau:
Heinrich Elli blau 2
Arne Isabell gelb 3
Linus Melanie grün 1
Marcus Line grau 4
Tobias Mira rot 5
Rot:
1. Lösung
Mira Geige Chemnitz 2 Jahre
Elli Harfe Linz 3 Monate
Isabell Klavier Bamberg 3 Jahre
Melanie Flöte Passau 9 Monate
Line Oboe München 5 Jahre
2. Lösung
Mira Geige Chemnitz 2 Jahre
Line Harfe Linz 3 Monate
Isabell Klavier Bamberg 3 Jahre
Melanie Flöte Passau 9 Monate
Elli Oboe München 5 Jahre



Aufgabe 2

374. Wertungsaufgabe
„Hallo Lisa, was bastelt du denn?“, fragte Mike, als er sah, dass sie quadratische Blätter faltete. „Ich falte  das Blatt so, dass ich ein Quadrat erhalte, das genau den halben Flächeninhalt hat wie mein Blatt vorher. Schau mal, das ist ganz einfach.“ Wie kann das Blatt gefaltet werden? 2 blaue Punkte und wieso kann man dann sicher sein, dass der Flächeninhalt wirklich halbiert wurde? + 2 blaue Punkte. Bernd kommt dazu und faltet ein Blatt (Quadrat ABCD) so, dass der Punkt B genau
auf die Mitte der Strecke CD zu liegen kommt. Die Knickkante streicht er glatt und nun sieht er drei rechtwinklige Dreiecke. Ein großes mit dem rechten Winkel bei D, ein mittleres mit dem rechten Winkel bei C und ein kleines dort, wo das gefaltete Teil über die Kante D nach A ragt. „Ob die drei  rechtwinkligen Dreiecke vielleicht sogar zueinander ähnlich sind?“, überlegt Mike. Für den Nachweis, dass alle drei Dreiecke zueinander ähnlich sind oder aber auch nicht – gibt es 6 rote Punkte.

Termin der Abgabe 29.11.2012 Deadline for solution is the 29th. november 2012.

“Hi Lisa, what are you making?”, Mike asked when he saw her folding square sheets of paper.
“I'm folding this sheet of paper so that I get a square that is exactly half the area of the original sheet. Look, it's really easy.”
How is the sheet folded? - two blue points. Why can she be sure that the area is really halved? - another 2 blue points.
Bernd arrives and folds a sheet (square ABCD) so that point B meets the centre of line segment CD. He smooths down the fold and now has three right triangles: A big one with the right angle at D, a medium one with the right angle at C and a small one where the folded part protrudes beyond the edge from D to A.
“I wonder if these right triangles are similar to each other?” Mike ponders.
Show that they are (not) – 6 red points
Lösung/solution:
Die Lösung für blau und rot von Jürgen Urbig, danke.
Gezeigt wird hier auch noch, dass die Dreiecke bei rot sogenannte "ägyptische Dreiecke" sind, das heißt die Seiten der Dreiecke verhalten sich wie 3:4:5. Der Inhalt der roten Aufgabe ist auch bekannt als der (1.) Satz von Haga.
als pdf




Aufgabe 3

375. Wertungsaufgabe
„Habt ihr schon von den Zauberpunkten beim Falten gehört?“, fragte Bernds Großvater, als er die letzte Aufgabe von Maria gezeigt bekam. Allgemeines Kopfschütteln. „Passt auf. Ich nehme zum Beispiel dieses quadratische Stück Papier (ABCD – 10 cm groß). Ich markiere auf der Kante CD einen Punkt X, der zwei cm von D entfernt ist. Zuerst falte ich B zu dem Punkt X und streiche die Faltlinie glatt. Jetzt klappe ich den umgefalteten Teil wieder zurück. Nun falte ich den Punkt A auf den Punkt X und streiche die Faltlinie glatt. Wenn ich wieder das Quadrat vor mir liegen habe, sehe ich einen Punkt M – den Schnittpunkt der beiden Faltlinien. Das ist der Zauberpunkt.“ Wieder allgemeines Kopfschütteln. „Nehmt mal einen Zirkel, stecht in M ein und nehmt die Entfernung zu X in den Zirkel. Wenn ihr es richtig gemacht habt, dann geht der Kreis durch X, aber auch durch A und B.“ „Cool, das ist ja Zauberei!“ Für die Bestimmung des Radius des „Zauberkreises“ durch Falten und Abmessen – 2 blaue Punkte. Bei Berechnung des Radius werden es 5 blaue Punkte. Der Nachweis, dass die Zauberei für jede Lage des Punktes X auf der Kante CD funktioniert, bringt 5 rote Punkte.

Termin der Abgabe 06.12.2012 Deadline for solution is the 6th. december 2012.

375
“Have you ever heard of the magic points when folding paper?”, Bernd's granddad asked when Maria showed him the last problem. Everyone shook their heads.
“Look, I've got this square piece of paper (ABCD . 10 cm). I'm marking a point X on side CD 2 cm away from D. Now I'm folding B to meet X and smooth the fold well. Fold back. Now I fold point A to X, smooth the fold and fold back. The intersections of the two folds is the magic point.” Again, collective shaking of heads.
“Take your compass and place the point of your compass at M and draw a circle through X. If you did everything as I said, the circle not only passes through X, but also through A and B.”
“Magic!”
Determine the radius of the “magic circle” by folding and measuring – 2 blue points. If you calculate the radius – 5 blue points.
Show that the magic works for any position of X on side CD – 5 red points.
Lösung/solution:
375 k
--> Bild groß <--
Günstig ist es nicht nur die "Knickfalten" zu haben, sondern auch die Strecke von X nach A bzw. C. Die Knickfalten sind dann die Mittelsenkrechten der Strecken. M ist somit der Schnittpunkt der Mittelsenkrechten des Dreiecks ABX. Dieser Schnittpunkt ist aber automatisch der Mittelpunkt des Umkreises eines Dreiecks. Somit müssen X, A und B auf einem Kreis liegen und das unabhängig von der Lage von X (zwischen C und D natürlich). Die Berechnung für auf r rund 6,5 cm. Verwendet werden kann die Tatsache der Halbierung der Seiten und des Senkrechtstehens der entsprechenden Funktionsbilder. Verwendet wurde aber die Formel für den Umkreisradius.




Aufgabe 4

376. Wertungsaufgabe

Mike hat aus regelmäßigen gleichgroßen Sechsecken ein Spielfeld gelegt und würfelt. 
376 „Was machst du denn da?“, fragt Lisa. „Schau mal, ich habe ein Startfeld. Alle Felder haben die gleiche Kantennotation (z. B. Norden die 1, Nordost die 2, …) Je nach Zahl, die ich würfele, verlasse ich das Feld und erreiche ein benachbartes Feld. Jetzt würfele ich wieder und verlasse das Feld wieder in die Richtung, die mir die gewürfelte Zahl vorgibt.“ „Ach, ich verstehe. Und was probierst du nun genau?“ „Ich versuche, mein Ausgangsfeld einmal zu umrunden und dann wieder dort anzukommen.“ Welche Wurfergebnisse müsste Mike erwürfeln, wenn die erste Zahl eine 6 ist und er mit einer minimalen Anzahl von Würfen seine Aufgabe erfüllen möchte.- 3 blaue Punkte Wie groß ist die Wahrscheinlichkeit für die Lösung(en?) von blau – 4 rote Punkte. Wie oft muss Mike würfeln, damit die Wahrscheinlichkeit, das Ausgangsfeld wieder zu erreichen, über 50 % liegt (ohne Umrundungszwang 3 rote Punkte, mit Umrundungszwang 5 rote Punkte) – das Spielfeld wird als unendlich groß angenommen.

Termin der Abgabe 13.12.2012 Deadline for solution is the 13th. december 2012.
 
Mike has laid out a game board using regular hexagons of equal size. Now he's throwing the dice.
376“What are you doing?”, Lisa asks.
“Look, here's the starting field. All hexagons are equally orientated (e.g. north = 1, north-east = 2, … ). Depending on what number I throw I leave one hexagon in a certain direction and arrive at the next. The I throw the dice again and move to the next field.”
“Ok, I see. And what exactly are you trying to achieve?”
“I try to go around my starting field and arrive back there.”
Which numbers would Mike have to throw if his first number was a 6 and he wanted to achieve his goal with a minimum of throws? - 3 blue points. What probability does this event have? - 4 red points.
How often would Mike have to throw the dice so that the probability to arrive back at the starting point is more than 50%? - 3 red points if he does not necessarily has to go round the starting field, 5 red points if he must do so.
Suppose an infinite game board.
Lösung/solution:

Spielfeld zum Probieren (pdf)

wird nachgereicht


Aufgabe 5

377. Wertungsaufgabe
„Neulich habe ich in einem Film gesehen, wie ein Junge an einem Automaten ein Computerspiel spielt. Mit seinem Joystick konnte er sein Auto nach links und rechts, aber auch nach oben und unten bewegen,“ sagte Bernd zu Mike. „Ja, so etwas kenne ich, das hatte mein Vater für seinen Atari 800.“ „Atari 800, das muss ja schon lange her sein, aber wird schon stimmen, denn der Film war aus dem Jahr 1988, das passt.“ Das Auto im Spiel war 0,8 cm groß und brauchte auf dem 50 cm breiten Spielfeld des Computers 2 Sekunden von links nach rechts. Wie schnell ist das Auto (in km/h) im Computerspiel? 3 blaue Punkte. Wenn das Auto im Spiel immer nach oben bewegt wird, taucht es nach dem Verlassen des Spielfeldes unten wieder auf. Bewegt man das Auto immer nach rechts, so taucht es nach dem Verlassen des Spielfeldes links wieder auf.  Kann es eine solche nicht kugelförmige Spielfläche auch in der Realität geben? Wenn ja, wie könnte diese aussehen, wenn nein, warum gibt es eine solche Spielfläche nicht. 4 rote Punkte

Termin der Abgabe 20.12.2012 Deadline for solution is the 20th. december 2012.
 
“The other day in a film I saw a boy playing a video game on an arcade machine. He used a joystick to move a car up and down, left and right,” Bernd said to Mike.
“Yes, I know that. My father used to have something like this for his Atari 800.”
“Atari 800, that's must have been some time ago, but it's probably right, because this film was from 1988, it fits.”
The car in the video game had a size of 0.8 cm. It took 2 seconds to move the car from left to right on the 50 cm wide screen. What is the speed of the car in the game in km/h? - 3 blue points
If you move the car only up and out of the screen it will reappear at the bottom of the sreen. Likewise if you move right over the edge, it will appear on the left side of the screen. Is such a non-spherical game field possible in real life. If yes, what could it look like, if no, why not. - 4 red points

Lösung/solution:
blau rund 0,9 km/h.
rot: bei der Fläche handelt es sich um einen Torus, also so etwas wie ein Donut.



Aufgabe 6

378. Wertungsaufgabe
„Du hattest doch neulich in einem (rechtwinkligen) Koordinatensystem ein regelmäßiges Sechseck konstruiert“, sagte Mike zu Bernd. „Stimmt, ich hatte die Punkte A (0; 0) und B (4; 1) vorgegeben und daraus das Sechseck ABCDEF (positiver Umlaufsinn) konstruiert.“ „Genau, das habe ich auch gemacht und dann habe ich die jeweils benachbarten Punkte des Sechsecks ABCDEF durch Geraden verbunden.“
„Du hast also die Seiten verlängert?“
„Genau, diese Geraden schneiden sich dann paarweise außerhalb des Sechsecks in 6 Punkten, die sich wiederum zu einem regelmäßigen Sechseck verbinden lassen.“ 4 blaue Punkte gibt es für die Koordinaten der Punkte C', D', E' und F' des zu konstruierenden Sechsecks A'B'C'D'E'F'.
Wie viel mal größer (Flächeninhalt) ist das Sechseck von Mike im Vergleich zum Sechseck von Bernd. 4 rote Punkte.

Termin der Abgabe 03.012.2013 Deadline for solution is the 3th. january 2012.
 
“You constructed a regular hexagon in a coordinate system a short while ago, didn't you?” Mike said to Bernd.
“I sure did. I started with the points A(0;0) and B(4;1) and constructed the hexagon ABCDEF (anti-clockwise).”
“Exactly. That's what I did, too. And then I joined neighbouring vertices by a straight line.”
“Instead of just line segments, you mean.”
“Exactly, because these lines meet in 6 points outside the hexagon which are the vertices of another regular hexagon.” - 4 blue points for the coordinates C', D', E' and F' of the new hexagon A'B'C'D'E'F'.
By what factor does the area of Mike's hexagon exceed the one constructed by Bernd? - 4 red points.
Lösung/solution:
Die Koordinaten der Punkte kann man dem Bild entnehmen:378
-->Bild groß<--
Eine Konstruktion nur mit Zirkel und Lineal ist auch nicht schwierig. Es wird ein gleichseitiges Dreieck mit der Länge der Strecke AB konstriert, dessen dritter Punkt (im 1. Quadraten wählen) wird zum Mittelpunkt M eines Kreises, auf dem die Punkte des kleinen Sechsecks liegen. Der Mittelpunkt M ist - wie man leicht sieht auch der Mittelpunkt des großen Sechsecks. Die 12 Dreiecke die zwischem kleinen und großen Sechseck enstehen haben alle den gleichen Flächeninhalt. (Gleiche Seitenlänge und Höhe lässt sich zeigen) Im kleinen Sechseck gibt es 6 genau so große Dreieecke. Also besteht das große Sechseck aus insgesamt 18 flächengleichen Dreiecken, damit genau 3 mal mehr wie das kleine Sechseck.
Agroß : Aklein = 3 :1 Nimmt ein n-Eck (ungleich 6, n>4), so ist das Verhältnis zwischen großem und kleinen n-Eck nicht mehr ganzzahlig.



Aufgabe 7

379. Wertungsaufgabe
379 k„Ist das eine neue Briefmarke für deine Sammlung?“, fragte Mike. „Schau doch mal genau hin“, sagte Lisa, „auf dem Brief siehst du eine Palindromzahl.“ „Stimmt die 21022012 kann man von links nach rechts und umgekehrt lesen, ohne dass diese sich ändert.“ Der 21. Februar 2012 wird in vielen Ländern unter Weglassung von Punkten oder Strichen so geschrieben, dass die Palindromzahl entsteht. 2 blaue Punkte für die Daten der beiden letzten 8-ziffrigen Palindromdaten vor 2012. 2 rote Punkte für die nächsten beiden 8-ziffrigen Palindromdaten nach 2012.

Termin der Abgabe 10.01.2013 Deadline for solution is the 10th. january 2013.

379
379 k
 “Is that a new stamp for your collection?”, Mike asked.
“You have to look closely”, Lisa said, “there is a palindromic number on the letter.”
“You're right, you can read 21022012 from left to right and vice versa without changing it.”
In many countries the 21st of February is noted without dots or hyphens in a way that makes such a palindromic number. 2 blue points for the last two palindromic dates of eight digits before 2012. 2 red points for the next two palindromic dates of eight digits after 2012.
Lösung/solution:
blau : 11.02.2011,    01.02.2010, 20.02.2002 , 10.02.2001 (Geburtstag von Felix H. aus der Klasse 5a) Noch weiter zurück, da ist es dann schon sehr lange her. 29.11.1192
rot: 02.02.2020, 12.02.2021, 22.02.2022,



Aufgabe 8

380. Wertungsaufgabe
„Bernd kannst du helfen?“, fragte Maria, „für unsere Mathematikgruppe will ich aus diesem 20 cm x 30 cm großen Blatt ein möglichst großes Netz eines Quaders (a x b x c) ausschneiden. 380 „Verstehe. Besser aber ist vielleicht, du gibt den Mitgliedern deiner Gruppe diese Aufgabe. Vorschlag: Eine der Kanten a (6,0 cm) des Netzes soll an der schmalen Seite des Blattes liegen. Der Rest ergibt sich dann (fast) von selbst.“, da hast du Recht“, sagte Maria.
4 blaue Punkte für das Volumen des Quaders, dessen Netz aus dem Blatt ausgeschnitten werden kann. 6 rote Punkte für das Netz, die Maße, eines Quaders, dessen Volumen maximal sein soll und dessen Netz auf das Blatt passt.

Termin der Abgabe 17.01.2013 Deadline for solution is the 17th. january 2013.
 
“Bernd, can you help?”, Maria asked, “I'd like to cut the net of a cuboid (a x b x c) from this single sheet of paper for our maths group. The sheet is 20cm x 30cm and the net should be as big as possible.” 380
„I understand. But perhaps it's better to put this problem to your maths group. Here's a suggestion: One of the sides of the net (a = 6cm) should be at the smaller side of the sheet. The rest will follow more or less automatically.“
“You're right”, Maria answered.
4 blue points for the volume of the cuboid whose net can be cut from the sheet of paper. 6 red points for the net of a cuboid whose volume is at the maximum.
Lösung/solution:
Lösung mittels --> Applet <--



Aufgabe 9

381. Wertungsaufgabe
„Ich habe auch mal eine Fadengrafik → Link ← erstellt“, sagt Lisa zu Maria. „Die sieht gut aus. Wie ich sehe, hast du im Koordinatensystem den Punkt (0;1) mit (10; 0), dann den Punkt (0;2) mit (9; 0), usw. zum Schluss (0;10) mit (1; 0) verbunden.“ „Stimmt genau.“

Die längste und die kürzeste „Fadenlänge“ - Verbindungsstrecken - sind zu bestimmen (Messung 3 blaue Punkte oder Berechnung 5). Die „Fäden“ der Grafik bilden letztlich eine gekrümmte Linie. Kreis?, Hyperbel?, Parabel? …? 5 rote Punkte
Termin der Abgabe 24.01.2013. Deadline for solution is the 24th. january 2013.
 
“Once I did a stitching card pattern→ Link ←”, Lisa told Maria.
“Looks nice. I see you used a coordinate system and connected points (0;1) with (10;1) and then (0;2) with (9;9) and so on until you finally connected (0;10) with (1;0).”
“Exactly.”
Find the shortest and the longest of these connections either by measuring them (3 blue points) or calculating (5 blue points).
The lines on the stitching card are tangents of a curved line. Circle? Hyperbola? Parabola? …? - 5 red points for a complete solution.
Lösung/solution:
Blau lässt sich durch eine entsprechende Konstruktion lösen. Genau so schnell geht es mit der Verwendung des Satzes des Pythagoras: Die längste Strecke (zwei mal vorhanden) ist rund 10,1 cm lang, die kürzeste Strecke (zwei mal vorhanden) ergibt sich zu 7,81 cm.
Lösung zu rot. Es handelt sich bei der gesuchte Kurve, auch Hüllkurve = Envolute, um eine Parabel.
Link zu einer Lösung



Aufgabe 10

382. Wertungsaufgabe
"Hallo Mike, du sitzt ja schon wieder vor einem Koordinatensystem. Was hast du vor?", fragt Bernd. "Ich will so eine Art Spirale konstruieren." "Klingt interessant und wie machst du das?" "Ich zeichne in das Koordinatensystem Geraden durch den Punkt (0; 0). Die erste Gerade ist
die x-Achse, dann im positiven Umlauf weitere Geraden, die um 30° gedreht werden. Nun zeichne ich den Punkt A (1; 0) ein.  Jetzt zeichne ich eine Senkrechte nach oben, bis ich auf die zweite Gerade treffe, dort entsteht der Punkt B. Dann zeichne ich in B eine Senkrechte, die die nächste Gerade im Punkt C trifft. Die Senkrechte im Punkt C trifft auf die y-Achse im Punkt D na und so weiter." "Stimmt, das wird eine Spirale." meint Lisa, die zu den beiden ins Zimmer getreten war.
Ein blauer Punkt für den Namen des Punktes, wenn die Spirale wieder auf die x-Achse trifft (+ 2 für dessen Koordinaten) und zwei blaue Punkte für die Lagebeschreibung des Punktes Z. Welche Koordinaten hat der Punkt Z? - 6 rote Punkte

Termin der Abgabe 31.01.2013. Deadline for solution is the 31th. january 2013.

382
“Hi Mike, you're sitting in front of a coordinate system, again. What are you up to?”, Bernd asked.
“I'd like to construct a kind of spiral.”
“Sound interesting. And how are you going to do it?”
“I draw straight lines into the coordinate system all through point (0;0). The first of these straight line is the x-axis and then, anti-clockwise more straight lines each turned 30 degrees. Now I mark point A (1;0). Then I draw a perpendicular to the x-axis which intersects the second of my straight lines in B. From there draw another perpendicular to intersect the next straight line in C. The perpendicular from point C will intersect the y-axis in point D and so on.”
“You're right. This becomes a spiral.” Lisa says when she enters the room.
“What's the denotation of the point where the spiral intersects the x-axis again – one blue point, what are it's coordinates – another 2 blue points -  and where will point Z be? - 2 blue points
What coordinates does point Z have? - 6 red points
Lösung/solution:
Die Lösung von U. Parsche, danke -->als pdf<--



Aufgabe 11

383. Wertungsaufgabe
Bernd sitzt am Computer und erforscht KFZ-Kennzeichen. „Das hat aber noch Zeit, bis du selber Auto fahren darfst“, sagt seine Schwester Maria. „Das stimmt schon, aber ich habe mal versucht, ein einfaches System zu finden, das ist mir nicht gelungen. Schau mal hier die Informationen für
Bamberg. Jedes Kennzeichen fängt mit BA an. Dann kommt ein Buchstabe mit bis zu vier Ziffern oder zwei Buchstaben mit bis zu 2 Ziffern. Es gibt keine führenden Nullen. Die Buchstaben B, F, G, I, O und Q dürfen nicht dabei sein. Ebenso ist HJ, KZ, NS, SA, und SS verboten.“ Du hast Recht,
das ist nicht so einfach.“ Wie viele Kennzeichen sind in Bamberg möglich? 6 blaue Punkte.
Wie lang (Anzahl der Zeichen) müssten Kennzeichen mindestens sein, wenn man weltweit einheitliche Kennzeichen verwenden würde. (26 Buchstaben des lateinischen Alphabets und Ziffern 0 bis 9) . Geschätzte Anzahl der Fortbewegungsmittel (mit Reserven der Reservierung) 7.000.000.000. (4 rote Punkte)

Termin der Abgabe 21.02.2013. Deadline for solution is the 21th. february 2013.
383

Bernd is at the computer researching German vehicle registration plates.
“There is still some time before you'll be allowed to drive a car”, his sister Maria rmarked.
“I know, I only tried to find a simple system. I didn't succeed. Look at the information about Bamberg. Each number plate starts with BA. The there is a letter and up to four digits or two letters followed by up to two digits. There are no left-hand zeros. Letters B, F, G, I, O and Q are not allowed. Likewise HJ, KZ, NS, SA and SS mustn't be used.”
“You're right, it's not that easy.”
How many registration codes are possible in Bamberg? - 6 blue points
How long (number of characters) would number plates have to be at least, if we used standardized codes worldwide? (26 letters of the Latin alphabet and digits 0 to 9). Estimated number of vehicles to be registered: 7,000,000,000 (with reserves). - 4 red points
Lösung/solution:
Die "blaue" Lösung von Linus, danke. --> als pdf <--
rot: Wenn man alle Fahrzeuge einfach nur durchnummeriert braucht man 10 Zeichen, denn die 7 Milliarden ist 10stellig. (Da gebe es noch viel "Platz" nach oben). Bei Buchstaben lässt sich überlegen. 1 Buchstabe 26 Möglichekiten 2 Buchstaben 26*26 = 26² = 676  Möglichkeiten, drei Buchstaben 26*26*26= 26³ Möglichkeiten. ...  mit 267 überschreiten man die 7 Milliarden. Also 7 Zeichen aus, um alle Fahrzeuge weltweit eindeutig zu kennzeichnen. (Für Schnellrechner: ermittle x in 26x >= 7000 000 000 --> x = log 7000 000 000/log 26.



Aufgabe 12

384. Wertungsaufgabe
Bernd trifft Mike im Treppenhaus der Schule. „Was ist denn mit dir? Musst du zur Strafe 100 mal die Treppe hoch und runter?“, fragte Bernd. „Nein, ich prüfe etwas nach. Die Treppenstufen sind von unten nach oben durchnummeriert. Beim Hochsteigen benutze ich entweder jede Stufe, ich darf auch auch eine auslassen – aber nicht mehr. Wenn ich nun drei Stufen habe, so gehe also Stufe 1, Stufe 2, Stufe 3 oder Stufe 2, Stufe 3 oder aber Stufe 1, Stufe 3. (kurz 1-2-3, 2-3 oder 1-3) Es sind also drei Möglichkeiten, wie ich eine dreistufige Treppe gehen kann.“ „Verstehe und nun arbeitest du dich so langsam nach oben.“

Wie viele Möglichkeiten des Treppensteigens gibt es, wenn es 5 Stufen sind? - 3 blaue Punkte

Wie viele Möglichkeiten gibt es, wenn die Treppe n Stufen hat? 4 rote Punkte.

Termin der Abgabe 28.02.2013. Deadline for solution is the 28th. february 2013.

384

Bernd meets Mike in the school's staircase.
“What's the matter with you? Did someone punish you to climb up and down the stairs for 100 times?”, Bernd asked.
“No, I'm verifying something. The steps are numbered from bottom to top. When I go up I either use each step or I may leave one out, but not more. So going up three steps means I could take step 1, step 2, step 3 or step 2, step 3 or step 1, step 3. In short: 1-2-3, 2-3 or 1-3. There are three possibilities to walk a three step stair.”
“I understand. And now you're slowly working your way up.”
How many possibilities are there to climb a 5-step stair? - 3 blue points
How many possibilities are there to climb a n-step stair? - 4 red points
Lösung/solution:
Hier die Lösung in Form einer Geschichte, dann an XXX, als doc.
Mehr zu den Fibonacci-Zahlen ist auch im Mathematiklexikon zu finden -eine Formal zur Berechnung inklusive.


Ergebnisse der Serie 32 blaue Punkte

Auswertung Serie 32 (blaue Liste)

Platz Name Ort Summe Aufgabe
373 374 375 376 377 378 379 380 381 382 383 384
1. Linus-Valentin Lohs Chemnitz 50 6 4 5 3 3 4 2 4 5 5 6 3
1. Uwe Parsche Chemnitz 50 6 4 5 3 3 4 2 4 5 5 6 3
1. Rafael Seidel Chemnitz 50 6 4 5 3 3 4 2 4 5 5 6 3
2. Doreen Naumann Duisburg 48 6 4 3 3 3 4 2 4 5 5 6 3
3. Gunnar Reinelt Chemnitz 45 6 4 3 3 3 4 2 3 5 5 4 3
4. Karolin Schuricht Chemnitz 31 6 4 - - - 4 2 - 5 4 6 -
5. Felicitas Güra Chemnitz 29 - 2 5 3 - - - - 5 5 6 3
5. Laura Schlosser Chemnitz 29 6 4 5 - - 3 2 4 5 - - -
6. Valentin Grundmann Chemnitz 28 6 - - 3 3 4 2 - 5 5 - -
7. Melanie Petz Chemnitz 25 - 2 2 3 - 4 2 4 3 5 - -
8. Heinrich Grossinger Chemnitz 24 - 2 2 3 3 4 2 - 5 - - 3
8. Elisa Parsche Chemnitz 24 6 4 5 3 - - 2 4 - - - -
9. GesaH Chemnitz 23 6 4 5 - - 3 2 - 3 - - -
9. Tom Straßer Chemnitz 23 - 4 2 3 - - 2 - 4 5 - 3
9. Marie-Christin Müller Erlenbach 23 - - - - - - - 4 5 5 6 3
10. Adrian Schlegel Chemnitz 21 6 - 2 3 - - 2 - - 5 - 3
11. Felix Helmert Chemnitz 20 - 4 2 3 3 - 2 4 - - - 2
12. Andree Dammann München 19 - - - - 3 4 2 - 5 5 - -
12. Mara Neudert Chemnitz 19 6 4 5 - - - 2 2 - - - -
13. Pauline Marschk Chemnitz 18 - 4 5 - - - - 4 5 - - -
13. Camilla Schreiter Chemnitz 18 - 4 5 - - - - 4 5 - - -
13. Martina Bausch Waldshut 18 6 4 - - 3 - 2 - - 3 - -
13. Jonas Frederik Otto Lichtenwalde 18 - 4 2 3 - - - 4 5 - - -
13. Sabine Fischbach Hessen 18 6 4 2 - - - - - - - 6 -
14. Felicitas Hastedt Chemnitz 17 - 2 2 3 - - 2 - 3 5 - -
14. Lena Rabbeau Chemnitz 17 6 2 - - - - - 4 5 - - -
15. Nele Mäding Chemnitz 16 - 4 - - - - - 4 5 - - -
15. Luisa Schlosser Chemnitz 16 6 - 5 - - 3 2 - - - - -
15. Line Mauersberger Chemnitz 16 - 4 - 3 - 4 2 - 3 - - -
15. Paul Arwed Guhlmann Chemnitz 16 - 4 2 3 - - 2 2 3 - - -
15. Johanna Börner Chemnitz 16 - - 2 3 3 - - 3 2 - - 3
16. Robin Seerig Chemnitz 15 6 4 - 3 - - - - - - - 2
17. Sophie Kalmer Chemnitz 14 - 2 - 3 - - - 4 5 - - -
17. Peye Mäding Chemnitz 14 - 4 2 3 - - - 2 3 - - -
17. Christian Wagner Bamberg 14 6 - - - - - 2 - - - 6 -
17. Pascal Graupner Chemnitz 14 - 4 2 - - - - 3 5 - - -
17. Saskia Schlosser Chemnitz 14 6 4 - - - - 2 2 - - - -
18. Marcel Seerig Chemnitz 13 - 2 2 - - - - 4 5 - - -
18. Florian A. Schönherr Chemnitz 13 - 4 2 - - - - 2 5 - - -
19. Felix Schrobback Chemnitz 12 - 4 2 3 - - - - 3 - - -
19. Ida Gwendolin Eichler Chemnitz 12 - - - 3 - - - 4 5 - - -
19. Christin Reichelt Chemnitz 12 - 4 2 3 - - - - 3 - - -
20. Rebecca Wagner Oberwiesenthal 11 6 - - - - - 2 - 3 - - -
21. Katrin Wolstein Bamberg 10 6 4 - - - - - - - - - -
21. Paula Geißler Chemnitz 10 - - 2 3 - - - - 5 - - -
21. Emma Irmscher Eibenberg 10 6 - 2 - - - 2 - - - - -
21. Elina Rech Chemnitz 10 - - - 3 - - 2 - - 5 - -
21. Elias Schmidt Chemnitz 10 - 3 2 - - - - - 5 - - -
21. Jürgen Urbig Chemnitz 10 6 4 - - - - - - - - - -
21. Jonathan Schlegel Chemnitz 10 - 2 2 3 1 - 2 - - - - -
22. Ulrike Böhme Chemnitz 9 - - 2 3 - - 2 - 2 - - -
22. Nadja Richter Chemnitz 9 - 4 2 3 - - - - - - - -
22. Carl Geißler Chemnitz 9 - 4 2 - - - - 3 - - - -
22. Kai-Lutz Wagner Chemnitz 9 - - 2 - - - 2 3 - - - -
23. Anna Georgi Chemnitz 8 - - 5 3 - - - - - - - -
23. Marvin Gülden Chemnitz 8 - - - - - - - - - 5 - 3
23. Jessica Ritter Chemnitz 8 - - - 3 3 - 2 - - - - -
23. Kira Grewolls Ulm 8 - - - - - - - - - - 5 3
24. Moritz Weber Chemnitz 7 - 2 - - - - - - - 5 - -
24. Karl Herrmann Chemnitz 7 - 4 2 - - - 1 - - - - -
24. Lilli Weiß Chemnitz 7 - - 5 - - - 2 - - - - -
24. Arne Weißbach Chemnitz 7 - - - - 3 - - 4 - - - -
25. Anna Grünert Chemnitz 6 - - 2 - - 4 - - - - - -
25. Marie Sophie Roß Chemnitz 6 6 - - - - - - - - - - -
25. Felix Karu Altach 6 - - - 4 - - 2 - - - - -
26. Elena Oelschlägel Chemnitz 5 - - 2 3 - - - - - - - -
26. Celine Strumpf Chemnitz 5 - 2 2 - - - - 1 - - - -
26. Vincent Baessler Chemnitz 5 - - 1 - - - - 4 - - - -
26. Valentin Sellin Chemnitz 5 - 3 - - - - 2 - - - - -
27. John Buttler Chemnitz 4 - 4 - - - - - - - - - -
27. Melina Seerig Chemnitz 4 - - - - - 2 2 - - - - -
27. Lisanne Brinkel Chemnitz 4 - 4 - - - - - - - - - -
27. Erik Walther Chemnitz 4 - 4 - - - - - - - - - -
27. Leon Grünert Chemnitz 4 - - - - - 4 - - - - - -
27. Susan Liebermann Chemnitz-Euba 4 - 4 - - - - - - - - - -
27. Lene Haag Chemnitz 4 - - - - - 4 - - - - - -
27. Tobias Morgenstern Chemnitz 4 - - - - - 4 - - - - - -
27. Andreas M. Dittersdorf 4 - 4 - - - - - - - - - -
27. Emilia Oelschlägel Chemnitz 4 - 4 - - - - - - - - - -
28. Elisa Bolte Chemnitz 3 - - - 3 - - - - - - - -
28. XXX ??? 3 - - - - - - - - - - - 3
28. Edda-Marie Penzlin Chemnitz 3 - - - - 3 - - - - - - -
29. Pepe Wurlitzer Chemnitz 2 - - 2 - - - - - - - - -
29. Tim Kasputtis Chemnitz 2 - - 2 - - - - - - - - -
29. Ina Jahre Zwickau 2 - - - - - - 2 - - - - -
29. Daniel Hufenbach Leipzig 2 - - - - - - 2 - - - - -
29. Lisa Berger Chemnitz 2 - - - - 2 - - - - - - -
29. Marie Schmieder Chemnitz 2 - - - - - - 2 - - - - -
29. Jule Irmscher Eibenberg 2 - - 2 - - - - - - - - -
29. Josephine Klotz Chemnitz 2 - - 2 - - - - - - - - -
29. Cynthia Raschkowsky Chemnitz 2 - - 2 - - - - - - - - -
29. Simon Winger Chemnitz 2 - - 2 - - - - - - - - -
29. Laurin Roßberg Chemnitz 2 - - - - 2 - - - - - - -
29. Tim Sigmund Chemnitz 2 - - - - 2 - - - - - - -
29. Alex Gähler Chemnitz 2 - - - - - 2 - - - - - -
29. Hannah-Sophie Schubert Chemnitz 2 - - - - - - 2 - - - - -
29. Joshua May Chemnitz 2 - - - - 2 - - - - - - -
29. Astrid Fischer Chemnitz 2 - - - - - - 2 - - - - -


Ergebnisse der Serie 32 rote Punkte

Auswertung Serie 32 (rote Liste)

Platz Name Ort Summe Aufgabe
373 374 375 376 377 378 379 380 381 382 383 384
1. Rafael Seidel Chemnitz 49 6 6 5 4 2 3 2 6 2 6 4 3
2. Uwe Parsche Chemnitz 48 6 6 5 - 2 3 2 6 5 6 4 3
3. Doreen Naumann Duisburg 45 6 4 2 4 2 3 2 6 4 6 4 2
4. Elisa Parsche Chemnitz 25 6 6 5 - - - 2 6 - - - -
5. Valentin Grundmann Chemnitz 20 6 - - 3 2 3 2 - 1 3 - -
5. Gunnar Reinelt Chemnitz 20 6 - - 3 2 3 2 - 1 3 - -
6. Felicitas Güra Chemnitz 17 - - 3 3 - - - - 1 6 - 4
6. Karolin Schuricht Chemnitz 17 6 - - - - 3 2 - 1 5 - -
7. Martina Bausch Waldshut 14 6 - - - 2 - 2 - - 4 - -
8. GesaH Chemnitz 13 6 - - - - 3 2 - 2 - - -
8. Marie-Christin Müller Erlenbach 13 - - - - - - - 2 2 3 3 3
8. Andree Dammann München 13 - - - - 2 3 2 - - 6 - -
8. Linus-Valentin Lohs Chemnitz 13 6 - 5 - - - 2 - - - - -
9. Felix Karu Altach 12 - - - 10 - - 2 - - - - -
9. Saskia Schlosser Chemnitz 12 6 6 - - - - - - - - - -
9. Jürgen Urbig Chemnitz 12 6 6 - - - - - - - - - -
10. Sabine Fischbach Hessen 11 6 4 - - - - - - - - 1 -
10. Christian Wagner Bamberg 11 6 - - - - - 1 - - - 4 -
10. Heinrich Grossinger Chemnitz 11 - - - 3 2 3 2 - 1 - - -
11. Laura Schlosser Chemnitz 9 4 - - - - 3 1 - 1 - - -
11. Melanie Petz Chemnitz 9 - - - - - 3 1 5 - - - -
12. Arne Weißbach Chemnitz 8 - - - - 2 - - 6 - - - -
12. Marcel Seerig Chemnitz 8 - 3 2 - - - - 1 2 - - -
12. Katrin Wolstein Bamberg 8 6 2 - - - - - - - - - -
12. Mara Neudert Chemnitz 8 6 - - - - - 2 - - - - -
13. Lisanne Brinkel Chemnitz 6 - 6 - - - - - - - - - -
13. Susan Liebermann Chemnitz-Euba 6 - 6 - - - - - - - - - -
13. Andreas M. Dittersdorf 6 - 6 - - - - - - - - - -
13. Nele Mäding Chemnitz 6 - - - - - - - 6 - - - -
13. Marie Sophie Roß Chemnitz 6 6 - - - - - - - - - - -
13. Luisa Schlosser Chemnitz 6 6 - - - - - - - - - - -
14. Adrian Schlegel Chemnitz 5 - - - - - - 2 - - 3 - -
15. Kira Grewolls Ulm 4 - - - - - - - - - - 4 -
15. Elina Rech Chemnitz 4 - - - - - - 1 - - 3 - -
15. XXX ??? 4 - - - - - - - - - - - 4
15. Jessica Ritter Chemnitz 4 - - - - 2 - 2 - - - - -
15. Tom Straßer Chemnitz 4 - - - 3 - - 1 - - - - -
16. Felix Helmert Chemnitz 3 - - - - - - 2 - - - - 1
16. Anna Grünert Chemnitz 3 - - - - - 3 - - - - - -
16. Moritz Weber Chemnitz 3 - - - - - - - - - 3 - -
16. Marvin Gülden Chemnitz 3 - - - - - - - - - 3 - -
16. Leon Grünert Chemnitz 3 - - - - - 3 - - - - - -
17. Camilla Schreiter Chemnitz 2 - - - - - - - - 2 - - -
17. Pauline Marschk Chemnitz 2 - - - - - - - - 2 - - -
17. Daniel Hufenbach Leipzig 2 - - - - - - 2 - - - - -
17. Ulrike Böhme Chemnitz 2 - - - - - - 2 - - - - -
17. Sophie Kalmer Chemnitz 2 - - - - - - - - 2 - - -
17. Jonathan Schlegel Chemnitz 2 - - - - - - 2 - - - - -
17. Emilia Oelschlägel Chemnitz 2 - 2 - - - - - - - - - -
17. Nadja Richter Chemnitz 2 - 2 - - - - - - - - - -
17. Ina Jahre Zwickau 2 - - - - - - 2 - - - - -
17. Paul Arwed Guhlmann Chemnitz 2 - - - - - - 2 - - - - -
17. Line Mauersberger Chemnitz 2 - - - - - - 2 - - - - -
17. Emma Irmscher Eibenberg 2 - - - - - - 2 - - - - -
17. Tobias Morgenstern Chemnitz 2 - - - - - 2 - - - - - -
17. Ida Gwendolin Eichler Chemnitz 2 - - - - - - - 2 - - - -
17. Hannah-Sophie Schubert Chemnitz 2 - - - - - - 2 - - - - -
17. Astrid Fischer Chemnitz 2 - - - - - - 2 - - - - -
18. Lilli Weiß Chemnitz 1 - - - - - - 1 - - - - -
18. Felicitas Hastedt Chemnitz 1 - - - - - - 1 - - - - -
18. Valentin Sellin Chemnitz 1 - - - - - - 1 - - - - -

Serie 31

Serie 31

Aufgabe 1

361. Wertungsaufgabe

Bernd und Mike trafen sich mit ihren Freunden Linus,Valentin und Adrian aus der Grundschulzeit. Inzwischen gehen sie alle an eine andere Schule. (Goetheschule, Schillerschule, Lessingschule, Galileischule und Gaussschule). Ihre Klasssenleiter (Arnold, Müller, Meier, Rust und Kühn) unterrichten sie jeweils in einem Fach (Mathematik, Deutsch, Englisch, Biologie bzw. Chemie)
Wer geht auf welche Schule, hat welchen Klassenlehrer, der welches Fach unterrichtet?
1. Adrian geht nicht auf die Goetheschule, hat nicht Herrn Müller, der kein Mathelehrer ist,  als Klassenlehrer.
2. Bernds Klassenlehrer heißt Kühn
3. Linus hat bei seinem Klassenlehrer Biologie
4. Mike geht in die Galileischule, hat aber bei seinem Klassenlehrer kein Englisch.
5. Herr Arnold unterrichtet Chemie, aber weder an der Galileischule noch an der Lessingschule
6. Der Klassenleiter, der an der Gaussschule arbeitet, unterrichtet Deutsch
7. Herr Rust ist Lehrer an der Schillerschule
6 blaue Punkte

Der Anlass des Treffens ist Adrians Geburtstag. (Geburtstage sind im Februar, April, Juni, August bzw. im Oktober). Sie feiern einmal  zu Hause, sonst aber aber im Garten, einer Eisdiele, einer Pizzeria bzw. beim Inder. Von ihren Eltern bekamen sie in diesem Jahr ein schickes Hemd, eine Uhr, ein Fahrrad, ein interessantes Buch bzw.. ein tolles Taschenmesser. Wer bekam was geschenkt, wo wurde gefeiert und in welchem Monat war das?
1. Bernd feiert im Februar, aber nicht in der Pizzaria.
2. Linus bekam das Buch, aber nicht im Juni.
3. Mike feierte zu Hause.
4. Die Uhr wurde im August verschenkt
5. Der Geburtstag von dem der das Hemd bekam, folgte auf den Geburtstag bei dem das Taschenmesser verschenkt wurde.
6. Im Juni wurde im Garten gefeiert, allerdings war das nicht der Geburtstag von Valentin. Auch das Fahrrad wurde nicht im Garten überreicht.
7. Der das Hemd bekam, hatte direkt nach dem Geburtstag, dessen Geburtstag in der Eisdiele gefeiert wurde.
6 rote Punkte

Termin der Abgabe 21.06.2012 Deadline for solution is the 21. june 2012.

361
logic puzzle
Bernd and Mike met up with their friends from primary school, Linus, Valentin and Adrian. Now they all go to different schools (Goethe School, Schiller School, Lessing School, Galilei School and Gauss School). Their form teachers (Arnold, Mueller, Meier, Rust and Kuehn) teach one subject each (Maths, German, English, Biology and Chemistry).
Who attends which school, who is their form teacher and which subject does he teach?
1. Adrian does not attend the Goethe School and his form teacher is not Mr Mueller who, by the way, doesn't teach Maths.
2. Bernd's form teacher is mr Kuehn.
3. Linus' form teacher teaches Biology.
4. Mike attends Galilei School, but his form teacher doesn't teach English.
5. Mr Arnold teaches Chemistry, but neither at Galilei, nor at Lessing School.
6. The form teacher working at Gauss School teaches German.
7. Mr Rust teaches at Schiller School.
- 6 blue points

The reason for the meeting is Adrian's birthday. (Birthdays are in February, April, June, August and October.) They celebrate at home or alternatively in the garden, in an ice cream parlour, a pizza place or in an Indian restaurant. As presents they got a nice shirt, a watch, a bicycle, an interesting book and a pen knife. Who got which present, where did they celebrate and when?
1. Bernd celebrates in February, but not in a pizza place.
2. Linus got the book, but not in June.
3. Mike celebrated at home
4. The watch was given as a present in August.
5. The birthday of the person who got the shirt followed the birthday that the pen knife was given as a present.
6. In June there was a garden party although it wasn't Valentin's birtday. Also the bicycle wasn't given as present in the garden.
7. The one who got the shirt celebrated his birthday directly after the one who celebrated in the ice cream parlour.
- 6 red points

Lösung/solution:
blau:
Bernd hat Hr Kühn in Deutsch auf der Gaussschule
Mike hat Hr. Meier in Mathe auf der Galileischule
Linus hat Hr. Müller in Bio auf der Lessingschule
Valentin hat Hr. Arnold in Chemie auf der Goethe
Adrian hat Hr. Rust in englisch auf der Schiller.

rot:

Februar, Bernd, Fahrrad, Inder
April, Valentin, Taschenmesser, Eisdiele
Juni, Adrian, Hemd, Garten
August, Mike, Uhr, Zuhause
Oktober, Linus, Buch, Pizzeria

Es gibt noch jeweils eine andere Lösung



Aufgabe 2


362. Wertungsaufgabe
Lisa hat eine Uhr (12 Stunden) aus Pappe vor sich liegen. Außer dem hat sie die Zahlen von 1 bis 11 aus Papier ausgeschnitten. „Was machst du da?“, fragte Maria. „Ich habe mir eine Aufgabe für unsere Mathematikgruppe ausgedacht. Die sollen die Papierzahlen, die ich hier auf den Zetteln habe. so auf die Uhr legen,dass die Uhr  zwar wieder die Zahlen von 1 bis 12 hat, aber benachbarte Zahlen sich um 2 oder drei unterscheiden.“ Gib eine Möglichkeit für eine solche Sortierung an. 3 blaue Punkte (am einfachsten schreibt sich Lösung auf, wenn man im Uhrzeigersinn die Zahlen nimmt und bei der Zahl beginnt, die auf die
ursprüngliche 1 gelegt wird. - Sollte man nachweisen, dass es keine Lösung gibt, die die Aufgabe erfüllt, so gibt es die Punkte auch) Wie lange bräuchte Lisa, wenn sie alle Möglichkeiten, die 11 Zettel auf der Uhr in irgend einer Form auf den ursprünglichen Uhrzahlen anzuordnen, ausprobieren würde und jede Umlegung 5 Sekunden dauerte? 3 rote Punkte
online testen

uhrTermin der Abgabe 28.06.2012 Deadline for solution is the 28. june 2012.
362
Lisa has got a clock (12 hours) made of cardboard in front of her. Also she has cut out the numbers from 1 to 11. Number 12 is already where it should be.
“What are you doing?”, Maria asks.
“I made up a problem for our maths group. They have to attach these paper numbers to the face of the clock in such a way that the clock shows all the numbers from 1 to 12 but that adjacent numbers differ by 2 or 3.”
One possibility of such an order – 3 blue points (also awarded if shown that no such solution exists)
How long would it take if Lisa tried all the possibilities to arrange the 11 numbers on the clock face taking 5 seconds for each try? - 3 red points
online test

Lösung/solution:
Hier die Lösung von Zach Markos, thanks --> pdf <--



Aufgabe 3

363. Wertungsaufgabe
Bernd trägt in ein (kartesisches) Koordinatensystem die Punkte (2; 3) und (4; 6) ein. „Was wird das?“, fragt Mike, der ins Zimmer kommt. „Ich suche die Koordinaten zweier weiterer Punkte, so dass  ich die 4 Punkte dann zu einem Quadrat verbinden kann“. Zwei blaue Punkte – die Angabe einer Möglichkeit ist ausreichend. Berechne den Flächeninhalt des größtmöglichen Quadrates, welches die obigen Punkte als Eckpunkte aufweist. 3 rote Punkte

Termin der Abgabe 06.09.2012 Deadline for solution is the 6. september 2012.

363
Bernd uses a cartesian coordinate system to mark points (2,3) and (4,6).  “What's this going to be?”, Mike asks upon entering the room.  “I'm looking for the coordinates of two more points so I can join them to get square.” - 2 blue points (one solution is sufficient)
Calculate the area of the biggest possible square that has above points as vertices. - 3 red points


Lösung/solution:
Wie man dem Bild entnehmen kann, gibt es drei Lösungen für blau. Die abzulesenden Koordinaten sind Bild erkennbar. Für rot eignen sich die zwei großen Quadrate. Die Kantenlänge lässt mit dem Satz des Pythagoras errechnen.
363 k
Bild in groß (enlarge)





Aufgabe 4

364. Wertungsaufgabe
Lisa und Maria schneiden ganz fleißig regelmäßige Sechsecke aus. Die Kantenlänge der Sechsecke  beträgt bei allen Sechsecken 2 cm. Mike kommt dazu und legt  aus 15 der Sechsecke ein „Dreieck“. „Das sieht aber gut aus“, meint Lisa. „Auch mir gefällt es, das sollte auf einer Bienenmesse
ausgestellt werden“, sagte Maria. Wie groß ist der Umfang der gelegten Figur? - 3 blaue Punkte. Die Mittelpunkte aller benachbarten Sechsecke der Figur von Mike werden mit einander verbunden.  Wie lang sind alle Verbindungslinien zusammen? 3 rote Punkte.
Termin der Abgabe 13.09.2012 Deadline for solution is the 13. september 2012.
Lisa and Maria are busy cutting out regular hexagons. The length of the edges in each hexagon is 2 cm. Mike joins them and uses 15 of these hexagons to form a kind of “triangle”.
“That looks nice”, Lisa remarks.
“I like it too”, Maria says. “It would sure be the latest craze on a beekeepers' fair.”
What's the perimeter of Mike's figure? - 3 blue points
Connect the centres of all neighbouring hexagons. How long are all the connecting lines together? - 3 red points
Lösung/solution:
Die Aufgabe war nicht so eindeutig wie sie sich möglicherweise liest.
Hier mal 2 Varianten:
Von Andree, danke  als pdf
Von Linus-V., danke als pdf
Nach eine Variante als Bild:
364-2





Aufgabe 5

365. Wertungsaufgabe
„Du Bernd, schau mal, einer aus unserer Mathematikgruppe hat sich dieses Geheimalphabet ausgedacht. Die Buchstaben des Alphabets werden durchnummeriert, allerdings wird das j übersprungen (j=i). Jede der Zahlen wird mit 4 multipliziert. Anschließend werden alle Zahlen noch addiert. Die so erhaltene Zahl wird dann für die Verschlüsselung genommen“. „Schöne Idee“, meint Bernd. Wie wird dann „Maria“ verschlüsselt? - 3 blaue Punkte.
Welcher Mathematiker verbirgt sich hinter der Zahl 284 – der Name besteht aus 7 Buchstaben und enthält ein e und ein b. 4 rote Punkte.

Termin der Abgabe 20.09.2012 Deadline for solution is the 20. september 2012.

“Bernd, take a look. Someone of our maths group made up this secret alphabet. The letters of the alphabet are numbered, with the exception of the letter 'j' (j=i). Each of these numbered is multiplied by 4. Finally all numbers are added. The result is the code”.
“Nice idea”, Bernd remarks.
How do you code 'Maria'? - 3 blue points
The name of which mathematician is coded as 284? The name consists of 7 letters and contains an 'e' and a 'b'. - 4 red points
Lösung/solution:
blau: Notiert man die Buchstaben und deren Zahlenwerte, so ergibt sich für Maria die Zahl 160.
rot: Aus der vorgegebenen Zahl auf den Namen zu schließen ist - wenn man keine weiteren Informationen hat - nicht eindeutig. So kann eine 12 für AAA, AB oder eben C stehen. Siebenbuchstabige deutsche Mathematiker, die in ihrem Namen ein ein e und ein n haben gibt es allerdings nicht so viele - Leibniz und Hilbert. Leibniz für auf 294 und Hilbert auf die 284. Gesucht war also Hilbert.


Aufgabe 6

366. Wertungsaufgabe
„Was machst du denn da?“, fragte Bernd seine Schwester Maria. „Ich will gerade beginnen, mach doch einfach mit.“ „Einverstanden.“ „Nimm zwei rechteckige Papierstreifen (2 x 20 cm). Klebe diese jeweils zu einem Ring zusammen, so dass sich die Klebestellen um 0,5 cm überlappen.“ „Habe ich.“ „Nun klebst du die beiden Ringe so an einer Stelle zusammen, so dass diese senkrecht zueinander sind.“ „Das sieht ja aus wie das Modell einer Kugel.“ „Genau. Nun schneidest du den einen Ring entlang der langen Mittellinie auseinander, anschließend den zweiten Ring.“ „Cool, das ist ja jetzt ein(e) ...“. Was ist entstanden und wie groß sind Innen- und Außenmaß des Gebildes – 3 blaue Punkte.
3 rote Punkte für die Maße des Gebildes, wenn die Maße der Streifen (a x b cm) sind und die Klebestellenüberlappung c cm groß ist. (c<a<<b)
Termin der Abgabe 27.09.2012 Deadline for solution is the 27. september 2012.
“What are you doing there?”, Bernd asks his sister Maria. “I'm only just starting. Come and join me.” “Why not.” “Look, take two rectangular strips of paper (2 x 20cm). Stick both of them together separately so that the splices overlap for 0.5cm.” “I've got that.” “Now stick the two rings together in one place and at a right angle.” “Looks like the model of a sphere.” “Exactly. Now cut one ring along its long centre line and then the other ring.” “Cool, I get a … .”
What do you get and what are the inner and outer measurements of this shape? - 3 blue points
3 red points for the measurements of the shape if the paper strips measure a x b centimetres and the overlap is c centimetres. (c<a<<b)

Lösung/solution:
Es entsteht ein "Quadratring" mit dem Außenmaß von 19,5 cm und einem Innenmaß von 17,5 cm. Der Ring selber ist also 1 cm breit. Darus lässt sich die allgemeine Lösung recht schnell ableiten. Außenmaß Streifenlänge - Überlappung (b-c) Innenmaß Außenmaß - Streifenbreite (b-c-a)


Aufgabe 7

367. Wertungsaufgabe
Mike repariert sein Fahrrad. Der Außendurchmesser des Vorderrades beträgt 71 cm. Die Spitze des Ventiles hat einen Abstand von 26 cm vom Mittelpunkt des Rades. Nach dem Ende der Reparatur dreht Mike das Vorderrad genau 12 mal, während das Rad noch steht. Welchen Weg legt die Ventilspitze während des Drehens zurück? - 3 blaue Punkte. (Formel siehe Mathematiklexikon). Nun setzt sich Mike auf das Rad und fährt ein kleines Stück.  Wieder dreht sich das Rad genau 12 mal. Welchen Weg legt die Spitze des Ventiles nun zurück, wenn sich das Ventil zu Beginn (und Ende) am tiefst möglichen Punkt des Rades befindet? - 6 rote Punkte.
Termin der Abgabe 04.10.2012 Deadline for solution is the 4th. october 2012.
Mike is repairing his bicycle. The outer diameter of his front wheel is 71cm. The tip of the valve is at a distance of 26cm from the centre of the wheel. When Mike is finished he lets the front wheel turn exactly 12 times while not moving the bicycle. What distance does the tip of the valve travel during thes 12 rotations? - 3 blue points
Now Mike gets on his bicycle and rides a short distance. Again his front wheel turns 12 times. What distance does the tip of the valve travel now if the valve started and ended at the lowest possible position?
Lösung/solution:

Hier die Lösung von M. Pfaffe, danke
als pdf


Aufgabe 8

368. Wertungsaufgabe
„Vor einiger Zeit (Aufgabe 354) hatten wir ein Dreieck ABC in ein Koordinatensystem gezeichnet.“; sagte Maria. „Kannst du mir noch mal die Koordinaten sagen?“, fragte Bernd. „Kein Problem.“ A (0; 0), B (5; 1) und C (4; 6) Da es sich um ein rechtwinkliges Dreieck handelt, ist es sicher kein Problem den Flächeninhalt auszurechnen – 3 blaue Punkte. Ist es möglich (oder
unmöglich) ein gleichseitiges Dreieck ABC mit A (0; 0) in ein Koordinatensystem zu zeichnen, so dass die Koordinaten von B und C auch ganzzahlig sind. 10 rote Punkte
Termin der Abgabe 11.10.2012 Deadline for solution is the 11th. october 2012.
368
“A while ago (problem 354) we drew a triangel ABC into a coordinate system”, Maria said.
“Could you give me the coordinates?”, Bernd asks her.
“No problem: A(0;0), B(5;1) and C(4;6)”.
As its is a right-angled triangle it should be easy to calculate its area. - 3 blue points.
Is it (im)possible to draw an equilateral triangle ABC with A at (0;0) into a coordinate system so that the coordinates of B and C are integers? - 10 red points
Lösung/solution:
Die Lösung wird nachgereicht, nur mal schon kurz: Blau Lösung A = 13 cm²
rot ein solches Dreieck gibt es (leider) nicht. Aber mit A (0; 0), B (41; 11)und C (11; 41) kommt man ganz schön nah heran.
\sqrt (1800) und sqrt {1800} und sqrt {1802} sind die Seitenlängen des Dreiecks ABC, also zwei ganz gleich und die dritte ist um 0,02 Einheiten länger.
Ausführliche Lösung, beruhend auf den Ideen von Rafael, danke.
368
Der Flächeninhalt des Dreiecks ergibt sich aus dem Flächeninhalt des Rechtecks ADEF vermindert um A1, A2 und A3. Die Flächeninhalte der Dreiecke (sind alle rechtwinklig) sind leicht zu berechnen. 5*6 - 5*1/2 - 5*1/2 - 4*6/2 = 30 - 2,5 - 2,5 - 12 = 13
Die Eigenschaft, dass das Dreieck ABC rechtwinklig war, wurde bei diesem Rechenweg gar nicht genutzt. Dieser Weg eignet sich also für jedes Dreieck, dessen Flächeninhalt zu bestimmen ist.
rot: Angenommen es existiere ein solches gleichseitiges Dreieck ABC mit der Seitenlänge a, dessen Koordinaten ganzzahlig seien.
Fall 1: B läge auf der x-Achse, dann wären die Koordinaten von C
 x_{\frac{a}{2}} und \pm \sqrt {3} \cdot \frac {a}{2}
Die x-Koordinate von C könnte ganzzahlig sein, wenn a durch zwei teilbar wäre, allerdings ist die y-Koordinate nie ganzzahlig, denn Wurzel (3) ist irrational (Formel für die Höhe im gleichseitigen Dreieck). Es gibt also kein solches Dreieck mit B auf der x-Achse.
Fall 2: C läge auf der y-Achse, denn wären die Koordinaten von B:
  \pm \sqrt {3} \cdot \frac {a}{2} und y_{\frac{a}{2}}
Die y-Koordinate von B könnte ganzzahlig sein, wenn a durch zwei teilbar wäre, allerdings ist die x-Koordinate nie ganzzahlig, denn Wurzel (3) ist irrational (Formel für die Höhe im gleichseitigen Dreieck). Es gibt also kein solches Dreieck mit C auf der y-Achse.
Fall 3: B und C lägen im 1. Quadraten. Dann ließe sich wie im Fall blau ein Rechteck finden. Die Koordinaten von D, E und F wären dann ganzzahlig, denn sie würden sich ja von den ganzzahligen Koordinaten von B und C ableiten. Der Flächeninhalt des Dreiecks ergäbe sich dann aus der Differenz des Rechtecks und den drei Dreiecken. Leicht zu sehen, dieser Flächeninhalt wäre ganzzahlig oder etwas mit ,5. Andererseits ließe sich für die Ermittlung der Seitenlänge a des Dreiecks ABC a² = xb² + yb² benutzen. a² ist aber ganzzahlig. Für den Flächeninhalt des gleichseitigen Dreiecks gilt aber auch:
 A = \frac{a^2}{4} \cdot \sqrt{3}
Der Flächeninhalt wäre dann aber wieder irrational, das aber steht im Widerspruch zum Rechenwegergebnis über das "Umgebungsrechteck". Es gibt also kein gleichseitiges Dreieck im Fall 3.
Weitere Fälle für die Lage des Dreiecks brauchen nicht untersucht zu werden, da sich diese durch geeignete Verschiebungen und Spiegelungen auf den Fall 3 zurückführen lassen.
Die Annahme der Existenz eines solchen Dreiecks ABC war also falsch.
(leider).




Aufgabe 9

369. Wertungsaufgabe
„Schau mal in dem Buch "Schönheit der Geometrie" von Miranda Lundy habe ich diese wunderschöne Konstruktion eines regelmäßigen Zwölfecks entdeckt.“ 369 Wie groß ist wohl der Umfang des Zwölfecks, wenn der Radius des inneren Kreises 4,0 cm beträgt und wie lang sind die längsten Diagonalen? - 3 blaue Punkte. Wie groß ist der Flächeninhalt des Zwölfecks? 3 rote Punkte

Termin der Abgabe 18.10.2012 Deadline for solution is the 18th. october 2012.
 
“Look, I found this beautiful construction of a regular dodecagon in this book Sacred Geometry by Miranda Lundy.”369 What is the perimeter of this dodecagon if the radius of the inscribed circle is 4cm and how long are the longest diagonals? - 3 blue points
What is the area of this dodecagon? - 3 red points.
Lösung/solution:
Lösung von U. Parsch, danke. als pdf




Aufgabe 10

370. Wertungsaufgabe
„Ich habe eine zauberhafte Primzahl entdeckt, es ist die 743", sagte Lisa. „Was ist daran zauberhaft?“, fragte Bernd. „Schau, wenn ich die erste Ziffer wegstreiche, so ist die verbleibende Zahl auch prim. Streiche ich die beiden ersten Ziffern weg, so ist es immer noch eine Primzahl, die übrig bleibt.“ „Das stimmt, gibt es viele solcher Zauberprimzahlen?“ „Das weiß ich nicht“, gab Lisa zu. Finde alle dreistelligen Zauberprimzahlen, deren erste Ziffer eine 1 ist. - 3 blaue Punkte
Welche der „blauen“ dreistelligen Zauberprimzahlen stellen die Reste von 4-stelligen Zauberprimzahlen dar? 3 rote Punkte -  Tipp Primtab im Lexikon verwenden. Anmerkung, die größte Zauberprimzahl hat 24 Stellen.

Termin der Abgabe 01.11.2012 Deadline for solution is the 1th. november 2012.
 
“I found a magic prime number, it's 743”, said Lisa.
“Why is it magic?”, Bernd wanted to know.
“Look, if you cross out the first digit, the remaining number is still prime. If you cross out the first two digits you still get a prime number.”
“That's right. Are there many of these magiv prime numbers?”
“I don't know”, Lisa admitted.
Find all three-digit magic primes beginning with 1. - 3 blue points.
Which of your “blue” three-digit primes are the remains of 4-digit primes? - 3 red points
Note: the biggest magic prime has 24 digits.
Lösung/ solution:
Einige der Löser haben die führende Nullen verwendet und kommen so auf 103, 107, 113, 137, 167, 173 und 197. die 103 zählt eher nicht dazu, denn 03 als Primzahl ???????
Für den roten Teil der Aufgabe setzt man am besten immer eine Ziffer vor blau und schaut, ob dies auf eine Primzahl führt.
Hier die Ergebnisse (ohne 103 und 107):
113: (2113) (5113) (6113)
137: (2137) (3137) (9137)
167: (3167) (5167) (8167)
173: (6173) (9173)
197: (5197) (6197)
Die Lister aller Zauberprimzahlen: --> hier <--




Aufgabe 11

370. Wertungsaufgabe
 
„Schau mal Mike, das ist ein Käferquadrat“; sagte Lisa. „Käferquadrat? Für mich ist das ein ganz normales Quadrat ABCD, welches 10 cm  groß ist.“ Stell dir vor, ein Käfer läuft von A nach C. Die Seitenfarbe aber mag der Käfer nicht, so dass er immer nur einen Zentimeter auf der Kante - mit Ausnahme der Kante BC -  läuft und ansonsten im Winkel von 90° (bezogen auf die Kanten) quer über das Quadrat, bis er wieder eine Kante erreicht, 1 cm darauf läuft, abbiegt, … bis er Punkt C erreicht hat. Wie lang ist der kürzeste Weg des Käfers von A nach C, wenn er in Richtung B startet? 2 blaue Punkte, 3 weitere blaue Punkte gibt es für eine (oder mehrere Formeln) für den Fall, dass das Quadrat n cm groß ist. (n natürliche Zahl)
Wie lang ist der Weg des Käfers, wenn er seine Wegabschnitte parallel zur Diagonalen BD wählt. 4 rote Punkte. 4 weitere roten Punkte sind möglich, wenn eine Formel (oder auch mehrere) für den allgemeinen Fall n entwickelt wird.

Termin der Abgabe 08.11.2012 Deadline for solution is the 8th. november 2012.
 
“Look Mike, a beetle square”, Lisa said.
“Beetle square? I see an ordinary square ABCD with 10cm sides.”
Imagine a beetle moving from A to C. It doesn't really like the ink the square has been drawn in so if the beetle moves along a side it only does so for 1 cm (except on side BC). So it moves along a side for 1cm, turns 90° moves across the square until it reaches another side, turns, moves along this side for 1cm, turns again, … , until it reaches point C.
How long is the shortest distance the beetle travels from A to C, if it starts in the direction of B? - 2 blue points. There are 3 more points awarded for a formula to calculate the distance for a square of n cm side length. (let n be natural number)
How far would our beetle travel if it moved parallel to the diagonal BD? - 4 red points. Again, another 4 red points are offered for a formula for the n-sided square.
Lösung/solution:

Die Lösungen sind dieses Mal als Applet ausgeführt:
-->blau<--
-->rot<--



Aufgabe 12


372. Wertungsaufgabe
„Wir haben heute mit unserer Mathematikgruppe „die böse Sieben“ gespielt“, sagte Marie zu Bernd, als sie nach Hause kam. „Wie geht das denn?“ „Es werden die Zahlen von 1 bis 100 der Reihe nach angesagt, wenn eine Zahl aber auf sieben endet oder durch sieben teilbar ist, dann muss statt der Zahl das Wort "böse" gesagt werden. Wer eine solche verbotene Zahl trotzdem sagt oder nicht weiß, wie es weiter geht, scheidet aus. Ist die 100 erreicht und es sind noch mehrere Teilnehmer im Rennen, so wird rückwärts gezählt, dann wieder vorwärts usw. Da es ganz schnell gehen muss, dauert das Spiel meist nicht so lange. Wir haben es noch erschwert, weil wir die Reihenfolge der Teilnehmer zufällig gewählt haben. Als wir das erste Mal die 100 erreicht hatten,
war noch keiner ausgeschieden und jeder hatte genau zweimal "böse" sagen müssen.“ Wie viele Teilnehmer gab es? - 2 blaue Punkte. Wie groß ist die Wahrscheinlichkeit, dass einer der Teilnehmer (Anzahl von blau) in der ersten Runde des Spieles 4 mal hintereinander „ausgelost“ wird? 3 rote Punkte

Termin der Abgabe 15.11.2012 Deadline for solution is the 15th. november 2012.

372
“Today we played 'the bad seven' in our maths group”, Marie told Bernd as she got home.
“What's this about?”
“You simply count from one to 100, one after the other. If you'd have to say a number containing a seven or even a number that is divisible by seven you have to say the word 'bad' instead. If you fail to do so you're out. If you reach 100 and there is still more than one player left you count backwards, then forward again and so on. The game usually doesn't take too long because you should count really fast. We made it even more difficult by picking the players at random for each new number. When we reached 100 for the first time nobody was out and everyone had said 'bad' exactly twice.”
How many participants were there 2 blue points
What's the probability that one of the players is chosen to count for 4 consecutive times?
Lösung/solution:
blau:
Es gibt die Zahlen, die durch sieben teilbar sind: 7; 14; 21; ... 98, das sind 14 Zahlen.
Dazu kommen Zahlen die auf 7 enden, aber nicht durch 7 teilbar sind: 17; 27; ... 97, das sind 8 Zahlen. Insgesamt also 22 Zahlen, das heißt es sind 11 Teilnehmer.
rot:
Einfache Variante, dass einer der 11 Teilnehmer vier mal hintereinander gezogen wird, ob "böse" oder nicht nicht ist: (1/11)4 =
0,0000683013455365.
Die Wahrscheinlichkeit, dass jeder Teilnehmer bis zur 11. böse-Zahl genau eine davon gesagt hat ist nicht auch nicht sehr groß: 0,000139905948868. Die Wahrscheinlichkeit für das Ergebnis für die Aufgabe blau zu berechnen, überlasse ich dem geneigten Leser.



Auswertung Serie 31 (blaue Liste)

Platz Name Ort Summe Aufgabe
361 362 363 364 365 366 367 368 369 370 371 372
1. Rafael Seidel Chemnitz 39 6 3 2 3 3 3 3 3 3 3 5 2
2. Doreen Naumann Duisburg 38 6 3 2 3 3 2 3 3 3 3 5 2
2. Linus-Valentin Lohs Chemnitz 38 6 3 2 3 3 3 3 3 3 3 4 2
3. Jürgen Urbig Chemnitz 37 6 3 2 3 3 3 3 3 3 3 5 -
4. Sabine Fischbach Hessen 25 6 3 2 3 3 - 2 3 - 3 - -
5. Elisa Parsche Chemnitz 24 - - 2 3 3 3 3 3 3 - 4 -
6. Uwe Parsche Chemnitz 22 - - - 3 3 3 3 3 3 - 4 -
7. Laura Schlosser Chemnitz 19 - - 2 3 3 3 - 2 - - 4 2
8. Valentin Grundmann Chemnitz 17 6 2 2 - 3 - - - - - 4 -
9. Luisa Schlosser Chemnitz 16 - - 2 3 - 3 3 3 2 - - -
10. Tom Straßer Chemnitz 15 6 3 1 - 3 - 2 - - - - -
11. Nele Mäding Chemnitz 14 6 - 2 3 - 3 - - - - - -
11. Heinrich Grossinger Chemnitz 14 6 3 2 - 3 - - - - - - -
12. Melanie Petz Chemnitz 13 - 3 2 - 3 2 3 - - - - -
12. Andree Dammann München 13 - - - 3 - - - - - 3 5 2
13. Mike Pfaffe Großenhain 12 - - - - - 3 3 3 3 - - -
13. Gunnar Reinelt Chemnitz 12 6 3 - - 3 - - - - - - -
14. Karolin Schuricht Chemnitz 11 - - 2 - 3 3 - - - 3 - -
14. Line Mauersberger Chemnitz 11 - - - - - - - - 2 3 4 2
15. Katrin Wolstein Bamberg 10 - - - - - - - - - 3 5 2
15. Felicitas Güra Chemnitz 10 6 1 - - 3 - - - - - - -
15. Elena Oelschlägel Chemnitz 10 - 3 2 - - - 3 - - - - -
16. Eden Markos Los Angeles 9 6 3 - - - - - - - - - -
16. Zach Markos Los Angeles 9 6 3 - - - - - - - - - -
16. Arne Weißbach Chemnitz 9 - 3 - - 3 - - 3 - - - -
16. Helene Fischer Chemnitz 9 4 - 2 - 3 - - - - - - -
16. Jessica Ritter Chemnitz 9 6 - - - 3 - - - - - - -
16. Felicitas Hastedt Chemnitz 9 - 3 2 - - - 3 - - - - -
17. Lilli Weiß Chemnitz 8 - 3 2 - 3 - - - - - - -
17. Simon Anders Chemnitz 8 - 3 2 - - - 3 - - - - -
17. Adrian Schlegel Chemnitz 8 - 3 - - 3 - 2 - - - - -
17. Felix Karu Altach 8 - 3 2 - 3 - - - - - - -
17. Elisa Bolte Chemnitz 8 - 3 2 - - - 3 - - - - -
17. Vincent Baessler Chemnitz 8 - - - - - 3 3 - - - - 2
17. Kai-Lutz Wagner Chemnitz 8 - - - 2 3 - - - - 3 - -
17. Lisanne Brinkel Chemnitz 8 - 3 2 - 3 - - - - - - -
17. Saskia Schlosser Chemnitz 8 - - 2 3 - - - 3 - - - -
18. Valentin Sellin Chemnitz 7 - 3 - - 2 - - - - 2 - -
18. Tobias Morgenstern Chemnitz 7 - - 2 2 3 - - - - - - -
19. Anna Georgi Chemnitz 6 - 3 - - 3 - - - - - - -
19. Pascal Graupner Chemnitz 6 - - - - 3 - 3 - - - - -
19. Christian Wagner Bamberg 6 6 - - - - - - - - - - -
19. Marcel Seerig Chemnitz 6 - - - 3 3 - - - - - - -
19. Anna Grünert Chemnitz 6 - 3 - - 3 - - - - - - -
19. Marie Sophie Roß Chemnitz 6 6 - - - - - - - - - - -
19. Elias Schmidt Chemnitz 6 - - - 3 3 - - - - - - -
20. Mara Neudert Chemnitz 5 - - 2 - 3 - - - - - - -
20. Shari Schmidt Chemnitz 5 - - 2 - 3 - - - - - - -
20. Tobias Richter Chemnitz 5 - - 2 - 3 - - - - - - -
20. Camilla Schreiter Chemnitz 5 - - 2 - 3 - - - - - - -
20. Lena Rabbeau Chemnitz 5 - - 2 - 3 - - - - - - -
20. Florian A. Schönherr Chemnitz 5 - - 2 - 3 - - - - - - -
20. Jonas Frederik Otto Lichtenwalde 5 - - 2 - 3 - - - - - - -
20. Selma Juhran Chemnitz 5 - - 2 - 3 - - - - - - -
20. Carl Geißler Chemnitz 5 - - 2 - 3 - - - - - - -
20. Pauline Marschk Chemnitz 5 - - 2 - 3 - - - - - - -
20. Sophie Kalmer Chemnitz 5 - - 2 - 3 - - - - - - -
20. Simon Winger Chemnitz 5 - 3 - - - - - - 2 - - -
20. Emma Irmscher Eibenberg 5 - - - - - 2 - - - - - -
20. Hannes Hohmann Chemnitz 5 - - 2 - 3 - - - - - - -
20. GesaH Chemnitz 5 - - 2 - 3 - - - - - - -
20. Marie Juhran Chemnitz 5 - - 2 - 3 - - - - - - -
20. Rebecca Wagner Oberwiesenthal 5 - - 2 - - - - 3 - - - -
20. Frederike Meiser Chemnitz 5 - - 2 - 3 - - - - - - -
20. Jessica Spindler Chemnitz 5 - - 2 - 3 - - - - - - -
21. Martina Bausch Waldshut 4 - - - - - - - - - - 4 -
21. Ida Gwendolin Eichler Chemnitz 4 - - - - 2 - - - - 2 - -
21. Lukas Thieme Chemnitz 4 - - 2 - 2 - - - - - - -
21. Paula Geißler Chemnitz 4 - - - - - - 2 2 - - - -
21. Paul Arwed Guhlmann Chemnitz 4 - - - - - - - - - - 2 2
21. Hannah-Sophie Schubert Chemnitz 4 - - 2 - - - - 2 - - - -
21. Nicklas Reichert Chemnitz 4 - - 2 - 2 - - - - - - -
22. Cynthia Raschkowsky Chemnitz 3 - 3 - - - - - - - - - -
22. Marvin Gülden Chemnitz 3 - - - - - - 3 - - - - -
22. Ulrike Böhme Chemnitz 3 - 3 - - - - - - - - - -
22. Elina Rech Chemnitz 3 - - - - 3 - - - - - - -
22. Robin König Chemnitz 3 - - - - - - - - - 3 - -
22. Lisa Berger Chemnitz 3 - - - - - - 3 - - - - -
22. Karl Herrmann Chemnitz 3 - - - - - - 3 - - - - -
22. XXX ??? 3 - 3 - - - - - - - - - -
22. Joel Magyar Chemnitz 3 - - - - 3 - - - - - - -
22. Hannes Eltner ???? 3 - - - - - 3 - - - - - -
22. Malte Lohs Chemnitz 3 - - - - - - - - - 3 - -
23. Julien Kaiser Chemnitz 2 - - - - - - - - - - 2 -
23. Peye Mäding Chemnitz 2 - - - - - - - - - - 2 -
23. Celine Strumpf Chemnitz 2 - - - - - - - 2 - - - -
23. Lydia Richter Chemnitz 2 - - - - - - - - - - 2 -
23. Noa Adamczak Chemnitz 2 - - - - - - - - - - 2 -
23. Alfred Groß Chemnitz 2 - - - - - - - - - - 2 -
23. Vincent Reich Chemnitz 2 - - - - - - - - - - 2 -
23. Mathis Ladstätter Chemnitz 2 - - - - - - - - - - 2 -
23. Emma Münzner Chemnitz 2 - - - - - - - - - - 2 -
23. Anke Morgenstern Chemnitz 2 - - - - - - - - - - 2 -
23. Lena Steinert Chemnitz 2 - - - - - - - - - - 2 -
23. Emely Arndt Chemnitz 2 - - - - - - - - - - 2 -
23. Lene Langenstraß Chemnitz 2 - - - - - - - - - - 2 -
23. Johann Otto Chemnitz 2 - - - - - - - - - - 2 -
23. Paula-Anthonia Turinsky Chemnitz 2 - - - - - - - - - - 2 -
23. Justin Nguyen Chemnitz 2 - - - - - - - - - - 2 -
23. Benedikt Schirrmeister Chemnitz 2 - - - - - - - - - - 2 -
23. Susanna Seidler Chemnitz 2 - - 2 - - - - - - - - -
23. Albin Uhlig Chemnitz 2 - - 2 - - - - - - - - -
23. Jule Schwalbe Chemnitz 2 - - 2 - - - - - - - - -
23. Melina Seerig Chemnitz 2 - - 2 - - - - - - - - -
23. Svenja Reinelt Chemnitz 2 - - - - - - 2 - - - - -
23. Benjamin Hildebrand Chemnitz 2 - - - - - - - - - - 2 -
23. Lene Haag Chemnitz 2 - - 2 - - - - - - - - -
23. Gwendolin Eichler Chemnitz 2 - - 2 - - - - - - - - -
23. Kevin Ngyen Chemnitz 2 - - 2 - - - - - - - - -
23. Celestina Montero Perez Chemnitz 2 - - 2 - - - - - - - - -
23. Marie Berger Chemnitz 2 - - 2 - - - - - - - - -
23. Paula Mühlmann Dittersdorf 2 - - 2 - - - - - - - - -
23. Carlo Klemm Chemnitz 2 - - 2 - - - - - - - - -
23. Leon Grünert Chemnitz 2 - - 2 - - - - - - - - -
23. Michelle Bühner Chemnitz 2 - - 2 - - - - - - - - -
23. Franz Kemter Chemnitz 2 - - 2 - - - - - - - - -
23. Jeremias Baryschnik Chemnitz 2 - - - - 2 - - - - - - -
23. Jonna Langrzik Chemnitz 2 - - - - - - - - - - 2 -
23. Paul Georgi Chemnitz 2 - - 2 - - - - - - - - -
23. Alex Gähler Chemnitz 2 - - - - - - - - - 2 - -
23. Marie Schmieder Chemnitz 2 - - 2 - - - - - - - - -
24. Till Schüppel Chemnitz 1 - - - - - - - - - - 1 -
24. Nadjeschda Günther Chemnitz 1 - - - - - - - - - - 1 -
24. Christin Reichelt Chemnitz 1 - - - - - - - - - - 1 -
24. Erik Arnold Chemnitz 1 - - - - - - - - - - 1 -
24. Moritz Weber Chemnitz 1 - - 1 - - - - - - - - -
24. Marie Albuschat Chemnitz 1 - - - - - - - - - - 1 -
24. Merlin Liesch Chemnitz 1 - - - - - - - - - - 1 -


Auswertung Serie 31 (rote Liste)

Platz Name Ort Summe Aufgabe
361 362 363 364 365 366 367 368 369 370 371 372
1. Rafael Seidel Chemnitz 55 6 3 3 3 4 3 6 10 3 3 8 3
2. Jürgen Urbig Chemnitz 47 6 3 3 3 4 3 4 8 3 3 7 -
3. Uwe Parsche Chemnitz 37 - - - 3 4 3 6 10 3 - 8 -
4. Elisa Parsche Chemnitz 33 - - 3 3 4 3 4 5 3 - 8 -
4. Doreen Naumann Duisburg 33 6 3 3 3 4 2 - 3 3 3 - 3
5. Mike Pfaffe Großenhain 22 - - - - - 3 6 10 3 - - -
6. Sabine Fischbach Hessen 21 6 3 3 2 4 - - - - 3 - -
7. Linus-Valentin Lohs Chemnitz 19 - 3 3 3 4 3 - - - 3 - -
8. Andree Dammann München 17 - - - 3 - - - - - 3 8 3
9. Katrin Wolstein Bamberg 14 - - - - - - - - - 3 8 3
10. Valentin Grundmann Chemnitz 11 - - 3 - 4 - - - - - 4 -
11. Laura Schlosser Chemnitz 10 - - 3 - 4 - - - - - 3 -
11. Felix Karu Altach 10 - 3 3 - 4 - - - - - - -
12. Karolin Schuricht Chemnitz 9 - - 2 - 4 - - - - 3 - -
12. Eden Markos Los Angeles 9 6 3 - - - - - - - - - -
12. Zach Markos Los Angeles 9 6 3 - - - - - - - - - -
13. Adrian Schlegel Chemnitz 7 - - - - 4 - - - - - - -
13. Heinrich Grossinger Chemnitz 7 - - 3 - 4 - - - - - - -
14. Carl Geißler Chemnitz 6 - - 2 - 4 - - - - - - -
14. Lena Rabbeau Chemnitz 6 - - 2 - 4 - - - - - - -
14. Marie Sophie Roß Chemnitz 6 6 - - - - - - - - - - -
14. Luisa Schlosser Chemnitz 6 - - 3 - - 3 - - - - - -
14. Sophie Kalmer Chemnitz 6 - - 2 - 4 - - - - - - -
14. Pauline Marschk Chemnitz 6 - - 2 - 4 - - - - - - -
14. GesaH Chemnitz 6 - - 2 - 4 - - - - - - -
14. Christian Wagner Bamberg 6 6 - - - - - - - - - - -
14. Mara Neudert Chemnitz 6 - - 2 - 4 - - - - - - -
14. Camilla Schreiter Chemnitz 6 - - 2 - 4 - - - - - - -
15. Tobias Morgenstern Chemnitz 5 - - 3 2 - - - - - - - -
16. Elina Rech Chemnitz 4 - - - - 4 - - - - - - -
16. Jonas Frederik Otto Lichtenwalde 4 - - - - 4 - - - - - - -
16. Lisanne Brinkel Chemnitz 4 - - - - 4 - - - - - - -
16. Kai-Lutz Wagner Chemnitz 4 - - - - 4 - - - - - - -
16. Pascal Graupner Chemnitz 4 - - - - 4 - - - - - - -
16. Felicitas Güra Chemnitz 4 - - - - 4 - - - - - - -
16. Elias Schmidt Chemnitz 4 - - - - 4 - - - - - - -
16. Tom Straßer Chemnitz 4 - - - - 4 - - - - - - -
16. Florian A. Schönherr Chemnitz 4 - - - - 4 - - - - - - -
16. Gunnar Reinelt Chemnitz 4 - - - - 4 - - - - - - -
17. Arne Weißbach Chemnitz 3 - - - - 3 - - - - - - -
17. Melanie Petz Chemnitz 3 - - 3 - - - - - - - - -
17. Frederike Meiser Chemnitz 3 - - 3 - - - - - - - - -
17. Marie Berger Chemnitz 3 - - 3 - - - - - - - - -
17. Nele Mäding Chemnitz 3 - - - - 3 - - - - - - -
17. Malte Lohs Chemnitz 3 - - - - - - - - - 3 - -
17. Marcel Seerig Chemnitz 3 - - - 3 - - - - - - - -
17. XXX ??? 3 - 3 - - - - - - - - - -
17. Shari Schmidt Chemnitz 3 - - 3 - - - - - - - - -
17. Robin König Chemnitz 3 - - - - - - - - - 3 - -
17. Moritz Weber Chemnitz 3 - - 3 - - - - - - - - -
17. Saskia Schlosser Chemnitz 3 - - 2 - - - - 1 - - - -
17. Lene Haag Chemnitz 3 - - 3 - - - - - - - - -
17. Susanna Seidler Chemnitz 3 - - 3 - - - - - - - - -
17. Helene Fischer Chemnitz 3 - - 3 - - - - - - - - -
18. Michelle Bühner Chemnitz 2 - - 2 - - - - - - - - -
18. Carlo Klemm Chemnitz 2 - - 2 - - - - - - - - -
18. Elisa Bolte Chemnitz 2 - - 2 - - - - - - - - -
18. Albin Uhlig Chemnitz 2 - - 2 - - - - - - - - -
18. Marie Juhran Chemnitz 2 - - 2 - - - - - - - - -
18. Nicklas Reichert Chemnitz 2 - - 2 - - - - - - - - -
18. Jessica Spindler Chemnitz 2 - - 2 - - - - - - - - -
18. Tobias Richter Chemnitz 2 - - 2 - - - - - - - - -
18. Gwendolin Eichler Chemnitz 2 - - 2 - - - - - - - - -
18. Ida Gwendolin Eichler Chemnitz 2 - - - - - - - - - 2 - -
18. Paula Geißler Chemnitz 2 - - 2 - - - - - - - - -
18. Hannes Hohmann Chemnitz 2 - - 2 - - - - - - - - -
18. Selma Juhran Chemnitz 2 - - 2 - - - - - - - - -
18. Kevin Ngyen Chemnitz 2 - - 2 - - - - - - - - -
18. Celestina Montero Perez Chemnitz 2 - - 2 - - - - - - - - -

Serie 30

Serie 30

Aufgabe 1

349. Wertungsaufgabe

Logikrätsel

Lisa und Maria haben zum Spieleabend eingeladen. Es kamen, Britta Becher, Gina Grau und Frieda Fink. Jede der Fünf hat einen anders farbigen Beutel für die Spielmarken vor sich liegen (rot, grün, blau, grau und gelb). Jede jedem der Beutel sind Spielmarken aus einem anderem Material (Eisen, Kupfer, Holz, Aluminium und Plastik). Die Anzahl der in einem Beutel enthaltenen Spielmarken ist unterschiedlich (50, 60, 70, 80 bzw. 90). Damit ergab sich vor dem eigentlichen Spielbeginn folgende Situation. (Hinweis:
Der Beutel von Gina hat die Farbe wie ihr Name schon sagt. Britta hat die meisten Spielmarken.
Maria hat keinen grünen Beutel vor sich liegen, sie und Frieda haben mehr als die 50 kupfernen Spielmarken.
Friedas Beutel ist nicht der mit den 60 Spielmarken und sie hat keine Spielmarken aus Holz.
Im grünen Beutel sind weniger Spielmarken als im roten Beutel, wobei im roten Beutel die Marken aus Eisen sind.
In dem Beutel von Lisa sind 20 Spielmarken mehr als in dem Beutel mit Aluminium.
Im blauen Beutel sind 70 Spielmarken.
Wer hat welche Anzahl und Art von Spielmarken, welche Farbe haben die Beutel? (5 blaue Punkte)

Bevor sie spielten unterhielten sie sich noch über das Einräumen der Zimmer in der neuen Schule (Zimmernummern: 1, 3, 4, 7 und 8). Jedes des Mädchen war für eines Zimmer verantwortlich gewesen und hatte die Bücher für je ein Unterrichtsfach ( Mathematik, Deutsch, Englisch, Frz. und Geschichte) in „ihren“ Raum gebracht. Die Klassenleiter, die für je einen Raum zuständig sind heißen Arnold, Instetten, Voss, Winkel und Zonk.

  1. Das Französischzimmer hat eine um 1 größere Nummer als das Zimmer von Herrn Winkel. Lisa hatte sich um dieses Zimmer gekümmert.
  2. Maria schleppte Geschichtsbücher.
  3. Das Zimmer von Herrn Zonk hat eine kleinere Nummer als das Zimmer wo Gina geholfen hatte.
  4. Das Zimmer von Herrn Instetten wurde mit Englischbüchern bestückt. Dieses Zimmer und auch das Zimmer wo Frieda geholfen hat, haben eine ungerade Zimmernummer.
  5. Im Zimmer 7 hat nicht das Mädchen gearbeitet, welches die Deutschbücher getragen hat.
  6. Zimmer 3 ist das Zimmer von Frau Arnold.

In welchem Zimmer haben die Mädchen geholfen, welche Bücher sind da jetzt drin und wer ist der jeweilige Klassenleiter? (5 rote Punkte)
Lösung bis zum 15.3.2012 Deadline for solution is 15th. march 2012

logic puzzle
Lisa and Maria have invited to a games evening. There were Brigitta Becher, Gina Grey and Fried Fink. Each of the five had a differently coloured bag of casino tokens in front of herself (red, green, blue, grey and yellow). In each bag are tokens made of a different material (iron, copper, wood, alloy and plastic). The number of tokens in each bag is different, too (50, 60, 70, 80 and 90). Before the actual start of the game there was the following situation:
The colour of Gina's bag matched her surname. Britta had most tokens. Maria didn't have a green bag in front of her. She and Frieda have more than the 50 copper tokens.
Friedas bag is not the one holding 60 tokens and she doesn't have wooden tokens. In the green bag there are less tokens than in the red one, albeit the tokens in the red bag are made of iron.
In Lisa's bag are 20 token more than in the bag holding the alloy tokens.
There are 70 tokens in the blue bag.
Who has got which number and material of tokens and what colour have the bags? - 5 blue points

Before they started playing they talked about how to furnish the rooms in the new school (room numbers 1, 3, 4, 7 and 8). Each of the girls had been responsible for one of the rooms and taken the books for one subject each into 'her' room (Maths, German, English, French and history). The teachers responsible for the rooms are Arnold, Instetten, Voss, Winkel and Zonk.
1. The French room's number is bigger by one than the number of Mr Winkler's room. Lisa took care of this room.
2. Maria had to haul the History books.
3. Mr Zonk's room has a smaller number than the one Gina had helped to fit out.
4. Mr Instetten's room is full of English books. This room has an odd room number just like the room that Frieda had helped with.
5. The girl who carried the German books didn't work in room number 7.
6. Room number 3 is Mrs Arnold's room.
In which room di the girls work, what books are there and who is the teacher? - 5 red points.

Lösung/solution:

auf eine Herleitung wird hier verzichtet:

blau:

Lisa: Holz, grün, 80

Maria: Aluminium, Gelb 60

Britta: Eisen, rot, 90

Gina: Kupfer, grau, 50

Frieda: Plastik, blau, 70

rot:

Lisa: Französisch, 8, Voss

Gina: Mathematik, 7, Winkel

Frieda: Deutsch, 3, Arnold

Britta: Englisch, 1, Instetten

Maria: Geschcihte, 4, Zonk

 


Aufgabe 2

350. Wertungsaufgabe
Bernd fährt mit seiner Familie ans Meer. Die Nummer ihres Wagons ist die 3024. Da meint Bernds Vater. „Das ist ja eine interessante Zahl.“ „Wie so?“, fragt Maria zurück. „Es ist das Produkt von vier aufeinander folgenden natürlichen Zahlen.“ Welche Zahlen sind es? - 2 blaue Punkte
Als sie endlich an ihrem Ferienhaus ankommen, sehen sie, dass die nebeneinander stehenden Häuser von 1 bis 11 durchnummeriert sind. Die Türen weisen zur Straße, die Terrassen haben einen Blick zum Meer.
„Welches ist unser Ferienhaus?“, fragt Bernd. Bernds Vater schmunzelt und meint: „Wenn wir auf der Terrasse mit dem Blick zum Meer sind, dann ist das Produkt der Häuseranzahl auf der linken Seite und der Anzahl der Häuser auf der rechten Seite um drei größer als das entsprechende Produkt, das unser linker Nachbar heraus bekommen würde, wenn er das von seinem Haus aus macht.“ Welche Hausnummer kann das Ferienhaus von Bernds Familie haben? - 3 rote Punkte

Termin der Abgabe 22.03.2012 Deadline for solution is the 23. march 2012.

Bernd goes to the sea with his family for a holiday. The number of their railway wagon is 3024.
Bernd's father remarks: “That's quite an interesting number.”
“Why's that?”, Maria asks wants to know. “Because it's the product of four subsequent numbers.”
What numbers are they? - 2 blue points.
When they arrive at their holiday apartment they see that the neighbouring houses are numbered from 1 to 11. The doors face the street, the terrace look out to the sea.
“Which one is ours?”, Bernd asks. Bernd's dad chuckles and says: “If we are on the terrace looking at the sea you'll find that the product of the number of houses to our left and the number of houses to our right exceeds by three the product that our left hand neighbour would calculate from his terrace accordingly.”
What number does the holiday apartment of Bernd's family have? - 3 red points

Lösung/solution:

blau:

Nun man könnte die Gleichung x * (x +1) * (x + 2) * (x + 3) =3024 oder man probiert mal schnell: 3024 - es kann wegen dert Teilbarkeitsregel für die 5 der faktor fünf nicht dabei sein. Damit kann man 1*2*3*4 oder 6*7*8*9 oder 11*12*13*14 ... probieren und wird mit 6*7*8*9 = 3024 fündig.

rot: Auch hier wird man mit Probieren schnell fertig.

Hausnummer Bernd Hausnummer linker Nachbar Produkt Bernd
(links * rechts)
Produkt linker Nachbar
(links * rechts)
Differenz der Produkte
Bernd - Nachbar
1 kein linker Nachbar
2 1 1*9 = 9 0 9
3 2 2*8 = 16 1*9 = 9 7
4 3 3*7 = 21 2*8 = 16 5
5 4 4*6 = 24 3 *7 = 21 3
6 5 5*5 = 25 4*6 = 24 1
7 6 6*4 = 24 5*5 = 25 -1
8 7 7*3 = 21 6*4 = 24 -3

 

 Wie man sieht passt nur die Aufgabenstellung auf den fall, dass Bernd im Haus 5 wohnt.

Eine Lösung mittels Gleichung stammt von Zach, thanks --> als pdf <--


Aufgabe 3

351. Wertungsaufgabe
Maria hat in ihrem Physikbuch eine Abbildung zweier Thermometer entdeckt. Das eine misst die Temperatur in Grad Celsius, das andere in
Grad Fahrenheit. Bernd kommt ins Zimmer, sieht das Bild und fängt gleich an zu suchen. „Du, schau mal, hier ist eine Formel zum Umrechnen der Einheiten.“  T_F = \frac{9}{5} \cdot T_C + 32 Es sind die Werte TC = 100 °C und TF = 100 °F in die jeweils andere Einheit umrechnen. (3 blaue Punkte) Bei welcher Temperatur zeigen die beiden Thermometer den gleichen Wert an? (3 rote Punkte)

Termin der Abgabe 29.03.2012 Deadline for solution is the 29. march 2012.

Maria has discovered an illustration of two thermometers. One measures the temperature in degree of Celcius, the other uses the Fahrenheit scale. Bernd enters the room, sees the picture and starts searching.
“Look, I here is a formula to convert the units.”

Lösung/solution:

blau:   T_F = \frac{9}{5} \cdot T_C + 32 Das Einsetzen von 100 °C führt auf 212 °F.

 T_F = \frac{9}{5} \cdot T_C + 32 |-32 \\  \frac{9}{5} \cdot T_C = T_F - 32 | \cdot \frac{5}{9} \\ T_C =  \frac{5}{9} \cdot T_F -  \frac{160}{9}

Das Einsetzen von 100 °F führt auf rund 38 °C.

rot: Die beiden Temperaturwerte sollen gleich sein. Viele haben eine Weile probiert, bis der in   T_F = \frac{9}{5} \cdot T_C + 32 eingesetzte Wert, auf das gleiche Ergebnis führt - geht natürlich. Oder aber: 

 T_C = \frac{9}{5} \cdot T_C + 32 |- \frac{9}{5} \cdot T_C  \\ - \frac{4}{5} \cdot T_C = 32 | \cdot - \frac{5}{4} \\ T_C = - 40

- 40 °C und - 40 °F stellen die gleiche Temperatur dar.

 


Aufgabe 4

352. Wertungsausfgabe
„Hallo Mike, kannst du dich noch an die Stammbrüche erinnern?“; fragte Bernd. „Waren das nicht die echten Brüche, deren Zähler immer 1 ist?“ „Stimmt genau“. „Die alten Ägypter haben viel mit solchen Stammbrüchen gerechnet.“ Wie viele von einander verschiedene Stammbrüche braucht man
mindestens, so dass deren Summe größer als 2 wird? (4 blaue Punkte). Welcher Bruch unterscheidet sich genau um 1 von seinem Kehrwert (Reziprok)? (3 rote Punkte)


Termin der Abgabe 05.04.2012 Deadline for solution is the 5. april 2012.

“Hi Mike, do you remember unit fractions?”; Bernd asked. “Do you mean the fractions whose numerator is always 1, but not 1/1?” “Exactly, the old Egyptians used them a lot.” How many different unit fractions do you need at least to add them up to a sum greater than 2? - 4 blue points
Which fraction differs from its reciprocal by exactly 1? - 3 red points

Lösung/solution:

from Eden, thanks --> pdf <--

Deutsch wird nach Ostern ergänzt


Aufgabe 5

353. Wertungsaufgabe
353„Störe meinen Kreis nicht“, sagt Bernd zu Mike, als der ins Zimmer kommt und dessen Schatten auf Bernds Zeichnung fällt. „Der Spruch ist aber nicht von dir und sei froh, dass ich dir nichts tue“, gab Mike schmunzelnd zurück. Mike schaut auf das Blatt von Bernd und erkennt einen Kreis, auf dem Bernd die Punkte A, B, C und D gegen den Uhrzeigersinn eingezeichnet hat. (siehe Bild)
Die vier Punkte werden zu einem Viereck verbunden. Wenn man weiß, dass in einem Dreieck die Innenwinkel zusammen immer 180° ergeben und dass zwei Winkel in einem gleichschenkligen Dreieck gleich groß sind, dann ist es gar nicht so schwer (auch ohne messen) zu zeigen, dass die gegenüberliegenden Winkel des Vierecks zusammen immer gerade 180° ergeben. (4 blaue Punkte). Wie aber zeigt man, dass AC = BC + CD gilt, wenn AB = BD = DA? (6 rote Punkte)


Termin der Abgabe 19.04.2012 Deadline for solution is the 19. april 2012.

353 “Do not disturb my circles”, Bernd says to Mike who enters the room casting a shadow on Bernd's drawing.
“That's not your phrase and anyway, you should be glad I don't want to harm you”, Mike replied with a chuckle. Mike looks at the sheet of paper in front of Bernd and sees a circle, on which Bernd has marked the points A, B, C and D anti-clockwise. (see figure)
The four points are connected to form a quadrilateral. If you keep in mind that the two angles at the base of an isosceles triangle are equal then it shouldn't be too difficult (even without measuring) to show that the opposite angles of our quadrilateral add up to 180°. - 4 blue points
How do you show, that AC = BC + CD?  - if AB = BD = DA - 6 red points

Lösung/solution:


Aufgabe 6

354. Wertungsaufgabe

Maria hat aus Pappe ein Dreieck ausgeschnitten und versucht es auf einer Bleistiftspitze zu balancieren, aber irgendwie will das nicht funktionieren. Bernd schaut eine Weile zu, dann fällt es ihm wieder ein.
„Du musst den Schwerpunkt des Dreiecks finden, dann geht es.“ „Ach, na klar, der Schwerpunkt eines Dreiecks ist doch der Schnittpunkt der Seitenhalbierenden.“ „Genau.“ Zeichne in ein Koordinatensystem die Punkte A(0; 0), B (5; 1) und C (2; 6). Welche Koordinaten hat der
Schwerpunkt des Dreiecks ABC? - 4 blaue Punkte.
Zeichnet man in das Koordinatensystem noch den Punkt D (0; 5) dann lässt sich das Viereck ABCD zeichnen. Welche Koordinaten hat dessen Schwerpunkt? (Berechnung 8 rote Punkte)


Termin der Abgabe 26.04.2012 Deadline for solution is the 26. april 2012.

Maria has cut out a cardboard triangle and tries to balance it on the tip of a pencil, which doesn't really work. Bernd keeps watching for a while before he remembers: “You have to find the centroid of the triangle, then it's easy.”
“Of course, the centroid. Isn't that the point of intersection of the triangle's medians?”
“Exactly.”
Draw a coordinate system and mark points A(0,0), B(5,1) and C(2,6). Which coordinates does the centroid of triangle ABC have? - 4 blue points
In the same coordinate system mark point D(0,5) and draw the quadrilateral ABCD. Which coordinates does its centroid have? - 8 red points for a calculation 

Lösung/ solution:

 schöne Lösung von Jürgen Urbis, danke: --> pdf <--


Aufgabe 7

355. Wertungsaufgabe

 „Vor kurzem hatten wir doch die Zahl des „goldenen Schnittes“ ausrechnet“, sagte Lisa zu Mike. „Stimmt, das war bei der Aufgabe 352. Wenn man eine Strecke AB im Verhältnis des goldenen Schnittes teilen will, so gilt es, einen Punkte C auf der Strecke AB zu finden, so dass z. B.  AB : AC = AC : BC gilt. Der Abschnitt AC ist in dem Fall das größere Teilstück und wird Major genannt, die Strecke BC ist dann der Minor. Ich habe eine schöne Konstruktion entdeckt, wie man den Punkt A finden kann, wenn man die Strecke BC vorgibt.“ „Lass hören.“ „Die Konstruktion ist eine
klassische – also ohne Messung zwischendurch. Die Strecke BC wird halbiert. Dann wird in B eine Senkrechte errichtet. Ein Punkt M auf der Senkrechten mit dem Abstand BC/2 wird festgelegt. Dieser Punkt M wird mit C durch eine Gerade verbunden. Anschließend wird ein Kreis um M
gezeichnet, der den Radius BC/2 hat. Dieser Kreis schneidet die Gerade AM in zwei Punkten X und Y, wobei Y weiter von C entfernt ist als X.
Jetzt wird ein Kreis um C gezogen mit dem Radius CY. Dieser Kreis schneidet die Gerade durch die Punkte C und B in zwei Punkten. Der Schnittpunkt, der weiter von B entfernt ist, wird mit A bezeichnet. Das ist der gesuchte Punkt A, denn es gilt nun  AB : AC = AC : BC“.
Wie lang ist AC, wenn BC 4,2 cm lang ist. (Ermittlung durch Rechnung oder Konstruktion – 4 blaue Punkte.) Es ist zu zeigen, dass die Konstruktionsvorschrift wirklich einen Punkt A ergibt,so dass der goldene Schnitt erfüllt ist. - 4 rote Punkte.

Termin der Abgabe 03.05.2012 Deadline for solution is the 3. may 2012.

355
“A while ago we calculated the value of the golden ratio, didn't we?”, Lisa said to Maik.
“That's right, problem 352 it was. If you want to divide a line segment AB according the golden ratio, you'd have to find a point C on AB so that for example AB : AC equals AC : BC. In this case AC is the bigger part and would be called 'major' while BC is 'minor'. I have come across a beautiful construction to find point A if BC is given.”
“Tell me.”
“It's a classic construction only using compass and straightedge. Bisect line segment BC. Then construct the perpendicular through B. Mark a point M on this perpendicular in a distance of BC/2. Connect M and C. Draw a circle centered at M with a radius of BC/2. This circle intersects line AM in two points X and Y with Y being further away from C than X. Now construct a circle centered at C with a radius of CY. This circle intersects the line through C and B in two points. Let the point of interesction that is further from B be A. That's the point A we were looking for, because AB : AC = AC : BC.”
How long is AC, if BC is 4.2 cm? Solution by calculation or construction – 4 blue points.
Show that above construction really gives you point A of the golden ratio. - 4 red points

Lösung/solution.

fibo7 k

Die Konstruktion verbirgt sich hinter der oberen rechten Konstruktion. Bei der Vorgabe von 4,2 cm ergibt sich AC zu 6,8 cm.

--> Block in groß <--

Solution from Zach, thanks --> pdf <--

 


Aufgabe 8

356. Wertungsaufgabe

„Hallo Bernd, wie ich sehe, hast du wieder mal ein großes Blatt Millimeterpapier aus meinem Schrank genommen“, sagte Bernds Vater. „Wozu brauchst das?“ „Na schau mal, unser Lehrer hat gesagt, dass wir in ein Koordinatensystem die Punkte (1; 5) und (4; 11) eintragen sollen. Diese
Punkte sind durch eine Gerade zu verbinden. Nun hat er uns gefragt, welcher der vier Punkte auch auf dieser Geraden liegt: (120; 243); (120; 244); (120; 245) oder (120; 246). Das Problem ist, dass dein großes Blatt leider nicht ausreicht, um die Punkte einzutragen.“ „Ich sehe auch ohne Eintragung, welcher der Punkte es sein muss“, sagte Bernds Vater. 3 blaue Punkte
Mike kam hinzu und gab Bernds Vater Recht. „Deine lineare Funktion ist ja noch einfach, aber unser Lehrer hat uns beauftragt herauszufinden, wann die Bilder zweier linearer Funktionen (y = f(x) = m1x + n1 und g(x) = m2x + n2) senkrecht zu einander verlaufen.“ - 5 rote Punkte für die
Bedingung und deren Herleitung.

Termin der Abgabe 17.05.2012 Deadline for solution is the 17. may 2012.

“Hi Bernd, I see you've taken a large sheet of graph paper from my cabinet”, Bernd's father said. “Waht dou you need that for?”
“Well look, our teacher said we are to put the points (1,5) and (4,11) into a coordinate system. These points are to be connected by a straight line. Then he asked us which of the following points are part of this line, too: (120,243), (120,244), (120,245) oder (120,246). The only problem is your big sheet of paper isn't big enough to mark these points.”
“I can see which points belong to the line without putting them in”, Bernd's father said. - 3 blue points
Mike joined them and agreed with Bernd's father. “Your linear function is easy enough, but our teacher asked us to find out under which conditions the graphs of the two linear functions (y = f(x) = m1x + n1 und g(x)= m2x + n2) are perpendicular to each other.” - 5 red points for the condition and its explanation

Lösung/solution:



Aufgabe 9

357. Wertungsaufgabe
„Ein Teilnehmer unserer Mathematikgruppe hat vom letzten Känguru-Wettbewerb diese Idee mitgebracht und ich habe mir das noch einmal aufgezeichnet“, sagte Maria zu Bernd, als der neugierig auf das Blatt schaute. 357-1„Das Bild besteht bei mir aus einem Rechteck der Größe (12 und 6 cm) und vier passenden Viertelkreisen. Wenn ich das Bild auseinander schneide,
so erhalte ich das 357-2 .“ Um wie viel Zentimeter unterscheiden sich die Umfänge der beiden Figuren? - drei blaue Punkte
Wie groß sind die Flächeninhalte und Umfänge der beiden Figuren (5 rote Punkte)

Termin der Abgabe 24.05.2012 Deadline for solution is the 24. may 2012.

 
“Someone from our maths group showed me this idea from the last Kangaroo Competition. I've drawn it again”, Maria said to Bernd who was closely examining the sheet of paper. 357-1
“The picture consists of a rectangle of 12 by 6 cm and four fitting quarters of a circle. If I cut the picture apart, I'll get this one.” 357-2
What's the difference between the perimeters of the two figures in centimetres? - 3 blue points
How big are the areas and perimeters of the two figures? - 5 red points

Lösung/solution:



Aufgabe 10

358. Wertungsaufgabe


„Das sieht aber interessant aus“, sagte Lisa zu Mike, der mit Zirkel und Lineal die Bogenblume konstruiert hatte.358 „Das war gar nicht so schwer, denn das Quadrat (4 cm) hatte ich schon und dann habe ich nur noch einen Kreis und vier Halbkreise passend konstruiert.“
Wie wird die Bogenblume konstruiert? - Konstruktionsbeschreibung 3 blaue Punkte
Wie groß ist der Flächeninhalt der vier grünen Flächen? - 4 rote Punkte.


Termin der Abgabe 31.05.2012 Deadline for solution is the 31. may 2012.


“That does look interesting”, Lisa said to Mike who had constructed a flower using compass and ruler. 358
“And it's not very difficult, because the square (4 cm) was given and I had only to construct the circle and the two semi-circles so they would fit.”
How is such a flower constructed? - 3 blue points for a description
What is the area of the four green shapes? - 4 red points

Lösung/solution:

blau: Zuerst zeichnet man das Quadrat mit der Kantenlänge von 4 cm.  Jetzt werden die Diagonalen des Quadrates gezeichnet. Deren Schnittpunkt ist der Mittelpunkt eines Kreises mit dem Radius der halben Diagonale. Nun werden die Seiten des Quadrtes halbiert. (mal ganz kurz --> Grundkonstruktion). Die so erhaltenen Mittelpunkte sind auch die Mittelpunkte der Halbkreises (mit r = halber Quadratseite).
rot: Für ein Quadrat der Länge a.
Mal man die gesamte Figur aus, so setzt diese sich aus dem Quadrat und den vier Halbkreisen zusammen. Davon muss man den Kreis mit dem Radius = halbe Diagonale (a/2* Wurzel(2) abziehen.
 A = a^2 + 4 \cdot \frac{1}{2} \cdot \Pi \cdot { (\frac{a}{2})}^2 - \Pi \cdot ( (\frac {a}{2}) \sqrt {2})^2 \\ A = a^2 + 2 \cdot  \Pi \cdot { (\frac{a^2}{4})} - \Pi \cdot ( (\frac {a^2}{4}) \cdot 2) \\ A = a^2
Die vier grünen Bogen haben den gleichen Flächeninhalt wie das Quadrat. Diese Aufgabe ist eine Anwendung von
http://schulmodell.eu/unterricht/84-unterrichtsfaecher/mathematik-unterricht/mathematik-themen/mathelexikon/1714-moendchen-des-hippokrates.html


Aufgabe 11

359. Wertungsaufgabe
„Musst du wieder mal die zweistelligen Quadratzahlen auswendig lernen?“, fragte Bernds Vater etwas verwundert, als er sah, dass sein Sohn diese Zahlen notierte. „Aber nein, ich stelle dreistellige Zahlen der Form ABC zusammen und zwar so, dass AB, aber auch BC jeweils eine Quadratzahl von meiner Liste sind.“ Pro Zahl gibt es einen blauen Punkt, wenn man zeigt, dass man mit Sicherheit keine solche Zahl ABC übersehen hat. Welches ist die größte Zahl ABCD..., die sich finden lässt, wenn immer zwei benachbarte Ziffern der Zahl eine Quadratzahl bilden? (AB; BC; CD; ...) - 2 rote Punkte

Termin der Abgabe 07.06.2012 Deadline for solution is the 7. june 2012.

“Do you have to memorize square numbers of double-digit numbers again?”, Bernd's dad asked a little surprised, when he saw his son note down these numbers.
“Of course not, I only make three-digit numbers of the form ABC so that AB is a square number of my list but also BC.”
You get one blue point for each of these numbers if you can show you haven't missed one of these ABC numbers.
Which one is the biggest number ABCD... that you can find, if two neighbouring digits of this number form a square number? (AB,BC;CD; … ) - 2 red points

Lösung/solution:


Aufgabe 12


360. Wertungsaufgabe
„Hallo Maria, bist du unter die Glücksspieler gegangen?“, fragte Bernd, als er seine Schwester mit den vielen Würfeln sah. „Aber nein, ich setze aus diesen acht Spielwürfeln (Zahlen 1, 2, …, 6 auf je einer Seite) einen Würfel (der Stufe 2) zusammen, so dass die von außen sichtbaren Zahlen eine größtmögliche Summe ergeben. Die kleinstmögliche Summe habe ich schon probiert.“ Wie groß ist die kleinstmögliche bzw. größtmögliche Summe? 4 blaue Punkte. Nimmt man 27 kleine Würfel, kann man den nächstgrößeren Würfel (Stufe 3) zusammensetzen, die Stufe vier erfordert dann schon 64 Spielwürfel. Bei welcher Stufe ergibt sich zum ersten Mal  eine Summe von mehr als 1000, wenn die Summe der sichtbaren Zahlen maximal sein soll. Bis zu welcher Stufe muss man kommen, wenn die Summe der sichtbaren Zahlen minimal sein soll, aber trotzdem mehr als 1000 ergeben soll. 6 rote Punkte.

Termin der Abgabe 07.06.2012 Deadline for solution is the 14. june 2012.

360
“Hi Maria, have you joined the gamblers, now?”, Bernd asked when he saw his sister with so many dice. “Of course not, I use these eight dice (one of the numbers 1, 2, … , 6 on each of the six faces)” to make a bigger (2nd order) cube, so that the pips facing outwards make the biggest possible sum. I've already found the smallest sum.” What is the smallest, what is the biggest possible sum? - 4 blue points With 27 dice you can make a 3rd order cube, for a 4th order cube you'd need 64 dice. What order would give you a sum of more than 1000 for the first time, if the highest possible number of pips faced outwards? What order would you need if the smallest possible sum of pips faced outwards, but still amount to more than 1000? - 6 red points

Lösung/solution:






Auswertung Serie 30 (blaue Liste)

Platz Name Ort Summe Aufgabe
349 350 351 352 353 354 355 356 357 358 359 360
1. Zach Markos Los Angeles 43 5 2 3 4 4 4 4 3 3 3 4 4
1. Rafael Seidel Chemnitz 43 5 2 3 4 4 4 4 3 3 3 4 4
2. Gunnar Reinelt Chemnitz 39 5 2 3 1 4 4 4 3 3 3 4 3
2. Doreen Naumann Duisburg 39 5 2 3 4 - 4 4 3 3 3 4 4
2. Sabine Fischbach Hessen 39 5 2 3 4 4 4 - 3 3 3 4 4
3. Jürgen Urbig Chemnitz 38 - 2 3 4 4 4 4 3 3 3 4 4
4. Linus-Valentin Lohs Chemnitz 35 5 2 3 2 4 - 2 3 3 3 4 4
6. Andree Dammann München 30 - 2 3 - 4 4 4 3 3 3 4 -
5. Valentin Grundmann Chemnitz 32 5 2 3 - 4 4 4 - - 3 4 3
7. Eden Markos Los Angeles 29 5 - 3 4 - - - 3 3 3 4 4
7. Heinrich Grossinger Chemnitz 29 5 2 - - 4 4 4 - - 3 4 3
8. Tom Straßer Chemnitz 24 5 2 3 - 4 - 4 - - 3 - 3
9. Tobias Morgenstern Chemnitz 22 5 2 3 4 - - - - - - 4 4
10. Melanie Petz Chemnitz 17 5 - 3 - - - - 3 3 3 - -
11. Felix Karu Altach 15 - - 3 - 4 4 - - - - - 4
12. Nele Mäding Chemnitz 14 5 - 3 - - - - - 3 3 - -
12. Lukas Thieme Chemnitz 14 5 2 3 4 - - - - - - - -
13. Celestina Montero Perez Chemnitz 13 - 2 3 4 - - - - - - - 4
14. Lene Haag Chemnitz 12 - 2 3 3 - - - - - - - 4
14. Adrian Schlegel Chemnitz 12 - 2 - - - - - - 3 3 4 -
14. Henrike Grundmann Chemnitz 12 - 2 2 4 - 4 - - - - - -
15. Ellen Wilde Chemnitz 11 5 2 - 4 - - - - - - - -
15. Marie Juhran Chemnitz 11 - - 3 4 - - - - - - - 4
15. Felicitas Güra Chemnitz 11 5 - 3 - - - - 3 - - - -
15. Julia Voigt Chemnitz 11 5 2 - 4 - - - - - - - -
16. Helene Fischer Chemnitz 10 4 2 - 4 - - - - - - - -
17. Astrid Fischer Chemnitz 9 - 2 3 4 - - - - - - - -
17. Felix Taubert Chemnitz 9 - 2 3 4 - - - - - - - -
17. Arne Weißbach Chemnitz 13 5 - - - - - 4 - - - - 4
17. Hannah Gebhardt Chemnitz 9 - 2 3 4 - - - - - - - -
17. Emilie Grossinger Chemnitz 9 - 2 3 4 - - - - - - - -
17. Paula Hartmannsdorf 9 - 2 3 - - - - - - - - 4
17. PC Zerbe Leipzig 9 - 2 3 4 - - - - - - - -
17. Frederike Meiser Chemnitz 9 - 2 3 4 - - - - - - - -
17. Tim Jechorek Chemnitz 9 - 2 3 4 - - - - - - - -
17. Leon Hoppe Chemnitz 9 - 2 3 4 - - - - - - - -
17. Felicitas Hastedt Chemnitz 9 - - 3 - - - - - 3 3 - -
18. Marcel Reichelt Chemnitz 8 - 2 2 4 - - - - - - - -
18. Elina Rech Chemnitz 8 5 - 3 - - - - - - - - -
18. Theresa Jänich Chemnitz 8 - 2 2 4 - - - - - - - -
19. Jessica Spindler Chemnitz 7 - - 3 4 - - - - - - - -
19. Kevin Ngyen Chemnitz 7 - - 3 4 - - - - - - - -
19. Anna Georgi Chemnitz 7 5 2 - - - - - - - - - -
19. Emily Neuwirth Chemnitz 7 - 2 2 3 - - - - - - - -
19. Hannes Hohmann Chemnitz 7 - - 3 4 - - - - - - - -
20. Simon Winger Chemnitz 6 - - - - - - - - 3 3 - -
20. Cynthia Raschkowsky Chemnitz 6 - - - - - - - - 3 3 - -
20. Marie Berger Chemnitz 6 - 2 - 4 - - - - - - - -
20. Tobias Richter Chemnitz 6 - 2 - 4 - - - - - - - -
20. Valentin Sellin Chemnitz 6 - 2 - - - - 4 - - - - -
20. Nicklas Reichert Chemnitz 6 - 2 - 4 - - - - - - - -
20. Moritz Duderstadt Chemnitz 6 - 2 - 4 - - - - - - - -
20. Joel Magyar Chemnitz 6 - 2 - 4 - - - - - - - -
20. Luis Raupach Chemnitz 6 - 2 - 4 - - - - - - - -
20. Lena Elisa Penzlin Chemnitz 6 - 2 - 4 - - - - - - - -
20. Anna Grünert Chemnitz 6 - - 2 4 - - - - - - - -
20. Ole Koelb Chemnitz 6 - - 2 4 - - - - - - - -
21. Christian Wagner Bamberg 5 5 - - - - - - - - - - -
21. Franz Kemter Chemnitz 5 - 2 - 3 - - - - - - - -
21. Annika Theumer Chemnitz 5 5 - - - - - - - - - - -
21. Ulrike Böhme Chemnitz 5 - 2 - - - - - - - 3 - -
21. Karl Herrmann Chemnitz 5 - 2 - - - - - - - - - 3
21. Friederike Lenk Chemnitz 5 - 2 - 3 - - - - - - - -
22. Willy Stöckel Chemnitz 4 - - - 4 - - - - - - - -
22. Paula Mühlmann Dittersdorf 4 - - - 4 - - - - - - - -
22. XXX ??? 4 - - - - - - - - - - - 4
22. Amarin Roßberg Chemnitz 4 - - - 4 - - - - - - - -
22. Lukas Kirchberg Chemnitz 4 - - - 4 - - - - - - - -
22. Gabriel Dammann München 4 - 2 - - - - - - 2 - - -
22. Ernesto Uhlmann Chemnitz 4 - - - 4 - - - - - - - -
22. Marvin Gülden Chemnitz 4 - - - - 4 - - - - - - -
22. Carlo Klemm Chemnitz 4 - - - 4 - - - - - - - -
22. Svenja Reinelt Chemnitz 4 - - - - - 4 - - - - - -
22. Tom Hartig Chemnitz 4 - - - 4 - - - - - - - -
23. Elena Oelschlägel Chemnitz 3 - - - - - - - - - - 3 -
23. Julia Ritter Chemnitz 3 - - - 3 - - - - - - - -
23. Elisa Bolte Chemnitz 3 - - - - - - - - - 3 - -
23. Julian Vass Chemnitz 3 - - - 3 - - - - - - - -
23. Ida Heuschkel Chemnitz 3 - - 3 - - - - - - - - -
23. Malte Lohs Chemnitz 3 - - - - - - - - - 3 - -
23. Selma Juhran Chemnitz 3 - - 3 - - - - - - - - -
23. Lilli Weiß Chemnitz 3 - - - - - - - - 3 - - -
23. Simon Anders Chemnitz 3 - - - - - - - - - 3 - -
23. Shari Schmidt Chemnitz 3 - - - 3 - - - - - - - -
23. Lisanne Brinkel Chemnitz 3 - - - - - - - - 3 - - -
23. Jessica Ritter Chemnitz 3 - - - - - - - - - 3 - -
24. Joshua May Chemnitz 2 - 2 - - - - - - - - - -
24. Sten-Niclas Wolter Chemnitz 2 - 2 - - - - - - - - - -
24. Hanna Kallenbach Chemnitz 2 - 2 - - - - - - - - - -
24. Clara Stöckel Chemnitz 2 - 2 - - - - - - - - - -
24. Luisa Franke Chemnitz 2 - 2 - - - - - - - - - -
24. Maria Dreßler Chemnitz 2 - 2 - - - - - - - - - -
24. Emmely Schöne Chemnitz 2 - 2 - - - - - - - - - -
24. Tim Missullis Chemnitz 2 - 2 - - - - - - - - - -
24. Jule Irmscher Eibenberg 2 - 2 - - - - - - - - - -
24. Josephine Klotz Chemnitz 2 - 2 - - - - - - - - - -
24. Moritz Weber Chemnitz 2 - 2 - - - - - - - - - -
24. Eva-Lotta Rümmler Chemnitz 2 - 2 - - - - - - - - - -
24. Michelle Wade Chemnitz 2 - 2 - - - - - - - - - -
24. Erik Walther Chemnitz 2 - 2 - - - - - - - - - -
24. Marvin Köllner Chemnitz 2 - 2 - - - - - - - - - -
24. Tim Sigmund Chemnitz 2 - 2 - - - - - - - - - -
24. Pit Hopke Chemnitz 2 - 2 - - - - - - - - - -
24. Katharina Zweiniger Chemnitz 2 - 2 - - - - - - - - - -

Auswertung Serie 30 (rote Liste)

Platz Name Ort Summe Aufgabe
349 350 351 352 353 354 355 356 357 358 359 360
1. Zach Markos Los Angeles 54 5 3 3 3 6 8 4 5 5 4 2 6
1. Rafael Seidel Chemnitz 54 5 3 3 3 6 8 4 5 5 4 2 6
2. Jürgen Urbig Chemnitz 49 - 3 3 3 6 8 4 5 5 4 2 6
3. Doreen Naumann Duisburg 47 5 3 3 3 - 8 3 5 5 4 2 6
4. Sabine Fischbach Hessen 41 5 3 3 2 - 8 - 3 5 4 2 6
5. Andree Dammann München 34 - 3 3 - - 8 4 5 5 4 2 -
6. Eden Markos Los Angeles 28 5 - 3 3 - - - - 5 4 2 6
7. Felix Karu Altach 23 - - 3 - 6 8 - - - - - 6
8. Linus-Valentin Lohs Chemnitz 22 3 3 3 3 - - - - 4 4 2 -
9. Gunnar Reinelt Chemnitz 16 5 2 3 - - - - - 4 - 2 -
9. Valentin Grundmann Chemnitz 16 5 2 3 - - - - - - 4 2 -
10. Heinrich Grossinger Chemnitz 13 5 2 - - - - - - - - 2 4
11. Melanie Petz Chemnitz 10 5 - - - - - - - 5 - - -
12. Astrid Fischer Chemnitz 9 - 3 3 3 - - - - - - - -
12. PC Zerbe Leipzig 9 - 3 3 3 - - - - - - - -
13. Felix Taubert Chemnitz 7 - 3 3 1 - - - - - - - -
13. Ulrike Böhme Chemnitz 7 - 3 - - - - - - - 4 - -
14. Julia Voigt Chemnitz 6 - 3 - 3 - - - - - - - -
14. Paula Hartmannsdorf 6 - 3 3 - - - - - - - - -
14. Arne Weißbach Chemnitz 6 6
14. Moritz Weber Chemnitz 6 - 3 3 - - - - - - - - -
14. XXX ??? 6 - - - - - - - - - - - 6
15. Cynthia Raschkowsky Chemnitz 5 - - - - - - - - 5 - - -
15. Christian Wagner Bamberg 5 5 - - - - - - - - - - -
15. Annika Theumer Chemnitz 5 5 - - - - - - - - - - -
16. Malte Lohs Chemnitz 4 - - - - - - - - - 4 - -
17. Lukas Thieme Chemnitz 3 - - 3 - - - - - - - - -
17. Willy Stöckel Chemnitz 3 - - - 3 - - - - - - - -
17. Elena Oelschlägel Chemnitz 3 - - - - - - - - - 3 - -
17. Lisanne Brinkel Chemnitz 3 - - - - - - - - 3 - - -
17. Marcel Reichelt Chemnitz 3 - - - 3 - - - - - - - -
17. Tom Hartig Chemnitz 3 - - - 3 - - - - - - - -
17. Stefan Giron Dresden 3 - - - 3 - - - - - - - -
17. Gabriel Dammann München 3 - 3 - - - - - - - - - -
17. Ernesto Uhlmann Chemnitz 3 - - - 3 - - - - - - - -
17. Ole Koelb Chemnitz 3 - - - 3 - - - - - - - -
17. Marie Juhran Chemnitz 3 - - 3 - - - - - - - - -
17. Emilie Grossinger Chemnitz 3 - - - 3 - - - - - - - -
17. Nele Mäding Chemnitz 3 - - - - - - - - 3 - - -
17. Hannah Gebhardt Chemnitz 3 - - - 3 - - - - - - - -
17. Leon Hoppe Chemnitz 3 - - 3 - - - - - - - - -
17. Tobias Richter Chemnitz 3 - 3 - - - - - - - - - -
17. Frederike Meiser Chemnitz 3 - - 3 - - - - - - - - -
17. Hannes Hohmann Chemnitz 3 - - 3 - - - - - - - - -
17. Celestina Montero Perez Chemnitz 3 - - 3 - - - - - - - - -
17. Ida Heuschkel Chemnitz 3 - - 3 - - - - - - - - -
17. Selma Juhran Chemnitz 3 - - 3 - - - - - - - - -
17. Kevin Ngyen Chemnitz 3 - - 3 - - - - - - - - -
17. Luis Raupach Chemnitz 3 - - - 3 - - - - - - - -
17. Lilli Weiß Chemnitz 3 - - - - - - - - 3 - - -
17. Tom Straßer Chemnitz 3 - - 3 - - - - - - - - -
17. Helene Fischer Chemnitz 3 - 3 - - - - - - - - - -
17. Simon Winger Chemnitz 3 - - - - - - - - 3 - - -
17. Lene Haag Chemnitz 3 - - 3 - - - - - - - - -
17. Felicitas Hastedt Chemnitz 3 - - - - - - - - 3 - - -
18. Rodrigo Vergara Luxemburg (Luxemburg) 2 - - - - 2 - - - - - - -
18. Anna Georgi Chemnitz 2 - 2 - - - - - - - - - -
18. Anna Grünert Chemnitz 2 - - 2 - - - - - - - - -
18. Lukas Kirchberg Chemnitz 2 - - - 2 - - - - - - - -
18. Adrian Schlegel Chemnitz 2 - - - - - - - - - - 2 -
19. Theresa Jänich Chemnitz 1 - - - 1 - - - - - - - -
19. Friederike Lenk Chemnitz 1 - - - 1 - - - - - - - -



Serie-29

Serie 29

Aufgabe 1

337. Wertungsaufgabe

Maria hatte vier Bekannte eingeladen. Hans, Georg, Otto und Moritz. Jeder wohnt in einer anderen Stadt: Chemnitz, Dresden, Leipzig und Zwickau. Schade, dass einer von denen nicht liest, einer keine Hunde mag, einer jünger bzw. kleiner als Maria ist. Andererseits gibt es einen Musikliebhaber, einen Triathleten, einen, der gern spazieren geht und einen Tischtennisspieler. Wer kommt woher und welche gute bzw. nicht so gute Eigenschaft hat er in den Augen von Maria?
1. Otto ist kleiner als Maria
2. Moritz ist nun gerade kein Triathlet.
3. Der aus Dresden ist jünger als Maria. Der Chemnitzer mag – wie Maria auch – die Musik, ist aber nicht der, der keine Hunde mag.
4. Der leidenschaftliche Spaziergänger ist gar kein Freund von Büchern – schade eigentlich.
5. Georg aus Zwickau mag Hunde. Ach ja, der Tischtennisspieler mag Hunde sehr.
6 blaue Punkte.
Abends gab es noch ein kleines Partyspiel. Die Jungs stellten sich nebeneinander auf. (Start bei A, B, C und D von links nach rechts gesehen.). Sie sollten mit drei Bällen jonglieren und dabei so schnell wie möglich 10 m zurücklegen. Hier werden nun sogar noch die Nachnamen (Ahrend, Berlin, Händler und Wächter) der 4 bekannt. Wie heißen die 4 mit vollständigem Namen, wer startete von welcher Position und in welcher Reihenfolge kamen sie ins „Ziel“?
1. Otto Ahrend stand beim Start direkt rechts neben dem, der dann gewann.
2. Der Starter vom Punkt D wurde nicht 3.
3. Händler, der nicht Hans heißt, wurde Zweiter.
4. Moritz startete von C aus, während Wächter direkt links neben Moritz startete.
5 rote Punkte

englisch version

Maria had invited four friends: Hans, Georg, Otto and Moritz. Each of them lived in another town: Chemnitz, Dresden, Leipzig and Zwickau. It's a pity, that one of them doesn't read, another one dislikes dogs, one is younger and one is smaller than Maria. On the other hand, there is one great lover of music, one triathlete, on who likes going walks and a table tennis player. Who lives where and which positive or not quite so positive qualities does he have in Maria's eyes?

  1. Otto is smaller than Maria.

  2. Moritz isn't exactly a triathlete.

  3. The person from Dresden is younger than Maria. The one from Chemnitz is - just like Maria – very fond of music, but is not the person who doesn't like dogs.

  4. The one who likes walks is not a friend of books, which is unfortunate if you think about it.

  5. Georg from Zwickau likes dogs. The table tennis player likes dogs a lot.

6 blue points.

In the evening there is a little party game: All the boys form a row. (A, B, C and D from left to right.) Now they have to juggle with three balls while running a distance of 10 metres. At this point we even get to know the boys' surnames (Ahrend, Berlin, Haendler and Waechter). What are their full names, who starts from which position, and in what order do they arrive?

  1. Otto Ahrend started right of the later winner.

  2. The one who started from position D wasn't third.

  3. Haendler, whose first name isn't Hans, came second.

  4. Moritz started from C, while Wächter started left of Moritz.

5 red points.

Lösung/solution:

Hier die Lösung im Stil des PM-Logiktrainers von Jürgen Urbig, danke.

--> als pdf <--


 

Aufgabe 2

338. Wertungsaufgabe

„Was gestaltest du denn da für ein Muster?“, fragte Lisa. „Ich habe hier einen Kreis mit einem Radius von 2,0 cm. Nun möchte ich genauso große Kreise um diesen herum zeichnen. Alle Kreise sollen den ersten berühren. Die Außenkreise dürfen sich auch berühren, aber nicht überschneiden.“, sagte Mike. Wie viele Kreise passen da wohl um den Kreis und wie kann man das konstruieren, ohne ein Lineal zum Messen zu verwenden? Es gibt pro Kreis einen blauen Punkt und weitere blaue Punkte für die Beschreibung. Nur eine Konstruktion abzugeben, zählt nicht. Wie groß ist der Flächeninhalt der „Lücken“ - zwischen Ausgangskreis und Außenkreisen, also der Flächeninhalt, der nicht bedeckt wird? 6 rote Punkte

english version

“What's that pattern you're creating?”, Lisa asked. “A flower?”

“I've got a circle with a radius of 2 cm. Now I'd like to draw circles with the same radius around this circle. All of these circles should be tangent to the first circle. The outer circles may be tangent to each other but shouldn't intersect.”, Mike explained. How many of these circles fit around the circle and how can you construct it without measuring.

One blue point for each circle and another one for describing the construction. “What's that pattern you're creating?”, Lisa asked. “A flower?”

“I've got a circle with a radius of 2 cm. Now I'd like to draw circles with the same radius around this circle. All of these circles should be tangent to the first circle. The outer circles may be tangent to each other but shouldn't intersect.”, Mike explained. How many of these circles fit around the circle and how can you construct it without measuring.

One blue point for each circle and another one for describing the construction. “What's that pattern you're creating?”, Lisa asked. “A flower?”

Drawing alone won't win a point.Drawing alone won't win a point.

Waht's the size of the area of the “gaps” between the circles, the area not covered by circles? - 6 red points.

Lösung/solution:

Solution from Markus, thanks --> pdf <--


Aufgabe 3

339. Wertungsaufgabe
„Du hattest doch letzte Woche die Aufgabe mit dem Zirkel“, sagte Bernd. „Stimmt, wieso?“, fragte Lisa zurück. „Der kann dir bestimmt bei der folgenden Aufgabe helfen.“ Es gibt einen quadratischen Pool (Kantenlänge 10 m und randvoll mit Wasser). Auf dem schwimmt ein Schiff, dessen Mast von einer Ecke 9,22 m und von einer benachbarten Ecke 2,24 m entfernt ist. Kann man mit einem 1,2 m langen Stab das Schiff erreichen? Begründete Antwort (Rechnung oder Konstruktionsbeschreibung) 4 blaue Punkte. Kann man die Spitze des Mastes mit dem Stab erreichen, wenn der Mast 1 m über dem Wasserspiegel aufragt. Wenn es nicht klappt, wie lang müsste der Stab sein? 3 rote Punkte
Anmerkung: Hand oder Arm wird nicht über dem Wasser eingesetzt.

english version
“Your last maths problem was the one using the compass, wasn't it?”, Bernd asked.

“That's right, what about it?” Lisa asked back.

“Because the same instrument may be of help for the following task.”

There is a pool in the shape of a 10-metre-square full of water. On it floates a ship whose mast is 9.22 metres away from one corner and 2.24 metres away from the neighbouring corner. Can you reach the ship with a rod of 1.2 metres length?

Answer with rationale – 4 blue points.

Can you reach the top of the mast if the mast rises 1 metre above the surface of the water. In case that is not possible: how long would the rod have to be? - 3 red points.

Note: hand or arm cannot be used to reach over the water.

Lösung/solution:

the solution from Zach, thanks --> pdf <--

Die Lösung von Linus-V, danke --> pdf <--


Aufgabe 4

340. Wertungsaufgabe

„Hallo Lisa, was grübelst du denn vor dich hin?“, fragte Mike, als Lisa vor sich hin starrend, ihn fast umrannte. „Entschuldigung, mir ging gerade die Idee eines Zahlenwörterbuches durch den Kopf.“ „Zahlenwörterbuch?“ Nun, alle natürlichen Zahlen werden als Zahlwort aufgeschrieben und dann werden die Zahlwörter – wie in einem Wörterbuch – alphabetisch geordnet. Ich habe mich gerade gefragt, an der wie vielten Stelle das Wort Eins kommt.“
An welche Stelle kommt die Eins, wenn die Zahlen von Null bis Neunundneunzig im Wörterbuch stehen sollen – 3 blaue Punkte. An welche Stelle kommt die Eins, wenn die Zahlen von Null bis zur Neunhundertneunundneunzigtausendneunhundertneunundneunzig im Wörterbuch stehen sollen – 6 rote Punkte. (bei rot gibt es – je nach Sprechweise – mehrere Lösungen)
Deutsches Wörterbuch!

english version
“Hi Lisa, what on earth are you thinking about?”, Mike asked when he got very nearly knocked over by Lisa who was gazing absent-mindedly into space.
“Oh, I'm sorry. I was mulling over the concept of a number dictionary.”
“A number dictionary?”
“Well, all natural numbers are written down as numerals and then ordered alphabetically, just like a dictionary. I was just asking myself in what position you'd find the numeral 'one'.”
Where would you find numeral 'one' if our dictionary contained numerals from zero to ninety-nine? - 3 blue points.
Which position would 'one' have if our dictionary contained all numerals from zero to nine hundred ninety-nine thousand nine hundred ninety-nine? - 6 red points
Use numerals without 'and'!

Lösung/solution:

blau: vor der EINS also e gibt es die Buchstaben, a, b, c und d. Zahlen die mit a anfangen: 8; 18, 28; 38; ..., 98 und die 80. Es sind also 11 Zahlen, die mit "A" beginnen. Das selbe bei d: 3; 13; ..., 93 und 30. Die Buchstaben b und c gibt es nicht am Anfang. Das "E" gibt es bei 11, 21, ... und 91., aber die alphabetisch hinter EI von Eins. Alle anderen Zahlwörter kommen danach im Alphabet. Die EINS steht also an 23. Stelle im Zahlenwörterbuch.

rot: Es sind systematatisch die dreistelligen Zahlen, dann die vierstelligen Zahlen usw. zu untersuchen. Unterschiede ergeben sich ob man Hundert (nach der EINS) oder aber Einhundert (vor der EINS) sagt. Bei vierstelligen Zahlen ist die Sprechweise ohne Ein oder mit ein nicht entscheidend - kommt in jedem Fall dahinter. ...... Je nach Variante kommt die EINS erst deutlich nach der 300.000sten bz. 400.000sten Stelle.

 


Aufgabe 5

341. Wertungsaufgabe
„Hallo Bernd, das war ja eine knifflige Aufgabe, so ein Wörterbuch zu erstellen“, sagte dessen Opa, als er diese Aufgabe – mit etwas Mühe - gelöst hatte. „Kennst du noch das Verfahren der Umwandlungen von gemeinen Brüchen in Dezimalbrüche?“ „Aber klar doch. Ich muss den Zähler des Bruches durch den Nenner dividieren. Als Ergebnis erhalte ich dann entweder einen endlichen oder einen periodischen Dezimalbruch.“ „Stimmt genau. Wenn ich dir einen gemeinen Bruch mit einem zweistelligen Nenner gebe – gekürzt habe ich schon – kannst du mir dann auch ohne Rechnen sagen, ob es ein endlicher oder ein periodischer Dezimalbruch wird?“ Für welche zweistelligen Nenner erhält man einen endlichen Dezimalbruch? Für je drei gefundene Nenner gibt einen blauen Punkte und noch zwei dazu für eine gute Begründung. Für 4 rote Punkte ist der Bruch 1/5 umzuwandeln, allerdings soll dies im Binärsystem erfolgen.

english version

“Hi Bernd, that was quite a hard problem, making such a dictionary”, his grandfather said, when he had solve the task – with some effort. “Say, do you still remember how to transform a common fraction into decimals?”
“Of course I do. I only have to divide the numerator by the denominator. As a result I either get a finite or infinite decimal fraction.”
“Exactly, if I showed you a common fraction with a two-digit denominator, which I have already cancelled down, would you be able to tell me without calculation whether you'd get a finite or infinite decimal fraction?”
Which two-digit denominators make a finite decimal? - one blue point for each set of three denominators plus two extra for a solid explanation. For 4 red points transform 1/5 using the binary system.

Lösung/solution:

 


Aufgabe 6

342. Wertungsaufgabe
„Hallo Bernd, was hast du denn eingekauft?“, fragte Mike. „Das ist eine ganz tolle Vase für meine Mutter zum Geburtstag.“ Da die so groß ist, vermute ich mal, dass die ziemlich teuer war.“ „Stimmt, aber ich hatte genug Geld (auch wenn es weniger als 100 Euro waren) mit, um sie zu bezahlen. Dir kann ich ja sagen, was die Vase gekostet hat. Als ich los ging, um die Vase zu kaufen, habe ich mein Geld gezählt. Die Vase hat genau die Hälfte von dem gekostet, was ich an Geld gezählt hatte. Jetzt habe ich halb so viele Euros wie vorher Cent, aber genau die gleiche Anzahl Cent wie vorher Euros.“ Mike schaute erst etwas verwirrt, aber dann wusste er, was Bernd bezahlt hatte. (3 blaue Punkte) Die „blaue Aufgabe“ ist ein Klassiker. Für 3 rote Punkte ist zu zeigen, ob es auch geht, einen Geldbetrag zu haben, so dass die Vase nur 1/3 des vorhandenen Geldes kostet (weniger als 100 Euro) und Bernd am Ende 2/3 so viele Euros hat wie vorher Cent, aber so viele Cent wie vorher Euros.

Zu lösen bis 12.11.2012. Deadline for solution is 12. january 2012

english version

problem 342
"Hi Bernd, what on earth did you buy?", Mike asked.
"It is an absolutely lovely vase for my mom's birthday."
"At that size it must have cost a fortune, I suppose."
"It sure has, but I had enough money to pay it (even though it was less than 100 Euros). I can tell you what it cost. Before I went shopping I had counted my money. The vase cost exactly half as much as I had counted. Now I have half as many Euros as I had Cents, but exactly as many Cents as I had Euros."
Mike looks a bit puzzled at first, but the he knew what Bernd had paid. (3 blue points)
The "blue problem" is a classic. For three red points you have to show whether there is an amount of money so that the vase cost only 1/3 of the money available (less then 100 Euros) and Bernd has two thirds as many Euros as he had Cents, but as many Cents as he had Euros.

Lösung/solution:

blau und rot Lösung von J. Urbrig, danke --> pdf <--

solution (blue and red) from Z. Markos, thanks --> pdf <--

 


Aufgabe 7

343. Wertungsaufgabe
Bernd und Mike sehen sich Bilder ihrer letzten Radtour an. Eine der Etappen führte von Linz nach Wien, das sind rund 195 Kilometer. „Da waren wir ganz schön schnell unterwegs“, meinte Bernd. „Das stimmt, aber wenn wir im Durchschnitt noch 4 km/h mehr hätten fahren können (oder wollen), dann wären wir sogar eine ganze Stunde eher in Wien gewesen.“ Wie schnell waren Bernd und Mike? 5 blaue Punkte – da es bei der Geschwindigkeit ein ganzzahliges Ergebnis ist, lässt sich für jüngere Teilnehmer die Aufgabe auch durch systematisches Probieren lösen. Zwischen Passau und Linz waren Bernd und Mike mit 35 km/h unterwegs. Dann sprintete Mike für 10 km mit 45 km/h los, drehte  blitzartig herum und fuhr wieder zu Bernd, der seine 35 km/h beibehalten hatte. Nach welcher Zeit waren die beiden zusammen, wenn Mike die gesamte Zeit 45 km/h fuhr? 4 rote Punkte.

Abgabe bis zum 19.1.2012, Deadline for solution is 19. January 2012
problem 343
Bernd and Mike are looking at photos of their last cycling trip. One of the stages went from Linz to Vienna, which are about 195 kilometres.
"We were very fast then", observed Mike.
"That's right, but if we could have gone (or had wanted to go) only 4 km/h faster on average, we would have been in Vienna a full hour earlier."
How fast were Bernd and Mike on average? - 5 blue points - The average speed is an integer, so for younger participants the problem should be solvable by trial and error.
Between Passau and Linz Bernd and Mike rode at an average speed of 35 km/h. Then, for 10 kilometres Mike went on a sprint of 45 km/h, turned abruptly and returned to Bernd who had been going at a steady 35 km/h. After what time did the meet again if Mike went at an average speed of 45 km/h over this period? - 4 red points

Lösung/solution:

blau: Lösungsvariante Probieren. Überlegung die Geschwindigkeit soll ganzzahlig sein, also nehme ich mal

20 km/h --> Zeit 9,75 h (195 : 20) --> höheres Tempo 24 km/h (20 + 4) --> 8,125 h -- 1,5 h schneller --> passt nicht

25 km/h --> Zeit 7,8 h --> 29 km/h --> 6,72 h --> 1,07.. Stunden --> passt nicht, ist aber sehr nah dran

26km/h --> Zeit 7,5 h --> 30 km/h --> 6,5 --> 1 Stunde --> passt . Die beiden waren mit 26 km/h unterwegs, bei 4km/h mehr hätten sie eine Stunde weniger gebraucht.

Lösung mittels Gleichungssystem blau und rot von Zach Markos, many thanks --> pdf <--

 


Aufgabe 8

344. Wertungsaufgabe

Aufgabe 344
Lisa zeichnet und rechnet mit Quadern. Maria hat in einem Mathematikbuch gelesen, dass von allen Quadern mit gleichem Volumen der Würfel (mit dem gleichen Volumen wie die Quader) die kleinste Oberfläche aller solcher Quader hat. Wie groß ist die Oberfläche eines Würfels, der ein Volumen von 8 m³ hat? (4 blaue Punkte).  Welche Abmessungen sollte ein „oben offener“ Quader haben (V = 8 m³), so dass die fünf Flächen eine möglichst kleine Oberfläche aufweisen – eine Variante aufzuschreiben reicht. (5 rote Punkte)

Lösung bis zum 26.1.2012, Deadline of solution 2012-01-26

english version

problem 344
Lisa draws and calculates cuboids. Maria read in her Maths book that of all cuboids with an equal volume the cube has the smallest surface area. How big is the surface area of a cube that has a volume of 8 m³? - 4 blue points
What measurements does an open cuboid of 8 m³ have (a box without a "lid"), so that the combined surface area of the five remaining faces is minimal? - One solution - 5 red points

Lösung/solution:

blau: Wenn das Volumen des Würfels 8 m³ groß ist, so mittels Wurzelziehen (3. Wurzel) oder etwas probieren die Kantenlänge des Würfels schnell gefunden. Die Oberfläche beträgt dann  6 * 2 m * 2 m = 24 m².

rot: Die mögliche schnelle - wie ich finde - geniale Überlegung ist die: Man nehme einen Würfel nicht von 8 m³, sondern mit 16 m³ Volumen. Kleiner geht dessen Oberfläche nicht. Schneidet man nun diesen Würfel "in der Mitte durch", erhält man zwei Quader die an einer Seite "offen" sind, je 8 m³ Volumen haben und der Oberfläche kleiner nicht sein kann. :-)

Zwei verschiedene Wege zur Lösung zeigen die Varianten von Jürgen U. und Zach M., danke

--> Lösung (deutsch) <-- --> english solution <--


Aufgabe 9

345. Wertungsaufgabe


345„Hallo Lisa, wie ich sehe, konstruierst du die Ying-Yang-Figur. Wie hast du das gemacht?“, fragte Bernd. „Schau auf das Bild und vollzieh die
Schritte nach, dann ist es ganz einfach. Zeichne einen Kreis mit dem Mittelpunkt M. Zeichne den Durchmesser BC ein. Halbiere CM und BM.
Zeichne die Halbkreise mit den Mittelpunkte E bzw. D. Nun brauchst du nur noch die Figur auszumalen und fertig.“ 345 2
Wie lässt sich die Konstruktion fortsetzen, so dass je zwei gleich aussehende (zu einander kongruente) schwarze bzw. weiße Teilflächen entstehen. (3 blaue Punkte).

Es ist eine Gerade zu finden, die die Figur aus dem blauen Teil der Aufgabe so teilt, dass je zwei schwarze und zwei
weiße Teilflächen entstehen, welche alle den gleichen Flächeninhalt haben. (5 rote Punkte). 













Lösung bis zum 2.2.2012 Deadline of solution is 2th. february 2012

problem 345

345 "Hi Lisa, I see you've constructed a Yin-Yang figure. How did you do it?", Bernd asked.
"Look at the picture and do it step by step. It's easy. Draw a circle around center M. Draw diameter BC. Bisect CM and BM. Draw semi-circles around the centers E and D. Colour the figure and you are done." 345 2
How can you continue this construction in order to get two congruent black and two white areas. - 3 blue points
Find a line that divides the figure of the blue challenge so that you get two black and two white subareas that all have the same area. - 5 red points

 

 

 

Lösung/solution.

Die (einfachste Lösung für die blaue Aufgabenstellung:

345-lsgAusgemalt dann so:

345-lsg 2

rot: Zeichnet man eine Gerade durch M im Winkel von 45° (nach unten) zum Durchmesser CB, so wird die weiße Fläche und die schwarze Fläche jeweils - auf den Flächeninhalt bezogen halbiert. Eine ausführliche Lösung von Zach Markos --> als pdf <-- , danke (thanks)


Aufgabe 10

346. Wertungsaufgabe


pythagoras-1„Sagt dir  3-4-5 etwas?“, fragte Mike. „Aber klar doch, das ist doch ein pythagoräisches Zahlentripel und zwar das kleinst mögliche.“ „Stimmt genau, diese Zahlen erfüllen den Satz des Pythagoras. a² + b² = c², mit a = 3, b = 4 und c = 5. Alle Tripel natürlicher Zahlen, die den Satz des
Pythagoras erfüllen, werden pythagoräische Zahlentripel genannt.“
(Anmerkung: Nimmt man zwei Zahlen, so nennt man es Paar (a; b), bei drei Zahlen (a; b; c) heißt es Tripel.)
Finde die natürlichen Zahlen b und c heraus, wenn 5² + b² = c² (3 blaue Punkte, wenn die Probe dabei ist.) Zu zeigen ist, dass jede natürliche Zahl (größer als 2) Bestandteil eines pythagoräischen Zahlentripels sein kann. (4 rote Punkte)

zu lösen bis 9.2.2012 Deadline of solution is 9.th. february 2012


problem 346
pythagoras-1"Does 3-4-5 ring a bell?", Mike asked.
"Sure it does. It's a Pythagorean triple, the smallest possible."
"Exactly, these numbers fulfil the Pythagorean theorem, a² + b² = c², with a = 3, b = 4 and c = 5. All triples of integers that fulfil the Pythagorean theorem are called Pythagorean triples."
(note: If you take two numbers you call it a pair (a; b), with three numbers (a, b, c) it Is called a triple.)
Find the integers b and c for 5² + b² = c². - 3 blue points if there is proof.
Show that any integer bigger than 2 can be part of an Pythagorean triple. (4 red points)

 

 

 

Lösung/solution:

Die Lösung von Felix K. --> als pdf <--

solution from  Zach, thanks --> pdf <--


Aufgabe 11:

347. Wertungsaufgabe:
Lisa sitzt vor einer Kiste mit sehr vielen gleich großen (und wunderbar saftigen) Apfelsinen. Sie nimmt die Apfelsinen und stapelt sie zu „tetraederförmigen“ Pyramiden. So braucht sie z.B. 4 Apfelsinen für eine Pyramide der Höhe 2. Dazu legt sie drei Apfelsinen – möglichst dicht - zu einem Dreieck und legt dann die vierte Kugel oben drauf. Mike kommt dazu. Er versucht eine möglichst hohe Pyramide mit dieser Form zu legen. Da meint Lisa: „Schau mal, diese Pyramide besteht aus so vielen Apfelsinen, dass sich daraus zwei gleichgroße Pyramide stapeln lassen, ohne dass eine Apfelsine zu viel ist oder dazu genommen werden muss.“ „Stimmt genau“, sagt Mike erstaunt.
Wie viele Apfelsinen braucht man, so dass man daraus eine große oder zwei gleiche - etwas kleinere - Pyramiden bauen kann? (4 blaue Punkte) Wie viele Apfelsinen braucht man, damit man eine große Pyramide bauen kann oder aber eben zwei kleinere, welche nicht gleich groß sein dürfen?
(5 rote Punkte) 

Lösung bis zum 1.3.2012 Deadline for solution is 1th march 2012

problem 347
Lisa is sitting in front of a box of delicious looking, equally sized oranges. She takes the oranges and stacks them into a tetrahedron-shaped pyramid. She need 4 oranges for a pyramide of height 2. To do that she puts three oranges in a tight triangle and puts the fourth on top. Mike joins her an tries to make an extra high pyramid. Lisa says: "Look, this pyramid is made of exactly the numer of oranges that you would need to make two smaller pyramids of equal size without adding or removing a single orange."
"You're right", Mike says amazedly.
How many oranges do you need to make one big pyramid or two smaller equally-sized pyramids? - 4 blue points
How many oranges do you need to make one big pyramid or two differently-sized pyramids? - 5 red points.

Lösung/solution:

blau: Die Lösung der Aufgabe kann man den Bildern entnehmen:

 

347-1

Aus diesen vier neben einander liegenden Schichten kann man eine Pyramide der Höhe vier (20 Apfelsinen) zusammensetzen. Aus den drei linke lässt sich eine Pyramide der Höhe 3 (10 Apfelsinen) zusammensetzen. Zwei Pyramiden der Höhe drei ergeben eine Pyramide der Höhe 4.

347-2

347-3

347-4

rot: Eine Pyramide der Höhe 8 (120 Apfelsinen) und eine Pyramide der Höhe 14 (560 Apfelsinen) lassen sich zu einer Pyramide der Höhe 15 (680 Apfelsinen) zusammensetzen. Es gibt aber noch weitere Lösungen für die Aufgaben stellung bei rot. (Allderdings nur eine einzige für den Typ blau.) Eine sehr ausführliche Darstellung der Lösungen, der Berechnungen der "Apfelsinenzahlen" und und und findet sich in der Lösung von Zach, many thanks. --> pdf <--


Aufgabe 12

348. Wertungsaufgabe
„Heute habe ich etwas Merkwürdiges gehört?“, sagte Maria. Bernd runzelt die Stirn. „Stell dir vor, da sagt einer aus unserem Matheclub, er hätte jetzt Meerschweinchen. Auf die Frage, wie viele, sagte er: Nehme ich zu 4/5 der Anzahl meiner Meerschweinchen 4/5 eines Meerschweinchens
hinzu, so erhalte ich die Anzahl meiner Meerschweinchen.“ Wie viele Meerschweinchen hat der Junge? (3 blaue Punkte) „Das ist noch gar nichts,“ erwiderte Bernd. „Es hört sich leicht an, aber … 20 + 20 + 20 + 10 + 10 = 80.“ „Wo ist da das Problem?“, fragte Mike, der gerade ins Zimmer trat. „Schreibe die Aufgabe mit englischen Zahlwörtern auf, also twenty + twenty …. (Rechenzeichen werden nicht übersetzt bzw. durch Buchstaben ersetzt.) Es sind dann genau 8 verschiedene Buchstaben, die in der Aufgabe vorkommen. Nun soll jeder Buchstabe durch eine Ziffer ersetzt werden – gleiche Buchstaben, gleiche Ziffern – verschiedene Buchstaben, verschiedene Ziffern.“ Welcher Buchstabe muss für welche Ziffer stehen, damit die mit Buchstaben geschriebene Aufgabe in eine richtig gelöste Aufgabe mit Ziffern überführt wird. (8 rote Punkte)
Tipp: Schreibe die „rote“ Aufgabe wie bei einer schriftlichen Additionsaufgabe auf.

Lösung bis zum 8.3.2012 Deadline for solution is 8th. march 2012

problem 348
"Today i heard something strange", Maria said. Bernd frowned.
"Imagine, one boy of our maths club said he had guinea pigs now. When asked how many, he replied: 'If I add to 4/5 of the number of my guinea pigs 4/5 of one guinea pig I'll get the number of my guinea pigs.' "
How many guinea pigs does he have? - 3 blue points
"That's nothing", Bernd replied. "Listen, it may sound easy, but ... 20 + 20+ 20 + 10 + 10 = 80."
"What's the problem?", Mike asked when entering the room.
"Well, write this task using English numerals like twenty + twenty ... . (operator symbols remain) You get exactly 8 different letters in this task. Now replace each letter by a digit - same letter, same digit - different letter - different digit. Which letter goes with which digit so that task written in letters corresponds to a correctly solved task using digits. - 8 red points
Hint: Write the task as in the columnar addition method.

Lösung/solution:

blau:

x sei die Anzahl aller Meerschweine, dann gilt  \frac{4}{5} + \frac {4}{5} \cdot x = x | - \frac {4}{5} \cdot x \\ \frac{4}{5} = \frac {1}{5} \cdot x | : \frac {1}{5} \\ x = 4

Es sind also 4 Meerschweine, eine Probe ist schnell gemacht.

rot:

twenty + twenty + twenty + ten + ten + ten = eighty 

123416 + 123416 + 123416 + 134 + 134 + 134 = 370516

Variante 1 von XXX - ein DELPHI Programm im Stil von PASCAL, danke:

var t,w,e,n,i,y,g,h : integer;
{sub} function ten:integer;
begin
  ten := (t*10+e)*10+n;
end;
{sub} function twenty:integer;
begin
  twenty := ((((t*10+w)*10+e)*10+n)*10+t)*10+y;
end;
{sub} function eighty:integer;
begin
  eighty := ((((e*10+i)*10+g)*10+h)*10+t)*10+y;
end;
begin
  for y := 0 to 9 do 
  for n := 0 to 9 do if y<>n then 
  for w := 0 to 9 do if (w<>y) AND (w<>n) then
  for e := 0 to 9 do if (e<>w) AND (e<>y) AND (e<>n) then
  for t := 0 to 9 do if (t<>y) AND (t<>n) AND (t<>w) AND (t<>e) then
  for i := 0 to 9 do if (i<>y) AND (i<>n) AND (i<>w) AND (i<>e) AND (i<>t) then
  for g := 0 to 9 do if (g<>y) AND (g<>n) AND (g<>w) AND (g<>e) AND (g<>t) AND (g<>i) then
  for h := 0 to 9 do if (h<>y) AND (h<>n) AND (h<>w) AND (h<>e) AND (h<>t) AND (h<>i) AND (h<>g) then
 
  if 3*twenty+2*ten = eighty  then "Ausgabe"

Variante 2 - Lösung von Felix Karu, danke

--> pdf <--

solution (english) from Zach, thanks

--> pdf <--


Auswertung der Serie 29

Anmerkung: Sollten sich bei mir noch irgendwelche Zettel mit Lösungen finden, dann werden die Punkte noch nachgetragen, sind dann aber nur bei der Liste mit den Gesamtpunktzahlen zu "erkennen".  http://www.schulmodell.eu/punkte.html

Es waren 52 blaue und 58 rote Punkte zu erreichen, insgesamt also 110.

Auswertung Serie 29 (blaue Liste)

Platz Name Ort Summe Aufgabe
  337 338 339 340 341 342 343 344 345 346 347 348
1. Zach Markos Los Angeles 52 6 9 4 3 5 3 5 4 3 3 4 3
1. Doreen Naumann Duisburg 52 6 9 4 3 5 3 5 4 3 3 4 3
1. Rafael Seidel Chemnitz 52 6 9 4 3 5 3 5 4 3 3 4 3
1. Linus-Valentin Lohs Chemnitz 52 6 9 4 3 5 3 5 4 3 3 4 3
2. Sabine Fischbach Hessen 45 6 9 4 3 5 3 5 4 - 3 - 3
3. Valentin Grundmann Chemnitz 44 6 9 4 3 - - 5 4 3 3 4 3
3. Jürgen Urbig Chemnitz 44 6 9 4 3 - 3 5 4 3 3 4 -
4. Andree Dammann München 37 - 9 4 3 5 3 5 4 - - 4 -
5. Gunnar Reinelt Chemnitz 34 6 9 4 - - - 5 4 - 3 - 3
6. Simon Anders Chemnitz 22 6 9 - - - - - 4 - 3 - -
7. Felicitas Hastedt Chemnitz 21 6 - 4 - - - 5 - 3 - - 3
8. Elena Oelschlägel Chemnitz 19 6 9 - - - - - 4 - - - -
8. Marcel Reichelt Chemnitz 19 6 - - 3 3 - - 4 - 3 - -
8. Tom Straßer Chemnitz 19 4 - - - - - 5 4 - 3 - 3
8. Felicitas Güra Chemnitz 19 6 - 4 - - - 5 4 - - - -
9. Nele Mäding Chemnitz 18 6 - 4 - - - 5 3 - - - -
9. Lena Elisa Penzlin Chemnitz 18 6 - - 3 - 3 - - 3 3 - -
9. Ole Koelb Chemnitz 18 6 - - 3 - 3 - - 3 3 - -
9. Tim Jechorek Chemnitz 18 6 - - 3 - - - - 3 3 - 3
9. Ellen Wilde Chemnitz 18 6 - - 3 - 3 - 4 - - - 2
9. Heinrich Grossinger Chemnitz 18 6 - - - - - 5 1 - 3 - 3
10. Felix Taubert Chemnitz 16 - 9 - 3 - - - 4 - - - -
10. Henrike Grundmann Chemnitz 16 6 - - 3 - 3 - 4 - - - -
10. Lisanne Brinkel Chemnitz 16 6 - - - - - - - 3 - 4 3
11. Marvin Gülden Chemnitz 15 6 9 - - - - - - - - - -
11. Uwe Parsche Chemnitz 15 6 9 - - - - - - - - - -
11. Karl Herrmann Chemnitz 15 6 - - - - - - 4 2 - - 3
11. Emily Neuwirth Chemnitz 15 6 - - 3 - 3 - - - 3 - -
11. Melanie Petz Chemnitz 15 - - 4 - - - 5 - 3 3 - -
12. Arne Weißbach Chemnitz 13 - - 4 - - - 5 4 - - - -
12. Tobias Morgenstern Chemnitz 13 - - - - 3 3 - 1 3 - - 3
12. Cynthia Raschkowsky Chemnitz 13 - 9 - - - - 4 - - - - -
12. Theresa Jänich Chemnitz 13 - - - 3 - 3 - 4 3 - - -
12. Valentin Sellin Chemnitz 13 6 - - - - - - 4 - - - 3
13. Moritz Duderstadt Chemnitz 12 6 - - 3 - 3 - - - - - -
13. Gabriel Dammann München 12 - - - - 5 3 - 4 - - - -
13. Paula Hartmannsdorf 12 - - - - 5 - - - - - 4 3
13. Lilli Weiß Chemnitz 12 6 - - - - - - - 3 - - 3
14. Lukas Thieme Chemnitz 11 - - - - - - 4 4 3 - - -
15. Julia Voigt Chemnitz 10 - - - 3 - 3 - 4 - - - -
15. Julian Vass Chemnitz 10 - - - 3 - - - 4 - 3 - -
15. Hannes Hohmann Chemnitz 10 - - - - - - - 4 3 - - 3
15. Jessica Ritter Chemnitz 10 6 - - - - - - 4 - - - -
15. Julia Ritter Chemnitz 10 - - - 3 - - - 4 - 3 - -
15. Celestina Montero Perez Chemnitz 10 - - - - - - - 4 3 - - 3
15. Marie Berger Chemnitz 10 - - - - - - - 4 3 - - 3
15. Elina Rech Chemnitz 10 6 - - - - - - 4 - - - -
16. Ernesto Uhlmann Chemnitz 9 - - - 3 - 3 - - 3 - - -
16. Lukas Kirchberg Chemnitz 9 - - - 3 - 3 - - 3 - - -
16. Willy Stöckel Chemnitz 9 - - - 3 - 3 - - 3 - - -
16. Tom Hartig Chemnitz 9 - - - 3 - - - - - 3 - 3
16. Anna Grünert Chemnitz 9 - 9 - - - - - - - - - -
16. Felix Karu Altach 9 - - - - - 3 - - - 3 - 3
16. Friederike Lenk Chemnitz 9 - - - 3 - 3 - - 3 - - -
16. Luis Raupach Chemnitz 9 - - - 3 2 - - - 2 - - 2
17. Hannah Gebhardt Chemnitz 8 - - - - 2 3 - - 3 - - -
17. Adrian Schlegel Chemnitz 8 - - 4 - - - - 4 - - - -
18. Emilie Grossinger Chemnitz 7 - - - 3 - - - 4 - - - -
18. Frederike Meiser Chemnitz 7 - - - - - - - 1 3 - - 3
18. Tobias Richter Chemnitz 7 - - - - - - - 4 3 - - -
19. Astrid Fischer Chemnitz 6 6 - - - - - - - - - - -
19. Helene Fischer Chemnitz 6 - - - - - - - - 3 - - 3
19. Paula Mühlmann Dittersdorf 6 - - - - - - - - 3 - - 3
19. Christian Wagner Bamberg 6 6 - - - - - - - - - - -
19. Lene Haag Chemnitz 6 - - - - - - - 4 2 - - -
19. Moritz Weber Chemnitz 6 6 - - - - - - - - - - -
20. XXX ??? 5 - - - - - - - - - 3 - 2
21. Katie Bessey Bath (Maine-USA) 4 - - - - - - - - - - 4 -
21. Hannes Eltner ???? 4 - - - - - - - 4 - - - -
21. Simon Winger Chemnitz 4 - - - - - - - - - - 4 -
21. Ulrike Böhme Chemnitz 4 - - 4 - - - - - - - - -
22. Tim Sigmund Chemnitz 3 - - - 3 - - - - - - - -
22. Anna Georgi Chemnitz 3 - - - - - - - - - - - 3
22. Jessica Spindler Chemnitz 3 - - - - - - - - - - - 3
22. Marie Juhran Chemnitz 3 - - - - - - - - - - - 3
22. Albin Uhlig Chemnitz 3 - - - - - - - - 3 - - -
22. Jonathan Dammann München 3 - - - - - 3 - - - - - -
22. Shari Schmidt Chemnitz 3 - - - - - - - - - - - 3
22. Clara Stöckel Chemnitz 3 - - - 3 - - - - - - - -
22. Johannes Allert Chemnitz 3 - - - 3 - - - - - - - -
22. Maria Dreßler Chemnitz 3 - - - 3 - - - - - - - -
22. Erik Walther Chemnitz 3 - - - 3 - - - - - - - -
22. Joshua May Chemnitz 3 - - - 3 - - - - - - - -
23. Jule Irmscher Eibenberg 2 - - - 2 - - - - - - - -
23. Marvin Köllner Chemnitz 2 - - - 2 - - - - - - - -
23. Amarin Roßberg Chemnitz 2 - - - 2 - - - - - - - -
23. Hanna Kallenbach Chemnitz 2 - - - 2 - - - - - - - -
23. Rebecca Wagner Oberwiesenthal 2 - - - - 2 - - - - - - -
23. Svenja Reinelt Chemnitz 2 - - - - 2 - - - - - - -
23. Hannah-Sophie Schubert Chemnitz 2 - - - - 2 - - - - - - -
23. Emmely Schöne Chemnitz 2 - - - 2 - - - - - - - -
23. Lina Krug Chemnitz 2 - - - 2 - - - - - - - -
23. Tim Missullis Chemnitz 2 - - - 2 - - - - - - - -
23. Katharina Zweiniger Chemnitz 2 - - - - - - - 1 1 - - -
23. Josephine Klotz Chemnitz 2 - - - 2 - - - - - - - -
23. Michelle Wade Chemnitz 2 - - - 2 - - - - - - - -
23. Laurin Roßberg Chemnitz 2 - - - 2 - - - - - - - -
23. Pit Hopke Chemnitz 2 - - - 2 - - - - - - - -
24. Franz Kemter Chemnitz 1 - - - - - - - 1 - - - -
24. Nicklas Reichert Chemnitz 1 - - - - 1 - - - - - - -
24. Kevin Ngyen Chemnitz 1 - - - - 1 - - - - - - -


Auswertung Serie 29 (rote Liste)

Platz Name Ort Summe Aufgabe
  337 338 339 340 341 342 343 344 345 346 347 348
1. Zach Markos Los Angeles 58 5 6 3 6 4 3 4 5 5 4 5 8
2. Doreen Naumann Duisburg 48 5 6 3 6 - 3 4 4 - 4 5 8
3. Jürgen Urbig Chemnitz 46 5 6 3 6 - 3 4 5 5 4 5 -
4. Rafael Seidel Chemnitz 38 - 6 3 6 - - - 5 5 - 5 8
4. Valentin Grundmann Chemnitz 38 5 6 2 2 - - - 1 5 4 5 8
4. Sabine Fischbach Hessen 38 5 6 3 6 4 2 2 2 1 3 - 4
5. Linus-Valentin Lohs Chemnitz 31 - - 3 6 - - 4 2 - 3 5 8
6. Gunnar Reinelt Chemnitz 22 5 6 3 - - - - 0 - - - 8
7. Andree Dammann München 21 - 6 1 - 4 - 4 2 - - 4 -
8. Heinrich Grossinger Chemnitz 17 5 - - - - - - - - 4 - 8
9. Felix Karu Altach 15 - - - - - 3 - - - 4 - 8
10. Simon Anders Chemnitz 12 5 6 - - - - - 1 - - - -
10. XXX ??? 12 - - - - - - - - - 4 - 8
11. Uwe Parsche Chemnitz 11 5 6 - - - - - - - - - -
12. Arne Weißbach Chemnitz 7 - - 3 - - - 4 - - - - -
13. Lisanne Brinkel Chemnitz 6 - - - - - - - - 1 - 5 -
13. Felix Taubert Chemnitz 6 - 6 - - - - - - - - - -
14. Paula Hartmannsdorf 5 - - - - - - - - - - 5 -
14. Emily Neuwirth Chemnitz 5 5 - - - - - - - - - - -
14. Christian Wagner Bamberg 5 5 - - - - - - - - - - -
14. Felicitas Güra Chemnitz 5 5 - - - - - - - - - - -
14. Simon Winger Chemnitz 5 - - - - - - - - - - 5 -
14. Astrid Fischer Chemnitz 5 5 - - - - - - - - - - -
14. Elena Oelschlägel Chemnitz 5 5 - - - - - - - - - - -
14. Katie Bessey Bath (Maine-USA) 5 - - - - - - - - - - 5 -
14. Valentin Sellin Chemnitz 5 5 - - - - - - - - - - -
14. Elina Rech Chemnitz 5 5 - - - - - - - - - - -
15. Nele Mäding Chemnitz 3 - - 3 - - - - - - - - -
15. Tobias Morgenstern Chemnitz 3 - - - - - 3 - - - - - -
15. Ulrike Böhme Chemnitz 3 - - 3 - - - - - - - - -
15. Melanie Petz Chemnitz 3 - - 3 - - - - - - - - -
16. Gabriel Dammann München 2 - - - - - - - 2 - - - -
17. Lilli Weiß Chemnitz 1 - - - - - - - - 1 - - -
17. Jessica Ritter Chemnitz 1 - - - - - - - 1 - - - -

Wechseln eines 500 Euroscheins

In der Wochenaufgabe 330 ging es u. a. um das Wechseln eines 500 Euroscheines

Hier nun die Lösungen komplett:

nur 200 Euroscheine geht nicht:

100 und 200 Euroscheine

1: 0 * 200 + 5 * 100
2: 1 * 200 + 3 * 100
3: 2 * 200 + 1 * 100

50, 100 und 200 Euroscheine

1: 0 * 200 + 0 * 100 + 10 * 50
2: 0 * 200 + 1 * 100 + 8 * 50
3: 0 * 200 + 2 * 100 + 6 * 50
4: 0 * 200 + 3 * 100 + 4 * 50
5: 0 * 200 + 4 * 100 + 2 * 50
6: 0 * 200 + 5 * 100 + 0 * 50
7: 1 * 200 + 0 * 100 + 6 * 50
8: 1 * 200 + 1 * 100 + 4 * 50
9: 1 * 200 + 2 * 100 + 2 * 50
10: 1 * 200 + 3 * 100 + 0 * 50
11: 2 * 200 + 0 * 100 + 2 * 50
12: 2 * 200 + 1 * 100 + 0 * 50

20, 50, 100 und 200 Euroscheine

1: 0 * 200 + 0 * 100 + 0 * 50 + 25 * 20
2: 0 * 200 + 0 * 100 + 2 * 50 + 20 * 20
3: 0 * 200 + 0 * 100 + 4 * 50 + 15 * 20
4: 0 * 200 + 0 * 100 + 6 * 50 + 10 * 20
5: 0 * 200 + 0 * 100 + 8 * 50 + 5 * 20
6: 0 * 200 + 0 * 100 + 10 * 50 + 0 * 20
7: 0 * 200 + 1 * 100 + 0 * 50 + 20 * 20
8: 0 * 200 + 1 * 100 + 2 * 50 + 15 * 20
9: 0 * 200 + 1 * 100 + 4 * 50 + 10 * 20
10: 0 * 200 + 1 * 100 + 6 * 50 + 5 * 20
11: 0 * 200 + 1 * 100 + 8 * 50 + 0 * 20
12: 0 * 200 + 2 * 100 + 0 * 50 + 15 * 20
13: 0 * 200 + 2 * 100 + 2 * 50 + 10 * 20
14: 0 * 200 + 2 * 100 + 4 * 50 + 5 * 20
15: 0 * 200 + 2 * 100 + 6 * 50 + 0 * 20
16: 0 * 200 + 3 * 100 + 0 * 50 + 10 * 20
17: 0 * 200 + 3 * 100 + 2 * 50 + 5 * 20
18: 0 * 200 + 3 * 100 + 4 * 50 + 0 * 20
19: 0 * 200 + 4 * 100 + 0 * 50 + 5 * 20
20: 0 * 200 + 4 * 100 + 2 * 50 + 0 * 20
21: 0 * 200 + 5 * 100 + 0 * 50 + 0 * 20
22: 1 * 200 + 0 * 100 + 0 * 50 + 15 * 20
23: 1 * 200 + 0 * 100 + 2 * 50 + 10 * 20
24: 1 * 200 + 0 * 100 + 4 * 50 + 5 * 20
25: 1 * 200 + 0 * 100 + 6 * 50 + 0 * 20
26: 1 * 200 + 1 * 100 + 0 * 50 + 10 * 20
27: 1 * 200 + 1 * 100 + 2 * 50 + 5 * 20
28: 1 * 200 + 1 * 100 + 4 * 50 + 0 * 20
29: 1 * 200 + 2 * 100 + 0 * 50 + 5 * 20
30: 1 * 200 + 2 * 100 + 2 * 50 + 0 * 20
31: 1 * 200 + 3 * 100 + 0 * 50 + 0 * 20
32: 2 * 200 + 0 * 100 + 0 * 50 + 5 * 20
33: 2 * 200 + 0 * 100 + 2 * 50 + 0 * 20
34: 2 * 200 + 1 * 100 + 0 * 50 + 0 * 20

10, 20, 50, 100 und 200 Euroscheine

1: 0 * 200 + 0 * 100 + 0 * 50 + 0 * 20 + 50 * 10
2: 0 * 200 + 0 * 100 + 0 * 50 + 1 * 20 + 48 * 10
3: 0 * 200 + 0 * 100 + 0 * 50 + 2 * 20 + 46 * 10
4: 0 * 200 + 0 * 100 + 0 * 50 + 3 * 20 + 44 * 10
5: 0 * 200 + 0 * 100 + 0 * 50 + 4 * 20 + 42 * 10
6: 0 * 200 + 0 * 100 + 0 * 50 + 5 * 20 + 40 * 10
7: 0 * 200 + 0 * 100 + 0 * 50 + 6 * 20 + 38 * 10
8: 0 * 200 + 0 * 100 + 0 * 50 + 7 * 20 + 36 * 10
9: 0 * 200 + 0 * 100 + 0 * 50 + 8 * 20 + 34 * 10
10: 0 * 200 + 0 * 100 + 0 * 50 + 9 * 20 + 32 * 10
11: 0 * 200 + 0 * 100 + 0 * 50 + 10 * 20 + 30 * 10
12: 0 * 200 + 0 * 100 + 0 * 50 + 11 * 20 + 28 * 10
13: 0 * 200 + 0 * 100 + 0 * 50 + 12 * 20 + 26 * 10
14: 0 * 200 + 0 * 100 + 0 * 50 + 13 * 20 + 24 * 10
15: 0 * 200 + 0 * 100 + 0 * 50 + 14 * 20 + 22 * 10
16: 0 * 200 + 0 * 100 + 0 * 50 + 15 * 20 + 20 * 10
17: 0 * 200 + 0 * 100 + 0 * 50 + 16 * 20 + 18 * 10
18: 0 * 200 + 0 * 100 + 0 * 50 + 17 * 20 + 16 * 10
19: 0 * 200 + 0 * 100 + 0 * 50 + 18 * 20 + 14 * 10
20: 0 * 200 + 0 * 100 + 0 * 50 + 19 * 20 + 12 * 10
21: 0 * 200 + 0 * 100 + 0 * 50 + 20 * 20 + 10 * 10
22: 0 * 200 + 0 * 100 + 0 * 50 + 21 * 20 + 8 * 10
23: 0 * 200 + 0 * 100 + 0 * 50 + 22 * 20 + 6 * 10
24: 0 * 200 + 0 * 100 + 0 * 50 + 23 * 20 + 4 * 10
25: 0 * 200 + 0 * 100 + 0 * 50 + 24 * 20 + 2 * 10
26: 0 * 200 + 0 * 100 + 0 * 50 + 25 * 20 + 0 * 10
27: 0 * 200 + 0 * 100 + 1 * 50 + 0 * 20 + 45 * 10
28: 0 * 200 + 0 * 100 + 1 * 50 + 1 * 20 + 43 * 10
29: 0 * 200 + 0 * 100 + 1 * 50 + 2 * 20 + 41 * 10
30: 0 * 200 + 0 * 100 + 1 * 50 + 3 * 20 + 39 * 10
31: 0 * 200 + 0 * 100 + 1 * 50 + 4 * 20 + 37 * 10
32: 0 * 200 + 0 * 100 + 1 * 50 + 5 * 20 + 35 * 10
33: 0 * 200 + 0 * 100 + 1 * 50 + 6 * 20 + 33 * 10
34: 0 * 200 + 0 * 100 + 1 * 50 + 7 * 20 + 31 * 10
35: 0 * 200 + 0 * 100 + 1 * 50 + 8 * 20 + 29 * 10
36: 0 * 200 + 0 * 100 + 1 * 50 + 9 * 20 + 27 * 10
37: 0 * 200 + 0 * 100 + 1 * 50 + 10 * 20 + 25 * 10
38: 0 * 200 + 0 * 100 + 1 * 50 + 11 * 20 + 23 * 10
39: 0 * 200 + 0 * 100 + 1 * 50 + 12 * 20 + 21 * 10
40: 0 * 200 + 0 * 100 + 1 * 50 + 13 * 20 + 19 * 10
41: 0 * 200 + 0 * 100 + 1 * 50 + 14 * 20 + 17 * 10
42: 0 * 200 + 0 * 100 + 1 * 50 + 15 * 20 + 15 * 10
43: 0 * 200 + 0 * 100 + 1 * 50 + 16 * 20 + 13 * 10
44: 0 * 200 + 0 * 100 + 1 * 50 + 17 * 20 + 11 * 10
45: 0 * 200 + 0 * 100 + 1 * 50 + 18 * 20 + 9 * 10
46: 0 * 200 + 0 * 100 + 1 * 50 + 19 * 20 + 7 * 10
47: 0 * 200 + 0 * 100 + 1 * 50 + 20 * 20 + 5 * 10
48: 0 * 200 + 0 * 100 + 1 * 50 + 21 * 20 + 3 * 10
49: 0 * 200 + 0 * 100 + 1 * 50 + 22 * 20 + 1 * 10
50: 0 * 200 + 0 * 100 + 2 * 50 + 0 * 20 + 40 * 10
51: 0 * 200 + 0 * 100 + 2 * 50 + 1 * 20 + 38 * 10
52: 0 * 200 + 0 * 100 + 2 * 50 + 2 * 20 + 36 * 10
53: 0 * 200 + 0 * 100 + 2 * 50 + 3 * 20 + 34 * 10
54: 0 * 200 + 0 * 100 + 2 * 50 + 4 * 20 + 32 * 10
55: 0 * 200 + 0 * 100 + 2 * 50 + 5 * 20 + 30 * 10
56: 0 * 200 + 0 * 100 + 2 * 50 + 6 * 20 + 28 * 10
57: 0 * 200 + 0 * 100 + 2 * 50 + 7 * 20 + 26 * 10
58: 0 * 200 + 0 * 100 + 2 * 50 + 8 * 20 + 24 * 10
59: 0 * 200 + 0 * 100 + 2 * 50 + 9 * 20 + 22 * 10
60: 0 * 200 + 0 * 100 + 2 * 50 + 10 * 20 + 20 * 10
61: 0 * 200 + 0 * 100 + 2 * 50 + 11 * 20 + 18 * 10
62: 0 * 200 + 0 * 100 + 2 * 50 + 12 * 20 + 16 * 10
63: 0 * 200 + 0 * 100 + 2 * 50 + 13 * 20 + 14 * 10
64: 0 * 200 + 0 * 100 + 2 * 50 + 14 * 20 + 12 * 10
65: 0 * 200 + 0 * 100 + 2 * 50 + 15 * 20 + 10 * 10
66: 0 * 200 + 0 * 100 + 2 * 50 + 16 * 20 + 8 * 10
67: 0 * 200 + 0 * 100 + 2 * 50 + 17 * 20 + 6 * 10
68: 0 * 200 + 0 * 100 + 2 * 50 + 18 * 20 + 4 * 10
69: 0 * 200 + 0 * 100 + 2 * 50 + 19 * 20 + 2 * 10
70: 0 * 200 + 0 * 100 + 2 * 50 + 20 * 20 + 0 * 10
71: 0 * 200 + 0 * 100 + 3 * 50 + 0 * 20 + 35 * 10
72: 0 * 200 + 0 * 100 + 3 * 50 + 1 * 20 + 33 * 10
73: 0 * 200 + 0 * 100 + 3 * 50 + 2 * 20 + 31 * 10
74: 0 * 200 + 0 * 100 + 3 * 50 + 3 * 20 + 29 * 10
75: 0 * 200 + 0 * 100 + 3 * 50 + 4 * 20 + 27 * 10
76: 0 * 200 + 0 * 100 + 3 * 50 + 5 * 20 + 25 * 10
77: 0 * 200 + 0 * 100 + 3 * 50 + 6 * 20 + 23 * 10
78: 0 * 200 + 0 * 100 + 3 * 50 + 7 * 20 + 21 * 10
79: 0 * 200 + 0 * 100 + 3 * 50 + 8 * 20 + 19 * 10
80: 0 * 200 + 0 * 100 + 3 * 50 + 9 * 20 + 17 * 10
81: 0 * 200 + 0 * 100 + 3 * 50 + 10 * 20 + 15 * 10
82: 0 * 200 + 0 * 100 + 3 * 50 + 11 * 20 + 13 * 10
83: 0 * 200 + 0 * 100 + 3 * 50 + 12 * 20 + 11 * 10
84: 0 * 200 + 0 * 100 + 3 * 50 + 13 * 20 + 9 * 10
85: 0 * 200 + 0 * 100 + 3 * 50 + 14 * 20 + 7 * 10
86: 0 * 200 + 0 * 100 + 3 * 50 + 15 * 20 + 5 * 10
87: 0 * 200 + 0 * 100 + 3 * 50 + 16 * 20 + 3 * 10
88: 0 * 200 + 0 * 100 + 3 * 50 + 17 * 20 + 1 * 10
89: 0 * 200 + 0 * 100 + 4 * 50 + 0 * 20 + 30 * 10
90: 0 * 200 + 0 * 100 + 4 * 50 + 1 * 20 + 28 * 10
91: 0 * 200 + 0 * 100 + 4 * 50 + 2 * 20 + 26 * 10
92: 0 * 200 + 0 * 100 + 4 * 50 + 3 * 20 + 24 * 10
93: 0 * 200 + 0 * 100 + 4 * 50 + 4 * 20 + 22 * 10
94: 0 * 200 + 0 * 100 + 4 * 50 + 5 * 20 + 20 * 10
95: 0 * 200 + 0 * 100 + 4 * 50 + 6 * 20 + 18 * 10
96: 0 * 200 + 0 * 100 + 4 * 50 + 7 * 20 + 16 * 10
97: 0 * 200 + 0 * 100 + 4 * 50 + 8 * 20 + 14 * 10
98: 0 * 200 + 0 * 100 + 4 * 50 + 9 * 20 + 12 * 10
99: 0 * 200 + 0 * 100 + 4 * 50 + 10 * 20 + 10 * 10
100: 0 * 200 + 0 * 100 + 4 * 50 + 11 * 20 + 8 * 10
101: 0 * 200 + 0 * 100 + 4 * 50 + 12 * 20 + 6 * 10
102: 0 * 200 + 0 * 100 + 4 * 50 + 13 * 20 + 4 * 10
103: 0 * 200 + 0 * 100 + 4 * 50 + 14 * 20 + 2 * 10
104: 0 * 200 + 0 * 100 + 4 * 50 + 15 * 20 + 0 * 10
105: 0 * 200 + 0 * 100 + 5 * 50 + 0 * 20 + 25 * 10
106: 0 * 200 + 0 * 100 + 5 * 50 + 1 * 20 + 23 * 10
107: 0 * 200 + 0 * 100 + 5 * 50 + 2 * 20 + 21 * 10
108: 0 * 200 + 0 * 100 + 5 * 50 + 3 * 20 + 19 * 10
109: 0 * 200 + 0 * 100 + 5 * 50 + 4 * 20 + 17 * 10
110: 0 * 200 + 0 * 100 + 5 * 50 + 5 * 20 + 15 * 10
111: 0 * 200 + 0 * 100 + 5 * 50 + 6 * 20 + 13 * 10
112: 0 * 200 + 0 * 100 + 5 * 50 + 7 * 20 + 11 * 10
113: 0 * 200 + 0 * 100 + 5 * 50 + 8 * 20 + 9 * 10
114: 0 * 200 + 0 * 100 + 5 * 50 + 9 * 20 + 7 * 10
115: 0 * 200 + 0 * 100 + 5 * 50 + 10 * 20 + 5 * 10
116: 0 * 200 + 0 * 100 + 5 * 50 + 11 * 20 + 3 * 10
117: 0 * 200 + 0 * 100 + 5 * 50 + 12 * 20 + 1 * 10
118: 0 * 200 + 0 * 100 + 6 * 50 + 0 * 20 + 20 * 10
119: 0 * 200 + 0 * 100 + 6 * 50 + 1 * 20 + 18 * 10
120: 0 * 200 + 0 * 100 + 6 * 50 + 2 * 20 + 16 * 10
121: 0 * 200 + 0 * 100 + 6 * 50 + 3 * 20 + 14 * 10
122: 0 * 200 + 0 * 100 + 6 * 50 + 4 * 20 + 12 * 10
123: 0 * 200 + 0 * 100 + 6 * 50 + 5 * 20 + 10 * 10
124: 0 * 200 + 0 * 100 + 6 * 50 + 6 * 20 + 8 * 10
125: 0 * 200 + 0 * 100 + 6 * 50 + 7 * 20 + 6 * 10
126: 0 * 200 + 0 * 100 + 6 * 50 + 8 * 20 + 4 * 10
127: 0 * 200 + 0 * 100 + 6 * 50 + 9 * 20 + 2 * 10
128: 0 * 200 + 0 * 100 + 6 * 50 + 10 * 20 + 0 * 10
129: 0 * 200 + 0 * 100 + 7 * 50 + 0 * 20 + 15 * 10
130: 0 * 200 + 0 * 100 + 7 * 50 + 1 * 20 + 13 * 10
131: 0 * 200 + 0 * 100 + 7 * 50 + 2 * 20 + 11 * 10
132: 0 * 200 + 0 * 100 + 7 * 50 + 3 * 20 + 9 * 10
133: 0 * 200 + 0 * 100 + 7 * 50 + 4 * 20 + 7 * 10
134: 0 * 200 + 0 * 100 + 7 * 50 + 5 * 20 + 5 * 10
135: 0 * 200 + 0 * 100 + 7 * 50 + 6 * 20 + 3 * 10
136: 0 * 200 + 0 * 100 + 7 * 50 + 7 * 20 + 1 * 10
137: 0 * 200 + 0 * 100 + 8 * 50 + 0 * 20 + 10 * 10
138: 0 * 200 + 0 * 100 + 8 * 50 + 1 * 20 + 8 * 10
139: 0 * 200 + 0 * 100 + 8 * 50 + 2 * 20 + 6 * 10
140: 0 * 200 + 0 * 100 + 8 * 50 + 3 * 20 + 4 * 10
141: 0 * 200 + 0 * 100 + 8 * 50 + 4 * 20 + 2 * 10
142: 0 * 200 + 0 * 100 + 8 * 50 + 5 * 20 + 0 * 10
143: 0 * 200 + 0 * 100 + 9 * 50 + 0 * 20 + 5 * 10
144: 0 * 200 + 0 * 100 + 9 * 50 + 1 * 20 + 3 * 10
145: 0 * 200 + 0 * 100 + 9 * 50 + 2 * 20 + 1 * 10
146: 0 * 200 + 0 * 100 + 10 * 50 + 0 * 20 + 0 * 10
147: 0 * 200 + 1 * 100 + 0 * 50 + 0 * 20 + 40 * 10
148: 0 * 200 + 1 * 100 + 0 * 50 + 1 * 20 + 38 * 10
149: 0 * 200 + 1 * 100 + 0 * 50 + 2 * 20 + 36 * 10
150: 0 * 200 + 1 * 100 + 0 * 50 + 3 * 20 + 34 * 10
151: 0 * 200 + 1 * 100 + 0 * 50 + 4 * 20 + 32 * 10
152: 0 * 200 + 1 * 100 + 0 * 50 + 5 * 20 + 30 * 10
153: 0 * 200 + 1 * 100 + 0 * 50 + 6 * 20 + 28 * 10
154: 0 * 200 + 1 * 100 + 0 * 50 + 7 * 20 + 26 * 10
155: 0 * 200 + 1 * 100 + 0 * 50 + 8 * 20 + 24 * 10
156: 0 * 200 + 1 * 100 + 0 * 50 + 9 * 20 + 22 * 10
157: 0 * 200 + 1 * 100 + 0 * 50 + 10 * 20 + 20 * 10
158: 0 * 200 + 1 * 100 + 0 * 50 + 11 * 20 + 18 * 10
159: 0 * 200 + 1 * 100 + 0 * 50 + 12 * 20 + 16 * 10
160: 0 * 200 + 1 * 100 + 0 * 50 + 13 * 20 + 14 * 10
161: 0 * 200 + 1 * 100 + 0 * 50 + 14 * 20 + 12 * 10
162: 0 * 200 + 1 * 100 + 0 * 50 + 15 * 20 + 10 * 10
163: 0 * 200 + 1 * 100 + 0 * 50 + 16 * 20 + 8 * 10
164: 0 * 200 + 1 * 100 + 0 * 50 + 17 * 20 + 6 * 10
165: 0 * 200 + 1 * 100 + 0 * 50 + 18 * 20 + 4 * 10
166: 0 * 200 + 1 * 100 + 0 * 50 + 19 * 20 + 2 * 10
167: 0 * 200 + 1 * 100 + 0 * 50 + 20 * 20 + 0 * 10
168: 0 * 200 + 1 * 100 + 1 * 50 + 0 * 20 + 35 * 10
169: 0 * 200 + 1 * 100 + 1 * 50 + 1 * 20 + 33 * 10
170: 0 * 200 + 1 * 100 + 1 * 50 + 2 * 20 + 31 * 10
171: 0 * 200 + 1 * 100 + 1 * 50 + 3 * 20 + 29 * 10
172: 0 * 200 + 1 * 100 + 1 * 50 + 4 * 20 + 27 * 10
173: 0 * 200 + 1 * 100 + 1 * 50 + 5 * 20 + 25 * 10
174: 0 * 200 + 1 * 100 + 1 * 50 + 6 * 20 + 23 * 10
175: 0 * 200 + 1 * 100 + 1 * 50 + 7 * 20 + 21 * 10
176: 0 * 200 + 1 * 100 + 1 * 50 + 8 * 20 + 19 * 10
177: 0 * 200 + 1 * 100 + 1 * 50 + 9 * 20 + 17 * 10
178: 0 * 200 + 1 * 100 + 1 * 50 + 10 * 20 + 15 * 10
179: 0 * 200 + 1 * 100 + 1 * 50 + 11 * 20 + 13 * 10
180: 0 * 200 + 1 * 100 + 1 * 50 + 12 * 20 + 11 * 10
181: 0 * 200 + 1 * 100 + 1 * 50 + 13 * 20 + 9 * 10
182: 0 * 200 + 1 * 100 + 1 * 50 + 14 * 20 + 7 * 10
183: 0 * 200 + 1 * 100 + 1 * 50 + 15 * 20 + 5 * 10
184: 0 * 200 + 1 * 100 + 1 * 50 + 16 * 20 + 3 * 10
185: 0 * 200 + 1 * 100 + 1 * 50 + 17 * 20 + 1 * 10
186: 0 * 200 + 1 * 100 + 2 * 50 + 0 * 20 + 30 * 10
187: 0 * 200 + 1 * 100 + 2 * 50 + 1 * 20 + 28 * 10
188: 0 * 200 + 1 * 100 + 2 * 50 + 2 * 20 + 26 * 10
189: 0 * 200 + 1 * 100 + 2 * 50 + 3 * 20 + 24 * 10
190: 0 * 200 + 1 * 100 + 2 * 50 + 4 * 20 + 22 * 10
191: 0 * 200 + 1 * 100 + 2 * 50 + 5 * 20 + 20 * 10
192: 0 * 200 + 1 * 100 + 2 * 50 + 6 * 20 + 18 * 10
193: 0 * 200 + 1 * 100 + 2 * 50 + 7 * 20 + 16 * 10
194: 0 * 200 + 1 * 100 + 2 * 50 + 8 * 20 + 14 * 10
195: 0 * 200 + 1 * 100 + 2 * 50 + 9 * 20 + 12 * 10
196: 0 * 200 + 1 * 100 + 2 * 50 + 10 * 20 + 10 * 10
197: 0 * 200 + 1 * 100 + 2 * 50 + 11 * 20 + 8 * 10
198: 0 * 200 + 1 * 100 + 2 * 50 + 12 * 20 + 6 * 10
199: 0 * 200 + 1 * 100 + 2 * 50 + 13 * 20 + 4 * 10
200: 0 * 200 + 1 * 100 + 2 * 50 + 14 * 20 + 2 * 10
201: 0 * 200 + 1 * 100 + 2 * 50 + 15 * 20 + 0 * 10
202: 0 * 200 + 1 * 100 + 3 * 50 + 0 * 20 + 25 * 10
203: 0 * 200 + 1 * 100 + 3 * 50 + 1 * 20 + 23 * 10
204: 0 * 200 + 1 * 100 + 3 * 50 + 2 * 20 + 21 * 10
205: 0 * 200 + 1 * 100 + 3 * 50 + 3 * 20 + 19 * 10
206: 0 * 200 + 1 * 100 + 3 * 50 + 4 * 20 + 17 * 10
207: 0 * 200 + 1 * 100 + 3 * 50 + 5 * 20 + 15 * 10
208: 0 * 200 + 1 * 100 + 3 * 50 + 6 * 20 + 13 * 10
209: 0 * 200 + 1 * 100 + 3 * 50 + 7 * 20 + 11 * 10
210: 0 * 200 + 1 * 100 + 3 * 50 + 8 * 20 + 9 * 10
211: 0 * 200 + 1 * 100 + 3 * 50 + 9 * 20 + 7 * 10
212: 0 * 200 + 1 * 100 + 3 * 50 + 10 * 20 + 5 * 10
213: 0 * 200 + 1 * 100 + 3 * 50 + 11 * 20 + 3 * 10
214: 0 * 200 + 1 * 100 + 3 * 50 + 12 * 20 + 1 * 10
215: 0 * 200 + 1 * 100 + 4 * 50 + 0 * 20 + 20 * 10
216: 0 * 200 + 1 * 100 + 4 * 50 + 1 * 20 + 18 * 10
217: 0 * 200 + 1 * 100 + 4 * 50 + 2 * 20 + 16 * 10
218: 0 * 200 + 1 * 100 + 4 * 50 + 3 * 20 + 14 * 10
219: 0 * 200 + 1 * 100 + 4 * 50 + 4 * 20 + 12 * 10
220: 0 * 200 + 1 * 100 + 4 * 50 + 5 * 20 + 10 * 10
221: 0 * 200 + 1 * 100 + 4 * 50 + 6 * 20 + 8 * 10
222: 0 * 200 + 1 * 100 + 4 * 50 + 7 * 20 + 6 * 10
223: 0 * 200 + 1 * 100 + 4 * 50 + 8 * 20 + 4 * 10
224: 0 * 200 + 1 * 100 + 4 * 50 + 9 * 20 + 2 * 10
225: 0 * 200 + 1 * 100 + 4 * 50 + 10 * 20 + 0 * 10
226: 0 * 200 + 1 * 100 + 5 * 50 + 0 * 20 + 15 * 10
227: 0 * 200 + 1 * 100 + 5 * 50 + 1 * 20 + 13 * 10
228: 0 * 200 + 1 * 100 + 5 * 50 + 2 * 20 + 11 * 10
229: 0 * 200 + 1 * 100 + 5 * 50 + 3 * 20 + 9 * 10
230: 0 * 200 + 1 * 100 + 5 * 50 + 4 * 20 + 7 * 10
231: 0 * 200 + 1 * 100 + 5 * 50 + 5 * 20 + 5 * 10
232: 0 * 200 + 1 * 100 + 5 * 50 + 6 * 20 + 3 * 10
233: 0 * 200 + 1 * 100 + 5 * 50 + 7 * 20 + 1 * 10
234: 0 * 200 + 1 * 100 + 6 * 50 + 0 * 20 + 10 * 10
235: 0 * 200 + 1 * 100 + 6 * 50 + 1 * 20 + 8 * 10
236: 0 * 200 + 1 * 100 + 6 * 50 + 2 * 20 + 6 * 10
237: 0 * 200 + 1 * 100 + 6 * 50 + 3 * 20 + 4 * 10
238: 0 * 200 + 1 * 100 + 6 * 50 + 4 * 20 + 2 * 10
239: 0 * 200 + 1 * 100 + 6 * 50 + 5 * 20 + 0 * 10
240: 0 * 200 + 1 * 100 + 7 * 50 + 0 * 20 + 5 * 10
241: 0 * 200 + 1 * 100 + 7 * 50 + 1 * 20 + 3 * 10
242: 0 * 200 + 1 * 100 + 7 * 50 + 2 * 20 + 1 * 10
243: 0 * 200 + 1 * 100 + 8 * 50 + 0 * 20 + 0 * 10
244: 0 * 200 + 2 * 100 + 0 * 50 + 0 * 20 + 30 * 10
245: 0 * 200 + 2 * 100 + 0 * 50 + 1 * 20 + 28 * 10
246: 0 * 200 + 2 * 100 + 0 * 50 + 2 * 20 + 26 * 10
247: 0 * 200 + 2 * 100 + 0 * 50 + 3 * 20 + 24 * 10
248: 0 * 200 + 2 * 100 + 0 * 50 + 4 * 20 + 22 * 10
249: 0 * 200 + 2 * 100 + 0 * 50 + 5 * 20 + 20 * 10
250: 0 * 200 + 2 * 100 + 0 * 50 + 6 * 20 + 18 * 10
251: 0 * 200 + 2 * 100 + 0 * 50 + 7 * 20 + 16 * 10
252: 0 * 200 + 2 * 100 + 0 * 50 + 8 * 20 + 14 * 10
253: 0 * 200 + 2 * 100 + 0 * 50 + 9 * 20 + 12 * 10
254: 0 * 200 + 2 * 100 + 0 * 50 + 10 * 20 + 10 * 10
255: 0 * 200 + 2 * 100 + 0 * 50 + 11 * 20 + 8 * 10
256: 0 * 200 + 2 * 100 + 0 * 50 + 12 * 20 + 6 * 10
257: 0 * 200 + 2 * 100 + 0 * 50 + 13 * 20 + 4 * 10
258: 0 * 200 + 2 * 100 + 0 * 50 + 14 * 20 + 2 * 10
259: 0 * 200 + 2 * 100 + 0 * 50 + 15 * 20 + 0 * 10
260: 0 * 200 + 2 * 100 + 1 * 50 + 0 * 20 + 25 * 10
261: 0 * 200 + 2 * 100 + 1 * 50 + 1 * 20 + 23 * 10
262: 0 * 200 + 2 * 100 + 1 * 50 + 2 * 20 + 21 * 10
263: 0 * 200 + 2 * 100 + 1 * 50 + 3 * 20 + 19 * 10
264: 0 * 200 + 2 * 100 + 1 * 50 + 4 * 20 + 17 * 10
265: 0 * 200 + 2 * 100 + 1 * 50 + 5 * 20 + 15 * 10
266: 0 * 200 + 2 * 100 + 1 * 50 + 6 * 20 + 13 * 10
267: 0 * 200 + 2 * 100 + 1 * 50 + 7 * 20 + 11 * 10
268: 0 * 200 + 2 * 100 + 1 * 50 + 8 * 20 + 9 * 10
269: 0 * 200 + 2 * 100 + 1 * 50 + 9 * 20 + 7 * 10
270: 0 * 200 + 2 * 100 + 1 * 50 + 10 * 20 + 5 * 10
271: 0 * 200 + 2 * 100 + 1 * 50 + 11 * 20 + 3 * 10
272: 0 * 200 + 2 * 100 + 1 * 50 + 12 * 20 + 1 * 10
273: 0 * 200 + 2 * 100 + 2 * 50 + 0 * 20 + 20 * 10
274: 0 * 200 + 2 * 100 + 2 * 50 + 1 * 20 + 18 * 10
275: 0 * 200 + 2 * 100 + 2 * 50 + 2 * 20 + 16 * 10
276: 0 * 200 + 2 * 100 + 2 * 50 + 3 * 20 + 14 * 10
277: 0 * 200 + 2 * 100 + 2 * 50 + 4 * 20 + 12 * 10
278: 0 * 200 + 2 * 100 + 2 * 50 + 5 * 20 + 10 * 10
279: 0 * 200 + 2 * 100 + 2 * 50 + 6 * 20 + 8 * 10
280: 0 * 200 + 2 * 100 + 2 * 50 + 7 * 20 + 6 * 10
281: 0 * 200 + 2 * 100 + 2 * 50 + 8 * 20 + 4 * 10
282: 0 * 200 + 2 * 100 + 2 * 50 + 9 * 20 + 2 * 10
283: 0 * 200 + 2 * 100 + 2 * 50 + 10 * 20 + 0 * 10
284: 0 * 200 + 2 * 100 + 3 * 50 + 0 * 20 + 15 * 10
285: 0 * 200 + 2 * 100 + 3 * 50 + 1 * 20 + 13 * 10
286: 0 * 200 + 2 * 100 + 3 * 50 + 2 * 20 + 11 * 10
287: 0 * 200 + 2 * 100 + 3 * 50 + 3 * 20 + 9 * 10
288: 0 * 200 + 2 * 100 + 3 * 50 + 4 * 20 + 7 * 10
289: 0 * 200 + 2 * 100 + 3 * 50 + 5 * 20 + 5 * 10
290: 0 * 200 + 2 * 100 + 3 * 50 + 6 * 20 + 3 * 10
291: 0 * 200 + 2 * 100 + 3 * 50 + 7 * 20 + 1 * 10
292: 0 * 200 + 2 * 100 + 4 * 50 + 0 * 20 + 10 * 10
293: 0 * 200 + 2 * 100 + 4 * 50 + 1 * 20 + 8 * 10
294: 0 * 200 + 2 * 100 + 4 * 50 + 2 * 20 + 6 * 10
295: 0 * 200 + 2 * 100 + 4 * 50 + 3 * 20 + 4 * 10
296: 0 * 200 + 2 * 100 + 4 * 50 + 4 * 20 + 2 * 10
297: 0 * 200 + 2 * 100 + 4 * 50 + 5 * 20 + 0 * 10
298: 0 * 200 + 2 * 100 + 5 * 50 + 0 * 20 + 5 * 10
299: 0 * 200 + 2 * 100 + 5 * 50 + 1 * 20 + 3 * 10
300: 0 * 200 + 2 * 100 + 5 * 50 + 2 * 20 + 1 * 10
301: 0 * 200 + 2 * 100 + 6 * 50 + 0 * 20 + 0 * 10
302: 0 * 200 + 3 * 100 + 0 * 50 + 0 * 20 + 20 * 10
303: 0 * 200 + 3 * 100 + 0 * 50 + 1 * 20 + 18 * 10
304: 0 * 200 + 3 * 100 + 0 * 50 + 2 * 20 + 16 * 10
305: 0 * 200 + 3 * 100 + 0 * 50 + 3 * 20 + 14 * 10
306: 0 * 200 + 3 * 100 + 0 * 50 + 4 * 20 + 12 * 10
307: 0 * 200 + 3 * 100 + 0 * 50 + 5 * 20 + 10 * 10
308: 0 * 200 + 3 * 100 + 0 * 50 + 6 * 20 + 8 * 10
309: 0 * 200 + 3 * 100 + 0 * 50 + 7 * 20 + 6 * 10
310: 0 * 200 + 3 * 100 + 0 * 50 + 8 * 20 + 4 * 10
311: 0 * 200 + 3 * 100 + 0 * 50 + 9 * 20 + 2 * 10
312: 0 * 200 + 3 * 100 + 0 * 50 + 10 * 20 + 0 * 10
313: 0 * 200 + 3 * 100 + 1 * 50 + 0 * 20 + 15 * 10
314: 0 * 200 + 3 * 100 + 1 * 50 + 1 * 20 + 13 * 10
315: 0 * 200 + 3 * 100 + 1 * 50 + 2 * 20 + 11 * 10
316: 0 * 200 + 3 * 100 + 1 * 50 + 3 * 20 + 9 * 10
317: 0 * 200 + 3 * 100 + 1 * 50 + 4 * 20 + 7 * 10
318: 0 * 200 + 3 * 100 + 1 * 50 + 5 * 20 + 5 * 10
319: 0 * 200 + 3 * 100 + 1 * 50 + 6 * 20 + 3 * 10
320: 0 * 200 + 3 * 100 + 1 * 50 + 7 * 20 + 1 * 10
321: 0 * 200 + 3 * 100 + 2 * 50 + 0 * 20 + 10 * 10
322: 0 * 200 + 3 * 100 + 2 * 50 + 1 * 20 + 8 * 10
323: 0 * 200 + 3 * 100 + 2 * 50 + 2 * 20 + 6 * 10
324: 0 * 200 + 3 * 100 + 2 * 50 + 3 * 20 + 4 * 10
325: 0 * 200 + 3 * 100 + 2 * 50 + 4 * 20 + 2 * 10
326: 0 * 200 + 3 * 100 + 2 * 50 + 5 * 20 + 0 * 10
327: 0 * 200 + 3 * 100 + 3 * 50 + 0 * 20 + 5 * 10
328: 0 * 200 + 3 * 100 + 3 * 50 + 1 * 20 + 3 * 10
329: 0 * 200 + 3 * 100 + 3 * 50 + 2 * 20 + 1 * 10
330: 0 * 200 + 3 * 100 + 4 * 50 + 0 * 20 + 0 * 10
331: 0 * 200 + 4 * 100 + 0 * 50 + 0 * 20 + 10 * 10
332: 0 * 200 + 4 * 100 + 0 * 50 + 1 * 20 + 8 * 10
333: 0 * 200 + 4 * 100 + 0 * 50 + 2 * 20 + 6 * 10
334: 0 * 200 + 4 * 100 + 0 * 50 + 3 * 20 + 4 * 10
335: 0 * 200 + 4 * 100 + 0 * 50 + 4 * 20 + 2 * 10
336: 0 * 200 + 4 * 100 + 0 * 50 + 5 * 20 + 0 * 10
337: 0 * 200 + 4 * 100 + 1 * 50 + 0 * 20 + 5 * 10
338: 0 * 200 + 4 * 100 + 1 * 50 + 1 * 20 + 3 * 10
339: 0 * 200 + 4 * 100 + 1 * 50 + 2 * 20 + 1 * 10
340: 0 * 200 + 4 * 100 + 2 * 50 + 0 * 20 + 0 * 10
341: 0 * 200 + 5 * 100 + 0 * 50 + 0 * 20 + 0 * 10
342: 1 * 200 + 0 * 100 + 0 * 50 + 0 * 20 + 30 * 10
343: 1 * 200 + 0 * 100 + 0 * 50 + 1 * 20 + 28 * 10
344: 1 * 200 + 0 * 100 + 0 * 50 + 2 * 20 + 26 * 10
345: 1 * 200 + 0 * 100 + 0 * 50 + 3 * 20 + 24 * 10
346: 1 * 200 + 0 * 100 + 0 * 50 + 4 * 20 + 22 * 10
347: 1 * 200 + 0 * 100 + 0 * 50 + 5 * 20 + 20 * 10
348: 1 * 200 + 0 * 100 + 0 * 50 + 6 * 20 + 18 * 10
349: 1 * 200 + 0 * 100 + 0 * 50 + 7 * 20 + 16 * 10
350: 1 * 200 + 0 * 100 + 0 * 50 + 8 * 20 + 14 * 10
351: 1 * 200 + 0 * 100 + 0 * 50 + 9 * 20 + 12 * 10
352: 1 * 200 + 0 * 100 + 0 * 50 + 10 * 20 + 10 * 10
353: 1 * 200 + 0 * 100 + 0 * 50 + 11 * 20 + 8 * 10
354: 1 * 200 + 0 * 100 + 0 * 50 + 12 * 20 + 6 * 10
355: 1 * 200 + 0 * 100 + 0 * 50 + 13 * 20 + 4 * 10
356: 1 * 200 + 0 * 100 + 0 * 50 + 14 * 20 + 2 * 10
357: 1 * 200 + 0 * 100 + 0 * 50 + 15 * 20 + 0 * 10
358: 1 * 200 + 0 * 100 + 1 * 50 + 0 * 20 + 25 * 10
359: 1 * 200 + 0 * 100 + 1 * 50 + 1 * 20 + 23 * 10
360: 1 * 200 + 0 * 100 + 1 * 50 + 2 * 20 + 21 * 10
361: 1 * 200 + 0 * 100 + 1 * 50 + 3 * 20 + 19 * 10
362: 1 * 200 + 0 * 100 + 1 * 50 + 4 * 20 + 17 * 10
363: 1 * 200 + 0 * 100 + 1 * 50 + 5 * 20 + 15 * 10
364: 1 * 200 + 0 * 100 + 1 * 50 + 6 * 20 + 13 * 10
365: 1 * 200 + 0 * 100 + 1 * 50 + 7 * 20 + 11 * 10
366: 1 * 200 + 0 * 100 + 1 * 50 + 8 * 20 + 9 * 10
367: 1 * 200 + 0 * 100 + 1 * 50 + 9 * 20 + 7 * 10
368: 1 * 200 + 0 * 100 + 1 * 50 + 10 * 20 + 5 * 10
369: 1 * 200 + 0 * 100 + 1 * 50 + 11 * 20 + 3 * 10
370: 1 * 200 + 0 * 100 + 1 * 50 + 12 * 20 + 1 * 10
371: 1 * 200 + 0 * 100 + 2 * 50 + 0 * 20 + 20 * 10
372: 1 * 200 + 0 * 100 + 2 * 50 + 1 * 20 + 18 * 10
373: 1 * 200 + 0 * 100 + 2 * 50 + 2 * 20 + 16 * 10
374: 1 * 200 + 0 * 100 + 2 * 50 + 3 * 20 + 14 * 10
375: 1 * 200 + 0 * 100 + 2 * 50 + 4 * 20 + 12 * 10
376: 1 * 200 + 0 * 100 + 2 * 50 + 5 * 20 + 10 * 10
377: 1 * 200 + 0 * 100 + 2 * 50 + 6 * 20 + 8 * 10
378: 1 * 200 + 0 * 100 + 2 * 50 + 7 * 20 + 6 * 10
379: 1 * 200 + 0 * 100 + 2 * 50 + 8 * 20 + 4 * 10
380: 1 * 200 + 0 * 100 + 2 * 50 + 9 * 20 + 2 * 10
381: 1 * 200 + 0 * 100 + 2 * 50 + 10 * 20 + 0 * 10
382: 1 * 200 + 0 * 100 + 3 * 50 + 0 * 20 + 15 * 10
383: 1 * 200 + 0 * 100 + 3 * 50 + 1 * 20 + 13 * 10
384: 1 * 200 + 0 * 100 + 3 * 50 + 2 * 20 + 11 * 10
385: 1 * 200 + 0 * 100 + 3 * 50 + 3 * 20 + 9 * 10
386: 1 * 200 + 0 * 100 + 3 * 50 + 4 * 20 + 7 * 10
387: 1 * 200 + 0 * 100 + 3 * 50 + 5 * 20 + 5 * 10
388: 1 * 200 + 0 * 100 + 3 * 50 + 6 * 20 + 3 * 10
389: 1 * 200 + 0 * 100 + 3 * 50 + 7 * 20 + 1 * 10
390: 1 * 200 + 0 * 100 + 4 * 50 + 0 * 20 + 10 * 10
391: 1 * 200 + 0 * 100 + 4 * 50 + 1 * 20 + 8 * 10
392: 1 * 200 + 0 * 100 + 4 * 50 + 2 * 20 + 6 * 10
393: 1 * 200 + 0 * 100 + 4 * 50 + 3 * 20 + 4 * 10
394: 1 * 200 + 0 * 100 + 4 * 50 + 4 * 20 + 2 * 10
395: 1 * 200 + 0 * 100 + 4 * 50 + 5 * 20 + 0 * 10
396: 1 * 200 + 0 * 100 + 5 * 50 + 0 * 20 + 5 * 10
397: 1 * 200 + 0 * 100 + 5 * 50 + 1 * 20 + 3 * 10
398: 1 * 200 + 0 * 100 + 5 * 50 + 2 * 20 + 1 * 10
399: 1 * 200 + 0 * 100 + 6 * 50 + 0 * 20 + 0 * 10
400: 1 * 200 + 1 * 100 + 0 * 50 + 0 * 20 + 20 * 10
401: 1 * 200 + 1 * 100 + 0 * 50 + 1 * 20 + 18 * 10
402: 1 * 200 + 1 * 100 + 0 * 50 + 2 * 20 + 16 * 10
403: 1 * 200 + 1 * 100 + 0 * 50 + 3 * 20 + 14 * 10
404: 1 * 200 + 1 * 100 + 0 * 50 + 4 * 20 + 12 * 10
405: 1 * 200 + 1 * 100 + 0 * 50 + 5 * 20 + 10 * 10
406: 1 * 200 + 1 * 100 + 0 * 50 + 6 * 20 + 8 * 10
407: 1 * 200 + 1 * 100 + 0 * 50 + 7 * 20 + 6 * 10
408: 1 * 200 + 1 * 100 + 0 * 50 + 8 * 20 + 4 * 10
409: 1 * 200 + 1 * 100 + 0 * 50 + 9 * 20 + 2 * 10
410: 1 * 200 + 1 * 100 + 0 * 50 + 10 * 20 + 0 * 10
411: 1 * 200 + 1 * 100 + 1 * 50 + 0 * 20 + 15 * 10
412: 1 * 200 + 1 * 100 + 1 * 50 + 1 * 20 + 13 * 10
413: 1 * 200 + 1 * 100 + 1 * 50 + 2 * 20 + 11 * 10
414: 1 * 200 + 1 * 100 + 1 * 50 + 3 * 20 + 9 * 10
415: 1 * 200 + 1 * 100 + 1 * 50 + 4 * 20 + 7 * 10
416: 1 * 200 + 1 * 100 + 1 * 50 + 5 * 20 + 5 * 10
417: 1 * 200 + 1 * 100 + 1 * 50 + 6 * 20 + 3 * 10
418: 1 * 200 + 1 * 100 + 1 * 50 + 7 * 20 + 1 * 10
419: 1 * 200 + 1 * 100 + 2 * 50 + 0 * 20 + 10 * 10
420: 1 * 200 + 1 * 100 + 2 * 50 + 1 * 20 + 8 * 10
421: 1 * 200 + 1 * 100 + 2 * 50 + 2 * 20 + 6 * 10
422: 1 * 200 + 1 * 100 + 2 * 50 + 3 * 20 + 4 * 10
423: 1 * 200 + 1 * 100 + 2 * 50 + 4 * 20 + 2 * 10
424: 1 * 200 + 1 * 100 + 2 * 50 + 5 * 20 + 0 * 10
425: 1 * 200 + 1 * 100 + 3 * 50 + 0 * 20 + 5 * 10
426: 1 * 200 + 1 * 100 + 3 * 50 + 1 * 20 + 3 * 10
427: 1 * 200 + 1 * 100 + 3 * 50 + 2 * 20 + 1 * 10
428: 1 * 200 + 1 * 100 + 4 * 50 + 0 * 20 + 0 * 10
429: 1 * 200 + 2 * 100 + 0 * 50 + 0 * 20 + 10 * 10
430: 1 * 200 + 2 * 100 + 0 * 50 + 1 * 20 + 8 * 10
431: 1 * 200 + 2 * 100 + 0 * 50 + 2 * 20 + 6 * 10
432: 1 * 200 + 2 * 100 + 0 * 50 + 3 * 20 + 4 * 10
433: 1 * 200 + 2 * 100 + 0 * 50 + 4 * 20 + 2 * 10
434: 1 * 200 + 2 * 100 + 0 * 50 + 5 * 20 + 0 * 10
435: 1 * 200 + 2 * 100 + 1 * 50 + 0 * 20 + 5 * 10
436: 1 * 200 + 2 * 100 + 1 * 50 + 1 * 20 + 3 * 10
437: 1 * 200 + 2 * 100 + 1 * 50 + 2 * 20 + 1 * 10
438: 1 * 200 + 2 * 100 + 2 * 50 + 0 * 20 + 0 * 10
439: 1 * 200 + 3 * 100 + 0 * 50 + 0 * 20 + 0 * 10
440: 2 * 200 + 0 * 100 + 0 * 50 + 0 * 20 + 10 * 10
441: 2 * 200 + 0 * 100 + 0 * 50 + 1 * 20 + 8 * 10
442: 2 * 200 + 0 * 100 + 0 * 50 + 2 * 20 + 6 * 10
443: 2 * 200 + 0 * 100 + 0 * 50 + 3 * 20 + 4 * 10
444: 2 * 200 + 0 * 100 + 0 * 50 + 4 * 20 + 2 * 10
445: 2 * 200 + 0 * 100 + 0 * 50 + 5 * 20 + 0 * 10
446: 2 * 200 + 0 * 100 + 1 * 50 + 0 * 20 + 5 * 10
447: 2 * 200 + 0 * 100 + 1 * 50 + 1 * 20 + 3 * 10
448: 2 * 200 + 0 * 100 + 1 * 50 + 2 * 20 + 1 * 10
449: 2 * 200 + 0 * 100 + 2 * 50 + 0 * 20 + 0 * 10
450: 2 * 200 + 1 * 100 + 0 * 50 + 0 * 20 + 0 * 10

5, 10, 20, 50, 100 und 200 Euroscheine

1: 0 * 200 + 0 * 100 + 0 * 50 + 0 * 20 + 0 * 10 + 100 * 5
2: 0 * 200 + 0 * 100 + 0 * 50 + 0 * 20 + 1 * 10 + 98 * 5
3: 0 * 200 + 0 * 100 + 0 * 50 + 0 * 20 + 2 * 10 + 96 * 5
4: 0 * 200 + 0 * 100 + 0 * 50 + 0 * 20 + 3 * 10 + 94 * 5
5: 0 * 200 + 0 * 100 + 0 * 50 + 0 * 20 + 4 * 10 + 92 * 5
6: 0 * 200 + 0 * 100 + 0 * 50 + 0 * 20 + 5 * 10 + 90 * 5
7: 0 * 200 + 0 * 100 + 0 * 50 + 0 * 20 + 6 * 10 + 88 * 5
8: 0 * 200 + 0 * 100 + 0 * 50 + 0 * 20 + 7 * 10 + 86 * 5
9: 0 * 200 + 0 * 100 + 0 * 50 + 0 * 20 + 8 * 10 + 84 * 5
10: 0 * 200 + 0 * 100 + 0 * 50 + 0 * 20 + 9 * 10 + 82 * 5
11: 0 * 200 + 0 * 100 + 0 * 50 + 0 * 20 + 10 * 10 + 80 * 5
12: 0 * 200 + 0 * 100 + 0 * 50 + 0 * 20 + 11 * 10 + 78 * 5
13: 0 * 200 + 0 * 100 + 0 * 50 + 0 * 20 + 12 * 10 + 76 * 5
14: 0 * 200 + 0 * 100 + 0 * 50 + 0 * 20 + 13 * 10 + 74 * 5
15: 0 * 200 + 0 * 100 + 0 * 50 + 0 * 20 + 14 * 10 + 72 * 5
16: 0 * 200 + 0 * 100 + 0 * 50 + 0 * 20 + 15 * 10 + 70 * 5
17: 0 * 200 + 0 * 100 + 0 * 50 + 0 * 20 + 16 * 10 + 68 * 5
18: 0 * 200 + 0 * 100 + 0 * 50 + 0 * 20 + 17 * 10 + 66 * 5
19: 0 * 200 + 0 * 100 + 0 * 50 + 0 * 20 + 18 * 10 + 64 * 5
20: 0 * 200 + 0 * 100 + 0 * 50 + 0 * 20 + 19 * 10 + 62 * 5
21: 0 * 200 + 0 * 100 + 0 * 50 + 0 * 20 + 20 * 10 + 60 * 5
22: 0 * 200 + 0 * 100 + 0 * 50 + 0 * 20 + 21 * 10 + 58 * 5
23: 0 * 200 + 0 * 100 + 0 * 50 + 0 * 20 + 22 * 10 + 56 * 5
24: 0 * 200 + 0 * 100 + 0 * 50 + 0 * 20 + 23 * 10 + 54 * 5
25: 0 * 200 + 0 * 100 + 0 * 50 + 0 * 20 + 24 * 10 + 52 * 5
26: 0 * 200 + 0 * 100 + 0 * 50 + 0 * 20 + 25 * 10 + 50 * 5
27: 0 * 200 + 0 * 100 + 0 * 50 + 0 * 20 + 26 * 10 + 48 * 5
28: 0 * 200 + 0 * 100 + 0 * 50 + 0 * 20 + 27 * 10 + 46 * 5
29: 0 * 200 + 0 * 100 + 0 * 50 + 0 * 20 + 28 * 10 + 44 * 5
30: 0 * 200 + 0 * 100 + 0 * 50 + 0 * 20 + 29 * 10 + 42 * 5
31: 0 * 200 + 0 * 100 + 0 * 50 + 0 * 20 + 30 * 10 + 40 * 5
32: 0 * 200 + 0 * 100 + 0 * 50 + 0 * 20 + 31 * 10 + 38 * 5
33: 0 * 200 + 0 * 100 + 0 * 50 + 0 * 20 + 32 * 10 + 36 * 5
34: 0 * 200 + 0 * 100 + 0 * 50 + 0 * 20 + 33 * 10 + 34 * 5
35: 0 * 200 + 0 * 100 + 0 * 50 + 0 * 20 + 34 * 10 + 32 * 5
36: 0 * 200 + 0 * 100 + 0 * 50 + 0 * 20 + 35 * 10 + 30 * 5
37: 0 * 200 + 0 * 100 + 0 * 50 + 0 * 20 + 36 * 10 + 28 * 5
38: 0 * 200 + 0 * 100 + 0 * 50 + 0 * 20 + 37 * 10 + 26 * 5
39: 0 * 200 + 0 * 100 + 0 * 50 + 0 * 20 + 38 * 10 + 24 * 5
40: 0 * 200 + 0 * 100 + 0 * 50 + 0 * 20 + 39 * 10 + 22 * 5
41: 0 * 200 + 0 * 100 + 0 * 50 + 0 * 20 + 40 * 10 + 20 * 5
42: 0 * 200 + 0 * 100 + 0 * 50 + 0 * 20 + 41 * 10 + 18 * 5
43: 0 * 200 + 0 * 100 + 0 * 50 + 0 * 20 + 42 * 10 + 16 * 5
44: 0 * 200 + 0 * 100 + 0 * 50 + 0 * 20 + 43 * 10 + 14 * 5
45: 0 * 200 + 0 * 100 + 0 * 50 + 0 * 20 + 44 * 10 + 12 * 5
46: 0 * 200 + 0 * 100 + 0 * 50 + 0 * 20 + 45 * 10 + 10 * 5
47: 0 * 200 + 0 * 100 + 0 * 50 + 0 * 20 + 46 * 10 + 8 * 5
48: 0 * 200 + 0 * 100 + 0 * 50 + 0 * 20 + 47 * 10 + 6 * 5
49: 0 * 200 + 0 * 100 + 0 * 50 + 0 * 20 + 48 * 10 + 4 * 5
50: 0 * 200 + 0 * 100 + 0 * 50 + 0 * 20 + 49 * 10 + 2 * 5
51: 0 * 200 + 0 * 100 + 0 * 50 + 0 * 20 + 50 * 10 + 0 * 5
52: 0 * 200 + 0 * 100 + 0 * 50 + 1 * 20 + 0 * 10 + 96 * 5
53: 0 * 200 + 0 * 100 + 0 * 50 + 1 * 20 + 1 * 10 + 94 * 5
54: 0 * 200 + 0 * 100 + 0 * 50 + 1 * 20 + 2 * 10 + 92 * 5
55: 0 * 200 + 0 * 100 + 0 * 50 + 1 * 20 + 3 * 10 + 90 * 5
56: 0 * 200 + 0 * 100 + 0 * 50 + 1 * 20 + 4 * 10 + 88 * 5
57: 0 * 200 + 0 * 100 + 0 * 50 + 1 * 20 + 5 * 10 + 86 * 5
58: 0 * 200 + 0 * 100 + 0 * 50 + 1 * 20 + 6 * 10 + 84 * 5
59: 0 * 200 + 0 * 100 + 0 * 50 + 1 * 20 + 7 * 10 + 82 * 5
60: 0 * 200 + 0 * 100 + 0 * 50 + 1 * 20 + 8 * 10 + 80 * 5
61: 0 * 200 + 0 * 100 + 0 * 50 + 1 * 20 + 9 * 10 + 78 * 5
62: 0 * 200 + 0 * 100 + 0 * 50 + 1 * 20 + 10 * 10 + 76 * 5
63: 0 * 200 + 0 * 100 + 0 * 50 + 1 * 20 + 11 * 10 + 74 * 5
64: 0 * 200 + 0 * 100 + 0 * 50 + 1 * 20 + 12 * 10 + 72 * 5
65: 0 * 200 + 0 * 100 + 0 * 50 + 1 * 20 + 13 * 10 + 70 * 5
66: 0 * 200 + 0 * 100 + 0 * 50 + 1 * 20 + 14 * 10 + 68 * 5
67: 0 * 200 + 0 * 100 + 0 * 50 + 1 * 20 + 15 * 10 + 66 * 5
68: 0 * 200 + 0 * 100 + 0 * 50 + 1 * 20 + 16 * 10 + 64 * 5
69: 0 * 200 + 0 * 100 + 0 * 50 + 1 * 20 + 17 * 10 + 62 * 5
70: 0 * 200 + 0 * 100 + 0 * 50 + 1 * 20 + 18 * 10 + 60 * 5
71: 0 * 200 + 0 * 100 + 0 * 50 + 1 * 20 + 19 * 10 + 58 * 5
72: 0 * 200 + 0 * 100 + 0 * 50 + 1 * 20 + 20 * 10 + 56 * 5
73: 0 * 200 + 0 * 100 + 0 * 50 + 1 * 20 + 21 * 10 + 54 * 5
74: 0 * 200 + 0 * 100 + 0 * 50 + 1 * 20 + 22 * 10 + 52 * 5
75: 0 * 200 + 0 * 100 + 0 * 50 + 1 * 20 + 23 * 10 + 50 * 5
76: 0 * 200 + 0 * 100 + 0 * 50 + 1 * 20 + 24 * 10 + 48 * 5
77: 0 * 200 + 0 * 100 + 0 * 50 + 1 * 20 + 25 * 10 + 46 * 5
78: 0 * 200 + 0 * 100 + 0 * 50 + 1 * 20 + 26 * 10 + 44 * 5
79: 0 * 200 + 0 * 100 + 0 * 50 + 1 * 20 + 27 * 10 + 42 * 5
80: 0 * 200 + 0 * 100 + 0 * 50 + 1 * 20 + 28 * 10 + 40 * 5
81: 0 * 200 + 0 * 100 + 0 * 50 + 1 * 20 + 29 * 10 + 38 * 5
82: 0 * 200 + 0 * 100 + 0 * 50 + 1 * 20 + 30 * 10 + 36 * 5
83: 0 * 200 + 0 * 100 + 0 * 50 + 1 * 20 + 31 * 10 + 34 * 5
84: 0 * 200 + 0 * 100 + 0 * 50 + 1 * 20 + 32 * 10 + 32 * 5
85: 0 * 200 + 0 * 100 + 0 * 50 + 1 * 20 + 33 * 10 + 30 * 5
86: 0 * 200 + 0 * 100 + 0 * 50 + 1 * 20 + 34 * 10 + 28 * 5
87: 0 * 200 + 0 * 100 + 0 * 50 + 1 * 20 + 35 * 10 + 26 * 5
88: 0 * 200 + 0 * 100 + 0 * 50 + 1 * 20 + 36 * 10 + 24 * 5
89: 0 * 200 + 0 * 100 + 0 * 50 + 1 * 20 + 37 * 10 + 22 * 5
90: 0 * 200 + 0 * 100 + 0 * 50 + 1 * 20 + 38 * 10 + 20 * 5
91: 0 * 200 + 0 * 100 + 0 * 50 + 1 * 20 + 39 * 10 + 18 * 5
92: 0 * 200 + 0 * 100 + 0 * 50 + 1 * 20 + 40 * 10 + 16 * 5
93: 0 * 200 + 0 * 100 + 0 * 50 + 1 * 20 + 41 * 10 + 14 * 5
94: 0 * 200 + 0 * 100 + 0 * 50 + 1 * 20 + 42 * 10 + 12 * 5
95: 0 * 200 + 0 * 100 + 0 * 50 + 1 * 20 + 43 * 10 + 10 * 5
96: 0 * 200 + 0 * 100 + 0 * 50 + 1 * 20 + 44 * 10 + 8 * 5
97: 0 * 200 + 0 * 100 + 0 * 50 + 1 * 20 + 45 * 10 + 6 * 5
98: 0 * 200 + 0 * 100 + 0 * 50 + 1 * 20 + 46 * 10 + 4 * 5
99: 0 * 200 + 0 * 100 + 0 * 50 + 1 * 20 + 47 * 10 + 2 * 5
100: 0 * 200 + 0 * 100 + 0 * 50 + 1 * 20 + 48 * 10 + 0 * 5
101: 0 * 200 + 0 * 100 + 0 * 50 + 2 * 20 + 0 * 10 + 92 * 5
102: 0 * 200 + 0 * 100 + 0 * 50 + 2 * 20 + 1 * 10 + 90 * 5
103: 0 * 200 + 0 * 100 + 0 * 50 + 2 * 20 + 2 * 10 + 88 * 5
104: 0 * 200 + 0 * 100 + 0 * 50 + 2 * 20 + 3 * 10 + 86 * 5
105: 0 * 200 + 0 * 100 + 0 * 50 + 2 * 20 + 4 * 10 + 84 * 5
106: 0 * 200 + 0 * 100 + 0 * 50 + 2 * 20 + 5 * 10 + 82 * 5
107: 0 * 200 + 0 * 100 + 0 * 50 + 2 * 20 + 6 * 10 + 80 * 5
108: 0 * 200 + 0 * 100 + 0 * 50 + 2 * 20 + 7 * 10 + 78 * 5
109: 0 * 200 + 0 * 100 + 0 * 50 + 2 * 20 + 8 * 10 + 76 * 5
110: 0 * 200 + 0 * 100 + 0 * 50 + 2 * 20 + 9 * 10 + 74 * 5
111: 0 * 200 + 0 * 100 + 0 * 50 + 2 * 20 + 10 * 10 + 72 * 5
112: 0 * 200 + 0 * 100 + 0 * 50 + 2 * 20 + 11 * 10 + 70 * 5
113: 0 * 200 + 0 * 100 + 0 * 50 + 2 * 20 + 12 * 10 + 68 * 5
114: 0 * 200 + 0 * 100 + 0 * 50 + 2 * 20 + 13 * 10 + 66 * 5
115: 0 * 200 + 0 * 100 + 0 * 50 + 2 * 20 + 14 * 10 + 64 * 5
116: 0 * 200 + 0 * 100 + 0 * 50 + 2 * 20 + 15 * 10 + 62 * 5
117: 0 * 200 + 0 * 100 + 0 * 50 + 2 * 20 + 16 * 10 + 60 * 5
118: 0 * 200 + 0 * 100 + 0 * 50 + 2 * 20 + 17 * 10 + 58 * 5
119: 0 * 200 + 0 * 100 + 0 * 50 + 2 * 20 + 18 * 10 + 56 * 5
120: 0 * 200 + 0 * 100 + 0 * 50 + 2 * 20 + 19 * 10 + 54 * 5
121: 0 * 200 + 0 * 100 + 0 * 50 + 2 * 20 + 20 * 10 + 52 * 5
122: 0 * 200 + 0 * 100 + 0 * 50 + 2 * 20 + 21 * 10 + 50 * 5
123: 0 * 200 + 0 * 100 + 0 * 50 + 2 * 20 + 22 * 10 + 48 * 5
124: 0 * 200 + 0 * 100 + 0 * 50 + 2 * 20 + 23 * 10 + 46 * 5
125: 0 * 200 + 0 * 100 + 0 * 50 + 2 * 20 + 24 * 10 + 44 * 5
126: 0 * 200 + 0 * 100 + 0 * 50 + 2 * 20 + 25 * 10 + 42 * 5
127: 0 * 200 + 0 * 100 + 0 * 50 + 2 * 20 + 26 * 10 + 40 * 5
128: 0 * 200 + 0 * 100 + 0 * 50 + 2 * 20 + 27 * 10 + 38 * 5
129: 0 * 200 + 0 * 100 + 0 * 50 + 2 * 20 + 28 * 10 + 36 * 5
130: 0 * 200 + 0 * 100 + 0 * 50 + 2 * 20 + 29 * 10 + 34 * 5
131: 0 * 200 + 0 * 100 + 0 * 50 + 2 * 20 + 30 * 10 + 32 * 5
132: 0 * 200 + 0 * 100 + 0 * 50 + 2 * 20 + 31 * 10 + 30 * 5
133: 0 * 200 + 0 * 100 + 0 * 50 + 2 * 20 + 32 * 10 + 28 * 5
134: 0 * 200 + 0 * 100 + 0 * 50 + 2 * 20 + 33 * 10 + 26 * 5
135: 0 * 200 + 0 * 100 + 0 * 50 + 2 * 20 + 34 * 10 + 24 * 5
136: 0 * 200 + 0 * 100 + 0 * 50 + 2 * 20 + 35 * 10 + 22 * 5
137: 0 * 200 + 0 * 100 + 0 * 50 + 2 * 20 + 36 * 10 + 20 * 5
138: 0 * 200 + 0 * 100 + 0 * 50 + 2 * 20 + 37 * 10 + 18 * 5
139: 0 * 200 + 0 * 100 + 0 * 50 + 2 * 20 + 38 * 10 + 16 * 5
140: 0 * 200 + 0 * 100 + 0 * 50 + 2 * 20 + 39 * 10 + 14 * 5
141: 0 * 200 + 0 * 100 + 0 * 50 + 2 * 20 + 40 * 10 + 12 * 5
142: 0 * 200 + 0 * 100 + 0 * 50 + 2 * 20 + 41 * 10 + 10 * 5
143: 0 * 200 + 0 * 100 + 0 * 50 + 2 * 20 + 42 * 10 + 8 * 5
144: 0 * 200 + 0 * 100 + 0 * 50 + 2 * 20 + 43 * 10 + 6 * 5
145: 0 * 200 + 0 * 100 + 0 * 50 + 2 * 20 + 44 * 10 + 4 * 5
146: 0 * 200 + 0 * 100 + 0 * 50 + 2 * 20 + 45 * 10 + 2 * 5
147: 0 * 200 + 0 * 100 + 0 * 50 + 2 * 20 + 46 * 10 + 0 * 5
148: 0 * 200 + 0 * 100 + 0 * 50 + 3 * 20 + 0 * 10 + 88 * 5
149: 0 * 200 + 0 * 100 + 0 * 50 + 3 * 20 + 1 * 10 + 86 * 5
150: 0 * 200 + 0 * 100 + 0 * 50 + 3 * 20 + 2 * 10 + 84 * 5
151: 0 * 200 + 0 * 100 + 0 * 50 + 3 * 20 + 3 * 10 + 82 * 5
152: 0 * 200 + 0 * 100 + 0 * 50 + 3 * 20 + 4 * 10 + 80 * 5
153: 0 * 200 + 0 * 100 + 0 * 50 + 3 * 20 + 5 * 10 + 78 * 5
154: 0 * 200 + 0 * 100 + 0 * 50 + 3 * 20 + 6 * 10 + 76 * 5
155: 0 * 200 + 0 * 100 + 0 * 50 + 3 * 20 + 7 * 10 + 74 * 5
156: 0 * 200 + 0 * 100 + 0 * 50 + 3 * 20 + 8 * 10 + 72 * 5
157: 0 * 200 + 0 * 100 + 0 * 50 + 3 * 20 + 9 * 10 + 70 * 5
158: 0 * 200 + 0 * 100 + 0 * 50 + 3 * 20 + 10 * 10 + 68 * 5
159: 0 * 200 + 0 * 100 + 0 * 50 + 3 * 20 + 11 * 10 + 66 * 5
160: 0 * 200 + 0 * 100 + 0 * 50 + 3 * 20 + 12 * 10 + 64 * 5
161: 0 * 200 + 0 * 100 + 0 * 50 + 3 * 20 + 13 * 10 + 62 * 5
162: 0 * 200 + 0 * 100 + 0 * 50 + 3 * 20 + 14 * 10 + 60 * 5
163: 0 * 200 + 0 * 100 + 0 * 50 + 3 * 20 + 15 * 10 + 58 * 5
164: 0 * 200 + 0 * 100 + 0 * 50 + 3 * 20 + 16 * 10 + 56 * 5
165: 0 * 200 + 0 * 100 + 0 * 50 + 3 * 20 + 17 * 10 + 54 * 5
166: 0 * 200 + 0 * 100 + 0 * 50 + 3 * 20 + 18 * 10 + 52 * 5
167: 0 * 200 + 0 * 100 + 0 * 50 + 3 * 20 + 19 * 10 + 50 * 5
168: 0 * 200 + 0 * 100 + 0 * 50 + 3 * 20 + 20 * 10 + 48 * 5
169: 0 * 200 + 0 * 100 + 0 * 50 + 3 * 20 + 21 * 10 + 46 * 5
170: 0 * 200 + 0 * 100 + 0 * 50 + 3 * 20 + 22 * 10 + 44 * 5
171: 0 * 200 + 0 * 100 + 0 * 50 + 3 * 20 + 23 * 10 + 42 * 5
172: 0 * 200 + 0 * 100 + 0 * 50 + 3 * 20 + 24 * 10 + 40 * 5
173: 0 * 200 + 0 * 100 + 0 * 50 + 3 * 20 + 25 * 10 + 38 * 5
174: 0 * 200 + 0 * 100 + 0 * 50 + 3 * 20 + 26 * 10 + 36 * 5
175: 0 * 200 + 0 * 100 + 0 * 50 + 3 * 20 + 27 * 10 + 34 * 5
176: 0 * 200 + 0 * 100 + 0 * 50 + 3 * 20 + 28 * 10 + 32 * 5
177: 0 * 200 + 0 * 100 + 0 * 50 + 3 * 20 + 29 * 10 + 30 * 5
178: 0 * 200 + 0 * 100 + 0 * 50 + 3 * 20 + 30 * 10 + 28 * 5
179: 0 * 200 + 0 * 100 + 0 * 50 + 3 * 20 + 31 * 10 + 26 * 5
180: 0 * 200 + 0 * 100 + 0 * 50 + 3 * 20 + 32 * 10 + 24 * 5
181: 0 * 200 + 0 * 100 + 0 * 50 + 3 * 20 + 33 * 10 + 22 * 5
182: 0 * 200 + 0 * 100 + 0 * 50 + 3 * 20 + 34 * 10 + 20 * 5
183: 0 * 200 + 0 * 100 + 0 * 50 + 3 * 20 + 35 * 10 + 18 * 5
184: 0 * 200 + 0 * 100 + 0 * 50 + 3 * 20 + 36 * 10 + 16 * 5
185: 0 * 200 + 0 * 100 + 0 * 50 + 3 * 20 + 37 * 10 + 14 * 5
186: 0 * 200 + 0 * 100 + 0 * 50 + 3 * 20 + 38 * 10 + 12 * 5
187: 0 * 200 + 0 * 100 + 0 * 50 + 3 * 20 + 39 * 10 + 10 * 5
188: 0 * 200 + 0 * 100 + 0 * 50 + 3 * 20 + 40 * 10 + 8 * 5
189: 0 * 200 + 0 * 100 + 0 * 50 + 3 * 20 + 41 * 10 + 6 * 5
190: 0 * 200 + 0 * 100 + 0 * 50 + 3 * 20 + 42 * 10 + 4 * 5
191: 0 * 200 + 0 * 100 + 0 * 50 + 3 * 20 + 43 * 10 + 2 * 5
192: 0 * 200 + 0 * 100 + 0 * 50 + 3 * 20 + 44 * 10 + 0 * 5
193: 0 * 200 + 0 * 100 + 0 * 50 + 4 * 20 + 0 * 10 + 84 * 5
194: 0 * 200 + 0 * 100 + 0 * 50 + 4 * 20 + 1 * 10 + 82 * 5
195: 0 * 200 + 0 * 100 + 0 * 50 + 4 * 20 + 2 * 10 + 80 * 5
196: 0 * 200 + 0 * 100 + 0 * 50 + 4 * 20 + 3 * 10 + 78 * 5
197: 0 * 200 + 0 * 100 + 0 * 50 + 4 * 20 + 4 * 10 + 76 * 5
198: 0 * 200 + 0 * 100 + 0 * 50 + 4 * 20 + 5 * 10 + 74 * 5
199: 0 * 200 + 0 * 100 + 0 * 50 + 4 * 20 + 6 * 10 + 72 * 5
200: 0 * 200 + 0 * 100 + 0 * 50 + 4 * 20 + 7 * 10 + 70 * 5
201: 0 * 200 + 0 * 100 + 0 * 50 + 4 * 20 + 8 * 10 + 68 * 5
202: 0 * 200 + 0 * 100 + 0 * 50 + 4 * 20 + 9 * 10 + 66 * 5
203: 0 * 200 + 0 * 100 + 0 * 50 + 4 * 20 + 10 * 10 + 64 * 5
204: 0 * 200 + 0 * 100 + 0 * 50 + 4 * 20 + 11 * 10 + 62 * 5
205: 0 * 200 + 0 * 100 + 0 * 50 + 4 * 20 + 12 * 10 + 60 * 5
206: 0 * 200 + 0 * 100 + 0 * 50 + 4 * 20 + 13 * 10 + 58 * 5
207: 0 * 200 + 0 * 100 + 0 * 50 + 4 * 20 + 14 * 10 + 56 * 5
208: 0 * 200 + 0 * 100 + 0 * 50 + 4 * 20 + 15 * 10 + 54 * 5
209: 0 * 200 + 0 * 100 + 0 * 50 + 4 * 20 + 16 * 10 + 52 * 5
210: 0 * 200 + 0 * 100 + 0 * 50 + 4 * 20 + 17 * 10 + 50 * 5
211: 0 * 200 + 0 * 100 + 0 * 50 + 4 * 20 + 18 * 10 + 48 * 5
212: 0 * 200 + 0 * 100 + 0 * 50 + 4 * 20 + 19 * 10 + 46 * 5
213: 0 * 200 + 0 * 100 + 0 * 50 + 4 * 20 + 20 * 10 + 44 * 5
214: 0 * 200 + 0 * 100 + 0 * 50 + 4 * 20 + 21 * 10 + 42 * 5
215: 0 * 200 + 0 * 100 + 0 * 50 + 4 * 20 + 22 * 10 + 40 * 5
216: 0 * 200 + 0 * 100 + 0 * 50 + 4 * 20 + 23 * 10 + 38 * 5
217: 0 * 200 + 0 * 100 + 0 * 50 + 4 * 20 + 24 * 10 + 36 * 5
218: 0 * 200 + 0 * 100 + 0 * 50 + 4 * 20 + 25 * 10 + 34 * 5
219: 0 * 200 + 0 * 100 + 0 * 50 + 4 * 20 + 26 * 10 + 32 * 5
220: 0 * 200 + 0 * 100 + 0 * 50 + 4 * 20 + 27 * 10 + 30 * 5
221: 0 * 200 + 0 * 100 + 0 * 50 + 4 * 20 + 28 * 10 + 28 * 5
222: 0 * 200 + 0 * 100 + 0 * 50 + 4 * 20 + 29 * 10 + 26 * 5
223: 0 * 200 + 0 * 100 + 0 * 50 + 4 * 20 + 30 * 10 + 24 * 5
224: 0 * 200 + 0 * 100 + 0 * 50 + 4 * 20 + 31 * 10 + 22 * 5
225: 0 * 200 + 0 * 100 + 0 * 50 + 4 * 20 + 32 * 10 + 20 * 5
226: 0 * 200 + 0 * 100 + 0 * 50 + 4 * 20 + 33 * 10 + 18 * 5
227: 0 * 200 + 0 * 100 + 0 * 50 + 4 * 20 + 34 * 10 + 16 * 5
228: 0 * 200 + 0 * 100 + 0 * 50 + 4 * 20 + 35 * 10 + 14 * 5
229: 0 * 200 + 0 * 100 + 0 * 50 + 4 * 20 + 36 * 10 + 12 * 5
230: 0 * 200 + 0 * 100 + 0 * 50 + 4 * 20 + 37 * 10 + 10 * 5
231: 0 * 200 + 0 * 100 + 0 * 50 + 4 * 20 + 38 * 10 + 8 * 5
232: 0 * 200 + 0 * 100 + 0 * 50 + 4 * 20 + 39 * 10 + 6 * 5
233: 0 * 200 + 0 * 100 + 0 * 50 + 4 * 20 + 40 * 10 + 4 * 5
234: 0 * 200 + 0 * 100 + 0 * 50 + 4 * 20 + 41 * 10 + 2 * 5
235: 0 * 200 + 0 * 100 + 0 * 50 + 4 * 20 + 42 * 10 + 0 * 5
236: 0 * 200 + 0 * 100 + 0 * 50 + 5 * 20 + 0 * 10 + 80 * 5
237: 0 * 200 + 0 * 100 + 0 * 50 + 5 * 20 + 1 * 10 + 78 * 5
238: 0 * 200 + 0 * 100 + 0 * 50 + 5 * 20 + 2 * 10 + 76 * 5
239: 0 * 200 + 0 * 100 + 0 * 50 + 5 * 20 + 3 * 10 + 74 * 5
240: 0 * 200 + 0 * 100 + 0 * 50 + 5 * 20 + 4 * 10 + 72 * 5
241: 0 * 200 + 0 * 100 + 0 * 50 + 5 * 20 + 5 * 10 + 70 * 5
242: 0 * 200 + 0 * 100 + 0 * 50 + 5 * 20 + 6 * 10 + 68 * 5
243: 0 * 200 + 0 * 100 + 0 * 50 + 5 * 20 + 7 * 10 + 66 * 5
244: 0 * 200 + 0 * 100 + 0 * 50 + 5 * 20 + 8 * 10 + 64 * 5
245: 0 * 200 + 0 * 100 + 0 * 50 + 5 * 20 + 9 * 10 + 62 * 5
246: 0 * 200 + 0 * 100 + 0 * 50 + 5 * 20 + 10 * 10 + 60 * 5
247: 0 * 200 + 0 * 100 + 0 * 50 + 5 * 20 + 11 * 10 + 58 * 5
248: 0 * 200 + 0 * 100 + 0 * 50 + 5 * 20 + 12 * 10 + 56 * 5
249: 0 * 200 + 0 * 100 + 0 * 50 + 5 * 20 + 13 * 10 + 54 * 5
250: 0 * 200 + 0 * 100 + 0 * 50 + 5 * 20 + 14 * 10 + 52 * 5
251: 0 * 200 + 0 * 100 + 0 * 50 + 5 * 20 + 15 * 10 + 50 * 5
252: 0 * 200 + 0 * 100 + 0 * 50 + 5 * 20 + 16 * 10 + 48 * 5
253: 0 * 200 + 0 * 100 + 0 * 50 + 5 * 20 + 17 * 10 + 46 * 5
254: 0 * 200 + 0 * 100 + 0 * 50 + 5 * 20 + 18 * 10 + 44 * 5
255: 0 * 200 + 0 * 100 + 0 * 50 + 5 * 20 + 19 * 10 + 42 * 5
256: 0 * 200 + 0 * 100 + 0 * 50 + 5 * 20 + 20 * 10 + 40 * 5
257: 0 * 200 + 0 * 100 + 0 * 50 + 5 * 20 + 21 * 10 + 38 * 5
258: 0 * 200 + 0 * 100 + 0 * 50 + 5 * 20 + 22 * 10 + 36 * 5
259: 0 * 200 + 0 * 100 + 0 * 50 + 5 * 20 + 23 * 10 + 34 * 5
260: 0 * 200 + 0 * 100 + 0 * 50 + 5 * 20 + 24 * 10 + 32 * 5
261: 0 * 200 + 0 * 100 + 0 * 50 + 5 * 20 + 25 * 10 + 30 * 5
262: 0 * 200 + 0 * 100 + 0 * 50 + 5 * 20 + 26 * 10 + 28 * 5
263: 0 * 200 + 0 * 100 + 0 * 50 + 5 * 20 + 27 * 10 + 26 * 5
264: 0 * 200 + 0 * 100 + 0 * 50 + 5 * 20 + 28 * 10 + 24 * 5
265: 0 * 200 + 0 * 100 + 0 * 50 + 5 * 20 + 29 * 10 + 22 * 5
266: 0 * 200 + 0 * 100 + 0 * 50 + 5 * 20 + 30 * 10 + 20 * 5
267: 0 * 200 + 0 * 100 + 0 * 50 + 5 * 20 + 31 * 10 + 18 * 5
268: 0 * 200 + 0 * 100 + 0 * 50 + 5 * 20 + 32 * 10 + 16 * 5
269: 0 * 200 + 0 * 100 + 0 * 50 + 5 * 20 + 33 * 10 + 14 * 5
270: 0 * 200 + 0 * 100 + 0 * 50 + 5 * 20 + 34 * 10 + 12 * 5
271: 0 * 200 + 0 * 100 + 0 * 50 + 5 * 20 + 35 * 10 + 10 * 5
272: 0 * 200 + 0 * 100 + 0 * 50 + 5 * 20 + 36 * 10 + 8 * 5
273: 0 * 200 + 0 * 100 + 0 * 50 + 5 * 20 + 37 * 10 + 6 * 5
274: 0 * 200 + 0 * 100 + 0 * 50 + 5 * 20 + 38 * 10 + 4 * 5
275: 0 * 200 + 0 * 100 + 0 * 50 + 5 * 20 + 39 * 10 + 2 * 5
276: 0 * 200 + 0 * 100 + 0 * 50 + 5 * 20 + 40 * 10 + 0 * 5
277: 0 * 200 + 0 * 100 + 0 * 50 + 6 * 20 + 0 * 10 + 76 * 5
278: 0 * 200 + 0 * 100 + 0 * 50 + 6 * 20 + 1 * 10 + 74 * 5
279: 0 * 200 + 0 * 100 + 0 * 50 + 6 * 20 + 2 * 10 + 72 * 5
280: 0 * 200 + 0 * 100 + 0 * 50 + 6 * 20 + 3 * 10 + 70 * 5
281: 0 * 200 + 0 * 100 + 0 * 50 + 6 * 20 + 4 * 10 + 68 * 5
282: 0 * 200 + 0 * 100 + 0 * 50 + 6 * 20 + 5 * 10 + 66 * 5
283: 0 * 200 + 0 * 100 + 0 * 50 + 6 * 20 + 6 * 10 + 64 * 5
284: 0 * 200 + 0 * 100 + 0 * 50 + 6 * 20 + 7 * 10 + 62 * 5
285: 0 * 200 + 0 * 100 + 0 * 50 + 6 * 20 + 8 * 10 + 60 * 5
286: 0 * 200 + 0 * 100 + 0 * 50 + 6 * 20 + 9 * 10 + 58 * 5
287: 0 * 200 + 0 * 100 + 0 * 50 + 6 * 20 + 10 * 10 + 56 * 5
288: 0 * 200 + 0 * 100 + 0 * 50 + 6 * 20 + 11 * 10 + 54 * 5
289: 0 * 200 + 0 * 100 + 0 * 50 + 6 * 20 + 12 * 10 + 52 * 5
290: 0 * 200 + 0 * 100 + 0 * 50 + 6 * 20 + 13 * 10 + 50 * 5
291: 0 * 200 + 0 * 100 + 0 * 50 + 6 * 20 + 14 * 10 + 48 * 5
292: 0 * 200 + 0 * 100 + 0 * 50 + 6 * 20 + 15 * 10 + 46 * 5
293: 0 * 200 + 0 * 100 + 0 * 50 + 6 * 20 + 16 * 10 + 44 * 5
294: 0 * 200 + 0 * 100 + 0 * 50 + 6 * 20 + 17 * 10 + 42 * 5
295: 0 * 200 + 0 * 100 + 0 * 50 + 6 * 20 + 18 * 10 + 40 * 5
296: 0 * 200 + 0 * 100 + 0 * 50 + 6 * 20 + 19 * 10 + 38 * 5
297: 0 * 200 + 0 * 100 + 0 * 50 + 6 * 20 + 20 * 10 + 36 * 5
298: 0 * 200 + 0 * 100 + 0 * 50 + 6 * 20 + 21 * 10 + 34 * 5
299: 0 * 200 + 0 * 100 + 0 * 50 + 6 * 20 + 22 * 10 + 32 * 5
300: 0 * 200 + 0 * 100 + 0 * 50 + 6 * 20 + 23 * 10 + 30 * 5
301: 0 * 200 + 0 * 100 + 0 * 50 + 6 * 20 + 24 * 10 + 28 * 5
302: 0 * 200 + 0 * 100 + 0 * 50 + 6 * 20 + 25 * 10 + 26 * 5
303: 0 * 200 + 0 * 100 + 0 * 50 + 6 * 20 + 26 * 10 + 24 * 5
304: 0 * 200 + 0 * 100 + 0 * 50 + 6 * 20 + 27 * 10 + 22 * 5
305: 0 * 200 + 0 * 100 + 0 * 50 + 6 * 20 + 28 * 10 + 20 * 5
306: 0 * 200 + 0 * 100 + 0 * 50 + 6 * 20 + 29 * 10 + 18 * 5
307: 0 * 200 + 0 * 100 + 0 * 50 + 6 * 20 + 30 * 10 + 16 * 5
308: 0 * 200 + 0 * 100 + 0 * 50 + 6 * 20 + 31 * 10 + 14 * 5
309: 0 * 200 + 0 * 100 + 0 * 50 + 6 * 20 + 32 * 10 + 12 * 5
310: 0 * 200 + 0 * 100 + 0 * 50 + 6 * 20 + 33 * 10 + 10 * 5
311: 0 * 200 + 0 * 100 + 0 * 50 + 6 * 20 + 34 * 10 + 8 * 5
312: 0 * 200 + 0 * 100 + 0 * 50 + 6 * 20 + 35 * 10 + 6 * 5
313: 0 * 200 + 0 * 100 + 0 * 50 + 6 * 20 + 36 * 10 + 4 * 5
314: 0 * 200 + 0 * 100 + 0 * 50 + 6 * 20 + 37 * 10 + 2 * 5
315: 0 * 200 + 0 * 100 + 0 * 50 + 6 * 20 + 38 * 10 + 0 * 5
316: 0 * 200 + 0 * 100 + 0 * 50 + 7 * 20 + 0 * 10 + 72 * 5
317: 0 * 200 + 0 * 100 + 0 * 50 + 7 * 20 + 1 * 10 + 70 * 5
318: 0 * 200 + 0 * 100 + 0 * 50 + 7 * 20 + 2 * 10 + 68 * 5
319: 0 * 200 + 0 * 100 + 0 * 50 + 7 * 20 + 3 * 10 + 66 * 5
320: 0 * 200 + 0 * 100 + 0 * 50 + 7 * 20 + 4 * 10 + 64 * 5
321: 0 * 200 + 0 * 100 + 0 * 50 + 7 * 20 + 5 * 10 + 62 * 5
322: 0 * 200 + 0 * 100 + 0 * 50 + 7 * 20 + 6 * 10 + 60 * 5
323: 0 * 200 + 0 * 100 + 0 * 50 + 7 * 20 + 7 * 10 + 58 * 5
324: 0 * 200 + 0 * 100 + 0 * 50 + 7 * 20 + 8 * 10 + 56 * 5
325: 0 * 200 + 0 * 100 + 0 * 50 + 7 * 20 + 9 * 10 + 54 * 5
326: 0 * 200 + 0 * 100 + 0 * 50 + 7 * 20 + 10 * 10 + 52 * 5
327: 0 * 200 + 0 * 100 + 0 * 50 + 7 * 20 + 11 * 10 + 50 * 5
328: 0 * 200 + 0 * 100 + 0 * 50 + 7 * 20 + 12 * 10 + 48 * 5
329: 0 * 200 + 0 * 100 + 0 * 50 + 7 * 20 + 13 * 10 + 46 * 5
330: 0 * 200 + 0 * 100 + 0 * 50 + 7 * 20 + 14 * 10 + 44 * 5
331: 0 * 200 + 0 * 100 + 0 * 50 + 7 * 20 + 15 * 10 + 42 * 5
332: 0 * 200 + 0 * 100 + 0 * 50 + 7 * 20 + 16 * 10 + 40 * 5
333: 0 * 200 + 0 * 100 + 0 * 50 + 7 * 20 + 17 * 10 + 38 * 5
334: 0 * 200 + 0 * 100 + 0 * 50 + 7 * 20 + 18 * 10 + 36 * 5
335: 0 * 200 + 0 * 100 + 0 * 50 + 7 * 20 + 19 * 10 + 34 * 5
336: 0 * 200 + 0 * 100 + 0 * 50 + 7 * 20 + 20 * 10 + 32 * 5
337: 0 * 200 + 0 * 100 + 0 * 50 + 7 * 20 + 21 * 10 + 30 * 5
338: 0 * 200 + 0 * 100 + 0 * 50 + 7 * 20 + 22 * 10 + 28 * 5
339: 0 * 200 + 0 * 100 + 0 * 50 + 7 * 20 + 23 * 10 + 26 * 5
340: 0 * 200 + 0 * 100 + 0 * 50 + 7 * 20 + 24 * 10 + 24 * 5
341: 0 * 200 + 0 * 100 + 0 * 50 + 7 * 20 + 25 * 10 + 22 * 5
342: 0 * 200 + 0 * 100 + 0 * 50 + 7 * 20 + 26 * 10 + 20 * 5
343: 0 * 200 + 0 * 100 + 0 * 50 + 7 * 20 + 27 * 10 + 18 * 5
344: 0 * 200 + 0 * 100 + 0 * 50 + 7 * 20 + 28 * 10 + 16 * 5
345: 0 * 200 + 0 * 100 + 0 * 50 + 7 * 20 + 29 * 10 + 14 * 5
346: 0 * 200 + 0 * 100 + 0 * 50 + 7 * 20 + 30 * 10 + 12 * 5
347: 0 * 200 + 0 * 100 + 0 * 50 + 7 * 20 + 31 * 10 + 10 * 5
348: 0 * 200 + 0 * 100 + 0 * 50 + 7 * 20 + 32 * 10 + 8 * 5
349: 0 * 200 + 0 * 100 + 0 * 50 + 7 * 20 + 33 * 10 + 6 * 5
350: 0 * 200 + 0 * 100 + 0 * 50 + 7 * 20 + 34 * 10 + 4 * 5
351: 0 * 200 + 0 * 100 + 0 * 50 + 7 * 20 + 35 * 10 + 2 * 5
352: 0 * 200 + 0 * 100 + 0 * 50 + 7 * 20 + 36 * 10 + 0 * 5
353: 0 * 200 + 0 * 100 + 0 * 50 + 8 * 20 + 0 * 10 + 68 * 5
354: 0 * 200 + 0 * 100 + 0 * 50 + 8 * 20 + 1 * 10 + 66 * 5
355: 0 * 200 + 0 * 100 + 0 * 50 + 8 * 20 + 2 * 10 + 64 * 5
356: 0 * 200 + 0 * 100 + 0 * 50 + 8 * 20 + 3 * 10 + 62 * 5
357: 0 * 200 + 0 * 100 + 0 * 50 + 8 * 20 + 4 * 10 + 60 * 5
358: 0 * 200 + 0 * 100 + 0 * 50 + 8 * 20 + 5 * 10 + 58 * 5
359: 0 * 200 + 0 * 100 + 0 * 50 + 8 * 20 + 6 * 10 + 56 * 5
360: 0 * 200 + 0 * 100 + 0 * 50 + 8 * 20 + 7 * 10 + 54 * 5
361: 0 * 200 + 0 * 100 + 0 * 50 + 8 * 20 + 8 * 10 + 52 * 5
362: 0 * 200 + 0 * 100 + 0 * 50 + 8 * 20 + 9 * 10 + 50 * 5
363: 0 * 200 + 0 * 100 + 0 * 50 + 8 * 20 + 10 * 10 + 48 * 5
364: 0 * 200 + 0 * 100 + 0 * 50 + 8 * 20 + 11 * 10 + 46 * 5
365: 0 * 200 + 0 * 100 + 0 * 50 + 8 * 20 + 12 * 10 + 44 * 5
366: 0 * 200 + 0 * 100 + 0 * 50 + 8 * 20 + 13 * 10 + 42 * 5
367: 0 * 200 + 0 * 100 + 0 * 50 + 8 * 20 + 14 * 10 + 40 * 5
368: 0 * 200 + 0 * 100 + 0 * 50 + 8 * 20 + 15 * 10 + 38 * 5
369: 0 * 200 + 0 * 100 + 0 * 50 + 8 * 20 + 16 * 10 + 36 * 5
370: 0 * 200 + 0 * 100 + 0 * 50 + 8 * 20 + 17 * 10 + 34 * 5
371: 0 * 200 + 0 * 100 + 0 * 50 + 8 * 20 + 18 * 10 + 32 * 5
372: 0 * 200 + 0 * 100 + 0 * 50 + 8 * 20 + 19 * 10 + 30 * 5
373: 0 * 200 + 0 * 100 + 0 * 50 + 8 * 20 + 20 * 10 + 28 * 5
374: 0 * 200 + 0 * 100 + 0 * 50 + 8 * 20 + 21 * 10 + 26 * 5
375: 0 * 200 + 0 * 100 + 0 * 50 + 8 * 20 + 22 * 10 + 24 * 5
376: 0 * 200 + 0 * 100 + 0 * 50 + 8 * 20 + 23 * 10 + 22 * 5
377: 0 * 200 + 0 * 100 + 0 * 50 + 8 * 20 + 24 * 10 + 20 * 5
378: 0 * 200 + 0 * 100 + 0 * 50 + 8 * 20 + 25 * 10 + 18 * 5
379: 0 * 200 + 0 * 100 + 0 * 50 + 8 * 20 + 26 * 10 + 16 * 5
380: 0 * 200 + 0 * 100 + 0 * 50 + 8 * 20 + 27 * 10 + 14 * 5
381: 0 * 200 + 0 * 100 + 0 * 50 + 8 * 20 + 28 * 10 + 12 * 5
382: 0 * 200 + 0 * 100 + 0 * 50 + 8 * 20 + 29 * 10 + 10 * 5
383: 0 * 200 + 0 * 100 + 0 * 50 + 8 * 20 + 30 * 10 + 8 * 5
384: 0 * 200 + 0 * 100 + 0 * 50 + 8 * 20 + 31 * 10 + 6 * 5
385: 0 * 200 + 0 * 100 + 0 * 50 + 8 * 20 + 32 * 10 + 4 * 5
386: 0 * 200 + 0 * 100 + 0 * 50 + 8 * 20 + 33 * 10 + 2 * 5
387: 0 * 200 + 0 * 100 + 0 * 50 + 8 * 20 + 34 * 10 + 0 * 5
388: 0 * 200 + 0 * 100 + 0 * 50 + 9 * 20 + 0 * 10 + 64 * 5
389: 0 * 200 + 0 * 100 + 0 * 50 + 9 * 20 + 1 * 10 + 62 * 5
390: 0 * 200 + 0 * 100 + 0 * 50 + 9 * 20 + 2 * 10 + 60 * 5
391: 0 * 200 + 0 * 100 + 0 * 50 + 9 * 20 + 3 * 10 + 58 * 5
392: 0 * 200 + 0 * 100 + 0 * 50 + 9 * 20 + 4 * 10 + 56 * 5
393: 0 * 200 + 0 * 100 + 0 * 50 + 9 * 20 + 5 * 10 + 54 * 5
394: 0 * 200 + 0 * 100 + 0 * 50 + 9 * 20 + 6 * 10 + 52 * 5
395: 0 * 200 + 0 * 100 + 0 * 50 + 9 * 20 + 7 * 10 + 50 * 5
396: 0 * 200 + 0 * 100 + 0 * 50 + 9 * 20 + 8 * 10 + 48 * 5
397: 0 * 200 + 0 * 100 + 0 * 50 + 9 * 20 + 9 * 10 + 46 * 5
398: 0 * 200 + 0 * 100 + 0 * 50 + 9 * 20 + 10 * 10 + 44 * 5
399: 0 * 200 + 0 * 100 + 0 * 50 + 9 * 20 + 11 * 10 + 42 * 5
400: 0 * 200 + 0 * 100 + 0 * 50 + 9 * 20 + 12 * 10 + 40 * 5
401: 0 * 200 + 0 * 100 + 0 * 50 + 9 * 20 + 13 * 10 + 38 * 5
402: 0 * 200 + 0 * 100 + 0 * 50 + 9 * 20 + 14 * 10 + 36 * 5
403: 0 * 200 + 0 * 100 + 0 * 50 + 9 * 20 + 15 * 10 + 34 * 5
404: 0 * 200 + 0 * 100 + 0 * 50 + 9 * 20 + 16 * 10 + 32 * 5
405: 0 * 200 + 0 * 100 + 0 * 50 + 9 * 20 + 17 * 10 + 30 * 5
406: 0 * 200 + 0 * 100 + 0 * 50 + 9 * 20 + 18 * 10 + 28 * 5
407: 0 * 200 + 0 * 100 + 0 * 50 + 9 * 20 + 19 * 10 + 26 * 5
408: 0 * 200 + 0 * 100 + 0 * 50 + 9 * 20 + 20 * 10 + 24 * 5
409: 0 * 200 + 0 * 100 + 0 * 50 + 9 * 20 + 21 * 10 + 22 * 5
410: 0 * 200 + 0 * 100 + 0 * 50 + 9 * 20 + 22 * 10 + 20 * 5
411: 0 * 200 + 0 * 100 + 0 * 50 + 9 * 20 + 23 * 10 + 18 * 5
412: 0 * 200 + 0 * 100 + 0 * 50 + 9 * 20 + 24 * 10 + 16 * 5
413: 0 * 200 + 0 * 100 + 0 * 50 + 9 * 20 + 25 * 10 + 14 * 5
414: 0 * 200 + 0 * 100 + 0 * 50 + 9 * 20 + 26 * 10 + 12 * 5
415: 0 * 200 + 0 * 100 + 0 * 50 + 9 * 20 + 27 * 10 + 10 * 5
416: 0 * 200 + 0 * 100 + 0 * 50 + 9 * 20 + 28 * 10 + 8 * 5
417: 0 * 200 + 0 * 100 + 0 * 50 + 9 * 20 + 29 * 10 + 6 * 5
418: 0 * 200 + 0 * 100 + 0 * 50 + 9 * 20 + 30 * 10 + 4 * 5
419: 0 * 200 + 0 * 100 + 0 * 50 + 9 * 20 + 31 * 10 + 2 * 5
420: 0 * 200 + 0 * 100 + 0 * 50 + 9 * 20 + 32 * 10 + 0 * 5
421: 0 * 200 + 0 * 100 + 0 * 50 + 10 * 20 + 0 * 10 + 60 * 5
422: 0 * 200 + 0 * 100 + 0 * 50 + 10 * 20 + 1 * 10 + 58 * 5
423: 0 * 200 + 0 * 100 + 0 * 50 + 10 * 20 + 2 * 10 + 56 * 5
424: 0 * 200 + 0 * 100 + 0 * 50 + 10 * 20 + 3 * 10 + 54 * 5
425: 0 * 200 + 0 * 100 + 0 * 50 + 10 * 20 + 4 * 10 + 52 * 5
426: 0 * 200 + 0 * 100 + 0 * 50 + 10 * 20 + 5 * 10 + 50 * 5
427: 0 * 200 + 0 * 100 + 0 * 50 + 10 * 20 + 6 * 10 + 48 * 5
428: 0 * 200 + 0 * 100 + 0 * 50 + 10 * 20 + 7 * 10 + 46 * 5
429: 0 * 200 + 0 * 100 + 0 * 50 + 10 * 20 + 8 * 10 + 44 * 5
430: 0 * 200 + 0 * 100 + 0 * 50 + 10 * 20 + 9 * 10 + 42 * 5
431: 0 * 200 + 0 * 100 + 0 * 50 + 10 * 20 + 10 * 10 + 40 * 5
432: 0 * 200 + 0 * 100 + 0 * 50 + 10 * 20 + 11 * 10 + 38 * 5
433: 0 * 200 + 0 * 100 + 0 * 50 + 10 * 20 + 12 * 10 + 36 * 5
434: 0 * 200 + 0 * 100 + 0 * 50 + 10 * 20 + 13 * 10 + 34 * 5
435: 0 * 200 + 0 * 100 + 0 * 50 + 10 * 20 + 14 * 10 + 32 * 5
436: 0 * 200 + 0 * 100 + 0 * 50 + 10 * 20 + 15 * 10 + 30 * 5
437: 0 * 200 + 0 * 100 + 0 * 50 + 10 * 20 + 16 * 10 + 28 * 5
438: 0 * 200 + 0 * 100 + 0 * 50 + 10 * 20 + 17 * 10 + 26 * 5
439: 0 * 200 + 0 * 100 + 0 * 50 + 10 * 20 + 18 * 10 + 24 * 5
440: 0 * 200 + 0 * 100 + 0 * 50 + 10 * 20 + 19 * 10 + 22 * 5
441: 0 * 200 + 0 * 100 + 0 * 50 + 10 * 20 + 20 * 10 + 20 * 5
442: 0 * 200 + 0 * 100 + 0 * 50 + 10 * 20 + 21 * 10 + 18 * 5
443: 0 * 200 + 0 * 100 + 0 * 50 + 10 * 20 + 22 * 10 + 16 * 5
444: 0 * 200 + 0 * 100 + 0 * 50 + 10 * 20 + 23 * 10 + 14 * 5
445: 0 * 200 + 0 * 100 + 0 * 50 + 10 * 20 + 24 * 10 + 12 * 5
446: 0 * 200 + 0 * 100 + 0 * 50 + 10 * 20 + 25 * 10 + 10 * 5
447: 0 * 200 + 0 * 100 + 0 * 50 + 10 * 20 + 26 * 10 + 8 * 5
448: 0 * 200 + 0 * 100 + 0 * 50 + 10 * 20 + 27 * 10 + 6 * 5
449: 0 * 200 + 0 * 100 + 0 * 50 + 10 * 20 + 28 * 10 + 4 * 5
450: 0 * 200 + 0 * 100 + 0 * 50 + 10 * 20 + 29 * 10 + 2 * 5
451: 0 * 200 + 0 * 100 + 0 * 50 + 10 * 20 + 30 * 10 + 0 * 5
452: 0 * 200 + 0 * 100 + 0 * 50 + 11 * 20 + 0 * 10 + 56 * 5
453: 0 * 200 + 0 * 100 + 0 * 50 + 11 * 20 + 1 * 10 + 54 * 5
454: 0 * 200 + 0 * 100 + 0 * 50 + 11 * 20 + 2 * 10 + 52 * 5
455: 0 * 200 + 0 * 100 + 0 * 50 + 11 * 20 + 3 * 10 + 50 * 5
456: 0 * 200 + 0 * 100 + 0 * 50 + 11 * 20 + 4 * 10 + 48 * 5
457: 0 * 200 + 0 * 100 + 0 * 50 + 11 * 20 + 5 * 10 + 46 * 5
458: 0 * 200 + 0 * 100 + 0 * 50 + 11 * 20 + 6 * 10 + 44 * 5
459: 0 * 200 + 0 * 100 + 0 * 50 + 11 * 20 + 7 * 10 + 42 * 5
460: 0 * 200 + 0 * 100 + 0 * 50 + 11 * 20 + 8 * 10 + 40 * 5
461: 0 * 200 + 0 * 100 + 0 * 50 + 11 * 20 + 9 * 10 + 38 * 5
462: 0 * 200 + 0 * 100 + 0 * 50 + 11 * 20 + 10 * 10 + 36 * 5
463: 0 * 200 + 0 * 100 + 0 * 50 + 11 * 20 + 11 * 10 + 34 * 5
464: 0 * 200 + 0 * 100 + 0 * 50 + 11 * 20 + 12 * 10 + 32 * 5
465: 0 * 200 + 0 * 100 + 0 * 50 + 11 * 20 + 13 * 10 + 30 * 5
466: 0 * 200 + 0 * 100 + 0 * 50 + 11 * 20 + 14 * 10 + 28 * 5
467: 0 * 200 + 0 * 100 + 0 * 50 + 11 * 20 + 15 * 10 + 26 * 5
468: 0 * 200 + 0 * 100 + 0 * 50 + 11 * 20 + 16 * 10 + 24 * 5
469: 0 * 200 + 0 * 100 + 0 * 50 + 11 * 20 + 17 * 10 + 22 * 5
470: 0 * 200 + 0 * 100 + 0 * 50 + 11 * 20 + 18 * 10 + 20 * 5
471: 0 * 200 + 0 * 100 + 0 * 50 + 11 * 20 + 19 * 10 + 18 * 5
472: 0 * 200 + 0 * 100 + 0 * 50 + 11 * 20 + 20 * 10 + 16 * 5
473: 0 * 200 + 0 * 100 + 0 * 50 + 11 * 20 + 21 * 10 + 14 * 5
474: 0 * 200 + 0 * 100 + 0 * 50 + 11 * 20 + 22 * 10 + 12 * 5
475: 0 * 200 + 0 * 100 + 0 * 50 + 11 * 20 + 23 * 10 + 10 * 5
476: 0 * 200 + 0 * 100 + 0 * 50 + 11 * 20 + 24 * 10 + 8 * 5
477: 0 * 200 + 0 * 100 + 0 * 50 + 11 * 20 + 25 * 10 + 6 * 5
478: 0 * 200 + 0 * 100 + 0 * 50 + 11 * 20 + 26 * 10 + 4 * 5
479: 0 * 200 + 0 * 100 + 0 * 50 + 11 * 20 + 27 * 10 + 2 * 5
480: 0 * 200 + 0 * 100 + 0 * 50 + 11 * 20 + 28 * 10 + 0 * 5
481: 0 * 200 + 0 * 100 + 0 * 50 + 12 * 20 + 0 * 10 + 52 * 5
482: 0 * 200 + 0 * 100 + 0 * 50 + 12 * 20 + 1 * 10 + 50 * 5
483: 0 * 200 + 0 * 100 + 0 * 50 + 12 * 20 + 2 * 10 + 48 * 5
484: 0 * 200 + 0 * 100 + 0 * 50 + 12 * 20 + 3 * 10 + 46 * 5
485: 0 * 200 + 0 * 100 + 0 * 50 + 12 * 20 + 4 * 10 + 44 * 5
486: 0 * 200 + 0 * 100 + 0 * 50 + 12 * 20 + 5 * 10 + 42 * 5
487: 0 * 200 + 0 * 100 + 0 * 50 + 12 * 20 + 6 * 10 + 40 * 5
488: 0 * 200 + 0 * 100 + 0 * 50 + 12 * 20 + 7 * 10 + 38 * 5
489: 0 * 200 + 0 * 100 + 0 * 50 + 12 * 20 + 8 * 10 + 36 * 5
490: 0 * 200 + 0 * 100 + 0 * 50 + 12 * 20 + 9 * 10 + 34 * 5
491: 0 * 200 + 0 * 100 + 0 * 50 + 12 * 20 + 10 * 10 + 32 * 5
492: 0 * 200 + 0 * 100 + 0 * 50 + 12 * 20 + 11 * 10 + 30 * 5
493: 0 * 200 + 0 * 100 + 0 * 50 + 12 * 20 + 12 * 10 + 28 * 5
494: 0 * 200 + 0 * 100 + 0 * 50 + 12 * 20 + 13 * 10 + 26 * 5
495: 0 * 200 + 0 * 100 + 0 * 50 + 12 * 20 + 14 * 10 + 24 * 5
496: 0 * 200 + 0 * 100 + 0 * 50 + 12 * 20 + 15 * 10 + 22 * 5
497: 0 * 200 + 0 * 100 + 0 * 50 + 12 * 20 + 16 * 10 + 20 * 5
498: 0 * 200 + 0 * 100 + 0 * 50 + 12 * 20 + 17 * 10 + 18 * 5
499: 0 * 200 + 0 * 100 + 0 * 50 + 12 * 20 + 18 * 10 + 16 * 5
500: 0 * 200 + 0 * 100 + 0 * 50 + 12 * 20 + 19 * 10 + 14 * 5
501: 0 * 200 + 0 * 100 + 0 * 50 + 12 * 20 + 20 * 10 + 12 * 5
502: 0 * 200 + 0 * 100 + 0 * 50 + 12 * 20 + 21 * 10 + 10 * 5
503: 0 * 200 + 0 * 100 + 0 * 50 + 12 * 20 + 22 * 10 + 8 * 5
504: 0 * 200 + 0 * 100 + 0 * 50 + 12 * 20 + 23 * 10 + 6 * 5
505: 0 * 200 + 0 * 100 + 0 * 50 + 12 * 20 + 24 * 10 + 4 * 5
506: 0 * 200 + 0 * 100 + 0 * 50 + 12 * 20 + 25 * 10 + 2 * 5
507: 0 * 200 + 0 * 100 + 0 * 50 + 12 * 20 + 26 * 10 + 0 * 5
508: 0 * 200 + 0 * 100 + 0 * 50 + 13 * 20 + 0 * 10 + 48 * 5
509: 0 * 200 + 0 * 100 + 0 * 50 + 13 * 20 + 1 * 10 + 46 * 5
510: 0 * 200 + 0 * 100 + 0 * 50 + 13 * 20 + 2 * 10 + 44 * 5
511: 0 * 200 + 0 * 100 + 0 * 50 + 13 * 20 + 3 * 10 + 42 * 5
512: 0 * 200 + 0 * 100 + 0 * 50 + 13 * 20 + 4 * 10 + 40 * 5
513: 0 * 200 + 0 * 100 + 0 * 50 + 13 * 20 + 5 * 10 + 38 * 5
514: 0 * 200 + 0 * 100 + 0 * 50 + 13 * 20 + 6 * 10 + 36 * 5
515: 0 * 200 + 0 * 100 + 0 * 50 + 13 * 20 + 7 * 10 + 34 * 5
516: 0 * 200 + 0 * 100 + 0 * 50 + 13 * 20 + 8 * 10 + 32 * 5
517: 0 * 200 + 0 * 100 + 0 * 50 + 13 * 20 + 9 * 10 + 30 * 5
518: 0 * 200 + 0 * 100 + 0 * 50 + 13 * 20 + 10 * 10 + 28 * 5
519: 0 * 200 + 0 * 100 + 0 * 50 + 13 * 20 + 11 * 10 + 26 * 5
520: 0 * 200 + 0 * 100 + 0 * 50 + 13 * 20 + 12 * 10 + 24 * 5
521: 0 * 200 + 0 * 100 + 0 * 50 + 13 * 20 + 13 * 10 + 22 * 5
522: 0 * 200 + 0 * 100 + 0 * 50 + 13 * 20 + 14 * 10 + 20 * 5
523: 0 * 200 + 0 * 100 + 0 * 50 + 13 * 20 + 15 * 10 + 18 * 5
524: 0 * 200 + 0 * 100 + 0 * 50 + 13 * 20 + 16 * 10 + 16 * 5
525: 0 * 200 + 0 * 100 + 0 * 50 + 13 * 20 + 17 * 10 + 14 * 5
526: 0 * 200 + 0 * 100 + 0 * 50 + 13 * 20 + 18 * 10 + 12 * 5
527: 0 * 200 + 0 * 100 + 0 * 50 + 13 * 20 + 19 * 10 + 10 * 5
528: 0 * 200 + 0 * 100 + 0 * 50 + 13 * 20 + 20 * 10 + 8 * 5
529: 0 * 200 + 0 * 100 + 0 * 50 + 13 * 20 + 21 * 10 + 6 * 5
530: 0 * 200 + 0 * 100 + 0 * 50 + 13 * 20 + 22 * 10 + 4 * 5
531: 0 * 200 + 0 * 100 + 0 * 50 + 13 * 20 + 23 * 10 + 2 * 5
532: 0 * 200 + 0 * 100 + 0 * 50 + 13 * 20 + 24 * 10 + 0 * 5
533: 0 * 200 + 0 * 100 + 0 * 50 + 14 * 20 + 0 * 10 + 44 * 5
534: 0 * 200 + 0 * 100 + 0 * 50 + 14 * 20 + 1 * 10 + 42 * 5
535: 0 * 200 + 0 * 100 + 0 * 50 + 14 * 20 + 2 * 10 + 40 * 5
536: 0 * 200 + 0 * 100 + 0 * 50 + 14 * 20 + 3 * 10 + 38 * 5
537: 0 * 200 + 0 * 100 + 0 * 50 + 14 * 20 + 4 * 10 + 36 * 5
538: 0 * 200 + 0 * 100 + 0 * 50 + 14 * 20 + 5 * 10 + 34 * 5
539: 0 * 200 + 0 * 100 + 0 * 50 + 14 * 20 + 6 * 10 + 32 * 5
540: 0 * 200 + 0 * 100 + 0 * 50 + 14 * 20 + 7 * 10 + 30 * 5
541: 0 * 200 + 0 * 100 + 0 * 50 + 14 * 20 + 8 * 10 + 28 * 5
542: 0 * 200 + 0 * 100 + 0 * 50 + 14 * 20 + 9 * 10 + 26 * 5
543: 0 * 200 + 0 * 100 + 0 * 50 + 14 * 20 + 10 * 10 + 24 * 5
544: 0 * 200 + 0 * 100 + 0 * 50 + 14 * 20 + 11 * 10 + 22 * 5
545: 0 * 200 + 0 * 100 + 0 * 50 + 14 * 20 + 12 * 10 + 20 * 5
546: 0 * 200 + 0 * 100 + 0 * 50 + 14 * 20 + 13 * 10 + 18 * 5
547: 0 * 200 + 0 * 100 + 0 * 50 + 14 * 20 + 14 * 10 + 16 * 5
548: 0 * 200 + 0 * 100 + 0 * 50 + 14 * 20 + 15 * 10 + 14 * 5
549: 0 * 200 + 0 * 100 + 0 * 50 + 14 * 20 + 16 * 10 + 12 * 5
550: 0 * 200 + 0 * 100 + 0 * 50 + 14 * 20 + 17 * 10 + 10 * 5
551: 0 * 200 + 0 * 100 + 0 * 50 + 14 * 20 + 18 * 10 + 8 * 5
552: 0 * 200 + 0 * 100 + 0 * 50 + 14 * 20 + 19 * 10 + 6 * 5
553: 0 * 200 + 0 * 100 + 0 * 50 + 14 * 20 + 20 * 10 + 4 * 5
554: 0 * 200 + 0 * 100 + 0 * 50 + 14 * 20 + 21 * 10 + 2 * 5
555: 0 * 200 + 0 * 100 + 0 * 50 + 14 * 20 + 22 * 10 + 0 * 5
556: 0 * 200 + 0 * 100 + 0 * 50 + 15 * 20 + 0 * 10 + 40 * 5
557: 0 * 200 + 0 * 100 + 0 * 50 + 15 * 20 + 1 * 10 + 38 * 5
558: 0 * 200 + 0 * 100 + 0 * 50 + 15 * 20 + 2 * 10 + 36 * 5
559: 0 * 200 + 0 * 100 + 0 * 50 + 15 * 20 + 3 * 10 + 34 * 5
560: 0 * 200 + 0 * 100 + 0 * 50 + 15 * 20 + 4 * 10 + 32 * 5
561: 0 * 200 + 0 * 100 + 0 * 50 + 15 * 20 + 5 * 10 + 30 * 5
562: 0 * 200 + 0 * 100 + 0 * 50 + 15 * 20 + 6 * 10 + 28 * 5
563: 0 * 200 + 0 * 100 + 0 * 50 + 15 * 20 + 7 * 10 + 26 * 5
564: 0 * 200 + 0 * 100 + 0 * 50 + 15 * 20 + 8 * 10 + 24 * 5
565: 0 * 200 + 0 * 100 + 0 * 50 + 15 * 20 + 9 * 10 + 22 * 5
566: 0 * 200 + 0 * 100 + 0 * 50 + 15 * 20 + 10 * 10 + 20 * 5
567: 0 * 200 + 0 * 100 + 0 * 50 + 15 * 20 + 11 * 10 + 18 * 5
568: 0 * 200 + 0 * 100 + 0 * 50 + 15 * 20 + 12 * 10 + 16 * 5
569: 0 * 200 + 0 * 100 + 0 * 50 + 15 * 20 + 13 * 10 + 14 * 5
570: 0 * 200 + 0 * 100 + 0 * 50 + 15 * 20 + 14 * 10 + 12 * 5
571: 0 * 200 + 0 * 100 + 0 * 50 + 15 * 20 + 15 * 10 + 10 * 5
572: 0 * 200 + 0 * 100 + 0 * 50 + 15 * 20 + 16 * 10 + 8 * 5
573: 0 * 200 + 0 * 100 + 0 * 50 + 15 * 20 + 17 * 10 + 6 * 5
574: 0 * 200 + 0 * 100 + 0 * 50 + 15 * 20 + 18 * 10 + 4 * 5
575: 0 * 200 + 0 * 100 + 0 * 50 + 15 * 20 + 19 * 10 + 2 * 5
576: 0 * 200 + 0 * 100 + 0 * 50 + 15 * 20 + 20 * 10 + 0 * 5
577: 0 * 200 + 0 * 100 + 0 * 50 + 16 * 20 + 0 * 10 + 36 * 5
578: 0 * 200 + 0 * 100 + 0 * 50 + 16 * 20 + 1 * 10 + 34 * 5
579: 0 * 200 + 0 * 100 + 0 * 50 + 16 * 20 + 2 * 10 + 32 * 5
580: 0 * 200 + 0 * 100 + 0 * 50 + 16 * 20 + 3 * 10 + 30 * 5
581: 0 * 200 + 0 * 100 + 0 * 50 + 16 * 20 + 4 * 10 + 28 * 5
582: 0 * 200 + 0 * 100 + 0 * 50 + 16 * 20 + 5 * 10 + 26 * 5
583: 0 * 200 + 0 * 100 + 0 * 50 + 16 * 20 + 6 * 10 + 24 * 5
584: 0 * 200 + 0 * 100 + 0 * 50 + 16 * 20 + 7 * 10 + 22 * 5
585: 0 * 200 + 0 * 100 + 0 * 50 + 16 * 20 + 8 * 10 + 20 * 5
586: 0 * 200 + 0 * 100 + 0 * 50 + 16 * 20 + 9 * 10 + 18 * 5
587: 0 * 200 + 0 * 100 + 0 * 50 + 16 * 20 + 10 * 10 + 16 * 5
588: 0 * 200 + 0 * 100 + 0 * 50 + 16 * 20 + 11 * 10 + 14 * 5
589: 0 * 200 + 0 * 100 + 0 * 50 + 16 * 20 + 12 * 10 + 12 * 5
590: 0 * 200 + 0 * 100 + 0 * 50 + 16 * 20 + 13 * 10 + 10 * 5
591: 0 * 200 + 0 * 100 + 0 * 50 + 16 * 20 + 14 * 10 + 8 * 5
592: 0 * 200 + 0 * 100 + 0 * 50 + 16 * 20 + 15 * 10 + 6 * 5
593: 0 * 200 + 0 * 100 + 0 * 50 + 16 * 20 + 16 * 10 + 4 * 5
594: 0 * 200 + 0 * 100 + 0 * 50 + 16 * 20 + 17 * 10 + 2 * 5
595: 0 * 200 + 0 * 100 + 0 * 50 + 16 * 20 + 18 * 10 + 0 * 5
596: 0 * 200 + 0 * 100 + 0 * 50 + 17 * 20 + 0 * 10 + 32 * 5
597: 0 * 200 + 0 * 100 + 0 * 50 + 17 * 20 + 1 * 10 + 30 * 5
598: 0 * 200 + 0 * 100 + 0 * 50 + 17 * 20 + 2 * 10 + 28 * 5
599: 0 * 200 + 0 * 100 + 0 * 50 + 17 * 20 + 3 * 10 + 26 * 5
600: 0 * 200 + 0 * 100 + 0 * 50 + 17 * 20 + 4 * 10 + 24 * 5
601: 0 * 200 + 0 * 100 + 0 * 50 + 17 * 20 + 5 * 10 + 22 * 5
602: 0 * 200 + 0 * 100 + 0 * 50 + 17 * 20 + 6 * 10 + 20 * 5
603: 0 * 200 + 0 * 100 + 0 * 50 + 17 * 20 + 7 * 10 + 18 * 5
604: 0 * 200 + 0 * 100 + 0 * 50 + 17 * 20 + 8 * 10 + 16 * 5
605: 0 * 200 + 0 * 100 + 0 * 50 + 17 * 20 + 9 * 10 + 14 * 5
606: 0 * 200 + 0 * 100 + 0 * 50 + 17 * 20 + 10 * 10 + 12 * 5
607: 0 * 200 + 0 * 100 + 0 * 50 + 17 * 20 + 11 * 10 + 10 * 5
608: 0 * 200 + 0 * 100 + 0 * 50 + 17 * 20 + 12 * 10 + 8 * 5
609: 0 * 200 + 0 * 100 + 0 * 50 + 17 * 20 + 13 * 10 + 6 * 5
610: 0 * 200 + 0 * 100 + 0 * 50 + 17 * 20 + 14 * 10 + 4 * 5
611: 0 * 200 + 0 * 100 + 0 * 50 + 17 * 20 + 15 * 10 + 2 * 5
612: 0 * 200 + 0 * 100 + 0 * 50 + 17 * 20 + 16 * 10 + 0 * 5
613: 0 * 200 + 0 * 100 + 0 * 50 + 18 * 20 + 0 * 10 + 28 * 5
614: 0 * 200 + 0 * 100 + 0 * 50 + 18 * 20 + 1 * 10 + 26 * 5
615: 0 * 200 + 0 * 100 + 0 * 50 + 18 * 20 + 2 * 10 + 24 * 5
616: 0 * 200 + 0 * 100 + 0 * 50 + 18 * 20 + 3 * 10 + 22 * 5
617: 0 * 200 + 0 * 100 + 0 * 50 + 18 * 20 + 4 * 10 + 20 * 5
618: 0 * 200 + 0 * 100 + 0 * 50 + 18 * 20 + 5 * 10 + 18 * 5
619: 0 * 200 + 0 * 100 + 0 * 50 + 18 * 20 + 6 * 10 + 16 * 5
620: 0 * 200 + 0 * 100 + 0 * 50 + 18 * 20 + 7 * 10 + 14 * 5
621: 0 * 200 + 0 * 100 + 0 * 50 + 18 * 20 + 8 * 10 + 12 * 5
622: 0 * 200 + 0 * 100 + 0 * 50 + 18 * 20 + 9 * 10 + 10 * 5
623: 0 * 200 + 0 * 100 + 0 * 50 + 18 * 20 + 10 * 10 + 8 * 5
624: 0 * 200 + 0 * 100 + 0 * 50 + 18 * 20 + 11 * 10 + 6 * 5
625: 0 * 200 + 0 * 100 + 0 * 50 + 18 * 20 + 12 * 10 + 4 * 5
626: 0 * 200 + 0 * 100 + 0 * 50 + 18 * 20 + 13 * 10 + 2 * 5
627: 0 * 200 + 0 * 100 + 0 * 50 + 18 * 20 + 14 * 10 + 0 * 5
628: 0 * 200 + 0 * 100 + 0 * 50 + 19 * 20 + 0 * 10 + 24 * 5
629: 0 * 200 + 0 * 100 + 0 * 50 + 19 * 20 + 1 * 10 + 22 * 5
630: 0 * 200 + 0 * 100 + 0 * 50 + 19 * 20 + 2 * 10 + 20 * 5
631: 0 * 200 + 0 * 100 + 0 * 50 + 19 * 20 + 3 * 10 + 18 * 5
632: 0 * 200 + 0 * 100 + 0 * 50 + 19 * 20 + 4 * 10 + 16 * 5
633: 0 * 200 + 0 * 100 + 0 * 50 + 19 * 20 + 5 * 10 + 14 * 5
634: 0 * 200 + 0 * 100 + 0 * 50 + 19 * 20 + 6 * 10 + 12 * 5
635: 0 * 200 + 0 * 100 + 0 * 50 + 19 * 20 + 7 * 10 + 10 * 5
636: 0 * 200 + 0 * 100 + 0 * 50 + 19 * 20 + 8 * 10 + 8 * 5
637: 0 * 200 + 0 * 100 + 0 * 50 + 19 * 20 + 9 * 10 + 6 * 5
638: 0 * 200 + 0 * 100 + 0 * 50 + 19 * 20 + 10 * 10 + 4 * 5
639: 0 * 200 + 0 * 100 + 0 * 50 + 19 * 20 + 11 * 10 + 2 * 5
640: 0 * 200 + 0 * 100 + 0 * 50 + 19 * 20 + 12 * 10 + 0 * 5
641: 0 * 200 + 0 * 100 + 0 * 50 + 20 * 20 + 0 * 10 + 20 * 5
642: 0 * 200 + 0 * 100 + 0 * 50 + 20 * 20 + 1 * 10 + 18 * 5
643: 0 * 200 + 0 * 100 + 0 * 50 + 20 * 20 + 2 * 10 + 16 * 5
644: 0 * 200 + 0 * 100 + 0 * 50 + 20 * 20 + 3 * 10 + 14 * 5
645: 0 * 200 + 0 * 100 + 0 * 50 + 20 * 20 + 4 * 10 + 12 * 5
646: 0 * 200 + 0 * 100 + 0 * 50 + 20 * 20 + 5 * 10 + 10 * 5
647: 0 * 200 + 0 * 100 + 0 * 50 + 20 * 20 + 6 * 10 + 8 * 5
648: 0 * 200 + 0 * 100 + 0 * 50 + 20 * 20 + 7 * 10 + 6 * 5
649: 0 * 200 + 0 * 100 + 0 * 50 + 20 * 20 + 8 * 10 + 4 * 5
650: 0 * 200 + 0 * 100 + 0 * 50 + 20 * 20 + 9 * 10 + 2 * 5
651: 0 * 200 + 0 * 100 + 0 * 50 + 20 * 20 + 10 * 10 + 0 * 5
652: 0 * 200 + 0 * 100 + 0 * 50 + 21 * 20 + 0 * 10 + 16 * 5
653: 0 * 200 + 0 * 100 + 0 * 50 + 21 * 20 + 1 * 10 + 14 * 5
654: 0 * 200 + 0 * 100 + 0 * 50 + 21 * 20 + 2 * 10 + 12 * 5
655: 0 * 200 + 0 * 100 + 0 * 50 + 21 * 20 + 3 * 10 + 10 * 5
656: 0 * 200 + 0 * 100 + 0 * 50 + 21 * 20 + 4 * 10 + 8 * 5
657: 0 * 200 + 0 * 100 + 0 * 50 + 21 * 20 + 5 * 10 + 6 * 5
658: 0 * 200 + 0 * 100 + 0 * 50 + 21 * 20 + 6 * 10 + 4 * 5
659: 0 * 200 + 0 * 100 + 0 * 50 + 21 * 20 + 7 * 10 + 2 * 5
660: 0 * 200 + 0 * 100 + 0 * 50 + 21 * 20 + 8 * 10 + 0 * 5
661: 0 * 200 + 0 * 100 + 0 * 50 + 22 * 20 + 0 * 10 + 12 * 5
662: 0 * 200 + 0 * 100 + 0 * 50 + 22 * 20 + 1 * 10 + 10 * 5
663: 0 * 200 + 0 * 100 + 0 * 50 + 22 * 20 + 2 * 10 + 8 * 5
664: 0 * 200 + 0 * 100 + 0 * 50 + 22 * 20 + 3 * 10 + 6 * 5
665: 0 * 200 + 0 * 100 + 0 * 50 + 22 * 20 + 4 * 10 + 4 * 5
666: 0 * 200 + 0 * 100 + 0 * 50 + 22 * 20 + 5 * 10 + 2 * 5
667: 0 * 200 + 0 * 100 + 0 * 50 + 22 * 20 + 6 * 10 + 0 * 5
668: 0 * 200 + 0 * 100 + 0 * 50 + 23 * 20 + 0 * 10 + 8 * 5
669: 0 * 200 + 0 * 100 + 0 * 50 + 23 * 20 + 1 * 10 + 6 * 5
670: 0 * 200 + 0 * 100 + 0 * 50 + 23 * 20 + 2 * 10 + 4 * 5
671: 0 * 200 + 0 * 100 + 0 * 50 + 23 * 20 + 3 * 10 + 2 * 5
672: 0 * 200 + 0 * 100 + 0 * 50 + 23 * 20 + 4 * 10 + 0 * 5
673: 0 * 200 + 0 * 100 + 0 * 50 + 24 * 20 + 0 * 10 + 4 * 5
674: 0 * 200 + 0 * 100 + 0 * 50 + 24 * 20 + 1 * 10 + 2 * 5
675: 0 * 200 + 0 * 100 + 0 * 50 + 24 * 20 + 2 * 10 + 0 * 5
676: 0 * 200 + 0 * 100 + 0 * 50 + 25 * 20 + 0 * 10 + 0 * 5
677: 0 * 200 + 0 * 100 + 1 * 50 + 0 * 20 + 0 * 10 + 90 * 5
678: 0 * 200 + 0 * 100 + 1 * 50 + 0 * 20 + 1 * 10 + 88 * 5
679: 0 * 200 + 0 * 100 + 1 * 50 + 0 * 20 + 2 * 10 + 86 * 5
680: 0 * 200 + 0 * 100 + 1 * 50 + 0 * 20 + 3 * 10 + 84 * 5
681: 0 * 200 + 0 * 100 + 1 * 50 + 0 * 20 + 4 * 10 + 82 * 5
682: 0 * 200 + 0 * 100 + 1 * 50 + 0 * 20 + 5 * 10 + 80 * 5
683: 0 * 200 + 0 * 100 + 1 * 50 + 0 * 20 + 6 * 10 + 78 * 5
684: 0 * 200 + 0 * 100 + 1 * 50 + 0 * 20 + 7 * 10 + 76 * 5
685: 0 * 200 + 0 * 100 + 1 * 50 + 0 * 20 + 8 * 10 + 74 * 5
686: 0 * 200 + 0 * 100 + 1 * 50 + 0 * 20 + 9 * 10 + 72 * 5
687: 0 * 200 + 0 * 100 + 1 * 50 + 0 * 20 + 10 * 10 + 70 * 5
688: 0 * 200 + 0 * 100 + 1 * 50 + 0 * 20 + 11 * 10 + 68 * 5
689: 0 * 200 + 0 * 100 + 1 * 50 + 0 * 20 + 12 * 10 + 66 * 5
690: 0 * 200 + 0 * 100 + 1 * 50 + 0 * 20 + 13 * 10 + 64 * 5
691: 0 * 200 + 0 * 100 + 1 * 50 + 0 * 20 + 14 * 10 + 62 * 5
692: 0 * 200 + 0 * 100 + 1 * 50 + 0 * 20 + 15 * 10 + 60 * 5
693: 0 * 200 + 0 * 100 + 1 * 50 + 0 * 20 + 16 * 10 + 58 * 5
694: 0 * 200 + 0 * 100 + 1 * 50 + 0 * 20 + 17 * 10 + 56 * 5
695: 0 * 200 + 0 * 100 + 1 * 50 + 0 * 20 + 18 * 10 + 54 * 5
696: 0 * 200 + 0 * 100 + 1 * 50 + 0 * 20 + 19 * 10 + 52 * 5
697: 0 * 200 + 0 * 100 + 1 * 50 + 0 * 20 + 20 * 10 + 50 * 5
698: 0 * 200 + 0 * 100 + 1 * 50 + 0 * 20 + 21 * 10 + 48 * 5
699: 0 * 200 + 0 * 100 + 1 * 50 + 0 * 20 + 22 * 10 + 46 * 5
700: 0 * 200 + 0 * 100 + 1 * 50 + 0 * 20 + 23 * 10 + 44 * 5
701: 0 * 200 + 0 * 100 + 1 * 50 + 0 * 20 + 24 * 10 + 42 * 5
702: 0 * 200 + 0 * 100 + 1 * 50 + 0 * 20 + 25 * 10 + 40 * 5
703: 0 * 200 + 0 * 100 + 1 * 50 + 0 * 20 + 26 * 10 + 38 * 5
704: 0 * 200 + 0 * 100 + 1 * 50 + 0 * 20 + 27 * 10 + 36 * 5
705: 0 * 200 + 0 * 100 + 1 * 50 + 0 * 20 + 28 * 10 + 34 * 5
706: 0 * 200 + 0 * 100 + 1 * 50 + 0 * 20 + 29 * 10 + 32 * 5
707: 0 * 200 + 0 * 100 + 1 * 50 + 0 * 20 + 30 * 10 + 30 * 5
708: 0 * 200 + 0 * 100 + 1 * 50 + 0 * 20 + 31 * 10 + 28 * 5
709: 0 * 200 + 0 * 100 + 1 * 50 + 0 * 20 + 32 * 10 + 26 * 5
710: 0 * 200 + 0 * 100 + 1 * 50 + 0 * 20 + 33 * 10 + 24 * 5
711: 0 * 200 + 0 * 100 + 1 * 50 + 0 * 20 + 34 * 10 + 22 * 5
712: 0 * 200 + 0 * 100 + 1 * 50 + 0 * 20 + 35 * 10 + 20 * 5
713: 0 * 200 + 0 * 100 + 1 * 50 + 0 * 20 + 36 * 10 + 18 * 5
714: 0 * 200 + 0 * 100 + 1 * 50 + 0 * 20 + 37 * 10 + 16 * 5
715: 0 * 200 + 0 * 100 + 1 * 50 + 0 * 20 + 38 * 10 + 14 * 5
716: 0 * 200 + 0 * 100 + 1 * 50 + 0 * 20 + 39 * 10 + 12 * 5
717: 0 * 200 + 0 * 100 + 1 * 50 + 0 * 20 + 40 * 10 + 10 * 5
718: 0 * 200 + 0 * 100 + 1 * 50 + 0 * 20 + 41 * 10 + 8 * 5
719: 0 * 200 + 0 * 100 + 1 * 50 + 0 * 20 + 42 * 10 + 6 * 5
720: 0 * 200 + 0 * 100 + 1 * 50 + 0 * 20 + 43 * 10 + 4 * 5
721: 0 * 200 + 0 * 100 + 1 * 50 + 0 * 20 + 44 * 10 + 2 * 5
722: 0 * 200 + 0 * 100 + 1 * 50 + 0 * 20 + 45 * 10 + 0 * 5
723: 0 * 200 + 0 * 100 + 1 * 50 + 1 * 20 + 0 * 10 + 86 * 5
724: 0 * 200 + 0 * 100 + 1 * 50 + 1 * 20 + 1 * 10 + 84 * 5
725: 0 * 200 + 0 * 100 + 1 * 50 + 1 * 20 + 2 * 10 + 82 * 5
726: 0 * 200 + 0 * 100 + 1 * 50 + 1 * 20 + 3 * 10 + 80 * 5
727: 0 * 200 + 0 * 100 + 1 * 50 + 1 * 20 + 4 * 10 + 78 * 5
728: 0 * 200 + 0 * 100 + 1 * 50 + 1 * 20 + 5 * 10 + 76 * 5
729: 0 * 200 + 0 * 100 + 1 * 50 + 1 * 20 + 6 * 10 + 74 * 5
730: 0 * 200 + 0 * 100 + 1 * 50 + 1 * 20 + 7 * 10 + 72 * 5
731: 0 * 200 + 0 * 100 + 1 * 50 + 1 * 20 + 8 * 10 + 70 * 5
732: 0 * 200 + 0 * 100 + 1 * 50 + 1 * 20 + 9 * 10 + 68 * 5
733: 0 * 200 + 0 * 100 + 1 * 50 + 1 * 20 + 10 * 10 + 66 * 5
734: 0 * 200 + 0 * 100 + 1 * 50 + 1 * 20 + 11 * 10 + 64 * 5
735: 0 * 200 + 0 * 100 + 1 * 50 + 1 * 20 + 12 * 10 + 62 * 5
736: 0 * 200 + 0 * 100 + 1 * 50 + 1 * 20 + 13 * 10 + 60 * 5
737: 0 * 200 + 0 * 100 + 1 * 50 + 1 * 20 + 14 * 10 + 58 * 5
738: 0 * 200 + 0 * 100 + 1 * 50 + 1 * 20 + 15 * 10 + 56 * 5
739: 0 * 200 + 0 * 100 + 1 * 50 + 1 * 20 + 16 * 10 + 54 * 5
740: 0 * 200 + 0 * 100 + 1 * 50 + 1 * 20 + 17 * 10 + 52 * 5
741: 0 * 200 + 0 * 100 + 1 * 50 + 1 * 20 + 18 * 10 + 50 * 5
742: 0 * 200 + 0 * 100 + 1 * 50 + 1 * 20 + 19 * 10 + 48 * 5
743: 0 * 200 + 0 * 100 + 1 * 50 + 1 * 20 + 20 * 10 + 46 * 5
744: 0 * 200 + 0 * 100 + 1 * 50 + 1 * 20 + 21 * 10 + 44 * 5
745: 0 * 200 + 0 * 100 + 1 * 50 + 1 * 20 + 22 * 10 + 42 * 5
746: 0 * 200 + 0 * 100 + 1 * 50 + 1 * 20 + 23 * 10 + 40 * 5
747: 0 * 200 + 0 * 100 + 1 * 50 + 1 * 20 + 24 * 10 + 38 * 5
748: 0 * 200 + 0 * 100 + 1 * 50 + 1 * 20 + 25 * 10 + 36 * 5
749: 0 * 200 + 0 * 100 + 1 * 50 + 1 * 20 + 26 * 10 + 34 * 5
750: 0 * 200 + 0 * 100 + 1 * 50 + 1 * 20 + 27 * 10 + 32 * 5
751: 0 * 200 + 0 * 100 + 1 * 50 + 1 * 20 + 28 * 10 + 30 * 5
752: 0 * 200 + 0 * 100 + 1 * 50 + 1 * 20 + 29 * 10 + 28 * 5
753: 0 * 200 + 0 * 100 + 1 * 50 + 1 * 20 + 30 * 10 + 26 * 5
754: 0 * 200 + 0 * 100 + 1 * 50 + 1 * 20 + 31 * 10 + 24 * 5
755: 0 * 200 + 0 * 100 + 1 * 50 + 1 * 20 + 32 * 10 + 22 * 5
756: 0 * 200 + 0 * 100 + 1 * 50 + 1 * 20 + 33 * 10 + 20 * 5
757: 0 * 200 + 0 * 100 + 1 * 50 + 1 * 20 + 34 * 10 + 18 * 5
758: 0 * 200 + 0 * 100 + 1 * 50 + 1 * 20 + 35 * 10 + 16 * 5
759: 0 * 200 + 0 * 100 + 1 * 50 + 1 * 20 + 36 * 10 + 14 * 5
760: 0 * 200 + 0 * 100 + 1 * 50 + 1 * 20 + 37 * 10 + 12 * 5
761: 0 * 200 + 0 * 100 + 1 * 50 + 1 * 20 + 38 * 10 + 10 * 5
762: 0 * 200 + 0 * 100 + 1 * 50 + 1 * 20 + 39 * 10 + 8 * 5
763: 0 * 200 + 0 * 100 + 1 * 50 + 1 * 20 + 40 * 10 + 6 * 5
764: 0 * 200 + 0 * 100 + 1 * 50 + 1 * 20 + 41 * 10 + 4 * 5
765: 0 * 200 + 0 * 100 + 1 * 50 + 1 * 20 + 42 * 10 + 2 * 5
766: 0 * 200 + 0 * 100 + 1 * 50 + 1 * 20 + 43 * 10 + 0 * 5
767: 0 * 200 + 0 * 100 + 1 * 50 + 2 * 20 + 0 * 10 + 82 * 5
768: 0 * 200 + 0 * 100 + 1 * 50 + 2 * 20 + 1 * 10 + 80 * 5
769: 0 * 200 + 0 * 100 + 1 * 50 + 2 * 20 + 2 * 10 + 78 * 5
770: 0 * 200 + 0 * 100 + 1 * 50 + 2 * 20 + 3 * 10 + 76 * 5
771: 0 * 200 + 0 * 100 + 1 * 50 + 2 * 20 + 4 * 10 + 74 * 5
772: 0 * 200 + 0 * 100 + 1 * 50 + 2 * 20 + 5 * 10 + 72 * 5
773: 0 * 200 + 0 * 100 + 1 * 50 + 2 * 20 + 6 * 10 + 70 * 5
774: 0 * 200 + 0 * 100 + 1 * 50 + 2 * 20 + 7 * 10 + 68 * 5
775: 0 * 200 + 0 * 100 + 1 * 50 + 2 * 20 + 8 * 10 + 66 * 5
776: 0 * 200 + 0 * 100 + 1 * 50 + 2 * 20 + 9 * 10 + 64 * 5
777: 0 * 200 + 0 * 100 + 1 * 50 + 2 * 20 + 10 * 10 + 62 * 5
778: 0 * 200 + 0 * 100 + 1 * 50 + 2 * 20 + 11 * 10 + 60 * 5
779: 0 * 200 + 0 * 100 + 1 * 50 + 2 * 20 + 12 * 10 + 58 * 5
780: 0 * 200 + 0 * 100 + 1 * 50 + 2 * 20 + 13 * 10 + 56 * 5
781: 0 * 200 + 0 * 100 + 1 * 50 + 2 * 20 + 14 * 10 + 54 * 5
782: 0 * 200 + 0 * 100 + 1 * 50 + 2 * 20 + 15 * 10 + 52 * 5
783: 0 * 200 + 0 * 100 + 1 * 50 + 2 * 20 + 16 * 10 + 50 * 5
784: 0 * 200 + 0 * 100 + 1 * 50 + 2 * 20 + 17 * 10 + 48 * 5
785: 0 * 200 + 0 * 100 + 1 * 50 + 2 * 20 + 18 * 10 + 46 * 5
786: 0 * 200 + 0 * 100 + 1 * 50 + 2 * 20 + 19 * 10 + 44 * 5
787: 0 * 200 + 0 * 100 + 1 * 50 + 2 * 20 + 20 * 10 + 42 * 5
788: 0 * 200 + 0 * 100 + 1 * 50 + 2 * 20 + 21 * 10 + 40 * 5
789: 0 * 200 + 0 * 100 + 1 * 50 + 2 * 20 + 22 * 10 + 38 * 5
790: 0 * 200 + 0 * 100 + 1 * 50 + 2 * 20 + 23 * 10 + 36 * 5
791: 0 * 200 + 0 * 100 + 1 * 50 + 2 * 20 + 24 * 10 + 34 * 5
792: 0 * 200 + 0 * 100 + 1 * 50 + 2 * 20 + 25 * 10 + 32 * 5
793: 0 * 200 + 0 * 100 + 1 * 50 + 2 * 20 + 26 * 10 + 30 * 5
794: 0 * 200 + 0 * 100 + 1 * 50 + 2 * 20 + 27 * 10 + 28 * 5
795: 0 * 200 + 0 * 100 + 1 * 50 + 2 * 20 + 28 * 10 + 26 * 5
796: 0 * 200 + 0 * 100 + 1 * 50 + 2 * 20 + 29 * 10 + 24 * 5
797: 0 * 200 + 0 * 100 + 1 * 50 + 2 * 20 + 30 * 10 + 22 * 5
798: 0 * 200 + 0 * 100 + 1 * 50 + 2 * 20 + 31 * 10 + 20 * 5
799: 0 * 200 + 0 * 100 + 1 * 50 + 2 * 20 + 32 * 10 + 18 * 5
800: 0 * 200 + 0 * 100 + 1 * 50 + 2 * 20 + 33 * 10 + 16 * 5
801: 0 * 200 + 0 * 100 + 1 * 50 + 2 * 20 + 34 * 10 + 14 * 5
802: 0 * 200 + 0 * 100 + 1 * 50 + 2 * 20 + 35 * 10 + 12 * 5
803: 0 * 200 + 0 * 100 + 1 * 50 + 2 * 20 + 36 * 10 + 10 * 5
804: 0 * 200 + 0 * 100 + 1 * 50 + 2 * 20 + 37 * 10 + 8 * 5
805: 0 * 200 + 0 * 100 + 1 * 50 + 2 * 20 + 38 * 10 + 6 * 5
806: 0 * 200 + 0 * 100 + 1 * 50 + 2 * 20 + 39 * 10 + 4 * 5
807: 0 * 200 + 0 * 100 + 1 * 50 + 2 * 20 + 40 * 10 + 2 * 5
808: 0 * 200 + 0 * 100 + 1 * 50 + 2 * 20 + 41 * 10 + 0 * 5
809: 0 * 200 + 0 * 100 + 1 * 50 + 3 * 20 + 0 * 10 + 78 * 5
810: 0 * 200 + 0 * 100 + 1 * 50 + 3 * 20 + 1 * 10 + 76 * 5
811: 0 * 200 + 0 * 100 + 1 * 50 + 3 * 20 + 2 * 10 + 74 * 5
812: 0 * 200 + 0 * 100 + 1 * 50 + 3 * 20 + 3 * 10 + 72 * 5
813: 0 * 200 + 0 * 100 + 1 * 50 + 3 * 20 + 4 * 10 + 70 * 5
814: 0 * 200 + 0 * 100 + 1 * 50 + 3 * 20 + 5 * 10 + 68 * 5
815: 0 * 200 + 0 * 100 + 1 * 50 + 3 * 20 + 6 * 10 + 66 * 5
816: 0 * 200 + 0 * 100 + 1 * 50 + 3 * 20 + 7 * 10 + 64 * 5
817: 0 * 200 + 0 * 100 + 1 * 50 + 3 * 20 + 8 * 10 + 62 * 5
818: 0 * 200 + 0 * 100 + 1 * 50 + 3 * 20 + 9 * 10 + 60 * 5
819: 0 * 200 + 0 * 100 + 1 * 50 + 3 * 20 + 10 * 10 + 58 * 5
820: 0 * 200 + 0 * 100 + 1 * 50 + 3 * 20 + 11 * 10 + 56 * 5
821: 0 * 200 + 0 * 100 + 1 * 50 + 3 * 20 + 12 * 10 + 54 * 5
822: 0 * 200 + 0 * 100 + 1 * 50 + 3 * 20 + 13 * 10 + 52 * 5
823: 0 * 200 + 0 * 100 + 1 * 50 + 3 * 20 + 14 * 10 + 50 * 5
824: 0 * 200 + 0 * 100 + 1 * 50 + 3 * 20 + 15 * 10 + 48 * 5
825: 0 * 200 + 0 * 100 + 1 * 50 + 3 * 20 + 16 * 10 + 46 * 5
826: 0 * 200 + 0 * 100 + 1 * 50 + 3 * 20 + 17 * 10 + 44 * 5
827: 0 * 200 + 0 * 100 + 1 * 50 + 3 * 20 + 18 * 10 + 42 * 5
828: 0 * 200 + 0 * 100 + 1 * 50 + 3 * 20 + 19 * 10 + 40 * 5
829: 0 * 200 + 0 * 100 + 1 * 50 + 3 * 20 + 20 * 10 + 38 * 5
830: 0 * 200 + 0 * 100 + 1 * 50 + 3 * 20 + 21 * 10 + 36 * 5
831: 0 * 200 + 0 * 100 + 1 * 50 + 3 * 20 + 22 * 10 + 34 * 5
832: 0 * 200 + 0 * 100 + 1 * 50 + 3 * 20 + 23 * 10 + 32 * 5
833: 0 * 200 + 0 * 100 + 1 * 50 + 3 * 20 + 24 * 10 + 30 * 5
834: 0 * 200 + 0 * 100 + 1 * 50 + 3 * 20 + 25 * 10 + 28 * 5
835: 0 * 200 + 0 * 100 + 1 * 50 + 3 * 20 + 26 * 10 + 26 * 5
836: 0 * 200 + 0 * 100 + 1 * 50 + 3 * 20 + 27 * 10 + 24 * 5
837: 0 * 200 + 0 * 100 + 1 * 50 + 3 * 20 + 28 * 10 + 22 * 5
838: 0 * 200 + 0 * 100 + 1 * 50 + 3 * 20 + 29 * 10 + 20 * 5
839: 0 * 200 + 0 * 100 + 1 * 50 + 3 * 20 + 30 * 10 + 18 * 5
840: 0 * 200 + 0 * 100 + 1 * 50 + 3 * 20 + 31 * 10 + 16 * 5
841: 0 * 200 + 0 * 100 + 1 * 50 + 3 * 20 + 32 * 10 + 14 * 5
842: 0 * 200 + 0 * 100 + 1 * 50 + 3 * 20 + 33 * 10 + 12 * 5
843: 0 * 200 + 0 * 100 + 1 * 50 + 3 * 20 + 34 * 10 + 10 * 5
844: 0 * 200 + 0 * 100 + 1 * 50 + 3 * 20 + 35 * 10 + 8 * 5
845: 0 * 200 + 0 * 100 + 1 * 50 + 3 * 20 + 36 * 10 + 6 * 5
846: 0 * 200 + 0 * 100 + 1 * 50 + 3 * 20 + 37 * 10 + 4 * 5
847: 0 * 200 + 0 * 100 + 1 * 50 + 3 * 20 + 38 * 10 + 2 * 5
848: 0 * 200 + 0 * 100 + 1 * 50 + 3 * 20 + 39 * 10 + 0 * 5
849: 0 * 200 + 0 * 100 + 1 * 50 + 4 * 20 + 0 * 10 + 74 * 5
850: 0 * 200 + 0 * 100 + 1 * 50 + 4 * 20 + 1 * 10 + 72 * 5
851: 0 * 200 + 0 * 100 + 1 * 50 + 4 * 20 + 2 * 10 + 70 * 5
852: 0 * 200 + 0 * 100 + 1 * 50 + 4 * 20 + 3 * 10 + 68 * 5
853: 0 * 200 + 0 * 100 + 1 * 50 + 4 * 20 + 4 * 10 + 66 * 5
854: 0 * 200 + 0 * 100 + 1 * 50 + 4 * 20 + 5 * 10 + 64 * 5
855: 0 * 200 + 0 * 100 + 1 * 50 + 4 * 20 + 6 * 10 + 62 * 5
856: 0 * 200 + 0 * 100 + 1 * 50 + 4 * 20 + 7 * 10 + 60 * 5
857: 0 * 200 + 0 * 100 + 1 * 50 + 4 * 20 + 8 * 10 + 58 * 5
858: 0 * 200 + 0 * 100 + 1 * 50 + 4 * 20 + 9 * 10 + 56 * 5
859: 0 * 200 + 0 * 100 + 1 * 50 + 4 * 20 + 10 * 10 + 54 * 5
860: 0 * 200 + 0 * 100 + 1 * 50 + 4 * 20 + 11 * 10 + 52 * 5
861: 0 * 200 + 0 * 100 + 1 * 50 + 4 * 20 + 12 * 10 + 50 * 5
862: 0 * 200 + 0 * 100 + 1 * 50 + 4 * 20 + 13 * 10 + 48 * 5
863: 0 * 200 + 0 * 100 + 1 * 50 + 4 * 20 + 14 * 10 + 46 * 5
864: 0 * 200 + 0 * 100 + 1 * 50 + 4 * 20 + 15 * 10 + 44 * 5
865: 0 * 200 + 0 * 100 + 1 * 50 + 4 * 20 + 16 * 10 + 42 * 5
866: 0 * 200 + 0 * 100 + 1 * 50 + 4 * 20 + 17 * 10 + 40 * 5
867: 0 * 200 + 0 * 100 + 1 * 50 + 4 * 20 + 18 * 10 + 38 * 5
868: 0 * 200 + 0 * 100 + 1 * 50 + 4 * 20 + 19 * 10 + 36 * 5
869: 0 * 200 + 0 * 100 + 1 * 50 + 4 * 20 + 20 * 10 + 34 * 5
870: 0 * 200 + 0 * 100 + 1 * 50 + 4 * 20 + 21 * 10 + 32 * 5
871: 0 * 200 + 0 * 100 + 1 * 50 + 4 * 20 + 22 * 10 + 30 * 5
872: 0 * 200 + 0 * 100 + 1 * 50 + 4 * 20 + 23 * 10 + 28 * 5
873: 0 * 200 + 0 * 100 + 1 * 50 + 4 * 20 + 24 * 10 + 26 * 5
874: 0 * 200 + 0 * 100 + 1 * 50 + 4 * 20 + 25 * 10 + 24 * 5
875: 0 * 200 + 0 * 100 + 1 * 50 + 4 * 20 + 26 * 10 + 22 * 5
876: 0 * 200 + 0 * 100 + 1 * 50 + 4 * 20 + 27 * 10 + 20 * 5
877: 0 * 200 + 0 * 100 + 1 * 50 + 4 * 20 + 28 * 10 + 18 * 5
878: 0 * 200 + 0 * 100 + 1 * 50 + 4 * 20 + 29 * 10 + 16 * 5
879: 0 * 200 + 0 * 100 + 1 * 50 + 4 * 20 + 30 * 10 + 14 * 5
880: 0 * 200 + 0 * 100 + 1 * 50 + 4 * 20 + 31 * 10 + 12 * 5
881: 0 * 200 + 0 * 100 + 1 * 50 + 4 * 20 + 32 * 10 + 10 * 5
882: 0 * 200 + 0 * 100 + 1 * 50 + 4 * 20 + 33 * 10 + 8 * 5
883: 0 * 200 + 0 * 100 + 1 * 50 + 4 * 20 + 34 * 10 + 6 * 5
884: 0 * 200 + 0 * 100 + 1 * 50 + 4 * 20 + 35 * 10 + 4 * 5
885: 0 * 200 + 0 * 100 + 1 * 50 + 4 * 20 + 36 * 10 + 2 * 5
886: 0 * 200 + 0 * 100 + 1 * 50 + 4 * 20 + 37 * 10 + 0 * 5
887: 0 * 200 + 0 * 100 + 1 * 50 + 5 * 20 + 0 * 10 + 70 * 5
888: 0 * 200 + 0 * 100 + 1 * 50 + 5 * 20 + 1 * 10 + 68 * 5
889: 0 * 200 + 0 * 100 + 1 * 50 + 5 * 20 + 2 * 10 + 66 * 5
890: 0 * 200 + 0 * 100 + 1 * 50 + 5 * 20 + 3 * 10 + 64 * 5
891: 0 * 200 + 0 * 100 + 1 * 50 + 5 * 20 + 4 * 10 + 62 * 5
892: 0 * 200 + 0 * 100 + 1 * 50 + 5 * 20 + 5 * 10 + 60 * 5
893: 0 * 200 + 0 * 100 + 1 * 50 + 5 * 20 + 6 * 10 + 58 * 5
894: 0 * 200 + 0 * 100 + 1 * 50 + 5 * 20 + 7 * 10 + 56 * 5
895: 0 * 200 + 0 * 100 + 1 * 50 + 5 * 20 + 8 * 10 + 54 * 5
896: 0 * 200 + 0 * 100 + 1 * 50 + 5 * 20 + 9 * 10 + 52 * 5
897: 0 * 200 + 0 * 100 + 1 * 50 + 5 * 20 + 10 * 10 + 50 * 5
898: 0 * 200 + 0 * 100 + 1 * 50 + 5 * 20 + 11 * 10 + 48 * 5
899: 0 * 200 + 0 * 100 + 1 * 50 + 5 * 20 + 12 * 10 + 46 * 5
900: 0 * 200 + 0 * 100 + 1 * 50 + 5 * 20 + 13 * 10 + 44 * 5
901: 0 * 200 + 0 * 100 + 1 * 50 + 5 * 20 + 14 * 10 + 42 * 5
902: 0 * 200 + 0 * 100 + 1 * 50 + 5 * 20 + 15 * 10 + 40 * 5
903: 0 * 200 + 0 * 100 + 1 * 50 + 5 * 20 + 16 * 10 + 38 * 5
904: 0 * 200 + 0 * 100 + 1 * 50 + 5 * 20 + 17 * 10 + 36 * 5
905: 0 * 200 + 0 * 100 + 1 * 50 + 5 * 20 + 18 * 10 + 34 * 5
906: 0 * 200 + 0 * 100 + 1 * 50 + 5 * 20 + 19 * 10 + 32 * 5
907: 0 * 200 + 0 * 100 + 1 * 50 + 5 * 20 + 20 * 10 + 30 * 5
908: 0 * 200 + 0 * 100 + 1 * 50 + 5 * 20 + 21 * 10 + 28 * 5
909: 0 * 200 + 0 * 100 + 1 * 50 + 5 * 20 + 22 * 10 + 26 * 5
910: 0 * 200 + 0 * 100 + 1 * 50 + 5 * 20 + 23 * 10 + 24 * 5
911: 0 * 200 + 0 * 100 + 1 * 50 + 5 * 20 + 24 * 10 + 22 * 5
912: 0 * 200 + 0 * 100 + 1 * 50 + 5 * 20 + 25 * 10 + 20 * 5
913: 0 * 200 + 0 * 100 + 1 * 50 + 5 * 20 + 26 * 10 + 18 * 5
914: 0 * 200 + 0 * 100 + 1 * 50 + 5 * 20 + 27 * 10 + 16 * 5
915: 0 * 200 + 0 * 100 + 1 * 50 + 5 * 20 + 28 * 10 + 14 * 5
916: 0 * 200 + 0 * 100 + 1 * 50 + 5 * 20 + 29 * 10 + 12 * 5
917: 0 * 200 + 0 * 100 + 1 * 50 + 5 * 20 + 30 * 10 + 10 * 5
918: 0 * 200 + 0 * 100 + 1 * 50 + 5 * 20 + 31 * 10 + 8 * 5
919: 0 * 200 + 0 * 100 + 1 * 50 + 5 * 20 + 32 * 10 + 6 * 5
920: 0 * 200 + 0 * 100 + 1 * 50 + 5 * 20 + 33 * 10 + 4 * 5
921: 0 * 200 + 0 * 100 + 1 * 50 + 5 * 20 + 34 * 10 + 2 * 5
922: 0 * 200 + 0 * 100 + 1 * 50 + 5 * 20 + 35 * 10 + 0 * 5
923: 0 * 200 + 0 * 100 + 1 * 50 + 6 * 20 + 0 * 10 + 66 * 5
924: 0 * 200 + 0 * 100 + 1 * 50 + 6 * 20 + 1 * 10 + 64 * 5
925: 0 * 200 + 0 * 100 + 1 * 50 + 6 * 20 + 2 * 10 + 62 * 5
926: 0 * 200 + 0 * 100 + 1 * 50 + 6 * 20 + 3 * 10 + 60 * 5
927: 0 * 200 + 0 * 100 + 1 * 50 + 6 * 20 + 4 * 10 + 58 * 5
928: 0 * 200 + 0 * 100 + 1 * 50 + 6 * 20 + 5 * 10 + 56 * 5
929: 0 * 200 + 0 * 100 + 1 * 50 + 6 * 20 + 6 * 10 + 54 * 5
930: 0 * 200 + 0 * 100 + 1 * 50 + 6 * 20 + 7 * 10 + 52 * 5
931: 0 * 200 + 0 * 100 + 1 * 50 + 6 * 20 + 8 * 10 + 50 * 5
932: 0 * 200 + 0 * 100 + 1 * 50 + 6 * 20 + 9 * 10 + 48 * 5
933: 0 * 200 + 0 * 100 + 1 * 50 + 6 * 20 + 10 * 10 + 46 * 5
934: 0 * 200 + 0 * 100 + 1 * 50 + 6 * 20 + 11 * 10 + 44 * 5
935: 0 * 200 + 0 * 100 + 1 * 50 + 6 * 20 + 12 * 10 + 42 * 5
936: 0 * 200 + 0 * 100 + 1 * 50 + 6 * 20 + 13 * 10 + 40 * 5
937: 0 * 200 + 0 * 100 + 1 * 50 + 6 * 20 + 14 * 10 + 38 * 5
938: 0 * 200 + 0 * 100 + 1 * 50 + 6 * 20 + 15 * 10 + 36 * 5
939: 0 * 200 + 0 * 100 + 1 * 50 + 6 * 20 + 16 * 10 + 34 * 5
940: 0 * 200 + 0 * 100 + 1 * 50 + 6 * 20 + 17 * 10 + 32 * 5
941: 0 * 200 + 0 * 100 + 1 * 50 + 6 * 20 + 18 * 10 + 30 * 5
942: 0 * 200 + 0 * 100 + 1 * 50 + 6 * 20 + 19 * 10 + 28 * 5
943: 0 * 200 + 0 * 100 + 1 * 50 + 6 * 20 + 20 * 10 + 26 * 5
944: 0 * 200 + 0 * 100 + 1 * 50 + 6 * 20 + 21 * 10 + 24 * 5
945: 0 * 200 + 0 * 100 + 1 * 50 + 6 * 20 + 22 * 10 + 22 * 5
946: 0 * 200 + 0 * 100 + 1 * 50 + 6 * 20 + 23 * 10 + 20 * 5
947: 0 * 200 + 0 * 100 + 1 * 50 + 6 * 20 + 24 * 10 + 18 * 5
948: 0 * 200 + 0 * 100 + 1 * 50 + 6 * 20 + 25 * 10 + 16 * 5
949: 0 * 200 + 0 * 100 + 1 * 50 + 6 * 20 + 26 * 10 + 14 * 5
950: 0 * 200 + 0 * 100 + 1 * 50 + 6 * 20 + 27 * 10 + 12 * 5
951: 0 * 200 + 0 * 100 + 1 * 50 + 6 * 20 + 28 * 10 + 10 * 5
952: 0 * 200 + 0 * 100 + 1 * 50 + 6 * 20 + 29 * 10 + 8 * 5
953: 0 * 200 + 0 * 100 + 1 * 50 + 6 * 20 + 30 * 10 + 6 * 5
954: 0 * 200 + 0 * 100 + 1 * 50 + 6 * 20 + 31 * 10 + 4 * 5
955: 0 * 200 + 0 * 100 + 1 * 50 + 6 * 20 + 32 * 10 + 2 * 5
956: 0 * 200 + 0 * 100 + 1 * 50 + 6 * 20 + 33 * 10 + 0 * 5
957: 0 * 200 + 0 * 100 + 1 * 50 + 7 * 20 + 0 * 10 + 62 * 5
958: 0 * 200 + 0 * 100 + 1 * 50 + 7 * 20 + 1 * 10 + 60 * 5
959: 0 * 200 + 0 * 100 + 1 * 50 + 7 * 20 + 2 * 10 + 58 * 5
960: 0 * 200 + 0 * 100 + 1 * 50 + 7 * 20 + 3 * 10 + 56 * 5
961: 0 * 200 + 0 * 100 + 1 * 50 + 7 * 20 + 4 * 10 + 54 * 5
962: 0 * 200 + 0 * 100 + 1 * 50 + 7 * 20 + 5 * 10 + 52 * 5
963: 0 * 200 + 0 * 100 + 1 * 50 + 7 * 20 + 6 * 10 + 50 * 5
964: 0 * 200 + 0 * 100 + 1 * 50 + 7 * 20 + 7 * 10 + 48 * 5
965: 0 * 200 + 0 * 100 + 1 * 50 + 7 * 20 + 8 * 10 + 46 * 5
966: 0 * 200 + 0 * 100 + 1 * 50 + 7 * 20 + 9 * 10 + 44 * 5
967: 0 * 200 + 0 * 100 + 1 * 50 + 7 * 20 + 10 * 10 + 42 * 5
968: 0 * 200 + 0 * 100 + 1 * 50 + 7 * 20 + 11 * 10 + 40 * 5
969: 0 * 200 + 0 * 100 + 1 * 50 + 7 * 20 + 12 * 10 + 38 * 5
970: 0 * 200 + 0 * 100 + 1 * 50 + 7 * 20 + 13 * 10 + 36 * 5
971: 0 * 200 + 0 * 100 + 1 * 50 + 7 * 20 + 14 * 10 + 34 * 5
972: 0 * 200 + 0 * 100 + 1 * 50 + 7 * 20 + 15 * 10 + 32 * 5
973: 0 * 200 + 0 * 100 + 1 * 50 + 7 * 20 + 16 * 10 + 30 * 5
974: 0 * 200 + 0 * 100 + 1 * 50 + 7 * 20 + 17 * 10 + 28 * 5
975: 0 * 200 + 0 * 100 + 1 * 50 + 7 * 20 + 18 * 10 + 26 * 5
976: 0 * 200 + 0 * 100 + 1 * 50 + 7 * 20 + 19 * 10 + 24 * 5
977: 0 * 200 + 0 * 100 + 1 * 50 + 7 * 20 + 20 * 10 + 22 * 5
978: 0 * 200 + 0 * 100 + 1 * 50 + 7 * 20 + 21 * 10 + 20 * 5
979: 0 * 200 + 0 * 100 + 1 * 50 + 7 * 20 + 22 * 10 + 18 * 5
980: 0 * 200 + 0 * 100 + 1 * 50 + 7 * 20 + 23 * 10 + 16 * 5
981: 0 * 200 + 0 * 100 + 1 * 50 + 7 * 20 + 24 * 10 + 14 * 5
982: 0 * 200 + 0 * 100 + 1 * 50 + 7 * 20 + 25 * 10 + 12 * 5
983: 0 * 200 + 0 * 100 + 1 * 50 + 7 * 20 + 26 * 10 + 10 * 5
984: 0 * 200 + 0 * 100 + 1 * 50 + 7 * 20 + 27 * 10 + 8 * 5
985: 0 * 200 + 0 * 100 + 1 * 50 + 7 * 20 + 28 * 10 + 6 * 5
986: 0 * 200 + 0 * 100 + 1 * 50 + 7 * 20 + 29 * 10 + 4 * 5
987: 0 * 200 + 0 * 100 + 1 * 50 + 7 * 20 + 30 * 10 + 2 * 5
988: 0 * 200 + 0 * 100 + 1 * 50 + 7 * 20 + 31 * 10 + 0 * 5
989: 0 * 200 + 0 * 100 + 1 * 50 + 8 * 20 + 0 * 10 + 58 * 5
990: 0 * 200 + 0 * 100 + 1 * 50 + 8 * 20 + 1 * 10 + 56 * 5
991: 0 * 200 + 0 * 100 + 1 * 50 + 8 * 20 + 2 * 10 + 54 * 5
992: 0 * 200 + 0 * 100 + 1 * 50 + 8 * 20 + 3 * 10 + 52 * 5
993: 0 * 200 + 0 * 100 + 1 * 50 + 8 * 20 + 4 * 10 + 50 * 5
994: 0 * 200 + 0 * 100 + 1 * 50 + 8 * 20 + 5 * 10 + 48 * 5
995: 0 * 200 + 0 * 100 + 1 * 50 + 8 * 20 + 6 * 10 + 46 * 5
996: 0 * 200 + 0 * 100 + 1 * 50 + 8 * 20 + 7 * 10 + 44 * 5
997: 0 * 200 + 0 * 100 + 1 * 50 + 8 * 20 + 8 * 10 + 42 * 5
998: 0 * 200 + 0 * 100 + 1 * 50 + 8 * 20 + 9 * 10 + 40 * 5
999: 0 * 200 + 0 * 100 + 1 * 50 + 8 * 20 + 10 * 10 + 38 * 5
1000: 0 * 200 + 0 * 100 + 1 * 50 + 8 * 20 + 11 * 10 + 36 * 5
1001: 0 * 200 + 0 * 100 + 1 * 50 + 8 * 20 + 12 * 10 + 34 * 5
1002: 0 * 200 + 0 * 100 + 1 * 50 + 8 * 20 + 13 * 10 + 32 * 5
1003: 0 * 200 + 0 * 100 + 1 * 50 + 8 * 20 + 14 * 10 + 30 * 5
1004: 0 * 200 + 0 * 100 + 1 * 50 + 8 * 20 + 15 * 10 + 28 * 5
1005: 0 * 200 + 0 * 100 + 1 * 50 + 8 * 20 + 16 * 10 + 26 * 5
1006: 0 * 200 + 0 * 100 + 1 * 50 + 8 * 20 + 17 * 10 + 24 * 5
1007: 0 * 200 + 0 * 100 + 1 * 50 + 8 * 20 + 18 * 10 + 22 * 5
1008: 0 * 200 + 0 * 100 + 1 * 50 + 8 * 20 + 19 * 10 + 20 * 5
1009: 0 * 200 + 0 * 100 + 1 * 50 + 8 * 20 + 20 * 10 + 18 * 5
1010: 0 * 200 + 0 * 100 + 1 * 50 + 8 * 20 + 21 * 10 + 16 * 5
1011: 0 * 200 + 0 * 100 + 1 * 50 + 8 * 20 + 22 * 10 + 14 * 5
1012: 0 * 200 + 0 * 100 + 1 * 50 + 8 * 20 + 23 * 10 + 12 * 5
1013: 0 * 200 + 0 * 100 + 1 * 50 + 8 * 20 + 24 * 10 + 10 * 5
1014: 0 * 200 + 0 * 100 + 1 * 50 + 8 * 20 + 25 * 10 + 8 * 5
1015: 0 * 200 + 0 * 100 + 1 * 50 + 8 * 20 + 26 * 10 + 6 * 5
1016: 0 * 200 + 0 * 100 + 1 * 50 + 8 * 20 + 27 * 10 + 4 * 5
1017: 0 * 200 + 0 * 100 + 1 * 50 + 8 * 20 + 28 * 10 + 2 * 5
1018: 0 * 200 + 0 * 100 + 1 * 50 + 8 * 20 + 29 * 10 + 0 * 5
1019: 0 * 200 + 0 * 100 + 1 * 50 + 9 * 20 + 0 * 10 + 54 * 5
1020: 0 * 200 + 0 * 100 + 1 * 50 + 9 * 20 + 1 * 10 + 52 * 5
1021: 0 * 200 + 0 * 100 + 1 * 50 + 9 * 20 + 2 * 10 + 50 * 5
1022: 0 * 200 + 0 * 100 + 1 * 50 + 9 * 20 + 3 * 10 + 48 * 5
1023: 0 * 200 + 0 * 100 + 1 * 50 + 9 * 20 + 4 * 10 + 46 * 5
1024: 0 * 200 + 0 * 100 + 1 * 50 + 9 * 20 + 5 * 10 + 44 * 5
1025: 0 * 200 + 0 * 100 + 1 * 50 + 9 * 20 + 6 * 10 + 42 * 5
1026: 0 * 200 + 0 * 100 + 1 * 50 + 9 * 20 + 7 * 10 + 40 * 5
1027: 0 * 200 + 0 * 100 + 1 * 50 + 9 * 20 + 8 * 10 + 38 * 5
1028: 0 * 200 + 0 * 100 + 1 * 50 + 9 * 20 + 9 * 10 + 36 * 5
1029: 0 * 200 + 0 * 100 + 1 * 50 + 9 * 20 + 10 * 10 + 34 * 5
1030: 0 * 200 + 0 * 100 + 1 * 50 + 9 * 20 + 11 * 10 + 32 * 5
1031: 0 * 200 + 0 * 100 + 1 * 50 + 9 * 20 + 12 * 10 + 30 * 5
1032: 0 * 200 + 0 * 100 + 1 * 50 + 9 * 20 + 13 * 10 + 28 * 5
1033: 0 * 200 + 0 * 100 + 1 * 50 + 9 * 20 + 14 * 10 + 26 * 5
1034: 0 * 200 + 0 * 100 + 1 * 50 + 9 * 20 + 15 * 10 + 24 * 5
1035: 0 * 200 + 0 * 100 + 1 * 50 + 9 * 20 + 16 * 10 + 22 * 5
1036: 0 * 200 + 0 * 100 + 1 * 50 + 9 * 20 + 17 * 10 + 20 * 5
1037: 0 * 200 + 0 * 100 + 1 * 50 + 9 * 20 + 18 * 10 + 18 * 5
1038: 0 * 200 + 0 * 100 + 1 * 50 + 9 * 20 + 19 * 10 + 16 * 5
1039: 0 * 200 + 0 * 100 + 1 * 50 + 9 * 20 + 20 * 10 + 14 * 5
1040: 0 * 200 + 0 * 100 + 1 * 50 + 9 * 20 + 21 * 10 + 12 * 5
1041: 0 * 200 + 0 * 100 + 1 * 50 + 9 * 20 + 22 * 10 + 10 * 5
1042: 0 * 200 + 0 * 100 + 1 * 50 + 9 * 20 + 23 * 10 + 8 * 5
1043: 0 * 200 + 0 * 100 + 1 * 50 + 9 * 20 + 24 * 10 + 6 * 5
1044: 0 * 200 + 0 * 100 + 1 * 50 + 9 * 20 + 25 * 10 + 4 * 5
1045: 0 * 200 + 0 * 100 + 1 * 50 + 9 * 20 + 26 * 10 + 2 * 5
1046: 0 * 200 + 0 * 100 + 1 * 50 + 9 * 20 + 27 * 10 + 0 * 5
1047: 0 * 200 + 0 * 100 + 1 * 50 + 10 * 20 + 0 * 10 + 50 * 5
1048: 0 * 200 + 0 * 100 + 1 * 50 + 10 * 20 + 1 * 10 + 48 * 5
1049: 0 * 200 + 0 * 100 + 1 * 50 + 10 * 20 + 2 * 10 + 46 * 5
1050: 0 * 200 + 0 * 100 + 1 * 50 + 10 * 20 + 3 * 10 + 44 * 5
1051: 0 * 200 + 0 * 100 + 1 * 50 + 10 * 20 + 4 * 10 + 42 * 5
1052: 0 * 200 + 0 * 100 + 1 * 50 + 10 * 20 + 5 * 10 + 40 * 5
1053: 0 * 200 + 0 * 100 + 1 * 50 + 10 * 20 + 6 * 10 + 38 * 5
1054: 0 * 200 + 0 * 100 + 1 * 50 + 10 * 20 + 7 * 10 + 36 * 5
1055: 0 * 200 + 0 * 100 + 1 * 50 + 10 * 20 + 8 * 10 + 34 * 5
1056: 0 * 200 + 0 * 100 + 1 * 50 + 10 * 20 + 9 * 10 + 32 * 5
1057: 0 * 200 + 0 * 100 + 1 * 50 + 10 * 20 + 10 * 10 + 30 * 5
1058: 0 * 200 + 0 * 100 + 1 * 50 + 10 * 20 + 11 * 10 + 28 * 5
1059: 0 * 200 + 0 * 100 + 1 * 50 + 10 * 20 + 12 * 10 + 26 * 5
1060: 0 * 200 + 0 * 100 + 1 * 50 + 10 * 20 + 13 * 10 + 24 * 5
1061: 0 * 200 + 0 * 100 + 1 * 50 + 10 * 20 + 14 * 10 + 22 * 5
1062: 0 * 200 + 0 * 100 + 1 * 50 + 10 * 20 + 15 * 10 + 20 * 5
1063: 0 * 200 + 0 * 100 + 1 * 50 + 10 * 20 + 16 * 10 + 18 * 5
1064: 0 * 200 + 0 * 100 + 1 * 50 + 10 * 20 + 17 * 10 + 16 * 5
1065: 0 * 200 + 0 * 100 + 1 * 50 + 10 * 20 + 18 * 10 + 14 * 5
1066: 0 * 200 + 0 * 100 + 1 * 50 + 10 * 20 + 19 * 10 + 12 * 5
1067: 0 * 200 + 0 * 100 + 1 * 50 + 10 * 20 + 20 * 10 + 10 * 5
1068: 0 * 200 + 0 * 100 + 1 * 50 + 10 * 20 + 21 * 10 + 8 * 5
1069: 0 * 200 + 0 * 100 + 1 * 50 + 10 * 20 + 22 * 10 + 6 * 5
1070: 0 * 200 + 0 * 100 + 1 * 50 + 10 * 20 + 23 * 10 + 4 * 5
1071: 0 * 200 + 0 * 100 + 1 * 50 + 10 * 20 + 24 * 10 + 2 * 5
1072: 0 * 200 + 0 * 100 + 1 * 50 + 10 * 20 + 25 * 10 + 0 * 5
1073: 0 * 200 + 0 * 100 + 1 * 50 + 11 * 20 + 0 * 10 + 46 * 5
1074: 0 * 200 + 0 * 100 + 1 * 50 + 11 * 20 + 1 * 10 + 44 * 5
1075: 0 * 200 + 0 * 100 + 1 * 50 + 11 * 20 + 2 * 10 + 42 * 5
1076: 0 * 200 + 0 * 100 + 1 * 50 + 11 * 20 + 3 * 10 + 40 * 5
1077: 0 * 200 + 0 * 100 + 1 * 50 + 11 * 20 + 4 * 10 + 38 * 5
1078: 0 * 200 + 0 * 100 + 1 * 50 + 11 * 20 + 5 * 10 + 36 * 5
1079: 0 * 200 + 0 * 100 + 1 * 50 + 11 * 20 + 6 * 10 + 34 * 5
1080: 0 * 200 + 0 * 100 + 1 * 50 + 11 * 20 + 7 * 10 + 32 * 5
1081: 0 * 200 + 0 * 100 + 1 * 50 + 11 * 20 + 8 * 10 + 30 * 5
1082: 0 * 200 + 0 * 100 + 1 * 50 + 11 * 20 + 9 * 10 + 28 * 5
1083: 0 * 200 + 0 * 100 + 1 * 50 + 11 * 20 + 10 * 10 + 26 * 5
1084: 0 * 200 + 0 * 100 + 1 * 50 + 11 * 20 + 11 * 10 + 24 * 5
1085: 0 * 200 + 0 * 100 + 1 * 50 + 11 * 20 + 12 * 10 + 22 * 5
1086: 0 * 200 + 0 * 100 + 1 * 50 + 11 * 20 + 13 * 10 + 20 * 5
1087: 0 * 200 + 0 * 100 + 1 * 50 + 11 * 20 + 14 * 10 + 18 * 5
1088: 0 * 200 + 0 * 100 + 1 * 50 + 11 * 20 + 15 * 10 + 16 * 5
1089: 0 * 200 + 0 * 100 + 1 * 50 + 11 * 20 + 16 * 10 + 14 * 5
1090: 0 * 200 + 0 * 100 + 1 * 50 + 11 * 20 + 17 * 10 + 12 * 5
1091: 0 * 200 + 0 * 100 + 1 * 50 + 11 * 20 + 18 * 10 + 10 * 5
1092: 0 * 200 + 0 * 100 + 1 * 50 + 11 * 20 + 19 * 10 + 8 * 5
1093: 0 * 200 + 0 * 100 + 1 * 50 + 11 * 20 + 20 * 10 + 6 * 5
1094: 0 * 200 + 0 * 100 + 1 * 50 + 11 * 20 + 21 * 10 + 4 * 5
1095: 0 * 200 + 0 * 100 + 1 * 50 + 11 * 20 + 22 * 10 + 2 * 5
1096: 0 * 200 + 0 * 100 + 1 * 50 + 11 * 20 + 23 * 10 + 0 * 5
1097: 0 * 200 + 0 * 100 + 1 * 50 + 12 * 20 + 0 * 10 + 42 * 5
1098: 0 * 200 + 0 * 100 + 1 * 50 + 12 * 20 + 1 * 10 + 40 * 5
1099: 0 * 200 + 0 * 100 + 1 * 50 + 12 * 20 + 2 * 10 + 38 * 5
1100: 0 * 200 + 0 * 100 + 1 * 50 + 12 * 20 + 3 * 10 + 36 * 5
1101: 0 * 200 + 0 * 100 + 1 * 50 + 12 * 20 + 4 * 10 + 34 * 5
1102: 0 * 200 + 0 * 100 + 1 * 50 + 12 * 20 + 5 * 10 + 32 * 5
1103: 0 * 200 + 0 * 100 + 1 * 50 + 12 * 20 + 6 * 10 + 30 * 5
1104: 0 * 200 + 0 * 100 + 1 * 50 + 12 * 20 + 7 * 10 + 28 * 5
1105: 0 * 200 + 0 * 100 + 1 * 50 + 12 * 20 + 8 * 10 + 26 * 5
1106: 0 * 200 + 0 * 100 + 1 * 50 + 12 * 20 + 9 * 10 + 24 * 5
1107: 0 * 200 + 0 * 100 + 1 * 50 + 12 * 20 + 10 * 10 + 22 * 5
1108: 0 * 200 + 0 * 100 + 1 * 50 + 12 * 20 + 11 * 10 + 20 * 5
1109: 0 * 200 + 0 * 100 + 1 * 50 + 12 * 20 + 12 * 10 + 18 * 5
1110: 0 * 200 + 0 * 100 + 1 * 50 + 12 * 20 + 13 * 10 + 16 * 5
1111: 0 * 200 + 0 * 100 + 1 * 50 + 12 * 20 + 14 * 10 + 14 * 5
1112: 0 * 200 + 0 * 100 + 1 * 50 + 12 * 20 + 15 * 10 + 12 * 5
1113: 0 * 200 + 0 * 100 + 1 * 50 + 12 * 20 + 16 * 10 + 10 * 5
1114: 0 * 200 + 0 * 100 + 1 * 50 + 12 * 20 + 17 * 10 + 8 * 5
1115: 0 * 200 + 0 * 100 + 1 * 50 + 12 * 20 + 18 * 10 + 6 * 5
1116: 0 * 200 + 0 * 100 + 1 * 50 + 12 * 20 + 19 * 10 + 4 * 5
1117: 0 * 200 + 0 * 100 + 1 * 50 + 12 * 20 + 20 * 10 + 2 * 5
1118: 0 * 200 + 0 * 100 + 1 * 50 + 12 * 20 + 21 * 10 + 0 * 5
1119: 0 * 200 + 0 * 100 + 1 * 50 + 13 * 20 + 0 * 10 + 38 * 5
1120: 0 * 200 + 0 * 100 + 1 * 50 + 13 * 20 + 1 * 10 + 36 * 5
1121: 0 * 200 + 0 * 100 + 1 * 50 + 13 * 20 + 2 * 10 + 34 * 5
1122: 0 * 200 + 0 * 100 + 1 * 50 + 13 * 20 + 3 * 10 + 32 * 5
1123: 0 * 200 + 0 * 100 + 1 * 50 + 13 * 20 + 4 * 10 + 30 * 5
1124: 0 * 200 + 0 * 100 + 1 * 50 + 13 * 20 + 5 * 10 + 28 * 5
1125: 0 * 200 + 0 * 100 + 1 * 50 + 13 * 20 + 6 * 10 + 26 * 5
1126: 0 * 200 + 0 * 100 + 1 * 50 + 13 * 20 + 7 * 10 + 24 * 5
1127: 0 * 200 + 0 * 100 + 1 * 50 + 13 * 20 + 8 * 10 + 22 * 5
1128: 0 * 200 + 0 * 100 + 1 * 50 + 13 * 20 + 9 * 10 + 20 * 5
1129: 0 * 200 + 0 * 100 + 1 * 50 + 13 * 20 + 10 * 10 + 18 * 5
1130: 0 * 200 + 0 * 100 + 1 * 50 + 13 * 20 + 11 * 10 + 16 * 5
1131: 0 * 200 + 0 * 100 + 1 * 50 + 13 * 20 + 12 * 10 + 14 * 5
1132: 0 * 200 + 0 * 100 + 1 * 50 + 13 * 20 + 13 * 10 + 12 * 5
1133: 0 * 200 + 0 * 100 + 1 * 50 + 13 * 20 + 14 * 10 + 10 * 5
1134: 0 * 200 + 0 * 100 + 1 * 50 + 13 * 20 + 15 * 10 + 8 * 5
1135: 0 * 200 + 0 * 100 + 1 * 50 + 13 * 20 + 16 * 10 + 6 * 5
1136: 0 * 200 + 0 * 100 + 1 * 50 + 13 * 20 + 17 * 10 + 4 * 5
1137: 0 * 200 + 0 * 100 + 1 * 50 + 13 * 20 + 18 * 10 + 2 * 5
1138: 0 * 200 + 0 * 100 + 1 * 50 + 13 * 20 + 19 * 10 + 0 * 5
1139: 0 * 200 + 0 * 100 + 1 * 50 + 14 * 20 + 0 * 10 + 34 * 5
1140: 0 * 200 + 0 * 100 + 1 * 50 + 14 * 20 + 1 * 10 + 32 * 5
1141: 0 * 200 + 0 * 100 + 1 * 50 + 14 * 20 + 2 * 10 + 30 * 5
1142: 0 * 200 + 0 * 100 + 1 * 50 + 14 * 20 + 3 * 10 + 28 * 5
1143: 0 * 200 + 0 * 100 + 1 * 50 + 14 * 20 + 4 * 10 + 26 * 5
1144: 0 * 200 + 0 * 100 + 1 * 50 + 14 * 20 + 5 * 10 + 24 * 5
1145: 0 * 200 + 0 * 100 + 1 * 50 + 14 * 20 + 6 * 10 + 22 * 5
1146: 0 * 200 + 0 * 100 + 1 * 50 + 14 * 20 + 7 * 10 + 20 * 5
1147: 0 * 200 + 0 * 100 + 1 * 50 + 14 * 20 + 8 * 10 + 18 * 5
1148: 0 * 200 + 0 * 100 + 1 * 50 + 14 * 20 + 9 * 10 + 16 * 5
1149: 0 * 200 + 0 * 100 + 1 * 50 + 14 * 20 + 10 * 10 + 14 * 5
1150: 0 * 200 + 0 * 100 + 1 * 50 + 14 * 20 + 11 * 10 + 12 * 5
1151: 0 * 200 + 0 * 100 + 1 * 50 + 14 * 20 + 12 * 10 + 10 * 5
1152: 0 * 200 + 0 * 100 + 1 * 50 + 14 * 20 + 13 * 10 + 8 * 5
1153: 0 * 200 + 0 * 100 + 1 * 50 + 14 * 20 + 14 * 10 + 6 * 5
1154: 0 * 200 + 0 * 100 + 1 * 50 + 14 * 20 + 15 * 10 + 4 * 5
1155: 0 * 200 + 0 * 100 + 1 * 50 + 14 * 20 + 16 * 10 + 2 * 5
1156: 0 * 200 + 0 * 100 + 1 * 50 + 14 * 20 + 17 * 10 + 0 * 5
1157: 0 * 200 + 0 * 100 + 1 * 50 + 15 * 20 + 0 * 10 + 30 * 5
1158: 0 * 200 + 0 * 100 + 1 * 50 + 15 * 20 + 1 * 10 + 28 * 5
1159: 0 * 200 + 0 * 100 + 1 * 50 + 15 * 20 + 2 * 10 + 26 * 5
1160: 0 * 200 + 0 * 100 + 1 * 50 + 15 * 20 + 3 * 10 + 24 * 5
1161: 0 * 200 + 0 * 100 + 1 * 50 + 15 * 20 + 4 * 10 + 22 * 5
1162: 0 * 200 + 0 * 100 + 1 * 50 + 15 * 20 + 5 * 10 + 20 * 5
1163: 0 * 200 + 0 * 100 + 1 * 50 + 15 * 20 + 6 * 10 + 18 * 5
1164: 0 * 200 + 0 * 100 + 1 * 50 + 15 * 20 + 7 * 10 + 16 * 5
1165: 0 * 200 + 0 * 100 + 1 * 50 + 15 * 20 + 8 * 10 + 14 * 5
1166: 0 * 200 + 0 * 100 + 1 * 50 + 15 * 20 + 9 * 10 + 12 * 5
1167: 0 * 200 + 0 * 100 + 1 * 50 + 15 * 20 + 10 * 10 + 10 * 5
1168: 0 * 200 + 0 * 100 + 1 * 50 + 15 * 20 + 11 * 10 + 8 * 5
1169: 0 * 200 + 0 * 100 + 1 * 50 + 15 * 20 + 12 * 10 + 6 * 5
1170: 0 * 200 + 0 * 100 + 1 * 50 + 15 * 20 + 13 * 10 + 4 * 5
1171: 0 * 200 + 0 * 100 + 1 * 50 + 15 * 20 + 14 * 10 + 2 * 5
1172: 0 * 200 + 0 * 100 + 1 * 50 + 15 * 20 + 15 * 10 + 0 * 5
1173: 0 * 200 + 0 * 100 + 1 * 50 + 16 * 20 + 0 * 10 + 26 * 5
1174: 0 * 200 + 0 * 100 + 1 * 50 + 16 * 20 + 1 * 10 + 24 * 5
1175: 0 * 200 + 0 * 100 + 1 * 50 + 16 * 20 + 2 * 10 + 22 * 5
1176: 0 * 200 + 0 * 100 + 1 * 50 + 16 * 20 + 3 * 10 + 20 * 5
1177: 0 * 200 + 0 * 100 + 1 * 50 + 16 * 20 + 4 * 10 + 18 * 5
1178: 0 * 200 + 0 * 100 + 1 * 50 + 16 * 20 + 5 * 10 + 16 * 5
1179: 0 * 200 + 0 * 100 + 1 * 50 + 16 * 20 + 6 * 10 + 14 * 5
1180: 0 * 200 + 0 * 100 + 1 * 50 + 16 * 20 + 7 * 10 + 12 * 5
1181: 0 * 200 + 0 * 100 + 1 * 50 + 16 * 20 + 8 * 10 + 10 * 5
1182: 0 * 200 + 0 * 100 + 1 * 50 + 16 * 20 + 9 * 10 + 8 * 5
1183: 0 * 200 + 0 * 100 + 1 * 50 + 16 * 20 + 10 * 10 + 6 * 5
1184: 0 * 200 + 0 * 100 + 1 * 50 + 16 * 20 + 11 * 10 + 4 * 5
1185: 0 * 200 + 0 * 100 + 1 * 50 + 16 * 20 + 12 * 10 + 2 * 5
1186: 0 * 200 + 0 * 100 + 1 * 50 + 16 * 20 + 13 * 10 + 0 * 5
1187: 0 * 200 + 0 * 100 + 1 * 50 + 17 * 20 + 0 * 10 + 22 * 5
1188: 0 * 200 + 0 * 100 + 1 * 50 + 17 * 20 + 1 * 10 + 20 * 5
1189: 0 * 200 + 0 * 100 + 1 * 50 + 17 * 20 + 2 * 10 + 18 * 5
1190: 0 * 200 + 0 * 100 + 1 * 50 + 17 * 20 + 3 * 10 + 16 * 5
1191: 0 * 200 + 0 * 100 + 1 * 50 + 17 * 20 + 4 * 10 + 14 * 5
1192: 0 * 200 + 0 * 100 + 1 * 50 + 17 * 20 + 5 * 10 + 12 * 5
1193: 0 * 200 + 0 * 100 + 1 * 50 + 17 * 20 + 6 * 10 + 10 * 5
1194: 0 * 200 + 0 * 100 + 1 * 50 + 17 * 20 + 7 * 10 + 8 * 5
1195: 0 * 200 + 0 * 100 + 1 * 50 + 17 * 20 + 8 * 10 + 6 * 5
1196: 0 * 200 + 0 * 100 + 1 * 50 + 17 * 20 + 9 * 10 + 4 * 5
1197: 0 * 200 + 0 * 100 + 1 * 50 + 17 * 20 + 10 * 10 + 2 * 5
1198: 0 * 200 + 0 * 100 + 1 * 50 + 17 * 20 + 11 * 10 + 0 * 5
1199: 0 * 200 + 0 * 100 + 1 * 50 + 18 * 20 + 0 * 10 + 18 * 5
1200: 0 * 200 + 0 * 100 + 1 * 50 + 18 * 20 + 1 * 10 + 16 * 5
1201: 0 * 200 + 0 * 100 + 1 * 50 + 18 * 20 + 2 * 10 + 14 * 5
1202: 0 * 200 + 0 * 100 + 1 * 50 + 18 * 20 + 3 * 10 + 12 * 5
1203: 0 * 200 + 0 * 100 + 1 * 50 + 18 * 20 + 4 * 10 + 10 * 5
1204: 0 * 200 + 0 * 100 + 1 * 50 + 18 * 20 + 5 * 10 + 8 * 5
1205: 0 * 200 + 0 * 100 + 1 * 50 + 18 * 20 + 6 * 10 + 6 * 5
1206: 0 * 200 + 0 * 100 + 1 * 50 + 18 * 20 + 7 * 10 + 4 * 5
1207: 0 * 200 + 0 * 100 + 1 * 50 + 18 * 20 + 8 * 10 + 2 * 5
1208: 0 * 200 + 0 * 100 + 1 * 50 + 18 * 20 + 9 * 10 + 0 * 5
1209: 0 * 200 + 0 * 100 + 1 * 50 + 19 * 20 + 0 * 10 + 14 * 5
1210: 0 * 200 + 0 * 100 + 1 * 50 + 19 * 20 + 1 * 10 + 12 * 5
1211: 0 * 200 + 0 * 100 + 1 * 50 + 19 * 20 + 2 * 10 + 10 * 5
1212: 0 * 200 + 0 * 100 + 1 * 50 + 19 * 20 + 3 * 10 + 8 * 5
1213: 0 * 200 + 0 * 100 + 1 * 50 + 19 * 20 + 4 * 10 + 6 * 5
1214: 0 * 200 + 0 * 100 + 1 * 50 + 19 * 20 + 5 * 10 + 4 * 5
1215: 0 * 200 + 0 * 100 + 1 * 50 + 19 * 20 + 6 * 10 + 2 * 5
1216: 0 * 200 + 0 * 100 + 1 * 50 + 19 * 20 + 7 * 10 + 0 * 5
1217: 0 * 200 + 0 * 100 + 1 * 50 + 20 * 20 + 0 * 10 + 10 * 5
1218: 0 * 200 + 0 * 100 + 1 * 50 + 20 * 20 + 1 * 10 + 8 * 5
1219: 0 * 200 + 0 * 100 + 1 * 50 + 20 * 20 + 2 * 10 + 6 * 5
1220: 0 * 200 + 0 * 100 + 1 * 50 + 20 * 20 + 3 * 10 + 4 * 5
1221: 0 * 200 + 0 * 100 + 1 * 50 + 20 * 20 + 4 * 10 + 2 * 5
1222: 0 * 200 + 0 * 100 + 1 * 50 + 20 * 20 + 5 * 10 + 0 * 5
1223: 0 * 200 + 0 * 100 + 1 * 50 + 21 * 20 + 0 * 10 + 6 * 5
1224: 0 * 200 + 0 * 100 + 1 * 50 + 21 * 20 + 1 * 10 + 4 * 5
1225: 0 * 200 + 0 * 100 + 1 * 50 + 21 * 20 + 2 * 10 + 2 * 5
1226: 0 * 200 + 0 * 100 + 1 * 50 + 21 * 20 + 3 * 10 + 0 * 5
1227: 0 * 200 + 0 * 100 + 1 * 50 + 22 * 20 + 0 * 10 + 2 * 5
1228: 0 * 200 + 0 * 100 + 1 * 50 + 22 * 20 + 1 * 10 + 0 * 5
1229: 0 * 200 + 0 * 100 + 2 * 50 + 0 * 20 + 0 * 10 + 80 * 5
1230: 0 * 200 + 0 * 100 + 2 * 50 + 0 * 20 + 1 * 10 + 78 * 5
1231: 0 * 200 + 0 * 100 + 2 * 50 + 0 * 20 + 2 * 10 + 76 * 5
1232: 0 * 200 + 0 * 100 + 2 * 50 + 0 * 20 + 3 * 10 + 74 * 5
1233: 0 * 200 + 0 * 100 + 2 * 50 + 0 * 20 + 4 * 10 + 72 * 5
1234: 0 * 200 + 0 * 100 + 2 * 50 + 0 * 20 + 5 * 10 + 70 * 5
1235: 0 * 200 + 0 * 100 + 2 * 50 + 0 * 20 + 6 * 10 + 68 * 5
1236: 0 * 200 + 0 * 100 + 2 * 50 + 0 * 20 + 7 * 10 + 66 * 5
1237: 0 * 200 + 0 * 100 + 2 * 50 + 0 * 20 + 8 * 10 + 64 * 5
1238: 0 * 200 + 0 * 100 + 2 * 50 + 0 * 20 + 9 * 10 + 62 * 5
1239: 0 * 200 + 0 * 100 + 2 * 50 + 0 * 20 + 10 * 10 + 60 * 5
1240: 0 * 200 + 0 * 100 + 2 * 50 + 0 * 20 + 11 * 10 + 58 * 5
1241: 0 * 200 + 0 * 100 + 2 * 50 + 0 * 20 + 12 * 10 + 56 * 5
1242: 0 * 200 + 0 * 100 + 2 * 50 + 0 * 20 + 13 * 10 + 54 * 5
1243: 0 * 200 + 0 * 100 + 2 * 50 + 0 * 20 + 14 * 10 + 52 * 5
1244: 0 * 200 + 0 * 100 + 2 * 50 + 0 * 20 + 15 * 10 + 50 * 5
1245: 0 * 200 + 0 * 100 + 2 * 50 + 0 * 20 + 16 * 10 + 48 * 5
1246: 0 * 200 + 0 * 100 + 2 * 50 + 0 * 20 + 17 * 10 + 46 * 5
1247: 0 * 200 + 0 * 100 + 2 * 50 + 0 * 20 + 18 * 10 + 44 * 5
1248: 0 * 200 + 0 * 100 + 2 * 50 + 0 * 20 + 19 * 10 + 42 * 5
1249: 0 * 200 + 0 * 100 + 2 * 50 + 0 * 20 + 20 * 10 + 40 * 5
1250: 0 * 200 + 0 * 100 + 2 * 50 + 0 * 20 + 21 * 10 + 38 * 5
1251: 0 * 200 + 0 * 100 + 2 * 50 + 0 * 20 + 22 * 10 + 36 * 5
1252: 0 * 200 + 0 * 100 + 2 * 50 + 0 * 20 + 23 * 10 + 34 * 5
1253: 0 * 200 + 0 * 100 + 2 * 50 + 0 * 20 + 24 * 10 + 32 * 5
1254: 0 * 200 + 0 * 100 + 2 * 50 + 0 * 20 + 25 * 10 + 30 * 5
1255: 0 * 200 + 0 * 100 + 2 * 50 + 0 * 20 + 26 * 10 + 28 * 5
1256: 0 * 200 + 0 * 100 + 2 * 50 + 0 * 20 + 27 * 10 + 26 * 5
1257: 0 * 200 + 0 * 100 + 2 * 50 + 0 * 20 + 28 * 10 + 24 * 5
1258: 0 * 200 + 0 * 100 + 2 * 50 + 0 * 20 + 29 * 10 + 22 * 5
1259: 0 * 200 + 0 * 100 + 2 * 50 + 0 * 20 + 30 * 10 + 20 * 5
1260: 0 * 200 + 0 * 100 + 2 * 50 + 0 * 20 + 31 * 10 + 18 * 5
1261: 0 * 200 + 0 * 100 + 2 * 50 + 0 * 20 + 32 * 10 + 16 * 5
1262: 0 * 200 + 0 * 100 + 2 * 50 + 0 * 20 + 33 * 10 + 14 * 5
1263: 0 * 200 + 0 * 100 + 2 * 50 + 0 * 20 + 34 * 10 + 12 * 5
1264: 0 * 200 + 0 * 100 + 2 * 50 + 0 * 20 + 35 * 10 + 10 * 5
1265: 0 * 200 + 0 * 100 + 2 * 50 + 0 * 20 + 36 * 10 + 8 * 5
1266: 0 * 200 + 0 * 100 + 2 * 50 + 0 * 20 + 37 * 10 + 6 * 5
1267: 0 * 200 + 0 * 100 + 2 * 50 + 0 * 20 + 38 * 10 + 4 * 5
1268: 0 * 200 + 0 * 100 + 2 * 50 + 0 * 20 + 39 * 10 + 2 * 5
1269: 0 * 200 + 0 * 100 + 2 * 50 + 0 * 20 + 40 * 10 + 0 * 5
1270: 0 * 200 + 0 * 100 + 2 * 50 + 1 * 20 + 0 * 10 + 76 * 5
1271: 0 * 200 + 0 * 100 + 2 * 50 + 1 * 20 + 1 * 10 + 74 * 5
1272: 0 * 200 + 0 * 100 + 2 * 50 + 1 * 20 + 2 * 10 + 72 * 5
1273: 0 * 200 + 0 * 100 + 2 * 50 + 1 * 20 + 3 * 10 + 70 * 5
1274: 0 * 200 + 0 * 100 + 2 * 50 + 1 * 20 + 4 * 10 + 68 * 5
1275: 0 * 200 + 0 * 100 + 2 * 50 + 1 * 20 + 5 * 10 + 66 * 5
1276: 0 * 200 + 0 * 100 + 2 * 50 + 1 * 20 + 6 * 10 + 64 * 5
1277: 0 * 200 + 0 * 100 + 2 * 50 + 1 * 20 + 7 * 10 + 62 * 5
1278: 0 * 200 + 0 * 100 + 2 * 50 + 1 * 20 + 8 * 10 + 60 * 5
1279: 0 * 200 + 0 * 100 + 2 * 50 + 1 * 20 + 9 * 10 + 58 * 5
1280: 0 * 200 + 0 * 100 + 2 * 50 + 1 * 20 + 10 * 10 + 56 * 5
1281: 0 * 200 + 0 * 100 + 2 * 50 + 1 * 20 + 11 * 10 + 54 * 5
1282: 0 * 200 + 0 * 100 + 2 * 50 + 1 * 20 + 12 * 10 + 52 * 5
1283: 0 * 200 + 0 * 100 + 2 * 50 + 1 * 20 + 13 * 10 + 50 * 5
1284: 0 * 200 + 0 * 100 + 2 * 50 + 1 * 20 + 14 * 10 + 48 * 5
1285: 0 * 200 + 0 * 100 + 2 * 50 + 1 * 20 + 15 * 10 + 46 * 5
1286: 0 * 200 + 0 * 100 + 2 * 50 + 1 * 20 + 16 * 10 + 44 * 5
1287: 0 * 200 + 0 * 100 + 2 * 50 + 1 * 20 + 17 * 10 + 42 * 5
1288: 0 * 200 + 0 * 100 + 2 * 50 + 1 * 20 + 18 * 10 + 40 * 5
1289: 0 * 200 + 0 * 100 + 2 * 50 + 1 * 20 + 19 * 10 + 38 * 5
1290: 0 * 200 + 0 * 100 + 2 * 50 + 1 * 20 + 20 * 10 + 36 * 5
1291: 0 * 200 + 0 * 100 + 2 * 50 + 1 * 20 + 21 * 10 + 34 * 5
1292: 0 * 200 + 0 * 100 + 2 * 50 + 1 * 20 + 22 * 10 + 32 * 5
1293: 0 * 200 + 0 * 100 + 2 * 50 + 1 * 20 + 23 * 10 + 30 * 5
1294: 0 * 200 + 0 * 100 + 2 * 50 + 1 * 20 + 24 * 10 + 28 * 5
1295: 0 * 200 + 0 * 100 + 2 * 50 + 1 * 20 + 25 * 10 + 26 * 5
1296: 0 * 200 + 0 * 100 + 2 * 50 + 1 * 20 + 26 * 10 + 24 * 5
1297: 0 * 200 + 0 * 100 + 2 * 50 + 1 * 20 + 27 * 10 + 22 * 5
1298: 0 * 200 + 0 * 100 + 2 * 50 + 1 * 20 + 28 * 10 + 20 * 5
1299: 0 * 200 + 0 * 100 + 2 * 50 + 1 * 20 + 29 * 10 + 18 * 5
1300: 0 * 200 + 0 * 100 + 2 * 50 + 1 * 20 + 30 * 10 + 16 * 5
1301: 0 * 200 + 0 * 100 + 2 * 50 + 1 * 20 + 31 * 10 + 14 * 5
1302: 0 * 200 + 0 * 100 + 2 * 50 + 1 * 20 + 32 * 10 + 12 * 5
1303: 0 * 200 + 0 * 100 + 2 * 50 + 1 * 20 + 33 * 10 + 10 * 5
1304: 0 * 200 + 0 * 100 + 2 * 50 + 1 * 20 + 34 * 10 + 8 * 5
1305: 0 * 200 + 0 * 100 + 2 * 50 + 1 * 20 + 35 * 10 + 6 * 5
1306: 0 * 200 + 0 * 100 + 2 * 50 + 1 * 20 + 36 * 10 + 4 * 5
1307: 0 * 200 + 0 * 100 + 2 * 50 + 1 * 20 + 37 * 10 + 2 * 5
1308: 0 * 200 + 0 * 100 + 2 * 50 + 1 * 20 + 38 * 10 + 0 * 5
1309: 0 * 200 + 0 * 100 + 2 * 50 + 2 * 20 + 0 * 10 + 72 * 5
1310: 0 * 200 + 0 * 100 + 2 * 50 + 2 * 20 + 1 * 10 + 70 * 5
1311: 0 * 200 + 0 * 100 + 2 * 50 + 2 * 20 + 2 * 10 + 68 * 5
1312: 0 * 200 + 0 * 100 + 2 * 50 + 2 * 20 + 3 * 10 + 66 * 5
1313: 0 * 200 + 0 * 100 + 2 * 50 + 2 * 20 + 4 * 10 + 64 * 5
1314: 0 * 200 + 0 * 100 + 2 * 50 + 2 * 20 + 5 * 10 + 62 * 5
1315: 0 * 200 + 0 * 100 + 2 * 50 + 2 * 20 + 6 * 10 + 60 * 5
1316: 0 * 200 + 0 * 100 + 2 * 50 + 2 * 20 + 7 * 10 + 58 * 5
1317: 0 * 200 + 0 * 100 + 2 * 50 + 2 * 20 + 8 * 10 + 56 * 5
1318: 0 * 200 + 0 * 100 + 2 * 50 + 2 * 20 + 9 * 10 + 54 * 5
1319: 0 * 200 + 0 * 100 + 2 * 50 + 2 * 20 + 10 * 10 + 52 * 5
1320: 0 * 200 + 0 * 100 + 2 * 50 + 2 * 20 + 11 * 10 + 50 * 5
1321: 0 * 200 + 0 * 100 + 2 * 50 + 2 * 20 + 12 * 10 + 48 * 5
1322: 0 * 200 + 0 * 100 + 2 * 50 + 2 * 20 + 13 * 10 + 46 * 5
1323: 0 * 200 + 0 * 100 + 2 * 50 + 2 * 20 + 14 * 10 + 44 * 5
1324: 0 * 200 + 0 * 100 + 2 * 50 + 2 * 20 + 15 * 10 + 42 * 5
1325: 0 * 200 + 0 * 100 + 2 * 50 + 2 * 20 + 16 * 10 + 40 * 5
1326: 0 * 200 + 0 * 100 + 2 * 50 + 2 * 20 + 17 * 10 + 38 * 5
1327: 0 * 200 + 0 * 100 + 2 * 50 + 2 * 20 + 18 * 10 + 36 * 5
1328: 0 * 200 + 0 * 100 + 2 * 50 + 2 * 20 + 19 * 10 + 34 * 5
1329: 0 * 200 + 0 * 100 + 2 * 50 + 2 * 20 + 20 * 10 + 32 * 5
1330: 0 * 200 + 0 * 100 + 2 * 50 + 2 * 20 + 21 * 10 + 30 * 5
1331: 0 * 200 + 0 * 100 + 2 * 50 + 2 * 20 + 22 * 10 + 28 * 5
1332: 0 * 200 + 0 * 100 + 2 * 50 + 2 * 20 + 23 * 10 + 26 * 5
1333: 0 * 200 + 0 * 100 + 2 * 50 + 2 * 20 + 24 * 10 + 24 * 5
1334: 0 * 200 + 0 * 100 + 2 * 50 + 2 * 20 + 25 * 10 + 22 * 5
1335: 0 * 200 + 0 * 100 + 2 * 50 + 2 * 20 + 26 * 10 + 20 * 5
1336: 0 * 200 + 0 * 100 + 2 * 50 + 2 * 20 + 27 * 10 + 18 * 5
1337: 0 * 200 + 0 * 100 + 2 * 50 + 2 * 20 + 28 * 10 + 16 * 5
1338: 0 * 200 + 0 * 100 + 2 * 50 + 2 * 20 + 29 * 10 + 14 * 5
1339: 0 * 200 + 0 * 100 + 2 * 50 + 2 * 20 + 30 * 10 + 12 * 5
1340: 0 * 200 + 0 * 100 + 2 * 50 + 2 * 20 + 31 * 10 + 10 * 5
1341: 0 * 200 + 0 * 100 + 2 * 50 + 2 * 20 + 32 * 10 + 8 * 5
1342: 0 * 200 + 0 * 100 + 2 * 50 + 2 * 20 + 33 * 10 + 6 * 5
1343: 0 * 200 + 0 * 100 + 2 * 50 + 2 * 20 + 34 * 10 + 4 * 5
1344: 0 * 200 + 0 * 100 + 2 * 50 + 2 * 20 + 35 * 10 + 2 * 5
1345: 0 * 200 + 0 * 100 + 2 * 50 + 2 * 20 + 36 * 10 + 0 * 5
1346: 0 * 200 + 0 * 100 + 2 * 50 + 3 * 20 + 0 * 10 + 68 * 5
1347: 0 * 200 + 0 * 100 + 2 * 50 + 3 * 20 + 1 * 10 + 66 * 5
1348: 0 * 200 + 0 * 100 + 2 * 50 + 3 * 20 + 2 * 10 + 64 * 5
1349: 0 * 200 + 0 * 100 + 2 * 50 + 3 * 20 + 3 * 10 + 62 * 5
1350: 0 * 200 + 0 * 100 + 2 * 50 + 3 * 20 + 4 * 10 + 60 * 5
1351: 0 * 200 + 0 * 100 + 2 * 50 + 3 * 20 + 5 * 10 + 58 * 5
1352: 0 * 200 + 0 * 100 + 2 * 50 + 3 * 20 + 6 * 10 + 56 * 5
1353: 0 * 200 + 0 * 100 + 2 * 50 + 3 * 20 + 7 * 10 + 54 * 5
1354: 0 * 200 + 0 * 100 + 2 * 50 + 3 * 20 + 8 * 10 + 52 * 5
1355: 0 * 200 + 0 * 100 + 2 * 50 + 3 * 20 + 9 * 10 + 50 * 5
1356: 0 * 200 + 0 * 100 + 2 * 50 + 3 * 20 + 10 * 10 + 48 * 5
1357: 0 * 200 + 0 * 100 + 2 * 50 + 3 * 20 + 11 * 10 + 46 * 5
1358: 0 * 200 + 0 * 100 + 2 * 50 + 3 * 20 + 12 * 10 + 44 * 5
1359: 0 * 200 + 0 * 100 + 2 * 50 + 3 * 20 + 13 * 10 + 42 * 5
1360: 0 * 200 + 0 * 100 + 2 * 50 + 3 * 20 + 14 * 10 + 40 * 5
1361: 0 * 200 + 0 * 100 + 2 * 50 + 3 * 20 + 15 * 10 + 38 * 5
1362: 0 * 200 + 0 * 100 + 2 * 50 + 3 * 20 + 16 * 10 + 36 * 5
1363: 0 * 200 + 0 * 100 + 2 * 50 + 3 * 20 + 17 * 10 + 34 * 5
1364: 0 * 200 + 0 * 100 + 2 * 50 + 3 * 20 + 18 * 10 + 32 * 5
1365: 0 * 200 + 0 * 100 + 2 * 50 + 3 * 20 + 19 * 10 + 30 * 5
1366: 0 * 200 + 0 * 100 + 2 * 50 + 3 * 20 + 20 * 10 + 28 * 5
1367: 0 * 200 + 0 * 100 + 2 * 50 + 3 * 20 + 21 * 10 + 26 * 5
1368: 0 * 200 + 0 * 100 + 2 * 50 + 3 * 20 + 22 * 10 + 24 * 5
1369: 0 * 200 + 0 * 100 + 2 * 50 + 3 * 20 + 23 * 10 + 22 * 5
1370: 0 * 200 + 0 * 100 + 2 * 50 + 3 * 20 + 24 * 10 + 20 * 5
1371: 0 * 200 + 0 * 100 + 2 * 50 + 3 * 20 + 25 * 10 + 18 * 5
1372: 0 * 200 + 0 * 100 + 2 * 50 + 3 * 20 + 26 * 10 + 16 * 5
1373: 0 * 200 + 0 * 100 + 2 * 50 + 3 * 20 + 27 * 10 + 14 * 5
1374: 0 * 200 + 0 * 100 + 2 * 50 + 3 * 20 + 28 * 10 + 12 * 5
1375: 0 * 200 + 0 * 100 + 2 * 50 + 3 * 20 + 29 * 10 + 10 * 5
1376: 0 * 200 + 0 * 100 + 2 * 50 + 3 * 20 + 30 * 10 + 8 * 5
1377: 0 * 200 + 0 * 100 + 2 * 50 + 3 * 20 + 31 * 10 + 6 * 5
1378: 0 * 200 + 0 * 100 + 2 * 50 + 3 * 20 + 32 * 10 + 4 * 5
1379: 0 * 200 + 0 * 100 + 2 * 50 + 3 * 20 + 33 * 10 + 2 * 5
1380: 0 * 200 + 0 * 100 + 2 * 50 + 3 * 20 + 34 * 10 + 0 * 5
1381: 0 * 200 + 0 * 100 + 2 * 50 + 4 * 20 + 0 * 10 + 64 * 5
1382: 0 * 200 + 0 * 100 + 2 * 50 + 4 * 20 + 1 * 10 + 62 * 5
1383: 0 * 200 + 0 * 100 + 2 * 50 + 4 * 20 + 2 * 10 + 60 * 5
1384: 0 * 200 + 0 * 100 + 2 * 50 + 4 * 20 + 3 * 10 + 58 * 5
1385: 0 * 200 + 0 * 100 + 2 * 50 + 4 * 20 + 4 * 10 + 56 * 5
1386: 0 * 200 + 0 * 100 + 2 * 50 + 4 * 20 + 5 * 10 + 54 * 5
1387: 0 * 200 + 0 * 100 + 2 * 50 + 4 * 20 + 6 * 10 + 52 * 5
1388: 0 * 200 + 0 * 100 + 2 * 50 + 4 * 20 + 7 * 10 + 50 * 5
1389: 0 * 200 + 0 * 100 + 2 * 50 + 4 * 20 + 8 * 10 + 48 * 5
1390: 0 * 200 + 0 * 100 + 2 * 50 + 4 * 20 + 9 * 10 + 46 * 5
1391: 0 * 200 + 0 * 100 + 2 * 50 + 4 * 20 + 10 * 10 + 44 * 5
1392: 0 * 200 + 0 * 100 + 2 * 50 + 4 * 20 + 11 * 10 + 42 * 5
1393: 0 * 200 + 0 * 100 + 2 * 50 + 4 * 20 + 12 * 10 + 40 * 5
1394: 0 * 200 + 0 * 100 + 2 * 50 + 4 * 20 + 13 * 10 + 38 * 5
1395: 0 * 200 + 0 * 100 + 2 * 50 + 4 * 20 + 14 * 10 + 36 * 5
1396: 0 * 200 + 0 * 100 + 2 * 50 + 4 * 20 + 15 * 10 + 34 * 5
1397: 0 * 200 + 0 * 100 + 2 * 50 + 4 * 20 + 16 * 10 + 32 * 5
1398: 0 * 200 + 0 * 100 + 2 * 50 + 4 * 20 + 17 * 10 + 30 * 5
1399: 0 * 200 + 0 * 100 + 2 * 50 + 4 * 20 + 18 * 10 + 28 * 5
1400: 0 * 200 + 0 * 100 + 2 * 50 + 4 * 20 + 19 * 10 + 26 * 5
1401: 0 * 200 + 0 * 100 + 2 * 50 + 4 * 20 + 20 * 10 + 24 * 5
1402: 0 * 200 + 0 * 100 + 2 * 50 + 4 * 20 + 21 * 10 + 22 * 5
1403: 0 * 200 + 0 * 100 + 2 * 50 + 4 * 20 + 22 * 10 + 20 * 5
1404: 0 * 200 + 0 * 100 + 2 * 50 + 4 * 20 + 23 * 10 + 18 * 5
1405: 0 * 200 + 0 * 100 + 2 * 50 + 4 * 20 + 24 * 10 + 16 * 5
1406: 0 * 200 + 0 * 100 + 2 * 50 + 4 * 20 + 25 * 10 + 14 * 5
1407: 0 * 200 + 0 * 100 + 2 * 50 + 4 * 20 + 26 * 10 + 12 * 5
1408: 0 * 200 + 0 * 100 + 2 * 50 + 4 * 20 + 27 * 10 + 10 * 5
1409: 0 * 200 + 0 * 100 + 2 * 50 + 4 * 20 + 28 * 10 + 8 * 5
1410: 0 * 200 + 0 * 100 + 2 * 50 + 4 * 20 + 29 * 10 + 6 * 5
1411: 0 * 200 + 0 * 100 + 2 * 50 + 4 * 20 + 30 * 10 + 4 * 5
1412: 0 * 200 + 0 * 100 + 2 * 50 + 4 * 20 + 31 * 10 + 2 * 5
1413: 0 * 200 + 0 * 100 + 2 * 50 + 4 * 20 + 32 * 10 + 0 * 5
1414: 0 * 200 + 0 * 100 + 2 * 50 + 5 * 20 + 0 * 10 + 60 * 5
1415: 0 * 200 + 0 * 100 + 2 * 50 + 5 * 20 + 1 * 10 + 58 * 5
1416: 0 * 200 + 0 * 100 + 2 * 50 + 5 * 20 + 2 * 10 + 56 * 5
1417: 0 * 200 + 0 * 100 + 2 * 50 + 5 * 20 + 3 * 10 + 54 * 5
1418: 0 * 200 + 0 * 100 + 2 * 50 + 5 * 20 + 4 * 10 + 52 * 5
1419: 0 * 200 + 0 * 100 + 2 * 50 + 5 * 20 + 5 * 10 + 50 * 5
1420: 0 * 200 + 0 * 100 + 2 * 50 + 5 * 20 + 6 * 10 + 48 * 5
1421: 0 * 200 + 0 * 100 + 2 * 50 + 5 * 20 + 7 * 10 + 46 * 5
1422: 0 * 200 + 0 * 100 + 2 * 50 + 5 * 20 + 8 * 10 + 44 * 5
1423: 0 * 200 + 0 * 100 + 2 * 50 + 5 * 20 + 9 * 10 + 42 * 5
1424: 0 * 200 + 0 * 100 + 2 * 50 + 5 * 20 + 10 * 10 + 40 * 5
1425: 0 * 200 + 0 * 100 + 2 * 50 + 5 * 20 + 11 * 10 + 38 * 5
1426: 0 * 200 + 0 * 100 + 2 * 50 + 5 * 20 + 12 * 10 + 36 * 5
1427: 0 * 200 + 0 * 100 + 2 * 50 + 5 * 20 + 13 * 10 + 34 * 5
1428: 0 * 200 + 0 * 100 + 2 * 50 + 5 * 20 + 14 * 10 + 32 * 5
1429: 0 * 200 + 0 * 100 + 2 * 50 + 5 * 20 + 15 * 10 + 30 * 5
1430: 0 * 200 + 0 * 100 + 2 * 50 + 5 * 20 + 16 * 10 + 28 * 5
1431: 0 * 200 + 0 * 100 + 2 * 50 + 5 * 20 + 17 * 10 + 26 * 5
1432: 0 * 200 + 0 * 100 + 2 * 50 + 5 * 20 + 18 * 10 + 24 * 5
1433: 0 * 200 + 0 * 100 + 2 * 50 + 5 * 20 + 19 * 10 + 22 * 5
1434: 0 * 200 + 0 * 100 + 2 * 50 + 5 * 20 + 20 * 10 + 20 * 5
1435: 0 * 200 + 0 * 100 + 2 * 50 + 5 * 20 + 21 * 10 + 18 * 5
1436: 0 * 200 + 0 * 100 + 2 * 50 + 5 * 20 + 22 * 10 + 16 * 5
1437: 0 * 200 + 0 * 100 + 2 * 50 + 5 * 20 + 23 * 10 + 14 * 5
1438: 0 * 200 + 0 * 100 + 2 * 50 + 5 * 20 + 24 * 10 + 12 * 5
1439: 0 * 200 + 0 * 100 + 2 * 50 + 5 * 20 + 25 * 10 + 10 * 5
1440: 0 * 200 + 0 * 100 + 2 * 50 + 5 * 20 + 26 * 10 + 8 * 5
1441: 0 * 200 + 0 * 100 + 2 * 50 + 5 * 20 + 27 * 10 + 6 * 5
1442: 0 * 200 + 0 * 100 + 2 * 50 + 5 * 20 + 28 * 10 + 4 * 5
1443: 0 * 200 + 0 * 100 + 2 * 50 + 5 * 20 + 29 * 10 + 2 * 5
1444: 0 * 200 + 0 * 100 + 2 * 50 + 5 * 20 + 30 * 10 + 0 * 5
1445: 0 * 200 + 0 * 100 + 2 * 50 + 6 * 20 + 0 * 10 + 56 * 5
1446: 0 * 200 + 0 * 100 + 2 * 50 + 6 * 20 + 1 * 10 + 54 * 5
1447: 0 * 200 + 0 * 100 + 2 * 50 + 6 * 20 + 2 * 10 + 52 * 5
1448: 0 * 200 + 0 * 100 + 2 * 50 + 6 * 20 + 3 * 10 + 50 * 5
1449: 0 * 200 + 0 * 100 + 2 * 50 + 6 * 20 + 4 * 10 + 48 * 5
1450: 0 * 200 + 0 * 100 + 2 * 50 + 6 * 20 + 5 * 10 + 46 * 5
1451: 0 * 200 + 0 * 100 + 2 * 50 + 6 * 20 + 6 * 10 + 44 * 5
1452: 0 * 200 + 0 * 100 + 2 * 50 + 6 * 20 + 7 * 10 + 42 * 5
1453: 0 * 200 + 0 * 100 + 2 * 50 + 6 * 20 + 8 * 10 + 40 * 5
1454: 0 * 200 + 0 * 100 + 2 * 50 + 6 * 20 + 9 * 10 + 38 * 5
1455: 0 * 200 + 0 * 100 + 2 * 50 + 6 * 20 + 10 * 10 + 36 * 5
1456: 0 * 200 + 0 * 100 + 2 * 50 + 6 * 20 + 11 * 10 + 34 * 5
1457: 0 * 200 + 0 * 100 + 2 * 50 + 6 * 20 + 12 * 10 + 32 * 5
1458: 0 * 200 + 0 * 100 + 2 * 50 + 6 * 20 + 13 * 10 + 30 * 5
1459: 0 * 200 + 0 * 100 + 2 * 50 + 6 * 20 + 14 * 10 + 28 * 5
1460: 0 * 200 + 0 * 100 + 2 * 50 + 6 * 20 + 15 * 10 + 26 * 5
1461: 0 * 200 + 0 * 100 + 2 * 50 + 6 * 20 + 16 * 10 + 24 * 5
1462: 0 * 200 + 0 * 100 + 2 * 50 + 6 * 20 + 17 * 10 + 22 * 5
1463: 0 * 200 + 0 * 100 + 2 * 50 + 6 * 20 + 18 * 10 + 20 * 5
1464: 0 * 200 + 0 * 100 + 2 * 50 + 6 * 20 + 19 * 10 + 18 * 5
1465: 0 * 200 + 0 * 100 + 2 * 50 + 6 * 20 + 20 * 10 + 16 * 5
1466: 0 * 200 + 0 * 100 + 2 * 50 + 6 * 20 + 21 * 10 + 14 * 5
1467: 0 * 200 + 0 * 100 + 2 * 50 + 6 * 20 + 22 * 10 + 12 * 5
1468: 0 * 200 + 0 * 100 + 2 * 50 + 6 * 20 + 23 * 10 + 10 * 5
1469: 0 * 200 + 0 * 100 + 2 * 50 + 6 * 20 + 24 * 10 + 8 * 5
1470: 0 * 200 + 0 * 100 + 2 * 50 + 6 * 20 + 25 * 10 + 6 * 5
1471: 0 * 200 + 0 * 100 + 2 * 50 + 6 * 20 + 26 * 10 + 4 * 5
1472: 0 * 200 + 0 * 100 + 2 * 50 + 6 * 20 + 27 * 10 + 2 * 5
1473: 0 * 200 + 0 * 100 + 2 * 50 + 6 * 20 + 28 * 10 + 0 * 5
1474: 0 * 200 + 0 * 100 + 2 * 50 + 7 * 20 + 0 * 10 + 52 * 5
1475: 0 * 200 + 0 * 100 + 2 * 50 + 7 * 20 + 1 * 10 + 50 * 5
1476: 0 * 200 + 0 * 100 + 2 * 50 + 7 * 20 + 2 * 10 + 48 * 5
1477: 0 * 200 + 0 * 100 + 2 * 50 + 7 * 20 + 3 * 10 + 46 * 5
1478: 0 * 200 + 0 * 100 + 2 * 50 + 7 * 20 + 4 * 10 + 44 * 5
1479: 0 * 200 + 0 * 100 + 2 * 50 + 7 * 20 + 5 * 10 + 42 * 5
1480: 0 * 200 + 0 * 100 + 2 * 50 + 7 * 20 + 6 * 10 + 40 * 5
1481: 0 * 200 + 0 * 100 + 2 * 50 + 7 * 20 + 7 * 10 + 38 * 5
1482: 0 * 200 + 0 * 100 + 2 * 50 + 7 * 20 + 8 * 10 + 36 * 5
1483: 0 * 200 + 0 * 100 + 2 * 50 + 7 * 20 + 9 * 10 + 34 * 5
1484: 0 * 200 + 0 * 100 + 2 * 50 + 7 * 20 + 10 * 10 + 32 * 5
1485: 0 * 200 + 0 * 100 + 2 * 50 + 7 * 20 + 11 * 10 + 30 * 5
1486: 0 * 200 + 0 * 100 + 2 * 50 + 7 * 20 + 12 * 10 + 28 * 5
1487: 0 * 200 + 0 * 100 + 2 * 50 + 7 * 20 + 13 * 10 + 26 * 5
1488: 0 * 200 + 0 * 100 + 2 * 50 + 7 * 20 + 14 * 10 + 24 * 5
1489: 0 * 200 + 0 * 100 + 2 * 50 + 7 * 20 + 15 * 10 + 22 * 5
1490: 0 * 200 + 0 * 100 + 2 * 50 + 7 * 20 + 16 * 10 + 20 * 5
1491: 0 * 200 + 0 * 100 + 2 * 50 + 7 * 20 + 17 * 10 + 18 * 5
1492: 0 * 200 + 0 * 100 + 2 * 50 + 7 * 20 + 18 * 10 + 16 * 5
1493: 0 * 200 + 0 * 100 + 2 * 50 + 7 * 20 + 19 * 10 + 14 * 5
1494: 0 * 200 + 0 * 100 + 2 * 50 + 7 * 20 + 20 * 10 + 12 * 5
1495: 0 * 200 + 0 * 100 + 2 * 50 + 7 * 20 + 21 * 10 + 10 * 5
1496: 0 * 200 + 0 * 100 + 2 * 50 + 7 * 20 + 22 * 10 + 8 * 5
1497: 0 * 200 + 0 * 100 + 2 * 50 + 7 * 20 + 23 * 10 + 6 * 5
1498: 0 * 200 + 0 * 100 + 2 * 50 + 7 * 20 + 24 * 10 + 4 * 5
1499: 0 * 200 + 0 * 100 + 2 * 50 + 7 * 20 + 25 * 10 + 2 * 5
1500: 0 * 200 + 0 * 100 + 2 * 50 + 7 * 20 + 26 * 10 + 0 * 5
1501: 0 * 200 + 0 * 100 + 2 * 50 + 8 * 20 + 0 * 10 + 48 * 5
1502: 0 * 200 + 0 * 100 + 2 * 50 + 8 * 20 + 1 * 10 + 46 * 5
1503: 0 * 200 + 0 * 100 + 2 * 50 + 8 * 20 + 2 * 10 + 44 * 5
1504: 0 * 200 + 0 * 100 + 2 * 50 + 8 * 20 + 3 * 10 + 42 * 5
1505: 0 * 200 + 0 * 100 + 2 * 50 + 8 * 20 + 4 * 10 + 40 * 5
1506: 0 * 200 + 0 * 100 + 2 * 50 + 8 * 20 + 5 * 10 + 38 * 5
1507: 0 * 200 + 0 * 100 + 2 * 50 + 8 * 20 + 6 * 10 + 36 * 5
1508: 0 * 200 + 0 * 100 + 2 * 50 + 8 * 20 + 7 * 10 + 34 * 5
1509: 0 * 200 + 0 * 100 + 2 * 50 + 8 * 20 + 8 * 10 + 32 * 5
1510: 0 * 200 + 0 * 100 + 2 * 50 + 8 * 20 + 9 * 10 + 30 * 5
1511: 0 * 200 + 0 * 100 + 2 * 50 + 8 * 20 + 10 * 10 + 28 * 5
1512: 0 * 200 + 0 * 100 + 2 * 50 + 8 * 20 + 11 * 10 + 26 * 5
1513: 0 * 200 + 0 * 100 + 2 * 50 + 8 * 20 + 12 * 10 + 24 * 5
1514: 0 * 200 + 0 * 100 + 2 * 50 + 8 * 20 + 13 * 10 + 22 * 5
1515: 0 * 200 + 0 * 100 + 2 * 50 + 8 * 20 + 14 * 10 + 20 * 5
1516: 0 * 200 + 0 * 100 + 2 * 50 + 8 * 20 + 15 * 10 + 18 * 5
1517: 0 * 200 + 0 * 100 + 2 * 50 + 8 * 20 + 16 * 10 + 16 * 5
1518: 0 * 200 + 0 * 100 + 2 * 50 + 8 * 20 + 17 * 10 + 14 * 5
1519: 0 * 200 + 0 * 100 + 2 * 50 + 8 * 20 + 18 * 10 + 12 * 5
1520: 0 * 200 + 0 * 100 + 2 * 50 + 8 * 20 + 19 * 10 + 10 * 5
1521: 0 * 200 + 0 * 100 + 2 * 50 + 8 * 20 + 20 * 10 + 8 * 5
1522: 0 * 200 + 0 * 100 + 2 * 50 + 8 * 20 + 21 * 10 + 6 * 5
1523: 0 * 200 + 0 * 100 + 2 * 50 + 8 * 20 + 22 * 10 + 4 * 5
1524: 0 * 200 + 0 * 100 + 2 * 50 + 8 * 20 + 23 * 10 + 2 * 5
1525: 0 * 200 + 0 * 100 + 2 * 50 + 8 * 20 + 24 * 10 + 0 * 5
1526: 0 * 200 + 0 * 100 + 2 * 50 + 9 * 20 + 0 * 10 + 44 * 5
1527: 0 * 200 + 0 * 100 + 2 * 50 + 9 * 20 + 1 * 10 + 42 * 5
1528: 0 * 200 + 0 * 100 + 2 * 50 + 9 * 20 + 2 * 10 + 40 * 5
1529: 0 * 200 + 0 * 100 + 2 * 50 + 9 * 20 + 3 * 10 + 38 * 5
1530: 0 * 200 + 0 * 100 + 2 * 50 + 9 * 20 + 4 * 10 + 36 * 5
1531: 0 * 200 + 0 * 100 + 2 * 50 + 9 * 20 + 5 * 10 + 34 * 5
1532: 0 * 200 + 0 * 100 + 2 * 50 + 9 * 20 + 6 * 10 + 32 * 5
1533: 0 * 200 + 0 * 100 + 2 * 50 + 9 * 20 + 7 * 10 + 30 * 5
1534: 0 * 200 + 0 * 100 + 2 * 50 + 9 * 20 + 8 * 10 + 28 * 5
1535: 0 * 200 + 0 * 100 + 2 * 50 + 9 * 20 + 9 * 10 + 26 * 5
1536: 0 * 200 + 0 * 100 + 2 * 50 + 9 * 20 + 10 * 10 + 24 * 5
1537: 0 * 200 + 0 * 100 + 2 * 50 + 9 * 20 + 11 * 10 + 22 * 5
1538: 0 * 200 + 0 * 100 + 2 * 50 + 9 * 20 + 12 * 10 + 20 * 5
1539: 0 * 200 + 0 * 100 + 2 * 50 + 9 * 20 + 13 * 10 + 18 * 5
1540: 0 * 200 + 0 * 100 + 2 * 50 + 9 * 20 + 14 * 10 + 16 * 5
1541: 0 * 200 + 0 * 100 + 2 * 50 + 9 * 20 + 15 * 10 + 14 * 5
1542: 0 * 200 + 0 * 100 + 2 * 50 + 9 * 20 + 16 * 10 + 12 * 5
1543: 0 * 200 + 0 * 100 + 2 * 50 + 9 * 20 + 17 * 10 + 10 * 5
1544: 0 * 200 + 0 * 100 + 2 * 50 + 9 * 20 + 18 * 10 + 8 * 5
1545: 0 * 200 + 0 * 100 + 2 * 50 + 9 * 20 + 19 * 10 + 6 * 5
1546: 0 * 200 + 0 * 100 + 2 * 50 + 9 * 20 + 20 * 10 + 4 * 5
1547: 0 * 200 + 0 * 100 + 2 * 50 + 9 * 20 + 21 * 10 + 2 * 5
1548: 0 * 200 + 0 * 100 + 2 * 50 + 9 * 20 + 22 * 10 + 0 * 5
1549: 0 * 200 + 0 * 100 + 2 * 50 + 10 * 20 + 0 * 10 + 40 * 5
1550: 0 * 200 + 0 * 100 + 2 * 50 + 10 * 20 + 1 * 10 + 38 * 5
1551: 0 * 200 + 0 * 100 + 2 * 50 + 10 * 20 + 2 * 10 + 36 * 5
1552: 0 * 200 + 0 * 100 + 2 * 50 + 10 * 20 + 3 * 10 + 34 * 5
1553: 0 * 200 + 0 * 100 + 2 * 50 + 10 * 20 + 4 * 10 + 32 * 5
1554: 0 * 200 + 0 * 100 + 2 * 50 + 10 * 20 + 5 * 10 + 30 * 5
1555: 0 * 200 + 0 * 100 + 2 * 50 + 10 * 20 + 6 * 10 + 28 * 5
1556: 0 * 200 + 0 * 100 + 2 * 50 + 10 * 20 + 7 * 10 + 26 * 5
1557: 0 * 200 + 0 * 100 + 2 * 50 + 10 * 20 + 8 * 10 + 24 * 5
1558: 0 * 200 + 0 * 100 + 2 * 50 + 10 * 20 + 9 * 10 + 22 * 5
1559: 0 * 200 + 0 * 100 + 2 * 50 + 10 * 20 + 10 * 10 + 20 * 5
1560: 0 * 200 + 0 * 100 + 2 * 50 + 10 * 20 + 11 * 10 + 18 * 5
1561: 0 * 200 + 0 * 100 + 2 * 50 + 10 * 20 + 12 * 10 + 16 * 5
1562: 0 * 200 + 0 * 100 + 2 * 50 + 10 * 20 + 13 * 10 + 14 * 5
1563: 0 * 200 + 0 * 100 + 2 * 50 + 10 * 20 + 14 * 10 + 12 * 5
1564: 0 * 200 + 0 * 100 + 2 * 50 + 10 * 20 + 15 * 10 + 10 * 5
1565: 0 * 200 + 0 * 100 + 2 * 50 + 10 * 20 + 16 * 10 + 8 * 5
1566: 0 * 200 + 0 * 100 + 2 * 50 + 10 * 20 + 17 * 10 + 6 * 5
1567: 0 * 200 + 0 * 100 + 2 * 50 + 10 * 20 + 18 * 10 + 4 * 5
1568: 0 * 200 + 0 * 100 + 2 * 50 + 10 * 20 + 19 * 10 + 2 * 5
1569: 0 * 200 + 0 * 100 + 2 * 50 + 10 * 20 + 20 * 10 + 0 * 5
1570: 0 * 200 + 0 * 100 + 2 * 50 + 11 * 20 + 0 * 10 + 36 * 5
1571: 0 * 200 + 0 * 100 + 2 * 50 + 11 * 20 + 1 * 10 + 34 * 5
1572: 0 * 200 + 0 * 100 + 2 * 50 + 11 * 20 + 2 * 10 + 32 * 5
1573: 0 * 200 + 0 * 100 + 2 * 50 + 11 * 20 + 3 * 10 + 30 * 5
1574: 0 * 200 + 0 * 100 + 2 * 50 + 11 * 20 + 4 * 10 + 28 * 5
1575: 0 * 200 + 0 * 100 + 2 * 50 + 11 * 20 + 5 * 10 + 26 * 5
1576: 0 * 200 + 0 * 100 + 2 * 50 + 11 * 20 + 6 * 10 + 24 * 5
1577: 0 * 200 + 0 * 100 + 2 * 50 + 11 * 20 + 7 * 10 + 22 * 5
1578: 0 * 200 + 0 * 100 + 2 * 50 + 11 * 20 + 8 * 10 + 20 * 5
1579: 0 * 200 + 0 * 100 + 2 * 50 + 11 * 20 + 9 * 10 + 18 * 5
1580: 0 * 200 + 0 * 100 + 2 * 50 + 11 * 20 + 10 * 10 + 16 * 5
1581: 0 * 200 + 0 * 100 + 2 * 50 + 11 * 20 + 11 * 10 + 14 * 5
1582: 0 * 200 + 0 * 100 + 2 * 50 + 11 * 20 + 12 * 10 + 12 * 5
1583: 0 * 200 + 0 * 100 + 2 * 50 + 11 * 20 + 13 * 10 + 10 * 5
1584: 0 * 200 + 0 * 100 + 2 * 50 + 11 * 20 + 14 * 10 + 8 * 5
1585: 0 * 200 + 0 * 100 + 2 * 50 + 11 * 20 + 15 * 10 + 6 * 5
1586: 0 * 200 + 0 * 100 + 2 * 50 + 11 * 20 + 16 * 10 + 4 * 5
1587: 0 * 200 + 0 * 100 + 2 * 50 + 11 * 20 + 17 * 10 + 2 * 5
1588: 0 * 200 + 0 * 100 + 2 * 50 + 11 * 20 + 18 * 10 + 0 * 5
1589: 0 * 200 + 0 * 100 + 2 * 50 + 12 * 20 + 0 * 10 + 32 * 5
1590: 0 * 200 + 0 * 100 + 2 * 50 + 12 * 20 + 1 * 10 + 30 * 5
1591: 0 * 200 + 0 * 100 + 2 * 50 + 12 * 20 + 2 * 10 + 28 * 5
1592: 0 * 200 + 0 * 100 + 2 * 50 + 12 * 20 + 3 * 10 + 26 * 5
1593: 0 * 200 + 0 * 100 + 2 * 50 + 12 * 20 + 4 * 10 + 24 * 5
1594: 0 * 200 + 0 * 100 + 2 * 50 + 12 * 20 + 5 * 10 + 22 * 5
1595: 0 * 200 + 0 * 100 + 2 * 50 + 12 * 20 + 6 * 10 + 20 * 5
1596: 0 * 200 + 0 * 100 + 2 * 50 + 12 * 20 + 7 * 10 + 18 * 5
1597: 0 * 200 + 0 * 100 + 2 * 50 + 12 * 20 + 8 * 10 + 16 * 5
1598: 0 * 200 + 0 * 100 + 2 * 50 + 12 * 20 + 9 * 10 + 14 * 5
1599: 0 * 200 + 0 * 100 + 2 * 50 + 12 * 20 + 10 * 10 + 12 * 5
1600: 0 * 200 + 0 * 100 + 2 * 50 + 12 * 20 + 11 * 10 + 10 * 5
1601: 0 * 200 + 0 * 100 + 2 * 50 + 12 * 20 + 12 * 10 + 8 * 5
1602: 0 * 200 + 0 * 100 + 2 * 50 + 12 * 20 + 13 * 10 + 6 * 5
1603: 0 * 200 + 0 * 100 + 2 * 50 + 12 * 20 + 14 * 10 + 4 * 5
1604: 0 * 200 + 0 * 100 + 2 * 50 + 12 * 20 + 15 * 10 + 2 * 5
1605: 0 * 200 + 0 * 100 + 2 * 50 + 12 * 20 + 16 * 10 + 0 * 5
1606: 0 * 200 + 0 * 100 + 2 * 50 + 13 * 20 + 0 * 10 + 28 * 5
1607: 0 * 200 + 0 * 100 + 2 * 50 + 13 * 20 + 1 * 10 + 26 * 5
1608: 0 * 200 + 0 * 100 + 2 * 50 + 13 * 20 + 2 * 10 + 24 * 5
1609: 0 * 200 + 0 * 100 + 2 * 50 + 13 * 20 + 3 * 10 + 22 * 5
1610: 0 * 200 + 0 * 100 + 2 * 50 + 13 * 20 + 4 * 10 + 20 * 5
1611: 0 * 200 + 0 * 100 + 2 * 50 + 13 * 20 + 5 * 10 + 18 * 5
1612: 0 * 200 + 0 * 100 + 2 * 50 + 13 * 20 + 6 * 10 + 16 * 5
1613: 0 * 200 + 0 * 100 + 2 * 50 + 13 * 20 + 7 * 10 + 14 * 5
1614: 0 * 200 + 0 * 100 + 2 * 50 + 13 * 20 + 8 * 10 + 12 * 5
1615: 0 * 200 + 0 * 100 + 2 * 50 + 13 * 20 + 9 * 10 + 10 * 5
1616: 0 * 200 + 0 * 100 + 2 * 50 + 13 * 20 + 10 * 10 + 8 * 5
1617: 0 * 200 + 0 * 100 + 2 * 50 + 13 * 20 + 11 * 10 + 6 * 5
1618: 0 * 200 + 0 * 100 + 2 * 50 + 13 * 20 + 12 * 10 + 4 * 5
1619: 0 * 200 + 0 * 100 + 2 * 50 + 13 * 20 + 13 * 10 + 2 * 5
1620: 0 * 200 + 0 * 100 + 2 * 50 + 13 * 20 + 14 * 10 + 0 * 5
1621: 0 * 200 + 0 * 100 + 2 * 50 + 14 * 20 + 0 * 10 + 24 * 5
1622: 0 * 200 + 0 * 100 + 2 * 50 + 14 * 20 + 1 * 10 + 22 * 5
1623: 0 * 200 + 0 * 100 + 2 * 50 + 14 * 20 + 2 * 10 + 20 * 5
1624: 0 * 200 + 0 * 100 + 2 * 50 + 14 * 20 + 3 * 10 + 18 * 5
1625: 0 * 200 + 0 * 100 + 2 * 50 + 14 * 20 + 4 * 10 + 16 * 5
1626: 0 * 200 + 0 * 100 + 2 * 50 + 14 * 20 + 5 * 10 + 14 * 5
1627: 0 * 200 + 0 * 100 + 2 * 50 + 14 * 20 + 6 * 10 + 12 * 5
1628: 0 * 200 + 0 * 100 + 2 * 50 + 14 * 20 + 7 * 10 + 10 * 5
1629: 0 * 200 + 0 * 100 + 2 * 50 + 14 * 20 + 8 * 10 + 8 * 5
1630: 0 * 200 + 0 * 100 + 2 * 50 + 14 * 20 + 9 * 10 + 6 * 5
1631: 0 * 200 + 0 * 100 + 2 * 50 + 14 * 20 + 10 * 10 + 4 * 5
1632: 0 * 200 + 0 * 100 + 2 * 50 + 14 * 20 + 11 * 10 + 2 * 5
1633: 0 * 200 + 0 * 100 + 2 * 50 + 14 * 20 + 12 * 10 + 0 * 5
1634: 0 * 200 + 0 * 100 + 2 * 50 + 15 * 20 + 0 * 10 + 20 * 5
1635: 0 * 200 + 0 * 100 + 2 * 50 + 15 * 20 + 1 * 10 + 18 * 5
1636: 0 * 200 + 0 * 100 + 2 * 50 + 15 * 20 + 2 * 10 + 16 * 5
1637: 0 * 200 + 0 * 100 + 2 * 50 + 15 * 20 + 3 * 10 + 14 * 5
1638: 0 * 200 + 0 * 100 + 2 * 50 + 15 * 20 + 4 * 10 + 12 * 5
1639: 0 * 200 + 0 * 100 + 2 * 50 + 15 * 20 + 5 * 10 + 10 * 5
1640: 0 * 200 + 0 * 100 + 2 * 50 + 15 * 20 + 6 * 10 + 8 * 5
1641: 0 * 200 + 0 * 100 + 2 * 50 + 15 * 20 + 7 * 10 + 6 * 5
1642: 0 * 200 + 0 * 100 + 2 * 50 + 15 * 20 + 8 * 10 + 4 * 5
1643: 0 * 200 + 0 * 100 + 2 * 50 + 15 * 20 + 9 * 10 + 2 * 5
1644: 0 * 200 + 0 * 100 + 2 * 50 + 15 * 20 + 10 * 10 + 0 * 5
1645: 0 * 200 + 0 * 100 + 2 * 50 + 16 * 20 + 0 * 10 + 16 * 5
1646: 0 * 200 + 0 * 100 + 2 * 50 + 16 * 20 + 1 * 10 + 14 * 5
1647: 0 * 200 + 0 * 100 + 2 * 50 + 16 * 20 + 2 * 10 + 12 * 5
1648: 0 * 200 + 0 * 100 + 2 * 50 + 16 * 20 + 3 * 10 + 10 * 5
1649: 0 * 200 + 0 * 100 + 2 * 50 + 16 * 20 + 4 * 10 + 8 * 5
1650: 0 * 200 + 0 * 100 + 2 * 50 + 16 * 20 + 5 * 10 + 6 * 5
1651: 0 * 200 + 0 * 100 + 2 * 50 + 16 * 20 + 6 * 10 + 4 * 5
1652: 0 * 200 + 0 * 100 + 2 * 50 + 16 * 20 + 7 * 10 + 2 * 5
1653: 0 * 200 + 0 * 100 + 2 * 50 + 16 * 20 + 8 * 10 + 0 * 5
1654: 0 * 200 + 0 * 100 + 2 * 50 + 17 * 20 + 0 * 10 + 12 * 5
1655: 0 * 200 + 0 * 100 + 2 * 50 + 17 * 20 + 1 * 10 + 10 * 5
1656: 0 * 200 + 0 * 100 + 2 * 50 + 17 * 20 + 2 * 10 + 8 * 5
1657: 0 * 200 + 0 * 100 + 2 * 50 + 17 * 20 + 3 * 10 + 6 * 5
1658: 0 * 200 + 0 * 100 + 2 * 50 + 17 * 20 + 4 * 10 + 4 * 5
1659: 0 * 200 + 0 * 100 + 2 * 50 + 17 * 20 + 5 * 10 + 2 * 5
1660: 0 * 200 + 0 * 100 + 2 * 50 + 17 * 20 + 6 * 10 + 0 * 5
1661: 0 * 200 + 0 * 100 + 2 * 50 + 18 * 20 + 0 * 10 + 8 * 5
1662: 0 * 200 + 0 * 100 + 2 * 50 + 18 * 20 + 1 * 10 + 6 * 5
1663: 0 * 200 + 0 * 100 + 2 * 50 + 18 * 20 + 2 * 10 + 4 * 5
1664: 0 * 200 + 0 * 100 + 2 * 50 + 18 * 20 + 3 * 10 + 2 * 5
1665: 0 * 200 + 0 * 100 + 2 * 50 + 18 * 20 + 4 * 10 + 0 * 5
1666: 0 * 200 + 0 * 100 + 2 * 50 + 19 * 20 + 0 * 10 + 4 * 5
1667: 0 * 200 + 0 * 100 + 2 * 50 + 19 * 20 + 1 * 10 + 2 * 5
1668: 0 * 200 + 0 * 100 + 2 * 50 + 19 * 20 + 2 * 10 + 0 * 5
1669: 0 * 200 + 0 * 100 + 2 * 50 + 20 * 20 + 0 * 10 + 0 * 5
1670: 0 * 200 + 0 * 100 + 3 * 50 + 0 * 20 + 0 * 10 + 70 * 5
1671: 0 * 200 + 0 * 100 + 3 * 50 + 0 * 20 + 1 * 10 + 68 * 5
1672: 0 * 200 + 0 * 100 + 3 * 50 + 0 * 20 + 2 * 10 + 66 * 5
1673: 0 * 200 + 0 * 100 + 3 * 50 + 0 * 20 + 3 * 10 + 64 * 5
1674: 0 * 200 + 0 * 100 + 3 * 50 + 0 * 20 + 4 * 10 + 62 * 5
1675: 0 * 200 + 0 * 100 + 3 * 50 + 0 * 20 + 5 * 10 + 60 * 5
1676: 0 * 200 + 0 * 100 + 3 * 50 + 0 * 20 + 6 * 10 + 58 * 5
1677: 0 * 200 + 0 * 100 + 3 * 50 + 0 * 20 + 7 * 10 + 56 * 5
1678: 0 * 200 + 0 * 100 + 3 * 50 + 0 * 20 + 8 * 10 + 54 * 5
1679: 0 * 200 + 0 * 100 + 3 * 50 + 0 * 20 + 9 * 10 + 52 * 5
1680: 0 * 200 + 0 * 100 + 3 * 50 + 0 * 20 + 10 * 10 + 50 * 5
1681: 0 * 200 + 0 * 100 + 3 * 50 + 0 * 20 + 11 * 10 + 48 * 5
1682: 0 * 200 + 0 * 100 + 3 * 50 + 0 * 20 + 12 * 10 + 46 * 5
1683: 0 * 200 + 0 * 100 + 3 * 50 + 0 * 20 + 13 * 10 + 44 * 5
1684: 0 * 200 + 0 * 100 + 3 * 50 + 0 * 20 + 14 * 10 + 42 * 5
1685: 0 * 200 + 0 * 100 + 3 * 50 + 0 * 20 + 15 * 10 + 40 * 5
1686: 0 * 200 + 0 * 100 + 3 * 50 + 0 * 20 + 16 * 10 + 38 * 5
1687: 0 * 200 + 0 * 100 + 3 * 50 + 0 * 20 + 17 * 10 + 36 * 5
1688: 0 * 200 + 0 * 100 + 3 * 50 + 0 * 20 + 18 * 10 + 34 * 5
1689: 0 * 200 + 0 * 100 + 3 * 50 + 0 * 20 + 19 * 10 + 32 * 5
1690: 0 * 200 + 0 * 100 + 3 * 50 + 0 * 20 + 20 * 10 + 30 * 5
1691: 0 * 200 + 0 * 100 + 3 * 50 + 0 * 20 + 21 * 10 + 28 * 5
1692: 0 * 200 + 0 * 100 + 3 * 50 + 0 * 20 + 22 * 10 + 26 * 5
1693: 0 * 200 + 0 * 100 + 3 * 50 + 0 * 20 + 23 * 10 + 24 * 5
1694: 0 * 200 + 0 * 100 + 3 * 50 + 0 * 20 + 24 * 10 + 22 * 5
1695: 0 * 200 + 0 * 100 + 3 * 50 + 0 * 20 + 25 * 10 + 20 * 5
1696: 0 * 200 + 0 * 100 + 3 * 50 + 0 * 20 + 26 * 10 + 18 * 5
1697: 0 * 200 + 0 * 100 + 3 * 50 + 0 * 20 + 27 * 10 + 16 * 5
1698: 0 * 200 + 0 * 100 + 3 * 50 + 0 * 20 + 28 * 10 + 14 * 5
1699: 0 * 200 + 0 * 100 + 3 * 50 + 0 * 20 + 29 * 10 + 12 * 5
1700: 0 * 200 + 0 * 100 + 3 * 50 + 0 * 20 + 30 * 10 + 10 * 5
1701: 0 * 200 + 0 * 100 + 3 * 50 + 0 * 20 + 31 * 10 + 8 * 5
1702: 0 * 200 + 0 * 100 + 3 * 50 + 0 * 20 + 32 * 10 + 6 * 5
1703: 0 * 200 + 0 * 100 + 3 * 50 + 0 * 20 + 33 * 10 + 4 * 5
1704: 0 * 200 + 0 * 100 + 3 * 50 + 0 * 20 + 34 * 10 + 2 * 5
1705: 0 * 200 + 0 * 100 + 3 * 50 + 0 * 20 + 35 * 10 + 0 * 5
1706: 0 * 200 + 0 * 100 + 3 * 50 + 1 * 20 + 0 * 10 + 66 * 5
1707: 0 * 200 + 0 * 100 + 3 * 50 + 1 * 20 + 1 * 10 + 64 * 5
1708: 0 * 200 + 0 * 100 + 3 * 50 + 1 * 20 + 2 * 10 + 62 * 5
1709: 0 * 200 + 0 * 100 + 3 * 50 + 1 * 20 + 3 * 10 + 60 * 5
1710: 0 * 200 + 0 * 100 + 3 * 50 + 1 * 20 + 4 * 10 + 58 * 5
1711: 0 * 200 + 0 * 100 + 3 * 50 + 1 * 20 + 5 * 10 + 56 * 5
1712: 0 * 200 + 0 * 100 + 3 * 50 + 1 * 20 + 6 * 10 + 54 * 5
1713: 0 * 200 + 0 * 100 + 3 * 50 + 1 * 20 + 7 * 10 + 52 * 5
1714: 0 * 200 + 0 * 100 + 3 * 50 + 1 * 20 + 8 * 10 + 50 * 5
1715: 0 * 200 + 0 * 100 + 3 * 50 + 1 * 20 + 9 * 10 + 48 * 5
1716: 0 * 200 + 0 * 100 + 3 * 50 + 1 * 20 + 10 * 10 + 46 * 5
1717: 0 * 200 + 0 * 100 + 3 * 50 + 1 * 20 + 11 * 10 + 44 * 5
1718: 0 * 200 + 0 * 100 + 3 * 50 + 1 * 20 + 12 * 10 + 42 * 5
1719: 0 * 200 + 0 * 100 + 3 * 50 + 1 * 20 + 13 * 10 + 40 * 5
1720: 0 * 200 + 0 * 100 + 3 * 50 + 1 * 20 + 14 * 10 + 38 * 5
1721: 0 * 200 + 0 * 100 + 3 * 50 + 1 * 20 + 15 * 10 + 36 * 5
1722: 0 * 200 + 0 * 100 + 3 * 50 + 1 * 20 + 16 * 10 + 34 * 5
1723: 0 * 200 + 0 * 100 + 3 * 50 + 1 * 20 + 17 * 10 + 32 * 5
1724: 0 * 200 + 0 * 100 + 3 * 50 + 1 * 20 + 18 * 10 + 30 * 5
1725: 0 * 200 + 0 * 100 + 3 * 50 + 1 * 20 + 19 * 10 + 28 * 5
1726: 0 * 200 + 0 * 100 + 3 * 50 + 1 * 20 + 20 * 10 + 26 * 5
1727: 0 * 200 + 0 * 100 + 3 * 50 + 1 * 20 + 21 * 10 + 24 * 5
1728: 0 * 200 + 0 * 100 + 3 * 50 + 1 * 20 + 22 * 10 + 22 * 5
1729: 0 * 200 + 0 * 100 + 3 * 50 + 1 * 20 + 23 * 10 + 20 * 5
1730: 0 * 200 + 0 * 100 + 3 * 50 + 1 * 20 + 24 * 10 + 18 * 5
1731: 0 * 200 + 0 * 100 + 3 * 50 + 1 * 20 + 25 * 10 + 16 * 5
1732: 0 * 200 + 0 * 100 + 3 * 50 + 1 * 20 + 26 * 10 + 14 * 5
1733: 0 * 200 + 0 * 100 + 3 * 50 + 1 * 20 + 27 * 10 + 12 * 5
1734: 0 * 200 + 0 * 100 + 3 * 50 + 1 * 20 + 28 * 10 + 10 * 5
1735: 0 * 200 + 0 * 100 + 3 * 50 + 1 * 20 + 29 * 10 + 8 * 5
1736: 0 * 200 + 0 * 100 + 3 * 50 + 1 * 20 + 30 * 10 + 6 * 5
1737: 0 * 200 + 0 * 100 + 3 * 50 + 1 * 20 + 31 * 10 + 4 * 5
1738: 0 * 200 + 0 * 100 + 3 * 50 + 1 * 20 + 32 * 10 + 2 * 5
1739: 0 * 200 + 0 * 100 + 3 * 50 + 1 * 20 + 33 * 10 + 0 * 5
1740: 0 * 200 + 0 * 100 + 3 * 50 + 2 * 20 + 0 * 10 + 62 * 5
1741: 0 * 200 + 0 * 100 + 3 * 50 + 2 * 20 + 1 * 10 + 60 * 5
1742: 0 * 200 + 0 * 100 + 3 * 50 + 2 * 20 + 2 * 10 + 58 * 5
1743: 0 * 200 + 0 * 100 + 3 * 50 + 2 * 20 + 3 * 10 + 56 * 5
1744: 0 * 200 + 0 * 100 + 3 * 50 + 2 * 20 + 4 * 10 + 54 * 5
1745: 0 * 200 + 0 * 100 + 3 * 50 + 2 * 20 + 5 * 10 + 52 * 5
1746: 0 * 200 + 0 * 100 + 3 * 50 + 2 * 20 + 6 * 10 + 50 * 5
1747: 0 * 200 + 0 * 100 + 3 * 50 + 2 * 20 + 7 * 10 + 48 * 5
1748: 0 * 200 + 0 * 100 + 3 * 50 + 2 * 20 + 8 * 10 + 46 * 5
1749: 0 * 200 + 0 * 100 + 3 * 50 + 2 * 20 + 9 * 10 + 44 * 5
1750: 0 * 200 + 0 * 100 + 3 * 50 + 2 * 20 + 10 * 10 + 42 * 5
1751: 0 * 200 + 0 * 100 + 3 * 50 + 2 * 20 + 11 * 10 + 40 * 5
1752: 0 * 200 + 0 * 100 + 3 * 50 + 2 * 20 + 12 * 10 + 38 * 5
1753: 0 * 200 + 0 * 100 + 3 * 50 + 2 * 20 + 13 * 10 + 36 * 5
1754: 0 * 200 + 0 * 100 + 3 * 50 + 2 * 20 + 14 * 10 + 34 * 5
1755: 0 * 200 + 0 * 100 + 3 * 50 + 2 * 20 + 15 * 10 + 32 * 5
1756: 0 * 200 + 0 * 100 + 3 * 50 + 2 * 20 + 16 * 10 + 30 * 5
1757: 0 * 200 + 0 * 100 + 3 * 50 + 2 * 20 + 17 * 10 + 28 * 5
1758: 0 * 200 + 0 * 100 + 3 * 50 + 2 * 20 + 18 * 10 + 26 * 5
1759: 0 * 200 + 0 * 100 + 3 * 50 + 2 * 20 + 19 * 10 + 24 * 5
1760: 0 * 200 + 0 * 100 + 3 * 50 + 2 * 20 + 20 * 10 + 22 * 5
1761: 0 * 200 + 0 * 100 + 3 * 50 + 2 * 20 + 21 * 10 + 20 * 5
1762: 0 * 200 + 0 * 100 + 3 * 50 + 2 * 20 + 22 * 10 + 18 * 5
1763: 0 * 200 + 0 * 100 + 3 * 50 + 2 * 20 + 23 * 10 + 16 * 5
1764: 0 * 200 + 0 * 100 + 3 * 50 + 2 * 20 + 24 * 10 + 14 * 5
1765: 0 * 200 + 0 * 100 + 3 * 50 + 2 * 20 + 25 * 10 + 12 * 5
1766: 0 * 200 + 0 * 100 + 3 * 50 + 2 * 20 + 26 * 10 + 10 * 5
1767: 0 * 200 + 0 * 100 + 3 * 50 + 2 * 20 + 27 * 10 + 8 * 5
1768: 0 * 200 + 0 * 100 + 3 * 50 + 2 * 20 + 28 * 10 + 6 * 5
1769: 0 * 200 + 0 * 100 + 3 * 50 + 2 * 20 + 29 * 10 + 4 * 5
1770: 0 * 200 + 0 * 100 + 3 * 50 + 2 * 20 + 30 * 10 + 2 * 5
1771: 0 * 200 + 0 * 100 + 3 * 50 + 2 * 20 + 31 * 10 + 0 * 5
1772: 0 * 200 + 0 * 100 + 3 * 50 + 3 * 20 + 0 * 10 + 58 * 5
1773: 0 * 200 + 0 * 100 + 3 * 50 + 3 * 20 + 1 * 10 + 56 * 5
1774: 0 * 200 + 0 * 100 + 3 * 50 + 3 * 20 + 2 * 10 + 54 * 5
1775: 0 * 200 + 0 * 100 + 3 * 50 + 3 * 20 + 3 * 10 + 52 * 5
1776: 0 * 200 + 0 * 100 + 3 * 50 + 3 * 20 + 4 * 10 + 50 * 5
1777: 0 * 200 + 0 * 100 + 3 * 50 + 3 * 20 + 5 * 10 + 48 * 5
1778: 0 * 200 + 0 * 100 + 3 * 50 + 3 * 20 + 6 * 10 + 46 * 5
1779: 0 * 200 + 0 * 100 + 3 * 50 + 3 * 20 + 7 * 10 + 44 * 5
1780: 0 * 200 + 0 * 100 + 3 * 50 + 3 * 20 + 8 * 10 + 42 * 5
1781: 0 * 200 + 0 * 100 + 3 * 50 + 3 * 20 + 9 * 10 + 40 * 5
1782: 0 * 200 + 0 * 100 + 3 * 50 + 3 * 20 + 10 * 10 + 38 * 5
1783: 0 * 200 + 0 * 100 + 3 * 50 + 3 * 20 + 11 * 10 + 36 * 5
1784: 0 * 200 + 0 * 100 + 3 * 50 + 3 * 20 + 12 * 10 + 34 * 5
1785: 0 * 200 + 0 * 100 + 3 * 50 + 3 * 20 + 13 * 10 + 32 * 5
1786: 0 * 200 + 0 * 100 + 3 * 50 + 3 * 20 + 14 * 10 + 30 * 5
1787: 0 * 200 + 0 * 100 + 3 * 50 + 3 * 20 + 15 * 10 + 28 * 5
1788: 0 * 200 + 0 * 100 + 3 * 50 + 3 * 20 + 16 * 10 + 26 * 5
1789: 0 * 200 + 0 * 100 + 3 * 50 + 3 * 20 + 17 * 10 + 24 * 5
1790: 0 * 200 + 0 * 100 + 3 * 50 + 3 * 20 + 18 * 10 + 22 * 5
1791: 0 * 200 + 0 * 100 + 3 * 50 + 3 * 20 + 19 * 10 + 20 * 5
1792: 0 * 200 + 0 * 100 + 3 * 50 + 3 * 20 + 20 * 10 + 18 * 5
1793: 0 * 200 + 0 * 100 + 3 * 50 + 3 * 20 + 21 * 10 + 16 * 5
1794: 0 * 200 + 0 * 100 + 3 * 50 + 3 * 20 + 22 * 10 + 14 * 5
1795: 0 * 200 + 0 * 100 + 3 * 50 + 3 * 20 + 23 * 10 + 12 * 5
1796: 0 * 200 + 0 * 100 + 3 * 50 + 3 * 20 + 24 * 10 + 10 * 5
1797: 0 * 200 + 0 * 100 + 3 * 50 + 3 * 20 + 25 * 10 + 8 * 5
1798: 0 * 200 + 0 * 100 + 3 * 50 + 3 * 20 + 26 * 10 + 6 * 5
1799: 0 * 200 + 0 * 100 + 3 * 50 + 3 * 20 + 27 * 10 + 4 * 5
1800: 0 * 200 + 0 * 100 + 3 * 50 + 3 * 20 + 28 * 10 + 2 * 5
1801: 0 * 200 + 0 * 100 + 3 * 50 + 3 * 20 + 29 * 10 + 0 * 5
1802: 0 * 200 + 0 * 100 + 3 * 50 + 4 * 20 + 0 * 10 + 54 * 5
1803: 0 * 200 + 0 * 100 + 3 * 50 + 4 * 20 + 1 * 10 + 52 * 5
1804: 0 * 200 + 0 * 100 + 3 * 50 + 4 * 20 + 2 * 10 + 50 * 5
1805: 0 * 200 + 0 * 100 + 3 * 50 + 4 * 20 + 3 * 10 + 48 * 5
1806: 0 * 200 + 0 * 100 + 3 * 50 + 4 * 20 + 4 * 10 + 46 * 5
1807: 0 * 200 + 0 * 100 + 3 * 50 + 4 * 20 + 5 * 10 + 44 * 5
1808: 0 * 200 + 0 * 100 + 3 * 50 + 4 * 20 + 6 * 10 + 42 * 5
1809: 0 * 200 + 0 * 100 + 3 * 50 + 4 * 20 + 7 * 10 + 40 * 5
1810: 0 * 200 + 0 * 100 + 3 * 50 + 4 * 20 + 8 * 10 + 38 * 5
1811: 0 * 200 + 0 * 100 + 3 * 50 + 4 * 20 + 9 * 10 + 36 * 5
1812: 0 * 200 + 0 * 100 + 3 * 50 + 4 * 20 + 10 * 10 + 34 * 5
1813: 0 * 200 + 0 * 100 + 3 * 50 + 4 * 20 + 11 * 10 + 32 * 5
1814: 0 * 200 + 0 * 100 + 3 * 50 + 4 * 20 + 12 * 10 + 30 * 5
1815: 0 * 200 + 0 * 100 + 3 * 50 + 4 * 20 + 13 * 10 + 28 * 5
1816: 0 * 200 + 0 * 100 + 3 * 50 + 4 * 20 + 14 * 10 + 26 * 5
1817: 0 * 200 + 0 * 100 + 3 * 50 + 4 * 20 + 15 * 10 + 24 * 5
1818: 0 * 200 + 0 * 100 + 3 * 50 + 4 * 20 + 16 * 10 + 22 * 5
1819: 0 * 200 + 0 * 100 + 3 * 50 + 4 * 20 + 17 * 10 + 20 * 5
1820: 0 * 200 + 0 * 100 + 3 * 50 + 4 * 20 + 18 * 10 + 18 * 5
1821: 0 * 200 + 0 * 100 + 3 * 50 + 4 * 20 + 19 * 10 + 16 * 5
1822: 0 * 200 + 0 * 100 + 3 * 50 + 4 * 20 + 20 * 10 + 14 * 5
1823: 0 * 200 + 0 * 100 + 3 * 50 + 4 * 20 + 21 * 10 + 12 * 5
1824: 0 * 200 + 0 * 100 + 3 * 50 + 4 * 20 + 22 * 10 + 10 * 5
1825: 0 * 200 + 0 * 100 + 3 * 50 + 4 * 20 + 23 * 10 + 8 * 5
1826: 0 * 200 + 0 * 100 + 3 * 50 + 4 * 20 + 24 * 10 + 6 * 5
1827: 0 * 200 + 0 * 100 + 3 * 50 + 4 * 20 + 25 * 10 + 4 * 5
1828: 0 * 200 + 0 * 100 + 3 * 50 + 4 * 20 + 26 * 10 + 2 * 5
1829: 0 * 200 + 0 * 100 + 3 * 50 + 4 * 20 + 27 * 10 + 0 * 5
1830: 0 * 200 + 0 * 100 + 3 * 50 + 5 * 20 + 0 * 10 + 50 * 5
1831: 0 * 200 + 0 * 100 + 3 * 50 + 5 * 20 + 1 * 10 + 48 * 5
1832: 0 * 200 + 0 * 100 + 3 * 50 + 5 * 20 + 2 * 10 + 46 * 5
1833: 0 * 200 + 0 * 100 + 3 * 50 + 5 * 20 + 3 * 10 + 44 * 5
1834: 0 * 200 + 0 * 100 + 3 * 50 + 5 * 20 + 4 * 10 + 42 * 5
1835: 0 * 200 + 0 * 100 + 3 * 50 + 5 * 20 + 5 * 10 + 40 * 5
1836: 0 * 200 + 0 * 100 + 3 * 50 + 5 * 20 + 6 * 10 + 38 * 5
1837: 0 * 200 + 0 * 100 + 3 * 50 + 5 * 20 + 7 * 10 + 36 * 5
1838: 0 * 200 + 0 * 100 + 3 * 50 + 5 * 20 + 8 * 10 + 34 * 5
1839: 0 * 200 + 0 * 100 + 3 * 50 + 5 * 20 + 9 * 10 + 32 * 5
1840: 0 * 200 + 0 * 100 + 3 * 50 + 5 * 20 + 10 * 10 + 30 * 5
1841: 0 * 200 + 0 * 100 + 3 * 50 + 5 * 20 + 11 * 10 + 28 * 5
1842: 0 * 200 + 0 * 100 + 3 * 50 + 5 * 20 + 12 * 10 + 26 * 5
1843: 0 * 200 + 0 * 100 + 3 * 50 + 5 * 20 + 13 * 10 + 24 * 5
1844: 0 * 200 + 0 * 100 + 3 * 50 + 5 * 20 + 14 * 10 + 22 * 5
1845: 0 * 200 + 0 * 100 + 3 * 50 + 5 * 20 + 15 * 10 + 20 * 5
1846: 0 * 200 + 0 * 100 + 3 * 50 + 5 * 20 + 16 * 10 + 18 * 5
1847: 0 * 200 + 0 * 100 + 3 * 50 + 5 * 20 + 17 * 10 + 16 * 5
1848: 0 * 200 + 0 * 100 + 3 * 50 + 5 * 20 + 18 * 10 + 14 * 5
1849: 0 * 200 + 0 * 100 + 3 * 50 + 5 * 20 + 19 * 10 + 12 * 5
1850: 0 * 200 + 0 * 100 + 3 * 50 + 5 * 20 + 20 * 10 + 10 * 5
1851: 0 * 200 + 0 * 100 + 3 * 50 + 5 * 20 + 21 * 10 + 8 * 5
1852: 0 * 200 + 0 * 100 + 3 * 50 + 5 * 20 + 22 * 10 + 6 * 5
1853: 0 * 200 + 0 * 100 + 3 * 50 + 5 * 20 + 23 * 10 + 4 * 5
1854: 0 * 200 + 0 * 100 + 3 * 50 + 5 * 20 + 24 * 10 + 2 * 5
1855: 0 * 200 + 0 * 100 + 3 * 50 + 5 * 20 + 25 * 10 + 0 * 5
1856: 0 * 200 + 0 * 100 + 3 * 50 + 6 * 20 + 0 * 10 + 46 * 5
1857: 0 * 200 + 0 * 100 + 3 * 50 + 6 * 20 + 1 * 10 + 44 * 5
1858: 0 * 200 + 0 * 100 + 3 * 50 + 6 * 20 + 2 * 10 + 42 * 5
1859: 0 * 200 + 0 * 100 + 3 * 50 + 6 * 20 + 3 * 10 + 40 * 5
1860: 0 * 200 + 0 * 100 + 3 * 50 + 6 * 20 + 4 * 10 + 38 * 5
1861: 0 * 200 + 0 * 100 + 3 * 50 + 6 * 20 + 5 * 10 + 36 * 5
1862: 0 * 200 + 0 * 100 + 3 * 50 + 6 * 20 + 6 * 10 + 34 * 5
1863: 0 * 200 + 0 * 100 + 3 * 50 + 6 * 20 + 7 * 10 + 32 * 5
1864: 0 * 200 + 0 * 100 + 3 * 50 + 6 * 20 + 8 * 10 + 30 * 5
1865: 0 * 200 + 0 * 100 + 3 * 50 + 6 * 20 + 9 * 10 + 28 * 5
1866: 0 * 200 + 0 * 100 + 3 * 50 + 6 * 20 + 10 * 10 + 26 * 5
1867: 0 * 200 + 0 * 100 + 3 * 50 + 6 * 20 + 11 * 10 + 24 * 5
1868: 0 * 200 + 0 * 100 + 3 * 50 + 6 * 20 + 12 * 10 + 22 * 5
1869: 0 * 200 + 0 * 100 + 3 * 50 + 6 * 20 + 13 * 10 + 20 * 5
1870: 0 * 200 + 0 * 100 + 3 * 50 + 6 * 20 + 14 * 10 + 18 * 5
1871: 0 * 200 + 0 * 100 + 3 * 50 + 6 * 20 + 15 * 10 + 16 * 5
1872: 0 * 200 + 0 * 100 + 3 * 50 + 6 * 20 + 16 * 10 + 14 * 5
1873: 0 * 200 + 0 * 100 + 3 * 50 + 6 * 20 + 17 * 10 + 12 * 5
1874: 0 * 200 + 0 * 100 + 3 * 50 + 6 * 20 + 18 * 10 + 10 * 5
1875: 0 * 200 + 0 * 100 + 3 * 50 + 6 * 20 + 19 * 10 + 8 * 5
1876: 0 * 200 + 0 * 100 + 3 * 50 + 6 * 20 + 20 * 10 + 6 * 5
1877: 0 * 200 + 0 * 100 + 3 * 50 + 6 * 20 + 21 * 10 + 4 * 5
1878: 0 * 200 + 0 * 100 + 3 * 50 + 6 * 20 + 22 * 10 + 2 * 5
1879: 0 * 200 + 0 * 100 + 3 * 50 + 6 * 20 + 23 * 10 + 0 * 5
1880: 0 * 200 + 0 * 100 + 3 * 50 + 7 * 20 + 0 * 10 + 42 * 5
1881: 0 * 200 + 0 * 100 + 3 * 50 + 7 * 20 + 1 * 10 + 40 * 5
1882: 0 * 200 + 0 * 100 + 3 * 50 + 7 * 20 + 2 * 10 + 38 * 5
1883: 0 * 200 + 0 * 100 + 3 * 50 + 7 * 20 + 3 * 10 + 36 * 5
1884: 0 * 200 + 0 * 100 + 3 * 50 + 7 * 20 + 4 * 10 + 34 * 5
1885: 0 * 200 + 0 * 100 + 3 * 50 + 7 * 20 + 5 * 10 + 32 * 5
1886: 0 * 200 + 0 * 100 + 3 * 50 + 7 * 20 + 6 * 10 + 30 * 5
1887: 0 * 200 + 0 * 100 + 3 * 50 + 7 * 20 + 7 * 10 + 28 * 5
1888: 0 * 200 + 0 * 100 + 3 * 50 + 7 * 20 + 8 * 10 + 26 * 5
1889: 0 * 200 + 0 * 100 + 3 * 50 + 7 * 20 + 9 * 10 + 24 * 5
1890: 0 * 200 + 0 * 100 + 3 * 50 + 7 * 20 + 10 * 10 + 22 * 5
1891: 0 * 200 + 0 * 100 + 3 * 50 + 7 * 20 + 11 * 10 + 20 * 5
1892: 0 * 200 + 0 * 100 + 3 * 50 + 7 * 20 + 12 * 10 + 18 * 5
1893: 0 * 200 + 0 * 100 + 3 * 50 + 7 * 20 + 13 * 10 + 16 * 5
1894: 0 * 200 + 0 * 100 + 3 * 50 + 7 * 20 + 14 * 10 + 14 * 5
1895: 0 * 200 + 0 * 100 + 3 * 50 + 7 * 20 + 15 * 10 + 12 * 5
1896: 0 * 200 + 0 * 100 + 3 * 50 + 7 * 20 + 16 * 10 + 10 * 5
1897: 0 * 200 + 0 * 100 + 3 * 50 + 7 * 20 + 17 * 10 + 8 * 5
1898: 0 * 200 + 0 * 100 + 3 * 50 + 7 * 20 + 18 * 10 + 6 * 5
1899: 0 * 200 + 0 * 100 + 3 * 50 + 7 * 20 + 19 * 10 + 4 * 5
1900: 0 * 200 + 0 * 100 + 3 * 50 + 7 * 20 + 20 * 10 + 2 * 5
1901: 0 * 200 + 0 * 100 + 3 * 50 + 7 * 20 + 21 * 10 + 0 * 5
1902: 0 * 200 + 0 * 100 + 3 * 50 + 8 * 20 + 0 * 10 + 38 * 5
1903: 0 * 200 + 0 * 100 + 3 * 50 + 8 * 20 + 1 * 10 + 36 * 5
1904: 0 * 200 + 0 * 100 + 3 * 50 + 8 * 20 + 2 * 10 + 34 * 5
1905: 0 * 200 + 0 * 100 + 3 * 50 + 8 * 20 + 3 * 10 + 32 * 5
1906: 0 * 200 + 0 * 100 + 3 * 50 + 8 * 20 + 4 * 10 + 30 * 5
1907: 0 * 200 + 0 * 100 + 3 * 50 + 8 * 20 + 5 * 10 + 28 * 5
1908: 0 * 200 + 0 * 100 + 3 * 50 + 8 * 20 + 6 * 10 + 26 * 5
1909: 0 * 200 + 0 * 100 + 3 * 50 + 8 * 20 + 7 * 10 + 24 * 5
1910: 0 * 200 + 0 * 100 + 3 * 50 + 8 * 20 + 8 * 10 + 22 * 5
1911: 0 * 200 + 0 * 100 + 3 * 50 + 8 * 20 + 9 * 10 + 20 * 5
1912: 0 * 200 + 0 * 100 + 3 * 50 + 8 * 20 + 10 * 10 + 18 * 5
1913: 0 * 200 + 0 * 100 + 3 * 50 + 8 * 20 + 11 * 10 + 16 * 5
1914: 0 * 200 + 0 * 100 + 3 * 50 + 8 * 20 + 12 * 10 + 14 * 5
1915: 0 * 200 + 0 * 100 + 3 * 50 + 8 * 20 + 13 * 10 + 12 * 5
1916: 0 * 200 + 0 * 100 + 3 * 50 + 8 * 20 + 14 * 10 + 10 * 5
1917: 0 * 200 + 0 * 100 + 3 * 50 + 8 * 20 + 15 * 10 + 8 * 5
1918: 0 * 200 + 0 * 100 + 3 * 50 + 8 * 20 + 16 * 10 + 6 * 5
1919: 0 * 200 + 0 * 100 + 3 * 50 + 8 * 20 + 17 * 10 + 4 * 5
1920: 0 * 200 + 0 * 100 + 3 * 50 + 8 * 20 + 18 * 10 + 2 * 5
1921: 0 * 200 + 0 * 100 + 3 * 50 + 8 * 20 + 19 * 10 + 0 * 5
1922: 0 * 200 + 0 * 100 + 3 * 50 + 9 * 20 + 0 * 10 + 34 * 5
1923: 0 * 200 + 0 * 100 + 3 * 50 + 9 * 20 + 1 * 10 + 32 * 5
1924: 0 * 200 + 0 * 100 + 3 * 50 + 9 * 20 + 2 * 10 + 30 * 5
1925: 0 * 200 + 0 * 100 + 3 * 50 + 9 * 20 + 3 * 10 + 28 * 5
1926: 0 * 200 + 0 * 100 + 3 * 50 + 9 * 20 + 4 * 10 + 26 * 5
1927: 0 * 200 + 0 * 100 + 3 * 50 + 9 * 20 + 5 * 10 + 24 * 5
1928: 0 * 200 + 0 * 100 + 3 * 50 + 9 * 20 + 6 * 10 + 22 * 5
1929: 0 * 200 + 0 * 100 + 3 * 50 + 9 * 20 + 7 * 10 + 20 * 5
1930: 0 * 200 + 0 * 100 + 3 * 50 + 9 * 20 + 8 * 10 + 18 * 5
1931: 0 * 200 + 0 * 100 + 3 * 50 + 9 * 20 + 9 * 10 + 16 * 5
1932: 0 * 200 + 0 * 100 + 3 * 50 + 9 * 20 + 10 * 10 + 14 * 5
1933: 0 * 200 + 0 * 100 + 3 * 50 + 9 * 20 + 11 * 10 + 12 * 5
1934: 0 * 200 + 0 * 100 + 3 * 50 + 9 * 20 + 12 * 10 + 10 * 5
1935: 0 * 200 + 0 * 100 + 3 * 50 + 9 * 20 + 13 * 10 + 8 * 5
1936: 0 * 200 + 0 * 100 + 3 * 50 + 9 * 20 + 14 * 10 + 6 * 5
1937: 0 * 200 + 0 * 100 + 3 * 50 + 9 * 20 + 15 * 10 + 4 * 5
1938: 0 * 200 + 0 * 100 + 3 * 50 + 9 * 20 + 16 * 10 + 2 * 5
1939: 0 * 200 + 0 * 100 + 3 * 50 + 9 * 20 + 17 * 10 + 0 * 5
1940: 0 * 200 + 0 * 100 + 3 * 50 + 10 * 20 + 0 * 10 + 30 * 5
1941: 0 * 200 + 0 * 100 + 3 * 50 + 10 * 20 + 1 * 10 + 28 * 5
1942: 0 * 200 + 0 * 100 + 3 * 50 + 10 * 20 + 2 * 10 + 26 * 5
1943: 0 * 200 + 0 * 100 + 3 * 50 + 10 * 20 + 3 * 10 + 24 * 5
1944: 0 * 200 + 0 * 100 + 3 * 50 + 10 * 20 + 4 * 10 + 22 * 5
1945: 0 * 200 + 0 * 100 + 3 * 50 + 10 * 20 + 5 * 10 + 20 * 5
1946: 0 * 200 + 0 * 100 + 3 * 50 + 10 * 20 + 6 * 10 + 18 * 5
1947: 0 * 200 + 0 * 100 + 3 * 50 + 10 * 20 + 7 * 10 + 16 * 5
1948: 0 * 200 + 0 * 100 + 3 * 50 + 10 * 20 + 8 * 10 + 14 * 5
1949: 0 * 200 + 0 * 100 + 3 * 50 + 10 * 20 + 9 * 10 + 12 * 5
1950: 0 * 200 + 0 * 100 + 3 * 50 + 10 * 20 + 10 * 10 + 10 * 5
1951: 0 * 200 + 0 * 100 + 3 * 50 + 10 * 20 + 11 * 10 + 8 * 5
1952: 0 * 200 + 0 * 100 + 3 * 50 + 10 * 20 + 12 * 10 + 6 * 5
1953: 0 * 200 + 0 * 100 + 3 * 50 + 10 * 20 + 13 * 10 + 4 * 5
1954: 0 * 200 + 0 * 100 + 3 * 50 + 10 * 20 + 14 * 10 + 2 * 5
1955: 0 * 200 + 0 * 100 + 3 * 50 + 10 * 20 + 15 * 10 + 0 * 5
1956: 0 * 200 + 0 * 100 + 3 * 50 + 11 * 20 + 0 * 10 + 26 * 5
1957: 0 * 200 + 0 * 100 + 3 * 50 + 11 * 20 + 1 * 10 + 24 * 5
1958: 0 * 200 + 0 * 100 + 3 * 50 + 11 * 20 + 2 * 10 + 22 * 5
1959: 0 * 200 + 0 * 100 + 3 * 50 + 11 * 20 + 3 * 10 + 20 * 5
1960: 0 * 200 + 0 * 100 + 3 * 50 + 11 * 20 + 4 * 10 + 18 * 5
1961: 0 * 200 + 0 * 100 + 3 * 50 + 11 * 20 + 5 * 10 + 16 * 5
1962: 0 * 200 + 0 * 100 + 3 * 50 + 11 * 20 + 6 * 10 + 14 * 5
1963: 0 * 200 + 0 * 100 + 3 * 50 + 11 * 20 + 7 * 10 + 12 * 5
1964: 0 * 200 + 0 * 100 + 3 * 50 + 11 * 20 + 8 * 10 + 10 * 5
1965: 0 * 200 + 0 * 100 + 3 * 50 + 11 * 20 + 9 * 10 + 8 * 5
1966: 0 * 200 + 0 * 100 + 3 * 50 + 11 * 20 + 10 * 10 + 6 * 5
1967: 0 * 200 + 0 * 100 + 3 * 50 + 11 * 20 + 11 * 10 + 4 * 5
1968: 0 * 200 + 0 * 100 + 3 * 50 + 11 * 20 + 12 * 10 + 2 * 5
1969: 0 * 200 + 0 * 100 + 3 * 50 + 11 * 20 + 13 * 10 + 0 * 5
1970: 0 * 200 + 0 * 100 + 3 * 50 + 12 * 20 + 0 * 10 + 22 * 5
1971: 0 * 200 + 0 * 100 + 3 * 50 + 12 * 20 + 1 * 10 + 20 * 5
1972: 0 * 200 + 0 * 100 + 3 * 50 + 12 * 20 + 2 * 10 + 18 * 5
1973: 0 * 200 + 0 * 100 + 3 * 50 + 12 * 20 + 3 * 10 + 16 * 5
1974: 0 * 200 + 0 * 100 + 3 * 50 + 12 * 20 + 4 * 10 + 14 * 5
1975: 0 * 200 + 0 * 100 + 3 * 50 + 12 * 20 + 5 * 10 + 12 * 5
1976: 0 * 200 + 0 * 100 + 3 * 50 + 12 * 20 + 6 * 10 + 10 * 5
1977: 0 * 200 + 0 * 100 + 3 * 50 + 12 * 20 + 7 * 10 + 8 * 5
1978: 0 * 200 + 0 * 100 + 3 * 50 + 12 * 20 + 8 * 10 + 6 * 5
1979: 0 * 200 + 0 * 100 + 3 * 50 + 12 * 20 + 9 * 10 + 4 * 5
1980: 0 * 200 + 0 * 100 + 3 * 50 + 12 * 20 + 10 * 10 + 2 * 5
1981: 0 * 200 + 0 * 100 + 3 * 50 + 12 * 20 + 11 * 10 + 0 * 5
1982: 0 * 200 + 0 * 100 + 3 * 50 + 13 * 20 + 0 * 10 + 18 * 5
1983: 0 * 200 + 0 * 100 + 3 * 50 + 13 * 20 + 1 * 10 + 16 * 5
1984: 0 * 200 + 0 * 100 + 3 * 50 + 13 * 20 + 2 * 10 + 14 * 5
1985: 0 * 200 + 0 * 100 + 3 * 50 + 13 * 20 + 3 * 10 + 12 * 5
1986: 0 * 200 + 0 * 100 + 3 * 50 + 13 * 20 + 4 * 10 + 10 * 5
1987: 0 * 200 + 0 * 100 + 3 * 50 + 13 * 20 + 5 * 10 + 8 * 5
1988: 0 * 200 + 0 * 100 + 3 * 50 + 13 * 20 + 6 * 10 + 6 * 5
1989: 0 * 200 + 0 * 100 + 3 * 50 + 13 * 20 + 7 * 10 + 4 * 5
1990: 0 * 200 + 0 * 100 + 3 * 50 + 13 * 20 + 8 * 10 + 2 * 5
1991: 0 * 200 + 0 * 100 + 3 * 50 + 13 * 20 + 9 * 10 + 0 * 5
1992: 0 * 200 + 0 * 100 + 3 * 50 + 14 * 20 + 0 * 10 + 14 * 5
1993: 0 * 200 + 0 * 100 + 3 * 50 + 14 * 20 + 1 * 10 + 12 * 5
1994: 0 * 200 + 0 * 100 + 3 * 50 + 14 * 20 + 2 * 10 + 10 * 5
1995: 0 * 200 + 0 * 100 + 3 * 50 + 14 * 20 + 3 * 10 + 8 * 5
1996: 0 * 200 + 0 * 100 + 3 * 50 + 14 * 20 + 4 * 10 + 6 * 5
1997: 0 * 200 + 0 * 100 + 3 * 50 + 14 * 20 + 5 * 10 + 4 * 5
1998: 0 * 200 + 0 * 100 + 3 * 50 + 14 * 20 + 6 * 10 + 2 * 5
1999: 0 * 200 + 0 * 100 + 3 * 50 + 14 * 20 + 7 * 10 + 0 * 5
2000: 0 * 200 + 0 * 100 + 3 * 50 + 15 * 20 + 0 * 10 + 10 * 5
2001: 0 * 200 + 0 * 100 + 3 * 50 + 15 * 20 + 1 * 10 + 8 * 5
2002: 0 * 200 + 0 * 100 + 3 * 50 + 15 * 20 + 2 * 10 + 6 * 5
2003: 0 * 200 + 0 * 100 + 3 * 50 + 15 * 20 + 3 * 10 + 4 * 5
2004: 0 * 200 + 0 * 100 + 3 * 50 + 15 * 20 + 4 * 10 + 2 * 5
2005: 0 * 200 + 0 * 100 + 3 * 50 + 15 * 20 + 5 * 10 + 0 * 5
2006: 0 * 200 + 0 * 100 + 3 * 50 + 16 * 20 + 0 * 10 + 6 * 5
2007: 0 * 200 + 0 * 100 + 3 * 50 + 16 * 20 + 1 * 10 + 4 * 5
2008: 0 * 200 + 0 * 100 + 3 * 50 + 16 * 20 + 2 * 10 + 2 * 5
2009: 0 * 200 + 0 * 100 + 3 * 50 + 16 * 20 + 3 * 10 + 0 * 5
2010: 0 * 200 + 0 * 100 + 3 * 50 + 17 * 20 + 0 * 10 + 2 * 5
2011: 0 * 200 + 0 * 100 + 3 * 50 + 17 * 20 + 1 * 10 + 0 * 5
2012: 0 * 200 + 0 * 100 + 4 * 50 + 0 * 20 + 0 * 10 + 60 * 5
2013: 0 * 200 + 0 * 100 + 4 * 50 + 0 * 20 + 1 * 10 + 58 * 5
2014: 0 * 200 + 0 * 100 + 4 * 50 + 0 * 20 + 2 * 10 + 56 * 5
2015: 0 * 200 + 0 * 100 + 4 * 50 + 0 * 20 + 3 * 10 + 54 * 5
2016: 0 * 200 + 0 * 100 + 4 * 50 + 0 * 20 + 4 * 10 + 52 * 5
2017: 0 * 200 + 0 * 100 + 4 * 50 + 0 * 20 + 5 * 10 + 50 * 5
2018: 0 * 200 + 0 * 100 + 4 * 50 + 0 * 20 + 6 * 10 + 48 * 5
2019: 0 * 200 + 0 * 100 + 4 * 50 + 0 * 20 + 7 * 10 + 46 * 5
2020: 0 * 200 + 0 * 100 + 4 * 50 + 0 * 20 + 8 * 10 + 44 * 5
2021: 0 * 200 + 0 * 100 + 4 * 50 + 0 * 20 + 9 * 10 + 42 * 5
2022: 0 * 200 + 0 * 100 + 4 * 50 + 0 * 20 + 10 * 10 + 40 * 5
2023: 0 * 200 + 0 * 100 + 4 * 50 + 0 * 20 + 11 * 10 + 38 * 5
2024: 0 * 200 + 0 * 100 + 4 * 50 + 0 * 20 + 12 * 10 + 36 * 5
2025: 0 * 200 + 0 * 100 + 4 * 50 + 0 * 20 + 13 * 10 + 34 * 5
2026: 0 * 200 + 0 * 100 + 4 * 50 + 0 * 20 + 14 * 10 + 32 * 5
2027: 0 * 200 + 0 * 100 + 4 * 50 + 0 * 20 + 15 * 10 + 30 * 5
2028: 0 * 200 + 0 * 100 + 4 * 50 + 0 * 20 + 16 * 10 + 28 * 5
2029: 0 * 200 + 0 * 100 + 4 * 50 + 0 * 20 + 17 * 10 + 26 * 5
2030: 0 * 200 + 0 * 100 + 4 * 50 + 0 * 20 + 18 * 10 + 24 * 5
2031: 0 * 200 + 0 * 100 + 4 * 50 + 0 * 20 + 19 * 10 + 22 * 5
2032: 0 * 200 + 0 * 100 + 4 * 50 + 0 * 20 + 20 * 10 + 20 * 5
2033: 0 * 200 + 0 * 100 + 4 * 50 + 0 * 20 + 21 * 10 + 18 * 5
2034: 0 * 200 + 0 * 100 + 4 * 50 + 0 * 20 + 22 * 10 + 16 * 5
2035: 0 * 200 + 0 * 100 + 4 * 50 + 0 * 20 + 23 * 10 + 14 * 5
2036: 0 * 200 + 0 * 100 + 4 * 50 + 0 * 20 + 24 * 10 + 12 * 5
2037: 0 * 200 + 0 * 100 + 4 * 50 + 0 * 20 + 25 * 10 + 10 * 5
2038: 0 * 200 + 0 * 100 + 4 * 50 + 0 * 20 + 26 * 10 + 8 * 5
2039: 0 * 200 + 0 * 100 + 4 * 50 + 0 * 20 + 27 * 10 + 6 * 5
2040: 0 * 200 + 0 * 100 + 4 * 50 + 0 * 20 + 28 * 10 + 4 * 5
2041: 0 * 200 + 0 * 100 + 4 * 50 + 0 * 20 + 29 * 10 + 2 * 5
2042: 0 * 200 + 0 * 100 + 4 * 50 + 0 * 20 + 30 * 10 + 0 * 5
2043: 0 * 200 + 0 * 100 + 4 * 50 + 1 * 20 + 0 * 10 + 56 * 5
2044: 0 * 200 + 0 * 100 + 4 * 50 + 1 * 20 + 1 * 10 + 54 * 5
2045: 0 * 200 + 0 * 100 + 4 * 50 + 1 * 20 + 2 * 10 + 52 * 5
2046: 0 * 200 + 0 * 100 + 4 * 50 + 1 * 20 + 3 * 10 + 50 * 5
2047: 0 * 200 + 0 * 100 + 4 * 50 + 1 * 20 + 4 * 10 + 48 * 5
2048: 0 * 200 + 0 * 100 + 4 * 50 + 1 * 20 + 5 * 10 + 46 * 5
2049: 0 * 200 + 0 * 100 + 4 * 50 + 1 * 20 + 6 * 10 + 44 * 5
2050: 0 * 200 + 0 * 100 + 4 * 50 + 1 * 20 + 7 * 10 + 42 * 5
2051: 0 * 200 + 0 * 100 + 4 * 50 + 1 * 20 + 8 * 10 + 40 * 5
2052: 0 * 200 + 0 * 100 + 4 * 50 + 1 * 20 + 9 * 10 + 38 * 5
2053: 0 * 200 + 0 * 100 + 4 * 50 + 1 * 20 + 10 * 10 + 36 * 5
2054: 0 * 200 + 0 * 100 + 4 * 50 + 1 * 20 + 11 * 10 + 34 * 5
2055: 0 * 200 + 0 * 100 + 4 * 50 + 1 * 20 + 12 * 10 + 32 * 5
2056: 0 * 200 + 0 * 100 + 4 * 50 + 1 * 20 + 13 * 10 + 30 * 5
2057: 0 * 200 + 0 * 100 + 4 * 50 + 1 * 20 + 14 * 10 + 28 * 5
2058: 0 * 200 + 0 * 100 + 4 * 50 + 1 * 20 + 15 * 10 + 26 * 5
2059: 0 * 200 + 0 * 100 + 4 * 50 + 1 * 20 + 16 * 10 + 24 * 5
2060: 0 * 200 + 0 * 100 + 4 * 50 + 1 * 20 + 17 * 10 + 22 * 5
2061: 0 * 200 + 0 * 100 + 4 * 50 + 1 * 20 + 18 * 10 + 20 * 5
2062: 0 * 200 + 0 * 100 + 4 * 50 + 1 * 20 + 19 * 10 + 18 * 5
2063: 0 * 200 + 0 * 100 + 4 * 50 + 1 * 20 + 20 * 10 + 16 * 5
2064: 0 * 200 + 0 * 100 + 4 * 50 + 1 * 20 + 21 * 10 + 14 * 5
2065: 0 * 200 + 0 * 100 + 4 * 50 + 1 * 20 + 22 * 10 + 12 * 5
2066: 0 * 200 + 0 * 100 + 4 * 50 + 1 * 20 + 23 * 10 + 10 * 5
2067: 0 * 200 + 0 * 100 + 4 * 50 + 1 * 20 + 24 * 10 + 8 * 5
2068: 0 * 200 + 0 * 100 + 4 * 50 + 1 * 20 + 25 * 10 + 6 * 5
2069: 0 * 200 + 0 * 100 + 4 * 50 + 1 * 20 + 26 * 10 + 4 * 5
2070: 0 * 200 + 0 * 100 + 4 * 50 + 1 * 20 + 27 * 10 + 2 * 5
2071: 0 * 200 + 0 * 100 + 4 * 50 + 1 * 20 + 28 * 10 + 0 * 5
2072: 0 * 200 + 0 * 100 + 4 * 50 + 2 * 20 + 0 * 10 + 52 * 5
2073: 0 * 200 + 0 * 100 + 4 * 50 + 2 * 20 + 1 * 10 + 50 * 5
2074: 0 * 200 + 0 * 100 + 4 * 50 + 2 * 20 + 2 * 10 + 48 * 5
2075: 0 * 200 + 0 * 100 + 4 * 50 + 2 * 20 + 3 * 10 + 46 * 5
2076: 0 * 200 + 0 * 100 + 4 * 50 + 2 * 20 + 4 * 10 + 44 * 5
2077: 0 * 200 + 0 * 100 + 4 * 50 + 2 * 20 + 5 * 10 + 42 * 5
2078: 0 * 200 + 0 * 100 + 4 * 50 + 2 * 20 + 6 * 10 + 40 * 5
2079: 0 * 200 + 0 * 100 + 4 * 50 + 2 * 20 + 7 * 10 + 38 * 5
2080: 0 * 200 + 0 * 100 + 4 * 50 + 2 * 20 + 8 * 10 + 36 * 5
2081: 0 * 200 + 0 * 100 + 4 * 50 + 2 * 20 + 9 * 10 + 34 * 5
2082: 0 * 200 + 0 * 100 + 4 * 50 + 2 * 20 + 10 * 10 + 32 * 5
2083: 0 * 200 + 0 * 100 + 4 * 50 + 2 * 20 + 11 * 10 + 30 * 5
2084: 0 * 200 + 0 * 100 + 4 * 50 + 2 * 20 + 12 * 10 + 28 * 5
2085: 0 * 200 + 0 * 100 + 4 * 50 + 2 * 20 + 13 * 10 + 26 * 5
2086: 0 * 200 + 0 * 100 + 4 * 50 + 2 * 20 + 14 * 10 + 24 * 5
2087: 0 * 200 + 0 * 100 + 4 * 50 + 2 * 20 + 15 * 10 + 22 * 5
2088: 0 * 200 + 0 * 100 + 4 * 50 + 2 * 20 + 16 * 10 + 20 * 5
2089: 0 * 200 + 0 * 100 + 4 * 50 + 2 * 20 + 17 * 10 + 18 * 5
2090: 0 * 200 + 0 * 100 + 4 * 50 + 2 * 20 + 18 * 10 + 16 * 5
2091: 0 * 200 + 0 * 100 + 4 * 50 + 2 * 20 + 19 * 10 + 14 * 5
2092: 0 * 200 + 0 * 100 + 4 * 50 + 2 * 20 + 20 * 10 + 12 * 5
2093: 0 * 200 + 0 * 100 + 4 * 50 + 2 * 20 + 21 * 10 + 10 * 5
2094: 0 * 200 + 0 * 100 + 4 * 50 + 2 * 20 + 22 * 10 + 8 * 5
2095: 0 * 200 + 0 * 100 + 4 * 50 + 2 * 20 + 23 * 10 + 6 * 5
2096: 0 * 200 + 0 * 100 + 4 * 50 + 2 * 20 + 24 * 10 + 4 * 5
2097: 0 * 200 + 0 * 100 + 4 * 50 + 2 * 20 + 25 * 10 + 2 * 5
2098: 0 * 200 + 0 * 100 + 4 * 50 + 2 * 20 + 26 * 10 + 0 * 5
2099: 0 * 200 + 0 * 100 + 4 * 50 + 3 * 20 + 0 * 10 + 48 * 5
2100: 0 * 200 + 0 * 100 + 4 * 50 + 3 * 20 + 1 * 10 + 46 * 5
2101: 0 * 200 + 0 * 100 + 4 * 50 + 3 * 20 + 2 * 10 + 44 * 5
2102: 0 * 200 + 0 * 100 + 4 * 50 + 3 * 20 + 3 * 10 + 42 * 5
2103: 0 * 200 + 0 * 100 + 4 * 50 + 3 * 20 + 4 * 10 + 40 * 5
2104: 0 * 200 + 0 * 100 + 4 * 50 + 3 * 20 + 5 * 10 + 38 * 5
2105: 0 * 200 + 0 * 100 + 4 * 50 + 3 * 20 + 6 * 10 + 36 * 5
2106: 0 * 200 + 0 * 100 + 4 * 50 + 3 * 20 + 7 * 10 + 34 * 5
2107: 0 * 200 + 0 * 100 + 4 * 50 + 3 * 20 + 8 * 10 + 32 * 5
2108: 0 * 200 + 0 * 100 + 4 * 50 + 3 * 20 + 9 * 10 + 30 * 5
2109: 0 * 200 + 0 * 100 + 4 * 50 + 3 * 20 + 10 * 10 + 28 * 5
2110: 0 * 200 + 0 * 100 + 4 * 50 + 3 * 20 + 11 * 10 + 26 * 5
2111: 0 * 200 + 0 * 100 + 4 * 50 + 3 * 20 + 12 * 10 + 24 * 5
2112: 0 * 200 + 0 * 100 + 4 * 50 + 3 * 20 + 13 * 10 + 22 * 5
2113: 0 * 200 + 0 * 100 + 4 * 50 + 3 * 20 + 14 * 10 + 20 * 5
2114: 0 * 200 + 0 * 100 + 4 * 50 + 3 * 20 + 15 * 10 + 18 * 5
2115: 0 * 200 + 0 * 100 + 4 * 50 + 3 * 20 + 16 * 10 + 16 * 5
2116: 0 * 200 + 0 * 100 + 4 * 50 + 3 * 20 + 17 * 10 + 14 * 5
2117: 0 * 200 + 0 * 100 + 4 * 50 + 3 * 20 + 18 * 10 + 12 * 5
2118: 0 * 200 + 0 * 100 + 4 * 50 + 3 * 20 + 19 * 10 + 10 * 5
2119: 0 * 200 + 0 * 100 + 4 * 50 + 3 * 20 + 20 * 10 + 8 * 5
2120: 0 * 200 + 0 * 100 + 4 * 50 + 3 * 20 + 21 * 10 + 6 * 5
2121: 0 * 200 + 0 * 100 + 4 * 50 + 3 * 20 + 22 * 10 + 4 * 5
2122: 0 * 200 + 0 * 100 + 4 * 50 + 3 * 20 + 23 * 10 + 2 * 5
2123: 0 * 200 + 0 * 100 + 4 * 50 + 3 * 20 + 24 * 10 + 0 * 5
2124: 0 * 200 + 0 * 100 + 4 * 50 + 4 * 20 + 0 * 10 + 44 * 5
2125: 0 * 200 + 0 * 100 + 4 * 50 + 4 * 20 + 1 * 10 + 42 * 5
2126: 0 * 200 + 0 * 100 + 4 * 50 + 4 * 20 + 2 * 10 + 40 * 5
2127: 0 * 200 + 0 * 100 + 4 * 50 + 4 * 20 + 3 * 10 + 38 * 5
2128: 0 * 200 + 0 * 100 + 4 * 50 + 4 * 20 + 4 * 10 + 36 * 5
2129: 0 * 200 + 0 * 100 + 4 * 50 + 4 * 20 + 5 * 10 + 34 * 5
2130: 0 * 200 + 0 * 100 + 4 * 50 + 4 * 20 + 6 * 10 + 32 * 5
2131: 0 * 200 + 0 * 100 + 4 * 50 + 4 * 20 + 7 * 10 + 30 * 5
2132: 0 * 200 + 0 * 100 + 4 * 50 + 4 * 20 + 8 * 10 + 28 * 5
2133: 0 * 200 + 0 * 100 + 4 * 50 + 4 * 20 + 9 * 10 + 26 * 5
2134: 0 * 200 + 0 * 100 + 4 * 50 + 4 * 20 + 10 * 10 + 24 * 5
2135: 0 * 200 + 0 * 100 + 4 * 50 + 4 * 20 + 11 * 10 + 22 * 5
2136: 0 * 200 + 0 * 100 + 4 * 50 + 4 * 20 + 12 * 10 + 20 * 5
2137: 0 * 200 + 0 * 100 + 4 * 50 + 4 * 20 + 13 * 10 + 18 * 5
2138: 0 * 200 + 0 * 100 + 4 * 50 + 4 * 20 + 14 * 10 + 16 * 5
2139: 0 * 200 + 0 * 100 + 4 * 50 + 4 * 20 + 15 * 10 + 14 * 5
2140: 0 * 200 + 0 * 100 + 4 * 50 + 4 * 20 + 16 * 10 + 12 * 5
2141: 0 * 200 + 0 * 100 + 4 * 50 + 4 * 20 + 17 * 10 + 10 * 5
2142: 0 * 200 + 0 * 100 + 4 * 50 + 4 * 20 + 18 * 10 + 8 * 5
2143: 0 * 200 + 0 * 100 + 4 * 50 + 4 * 20 + 19 * 10 + 6 * 5
2144: 0 * 200 + 0 * 100 + 4 * 50 + 4 * 20 + 20 * 10 + 4 * 5
2145: 0 * 200 + 0 * 100 + 4 * 50 + 4 * 20 + 21 * 10 + 2 * 5
2146: 0 * 200 + 0 * 100 + 4 * 50 + 4 * 20 + 22 * 10 + 0 * 5
2147: 0 * 200 + 0 * 100 + 4 * 50 + 5 * 20 + 0 * 10 + 40 * 5
2148: 0 * 200 + 0 * 100 + 4 * 50 + 5 * 20 + 1 * 10 + 38 * 5
2149: 0 * 200 + 0 * 100 + 4 * 50 + 5 * 20 + 2 * 10 + 36 * 5
2150: 0 * 200 + 0 * 100 + 4 * 50 + 5 * 20 + 3 * 10 + 34 * 5
2151: 0 * 200 + 0 * 100 + 4 * 50 + 5 * 20 + 4 * 10 + 32 * 5
2152: 0 * 200 + 0 * 100 + 4 * 50 + 5 * 20 + 5 * 10 + 30 * 5
2153: 0 * 200 + 0 * 100 + 4 * 50 + 5 * 20 + 6 * 10 + 28 * 5
2154: 0 * 200 + 0 * 100 + 4 * 50 + 5 * 20 + 7 * 10 + 26 * 5
2155: 0 * 200 + 0 * 100 + 4 * 50 + 5 * 20 + 8 * 10 + 24 * 5
2156: 0 * 200 + 0 * 100 + 4 * 50 + 5 * 20 + 9 * 10 + 22 * 5
2157: 0 * 200 + 0 * 100 + 4 * 50 + 5 * 20 + 10 * 10 + 20 * 5
2158: 0 * 200 + 0 * 100 + 4 * 50 + 5 * 20 + 11 * 10 + 18 * 5
2159: 0 * 200 + 0 * 100 + 4 * 50 + 5 * 20 + 12 * 10 + 16 * 5
2160: 0 * 200 + 0 * 100 + 4 * 50 + 5 * 20 + 13 * 10 + 14 * 5
2161: 0 * 200 + 0 * 100 + 4 * 50 + 5 * 20 + 14 * 10 + 12 * 5
2162: 0 * 200 + 0 * 100 + 4 * 50 + 5 * 20 + 15 * 10 + 10 * 5
2163: 0 * 200 + 0 * 100 + 4 * 50 + 5 * 20 + 16 * 10 + 8 * 5
2164: 0 * 200 + 0 * 100 + 4 * 50 + 5 * 20 + 17 * 10 + 6 * 5
2165: 0 * 200 + 0 * 100 + 4 * 50 + 5 * 20 + 18 * 10 + 4 * 5
2166: 0 * 200 + 0 * 100 + 4 * 50 + 5 * 20 + 19 * 10 + 2 * 5
2167: 0 * 200 + 0 * 100 + 4 * 50 + 5 * 20 + 20 * 10 + 0 * 5
2168: 0 * 200 + 0 * 100 + 4 * 50 + 6 * 20 + 0 * 10 + 36 * 5
2169: 0 * 200 + 0 * 100 + 4 * 50 + 6 * 20 + 1 * 10 + 34 * 5
2170: 0 * 200 + 0 * 100 + 4 * 50 + 6 * 20 + 2 * 10 + 32 * 5
2171: 0 * 200 + 0 * 100 + 4 * 50 + 6 * 20 + 3 * 10 + 30 * 5
2172: 0 * 200 + 0 * 100 + 4 * 50 + 6 * 20 + 4 * 10 + 28 * 5
2173: 0 * 200 + 0 * 100 + 4 * 50 + 6 * 20 + 5 * 10 + 26 * 5
2174: 0 * 200 + 0 * 100 + 4 * 50 + 6 * 20 + 6 * 10 + 24 * 5
2175: 0 * 200 + 0 * 100 + 4 * 50 + 6 * 20 + 7 * 10 + 22 * 5
2176: 0 * 200 + 0 * 100 + 4 * 50 + 6 * 20 + 8 * 10 + 20 * 5
2177: 0 * 200 + 0 * 100 + 4 * 50 + 6 * 20 + 9 * 10 + 18 * 5
2178: 0 * 200 + 0 * 100 + 4 * 50 + 6 * 20 + 10 * 10 + 16 * 5
2179: 0 * 200 + 0 * 100 + 4 * 50 + 6 * 20 + 11 * 10 + 14 * 5
2180: 0 * 200 + 0 * 100 + 4 * 50 + 6 * 20 + 12 * 10 + 12 * 5
2181: 0 * 200 + 0 * 100 + 4 * 50 + 6 * 20 + 13 * 10 + 10 * 5
2182: 0 * 200 + 0 * 100 + 4 * 50 + 6 * 20 + 14 * 10 + 8 * 5
2183: 0 * 200 + 0 * 100 + 4 * 50 + 6 * 20 + 15 * 10 + 6 * 5
2184: 0 * 200 + 0 * 100 + 4 * 50 + 6 * 20 + 16 * 10 + 4 * 5
2185: 0 * 200 + 0 * 100 + 4 * 50 + 6 * 20 + 17 * 10 + 2 * 5
2186: 0 * 200 + 0 * 100 + 4 * 50 + 6 * 20 + 18 * 10 + 0 * 5
2187: 0 * 200 + 0 * 100 + 4 * 50 + 7 * 20 + 0 * 10 + 32 * 5
2188: 0 * 200 + 0 * 100 + 4 * 50 + 7 * 20 + 1 * 10 + 30 * 5
2189: 0 * 200 + 0 * 100 + 4 * 50 + 7 * 20 + 2 * 10 + 28 * 5
2190: 0 * 200 + 0 * 100 + 4 * 50 + 7 * 20 + 3 * 10 + 26 * 5
2191: 0 * 200 + 0 * 100 + 4 * 50 + 7 * 20 + 4 * 10 + 24 * 5
2192: 0 * 200 + 0 * 100 + 4 * 50 + 7 * 20 + 5 * 10 + 22 * 5
2193: 0 * 200 + 0 * 100 + 4 * 50 + 7 * 20 + 6 * 10 + 20 * 5
2194: 0 * 200 + 0 * 100 + 4 * 50 + 7 * 20 + 7 * 10 + 18 * 5
2195: 0 * 200 + 0 * 100 + 4 * 50 + 7 * 20 + 8 * 10 + 16 * 5
2196: 0 * 200 + 0 * 100 + 4 * 50 + 7 * 20 + 9 * 10 + 14 * 5
2197: 0 * 200 + 0 * 100 + 4 * 50 + 7 * 20 + 10 * 10 + 12 * 5
2198: 0 * 200 + 0 * 100 + 4 * 50 + 7 * 20 + 11 * 10 + 10 * 5
2199: 0 * 200 + 0 * 100 + 4 * 50 + 7 * 20 + 12 * 10 + 8 * 5
2200: 0 * 200 + 0 * 100 + 4 * 50 + 7 * 20 + 13 * 10 + 6 * 5
2201: 0 * 200 + 0 * 100 + 4 * 50 + 7 * 20 + 14 * 10 + 4 * 5
2202: 0 * 200 + 0 * 100 + 4 * 50 + 7 * 20 + 15 * 10 + 2 * 5
2203: 0 * 200 + 0 * 100 + 4 * 50 + 7 * 20 + 16 * 10 + 0 * 5
2204: 0 * 200 + 0 * 100 + 4 * 50 + 8 * 20 + 0 * 10 + 28 * 5
2205: 0 * 200 + 0 * 100 + 4 * 50 + 8 * 20 + 1 * 10 + 26 * 5
2206: 0 * 200 + 0 * 100 + 4 * 50 + 8 * 20 + 2 * 10 + 24 * 5
2207: 0 * 200 + 0 * 100 + 4 * 50 + 8 * 20 + 3 * 10 + 22 * 5
2208: 0 * 200 + 0 * 100 + 4 * 50 + 8 * 20 + 4 * 10 + 20 * 5
2209: 0 * 200 + 0 * 100 + 4 * 50 + 8 * 20 + 5 * 10 + 18 * 5
2210: 0 * 200 + 0 * 100 + 4 * 50 + 8 * 20 + 6 * 10 + 16 * 5
2211: 0 * 200 + 0 * 100 + 4 * 50 + 8 * 20 + 7 * 10 + 14 * 5
2212: 0 * 200 + 0 * 100 + 4 * 50 + 8 * 20 + 8 * 10 + 12 * 5
2213: 0 * 200 + 0 * 100 + 4 * 50 + 8 * 20 + 9 * 10 + 10 * 5
2214: 0 * 200 + 0 * 100 + 4 * 50 + 8 * 20 + 10 * 10 + 8 * 5
2215: 0 * 200 + 0 * 100 + 4 * 50 + 8 * 20 + 11 * 10 + 6 * 5
2216: 0 * 200 + 0 * 100 + 4 * 50 + 8 * 20 + 12 * 10 + 4 * 5
2217: 0 * 200 + 0 * 100 + 4 * 50 + 8 * 20 + 13 * 10 + 2 * 5
2218: 0 * 200 + 0 * 100 + 4 * 50 + 8 * 20 + 14 * 10 + 0 * 5
2219: 0 * 200 + 0 * 100 + 4 * 50 + 9 * 20 + 0 * 10 + 24 * 5
2220: 0 * 200 + 0 * 100 + 4 * 50 + 9 * 20 + 1 * 10 + 22 * 5
2221: 0 * 200 + 0 * 100 + 4 * 50 + 9 * 20 + 2 * 10 + 20 * 5
2222: 0 * 200 + 0 * 100 + 4 * 50 + 9 * 20 + 3 * 10 + 18 * 5
2223: 0 * 200 + 0 * 100 + 4 * 50 + 9 * 20 + 4 * 10 + 16 * 5
2224: 0 * 200 + 0 * 100 + 4 * 50 + 9 * 20 + 5 * 10 + 14 * 5
2225: 0 * 200 + 0 * 100 + 4 * 50 + 9 * 20 + 6 * 10 + 12 * 5
2226: 0 * 200 + 0 * 100 + 4 * 50 + 9 * 20 + 7 * 10 + 10 * 5
2227: 0 * 200 + 0 * 100 + 4 * 50 + 9 * 20 + 8 * 10 + 8 * 5
2228: 0 * 200 + 0 * 100 + 4 * 50 + 9 * 20 + 9 * 10 + 6 * 5
2229: 0 * 200 + 0 * 100 + 4 * 50 + 9 * 20 + 10 * 10 + 4 * 5
2230: 0 * 200 + 0 * 100 + 4 * 50 + 9 * 20 + 11 * 10 + 2 * 5
2231: 0 * 200 + 0 * 100 + 4 * 50 + 9 * 20 + 12 * 10 + 0 * 5
2232: 0 * 200 + 0 * 100 + 4 * 50 + 10 * 20 + 0 * 10 + 20 * 5
2233: 0 * 200 + 0 * 100 + 4 * 50 + 10 * 20 + 1 * 10 + 18 * 5
2234: 0 * 200 + 0 * 100 + 4 * 50 + 10 * 20 + 2 * 10 + 16 * 5
2235: 0 * 200 + 0 * 100 + 4 * 50 + 10 * 20 + 3 * 10 + 14 * 5
2236: 0 * 200 + 0 * 100 + 4 * 50 + 10 * 20 + 4 * 10 + 12 * 5
2237: 0 * 200 + 0 * 100 + 4 * 50 + 10 * 20 + 5 * 10 + 10 * 5
2238: 0 * 200 + 0 * 100 + 4 * 50 + 10 * 20 + 6 * 10 + 8 * 5
2239: 0 * 200 + 0 * 100 + 4 * 50 + 10 * 20 + 7 * 10 + 6 * 5
2240: 0 * 200 + 0 * 100 + 4 * 50 + 10 * 20 + 8 * 10 + 4 * 5
2241: 0 * 200 + 0 * 100 + 4 * 50 + 10 * 20 + 9 * 10 + 2 * 5
2242: 0 * 200 + 0 * 100 + 4 * 50 + 10 * 20 + 10 * 10 + 0 * 5
2243: 0 * 200 + 0 * 100 + 4 * 50 + 11 * 20 + 0 * 10 + 16 * 5
2244: 0 * 200 + 0 * 100 + 4 * 50 + 11 * 20 + 1 * 10 + 14 * 5
2245: 0 * 200 + 0 * 100 + 4 * 50 + 11 * 20 + 2 * 10 + 12 * 5
2246: 0 * 200 + 0 * 100 + 4 * 50 + 11 * 20 + 3 * 10 + 10 * 5
2247: 0 * 200 + 0 * 100 + 4 * 50 + 11 * 20 + 4 * 10 + 8 * 5
2248: 0 * 200 + 0 * 100 + 4 * 50 + 11 * 20 + 5 * 10 + 6 * 5
2249: 0 * 200 + 0 * 100 + 4 * 50 + 11 * 20 + 6 * 10 + 4 * 5
2250: 0 * 200 + 0 * 100 + 4 * 50 + 11 * 20 + 7 * 10 + 2 * 5
2251: 0 * 200 + 0 * 100 + 4 * 50 + 11 * 20 + 8 * 10 + 0 * 5
2252: 0 * 200 + 0 * 100 + 4 * 50 + 12 * 20 + 0 * 10 + 12 * 5
2253: 0 * 200 + 0 * 100 + 4 * 50 + 12 * 20 + 1 * 10 + 10 * 5
2254: 0 * 200 + 0 * 100 + 4 * 50 + 12 * 20 + 2 * 10 + 8 * 5
2255: 0 * 200 + 0 * 100 + 4 * 50 + 12 * 20 + 3 * 10 + 6 * 5
2256: 0 * 200 + 0 * 100 + 4 * 50 + 12 * 20 + 4 * 10 + 4 * 5
2257: 0 * 200 + 0 * 100 + 4 * 50 + 12 * 20 + 5 * 10 + 2 * 5
2258: 0 * 200 + 0 * 100 + 4 * 50 + 12 * 20 + 6 * 10 + 0 * 5
2259: 0 * 200 + 0 * 100 + 4 * 50 + 13 * 20 + 0 * 10 + 8 * 5
2260: 0 * 200 + 0 * 100 + 4 * 50 + 13 * 20 + 1 * 10 + 6 * 5
2261: 0 * 200 + 0 * 100 + 4 * 50 + 13 * 20 + 2 * 10 + 4 * 5
2262: 0 * 200 + 0 * 100 + 4 * 50 + 13 * 20 + 3 * 10 + 2 * 5
2263: 0 * 200 + 0 * 100 + 4 * 50 + 13 * 20 + 4 * 10 + 0 * 5
2264: 0 * 200 + 0 * 100 + 4 * 50 + 14 * 20 + 0 * 10 + 4 * 5
2265: 0 * 200 + 0 * 100 + 4 * 50 + 14 * 20 + 1 * 10 + 2 * 5
2266: 0 * 200 + 0 * 100 + 4 * 50 + 14 * 20 + 2 * 10 + 0 * 5
2267: 0 * 200 + 0 * 100 + 4 * 50 + 15 * 20 + 0 * 10 + 0 * 5
2268: 0 * 200 + 0 * 100 + 5 * 50 + 0 * 20 + 0 * 10 + 50 * 5
2269: 0 * 200 + 0 * 100 + 5 * 50 + 0 * 20 + 1 * 10 + 48 * 5
2270: 0 * 200 + 0 * 100 + 5 * 50 + 0 * 20 + 2 * 10 + 46 * 5
2271: 0 * 200 + 0 * 100 + 5 * 50 + 0 * 20 + 3 * 10 + 44 * 5
2272: 0 * 200 + 0 * 100 + 5 * 50 + 0 * 20 + 4 * 10 + 42 * 5
2273: 0 * 200 + 0 * 100 + 5 * 50 + 0 * 20 + 5 * 10 + 40 * 5
2274: 0 * 200 + 0 * 100 + 5 * 50 + 0 * 20 + 6 * 10 + 38 * 5
2275: 0 * 200 + 0 * 100 + 5 * 50 + 0 * 20 + 7 * 10 + 36 * 5
2276: 0 * 200 + 0 * 100 + 5 * 50 + 0 * 20 + 8 * 10 + 34 * 5
2277: 0 * 200 + 0 * 100 + 5 * 50 + 0 * 20 + 9 * 10 + 32 * 5
2278: 0 * 200 + 0 * 100 + 5 * 50 + 0 * 20 + 10 * 10 + 30 * 5
2279: 0 * 200 + 0 * 100 + 5 * 50 + 0 * 20 + 11 * 10 + 28 * 5
2280: 0 * 200 + 0 * 100 + 5 * 50 + 0 * 20 + 12 * 10 + 26 * 5
2281: 0 * 200 + 0 * 100 + 5 * 50 + 0 * 20 + 13 * 10 + 24 * 5
2282: 0 * 200 + 0 * 100 + 5 * 50 + 0 * 20 + 14 * 10 + 22 * 5
2283: 0 * 200 + 0 * 100 + 5 * 50 + 0 * 20 + 15 * 10 + 20 * 5
2284: 0 * 200 + 0 * 100 + 5 * 50 + 0 * 20 + 16 * 10 + 18 * 5
2285: 0 * 200 + 0 * 100 + 5 * 50 + 0 * 20 + 17 * 10 + 16 * 5
2286: 0 * 200 + 0 * 100 + 5 * 50 + 0 * 20 + 18 * 10 + 14 * 5
2287: 0 * 200 + 0 * 100 + 5 * 50 + 0 * 20 + 19 * 10 + 12 * 5
2288: 0 * 200 + 0 * 100 + 5 * 50 + 0 * 20 + 20 * 10 + 10 * 5
2289: 0 * 200 + 0 * 100 + 5 * 50 + 0 * 20 + 21 * 10 + 8 * 5
2290: 0 * 200 + 0 * 100 + 5 * 50 + 0 * 20 + 22 * 10 + 6 * 5
2291: 0 * 200 + 0 * 100 + 5 * 50 + 0 * 20 + 23 * 10 + 4 * 5
2292: 0 * 200 + 0 * 100 + 5 * 50 + 0 * 20 + 24 * 10 + 2 * 5
2293: 0 * 200 + 0 * 100 + 5 * 50 + 0 * 20 + 25 * 10 + 0 * 5
2294: 0 * 200 + 0 * 100 + 5 * 50 + 1 * 20 + 0 * 10 + 46 * 5
2295: 0 * 200 + 0 * 100 + 5 * 50 + 1 * 20 + 1 * 10 + 44 * 5
2296: 0 * 200 + 0 * 100 + 5 * 50 + 1 * 20 + 2 * 10 + 42 * 5
2297: 0 * 200 + 0 * 100 + 5 * 50 + 1 * 20 + 3 * 10 + 40 * 5
2298: 0 * 200 + 0 * 100 + 5 * 50 + 1 * 20 + 4 * 10 + 38 * 5
2299: 0 * 200 + 0 * 100 + 5 * 50 + 1 * 20 + 5 * 10 + 36 * 5
2300: 0 * 200 + 0 * 100 + 5 * 50 + 1 * 20 + 6 * 10 + 34 * 5
2301: 0 * 200 + 0 * 100 + 5 * 50 + 1 * 20 + 7 * 10 + 32 * 5
2302: 0 * 200 + 0 * 100 + 5 * 50 + 1 * 20 + 8 * 10 + 30 * 5
2303: 0 * 200 + 0 * 100 + 5 * 50 + 1 * 20 + 9 * 10 + 28 * 5
2304: 0 * 200 + 0 * 100 + 5 * 50 + 1 * 20 + 10 * 10 + 26 * 5
2305: 0 * 200 + 0 * 100 + 5 * 50 + 1 * 20 + 11 * 10 + 24 * 5
2306: 0 * 200 + 0 * 100 + 5 * 50 + 1 * 20 + 12 * 10 + 22 * 5
2307: 0 * 200 + 0 * 100 + 5 * 50 + 1 * 20 + 13 * 10 + 20 * 5
2308: 0 * 200 + 0 * 100 + 5 * 50 + 1 * 20 + 14 * 10 + 18 * 5
2309: 0 * 200 + 0 * 100 + 5 * 50 + 1 * 20 + 15 * 10 + 16 * 5
2310: 0 * 200 + 0 * 100 + 5 * 50 + 1 * 20 + 16 * 10 + 14 * 5
2311: 0 * 200 + 0 * 100 + 5 * 50 + 1 * 20 + 17 * 10 + 12 * 5
2312: 0 * 200 + 0 * 100 + 5 * 50 + 1 * 20 + 18 * 10 + 10 * 5
2313: 0 * 200 + 0 * 100 + 5 * 50 + 1 * 20 + 19 * 10 + 8 * 5
2314: 0 * 200 + 0 * 100 + 5 * 50 + 1 * 20 + 20 * 10 + 6 * 5
2315: 0 * 200 + 0 * 100 + 5 * 50 + 1 * 20 + 21 * 10 + 4 * 5
2316: 0 * 200 + 0 * 100 + 5 * 50 + 1 * 20 + 22 * 10 + 2 * 5
2317: 0 * 200 + 0 * 100 + 5 * 50 + 1 * 20 + 23 * 10 + 0 * 5
2318: 0 * 200 + 0 * 100 + 5 * 50 + 2 * 20 + 0 * 10 + 42 * 5
2319: 0 * 200 + 0 * 100 + 5 * 50 + 2 * 20 + 1 * 10 + 40 * 5
2320: 0 * 200 + 0 * 100 + 5 * 50 + 2 * 20 + 2 * 10 + 38 * 5
2321: 0 * 200 + 0 * 100 + 5 * 50 + 2 * 20 + 3 * 10 + 36 * 5
2322: 0 * 200 + 0 * 100 + 5 * 50 + 2 * 20 + 4 * 10 + 34 * 5
2323: 0 * 200 + 0 * 100 + 5 * 50 + 2 * 20 + 5 * 10 + 32 * 5
2324: 0 * 200 + 0 * 100 + 5 * 50 + 2 * 20 + 6 * 10 + 30 * 5
2325: 0 * 200 + 0 * 100 + 5 * 50 + 2 * 20 + 7 * 10 + 28 * 5
2326: 0 * 200 + 0 * 100 + 5 * 50 + 2 * 20 + 8 * 10 + 26 * 5
2327: 0 * 200 + 0 * 100 + 5 * 50 + 2 * 20 + 9 * 10 + 24 * 5
2328: 0 * 200 + 0 * 100 + 5 * 50 + 2 * 20 + 10 * 10 + 22 * 5
2329: 0 * 200 + 0 * 100 + 5 * 50 + 2 * 20 + 11 * 10 + 20 * 5
2330: 0 * 200 + 0 * 100 + 5 * 50 + 2 * 20 + 12 * 10 + 18 * 5
2331: 0 * 200 + 0 * 100 + 5 * 50 + 2 * 20 + 13 * 10 + 16 * 5
2332: 0 * 200 + 0 * 100 + 5 * 50 + 2 * 20 + 14 * 10 + 14 * 5
2333: 0 * 200 + 0 * 100 + 5 * 50 + 2 * 20 + 15 * 10 + 12 * 5
2334: 0 * 200 + 0 * 100 + 5 * 50 + 2 * 20 + 16 * 10 + 10 * 5
2335: 0 * 200 + 0 * 100 + 5 * 50 + 2 * 20 + 17 * 10 + 8 * 5
2336: 0 * 200 + 0 * 100 + 5 * 50 + 2 * 20 + 18 * 10 + 6 * 5
2337: 0 * 200 + 0 * 100 + 5 * 50 + 2 * 20 + 19 * 10 + 4 * 5
2338: 0 * 200 + 0 * 100 + 5 * 50 + 2 * 20 + 20 * 10 + 2 * 5
2339: 0 * 200 + 0 * 100 + 5 * 50 + 2 * 20 + 21 * 10 + 0 * 5
2340: 0 * 200 + 0 * 100 + 5 * 50 + 3 * 20 + 0 * 10 + 38 * 5
2341: 0 * 200 + 0 * 100 + 5 * 50 + 3 * 20 + 1 * 10 + 36 * 5
2342: 0 * 200 + 0 * 100 + 5 * 50 + 3 * 20 + 2 * 10 + 34 * 5
2343: 0 * 200 + 0 * 100 + 5 * 50 + 3 * 20 + 3 * 10 + 32 * 5
2344: 0 * 200 + 0 * 100 + 5 * 50 + 3 * 20 + 4 * 10 + 30 * 5
2345: 0 * 200 + 0 * 100 + 5 * 50 + 3 * 20 + 5 * 10 + 28 * 5
2346: 0 * 200 + 0 * 100 + 5 * 50 + 3 * 20 + 6 * 10 + 26 * 5
2347: 0 * 200 + 0 * 100 + 5 * 50 + 3 * 20 + 7 * 10 + 24 * 5
2348: 0 * 200 + 0 * 100 + 5 * 50 + 3 * 20 + 8 * 10 + 22 * 5
2349: 0 * 200 + 0 * 100 + 5 * 50 + 3 * 20 + 9 * 10 + 20 * 5
2350: 0 * 200 + 0 * 100 + 5 * 50 + 3 * 20 + 10 * 10 + 18 * 5
2351: 0 * 200 + 0 * 100 + 5 * 50 + 3 * 20 + 11 * 10 + 16 * 5
2352: 0 * 200 + 0 * 100 + 5 * 50 + 3 * 20 + 12 * 10 + 14 * 5
2353: 0 * 200 + 0 * 100 + 5 * 50 + 3 * 20 + 13 * 10 + 12 * 5
2354: 0 * 200 + 0 * 100 + 5 * 50 + 3 * 20 + 14 * 10 + 10 * 5
2355: 0 * 200 + 0 * 100 + 5 * 50 + 3 * 20 + 15 * 10 + 8 * 5
2356: 0 * 200 + 0 * 100 + 5 * 50 + 3 * 20 + 16 * 10 + 6 * 5
2357: 0 * 200 + 0 * 100 + 5 * 50 + 3 * 20 + 17 * 10 + 4 * 5
2358: 0 * 200 + 0 * 100 + 5 * 50 + 3 * 20 + 18 * 10 + 2 * 5
2359: 0 * 200 + 0 * 100 + 5 * 50 + 3 * 20 + 19 * 10 + 0 * 5
2360: 0 * 200 + 0 * 100 + 5 * 50 + 4 * 20 + 0 * 10 + 34 * 5
2361: 0 * 200 + 0 * 100 + 5 * 50 + 4 * 20 + 1 * 10 + 32 * 5
2362: 0 * 200 + 0 * 100 + 5 * 50 + 4 * 20 + 2 * 10 + 30 * 5
2363: 0 * 200 + 0 * 100 + 5 * 50 + 4 * 20 + 3 * 10 + 28 * 5
2364: 0 * 200 + 0 * 100 + 5 * 50 + 4 * 20 + 4 * 10 + 26 * 5
2365: 0 * 200 + 0 * 100 + 5 * 50 + 4 * 20 + 5 * 10 + 24 * 5
2366: 0 * 200 + 0 * 100 + 5 * 50 + 4 * 20 + 6 * 10 + 22 * 5
2367: 0 * 200 + 0 * 100 + 5 * 50 + 4 * 20 + 7 * 10 + 20 * 5
2368: 0 * 200 + 0 * 100 + 5 * 50 + 4 * 20 + 8 * 10 + 18 * 5
2369: 0 * 200 + 0 * 100 + 5 * 50 + 4 * 20 + 9 * 10 + 16 * 5
2370: 0 * 200 + 0 * 100 + 5 * 50 + 4 * 20 + 10 * 10 + 14 * 5
2371: 0 * 200 + 0 * 100 + 5 * 50 + 4 * 20 + 11 * 10 + 12 * 5
2372: 0 * 200 + 0 * 100 + 5 * 50 + 4 * 20 + 12 * 10 + 10 * 5
2373: 0 * 200 + 0 * 100 + 5 * 50 + 4 * 20 + 13 * 10 + 8 * 5
2374: 0 * 200 + 0 * 100 + 5 * 50 + 4 * 20 + 14 * 10 + 6 * 5
2375: 0 * 200 + 0 * 100 + 5 * 50 + 4 * 20 + 15 * 10 + 4 * 5
2376: 0 * 200 + 0 * 100 + 5 * 50 + 4 * 20 + 16 * 10 + 2 * 5
2377: 0 * 200 + 0 * 100 + 5 * 50 + 4 * 20 + 17 * 10 + 0 * 5
2378: 0 * 200 + 0 * 100 + 5 * 50 + 5 * 20 + 0 * 10 + 30 * 5
2379: 0 * 200 + 0 * 100 + 5 * 50 + 5 * 20 + 1 * 10 + 28 * 5
2380: 0 * 200 + 0 * 100 + 5 * 50 + 5 * 20 + 2 * 10 + 26 * 5
2381: 0 * 200 + 0 * 100 + 5 * 50 + 5 * 20 + 3 * 10 + 24 * 5
2382: 0 * 200 + 0 * 100 + 5 * 50 + 5 * 20 + 4 * 10 + 22 * 5
2383: 0 * 200 + 0 * 100 + 5 * 50 + 5 * 20 + 5 * 10 + 20 * 5
2384: 0 * 200 + 0 * 100 + 5 * 50 + 5 * 20 + 6 * 10 + 18 * 5
2385: 0 * 200 + 0 * 100 + 5 * 50 + 5 * 20 + 7 * 10 + 16 * 5
2386: 0 * 200 + 0 * 100 + 5 * 50 + 5 * 20 + 8 * 10 + 14 * 5
2387: 0 * 200 + 0 * 100 + 5 * 50 + 5 * 20 + 9 * 10 + 12 * 5
2388: 0 * 200 + 0 * 100 + 5 * 50 + 5 * 20 + 10 * 10 + 10 * 5
2389: 0 * 200 + 0 * 100 + 5 * 50 + 5 * 20 + 11 * 10 + 8 * 5
2390: 0 * 200 + 0 * 100 + 5 * 50 + 5 * 20 + 12 * 10 + 6 * 5
2391: 0 * 200 + 0 * 100 + 5 * 50 + 5 * 20 + 13 * 10 + 4 * 5
2392: 0 * 200 + 0 * 100 + 5 * 50 + 5 * 20 + 14 * 10 + 2 * 5
2393: 0 * 200 + 0 * 100 + 5 * 50 + 5 * 20 + 15 * 10 + 0 * 5
2394: 0 * 200 + 0 * 100 + 5 * 50 + 6 * 20 + 0 * 10 + 26 * 5
2395: 0 * 200 + 0 * 100 + 5 * 50 + 6 * 20 + 1 * 10 + 24 * 5
2396: 0 * 200 + 0 * 100 + 5 * 50 + 6 * 20 + 2 * 10 + 22 * 5
2397: 0 * 200 + 0 * 100 + 5 * 50 + 6 * 20 + 3 * 10 + 20 * 5
2398: 0 * 200 + 0 * 100 + 5 * 50 + 6 * 20 + 4 * 10 + 18 * 5
2399: 0 * 200 + 0 * 100 + 5 * 50 + 6 * 20 + 5 * 10 + 16 * 5
2400: 0 * 200 + 0 * 100 + 5 * 50 + 6 * 20 + 6 * 10 + 14 * 5
2401: 0 * 200 + 0 * 100 + 5 * 50 + 6 * 20 + 7 * 10 + 12 * 5
2402: 0 * 200 + 0 * 100 + 5 * 50 + 6 * 20 + 8 * 10 + 10 * 5
2403: 0 * 200 + 0 * 100 + 5 * 50 + 6 * 20 + 9 * 10 + 8 * 5
2404: 0 * 200 + 0 * 100 + 5 * 50 + 6 * 20 + 10 * 10 + 6 * 5
2405: 0 * 200 + 0 * 100 + 5 * 50 + 6 * 20 + 11 * 10 + 4 * 5
2406: 0 * 200 + 0 * 100 + 5 * 50 + 6 * 20 + 12 * 10 + 2 * 5
2407: 0 * 200 + 0 * 100 + 5 * 50 + 6 * 20 + 13 * 10 + 0 * 5
2408: 0 * 200 + 0 * 100 + 5 * 50 + 7 * 20 + 0 * 10 + 22 * 5
2409: 0 * 200 + 0 * 100 + 5 * 50 + 7 * 20 + 1 * 10 + 20 * 5
2410: 0 * 200 + 0 * 100 + 5 * 50 + 7 * 20 + 2 * 10 + 18 * 5
2411: 0 * 200 + 0 * 100 + 5 * 50 + 7 * 20 + 3 * 10 + 16 * 5
2412: 0 * 200 + 0 * 100 + 5 * 50 + 7 * 20 + 4 * 10 + 14 * 5
2413: 0 * 200 + 0 * 100 + 5 * 50 + 7 * 20 + 5 * 10 + 12 * 5
2414: 0 * 200 + 0 * 100 + 5 * 50 + 7 * 20 + 6 * 10 + 10 * 5
2415: 0 * 200 + 0 * 100 + 5 * 50 + 7 * 20 + 7 * 10 + 8 * 5
2416: 0 * 200 + 0 * 100 + 5 * 50 + 7 * 20 + 8 * 10 + 6 * 5
2417: 0 * 200 + 0 * 100 + 5 * 50 + 7 * 20 + 9 * 10 + 4 * 5
2418: 0 * 200 + 0 * 100 + 5 * 50 + 7 * 20 + 10 * 10 + 2 * 5
2419: 0 * 200 + 0 * 100 + 5 * 50 + 7 * 20 + 11 * 10 + 0 * 5
2420: 0 * 200 + 0 * 100 + 5 * 50 + 8 * 20 + 0 * 10 + 18 * 5
2421: 0 * 200 + 0 * 100 + 5 * 50 + 8 * 20 + 1 * 10 + 16 * 5
2422: 0 * 200 + 0 * 100 + 5 * 50 + 8 * 20 + 2 * 10 + 14 * 5
2423: 0 * 200 + 0 * 100 + 5 * 50 + 8 * 20 + 3 * 10 + 12 * 5
2424: 0 * 200 + 0 * 100 + 5 * 50 + 8 * 20 + 4 * 10 + 10 * 5
2425: 0 * 200 + 0 * 100 + 5 * 50 + 8 * 20 + 5 * 10 + 8 * 5
2426: 0 * 200 + 0 * 100 + 5 * 50 + 8 * 20 + 6 * 10 + 6 * 5
2427: 0 * 200 + 0 * 100 + 5 * 50 + 8 * 20 + 7 * 10 + 4 * 5
2428: 0 * 200 + 0 * 100 + 5 * 50 + 8 * 20 + 8 * 10 + 2 * 5
2429: 0 * 200 + 0 * 100 + 5 * 50 + 8 * 20 + 9 * 10 + 0 * 5
2430: 0 * 200 + 0 * 100 + 5 * 50 + 9 * 20 + 0 * 10 + 14 * 5
2431: 0 * 200 + 0 * 100 + 5 * 50 + 9 * 20 + 1 * 10 + 12 * 5
2432: 0 * 200 + 0 * 100 + 5 * 50 + 9 * 20 + 2 * 10 + 10 * 5
2433: 0 * 200 + 0 * 100 + 5 * 50 + 9 * 20 + 3 * 10 + 8 * 5
2434: 0 * 200 + 0 * 100 + 5 * 50 + 9 * 20 + 4 * 10 + 6 * 5
2435: 0 * 200 + 0 * 100 + 5 * 50 + 9 * 20 + 5 * 10 + 4 * 5
2436: 0 * 200 + 0 * 100 + 5 * 50 + 9 * 20 + 6 * 10 + 2 * 5
2437: 0 * 200 + 0 * 100 + 5 * 50 + 9 * 20 + 7 * 10 + 0 * 5
2438: 0 * 200 + 0 * 100 + 5 * 50 + 10 * 20 + 0 * 10 + 10 * 5
2439: 0 * 200 + 0 * 100 + 5 * 50 + 10 * 20 + 1 * 10 + 8 * 5
2440: 0 * 200 + 0 * 100 + 5 * 50 + 10 * 20 + 2 * 10 + 6 * 5
2441: 0 * 200 + 0 * 100 + 5 * 50 + 10 * 20 + 3 * 10 + 4 * 5
2442: 0 * 200 + 0 * 100 + 5 * 50 + 10 * 20 + 4 * 10 + 2 * 5
2443: 0 * 200 + 0 * 100 + 5 * 50 + 10 * 20 + 5 * 10 + 0 * 5
2444: 0 * 200 + 0 * 100 + 5 * 50 + 11 * 20 + 0 * 10 + 6 * 5
2445: 0 * 200 + 0 * 100 + 5 * 50 + 11 * 20 + 1 * 10 + 4 * 5
2446: 0 * 200 + 0 * 100 + 5 * 50 + 11 * 20 + 2 * 10 + 2 * 5
2447: 0 * 200 + 0 * 100 + 5 * 50 + 11 * 20 + 3 * 10 + 0 * 5
2448: 0 * 200 + 0 * 100 + 5 * 50 + 12 * 20 + 0 * 10 + 2 * 5
2449: 0 * 200 + 0 * 100 + 5 * 50 + 12 * 20 + 1 * 10 + 0 * 5
2450: 0 * 200 + 0 * 100 + 6 * 50 + 0 * 20 + 0 * 10 + 40 * 5
2451: 0 * 200 + 0 * 100 + 6 * 50 + 0 * 20 + 1 * 10 + 38 * 5
2452: 0 * 200 + 0 * 100 + 6 * 50 + 0 * 20 + 2 * 10 + 36 * 5
2453: 0 * 200 + 0 * 100 + 6 * 50 + 0 * 20 + 3 * 10 + 34 * 5
2454: 0 * 200 + 0 * 100 + 6 * 50 + 0 * 20 + 4 * 10 + 32 * 5
2455: 0 * 200 + 0 * 100 + 6 * 50 + 0 * 20 + 5 * 10 + 30 * 5
2456: 0 * 200 + 0 * 100 + 6 * 50 + 0 * 20 + 6 * 10 + 28 * 5
2457: 0 * 200 + 0 * 100 + 6 * 50 + 0 * 20 + 7 * 10 + 26 * 5
2458: 0 * 200 + 0 * 100 + 6 * 50 + 0 * 20 + 8 * 10 + 24 * 5
2459: 0 * 200 + 0 * 100 + 6 * 50 + 0 * 20 + 9 * 10 + 22 * 5
2460: 0 * 200 + 0 * 100 + 6 * 50 + 0 * 20 + 10 * 10 + 20 * 5
2461: 0 * 200 + 0 * 100 + 6 * 50 + 0 * 20 + 11 * 10 + 18 * 5
2462: 0 * 200 + 0 * 100 + 6 * 50 + 0 * 20 + 12 * 10 + 16 * 5
2463: 0 * 200 + 0 * 100 + 6 * 50 + 0 * 20 + 13 * 10 + 14 * 5
2464: 0 * 200 + 0 * 100 + 6 * 50 + 0 * 20 + 14 * 10 + 12 * 5
2465: 0 * 200 + 0 * 100 + 6 * 50 + 0 * 20 + 15 * 10 + 10 * 5
2466: 0 * 200 + 0 * 100 + 6 * 50 + 0 * 20 + 16 * 10 + 8 * 5
2467: 0 * 200 + 0 * 100 + 6 * 50 + 0 * 20 + 17 * 10 + 6 * 5
2468: 0 * 200 + 0 * 100 + 6 * 50 + 0 * 20 + 18 * 10 + 4 * 5
2469: 0 * 200 + 0 * 100 + 6 * 50 + 0 * 20 + 19 * 10 + 2 * 5
2470: 0 * 200 + 0 * 100 + 6 * 50 + 0 * 20 + 20 * 10 + 0 * 5
2471: 0 * 200 + 0 * 100 + 6 * 50 + 1 * 20 + 0 * 10 + 36 * 5
2472: 0 * 200 + 0 * 100 + 6 * 50 + 1 * 20 + 1 * 10 + 34 * 5
2473: 0 * 200 + 0 * 100 + 6 * 50 + 1 * 20 + 2 * 10 + 32 * 5
2474: 0 * 200 + 0 * 100 + 6 * 50 + 1 * 20 + 3 * 10 + 30 * 5
2475: 0 * 200 + 0 * 100 + 6 * 50 + 1 * 20 + 4 * 10 + 28 * 5
2476: 0 * 200 + 0 * 100 + 6 * 50 + 1 * 20 + 5 * 10 + 26 * 5
2477: 0 * 200 + 0 * 100 + 6 * 50 + 1 * 20 + 6 * 10 + 24 * 5
2478: 0 * 200 + 0 * 100 + 6 * 50 + 1 * 20 + 7 * 10 + 22 * 5
2479: 0 * 200 + 0 * 100 + 6 * 50 + 1 * 20 + 8 * 10 + 20 * 5
2480: 0 * 200 + 0 * 100 + 6 * 50 + 1 * 20 + 9 * 10 + 18 * 5
2481: 0 * 200 + 0 * 100 + 6 * 50 + 1 * 20 + 10 * 10 + 16 * 5
2482: 0 * 200 + 0 * 100 + 6 * 50 + 1 * 20 + 11 * 10 + 14 * 5
2483: 0 * 200 + 0 * 100 + 6 * 50 + 1 * 20 + 12 * 10 + 12 * 5
2484: 0 * 200 + 0 * 100 + 6 * 50 + 1 * 20 + 13 * 10 + 10 * 5
2485: 0 * 200 + 0 * 100 + 6 * 50 + 1 * 20 + 14 * 10 + 8 * 5
2486: 0 * 200 + 0 * 100 + 6 * 50 + 1 * 20 + 15 * 10 + 6 * 5
2487: 0 * 200 + 0 * 100 + 6 * 50 + 1 * 20 + 16 * 10 + 4 * 5
2488: 0 * 200 + 0 * 100 + 6 * 50 + 1 * 20 + 17 * 10 + 2 * 5
2489: 0 * 200 + 0 * 100 + 6 * 50 + 1 * 20 + 18 * 10 + 0 * 5
2490: 0 * 200 + 0 * 100 + 6 * 50 + 2 * 20 + 0 * 10 + 32 * 5
2491: 0 * 200 + 0 * 100 + 6 * 50 + 2 * 20 + 1 * 10 + 30 * 5
2492: 0 * 200 + 0 * 100 + 6 * 50 + 2 * 20 + 2 * 10 + 28 * 5
2493: 0 * 200 + 0 * 100 + 6 * 50 + 2 * 20 + 3 * 10 + 26 * 5
2494: 0 * 200 + 0 * 100 + 6 * 50 + 2 * 20 + 4 * 10 + 24 * 5
2495: 0 * 200 + 0 * 100 + 6 * 50 + 2 * 20 + 5 * 10 + 22 * 5
2496: 0 * 200 + 0 * 100 + 6 * 50 + 2 * 20 + 6 * 10 + 20 * 5
2497: 0 * 200 + 0 * 100 + 6 * 50 + 2 * 20 + 7 * 10 + 18 * 5
2498: 0 * 200 + 0 * 100 + 6 * 50 + 2 * 20 + 8 * 10 + 16 * 5
2499: 0 * 200 + 0 * 100 + 6 * 50 + 2 * 20 + 9 * 10 + 14 * 5
2500: 0 * 200 + 0 * 100 + 6 * 50 + 2 * 20 + 10 * 10 + 12 * 5
2501: 0 * 200 + 0 * 100 + 6 * 50 + 2 * 20 + 11 * 10 + 10 * 5
2502: 0 * 200 + 0 * 100 + 6 * 50 + 2 * 20 + 12 * 10 + 8 * 5
2503: 0 * 200 + 0 * 100 + 6 * 50 + 2 * 20 + 13 * 10 + 6 * 5
2504: 0 * 200 + 0 * 100 + 6 * 50 + 2 * 20 + 14 * 10 + 4 * 5
2505: 0 * 200 + 0 * 100 + 6 * 50 + 2 * 20 + 15 * 10 + 2 * 5
2506: 0 * 200 + 0 * 100 + 6 * 50 + 2 * 20 + 16 * 10 + 0 * 5
2507: 0 * 200 + 0 * 100 + 6 * 50 + 3 * 20 + 0 * 10 + 28 * 5
2508: 0 * 200 + 0 * 100 + 6 * 50 + 3 * 20 + 1 * 10 + 26 * 5
2509: 0 * 200 + 0 * 100 + 6 * 50 + 3 * 20 + 2 * 10 + 24 * 5
2510: 0 * 200 + 0 * 100 + 6 * 50 + 3 * 20 + 3 * 10 + 22 * 5
2511: 0 * 200 + 0 * 100 + 6 * 50 + 3 * 20 + 4 * 10 + 20 * 5
2512: 0 * 200 + 0 * 100 + 6 * 50 + 3 * 20 + 5 * 10 + 18 * 5
2513: 0 * 200 + 0 * 100 + 6 * 50 + 3 * 20 + 6 * 10 + 16 * 5
2514: 0 * 200 + 0 * 100 + 6 * 50 + 3 * 20 + 7 * 10 + 14 * 5
2515: 0 * 200 + 0 * 100 + 6 * 50 + 3 * 20 + 8 * 10 + 12 * 5
2516: 0 * 200 + 0 * 100 + 6 * 50 + 3 * 20 + 9 * 10 + 10 * 5
2517: 0 * 200 + 0 * 100 + 6 * 50 + 3 * 20 + 10 * 10 + 8 * 5
2518: 0 * 200 + 0 * 100 + 6 * 50 + 3 * 20 + 11 * 10 + 6 * 5
2519: 0 * 200 + 0 * 100 + 6 * 50 + 3 * 20 + 12 * 10 + 4 * 5
2520: 0 * 200 + 0 * 100 + 6 * 50 + 3 * 20 + 13 * 10 + 2 * 5
2521: 0 * 200 + 0 * 100 + 6 * 50 + 3 * 20 + 14 * 10 + 0 * 5
2522: 0 * 200 + 0 * 100 + 6 * 50 + 4 * 20 + 0 * 10 + 24 * 5
2523: 0 * 200 + 0 * 100 + 6 * 50 + 4 * 20 + 1 * 10 + 22 * 5
2524: 0 * 200 + 0 * 100 + 6 * 50 + 4 * 20 + 2 * 10 + 20 * 5
2525: 0 * 200 + 0 * 100 + 6 * 50 + 4 * 20 + 3 * 10 + 18 * 5
2526: 0 * 200 + 0 * 100 + 6 * 50 + 4 * 20 + 4 * 10 + 16 * 5
2527: 0 * 200 + 0 * 100 + 6 * 50 + 4 * 20 + 5 * 10 + 14 * 5
2528: 0 * 200 + 0 * 100 + 6 * 50 + 4 * 20 + 6 * 10 + 12 * 5
2529: 0 * 200 + 0 * 100 + 6 * 50 + 4 * 20 + 7 * 10 + 10 * 5
2530: 0 * 200 + 0 * 100 + 6 * 50 + 4 * 20 + 8 * 10 + 8 * 5
2531: 0 * 200 + 0 * 100 + 6 * 50 + 4 * 20 + 9 * 10 + 6 * 5
2532: 0 * 200 + 0 * 100 + 6 * 50 + 4 * 20 + 10 * 10 + 4 * 5
2533: 0 * 200 + 0 * 100 + 6 * 50 + 4 * 20 + 11 * 10 + 2 * 5
2534: 0 * 200 + 0 * 100 + 6 * 50 + 4 * 20 + 12 * 10 + 0 * 5
2535: 0 * 200 + 0 * 100 + 6 * 50 + 5 * 20 + 0 * 10 + 20 * 5
2536: 0 * 200 + 0 * 100 + 6 * 50 + 5 * 20 + 1 * 10 + 18 * 5
2537: 0 * 200 + 0 * 100 + 6 * 50 + 5 * 20 + 2 * 10 + 16 * 5
2538: 0 * 200 + 0 * 100 + 6 * 50 + 5 * 20 + 3 * 10 + 14 * 5
2539: 0 * 200 + 0 * 100 + 6 * 50 + 5 * 20 + 4 * 10 + 12 * 5
2540: 0 * 200 + 0 * 100 + 6 * 50 + 5 * 20 + 5 * 10 + 10 * 5
2541: 0 * 200 + 0 * 100 + 6 * 50 + 5 * 20 + 6 * 10 + 8 * 5
2542: 0 * 200 + 0 * 100 + 6 * 50 + 5 * 20 + 7 * 10 + 6 * 5
2543: 0 * 200 + 0 * 100 + 6 * 50 + 5 * 20 + 8 * 10 + 4 * 5
2544: 0 * 200 + 0 * 100 + 6 * 50 + 5 * 20 + 9 * 10 + 2 * 5
2545: 0 * 200 + 0 * 100 + 6 * 50 + 5 * 20 + 10 * 10 + 0 * 5
2546: 0 * 200 + 0 * 100 + 6 * 50 + 6 * 20 + 0 * 10 + 16 * 5
2547: 0 * 200 + 0 * 100 + 6 * 50 + 6 * 20 + 1 * 10 + 14 * 5
2548: 0 * 200 + 0 * 100 + 6 * 50 + 6 * 20 + 2 * 10 + 12 * 5
2549: 0 * 200 + 0 * 100 + 6 * 50 + 6 * 20 + 3 * 10 + 10 * 5
2550: 0 * 200 + 0 * 100 + 6 * 50 + 6 * 20 + 4 * 10 + 8 * 5
2551: 0 * 200 + 0 * 100 + 6 * 50 + 6 * 20 + 5 * 10 + 6 * 5
2552: 0 * 200 + 0 * 100 + 6 * 50 + 6 * 20 + 6 * 10 + 4 * 5
2553: 0 * 200 + 0 * 100 + 6 * 50 + 6 * 20 + 7 * 10 + 2 * 5
2554: 0 * 200 + 0 * 100 + 6 * 50 + 6 * 20 + 8 * 10 + 0 * 5
2555: 0 * 200 + 0 * 100 + 6 * 50 + 7 * 20 + 0 * 10 + 12 * 5
2556: 0 * 200 + 0 * 100 + 6 * 50 + 7 * 20 + 1 * 10 + 10 * 5
2557: 0 * 200 + 0 * 100 + 6 * 50 + 7 * 20 + 2 * 10 + 8 * 5
2558: 0 * 200 + 0 * 100 + 6 * 50 + 7 * 20 + 3 * 10 + 6 * 5
2559: 0 * 200 + 0 * 100 + 6 * 50 + 7 * 20 + 4 * 10 + 4 * 5
2560: 0 * 200 + 0 * 100 + 6 * 50 + 7 * 20 + 5 * 10 + 2 * 5
2561: 0 * 200 + 0 * 100 + 6 * 50 + 7 * 20 + 6 * 10 + 0 * 5
2562: 0 * 200 + 0 * 100 + 6 * 50 + 8 * 20 + 0 * 10 + 8 * 5
2563: 0 * 200 + 0 * 100 + 6 * 50 + 8 * 20 + 1 * 10 + 6 * 5
2564: 0 * 200 + 0 * 100 + 6 * 50 + 8 * 20 + 2 * 10 + 4 * 5
2565: 0 * 200 + 0 * 100 + 6 * 50 + 8 * 20 + 3 * 10 + 2 * 5
2566: 0 * 200 + 0 * 100 + 6 * 50 + 8 * 20 + 4 * 10 + 0 * 5
2567: 0 * 200 + 0 * 100 + 6 * 50 + 9 * 20 + 0 * 10 + 4 * 5
2568: 0 * 200 + 0 * 100 + 6 * 50 + 9 * 20 + 1 * 10 + 2 * 5
2569: 0 * 200 + 0 * 100 + 6 * 50 + 9 * 20 + 2 * 10 + 0 * 5
2570: 0 * 200 + 0 * 100 + 6 * 50 + 10 * 20 + 0 * 10 + 0 * 5
2571: 0 * 200 + 0 * 100 + 7 * 50 + 0 * 20 + 0 * 10 + 30 * 5
2572: 0 * 200 + 0 * 100 + 7 * 50 + 0 * 20 + 1 * 10 + 28 * 5
2573: 0 * 200 + 0 * 100 + 7 * 50 + 0 * 20 + 2 * 10 + 26 * 5
2574: 0 * 200 + 0 * 100 + 7 * 50 + 0 * 20 + 3 * 10 + 24 * 5
2575: 0 * 200 + 0 * 100 + 7 * 50 + 0 * 20 + 4 * 10 + 22 * 5
2576: 0 * 200 + 0 * 100 + 7 * 50 + 0 * 20 + 5 * 10 + 20 * 5
2577: 0 * 200 + 0 * 100 + 7 * 50 + 0 * 20 + 6 * 10 + 18 * 5
2578: 0 * 200 + 0 * 100 + 7 * 50 + 0 * 20 + 7 * 10 + 16 * 5
2579: 0 * 200 + 0 * 100 + 7 * 50 + 0 * 20 + 8 * 10 + 14 * 5
2580: 0 * 200 + 0 * 100 + 7 * 50 + 0 * 20 + 9 * 10 + 12 * 5
2581: 0 * 200 + 0 * 100 + 7 * 50 + 0 * 20 + 10 * 10 + 10 * 5
2582: 0 * 200 + 0 * 100 + 7 * 50 + 0 * 20 + 11 * 10 + 8 * 5
2583: 0 * 200 + 0 * 100 + 7 * 50 + 0 * 20 + 12 * 10 + 6 * 5
2584: 0 * 200 + 0 * 100 + 7 * 50 + 0 * 20 + 13 * 10 + 4 * 5
2585: 0 * 200 + 0 * 100 + 7 * 50 + 0 * 20 + 14 * 10 + 2 * 5
2586: 0 * 200 + 0 * 100 + 7 * 50 + 0 * 20 + 15 * 10 + 0 * 5
2587: 0 * 200 + 0 * 100 + 7 * 50 + 1 * 20 + 0 * 10 + 26 * 5
2588: 0 * 200 + 0 * 100 + 7 * 50 + 1 * 20 + 1 * 10 + 24 * 5
2589: 0 * 200 + 0 * 100 + 7 * 50 + 1 * 20 + 2 * 10 + 22 * 5
2590: 0 * 200 + 0 * 100 + 7 * 50 + 1 * 20 + 3 * 10 + 20 * 5
2591: 0 * 200 + 0 * 100 + 7 * 50 + 1 * 20 + 4 * 10 + 18 * 5
2592: 0 * 200 + 0 * 100 + 7 * 50 + 1 * 20 + 5 * 10 + 16 * 5
2593: 0 * 200 + 0 * 100 + 7 * 50 + 1 * 20 + 6 * 10 + 14 * 5
2594: 0 * 200 + 0 * 100 + 7 * 50 + 1 * 20 + 7 * 10 + 12 * 5
2595: 0 * 200 + 0 * 100 + 7 * 50 + 1 * 20 + 8 * 10 + 10 * 5
2596: 0 * 200 + 0 * 100 + 7 * 50 + 1 * 20 + 9 * 10 + 8 * 5
2597: 0 * 200 + 0 * 100 + 7 * 50 + 1 * 20 + 10 * 10 + 6 * 5
2598: 0 * 200 + 0 * 100 + 7 * 50 + 1 * 20 + 11 * 10 + 4 * 5
2599: 0 * 200 + 0 * 100 + 7 * 50 + 1 * 20 + 12 * 10 + 2 * 5
2600: 0 * 200 + 0 * 100 + 7 * 50 + 1 * 20 + 13 * 10 + 0 * 5
2601: 0 * 200 + 0 * 100 + 7 * 50 + 2 * 20 + 0 * 10 + 22 * 5
2602: 0 * 200 + 0 * 100 + 7 * 50 + 2 * 20 + 1 * 10 + 20 * 5
2603: 0 * 200 + 0 * 100 + 7 * 50 + 2 * 20 + 2 * 10 + 18 * 5
2604: 0 * 200 + 0 * 100 + 7 * 50 + 2 * 20 + 3 * 10 + 16 * 5
2605: 0 * 200 + 0 * 100 + 7 * 50 + 2 * 20 + 4 * 10 + 14 * 5
2606: 0 * 200 + 0 * 100 + 7 * 50 + 2 * 20 + 5 * 10 + 12 * 5
2607: 0 * 200 + 0 * 100 + 7 * 50 + 2 * 20 + 6 * 10 + 10 * 5
2608: 0 * 200 + 0 * 100 + 7 * 50 + 2 * 20 + 7 * 10 + 8 * 5
2609: 0 * 200 + 0 * 100 + 7 * 50 + 2 * 20 + 8 * 10 + 6 * 5
2610: 0 * 200 + 0 * 100 + 7 * 50 + 2 * 20 + 9 * 10 + 4 * 5
2611: 0 * 200 + 0 * 100 + 7 * 50 + 2 * 20 + 10 * 10 + 2 * 5
2612: 0 * 200 + 0 * 100 + 7 * 50 + 2 * 20 + 11 * 10 + 0 * 5
2613: 0 * 200 + 0 * 100 + 7 * 50 + 3 * 20 + 0 * 10 + 18 * 5
2614: 0 * 200 + 0 * 100 + 7 * 50 + 3 * 20 + 1 * 10 + 16 * 5
2615: 0 * 200 + 0 * 100 + 7 * 50 + 3 * 20 + 2 * 10 + 14 * 5
2616: 0 * 200 + 0 * 100 + 7 * 50 + 3 * 20 + 3 * 10 + 12 * 5
2617: 0 * 200 + 0 * 100 + 7 * 50 + 3 * 20 + 4 * 10 + 10 * 5
2618: 0 * 200 + 0 * 100 + 7 * 50 + 3 * 20 + 5 * 10 + 8 * 5
2619: 0 * 200 + 0 * 100 + 7 * 50 + 3 * 20 + 6 * 10 + 6 * 5
2620: 0 * 200 + 0 * 100 + 7 * 50 + 3 * 20 + 7 * 10 + 4 * 5
2621: 0 * 200 + 0 * 100 + 7 * 50 + 3 * 20 + 8 * 10 + 2 * 5
2622: 0 * 200 + 0 * 100 + 7 * 50 + 3 * 20 + 9 * 10 + 0 * 5
2623: 0 * 200 + 0 * 100 + 7 * 50 + 4 * 20 + 0 * 10 + 14 * 5
2624: 0 * 200 + 0 * 100 + 7 * 50 + 4 * 20 + 1 * 10 + 12 * 5
2625: 0 * 200 + 0 * 100 + 7 * 50 + 4 * 20 + 2 * 10 + 10 * 5
2626: 0 * 200 + 0 * 100 + 7 * 50 + 4 * 20 + 3 * 10 + 8 * 5
2627: 0 * 200 + 0 * 100 + 7 * 50 + 4 * 20 + 4 * 10 + 6 * 5
2628: 0 * 200 + 0 * 100 + 7 * 50 + 4 * 20 + 5 * 10 + 4 * 5
2629: 0 * 200 + 0 * 100 + 7 * 50 + 4 * 20 + 6 * 10 + 2 * 5
2630: 0 * 200 + 0 * 100 + 7 * 50 + 4 * 20 + 7 * 10 + 0 * 5
2631: 0 * 200 + 0 * 100 + 7 * 50 + 5 * 20 + 0 * 10 + 10 * 5
2632: 0 * 200 + 0 * 100 + 7 * 50 + 5 * 20 + 1 * 10 + 8 * 5
2633: 0 * 200 + 0 * 100 + 7 * 50 + 5 * 20 + 2 * 10 + 6 * 5
2634: 0 * 200 + 0 * 100 + 7 * 50 + 5 * 20 + 3 * 10 + 4 * 5
2635: 0 * 200 + 0 * 100 + 7 * 50 + 5 * 20 + 4 * 10 + 2 * 5
2636: 0 * 200 + 0 * 100 + 7 * 50 + 5 * 20 + 5 * 10 + 0 * 5
2637: 0 * 200 + 0 * 100 + 7 * 50 + 6 * 20 + 0 * 10 + 6 * 5
2638: 0 * 200 + 0 * 100 + 7 * 50 + 6 * 20 + 1 * 10 + 4 * 5
2639: 0 * 200 + 0 * 100 + 7 * 50 + 6 * 20 + 2 * 10 + 2 * 5
2640: 0 * 200 + 0 * 100 + 7 * 50 + 6 * 20 + 3 * 10 + 0 * 5
2641: 0 * 200 + 0 * 100 + 7 * 50 + 7 * 20 + 0 * 10 + 2 * 5
2642: 0 * 200 + 0 * 100 + 7 * 50 + 7 * 20 + 1 * 10 + 0 * 5
2643: 0 * 200 + 0 * 100 + 8 * 50 + 0 * 20 + 0 * 10 + 20 * 5
2644: 0 * 200 + 0 * 100 + 8 * 50 + 0 * 20 + 1 * 10 + 18 * 5
2645: 0 * 200 + 0 * 100 + 8 * 50 + 0 * 20 + 2 * 10 + 16 * 5
2646: 0 * 200 + 0 * 100 + 8 * 50 + 0 * 20 + 3 * 10 + 14 * 5
2647: 0 * 200 + 0 * 100 + 8 * 50 + 0 * 20 + 4 * 10 + 12 * 5
2648: 0 * 200 + 0 * 100 + 8 * 50 + 0 * 20 + 5 * 10 + 10 * 5
2649: 0 * 200 + 0 * 100 + 8 * 50 + 0 * 20 + 6 * 10 + 8 * 5
2650: 0 * 200 + 0 * 100 + 8 * 50 + 0 * 20 + 7 * 10 + 6 * 5
2651: 0 * 200 + 0 * 100 + 8 * 50 + 0 * 20 + 8 * 10 + 4 * 5
2652: 0 * 200 + 0 * 100 + 8 * 50 + 0 * 20 + 9 * 10 + 2 * 5
2653: 0 * 200 + 0 * 100 + 8 * 50 + 0 * 20 + 10 * 10 + 0 * 5
2654: 0 * 200 + 0 * 100 + 8 * 50 + 1 * 20 + 0 * 10 + 16 * 5
2655: 0 * 200 + 0 * 100 + 8 * 50 + 1 * 20 + 1 * 10 + 14 * 5
2656: 0 * 200 + 0 * 100 + 8 * 50 + 1 * 20 + 2 * 10 + 12 * 5
2657: 0 * 200 + 0 * 100 + 8 * 50 + 1 * 20 + 3 * 10 + 10 * 5
2658: 0 * 200 + 0 * 100 + 8 * 50 + 1 * 20 + 4 * 10 + 8 * 5
2659: 0 * 200 + 0 * 100 + 8 * 50 + 1 * 20 + 5 * 10 + 6 * 5
2660: 0 * 200 + 0 * 100 + 8 * 50 + 1 * 20 + 6 * 10 + 4 * 5
2661: 0 * 200 + 0 * 100 + 8 * 50 + 1 * 20 + 7 * 10 + 2 * 5
2662: 0 * 200 + 0 * 100 + 8 * 50 + 1 * 20 + 8 * 10 + 0 * 5
2663: 0 * 200 + 0 * 100 + 8 * 50 + 2 * 20 + 0 * 10 + 12 * 5
2664: 0 * 200 + 0 * 100 + 8 * 50 + 2 * 20 + 1 * 10 + 10 * 5
2665: 0 * 200 + 0 * 100 + 8 * 50 + 2 * 20 + 2 * 10 + 8 * 5
2666: 0 * 200 + 0 * 100 + 8 * 50 + 2 * 20 + 3 * 10 + 6 * 5
2667: 0 * 200 + 0 * 100 + 8 * 50 + 2 * 20 + 4 * 10 + 4 * 5
2668: 0 * 200 + 0 * 100 + 8 * 50 + 2 * 20 + 5 * 10 + 2 * 5
2669: 0 * 200 + 0 * 100 + 8 * 50 + 2 * 20 + 6 * 10 + 0 * 5
2670: 0 * 200 + 0 * 100 + 8 * 50 + 3 * 20 + 0 * 10 + 8 * 5
2671: 0 * 200 + 0 * 100 + 8 * 50 + 3 * 20 + 1 * 10 + 6 * 5
2672: 0 * 200 + 0 * 100 + 8 * 50 + 3 * 20 + 2 * 10 + 4 * 5
2673: 0 * 200 + 0 * 100 + 8 * 50 + 3 * 20 + 3 * 10 + 2 * 5
2674: 0 * 200 + 0 * 100 + 8 * 50 + 3 * 20 + 4 * 10 + 0 * 5
2675: 0 * 200 + 0 * 100 + 8 * 50 + 4 * 20 + 0 * 10 + 4 * 5
2676: 0 * 200 + 0 * 100 + 8 * 50 + 4 * 20 + 1 * 10 + 2 * 5
2677: 0 * 200 + 0 * 100 + 8 * 50 + 4 * 20 + 2 * 10 + 0 * 5
2678: 0 * 200 + 0 * 100 + 8 * 50 + 5 * 20 + 0 * 10 + 0 * 5
2679: 0 * 200 + 0 * 100 + 9 * 50 + 0 * 20 + 0 * 10 + 10 * 5
2680: 0 * 200 + 0 * 100 + 9 * 50 + 0 * 20 + 1 * 10 + 8 * 5
2681: 0 * 200 + 0 * 100 + 9 * 50 + 0 * 20 + 2 * 10 + 6 * 5
2682: 0 * 200 + 0 * 100 + 9 * 50 + 0 * 20 + 3 * 10 + 4 * 5
2683: 0 * 200 + 0 * 100 + 9 * 50 + 0 * 20 + 4 * 10 + 2 * 5
2684: 0 * 200 + 0 * 100 + 9 * 50 + 0 * 20 + 5 * 10 + 0 * 5
2685: 0 * 200 + 0 * 100 + 9 * 50 + 1 * 20 + 0 * 10 + 6 * 5
2686: 0 * 200 + 0 * 100 + 9 * 50 + 1 * 20 + 1 * 10 + 4 * 5
2687: 0 * 200 + 0 * 100 + 9 * 50 + 1 * 20 + 2 * 10 + 2 * 5
2688: 0 * 200 + 0 * 100 + 9 * 50 + 1 * 20 + 3 * 10 + 0 * 5
2689: 0 * 200 + 0 * 100 + 9 * 50 + 2 * 20 + 0 * 10 + 2 * 5
2690: 0 * 200 + 0 * 100 + 9 * 50 + 2 * 20 + 1 * 10 + 0 * 5
2691: 0 * 200 + 0 * 100 + 10 * 50 + 0 * 20 + 0 * 10 + 0 * 5
2692: 0 * 200 + 1 * 100 + 0 * 50 + 0 * 20 + 0 * 10 + 80 * 5
2693: 0 * 200 + 1 * 100 + 0 * 50 + 0 * 20 + 1 * 10 + 78 * 5
2694: 0 * 200 + 1 * 100 + 0 * 50 + 0 * 20 + 2 * 10 + 76 * 5
2695: 0 * 200 + 1 * 100 + 0 * 50 + 0 * 20 + 3 * 10 + 74 * 5
2696: 0 * 200 + 1 * 100 + 0 * 50 + 0 * 20 + 4 * 10 + 72 * 5
2697: 0 * 200 + 1 * 100 + 0 * 50 + 0 * 20 + 5 * 10 + 70 * 5
2698: 0 * 200 + 1 * 100 + 0 * 50 + 0 * 20 + 6 * 10 + 68 * 5
2699: 0 * 200 + 1 * 100 + 0 * 50 + 0 * 20 + 7 * 10 + 66 * 5
2700: 0 * 200 + 1 * 100 + 0 * 50 + 0 * 20 + 8 * 10 + 64 * 5
2701: 0 * 200 + 1 * 100 + 0 * 50 + 0 * 20 + 9 * 10 + 62 * 5
2702: 0 * 200 + 1 * 100 + 0 * 50 + 0 * 20 + 10 * 10 + 60 * 5
2703: 0 * 200 + 1 * 100 + 0 * 50 + 0 * 20 + 11 * 10 + 58 * 5
2704: 0 * 200 + 1 * 100 + 0 * 50 + 0 * 20 + 12 * 10 + 56 * 5
2705: 0 * 200 + 1 * 100 + 0 * 50 + 0 * 20 + 13 * 10 + 54 * 5
2706: 0 * 200 + 1 * 100 + 0 * 50 + 0 * 20 + 14 * 10 + 52 * 5
2707: 0 * 200 + 1 * 100 + 0 * 50 + 0 * 20 + 15 * 10 + 50 * 5
2708: 0 * 200 + 1 * 100 + 0 * 50 + 0 * 20 + 16 * 10 + 48 * 5
2709: 0 * 200 + 1 * 100 + 0 * 50 + 0 * 20 + 17 * 10 + 46 * 5
2710: 0 * 200 + 1 * 100 + 0 * 50 + 0 * 20 + 18 * 10 + 44 * 5
2711: 0 * 200 + 1 * 100 + 0 * 50 + 0 * 20 + 19 * 10 + 42 * 5
2712: 0 * 200 + 1 * 100 + 0 * 50 + 0 * 20 + 20 * 10 + 40 * 5
2713: 0 * 200 + 1 * 100 + 0 * 50 + 0 * 20 + 21 * 10 + 38 * 5
2714: 0 * 200 + 1 * 100 + 0 * 50 + 0 * 20 + 22 * 10 + 36 * 5
2715: 0 * 200 + 1 * 100 + 0 * 50 + 0 * 20 + 23 * 10 + 34 * 5
2716: 0 * 200 + 1 * 100 + 0 * 50 + 0 * 20 + 24 * 10 + 32 * 5
2717: 0 * 200 + 1 * 100 + 0 * 50 + 0 * 20 + 25 * 10 + 30 * 5
2718: 0 * 200 + 1 * 100 + 0 * 50 + 0 * 20 + 26 * 10 + 28 * 5
2719: 0 * 200 + 1 * 100 + 0 * 50 + 0 * 20 + 27 * 10 + 26 * 5
2720: 0 * 200 + 1 * 100 + 0 * 50 + 0 * 20 + 28 * 10 + 24 * 5
2721: 0 * 200 + 1 * 100 + 0 * 50 + 0 * 20 + 29 * 10 + 22 * 5
2722: 0 * 200 + 1 * 100 + 0 * 50 + 0 * 20 + 30 * 10 + 20 * 5
2723: 0 * 200 + 1 * 100 + 0 * 50 + 0 * 20 + 31 * 10 + 18 * 5
2724: 0 * 200 + 1 * 100 + 0 * 50 + 0 * 20 + 32 * 10 + 16 * 5
2725: 0 * 200 + 1 * 100 + 0 * 50 + 0 * 20 + 33 * 10 + 14 * 5
2726: 0 * 200 + 1 * 100 + 0 * 50 + 0 * 20 + 34 * 10 + 12 * 5
2727: 0 * 200 + 1 * 100 + 0 * 50 + 0 * 20 + 35 * 10 + 10 * 5
2728: 0 * 200 + 1 * 100 + 0 * 50 + 0 * 20 + 36 * 10 + 8 * 5
2729: 0 * 200 + 1 * 100 + 0 * 50 + 0 * 20 + 37 * 10 + 6 * 5
2730: 0 * 200 + 1 * 100 + 0 * 50 + 0 * 20 + 38 * 10 + 4 * 5
2731: 0 * 200 + 1 * 100 + 0 * 50 + 0 * 20 + 39 * 10 + 2 * 5
2732: 0 * 200 + 1 * 100 + 0 * 50 + 0 * 20 + 40 * 10 + 0 * 5
2733: 0 * 200 + 1 * 100 + 0 * 50 + 1 * 20 + 0 * 10 + 76 * 5
2734: 0 * 200 + 1 * 100 + 0 * 50 + 1 * 20 + 1 * 10 + 74 * 5
2735: 0 * 200 + 1 * 100 + 0 * 50 + 1 * 20 + 2 * 10 + 72 * 5
2736: 0 * 200 + 1 * 100 + 0 * 50 + 1 * 20 + 3 * 10 + 70 * 5
2737: 0 * 200 + 1 * 100 + 0 * 50 + 1 * 20 + 4 * 10 + 68 * 5
2738: 0 * 200 + 1 * 100 + 0 * 50 + 1 * 20 + 5 * 10 + 66 * 5
2739: 0 * 200 + 1 * 100 + 0 * 50 + 1 * 20 + 6 * 10 + 64 * 5
2740: 0 * 200 + 1 * 100 + 0 * 50 + 1 * 20 + 7 * 10 + 62 * 5
2741: 0 * 200 + 1 * 100 + 0 * 50 + 1 * 20 + 8 * 10 + 60 * 5
2742: 0 * 200 + 1 * 100 + 0 * 50 + 1 * 20 + 9 * 10 + 58 * 5
2743: 0 * 200 + 1 * 100 + 0 * 50 + 1 * 20 + 10 * 10 + 56 * 5
2744: 0 * 200 + 1 * 100 + 0 * 50 + 1 * 20 + 11 * 10 + 54 * 5
2745: 0 * 200 + 1 * 100 + 0 * 50 + 1 * 20 + 12 * 10 + 52 * 5
2746: 0 * 200 + 1 * 100 + 0 * 50 + 1 * 20 + 13 * 10 + 50 * 5
2747: 0 * 200 + 1 * 100 + 0 * 50 + 1 * 20 + 14 * 10 + 48 * 5
2748: 0 * 200 + 1 * 100 + 0 * 50 + 1 * 20 + 15 * 10 + 46 * 5
2749: 0 * 200 + 1 * 100 + 0 * 50 + 1 * 20 + 16 * 10 + 44 * 5
2750: 0 * 200 + 1 * 100 + 0 * 50 + 1 * 20 + 17 * 10 + 42 * 5
2751: 0 * 200 + 1 * 100 + 0 * 50 + 1 * 20 + 18 * 10 + 40 * 5
2752: 0 * 200 + 1 * 100 + 0 * 50 + 1 * 20 + 19 * 10 + 38 * 5
2753: 0 * 200 + 1 * 100 + 0 * 50 + 1 * 20 + 20 * 10 + 36 * 5
2754: 0 * 200 + 1 * 100 + 0 * 50 + 1 * 20 + 21 * 10 + 34 * 5
2755: 0 * 200 + 1 * 100 + 0 * 50 + 1 * 20 + 22 * 10 + 32 * 5
2756: 0 * 200 + 1 * 100 + 0 * 50 + 1 * 20 + 23 * 10 + 30 * 5
2757: 0 * 200 + 1 * 100 + 0 * 50 + 1 * 20 + 24 * 10 + 28 * 5
2758: 0 * 200 + 1 * 100 + 0 * 50 + 1 * 20 + 25 * 10 + 26 * 5
2759: 0 * 200 + 1 * 100 + 0 * 50 + 1 * 20 + 26 * 10 + 24 * 5
2760: 0 * 200 + 1 * 100 + 0 * 50 + 1 * 20 + 27 * 10 + 22 * 5
2761: 0 * 200 + 1 * 100 + 0 * 50 + 1 * 20 + 28 * 10 + 20 * 5
2762: 0 * 200 + 1 * 100 + 0 * 50 + 1 * 20 + 29 * 10 + 18 * 5
2763: 0 * 200 + 1 * 100 + 0 * 50 + 1 * 20 + 30 * 10 + 16 * 5
2764: 0 * 200 + 1 * 100 + 0 * 50 + 1 * 20 + 31 * 10 + 14 * 5
2765: 0 * 200 + 1 * 100 + 0 * 50 + 1 * 20 + 32 * 10 + 12 * 5
2766: 0 * 200 + 1 * 100 + 0 * 50 + 1 * 20 + 33 * 10 + 10 * 5
2767: 0 * 200 + 1 * 100 + 0 * 50 + 1 * 20 + 34 * 10 + 8 * 5
2768: 0 * 200 + 1 * 100 + 0 * 50 + 1 * 20 + 35 * 10 + 6 * 5
2769: 0 * 200 + 1 * 100 + 0 * 50 + 1 * 20 + 36 * 10 + 4 * 5
2770: 0 * 200 + 1 * 100 + 0 * 50 + 1 * 20 + 37 * 10 + 2 * 5
2771: 0 * 200 + 1 * 100 + 0 * 50 + 1 * 20 + 38 * 10 + 0 * 5
2772: 0 * 200 + 1 * 100 + 0 * 50 + 2 * 20 + 0 * 10 + 72 * 5
2773: 0 * 200 + 1 * 100 + 0 * 50 + 2 * 20 + 1 * 10 + 70 * 5
2774: 0 * 200 + 1 * 100 + 0 * 50 + 2 * 20 + 2 * 10 + 68 * 5
2775: 0 * 200 + 1 * 100 + 0 * 50 + 2 * 20 + 3 * 10 + 66 * 5
2776: 0 * 200 + 1 * 100 + 0 * 50 + 2 * 20 + 4 * 10 + 64 * 5
2777: 0 * 200 + 1 * 100 + 0 * 50 + 2 * 20 + 5 * 10 + 62 * 5
2778: 0 * 200 + 1 * 100 + 0 * 50 + 2 * 20 + 6 * 10 + 60 * 5
2779: 0 * 200 + 1 * 100 + 0 * 50 + 2 * 20 + 7 * 10 + 58 * 5
2780: 0 * 200 + 1 * 100 + 0 * 50 + 2 * 20 + 8 * 10 + 56 * 5
2781: 0 * 200 + 1 * 100 + 0 * 50 + 2 * 20 + 9 * 10 + 54 * 5
2782: 0 * 200 + 1 * 100 + 0 * 50 + 2 * 20 + 10 * 10 + 52 * 5
2783: 0 * 200 + 1 * 100 + 0 * 50 + 2 * 20 + 11 * 10 + 50 * 5
2784: 0 * 200 + 1 * 100 + 0 * 50 + 2 * 20 + 12 * 10 + 48 * 5
2785: 0 * 200 + 1 * 100 + 0 * 50 + 2 * 20 + 13 * 10 + 46 * 5
2786: 0 * 200 + 1 * 100 + 0 * 50 + 2 * 20 + 14 * 10 + 44 * 5
2787: 0 * 200 + 1 * 100 + 0 * 50 + 2 * 20 + 15 * 10 + 42 * 5
2788: 0 * 200 + 1 * 100 + 0 * 50 + 2 * 20 + 16 * 10 + 40 * 5
2789: 0 * 200 + 1 * 100 + 0 * 50 + 2 * 20 + 17 * 10 + 38 * 5
2790: 0 * 200 + 1 * 100 + 0 * 50 + 2 * 20 + 18 * 10 + 36 * 5
2791: 0 * 200 + 1 * 100 + 0 * 50 + 2 * 20 + 19 * 10 + 34 * 5
2792: 0 * 200 + 1 * 100 + 0 * 50 + 2 * 20 + 20 * 10 + 32 * 5
2793: 0 * 200 + 1 * 100 + 0 * 50 + 2 * 20 + 21 * 10 + 30 * 5
2794: 0 * 200 + 1 * 100 + 0 * 50 + 2 * 20 + 22 * 10 + 28 * 5
2795: 0 * 200 + 1 * 100 + 0 * 50 + 2 * 20 + 23 * 10 + 26 * 5
2796: 0 * 200 + 1 * 100 + 0 * 50 + 2 * 20 + 24 * 10 + 24 * 5
2797: 0 * 200 + 1 * 100 + 0 * 50 + 2 * 20 + 25 * 10 + 22 * 5
2798: 0 * 200 + 1 * 100 + 0 * 50 + 2 * 20 + 26 * 10 + 20 * 5
2799: 0 * 200 + 1 * 100 + 0 * 50 + 2 * 20 + 27 * 10 + 18 * 5
2800: 0 * 200 + 1 * 100 + 0 * 50 + 2 * 20 + 28 * 10 + 16 * 5
2801: 0 * 200 + 1 * 100 + 0 * 50 + 2 * 20 + 29 * 10 + 14 * 5
2802: 0 * 200 + 1 * 100 + 0 * 50 + 2 * 20 + 30 * 10 + 12 * 5
2803: 0 * 200 + 1 * 100 + 0 * 50 + 2 * 20 + 31 * 10 + 10 * 5
2804: 0 * 200 + 1 * 100 + 0 * 50 + 2 * 20 + 32 * 10 + 8 * 5
2805: 0 * 200 + 1 * 100 + 0 * 50 + 2 * 20 + 33 * 10 + 6 * 5
2806: 0 * 200 + 1 * 100 + 0 * 50 + 2 * 20 + 34 * 10 + 4 * 5
2807: 0 * 200 + 1 * 100 + 0 * 50 + 2 * 20 + 35 * 10 + 2 * 5
2808: 0 * 200 + 1 * 100 + 0 * 50 + 2 * 20 + 36 * 10 + 0 * 5
2809: 0 * 200 + 1 * 100 + 0 * 50 + 3 * 20 + 0 * 10 + 68 * 5
2810: 0 * 200 + 1 * 100 + 0 * 50 + 3 * 20 + 1 * 10 + 66 * 5
2811: 0 * 200 + 1 * 100 + 0 * 50 + 3 * 20 + 2 * 10 + 64 * 5
2812: 0 * 200 + 1 * 100 + 0 * 50 + 3 * 20 + 3 * 10 + 62 * 5
2813: 0 * 200 + 1 * 100 + 0 * 50 + 3 * 20 + 4 * 10 + 60 * 5
2814: 0 * 200 + 1 * 100 + 0 * 50 + 3 * 20 + 5 * 10 + 58 * 5
2815: 0 * 200 + 1 * 100 + 0 * 50 + 3 * 20 + 6 * 10 + 56 * 5
2816: 0 * 200 + 1 * 100 + 0 * 50 + 3 * 20 + 7 * 10 + 54 * 5
2817: 0 * 200 + 1 * 100 + 0 * 50 + 3 * 20 + 8 * 10 + 52 * 5
2818: 0 * 200 + 1 * 100 + 0 * 50 + 3 * 20 + 9 * 10 + 50 * 5
2819: 0 * 200 + 1 * 100 + 0 * 50 + 3 * 20 + 10 * 10 + 48 * 5
2820: 0 * 200 + 1 * 100 + 0 * 50 + 3 * 20 + 11 * 10 + 46 * 5
2821: 0 * 200 + 1 * 100 + 0 * 50 + 3 * 20 + 12 * 10 + 44 * 5
2822: 0 * 200 + 1 * 100 + 0 * 50 + 3 * 20 + 13 * 10 + 42 * 5
2823: 0 * 200 + 1 * 100 + 0 * 50 + 3 * 20 + 14 * 10 + 40 * 5
2824: 0 * 200 + 1 * 100 + 0 * 50 + 3 * 20 + 15 * 10 + 38 * 5
2825: 0 * 200 + 1 * 100 + 0 * 50 + 3 * 20 + 16 * 10 + 36 * 5
2826: 0 * 200 + 1 * 100 + 0 * 50 + 3 * 20 + 17 * 10 + 34 * 5
2827: 0 * 200 + 1 * 100 + 0 * 50 + 3 * 20 + 18 * 10 + 32 * 5
2828: 0 * 200 + 1 * 100 + 0 * 50 + 3 * 20 + 19 * 10 + 30 * 5
2829: 0 * 200 + 1 * 100 + 0 * 50 + 3 * 20 + 20 * 10 + 28 * 5
2830: 0 * 200 + 1 * 100 + 0 * 50 + 3 * 20 + 21 * 10 + 26 * 5
2831: 0 * 200 + 1 * 100 + 0 * 50 + 3 * 20 + 22 * 10 + 24 * 5
2832: 0 * 200 + 1 * 100 + 0 * 50 + 3 * 20 + 23 * 10 + 22 * 5
2833: 0 * 200 + 1 * 100 + 0 * 50 + 3 * 20 + 24 * 10 + 20 * 5
2834: 0 * 200 + 1 * 100 + 0 * 50 + 3 * 20 + 25 * 10 + 18 * 5
2835: 0 * 200 + 1 * 100 + 0 * 50 + 3 * 20 + 26 * 10 + 16 * 5
2836: 0 * 200 + 1 * 100 + 0 * 50 + 3 * 20 + 27 * 10 + 14 * 5
2837: 0 * 200 + 1 * 100 + 0 * 50 + 3 * 20 + 28 * 10 + 12 * 5
2838: 0 * 200 + 1 * 100 + 0 * 50 + 3 * 20 + 29 * 10 + 10 * 5
2839: 0 * 200 + 1 * 100 + 0 * 50 + 3 * 20 + 30 * 10 + 8 * 5
2840: 0 * 200 + 1 * 100 + 0 * 50 + 3 * 20 + 31 * 10 + 6 * 5
2841: 0 * 200 + 1 * 100 + 0 * 50 + 3 * 20 + 32 * 10 + 4 * 5
2842: 0 * 200 + 1 * 100 + 0 * 50 + 3 * 20 + 33 * 10 + 2 * 5
2843: 0 * 200 + 1 * 100 + 0 * 50 + 3 * 20 + 34 * 10 + 0 * 5
2844: 0 * 200 + 1 * 100 + 0 * 50 + 4 * 20 + 0 * 10 + 64 * 5
2845: 0 * 200 + 1 * 100 + 0 * 50 + 4 * 20 + 1 * 10 + 62 * 5
2846: 0 * 200 + 1 * 100 + 0 * 50 + 4 * 20 + 2 * 10 + 60 * 5
2847: 0 * 200 + 1 * 100 + 0 * 50 + 4 * 20 + 3 * 10 + 58 * 5
2848: 0 * 200 + 1 * 100 + 0 * 50 + 4 * 20 + 4 * 10 + 56 * 5
2849: 0 * 200 + 1 * 100 + 0 * 50 + 4 * 20 + 5 * 10 + 54 * 5
2850: 0 * 200 + 1 * 100 + 0 * 50 + 4 * 20 + 6 * 10 + 52 * 5
2851: 0 * 200 + 1 * 100 + 0 * 50 + 4 * 20 + 7 * 10 + 50 * 5
2852: 0 * 200 + 1 * 100 + 0 * 50 + 4 * 20 + 8 * 10 + 48 * 5
2853: 0 * 200 + 1 * 100 + 0 * 50 + 4 * 20 + 9 * 10 + 46 * 5
2854: 0 * 200 + 1 * 100 + 0 * 50 + 4 * 20 + 10 * 10 + 44 * 5
2855: 0 * 200 + 1 * 100 + 0 * 50 + 4 * 20 + 11 * 10 + 42 * 5
2856: 0 * 200 + 1 * 100 + 0 * 50 + 4 * 20 + 12 * 10 + 40 * 5
2857: 0 * 200 + 1 * 100 + 0 * 50 + 4 * 20 + 13 * 10 + 38 * 5
2858: 0 * 200 + 1 * 100 + 0 * 50 + 4 * 20 + 14 * 10 + 36 * 5
2859: 0 * 200 + 1 * 100 + 0 * 50 + 4 * 20 + 15 * 10 + 34 * 5
2860: 0 * 200 + 1 * 100 + 0 * 50 + 4 * 20 + 16 * 10 + 32 * 5
2861: 0 * 200 + 1 * 100 + 0 * 50 + 4 * 20 + 17 * 10 + 30 * 5
2862: 0 * 200 + 1 * 100 + 0 * 50 + 4 * 20 + 18 * 10 + 28 * 5
2863: 0 * 200 + 1 * 100 + 0 * 50 + 4 * 20 + 19 * 10 + 26 * 5
2864: 0 * 200 + 1 * 100 + 0 * 50 + 4 * 20 + 20 * 10 + 24 * 5
2865: 0 * 200 + 1 * 100 + 0 * 50 + 4 * 20 + 21 * 10 + 22 * 5
2866: 0 * 200 + 1 * 100 + 0 * 50 + 4 * 20 + 22 * 10 + 20 * 5
2867: 0 * 200 + 1 * 100 + 0 * 50 + 4 * 20 + 23 * 10 + 18 * 5
2868: 0 * 200 + 1 * 100 + 0 * 50 + 4 * 20 + 24 * 10 + 16 * 5
2869: 0 * 200 + 1 * 100 + 0 * 50 + 4 * 20 + 25 * 10 + 14 * 5
2870: 0 * 200 + 1 * 100 + 0 * 50 + 4 * 20 + 26 * 10 + 12 * 5
2871: 0 * 200 + 1 * 100 + 0 * 50 + 4 * 20 + 27 * 10 + 10 * 5
2872: 0 * 200 + 1 * 100 + 0 * 50 + 4 * 20 + 28 * 10 + 8 * 5
2873: 0 * 200 + 1 * 100 + 0 * 50 + 4 * 20 + 29 * 10 + 6 * 5
2874: 0 * 200 + 1 * 100 + 0 * 50 + 4 * 20 + 30 * 10 + 4 * 5
2875: 0 * 200 + 1 * 100 + 0 * 50 + 4 * 20 + 31 * 10 + 2 * 5
2876: 0 * 200 + 1 * 100 + 0 * 50 + 4 * 20 + 32 * 10 + 0 * 5
2877: 0 * 200 + 1 * 100 + 0 * 50 + 5 * 20 + 0 * 10 + 60 * 5
2878: 0 * 200 + 1 * 100 + 0 * 50 + 5 * 20 + 1 * 10 + 58 * 5
2879: 0 * 200 + 1 * 100 + 0 * 50 + 5 * 20 + 2 * 10 + 56 * 5
2880: 0 * 200 + 1 * 100 + 0 * 50 + 5 * 20 + 3 * 10 + 54 * 5
2881: 0 * 200 + 1 * 100 + 0 * 50 + 5 * 20 + 4 * 10 + 52 * 5
2882: 0 * 200 + 1 * 100 + 0 * 50 + 5 * 20 + 5 * 10 + 50 * 5
2883: 0 * 200 + 1 * 100 + 0 * 50 + 5 * 20 + 6 * 10 + 48 * 5
2884: 0 * 200 + 1 * 100 + 0 * 50 + 5 * 20 + 7 * 10 + 46 * 5
2885: 0 * 200 + 1 * 100 + 0 * 50 + 5 * 20 + 8 * 10 + 44 * 5
2886: 0 * 200 + 1 * 100 + 0 * 50 + 5 * 20 + 9 * 10 + 42 * 5
2887: 0 * 200 + 1 * 100 + 0 * 50 + 5 * 20 + 10 * 10 + 40 * 5
2888: 0 * 200 + 1 * 100 + 0 * 50 + 5 * 20 + 11 * 10 + 38 * 5
2889: 0 * 200 + 1 * 100 + 0 * 50 + 5 * 20 + 12 * 10 + 36 * 5
2890: 0 * 200 + 1 * 100 + 0 * 50 + 5 * 20 + 13 * 10 + 34 * 5
2891: 0 * 200 + 1 * 100 + 0 * 50 + 5 * 20 + 14 * 10 + 32 * 5
2892: 0 * 200 + 1 * 100 + 0 * 50 + 5 * 20 + 15 * 10 + 30 * 5
2893: 0 * 200 + 1 * 100 + 0 * 50 + 5 * 20 + 16 * 10 + 28 * 5
2894: 0 * 200 + 1 * 100 + 0 * 50 + 5 * 20 + 17 * 10 + 26 * 5
2895: 0 * 200 + 1 * 100 + 0 * 50 + 5 * 20 + 18 * 10 + 24 * 5
2896: 0 * 200 + 1 * 100 + 0 * 50 + 5 * 20 + 19 * 10 + 22 * 5
2897: 0 * 200 + 1 * 100 + 0 * 50 + 5 * 20 + 20 * 10 + 20 * 5
2898: 0 * 200 + 1 * 100 + 0 * 50 + 5 * 20 + 21 * 10 + 18 * 5
2899: 0 * 200 + 1 * 100 + 0 * 50 + 5 * 20 + 22 * 10 + 16 * 5
2900: 0 * 200 + 1 * 100 + 0 * 50 + 5 * 20 + 23 * 10 + 14 * 5
2901: 0 * 200 + 1 * 100 + 0 * 50 + 5 * 20 + 24 * 10 + 12 * 5
2902: 0 * 200 + 1 * 100 + 0 * 50 + 5 * 20 + 25 * 10 + 10 * 5
2903: 0 * 200 + 1 * 100 + 0 * 50 + 5 * 20 + 26 * 10 + 8 * 5
2904: 0 * 200 + 1 * 100 + 0 * 50 + 5 * 20 + 27 * 10 + 6 * 5
2905: 0 * 200 + 1 * 100 + 0 * 50 + 5 * 20 + 28 * 10 + 4 * 5
2906: 0 * 200 + 1 * 100 + 0 * 50 + 5 * 20 + 29 * 10 + 2 * 5
2907: 0 * 200 + 1 * 100 + 0 * 50 + 5 * 20 + 30 * 10 + 0 * 5
2908: 0 * 200 + 1 * 100 + 0 * 50 + 6 * 20 + 0 * 10 + 56 * 5
2909: 0 * 200 + 1 * 100 + 0 * 50 + 6 * 20 + 1 * 10 + 54 * 5
2910: 0 * 200 + 1 * 100 + 0 * 50 + 6 * 20 + 2 * 10 + 52 * 5
2911: 0 * 200 + 1 * 100 + 0 * 50 + 6 * 20 + 3 * 10 + 50 * 5
2912: 0 * 200 + 1 * 100 + 0 * 50 + 6 * 20 + 4 * 10 + 48 * 5
2913: 0 * 200 + 1 * 100 + 0 * 50 + 6 * 20 + 5 * 10 + 46 * 5
2914: 0 * 200 + 1 * 100 + 0 * 50 + 6 * 20 + 6 * 10 + 44 * 5
2915: 0 * 200 + 1 * 100 + 0 * 50 + 6 * 20 + 7 * 10 + 42 * 5
2916: 0 * 200 + 1 * 100 + 0 * 50 + 6 * 20 + 8 * 10 + 40 * 5
2917: 0 * 200 + 1 * 100 + 0 * 50 + 6 * 20 + 9 * 10 + 38 * 5
2918: 0 * 200 + 1 * 100 + 0 * 50 + 6 * 20 + 10 * 10 + 36 * 5
2919: 0 * 200 + 1 * 100 + 0 * 50 + 6 * 20 + 11 * 10 + 34 * 5
2920: 0 * 200 + 1 * 100 + 0 * 50 + 6 * 20 + 12 * 10 + 32 * 5
2921: 0 * 200 + 1 * 100 + 0 * 50 + 6 * 20 + 13 * 10 + 30 * 5
2922: 0 * 200 + 1 * 100 + 0 * 50 + 6 * 20 + 14 * 10 + 28 * 5
2923: 0 * 200 + 1 * 100 + 0 * 50 + 6 * 20 + 15 * 10 + 26 * 5
2924: 0 * 200 + 1 * 100 + 0 * 50 + 6 * 20 + 16 * 10 + 24 * 5
2925: 0 * 200 + 1 * 100 + 0 * 50 + 6 * 20 + 17 * 10 + 22 * 5
2926: 0 * 200 + 1 * 100 + 0 * 50 + 6 * 20 + 18 * 10 + 20 * 5
2927: 0 * 200 + 1 * 100 + 0 * 50 + 6 * 20 + 19 * 10 + 18 * 5
2928: 0 * 200 + 1 * 100 + 0 * 50 + 6 * 20 + 20 * 10 + 16 * 5
2929: 0 * 200 + 1 * 100 + 0 * 50 + 6 * 20 + 21 * 10 + 14 * 5
2930: 0 * 200 + 1 * 100 + 0 * 50 + 6 * 20 + 22 * 10 + 12 * 5
2931: 0 * 200 + 1 * 100 + 0 * 50 + 6 * 20 + 23 * 10 + 10 * 5
2932: 0 * 200 + 1 * 100 + 0 * 50 + 6 * 20 + 24 * 10 + 8 * 5
2933: 0 * 200 + 1 * 100 + 0 * 50 + 6 * 20 + 25 * 10 + 6 * 5
2934: 0 * 200 + 1 * 100 + 0 * 50 + 6 * 20 + 26 * 10 + 4 * 5
2935: 0 * 200 + 1 * 100 + 0 * 50 + 6 * 20 + 27 * 10 + 2 * 5
2936: 0 * 200 + 1 * 100 + 0 * 50 + 6 * 20 + 28 * 10 + 0 * 5
2937: 0 * 200 + 1 * 100 + 0 * 50 + 7 * 20 + 0 * 10 + 52 * 5
2938: 0 * 200 + 1 * 100 + 0 * 50 + 7 * 20 + 1 * 10 + 50 * 5
2939: 0 * 200 + 1 * 100 + 0 * 50 + 7 * 20 + 2 * 10 + 48 * 5
2940: 0 * 200 + 1 * 100 + 0 * 50 + 7 * 20 + 3 * 10 + 46 * 5
2941: 0 * 200 + 1 * 100 + 0 * 50 + 7 * 20 + 4 * 10 + 44 * 5
2942: 0 * 200 + 1 * 100 + 0 * 50 + 7 * 20 + 5 * 10 + 42 * 5
2943: 0 * 200 + 1 * 100 + 0 * 50 + 7 * 20 + 6 * 10 + 40 * 5
2944: 0 * 200 + 1 * 100 + 0 * 50 + 7 * 20 + 7 * 10 + 38 * 5
2945: 0 * 200 + 1 * 100 + 0 * 50 + 7 * 20 + 8 * 10 + 36 * 5
2946: 0 * 200 + 1 * 100 + 0 * 50 + 7 * 20 + 9 * 10 + 34 * 5
2947: 0 * 200 + 1 * 100 + 0 * 50 + 7 * 20 + 10 * 10 + 32 * 5
2948: 0 * 200 + 1 * 100 + 0 * 50 + 7 * 20 + 11 * 10 + 30 * 5
2949: 0 * 200 + 1 * 100 + 0 * 50 + 7 * 20 + 12 * 10 + 28 * 5
2950: 0 * 200 + 1 * 100 + 0 * 50 + 7 * 20 + 13 * 10 + 26 * 5
2951: 0 * 200 + 1 * 100 + 0 * 50 + 7 * 20 + 14 * 10 + 24 * 5
2952: 0 * 200 + 1 * 100 + 0 * 50 + 7 * 20 + 15 * 10 + 22 * 5
2953: 0 * 200 + 1 * 100 + 0 * 50 + 7 * 20 + 16 * 10 + 20 * 5
2954: 0 * 200 + 1 * 100 + 0 * 50 + 7 * 20 + 17 * 10 + 18 * 5
2955: 0 * 200 + 1 * 100 + 0 * 50 + 7 * 20 + 18 * 10 + 16 * 5
2956: 0 * 200 + 1 * 100 + 0 * 50 + 7 * 20 + 19 * 10 + 14 * 5
2957: 0 * 200 + 1 * 100 + 0 * 50 + 7 * 20 + 20 * 10 + 12 * 5
2958: 0 * 200 + 1 * 100 + 0 * 50 + 7 * 20 + 21 * 10 + 10 * 5
2959: 0 * 200 + 1 * 100 + 0 * 50 + 7 * 20 + 22 * 10 + 8 * 5
2960: 0 * 200 + 1 * 100 + 0 * 50 + 7 * 20 + 23 * 10 + 6 * 5
2961: 0 * 200 + 1 * 100 + 0 * 50 + 7 * 20 + 24 * 10 + 4 * 5
2962: 0 * 200 + 1 * 100 + 0 * 50 + 7 * 20 + 25 * 10 + 2 * 5
2963: 0 * 200 + 1 * 100 + 0 * 50 + 7 * 20 + 26 * 10 + 0 * 5
2964: 0 * 200 + 1 * 100 + 0 * 50 + 8 * 20 + 0 * 10 + 48 * 5
2965: 0 * 200 + 1 * 100 + 0 * 50 + 8 * 20 + 1 * 10 + 46 * 5
2966: 0 * 200 + 1 * 100 + 0 * 50 + 8 * 20 + 2 * 10 + 44 * 5
2967: 0 * 200 + 1 * 100 + 0 * 50 + 8 * 20 + 3 * 10 + 42 * 5
2968: 0 * 200 + 1 * 100 + 0 * 50 + 8 * 20 + 4 * 10 + 40 * 5
2969: 0 * 200 + 1 * 100 + 0 * 50 + 8 * 20 + 5 * 10 + 38 * 5
2970: 0 * 200 + 1 * 100 + 0 * 50 + 8 * 20 + 6 * 10 + 36 * 5
2971: 0 * 200 + 1 * 100 + 0 * 50 + 8 * 20 + 7 * 10 + 34 * 5
2972: 0 * 200 + 1 * 100 + 0 * 50 + 8 * 20 + 8 * 10 + 32 * 5
2973: 0 * 200 + 1 * 100 + 0 * 50 + 8 * 20 + 9 * 10 + 30 * 5
2974: 0 * 200 + 1 * 100 + 0 * 50 + 8 * 20 + 10 * 10 + 28 * 5
2975: 0 * 200 + 1 * 100 + 0 * 50 + 8 * 20 + 11 * 10 + 26 * 5
2976: 0 * 200 + 1 * 100 + 0 * 50 + 8 * 20 + 12 * 10 + 24 * 5
2977: 0 * 200 + 1 * 100 + 0 * 50 + 8 * 20 + 13 * 10 + 22 * 5
2978: 0 * 200 + 1 * 100 + 0 * 50 + 8 * 20 + 14 * 10 + 20 * 5
2979: 0 * 200 + 1 * 100 + 0 * 50 + 8 * 20 + 15 * 10 + 18 * 5
2980: 0 * 200 + 1 * 100 + 0 * 50 + 8 * 20 + 16 * 10 + 16 * 5
2981: 0 * 200 + 1 * 100 + 0 * 50 + 8 * 20 + 17 * 10 + 14 * 5
2982: 0 * 200 + 1 * 100 + 0 * 50 + 8 * 20 + 18 * 10 + 12 * 5
2983: 0 * 200 + 1 * 100 + 0 * 50 + 8 * 20 + 19 * 10 + 10 * 5
2984: 0 * 200 + 1 * 100 + 0 * 50 + 8 * 20 + 20 * 10 + 8 * 5
2985: 0 * 200 + 1 * 100 + 0 * 50 + 8 * 20 + 21 * 10 + 6 * 5
2986: 0 * 200 + 1 * 100 + 0 * 50 + 8 * 20 + 22 * 10 + 4 * 5
2987: 0 * 200 + 1 * 100 + 0 * 50 + 8 * 20 + 23 * 10 + 2 * 5
2988: 0 * 200 + 1 * 100 + 0 * 50 + 8 * 20 + 24 * 10 + 0 * 5
2989: 0 * 200 + 1 * 100 + 0 * 50 + 9 * 20 + 0 * 10 + 44 * 5
2990: 0 * 200 + 1 * 100 + 0 * 50 + 9 * 20 + 1 * 10 + 42 * 5
2991: 0 * 200 + 1 * 100 + 0 * 50 + 9 * 20 + 2 * 10 + 40 * 5
2992: 0 * 200 + 1 * 100 + 0 * 50 + 9 * 20 + 3 * 10 + 38 * 5
2993: 0 * 200 + 1 * 100 + 0 * 50 + 9 * 20 + 4 * 10 + 36 * 5
2994: 0 * 200 + 1 * 100 + 0 * 50 + 9 * 20 + 5 * 10 + 34 * 5
2995: 0 * 200 + 1 * 100 + 0 * 50 + 9 * 20 + 6 * 10 + 32 * 5
2996: 0 * 200 + 1 * 100 + 0 * 50 + 9 * 20 + 7 * 10 + 30 * 5
2997: 0 * 200 + 1 * 100 + 0 * 50 + 9 * 20 + 8 * 10 + 28 * 5
2998: 0 * 200 + 1 * 100 + 0 * 50 + 9 * 20 + 9 * 10 + 26 * 5
2999: 0 * 200 + 1 * 100 + 0 * 50 + 9 * 20 + 10 * 10 + 24 * 5
3000: 0 * 200 + 1 * 100 + 0 * 50 + 9 * 20 + 11 * 10 + 22 * 5
3001: 0 * 200 + 1 * 100 + 0 * 50 + 9 * 20 + 12 * 10 + 20 * 5
3002: 0 * 200 + 1 * 100 + 0 * 50 + 9 * 20 + 13 * 10 + 18 * 5
3003: 0 * 200 + 1 * 100 + 0 * 50 + 9 * 20 + 14 * 10 + 16 * 5
3004: 0 * 200 + 1 * 100 + 0 * 50 + 9 * 20 + 15 * 10 + 14 * 5
3005: 0 * 200 + 1 * 100 + 0 * 50 + 9 * 20 + 16 * 10 + 12 * 5
3006: 0 * 200 + 1 * 100 + 0 * 50 + 9 * 20 + 17 * 10 + 10 * 5
3007: 0 * 200 + 1 * 100 + 0 * 50 + 9 * 20 + 18 * 10 + 8 * 5
3008: 0 * 200 + 1 * 100 + 0 * 50 + 9 * 20 + 19 * 10 + 6 * 5
3009: 0 * 200 + 1 * 100 + 0 * 50 + 9 * 20 + 20 * 10 + 4 * 5
3010: 0 * 200 + 1 * 100 + 0 * 50 + 9 * 20 + 21 * 10 + 2 * 5
3011: 0 * 200 + 1 * 100 + 0 * 50 + 9 * 20 + 22 * 10 + 0 * 5
3012: 0 * 200 + 1 * 100 + 0 * 50 + 10 * 20 + 0 * 10 + 40 * 5
3013: 0 * 200 + 1 * 100 + 0 * 50 + 10 * 20 + 1 * 10 + 38 * 5
3014: 0 * 200 + 1 * 100 + 0 * 50 + 10 * 20 + 2 * 10 + 36 * 5
3015: 0 * 200 + 1 * 100 + 0 * 50 + 10 * 20 + 3 * 10 + 34 * 5
3016: 0 * 200 + 1 * 100 + 0 * 50 + 10 * 20 + 4 * 10 + 32 * 5
3017: 0 * 200 + 1 * 100 + 0 * 50 + 10 * 20 + 5 * 10 + 30 * 5
3018: 0 * 200 + 1 * 100 + 0 * 50 + 10 * 20 + 6 * 10 + 28 * 5
3019: 0 * 200 + 1 * 100 + 0 * 50 + 10 * 20 + 7 * 10 + 26 * 5
3020: 0 * 200 + 1 * 100 + 0 * 50 + 10 * 20 + 8 * 10 + 24 * 5
3021: 0 * 200 + 1 * 100 + 0 * 50 + 10 * 20 + 9 * 10 + 22 * 5
3022: 0 * 200 + 1 * 100 + 0 * 50 + 10 * 20 + 10 * 10 + 20 * 5
3023: 0 * 200 + 1 * 100 + 0 * 50 + 10 * 20 + 11 * 10 + 18 * 5
3024: 0 * 200 + 1 * 100 + 0 * 50 + 10 * 20 + 12 * 10 + 16 * 5
3025: 0 * 200 + 1 * 100 + 0 * 50 + 10 * 20 + 13 * 10 + 14 * 5
3026: 0 * 200 + 1 * 100 + 0 * 50 + 10 * 20 + 14 * 10 + 12 * 5
3027: 0 * 200 + 1 * 100 + 0 * 50 + 10 * 20 + 15 * 10 + 10 * 5
3028: 0 * 200 + 1 * 100 + 0 * 50 + 10 * 20 + 16 * 10 + 8 * 5
3029: 0 * 200 + 1 * 100 + 0 * 50 + 10 * 20 + 17 * 10 + 6 * 5
3030: 0 * 200 + 1 * 100 + 0 * 50 + 10 * 20 + 18 * 10 + 4 * 5
3031: 0 * 200 + 1 * 100 + 0 * 50 + 10 * 20 + 19 * 10 + 2 * 5
3032: 0 * 200 + 1 * 100 + 0 * 50 + 10 * 20 + 20 * 10 + 0 * 5
3033: 0 * 200 + 1 * 100 + 0 * 50 + 11 * 20 + 0 * 10 + 36 * 5
3034: 0 * 200 + 1 * 100 + 0 * 50 + 11 * 20 + 1 * 10 + 34 * 5
3035: 0 * 200 + 1 * 100 + 0 * 50 + 11 * 20 + 2 * 10 + 32 * 5
3036: 0 * 200 + 1 * 100 + 0 * 50 + 11 * 20 + 3 * 10 + 30 * 5
3037: 0 * 200 + 1 * 100 + 0 * 50 + 11 * 20 + 4 * 10 + 28 * 5
3038: 0 * 200 + 1 * 100 + 0 * 50 + 11 * 20 + 5 * 10 + 26 * 5
3039: 0 * 200 + 1 * 100 + 0 * 50 + 11 * 20 + 6 * 10 + 24 * 5
3040: 0 * 200 + 1 * 100 + 0 * 50 + 11 * 20 + 7 * 10 + 22 * 5
3041: 0 * 200 + 1 * 100 + 0 * 50 + 11 * 20 + 8 * 10 + 20 * 5
3042: 0 * 200 + 1 * 100 + 0 * 50 + 11 * 20 + 9 * 10 + 18 * 5
3043: 0 * 200 + 1 * 100 + 0 * 50 + 11 * 20 + 10 * 10 + 16 * 5
3044: 0 * 200 + 1 * 100 + 0 * 50 + 11 * 20 + 11 * 10 + 14 * 5
3045: 0 * 200 + 1 * 100 + 0 * 50 + 11 * 20 + 12 * 10 + 12 * 5
3046: 0 * 200 + 1 * 100 + 0 * 50 + 11 * 20 + 13 * 10 + 10 * 5
3047: 0 * 200 + 1 * 100 + 0 * 50 + 11 * 20 + 14 * 10 + 8 * 5
3048: 0 * 200 + 1 * 100 + 0 * 50 + 11 * 20 + 15 * 10 + 6 * 5
3049: 0 * 200 + 1 * 100 + 0 * 50 + 11 * 20 + 16 * 10 + 4 * 5
3050: 0 * 200 + 1 * 100 + 0 * 50 + 11 * 20 + 17 * 10 + 2 * 5
3051: 0 * 200 + 1 * 100 + 0 * 50 + 11 * 20 + 18 * 10 + 0 * 5
3052: 0 * 200 + 1 * 100 + 0 * 50 + 12 * 20 + 0 * 10 + 32 * 5
3053: 0 * 200 + 1 * 100 + 0 * 50 + 12 * 20 + 1 * 10 + 30 * 5
3054: 0 * 200 + 1 * 100 + 0 * 50 + 12 * 20 + 2 * 10 + 28 * 5
3055: 0 * 200 + 1 * 100 + 0 * 50 + 12 * 20 + 3 * 10 + 26 * 5
3056: 0 * 200 + 1 * 100 + 0 * 50 + 12 * 20 + 4 * 10 + 24 * 5
3057: 0 * 200 + 1 * 100 + 0 * 50 + 12 * 20 + 5 * 10 + 22 * 5
3058: 0 * 200 + 1 * 100 + 0 * 50 + 12 * 20 + 6 * 10 + 20 * 5
3059: 0 * 200 + 1 * 100 + 0 * 50 + 12 * 20 + 7 * 10 + 18 * 5
3060: 0 * 200 + 1 * 100 + 0 * 50 + 12 * 20 + 8 * 10 + 16 * 5
3061: 0 * 200 + 1 * 100 + 0 * 50 + 12 * 20 + 9 * 10 + 14 * 5
3062: 0 * 200 + 1 * 100 + 0 * 50 + 12 * 20 + 10 * 10 + 12 * 5
3063: 0 * 200 + 1 * 100 + 0 * 50 + 12 * 20 + 11 * 10 + 10 * 5
3064: 0 * 200 + 1 * 100 + 0 * 50 + 12 * 20 + 12 * 10 + 8 * 5
3065: 0 * 200 + 1 * 100 + 0 * 50 + 12 * 20 + 13 * 10 + 6 * 5
3066: 0 * 200 + 1 * 100 + 0 * 50 + 12 * 20 + 14 * 10 + 4 * 5
3067: 0 * 200 + 1 * 100 + 0 * 50 + 12 * 20 + 15 * 10 + 2 * 5
3068: 0 * 200 + 1 * 100 + 0 * 50 + 12 * 20 + 16 * 10 + 0 * 5
3069: 0 * 200 + 1 * 100 + 0 * 50 + 13 * 20 + 0 * 10 + 28 * 5
3070: 0 * 200 + 1 * 100 + 0 * 50 + 13 * 20 + 1 * 10 + 26 * 5
3071: 0 * 200 + 1 * 100 + 0 * 50 + 13 * 20 + 2 * 10 + 24 * 5
3072: 0 * 200 + 1 * 100 + 0 * 50 + 13 * 20 + 3 * 10 + 22 * 5
3073: 0 * 200 + 1 * 100 + 0 * 50 + 13 * 20 + 4 * 10 + 20 * 5
3074: 0 * 200 + 1 * 100 + 0 * 50 + 13 * 20 + 5 * 10 + 18 * 5
3075: 0 * 200 + 1 * 100 + 0 * 50 + 13 * 20 + 6 * 10 + 16 * 5
3076: 0 * 200 + 1 * 100 + 0 * 50 + 13 * 20 + 7 * 10 + 14 * 5
3077: 0 * 200 + 1 * 100 + 0 * 50 + 13 * 20 + 8 * 10 + 12 * 5
3078: 0 * 200 + 1 * 100 + 0 * 50 + 13 * 20 + 9 * 10 + 10 * 5
3079: 0 * 200 + 1 * 100 + 0 * 50 + 13 * 20 + 10 * 10 + 8 * 5
3080: 0 * 200 + 1 * 100 + 0 * 50 + 13 * 20 + 11 * 10 + 6 * 5
3081: 0 * 200 + 1 * 100 + 0 * 50 + 13 * 20 + 12 * 10 + 4 * 5
3082: 0 * 200 + 1 * 100 + 0 * 50 + 13 * 20 + 13 * 10 + 2 * 5
3083: 0 * 200 + 1 * 100 + 0 * 50 + 13 * 20 + 14 * 10 + 0 * 5
3084: 0 * 200 + 1 * 100 + 0 * 50 + 14 * 20 + 0 * 10 + 24 * 5
3085: 0 * 200 + 1 * 100 + 0 * 50 + 14 * 20 + 1 * 10 + 22 * 5
3086: 0 * 200 + 1 * 100 + 0 * 50 + 14 * 20 + 2 * 10 + 20 * 5
3087: 0 * 200 + 1 * 100 + 0 * 50 + 14 * 20 + 3 * 10 + 18 * 5
3088: 0 * 200 + 1 * 100 + 0 * 50 + 14 * 20 + 4 * 10 + 16 * 5
3089: 0 * 200 + 1 * 100 + 0 * 50 + 14 * 20 + 5 * 10 + 14 * 5
3090: 0 * 200 + 1 * 100 + 0 * 50 + 14 * 20 + 6 * 10 + 12 * 5
3091: 0 * 200 + 1 * 100 + 0 * 50 + 14 * 20 + 7 * 10 + 10 * 5
3092: 0 * 200 + 1 * 100 + 0 * 50 + 14 * 20 + 8 * 10 + 8 * 5
3093: 0 * 200 + 1 * 100 + 0 * 50 + 14 * 20 + 9 * 10 + 6 * 5
3094: 0 * 200 + 1 * 100 + 0 * 50 + 14 * 20 + 10 * 10 + 4 * 5
3095: 0 * 200 + 1 * 100 + 0 * 50 + 14 * 20 + 11 * 10 + 2 * 5
3096: 0 * 200 + 1 * 100 + 0 * 50 + 14 * 20 + 12 * 10 + 0 * 5
3097: 0 * 200 + 1 * 100 + 0 * 50 + 15 * 20 + 0 * 10 + 20 * 5
3098: 0 * 200 + 1 * 100 + 0 * 50 + 15 * 20 + 1 * 10 + 18 * 5
3099: 0 * 200 + 1 * 100 + 0 * 50 + 15 * 20 + 2 * 10 + 16 * 5
3100: 0 * 200 + 1 * 100 + 0 * 50 + 15 * 20 + 3 * 10 + 14 * 5
3101: 0 * 200 + 1 * 100 + 0 * 50 + 15 * 20 + 4 * 10 + 12 * 5
3102: 0 * 200 + 1 * 100 + 0 * 50 + 15 * 20 + 5 * 10 + 10 * 5
3103: 0 * 200 + 1 * 100 + 0 * 50 + 15 * 20 + 6 * 10 + 8 * 5
3104: 0 * 200 + 1 * 100 + 0 * 50 + 15 * 20 + 7 * 10 + 6 * 5
3105: 0 * 200 + 1 * 100 + 0 * 50 + 15 * 20 + 8 * 10 + 4 * 5
3106: 0 * 200 + 1 * 100 + 0 * 50 + 15 * 20 + 9 * 10 + 2 * 5
3107: 0 * 200 + 1 * 100 + 0 * 50 + 15 * 20 + 10 * 10 + 0 * 5
3108: 0 * 200 + 1 * 100 + 0 * 50 + 16 * 20 + 0 * 10 + 16 * 5
3109: 0 * 200 + 1 * 100 + 0 * 50 + 16 * 20 + 1 * 10 + 14 * 5
3110: 0 * 200 + 1 * 100 + 0 * 50 + 16 * 20 + 2 * 10 + 12 * 5
3111: 0 * 200 + 1 * 100 + 0 * 50 + 16 * 20 + 3 * 10 + 10 * 5
3112: 0 * 200 + 1 * 100 + 0 * 50 + 16 * 20 + 4 * 10 + 8 * 5
3113: 0 * 200 + 1 * 100 + 0 * 50 + 16 * 20 + 5 * 10 + 6 * 5
3114: 0 * 200 + 1 * 100 + 0 * 50 + 16 * 20 + 6 * 10 + 4 * 5
3115: 0 * 200 + 1 * 100 + 0 * 50 + 16 * 20 + 7 * 10 + 2 * 5
3116: 0 * 200 + 1 * 100 + 0 * 50 + 16 * 20 + 8 * 10 + 0 * 5
3117: 0 * 200 + 1 * 100 + 0 * 50 + 17 * 20 + 0 * 10 + 12 * 5
3118: 0 * 200 + 1 * 100 + 0 * 50 + 17 * 20 + 1 * 10 + 10 * 5
3119: 0 * 200 + 1 * 100 + 0 * 50 + 17 * 20 + 2 * 10 + 8 * 5
3120: 0 * 200 + 1 * 100 + 0 * 50 + 17 * 20 + 3 * 10 + 6 * 5
3121: 0 * 200 + 1 * 100 + 0 * 50 + 17 * 20 + 4 * 10 + 4 * 5
3122: 0 * 200 + 1 * 100 + 0 * 50 + 17 * 20 + 5 * 10 + 2 * 5
3123: 0 * 200 + 1 * 100 + 0 * 50 + 17 * 20 + 6 * 10 + 0 * 5
3124: 0 * 200 + 1 * 100 + 0 * 50 + 18 * 20 + 0 * 10 + 8 * 5
3125: 0 * 200 + 1 * 100 + 0 * 50 + 18 * 20 + 1 * 10 + 6 * 5
3126: 0 * 200 + 1 * 100 + 0 * 50 + 18 * 20 + 2 * 10 + 4 * 5
3127: 0 * 200 + 1 * 100 + 0 * 50 + 18 * 20 + 3 * 10 + 2 * 5
3128: 0 * 200 + 1 * 100 + 0 * 50 + 18 * 20 + 4 * 10 + 0 * 5
3129: 0 * 200 + 1 * 100 + 0 * 50 + 19 * 20 + 0 * 10 + 4 * 5
3130: 0 * 200 + 1 * 100 + 0 * 50 + 19 * 20 + 1 * 10 + 2 * 5
3131: 0 * 200 + 1 * 100 + 0 * 50 + 19 * 20 + 2 * 10 + 0 * 5
3132: 0 * 200 + 1 * 100 + 0 * 50 + 20 * 20 + 0 * 10 + 0 * 5
3133: 0 * 200 + 1 * 100 + 1 * 50 + 0 * 20 + 0 * 10 + 70 * 5
3134: 0 * 200 + 1 * 100 + 1 * 50 + 0 * 20 + 1 * 10 + 68 * 5
3135: 0 * 200 + 1 * 100 + 1 * 50 + 0 * 20 + 2 * 10 + 66 * 5
3136: 0 * 200 + 1 * 100 + 1 * 50 + 0 * 20 + 3 * 10 + 64 * 5
3137: 0 * 200 + 1 * 100 + 1 * 50 + 0 * 20 + 4 * 10 + 62 * 5
3138: 0 * 200 + 1 * 100 + 1 * 50 + 0 * 20 + 5 * 10 + 60 * 5
3139: 0 * 200 + 1 * 100 + 1 * 50 + 0 * 20 + 6 * 10 + 58 * 5
3140: 0 * 200 + 1 * 100 + 1 * 50 + 0 * 20 + 7 * 10 + 56 * 5
3141: 0 * 200 + 1 * 100 + 1 * 50 + 0 * 20 + 8 * 10 + 54 * 5
3142: 0 * 200 + 1 * 100 + 1 * 50 + 0 * 20 + 9 * 10 + 52 * 5
3143: 0 * 200 + 1 * 100 + 1 * 50 + 0 * 20 + 10 * 10 + 50 * 5
3144: 0 * 200 + 1 * 100 + 1 * 50 + 0 * 20 + 11 * 10 + 48 * 5
3145: 0 * 200 + 1 * 100 + 1 * 50 + 0 * 20 + 12 * 10 + 46 * 5
3146: 0 * 200 + 1 * 100 + 1 * 50 + 0 * 20 + 13 * 10 + 44 * 5
3147: 0 * 200 + 1 * 100 + 1 * 50 + 0 * 20 + 14 * 10 + 42 * 5
3148: 0 * 200 + 1 * 100 + 1 * 50 + 0 * 20 + 15 * 10 + 40 * 5
3149: 0 * 200 + 1 * 100 + 1 * 50 + 0 * 20 + 16 * 10 + 38 * 5
3150: 0 * 200 + 1 * 100 + 1 * 50 + 0 * 20 + 17 * 10 + 36 * 5
3151: 0 * 200 + 1 * 100 + 1 * 50 + 0 * 20 + 18 * 10 + 34 * 5
3152: 0 * 200 + 1 * 100 + 1 * 50 + 0 * 20 + 19 * 10 + 32 * 5
3153: 0 * 200 + 1 * 100 + 1 * 50 + 0 * 20 + 20 * 10 + 30 * 5
3154: 0 * 200 + 1 * 100 + 1 * 50 + 0 * 20 + 21 * 10 + 28 * 5
3155: 0 * 200 + 1 * 100 + 1 * 50 + 0 * 20 + 22 * 10 + 26 * 5
3156: 0 * 200 + 1 * 100 + 1 * 50 + 0 * 20 + 23 * 10 + 24 * 5
3157: 0 * 200 + 1 * 100 + 1 * 50 + 0 * 20 + 24 * 10 + 22 * 5
3158: 0 * 200 + 1 * 100 + 1 * 50 + 0 * 20 + 25 * 10 + 20 * 5
3159: 0 * 200 + 1 * 100 + 1 * 50 + 0 * 20 + 26 * 10 + 18 * 5
3160: 0 * 200 + 1 * 100 + 1 * 50 + 0 * 20 + 27 * 10 + 16 * 5
3161: 0 * 200 + 1 * 100 + 1 * 50 + 0 * 20 + 28 * 10 + 14 * 5
3162: 0 * 200 + 1 * 100 + 1 * 50 + 0 * 20 + 29 * 10 + 12 * 5
3163: 0 * 200 + 1 * 100 + 1 * 50 + 0 * 20 + 30 * 10 + 10 * 5
3164: 0 * 200 + 1 * 100 + 1 * 50 + 0 * 20 + 31 * 10 + 8 * 5
3165: 0 * 200 + 1 * 100 + 1 * 50 + 0 * 20 + 32 * 10 + 6 * 5
3166: 0 * 200 + 1 * 100 + 1 * 50 + 0 * 20 + 33 * 10 + 4 * 5
3167: 0 * 200 + 1 * 100 + 1 * 50 + 0 * 20 + 34 * 10 + 2 * 5
3168: 0 * 200 + 1 * 100 + 1 * 50 + 0 * 20 + 35 * 10 + 0 * 5
3169: 0 * 200 + 1 * 100 + 1 * 50 + 1 * 20 + 0 * 10 + 66 * 5
3170: 0 * 200 + 1 * 100 + 1 * 50 + 1 * 20 + 1 * 10 + 64 * 5
3171: 0 * 200 + 1 * 100 + 1 * 50 + 1 * 20 + 2 * 10 + 62 * 5
3172: 0 * 200 + 1 * 100 + 1 * 50 + 1 * 20 + 3 * 10 + 60 * 5
3173: 0 * 200 + 1 * 100 + 1 * 50 + 1 * 20 + 4 * 10 + 58 * 5
3174: 0 * 200 + 1 * 100 + 1 * 50 + 1 * 20 + 5 * 10 + 56 * 5
3175: 0 * 200 + 1 * 100 + 1 * 50 + 1 * 20 + 6 * 10 + 54 * 5
3176: 0 * 200 + 1 * 100 + 1 * 50 + 1 * 20 + 7 * 10 + 52 * 5
3177: 0 * 200 + 1 * 100 + 1 * 50 + 1 * 20 + 8 * 10 + 50 * 5
3178: 0 * 200 + 1 * 100 + 1 * 50 + 1 * 20 + 9 * 10 + 48 * 5
3179: 0 * 200 + 1 * 100 + 1 * 50 + 1 * 20 + 10 * 10 + 46 * 5
3180: 0 * 200 + 1 * 100 + 1 * 50 + 1 * 20 + 11 * 10 + 44 * 5
3181: 0 * 200 + 1 * 100 + 1 * 50 + 1 * 20 + 12 * 10 + 42 * 5
3182: 0 * 200 + 1 * 100 + 1 * 50 + 1 * 20 + 13 * 10 + 40 * 5
3183: 0 * 200 + 1 * 100 + 1 * 50 + 1 * 20 + 14 * 10 + 38 * 5
3184: 0 * 200 + 1 * 100 + 1 * 50 + 1 * 20 + 15 * 10 + 36 * 5
3185: 0 * 200 + 1 * 100 + 1 * 50 + 1 * 20 + 16 * 10 + 34 * 5
3186: 0 * 200 + 1 * 100 + 1 * 50 + 1 * 20 + 17 * 10 + 32 * 5
3187: 0 * 200 + 1 * 100 + 1 * 50 + 1 * 20 + 18 * 10 + 30 * 5
3188: 0 * 200 + 1 * 100 + 1 * 50 + 1 * 20 + 19 * 10 + 28 * 5
3189: 0 * 200 + 1 * 100 + 1 * 50 + 1 * 20 + 20 * 10 + 26 * 5
3190: 0 * 200 + 1 * 100 + 1 * 50 + 1 * 20 + 21 * 10 + 24 * 5
3191: 0 * 200 + 1 * 100 + 1 * 50 + 1 * 20 + 22 * 10 + 22 * 5
3192: 0 * 200 + 1 * 100 + 1 * 50 + 1 * 20 + 23 * 10 + 20 * 5
3193: 0 * 200 + 1 * 100 + 1 * 50 + 1 * 20 + 24 * 10 + 18 * 5
3194: 0 * 200 + 1 * 100 + 1 * 50 + 1 * 20 + 25 * 10 + 16 * 5
3195: 0 * 200 + 1 * 100 + 1 * 50 + 1 * 20 + 26 * 10 + 14 * 5
3196: 0 * 200 + 1 * 100 + 1 * 50 + 1 * 20 + 27 * 10 + 12 * 5
3197: 0 * 200 + 1 * 100 + 1 * 50 + 1 * 20 + 28 * 10 + 10 * 5
3198: 0 * 200 + 1 * 100 + 1 * 50 + 1 * 20 + 29 * 10 + 8 * 5
3199: 0 * 200 + 1 * 100 + 1 * 50 + 1 * 20 + 30 * 10 + 6 * 5
3200: 0 * 200 + 1 * 100 + 1 * 50 + 1 * 20 + 31 * 10 + 4 * 5
3201: 0 * 200 + 1 * 100 + 1 * 50 + 1 * 20 + 32 * 10 + 2 * 5
3202: 0 * 200 + 1 * 100 + 1 * 50 + 1 * 20 + 33 * 10 + 0 * 5
3203: 0 * 200 + 1 * 100 + 1 * 50 + 2 * 20 + 0 * 10 + 62 * 5
3204: 0 * 200 + 1 * 100 + 1 * 50 + 2 * 20 + 1 * 10 + 60 * 5
3205: 0 * 200 + 1 * 100 + 1 * 50 + 2 * 20 + 2 * 10 + 58 * 5
3206: 0 * 200 + 1 * 100 + 1 * 50 + 2 * 20 + 3 * 10 + 56 * 5
3207: 0 * 200 + 1 * 100 + 1 * 50 + 2 * 20 + 4 * 10 + 54 * 5
3208: 0 * 200 + 1 * 100 + 1 * 50 + 2 * 20 + 5 * 10 + 52 * 5
3209: 0 * 200 + 1 * 100 + 1 * 50 + 2 * 20 + 6 * 10 + 50 * 5
3210: 0 * 200 + 1 * 100 + 1 * 50 + 2 * 20 + 7 * 10 + 48 * 5
3211: 0 * 200 + 1 * 100 + 1 * 50 + 2 * 20 + 8 * 10 + 46 * 5
3212: 0 * 200 + 1 * 100 + 1 * 50 + 2 * 20 + 9 * 10 + 44 * 5
3213: 0 * 200 + 1 * 100 + 1 * 50 + 2 * 20 + 10 * 10 + 42 * 5
3214: 0 * 200 + 1 * 100 + 1 * 50 + 2 * 20 + 11 * 10 + 40 * 5
3215: 0 * 200 + 1 * 100 + 1 * 50 + 2 * 20 + 12 * 10 + 38 * 5
3216: 0 * 200 + 1 * 100 + 1 * 50 + 2 * 20 + 13 * 10 + 36 * 5
3217: 0 * 200 + 1 * 100 + 1 * 50 + 2 * 20 + 14 * 10 + 34 * 5
3218: 0 * 200 + 1 * 100 + 1 * 50 + 2 * 20 + 15 * 10 + 32 * 5
3219: 0 * 200 + 1 * 100 + 1 * 50 + 2 * 20 + 16 * 10 + 30 * 5
3220: 0 * 200 + 1 * 100 + 1 * 50 + 2 * 20 + 17 * 10 + 28 * 5
3221: 0 * 200 + 1 * 100 + 1 * 50 + 2 * 20 + 18 * 10 + 26 * 5
3222: 0 * 200 + 1 * 100 + 1 * 50 + 2 * 20 + 19 * 10 + 24 * 5
3223: 0 * 200 + 1 * 100 + 1 * 50 + 2 * 20 + 20 * 10 + 22 * 5
3224: 0 * 200 + 1 * 100 + 1 * 50 + 2 * 20 + 21 * 10 + 20 * 5
3225: 0 * 200 + 1 * 100 + 1 * 50 + 2 * 20 + 22 * 10 + 18 * 5
3226: 0 * 200 + 1 * 100 + 1 * 50 + 2 * 20 + 23 * 10 + 16 * 5
3227: 0 * 200 + 1 * 100 + 1 * 50 + 2 * 20 + 24 * 10 + 14 * 5
3228: 0 * 200 + 1 * 100 + 1 * 50 + 2 * 20 + 25 * 10 + 12 * 5
3229: 0 * 200 + 1 * 100 + 1 * 50 + 2 * 20 + 26 * 10 + 10 * 5
3230: 0 * 200 + 1 * 100 + 1 * 50 + 2 * 20 + 27 * 10 + 8 * 5
3231: 0 * 200 + 1 * 100 + 1 * 50 + 2 * 20 + 28 * 10 + 6 * 5
3232: 0 * 200 + 1 * 100 + 1 * 50 + 2 * 20 + 29 * 10 + 4 * 5
3233: 0 * 200 + 1 * 100 + 1 * 50 + 2 * 20 + 30 * 10 + 2 * 5
3234: 0 * 200 + 1 * 100 + 1 * 50 + 2 * 20 + 31 * 10 + 0 * 5
3235: 0 * 200 + 1 * 100 + 1 * 50 + 3 * 20 + 0 * 10 + 58 * 5
3236: 0 * 200 + 1 * 100 + 1 * 50 + 3 * 20 + 1 * 10 + 56 * 5
3237: 0 * 200 + 1 * 100 + 1 * 50 + 3 * 20 + 2 * 10 + 54 * 5
3238: 0 * 200 + 1 * 100 + 1 * 50 + 3 * 20 + 3 * 10 + 52 * 5
3239: 0 * 200 + 1 * 100 + 1 * 50 + 3 * 20 + 4 * 10 + 50 * 5
3240: 0 * 200 + 1 * 100 + 1 * 50 + 3 * 20 + 5 * 10 + 48 * 5
3241: 0 * 200 + 1 * 100 + 1 * 50 + 3 * 20 + 6 * 10 + 46 * 5
3242: 0 * 200 + 1 * 100 + 1 * 50 + 3 * 20 + 7 * 10 + 44 * 5
3243: 0 * 200 + 1 * 100 + 1 * 50 + 3 * 20 + 8 * 10 + 42 * 5
3244: 0 * 200 + 1 * 100 + 1 * 50 + 3 * 20 + 9 * 10 + 40 * 5
3245: 0 * 200 + 1 * 100 + 1 * 50 + 3 * 20 + 10 * 10 + 38 * 5
3246: 0 * 200 + 1 * 100 + 1 * 50 + 3 * 20 + 11 * 10 + 36 * 5
3247: 0 * 200 + 1 * 100 + 1 * 50 + 3 * 20 + 12 * 10 + 34 * 5
3248: 0 * 200 + 1 * 100 + 1 * 50 + 3 * 20 + 13 * 10 + 32 * 5
3249: 0 * 200 + 1 * 100 + 1 * 50 + 3 * 20 + 14 * 10 + 30 * 5
3250: 0 * 200 + 1 * 100 + 1 * 50 + 3 * 20 + 15 * 10 + 28 * 5
3251: 0 * 200 + 1 * 100 + 1 * 50 + 3 * 20 + 16 * 10 + 26 * 5
3252: 0 * 200 + 1 * 100 + 1 * 50 + 3 * 20 + 17 * 10 + 24 * 5
3253: 0 * 200 + 1 * 100 + 1 * 50 + 3 * 20 + 18 * 10 + 22 * 5
3254: 0 * 200 + 1 * 100 + 1 * 50 + 3 * 20 + 19 * 10 + 20 * 5
3255: 0 * 200 + 1 * 100 + 1 * 50 + 3 * 20 + 20 * 10 + 18 * 5
3256: 0 * 200 + 1 * 100 + 1 * 50 + 3 * 20 + 21 * 10 + 16 * 5
3257: 0 * 200 + 1 * 100 + 1 * 50 + 3 * 20 + 22 * 10 + 14 * 5
3258: 0 * 200 + 1 * 100 + 1 * 50 + 3 * 20 + 23 * 10 + 12 * 5
3259: 0 * 200 + 1 * 100 + 1 * 50 + 3 * 20 + 24 * 10 + 10 * 5
3260: 0 * 200 + 1 * 100 + 1 * 50 + 3 * 20 + 25 * 10 + 8 * 5
3261: 0 * 200 + 1 * 100 + 1 * 50 + 3 * 20 + 26 * 10 + 6 * 5
3262: 0 * 200 + 1 * 100 + 1 * 50 + 3 * 20 + 27 * 10 + 4 * 5
3263: 0 * 200 + 1 * 100 + 1 * 50 + 3 * 20 + 28 * 10 + 2 * 5
3264: 0 * 200 + 1 * 100 + 1 * 50 + 3 * 20 + 29 * 10 + 0 * 5
3265: 0 * 200 + 1 * 100 + 1 * 50 + 4 * 20 + 0 * 10 + 54 * 5
3266: 0 * 200 + 1 * 100 + 1 * 50 + 4 * 20 + 1 * 10 + 52 * 5
3267: 0 * 200 + 1 * 100 + 1 * 50 + 4 * 20 + 2 * 10 + 50 * 5
3268: 0 * 200 + 1 * 100 + 1 * 50 + 4 * 20 + 3 * 10 + 48 * 5
3269: 0 * 200 + 1 * 100 + 1 * 50 + 4 * 20 + 4 * 10 + 46 * 5
3270: 0 * 200 + 1 * 100 + 1 * 50 + 4 * 20 + 5 * 10 + 44 * 5
3271: 0 * 200 + 1 * 100 + 1 * 50 + 4 * 20 + 6 * 10 + 42 * 5
3272: 0 * 200 + 1 * 100 + 1 * 50 + 4 * 20 + 7 * 10 + 40 * 5
3273: 0 * 200 + 1 * 100 + 1 * 50 + 4 * 20 + 8 * 10 + 38 * 5
3274: 0 * 200 + 1 * 100 + 1 * 50 + 4 * 20 + 9 * 10 + 36 * 5
3275: 0 * 200 + 1 * 100 + 1 * 50 + 4 * 20 + 10 * 10 + 34 * 5
3276: 0 * 200 + 1 * 100 + 1 * 50 + 4 * 20 + 11 * 10 + 32 * 5
3277: 0 * 200 + 1 * 100 + 1 * 50 + 4 * 20 + 12 * 10 + 30 * 5
3278: 0 * 200 + 1 * 100 + 1 * 50 + 4 * 20 + 13 * 10 + 28 * 5
3279: 0 * 200 + 1 * 100 + 1 * 50 + 4 * 20 + 14 * 10 + 26 * 5
3280: 0 * 200 + 1 * 100 + 1 * 50 + 4 * 20 + 15 * 10 + 24 * 5
3281: 0 * 200 + 1 * 100 + 1 * 50 + 4 * 20 + 16 * 10 + 22 * 5
3282: 0 * 200 + 1 * 100 + 1 * 50 + 4 * 20 + 17 * 10 + 20 * 5
3283: 0 * 200 + 1 * 100 + 1 * 50 + 4 * 20 + 18 * 10 + 18 * 5
3284: 0 * 200 + 1 * 100 + 1 * 50 + 4 * 20 + 19 * 10 + 16 * 5
3285: 0 * 200 + 1 * 100 + 1 * 50 + 4 * 20 + 20 * 10 + 14 * 5
3286: 0 * 200 + 1 * 100 + 1 * 50 + 4 * 20 + 21 * 10 + 12 * 5
3287: 0 * 200 + 1 * 100 + 1 * 50 + 4 * 20 + 22 * 10 + 10 * 5
3288: 0 * 200 + 1 * 100 + 1 * 50 + 4 * 20 + 23 * 10 + 8 * 5
3289: 0 * 200 + 1 * 100 + 1 * 50 + 4 * 20 + 24 * 10 + 6 * 5
3290: 0 * 200 + 1 * 100 + 1 * 50 + 4 * 20 + 25 * 10 + 4 * 5
3291: 0 * 200 + 1 * 100 + 1 * 50 + 4 * 20 + 26 * 10 + 2 * 5
3292: 0 * 200 + 1 * 100 + 1 * 50 + 4 * 20 + 27 * 10 + 0 * 5
3293: 0 * 200 + 1 * 100 + 1 * 50 + 5 * 20 + 0 * 10 + 50 * 5
3294: 0 * 200 + 1 * 100 + 1 * 50 + 5 * 20 + 1 * 10 + 48 * 5
3295: 0 * 200 + 1 * 100 + 1 * 50 + 5 * 20 + 2 * 10 + 46 * 5
3296: 0 * 200 + 1 * 100 + 1 * 50 + 5 * 20 + 3 * 10 + 44 * 5
3297: 0 * 200 + 1 * 100 + 1 * 50 + 5 * 20 + 4 * 10 + 42 * 5
3298: 0 * 200 + 1 * 100 + 1 * 50 + 5 * 20 + 5 * 10 + 40 * 5
3299: 0 * 200 + 1 * 100 + 1 * 50 + 5 * 20 + 6 * 10 + 38 * 5
3300: 0 * 200 + 1 * 100 + 1 * 50 + 5 * 20 + 7 * 10 + 36 * 5
3301: 0 * 200 + 1 * 100 + 1 * 50 + 5 * 20 + 8 * 10 + 34 * 5
3302: 0 * 200 + 1 * 100 + 1 * 50 + 5 * 20 + 9 * 10 + 32 * 5
3303: 0 * 200 + 1 * 100 + 1 * 50 + 5 * 20 + 10 * 10 + 30 * 5
3304: 0 * 200 + 1 * 100 + 1 * 50 + 5 * 20 + 11 * 10 + 28 * 5
3305: 0 * 200 + 1 * 100 + 1 * 50 + 5 * 20 + 12 * 10 + 26 * 5
3306: 0 * 200 + 1 * 100 + 1 * 50 + 5 * 20 + 13 * 10 + 24 * 5
3307: 0 * 200 + 1 * 100 + 1 * 50 + 5 * 20 + 14 * 10 + 22 * 5
3308: 0 * 200 + 1 * 100 + 1 * 50 + 5 * 20 + 15 * 10 + 20 * 5
3309: 0 * 200 + 1 * 100 + 1 * 50 + 5 * 20 + 16 * 10 + 18 * 5
3310: 0 * 200 + 1 * 100 + 1 * 50 + 5 * 20 + 17 * 10 + 16 * 5
3311: 0 * 200 + 1 * 100 + 1 * 50 + 5 * 20 + 18 * 10 + 14 * 5
3312: 0 * 200 + 1 * 100 + 1 * 50 + 5 * 20 + 19 * 10 + 12 * 5
3313: 0 * 200 + 1 * 100 + 1 * 50 + 5 * 20 + 20 * 10 + 10 * 5
3314: 0 * 200 + 1 * 100 + 1 * 50 + 5 * 20 + 21 * 10 + 8 * 5
3315: 0 * 200 + 1 * 100 + 1 * 50 + 5 * 20 + 22 * 10 + 6 * 5
3316: 0 * 200 + 1 * 100 + 1 * 50 + 5 * 20 + 23 * 10 + 4 * 5
3317: 0 * 200 + 1 * 100 + 1 * 50 + 5 * 20 + 24 * 10 + 2 * 5
3318: 0 * 200 + 1 * 100 + 1 * 50 + 5 * 20 + 25 * 10 + 0 * 5
3319: 0 * 200 + 1 * 100 + 1 * 50 + 6 * 20 + 0 * 10 + 46 * 5
3320: 0 * 200 + 1 * 100 + 1 * 50 + 6 * 20 + 1 * 10 + 44 * 5
3321: 0 * 200 + 1 * 100 + 1 * 50 + 6 * 20 + 2 * 10 + 42 * 5
3322: 0 * 200 + 1 * 100 + 1 * 50 + 6 * 20 + 3 * 10 + 40 * 5
3323: 0 * 200 + 1 * 100 + 1 * 50 + 6 * 20 + 4 * 10 + 38 * 5
3324: 0 * 200 + 1 * 100 + 1 * 50 + 6 * 20 + 5 * 10 + 36 * 5
3325: 0 * 200 + 1 * 100 + 1 * 50 + 6 * 20 + 6 * 10 + 34 * 5
3326: 0 * 200 + 1 * 100 + 1 * 50 + 6 * 20 + 7 * 10 + 32 * 5
3327: 0 * 200 + 1 * 100 + 1 * 50 + 6 * 20 + 8 * 10 + 30 * 5
3328: 0 * 200 + 1 * 100 + 1 * 50 + 6 * 20 + 9 * 10 + 28 * 5
3329: 0 * 200 + 1 * 100 + 1 * 50 + 6 * 20 + 10 * 10 + 26 * 5
3330: 0 * 200 + 1 * 100 + 1 * 50 + 6 * 20 + 11 * 10 + 24 * 5
3331: 0 * 200 + 1 * 100 + 1 * 50 + 6 * 20 + 12 * 10 + 22 * 5
3332: 0 * 200 + 1 * 100 + 1 * 50 + 6 * 20 + 13 * 10 + 20 * 5
3333: 0 * 200 + 1 * 100 + 1 * 50 + 6 * 20 + 14 * 10 + 18 * 5
3334: 0 * 200 + 1 * 100 + 1 * 50 + 6 * 20 + 15 * 10 + 16 * 5
3335: 0 * 200 + 1 * 100 + 1 * 50 + 6 * 20 + 16 * 10 + 14 * 5
3336: 0 * 200 + 1 * 100 + 1 * 50 + 6 * 20 + 17 * 10 + 12 * 5
3337: 0 * 200 + 1 * 100 + 1 * 50 + 6 * 20 + 18 * 10 + 10 * 5
3338: 0 * 200 + 1 * 100 + 1 * 50 + 6 * 20 + 19 * 10 + 8 * 5
3339: 0 * 200 + 1 * 100 + 1 * 50 + 6 * 20 + 20 * 10 + 6 * 5
3340: 0 * 200 + 1 * 100 + 1 * 50 + 6 * 20 + 21 * 10 + 4 * 5
3341: 0 * 200 + 1 * 100 + 1 * 50 + 6 * 20 + 22 * 10 + 2 * 5
3342: 0 * 200 + 1 * 100 + 1 * 50 + 6 * 20 + 23 * 10 + 0 * 5
3343: 0 * 200 + 1 * 100 + 1 * 50 + 7 * 20 + 0 * 10 + 42 * 5
3344: 0 * 200 + 1 * 100 + 1 * 50 + 7 * 20 + 1 * 10 + 40 * 5
3345: 0 * 200 + 1 * 100 + 1 * 50 + 7 * 20 + 2 * 10 + 38 * 5
3346: 0 * 200 + 1 * 100 + 1 * 50 + 7 * 20 + 3 * 10 + 36 * 5
3347: 0 * 200 + 1 * 100 + 1 * 50 + 7 * 20 + 4 * 10 + 34 * 5
3348: 0 * 200 + 1 * 100 + 1 * 50 + 7 * 20 + 5 * 10 + 32 * 5
3349: 0 * 200 + 1 * 100 + 1 * 50 + 7 * 20 + 6 * 10 + 30 * 5
3350: 0 * 200 + 1 * 100 + 1 * 50 + 7 * 20 + 7 * 10 + 28 * 5
3351: 0 * 200 + 1 * 100 + 1 * 50 + 7 * 20 + 8 * 10 + 26 * 5
3352: 0 * 200 + 1 * 100 + 1 * 50 + 7 * 20 + 9 * 10 + 24 * 5
3353: 0 * 200 + 1 * 100 + 1 * 50 + 7 * 20 + 10 * 10 + 22 * 5
3354: 0 * 200 + 1 * 100 + 1 * 50 + 7 * 20 + 11 * 10 + 20 * 5
3355: 0 * 200 + 1 * 100 + 1 * 50 + 7 * 20 + 12 * 10 + 18 * 5
3356: 0 * 200 + 1 * 100 + 1 * 50 + 7 * 20 + 13 * 10 + 16 * 5
3357: 0 * 200 + 1 * 100 + 1 * 50 + 7 * 20 + 14 * 10 + 14 * 5
3358: 0 * 200 + 1 * 100 + 1 * 50 + 7 * 20 + 15 * 10 + 12 * 5
3359: 0 * 200 + 1 * 100 + 1 * 50 + 7 * 20 + 16 * 10 + 10 * 5
3360: 0 * 200 + 1 * 100 + 1 * 50 + 7 * 20 + 17 * 10 + 8 * 5
3361: 0 * 200 + 1 * 100 + 1 * 50 + 7 * 20 + 18 * 10 + 6 * 5
3362: 0 * 200 + 1 * 100 + 1 * 50 + 7 * 20 + 19 * 10 + 4 * 5
3363: 0 * 200 + 1 * 100 + 1 * 50 + 7 * 20 + 20 * 10 + 2 * 5
3364: 0 * 200 + 1 * 100 + 1 * 50 + 7 * 20 + 21 * 10 + 0 * 5
3365: 0 * 200 + 1 * 100 + 1 * 50 + 8 * 20 + 0 * 10 + 38 * 5
3366: 0 * 200 + 1 * 100 + 1 * 50 + 8 * 20 + 1 * 10 + 36 * 5
3367: 0 * 200 + 1 * 100 + 1 * 50 + 8 * 20 + 2 * 10 + 34 * 5
3368: 0 * 200 + 1 * 100 + 1 * 50 + 8 * 20 + 3 * 10 + 32 * 5
3369: 0 * 200 + 1 * 100 + 1 * 50 + 8 * 20 + 4 * 10 + 30 * 5
3370: 0 * 200 + 1 * 100 + 1 * 50 + 8 * 20 + 5 * 10 + 28 * 5
3371: 0 * 200 + 1 * 100 + 1 * 50 + 8 * 20 + 6 * 10 + 26 * 5
3372: 0 * 200 + 1 * 100 + 1 * 50 + 8 * 20 + 7 * 10 + 24 * 5
3373: 0 * 200 + 1 * 100 + 1 * 50 + 8 * 20 + 8 * 10 + 22 * 5
3374: 0 * 200 + 1 * 100 + 1 * 50 + 8 * 20 + 9 * 10 + 20 * 5
3375: 0 * 200 + 1 * 100 + 1 * 50 + 8 * 20 + 10 * 10 + 18 * 5
3376: 0 * 200 + 1 * 100 + 1 * 50 + 8 * 20 + 11 * 10 + 16 * 5
3377: 0 * 200 + 1 * 100 + 1 * 50 + 8 * 20 + 12 * 10 + 14 * 5
3378: 0 * 200 + 1 * 100 + 1 * 50 + 8 * 20 + 13 * 10 + 12 * 5
3379: 0 * 200 + 1 * 100 + 1 * 50 + 8 * 20 + 14 * 10 + 10 * 5
3380: 0 * 200 + 1 * 100 + 1 * 50 + 8 * 20 + 15 * 10 + 8 * 5
3381: 0 * 200 + 1 * 100 + 1 * 50 + 8 * 20 + 16 * 10 + 6 * 5
3382: 0 * 200 + 1 * 100 + 1 * 50 + 8 * 20 + 17 * 10 + 4 * 5
3383: 0 * 200 + 1 * 100 + 1 * 50 + 8 * 20 + 18 * 10 + 2 * 5
3384: 0 * 200 + 1 * 100 + 1 * 50 + 8 * 20 + 19 * 10 + 0 * 5
3385: 0 * 200 + 1 * 100 + 1 * 50 + 9 * 20 + 0 * 10 + 34 * 5
3386: 0 * 200 + 1 * 100 + 1 * 50 + 9 * 20 + 1 * 10 + 32 * 5
3387: 0 * 200 + 1 * 100 + 1 * 50 + 9 * 20 + 2 * 10 + 30 * 5
3388: 0 * 200 + 1 * 100 + 1 * 50 + 9 * 20 + 3 * 10 + 28 * 5
3389: 0 * 200 + 1 * 100 + 1 * 50 + 9 * 20 + 4 * 10 + 26 * 5
3390: 0 * 200 + 1 * 100 + 1 * 50 + 9 * 20 + 5 * 10 + 24 * 5
3391: 0 * 200 + 1 * 100 + 1 * 50 + 9 * 20 + 6 * 10 + 22 * 5
3392: 0 * 200 + 1 * 100 + 1 * 50 + 9 * 20 + 7 * 10 + 20 * 5
3393: 0 * 200 + 1 * 100 + 1 * 50 + 9 * 20 + 8 * 10 + 18 * 5
3394: 0 * 200 + 1 * 100 + 1 * 50 + 9 * 20 + 9 * 10 + 16 * 5
3395: 0 * 200 + 1 * 100 + 1 * 50 + 9 * 20 + 10 * 10 + 14 * 5
3396: 0 * 200 + 1 * 100 + 1 * 50 + 9 * 20 + 11 * 10 + 12 * 5
3397: 0 * 200 + 1 * 100 + 1 * 50 + 9 * 20 + 12 * 10 + 10 * 5
3398: 0 * 200 + 1 * 100 + 1 * 50 + 9 * 20 + 13 * 10 + 8 * 5
3399: 0 * 200 + 1 * 100 + 1 * 50 + 9 * 20 + 14 * 10 + 6 * 5
3400: 0 * 200 + 1 * 100 + 1 * 50 + 9 * 20 + 15 * 10 + 4 * 5
3401: 0 * 200 + 1 * 100 + 1 * 50 + 9 * 20 + 16 * 10 + 2 * 5
3402: 0 * 200 + 1 * 100 + 1 * 50 + 9 * 20 + 17 * 10 + 0 * 5
3403: 0 * 200 + 1 * 100 + 1 * 50 + 10 * 20 + 0 * 10 + 30 * 5
3404: 0 * 200 + 1 * 100 + 1 * 50 + 10 * 20 + 1 * 10 + 28 * 5
3405: 0 * 200 + 1 * 100 + 1 * 50 + 10 * 20 + 2 * 10 + 26 * 5
3406: 0 * 200 + 1 * 100 + 1 * 50 + 10 * 20 + 3 * 10 + 24 * 5
3407: 0 * 200 + 1 * 100 + 1 * 50 + 10 * 20 + 4 * 10 + 22 * 5
3408: 0 * 200 + 1 * 100 + 1 * 50 + 10 * 20 + 5 * 10 + 20 * 5
3409: 0 * 200 + 1 * 100 + 1 * 50 + 10 * 20 + 6 * 10 + 18 * 5
3410: 0 * 200 + 1 * 100 + 1 * 50 + 10 * 20 + 7 * 10 + 16 * 5
3411: 0 * 200 + 1 * 100 + 1 * 50 + 10 * 20 + 8 * 10 + 14 * 5
3412: 0 * 200 + 1 * 100 + 1 * 50 + 10 * 20 + 9 * 10 + 12 * 5
3413: 0 * 200 + 1 * 100 + 1 * 50 + 10 * 20 + 10 * 10 + 10 * 5
3414: 0 * 200 + 1 * 100 + 1 * 50 + 10 * 20 + 11 * 10 + 8 * 5
3415: 0 * 200 + 1 * 100 + 1 * 50 + 10 * 20 + 12 * 10 + 6 * 5
3416: 0 * 200 + 1 * 100 + 1 * 50 + 10 * 20 + 13 * 10 + 4 * 5
3417: 0 * 200 + 1 * 100 + 1 * 50 + 10 * 20 + 14 * 10 + 2 * 5
3418: 0 * 200 + 1 * 100 + 1 * 50 + 10 * 20 + 15 * 10 + 0 * 5
3419: 0 * 200 + 1 * 100 + 1 * 50 + 11 * 20 + 0 * 10 + 26 * 5
3420: 0 * 200 + 1 * 100 + 1 * 50 + 11 * 20 + 1 * 10 + 24 * 5
3421: 0 * 200 + 1 * 100 + 1 * 50 + 11 * 20 + 2 * 10 + 22 * 5
3422: 0 * 200 + 1 * 100 + 1 * 50 + 11 * 20 + 3 * 10 + 20 * 5
3423: 0 * 200 + 1 * 100 + 1 * 50 + 11 * 20 + 4 * 10 + 18 * 5
3424: 0 * 200 + 1 * 100 + 1 * 50 + 11 * 20 + 5 * 10 + 16 * 5
3425: 0 * 200 + 1 * 100 + 1 * 50 + 11 * 20 + 6 * 10 + 14 * 5
3426: 0 * 200 + 1 * 100 + 1 * 50 + 11 * 20 + 7 * 10 + 12 * 5
3427: 0 * 200 + 1 * 100 + 1 * 50 + 11 * 20 + 8 * 10 + 10 * 5
3428: 0 * 200 + 1 * 100 + 1 * 50 + 11 * 20 + 9 * 10 + 8 * 5
3429: 0 * 200 + 1 * 100 + 1 * 50 + 11 * 20 + 10 * 10 + 6 * 5
3430: 0 * 200 + 1 * 100 + 1 * 50 + 11 * 20 + 11 * 10 + 4 * 5
3431: 0 * 200 + 1 * 100 + 1 * 50 + 11 * 20 + 12 * 10 + 2 * 5
3432: 0 * 200 + 1 * 100 + 1 * 50 + 11 * 20 + 13 * 10 + 0 * 5
3433: 0 * 200 + 1 * 100 + 1 * 50 + 12 * 20 + 0 * 10 + 22 * 5
3434: 0 * 200 + 1 * 100 + 1 * 50 + 12 * 20 + 1 * 10 + 20 * 5
3435: 0 * 200 + 1 * 100 + 1 * 50 + 12 * 20 + 2 * 10 + 18 * 5
3436: 0 * 200 + 1 * 100 + 1 * 50 + 12 * 20 + 3 * 10 + 16 * 5
3437: 0 * 200 + 1 * 100 + 1 * 50 + 12 * 20 + 4 * 10 + 14 * 5
3438: 0 * 200 + 1 * 100 + 1 * 50 + 12 * 20 + 5 * 10 + 12 * 5
3439: 0 * 200 + 1 * 100 + 1 * 50 + 12 * 20 + 6 * 10 + 10 * 5
3440: 0 * 200 + 1 * 100 + 1 * 50 + 12 * 20 + 7 * 10 + 8 * 5
3441: 0 * 200 + 1 * 100 + 1 * 50 + 12 * 20 + 8 * 10 + 6 * 5
3442: 0 * 200 + 1 * 100 + 1 * 50 + 12 * 20 + 9 * 10 + 4 * 5
3443: 0 * 200 + 1 * 100 + 1 * 50 + 12 * 20 + 10 * 10 + 2 * 5
3444: 0 * 200 + 1 * 100 + 1 * 50 + 12 * 20 + 11 * 10 + 0 * 5
3445: 0 * 200 + 1 * 100 + 1 * 50 + 13 * 20 + 0 * 10 + 18 * 5
3446: 0 * 200 + 1 * 100 + 1 * 50 + 13 * 20 + 1 * 10 + 16 * 5
3447: 0 * 200 + 1 * 100 + 1 * 50 + 13 * 20 + 2 * 10 + 14 * 5
3448: 0 * 200 + 1 * 100 + 1 * 50 + 13 * 20 + 3 * 10 + 12 * 5
3449: 0 * 200 + 1 * 100 + 1 * 50 + 13 * 20 + 4 * 10 + 10 * 5
3450: 0 * 200 + 1 * 100 + 1 * 50 + 13 * 20 + 5 * 10 + 8 * 5
3451: 0 * 200 + 1 * 100 + 1 * 50 + 13 * 20 + 6 * 10 + 6 * 5
3452: 0 * 200 + 1 * 100 + 1 * 50 + 13 * 20 + 7 * 10 + 4 * 5
3453: 0 * 200 + 1 * 100 + 1 * 50 + 13 * 20 + 8 * 10 + 2 * 5
3454: 0 * 200 + 1 * 100 + 1 * 50 + 13 * 20 + 9 * 10 + 0 * 5
3455: 0 * 200 + 1 * 100 + 1 * 50 + 14 * 20 + 0 * 10 + 14 * 5
3456: 0 * 200 + 1 * 100 + 1 * 50 + 14 * 20 + 1 * 10 + 12 * 5
3457: 0 * 200 + 1 * 100 + 1 * 50 + 14 * 20 + 2 * 10 + 10 * 5
3458: 0 * 200 + 1 * 100 + 1 * 50 + 14 * 20 + 3 * 10 + 8 * 5
3459: 0 * 200 + 1 * 100 + 1 * 50 + 14 * 20 + 4 * 10 + 6 * 5
3460: 0 * 200 + 1 * 100 + 1 * 50 + 14 * 20 + 5 * 10 + 4 * 5
3461: 0 * 200 + 1 * 100 + 1 * 50 + 14 * 20 + 6 * 10 + 2 * 5
3462: 0 * 200 + 1 * 100 + 1 * 50 + 14 * 20 + 7 * 10 + 0 * 5
3463: 0 * 200 + 1 * 100 + 1 * 50 + 15 * 20 + 0 * 10 + 10 * 5
3464: 0 * 200 + 1 * 100 + 1 * 50 + 15 * 20 + 1 * 10 + 8 * 5
3465: 0 * 200 + 1 * 100 + 1 * 50 + 15 * 20 + 2 * 10 + 6 * 5
3466: 0 * 200 + 1 * 100 + 1 * 50 + 15 * 20 + 3 * 10 + 4 * 5
3467: 0 * 200 + 1 * 100 + 1 * 50 + 15 * 20 + 4 * 10 + 2 * 5
3468: 0 * 200 + 1 * 100 + 1 * 50 + 15 * 20 + 5 * 10 + 0 * 5
3469: 0 * 200 + 1 * 100 + 1 * 50 + 16 * 20 + 0 * 10 + 6 * 5
3470: 0 * 200 + 1 * 100 + 1 * 50 + 16 * 20 + 1 * 10 + 4 * 5
3471: 0 * 200 + 1 * 100 + 1 * 50 + 16 * 20 + 2 * 10 + 2 * 5
3472: 0 * 200 + 1 * 100 + 1 * 50 + 16 * 20 + 3 * 10 + 0 * 5
3473: 0 * 200 + 1 * 100 + 1 * 50 + 17 * 20 + 0 * 10 + 2 * 5
3474: 0 * 200 + 1 * 100 + 1 * 50 + 17 * 20 + 1 * 10 + 0 * 5
3475: 0 * 200 + 1 * 100 + 2 * 50 + 0 * 20 + 0 * 10 + 60 * 5
3476: 0 * 200 + 1 * 100 + 2 * 50 + 0 * 20 + 1 * 10 + 58 * 5
3477: 0 * 200 + 1 * 100 + 2 * 50 + 0 * 20 + 2 * 10 + 56 * 5
3478: 0 * 200 + 1 * 100 + 2 * 50 + 0 * 20 + 3 * 10 + 54 * 5
3479: 0 * 200 + 1 * 100 + 2 * 50 + 0 * 20 + 4 * 10 + 52 * 5
3480: 0 * 200 + 1 * 100 + 2 * 50 + 0 * 20 + 5 * 10 + 50 * 5
3481: 0 * 200 + 1 * 100 + 2 * 50 + 0 * 20 + 6 * 10 + 48 * 5
3482: 0 * 200 + 1 * 100 + 2 * 50 + 0 * 20 + 7 * 10 + 46 * 5
3483: 0 * 200 + 1 * 100 + 2 * 50 + 0 * 20 + 8 * 10 + 44 * 5
3484: 0 * 200 + 1 * 100 + 2 * 50 + 0 * 20 + 9 * 10 + 42 * 5
3485: 0 * 200 + 1 * 100 + 2 * 50 + 0 * 20 + 10 * 10 + 40 * 5
3486: 0 * 200 + 1 * 100 + 2 * 50 + 0 * 20 + 11 * 10 + 38 * 5
3487: 0 * 200 + 1 * 100 + 2 * 50 + 0 * 20 + 12 * 10 + 36 * 5
3488: 0 * 200 + 1 * 100 + 2 * 50 + 0 * 20 + 13 * 10 + 34 * 5
3489: 0 * 200 + 1 * 100 + 2 * 50 + 0 * 20 + 14 * 10 + 32 * 5
3490: 0 * 200 + 1 * 100 + 2 * 50 + 0 * 20 + 15 * 10 + 30 * 5
3491: 0 * 200 + 1 * 100 + 2 * 50 + 0 * 20 + 16 * 10 + 28 * 5
3492: 0 * 200 + 1 * 100 + 2 * 50 + 0 * 20 + 17 * 10 + 26 * 5
3493: 0 * 200 + 1 * 100 + 2 * 50 + 0 * 20 + 18 * 10 + 24 * 5
3494: 0 * 200 + 1 * 100 + 2 * 50 + 0 * 20 + 19 * 10 + 22 * 5
3495: 0 * 200 + 1 * 100 + 2 * 50 + 0 * 20 + 20 * 10 + 20 * 5
3496: 0 * 200 + 1 * 100 + 2 * 50 + 0 * 20 + 21 * 10 + 18 * 5
3497: 0 * 200 + 1 * 100 + 2 * 50 + 0 * 20 + 22 * 10 + 16 * 5
3498: 0 * 200 + 1 * 100 + 2 * 50 + 0 * 20 + 23 * 10 + 14 * 5
3499: 0 * 200 + 1 * 100 + 2 * 50 + 0 * 20 + 24 * 10 + 12 * 5
3500: 0 * 200 + 1 * 100 + 2 * 50 + 0 * 20 + 25 * 10 + 10 * 5
3501: 0 * 200 + 1 * 100 + 2 * 50 + 0 * 20 + 26 * 10 + 8 * 5
3502: 0 * 200 + 1 * 100 + 2 * 50 + 0 * 20 + 27 * 10 + 6 * 5
3503: 0 * 200 + 1 * 100 + 2 * 50 + 0 * 20 + 28 * 10 + 4 * 5
3504: 0 * 200 + 1 * 100 + 2 * 50 + 0 * 20 + 29 * 10 + 2 * 5
3505: 0 * 200 + 1 * 100 + 2 * 50 + 0 * 20 + 30 * 10 + 0 * 5
3506: 0 * 200 + 1 * 100 + 2 * 50 + 1 * 20 + 0 * 10 + 56 * 5
3507: 0 * 200 + 1 * 100 + 2 * 50 + 1 * 20 + 1 * 10 + 54 * 5
3508: 0 * 200 + 1 * 100 + 2 * 50 + 1 * 20 + 2 * 10 + 52 * 5
3509: 0 * 200 + 1 * 100 + 2 * 50 + 1 * 20 + 3 * 10 + 50 * 5
3510: 0 * 200 + 1 * 100 + 2 * 50 + 1 * 20 + 4 * 10 + 48 * 5
3511: 0 * 200 + 1 * 100 + 2 * 50 + 1 * 20 + 5 * 10 + 46 * 5
3512: 0 * 200 + 1 * 100 + 2 * 50 + 1 * 20 + 6 * 10 + 44 * 5
3513: 0 * 200 + 1 * 100 + 2 * 50 + 1 * 20 + 7 * 10 + 42 * 5
3514: 0 * 200 + 1 * 100 + 2 * 50 + 1 * 20 + 8 * 10 + 40 * 5
3515: 0 * 200 + 1 * 100 + 2 * 50 + 1 * 20 + 9 * 10 + 38 * 5
3516: 0 * 200 + 1 * 100 + 2 * 50 + 1 * 20 + 10 * 10 + 36 * 5
3517: 0 * 200 + 1 * 100 + 2 * 50 + 1 * 20 + 11 * 10 + 34 * 5
3518: 0 * 200 + 1 * 100 + 2 * 50 + 1 * 20 + 12 * 10 + 32 * 5
3519: 0 * 200 + 1 * 100 + 2 * 50 + 1 * 20 + 13 * 10 + 30 * 5
3520: 0 * 200 + 1 * 100 + 2 * 50 + 1 * 20 + 14 * 10 + 28 * 5
3521: 0 * 200 + 1 * 100 + 2 * 50 + 1 * 20 + 15 * 10 + 26 * 5
3522: 0 * 200 + 1 * 100 + 2 * 50 + 1 * 20 + 16 * 10 + 24 * 5
3523: 0 * 200 + 1 * 100 + 2 * 50 + 1 * 20 + 17 * 10 + 22 * 5
3524: 0 * 200 + 1 * 100 + 2 * 50 + 1 * 20 + 18 * 10 + 20 * 5
3525: 0 * 200 + 1 * 100 + 2 * 50 + 1 * 20 + 19 * 10 + 18 * 5
3526: 0 * 200 + 1 * 100 + 2 * 50 + 1 * 20 + 20 * 10 + 16 * 5
3527: 0 * 200 + 1 * 100 + 2 * 50 + 1 * 20 + 21 * 10 + 14 * 5
3528: 0 * 200 + 1 * 100 + 2 * 50 + 1 * 20 + 22 * 10 + 12 * 5
3529: 0 * 200 + 1 * 100 + 2 * 50 + 1 * 20 + 23 * 10 + 10 * 5
3530: 0 * 200 + 1 * 100 + 2 * 50 + 1 * 20 + 24 * 10 + 8 * 5
3531: 0 * 200 + 1 * 100 + 2 * 50 + 1 * 20 + 25 * 10 + 6 * 5
3532: 0 * 200 + 1 * 100 + 2 * 50 + 1 * 20 + 26 * 10 + 4 * 5
3533: 0 * 200 + 1 * 100 + 2 * 50 + 1 * 20 + 27 * 10 + 2 * 5
3534: 0 * 200 + 1 * 100 + 2 * 50 + 1 * 20 + 28 * 10 + 0 * 5
3535: 0 * 200 + 1 * 100 + 2 * 50 + 2 * 20 + 0 * 10 + 52 * 5
3536: 0 * 200 + 1 * 100 + 2 * 50 + 2 * 20 + 1 * 10 + 50 * 5
3537: 0 * 200 + 1 * 100 + 2 * 50 + 2 * 20 + 2 * 10 + 48 * 5
3538: 0 * 200 + 1 * 100 + 2 * 50 + 2 * 20 + 3 * 10 + 46 * 5
3539: 0 * 200 + 1 * 100 + 2 * 50 + 2 * 20 + 4 * 10 + 44 * 5
3540: 0 * 200 + 1 * 100 + 2 * 50 + 2 * 20 + 5 * 10 + 42 * 5
3541: 0 * 200 + 1 * 100 + 2 * 50 + 2 * 20 + 6 * 10 + 40 * 5
3542: 0 * 200 + 1 * 100 + 2 * 50 + 2 * 20 + 7 * 10 + 38 * 5
3543: 0 * 200 + 1 * 100 + 2 * 50 + 2 * 20 + 8 * 10 + 36 * 5
3544: 0 * 200 + 1 * 100 + 2 * 50 + 2 * 20 + 9 * 10 + 34 * 5
3545: 0 * 200 + 1 * 100 + 2 * 50 + 2 * 20 + 10 * 10 + 32 * 5
3546: 0 * 200 + 1 * 100 + 2 * 50 + 2 * 20 + 11 * 10 + 30 * 5
3547: 0 * 200 + 1 * 100 + 2 * 50 + 2 * 20 + 12 * 10 + 28 * 5
3548: 0 * 200 + 1 * 100 + 2 * 50 + 2 * 20 + 13 * 10 + 26 * 5
3549: 0 * 200 + 1 * 100 + 2 * 50 + 2 * 20 + 14 * 10 + 24 * 5
3550: 0 * 200 + 1 * 100 + 2 * 50 + 2 * 20 + 15 * 10 + 22 * 5
3551: 0 * 200 + 1 * 100 + 2 * 50 + 2 * 20 + 16 * 10 + 20 * 5
3552: 0 * 200 + 1 * 100 + 2 * 50 + 2 * 20 + 17 * 10 + 18 * 5
3553: 0 * 200 + 1 * 100 + 2 * 50 + 2 * 20 + 18 * 10 + 16 * 5
3554: 0 * 200 + 1 * 100 + 2 * 50 + 2 * 20 + 19 * 10 + 14 * 5
3555: 0 * 200 + 1 * 100 + 2 * 50 + 2 * 20 + 20 * 10 + 12 * 5
3556: 0 * 200 + 1 * 100 + 2 * 50 + 2 * 20 + 21 * 10 + 10 * 5
3557: 0 * 200 + 1 * 100 + 2 * 50 + 2 * 20 + 22 * 10 + 8 * 5
3558: 0 * 200 + 1 * 100 + 2 * 50 + 2 * 20 + 23 * 10 + 6 * 5
3559: 0 * 200 + 1 * 100 + 2 * 50 + 2 * 20 + 24 * 10 + 4 * 5
3560: 0 * 200 + 1 * 100 + 2 * 50 + 2 * 20 + 25 * 10 + 2 * 5
3561: 0 * 200 + 1 * 100 + 2 * 50 + 2 * 20 + 26 * 10 + 0 * 5
3562: 0 * 200 + 1 * 100 + 2 * 50 + 3 * 20 + 0 * 10 + 48 * 5
3563: 0 * 200 + 1 * 100 + 2 * 50 + 3 * 20 + 1 * 10 + 46 * 5
3564: 0 * 200 + 1 * 100 + 2 * 50 + 3 * 20 + 2 * 10 + 44 * 5
3565: 0 * 200 + 1 * 100 + 2 * 50 + 3 * 20 + 3 * 10 + 42 * 5
3566: 0 * 200 + 1 * 100 + 2 * 50 + 3 * 20 + 4 * 10 + 40 * 5
3567: 0 * 200 + 1 * 100 + 2 * 50 + 3 * 20 + 5 * 10 + 38 * 5
3568: 0 * 200 + 1 * 100 + 2 * 50 + 3 * 20 + 6 * 10 + 36 * 5
3569: 0 * 200 + 1 * 100 + 2 * 50 + 3 * 20 + 7 * 10 + 34 * 5
3570: 0 * 200 + 1 * 100 + 2 * 50 + 3 * 20 + 8 * 10 + 32 * 5
3571: 0 * 200 + 1 * 100 + 2 * 50 + 3 * 20 + 9 * 10 + 30 * 5
3572: 0 * 200 + 1 * 100 + 2 * 50 + 3 * 20 + 10 * 10 + 28 * 5
3573: 0 * 200 + 1 * 100 + 2 * 50 + 3 * 20 + 11 * 10 + 26 * 5
3574: 0 * 200 + 1 * 100 + 2 * 50 + 3 * 20 + 12 * 10 + 24 * 5
3575: 0 * 200 + 1 * 100 + 2 * 50 + 3 * 20 + 13 * 10 + 22 * 5
3576: 0 * 200 + 1 * 100 + 2 * 50 + 3 * 20 + 14 * 10 + 20 * 5
3577: 0 * 200 + 1 * 100 + 2 * 50 + 3 * 20 + 15 * 10 + 18 * 5
3578: 0 * 200 + 1 * 100 + 2 * 50 + 3 * 20 + 16 * 10 + 16 * 5
3579: 0 * 200 + 1 * 100 + 2 * 50 + 3 * 20 + 17 * 10 + 14 * 5
3580: 0 * 200 + 1 * 100 + 2 * 50 + 3 * 20 + 18 * 10 + 12 * 5
3581: 0 * 200 + 1 * 100 + 2 * 50 + 3 * 20 + 19 * 10 + 10 * 5
3582: 0 * 200 + 1 * 100 + 2 * 50 + 3 * 20 + 20 * 10 + 8 * 5
3583: 0 * 200 + 1 * 100 + 2 * 50 + 3 * 20 + 21 * 10 + 6 * 5
3584: 0 * 200 + 1 * 100 + 2 * 50 + 3 * 20 + 22 * 10 + 4 * 5
3585: 0 * 200 + 1 * 100 + 2 * 50 + 3 * 20 + 23 * 10 + 2 * 5
3586: 0 * 200 + 1 * 100 + 2 * 50 + 3 * 20 + 24 * 10 + 0 * 5
3587: 0 * 200 + 1 * 100 + 2 * 50 + 4 * 20 + 0 * 10 + 44 * 5
3588: 0 * 200 + 1 * 100 + 2 * 50 + 4 * 20 + 1 * 10 + 42 * 5
3589: 0 * 200 + 1 * 100 + 2 * 50 + 4 * 20 + 2 * 10 + 40 * 5
3590: 0 * 200 + 1 * 100 + 2 * 50 + 4 * 20 + 3 * 10 + 38 * 5
3591: 0 * 200 + 1 * 100 + 2 * 50 + 4 * 20 + 4 * 10 + 36 * 5
3592: 0 * 200 + 1 * 100 + 2 * 50 + 4 * 20 + 5 * 10 + 34 * 5
3593: 0 * 200 + 1 * 100 + 2 * 50 + 4 * 20 + 6 * 10 + 32 * 5
3594: 0 * 200 + 1 * 100 + 2 * 50 + 4 * 20 + 7 * 10 + 30 * 5
3595: 0 * 200 + 1 * 100 + 2 * 50 + 4 * 20 + 8 * 10 + 28 * 5
3596: 0 * 200 + 1 * 100 + 2 * 50 + 4 * 20 + 9 * 10 + 26 * 5
3597: 0 * 200 + 1 * 100 + 2 * 50 + 4 * 20 + 10 * 10 + 24 * 5
3598: 0 * 200 + 1 * 100 + 2 * 50 + 4 * 20 + 11 * 10 + 22 * 5
3599: 0 * 200 + 1 * 100 + 2 * 50 + 4 * 20 + 12 * 10 + 20 * 5
3600: 0 * 200 + 1 * 100 + 2 * 50 + 4 * 20 + 13 * 10 + 18 * 5
3601: 0 * 200 + 1 * 100 + 2 * 50 + 4 * 20 + 14 * 10 + 16 * 5
3602: 0 * 200 + 1 * 100 + 2 * 50 + 4 * 20 + 15 * 10 + 14 * 5
3603: 0 * 200 + 1 * 100 + 2 * 50 + 4 * 20 + 16 * 10 + 12 * 5
3604: 0 * 200 + 1 * 100 + 2 * 50 + 4 * 20 + 17 * 10 + 10 * 5
3605: 0 * 200 + 1 * 100 + 2 * 50 + 4 * 20 + 18 * 10 + 8 * 5
3606: 0 * 200 + 1 * 100 + 2 * 50 + 4 * 20 + 19 * 10 + 6 * 5
3607: 0 * 200 + 1 * 100 + 2 * 50 + 4 * 20 + 20 * 10 + 4 * 5
3608: 0 * 200 + 1 * 100 + 2 * 50 + 4 * 20 + 21 * 10 + 2 * 5
3609: 0 * 200 + 1 * 100 + 2 * 50 + 4 * 20 + 22 * 10 + 0 * 5
3610: 0 * 200 + 1 * 100 + 2 * 50 + 5 * 20 + 0 * 10 + 40 * 5
3611: 0 * 200 + 1 * 100 + 2 * 50 + 5 * 20 + 1 * 10 + 38 * 5
3612: 0 * 200 + 1 * 100 + 2 * 50 + 5 * 20 + 2 * 10 + 36 * 5
3613: 0 * 200 + 1 * 100 + 2 * 50 + 5 * 20 + 3 * 10 + 34 * 5
3614: 0 * 200 + 1 * 100 + 2 * 50 + 5 * 20 + 4 * 10 + 32 * 5
3615: 0 * 200 + 1 * 100 + 2 * 50 + 5 * 20 + 5 * 10 + 30 * 5
3616: 0 * 200 + 1 * 100 + 2 * 50 + 5 * 20 + 6 * 10 + 28 * 5
3617: 0 * 200 + 1 * 100 + 2 * 50 + 5 * 20 + 7 * 10 + 26 * 5
3618: 0 * 200 + 1 * 100 + 2 * 50 + 5 * 20 + 8 * 10 + 24 * 5
3619: 0 * 200 + 1 * 100 + 2 * 50 + 5 * 20 + 9 * 10 + 22 * 5
3620: 0 * 200 + 1 * 100 + 2 * 50 + 5 * 20 + 10 * 10 + 20 * 5
3621: 0 * 200 + 1 * 100 + 2 * 50 + 5 * 20 + 11 * 10 + 18 * 5
3622: 0 * 200 + 1 * 100 + 2 * 50 + 5 * 20 + 12 * 10 + 16 * 5
3623: 0 * 200 + 1 * 100 + 2 * 50 + 5 * 20 + 13 * 10 + 14 * 5
3624: 0 * 200 + 1 * 100 + 2 * 50 + 5 * 20 + 14 * 10 + 12 * 5
3625: 0 * 200 + 1 * 100 + 2 * 50 + 5 * 20 + 15 * 10 + 10 * 5
3626: 0 * 200 + 1 * 100 + 2 * 50 + 5 * 20 + 16 * 10 + 8 * 5
3627: 0 * 200 + 1 * 100 + 2 * 50 + 5 * 20 + 17 * 10 + 6 * 5
3628: 0 * 200 + 1 * 100 + 2 * 50 + 5 * 20 + 18 * 10 + 4 * 5
3629: 0 * 200 + 1 * 100 + 2 * 50 + 5 * 20 + 19 * 10 + 2 * 5
3630: 0 * 200 + 1 * 100 + 2 * 50 + 5 * 20 + 20 * 10 + 0 * 5
3631: 0 * 200 + 1 * 100 + 2 * 50 + 6 * 20 + 0 * 10 + 36 * 5
3632: 0 * 200 + 1 * 100 + 2 * 50 + 6 * 20 + 1 * 10 + 34 * 5
3633: 0 * 200 + 1 * 100 + 2 * 50 + 6 * 20 + 2 * 10 + 32 * 5
3634: 0 * 200 + 1 * 100 + 2 * 50 + 6 * 20 + 3 * 10 + 30 * 5
3635: 0 * 200 + 1 * 100 + 2 * 50 + 6 * 20 + 4 * 10 + 28 * 5
3636: 0 * 200 + 1 * 100 + 2 * 50 + 6 * 20 + 5 * 10 + 26 * 5
3637: 0 * 200 + 1 * 100 + 2 * 50 + 6 * 20 + 6 * 10 + 24 * 5
3638: 0 * 200 + 1 * 100 + 2 * 50 + 6 * 20 + 7 * 10 + 22 * 5
3639: 0 * 200 + 1 * 100 + 2 * 50 + 6 * 20 + 8 * 10 + 20 * 5
3640: 0 * 200 + 1 * 100 + 2 * 50 + 6 * 20 + 9 * 10 + 18 * 5
3641: 0 * 200 + 1 * 100 + 2 * 50 + 6 * 20 + 10 * 10 + 16 * 5
3642: 0 * 200 + 1 * 100 + 2 * 50 + 6 * 20 + 11 * 10 + 14 * 5
3643: 0 * 200 + 1 * 100 + 2 * 50 + 6 * 20 + 12 * 10 + 12 * 5
3644: 0 * 200 + 1 * 100 + 2 * 50 + 6 * 20 + 13 * 10 + 10 * 5
3645: 0 * 200 + 1 * 100 + 2 * 50 + 6 * 20 + 14 * 10 + 8 * 5
3646: 0 * 200 + 1 * 100 + 2 * 50 + 6 * 20 + 15 * 10 + 6 * 5
3647: 0 * 200 + 1 * 100 + 2 * 50 + 6 * 20 + 16 * 10 + 4 * 5
3648: 0 * 200 + 1 * 100 + 2 * 50 + 6 * 20 + 17 * 10 + 2 * 5
3649: 0 * 200 + 1 * 100 + 2 * 50 + 6 * 20 + 18 * 10 + 0 * 5
3650: 0 * 200 + 1 * 100 + 2 * 50 + 7 * 20 + 0 * 10 + 32 * 5
3651: 0 * 200 + 1 * 100 + 2 * 50 + 7 * 20 + 1 * 10 + 30 * 5
3652: 0 * 200 + 1 * 100 + 2 * 50 + 7 * 20 + 2 * 10 + 28 * 5
3653: 0 * 200 + 1 * 100 + 2 * 50 + 7 * 20 + 3 * 10 + 26 * 5
3654: 0 * 200 + 1 * 100 + 2 * 50 + 7 * 20 + 4 * 10 + 24 * 5
3655: 0 * 200 + 1 * 100 + 2 * 50 + 7 * 20 + 5 * 10 + 22 * 5
3656: 0 * 200 + 1 * 100 + 2 * 50 + 7 * 20 + 6 * 10 + 20 * 5
3657: 0 * 200 + 1 * 100 + 2 * 50 + 7 * 20 + 7 * 10 + 18 * 5
3658: 0 * 200 + 1 * 100 + 2 * 50 + 7 * 20 + 8 * 10 + 16 * 5
3659: 0 * 200 + 1 * 100 + 2 * 50 + 7 * 20 + 9 * 10 + 14 * 5
3660: 0 * 200 + 1 * 100 + 2 * 50 + 7 * 20 + 10 * 10 + 12 * 5
3661: 0 * 200 + 1 * 100 + 2 * 50 + 7 * 20 + 11 * 10 + 10 * 5
3662: 0 * 200 + 1 * 100 + 2 * 50 + 7 * 20 + 12 * 10 + 8 * 5
3663: 0 * 200 + 1 * 100 + 2 * 50 + 7 * 20 + 13 * 10 + 6 * 5
3664: 0 * 200 + 1 * 100 + 2 * 50 + 7 * 20 + 14 * 10 + 4 * 5
3665: 0 * 200 + 1 * 100 + 2 * 50 + 7 * 20 + 15 * 10 + 2 * 5
3666: 0 * 200 + 1 * 100 + 2 * 50 + 7 * 20 + 16 * 10 + 0 * 5
3667: 0 * 200 + 1 * 100 + 2 * 50 + 8 * 20 + 0 * 10 + 28 * 5
3668: 0 * 200 + 1 * 100 + 2 * 50 + 8 * 20 + 1 * 10 + 26 * 5
3669: 0 * 200 + 1 * 100 + 2 * 50 + 8 * 20 + 2 * 10 + 24 * 5
3670: 0 * 200 + 1 * 100 + 2 * 50 + 8 * 20 + 3 * 10 + 22 * 5
3671: 0 * 200 + 1 * 100 + 2 * 50 + 8 * 20 + 4 * 10 + 20 * 5
3672: 0 * 200 + 1 * 100 + 2 * 50 + 8 * 20 + 5 * 10 + 18 * 5
3673: 0 * 200 + 1 * 100 + 2 * 50 + 8 * 20 + 6 * 10 + 16 * 5
3674: 0 * 200 + 1 * 100 + 2 * 50 + 8 * 20 + 7 * 10 + 14 * 5
3675: 0 * 200 + 1 * 100 + 2 * 50 + 8 * 20 + 8 * 10 + 12 * 5
3676: 0 * 200 + 1 * 100 + 2 * 50 + 8 * 20 + 9 * 10 + 10 * 5
3677: 0 * 200 + 1 * 100 + 2 * 50 + 8 * 20 + 10 * 10 + 8 * 5
3678: 0 * 200 + 1 * 100 + 2 * 50 + 8 * 20 + 11 * 10 + 6 * 5
3679: 0 * 200 + 1 * 100 + 2 * 50 + 8 * 20 + 12 * 10 + 4 * 5
3680: 0 * 200 + 1 * 100 + 2 * 50 + 8 * 20 + 13 * 10 + 2 * 5
3681: 0 * 200 + 1 * 100 + 2 * 50 + 8 * 20 + 14 * 10 + 0 * 5
3682: 0 * 200 + 1 * 100 + 2 * 50 + 9 * 20 + 0 * 10 + 24 * 5
3683: 0 * 200 + 1 * 100 + 2 * 50 + 9 * 20 + 1 * 10 + 22 * 5
3684: 0 * 200 + 1 * 100 + 2 * 50 + 9 * 20 + 2 * 10 + 20 * 5
3685: 0 * 200 + 1 * 100 + 2 * 50 + 9 * 20 + 3 * 10 + 18 * 5
3686: 0 * 200 + 1 * 100 + 2 * 50 + 9 * 20 + 4 * 10 + 16 * 5
3687: 0 * 200 + 1 * 100 + 2 * 50 + 9 * 20 + 5 * 10 + 14 * 5
3688: 0 * 200 + 1 * 100 + 2 * 50 + 9 * 20 + 6 * 10 + 12 * 5
3689: 0 * 200 + 1 * 100 + 2 * 50 + 9 * 20 + 7 * 10 + 10 * 5
3690: 0 * 200 + 1 * 100 + 2 * 50 + 9 * 20 + 8 * 10 + 8 * 5
3691: 0 * 200 + 1 * 100 + 2 * 50 + 9 * 20 + 9 * 10 + 6 * 5
3692: 0 * 200 + 1 * 100 + 2 * 50 + 9 * 20 + 10 * 10 + 4 * 5
3693: 0 * 200 + 1 * 100 + 2 * 50 + 9 * 20 + 11 * 10 + 2 * 5
3694: 0 * 200 + 1 * 100 + 2 * 50 + 9 * 20 + 12 * 10 + 0 * 5
3695: 0 * 200 + 1 * 100 + 2 * 50 + 10 * 20 + 0 * 10 + 20 * 5
3696: 0 * 200 + 1 * 100 + 2 * 50 + 10 * 20 + 1 * 10 + 18 * 5
3697: 0 * 200 + 1 * 100 + 2 * 50 + 10 * 20 + 2 * 10 + 16 * 5
3698: 0 * 200 + 1 * 100 + 2 * 50 + 10 * 20 + 3 * 10 + 14 * 5
3699: 0 * 200 + 1 * 100 + 2 * 50 + 10 * 20 + 4 * 10 + 12 * 5
3700: 0 * 200 + 1 * 100 + 2 * 50 + 10 * 20 + 5 * 10 + 10 * 5
3701: 0 * 200 + 1 * 100 + 2 * 50 + 10 * 20 + 6 * 10 + 8 * 5
3702: 0 * 200 + 1 * 100 + 2 * 50 + 10 * 20 + 7 * 10 + 6 * 5
3703: 0 * 200 + 1 * 100 + 2 * 50 + 10 * 20 + 8 * 10 + 4 * 5
3704: 0 * 200 + 1 * 100 + 2 * 50 + 10 * 20 + 9 * 10 + 2 * 5
3705: 0 * 200 + 1 * 100 + 2 * 50 + 10 * 20 + 10 * 10 + 0 * 5
3706: 0 * 200 + 1 * 100 + 2 * 50 + 11 * 20 + 0 * 10 + 16 * 5
3707: 0 * 200 + 1 * 100 + 2 * 50 + 11 * 20 + 1 * 10 + 14 * 5
3708: 0 * 200 + 1 * 100 + 2 * 50 + 11 * 20 + 2 * 10 + 12 * 5
3709: 0 * 200 + 1 * 100 + 2 * 50 + 11 * 20 + 3 * 10 + 10 * 5
3710: 0 * 200 + 1 * 100 + 2 * 50 + 11 * 20 + 4 * 10 + 8 * 5
3711: 0 * 200 + 1 * 100 + 2 * 50 + 11 * 20 + 5 * 10 + 6 * 5
3712: 0 * 200 + 1 * 100 + 2 * 50 + 11 * 20 + 6 * 10 + 4 * 5
3713: 0 * 200 + 1 * 100 + 2 * 50 + 11 * 20 + 7 * 10 + 2 * 5
3714: 0 * 200 + 1 * 100 + 2 * 50 + 11 * 20 + 8 * 10 + 0 * 5
3715: 0 * 200 + 1 * 100 + 2 * 50 + 12 * 20 + 0 * 10 + 12 * 5
3716: 0 * 200 + 1 * 100 + 2 * 50 + 12 * 20 + 1 * 10 + 10 * 5
3717: 0 * 200 + 1 * 100 + 2 * 50 + 12 * 20 + 2 * 10 + 8 * 5
3718: 0 * 200 + 1 * 100 + 2 * 50 + 12 * 20 + 3 * 10 + 6 * 5
3719: 0 * 200 + 1 * 100 + 2 * 50 + 12 * 20 + 4 * 10 + 4 * 5
3720: 0 * 200 + 1 * 100 + 2 * 50 + 12 * 20 + 5 * 10 + 2 * 5
3721: 0 * 200 + 1 * 100 + 2 * 50 + 12 * 20 + 6 * 10 + 0 * 5
3722: 0 * 200 + 1 * 100 + 2 * 50 + 13 * 20 + 0 * 10 + 8 * 5
3723: 0 * 200 + 1 * 100 + 2 * 50 + 13 * 20 + 1 * 10 + 6 * 5
3724: 0 * 200 + 1 * 100 + 2 * 50 + 13 * 20 + 2 * 10 + 4 * 5
3725: 0 * 200 + 1 * 100 + 2 * 50 + 13 * 20 + 3 * 10 + 2 * 5
3726: 0 * 200 + 1 * 100 + 2 * 50 + 13 * 20 + 4 * 10 + 0 * 5
3727: 0 * 200 + 1 * 100 + 2 * 50 + 14 * 20 + 0 * 10 + 4 * 5
3728: 0 * 200 + 1 * 100 + 2 * 50 + 14 * 20 + 1 * 10 + 2 * 5
3729: 0 * 200 + 1 * 100 + 2 * 50 + 14 * 20 + 2 * 10 + 0 * 5
3730: 0 * 200 + 1 * 100 + 2 * 50 + 15 * 20 + 0 * 10 + 0 * 5
3731: 0 * 200 + 1 * 100 + 3 * 50 + 0 * 20 + 0 * 10 + 50 * 5
3732: 0 * 200 + 1 * 100 + 3 * 50 + 0 * 20 + 1 * 10 + 48 * 5
3733: 0 * 200 + 1 * 100 + 3 * 50 + 0 * 20 + 2 * 10 + 46 * 5
3734: 0 * 200 + 1 * 100 + 3 * 50 + 0 * 20 + 3 * 10 + 44 * 5
3735: 0 * 200 + 1 * 100 + 3 * 50 + 0 * 20 + 4 * 10 + 42 * 5
3736: 0 * 200 + 1 * 100 + 3 * 50 + 0 * 20 + 5 * 10 + 40 * 5
3737: 0 * 200 + 1 * 100 + 3 * 50 + 0 * 20 + 6 * 10 + 38 * 5
3738: 0 * 200 + 1 * 100 + 3 * 50 + 0 * 20 + 7 * 10 + 36 * 5
3739: 0 * 200 + 1 * 100 + 3 * 50 + 0 * 20 + 8 * 10 + 34 * 5
3740: 0 * 200 + 1 * 100 + 3 * 50 + 0 * 20 + 9 * 10 + 32 * 5
3741: 0 * 200 + 1 * 100 + 3 * 50 + 0 * 20 + 10 * 10 + 30 * 5
3742: 0 * 200 + 1 * 100 + 3 * 50 + 0 * 20 + 11 * 10 + 28 * 5
3743: 0 * 200 + 1 * 100 + 3 * 50 + 0 * 20 + 12 * 10 + 26 * 5
3744: 0 * 200 + 1 * 100 + 3 * 50 + 0 * 20 + 13 * 10 + 24 * 5
3745: 0 * 200 + 1 * 100 + 3 * 50 + 0 * 20 + 14 * 10 + 22 * 5
3746: 0 * 200 + 1 * 100 + 3 * 50 + 0 * 20 + 15 * 10 + 20 * 5
3747: 0 * 200 + 1 * 100 + 3 * 50 + 0 * 20 + 16 * 10 + 18 * 5
3748: 0 * 200 + 1 * 100 + 3 * 50 + 0 * 20 + 17 * 10 + 16 * 5
3749: 0 * 200 + 1 * 100 + 3 * 50 + 0 * 20 + 18 * 10 + 14 * 5
3750: 0 * 200 + 1 * 100 + 3 * 50 + 0 * 20 + 19 * 10 + 12 * 5
3751: 0 * 200 + 1 * 100 + 3 * 50 + 0 * 20 + 20 * 10 + 10 * 5
3752: 0 * 200 + 1 * 100 + 3 * 50 + 0 * 20 + 21 * 10 + 8 * 5
3753: 0 * 200 + 1 * 100 + 3 * 50 + 0 * 20 + 22 * 10 + 6 * 5
3754: 0 * 200 + 1 * 100 + 3 * 50 + 0 * 20 + 23 * 10 + 4 * 5
3755: 0 * 200 + 1 * 100 + 3 * 50 + 0 * 20 + 24 * 10 + 2 * 5
3756: 0 * 200 + 1 * 100 + 3 * 50 + 0 * 20 + 25 * 10 + 0 * 5
3757: 0 * 200 + 1 * 100 + 3 * 50 + 1 * 20 + 0 * 10 + 46 * 5
3758: 0 * 200 + 1 * 100 + 3 * 50 + 1 * 20 + 1 * 10 + 44 * 5
3759: 0 * 200 + 1 * 100 + 3 * 50 + 1 * 20 + 2 * 10 + 42 * 5
3760: 0 * 200 + 1 * 100 + 3 * 50 + 1 * 20 + 3 * 10 + 40 * 5
3761: 0 * 200 + 1 * 100 + 3 * 50 + 1 * 20 + 4 * 10 + 38 * 5
3762: 0 * 200 + 1 * 100 + 3 * 50 + 1 * 20 + 5 * 10 + 36 * 5
3763: 0 * 200 + 1 * 100 + 3 * 50 + 1 * 20 + 6 * 10 + 34 * 5
3764: 0 * 200 + 1 * 100 + 3 * 50 + 1 * 20 + 7 * 10 + 32 * 5
3765: 0 * 200 + 1 * 100 + 3 * 50 + 1 * 20 + 8 * 10 + 30 * 5
3766: 0 * 200 + 1 * 100 + 3 * 50 + 1 * 20 + 9 * 10 + 28 * 5
3767: 0 * 200 + 1 * 100 + 3 * 50 + 1 * 20 + 10 * 10 + 26 * 5
3768: 0 * 200 + 1 * 100 + 3 * 50 + 1 * 20 + 11 * 10 + 24 * 5
3769: 0 * 200 + 1 * 100 + 3 * 50 + 1 * 20 + 12 * 10 + 22 * 5
3770: 0 * 200 + 1 * 100 + 3 * 50 + 1 * 20 + 13 * 10 + 20 * 5
3771: 0 * 200 + 1 * 100 + 3 * 50 + 1 * 20 + 14 * 10 + 18 * 5
3772: 0 * 200 + 1 * 100 + 3 * 50 + 1 * 20 + 15 * 10 + 16 * 5
3773: 0 * 200 + 1 * 100 + 3 * 50 + 1 * 20 + 16 * 10 + 14 * 5
3774: 0 * 200 + 1 * 100 + 3 * 50 + 1 * 20 + 17 * 10 + 12 * 5
3775: 0 * 200 + 1 * 100 + 3 * 50 + 1 * 20 + 18 * 10 + 10 * 5
3776: 0 * 200 + 1 * 100 + 3 * 50 + 1 * 20 + 19 * 10 + 8 * 5
3777: 0 * 200 + 1 * 100 + 3 * 50 + 1 * 20 + 20 * 10 + 6 * 5
3778: 0 * 200 + 1 * 100 + 3 * 50 + 1 * 20 + 21 * 10 + 4 * 5
3779: 0 * 200 + 1 * 100 + 3 * 50 + 1 * 20 + 22 * 10 + 2 * 5
3780: 0 * 200 + 1 * 100 + 3 * 50 + 1 * 20 + 23 * 10 + 0 * 5
3781: 0 * 200 + 1 * 100 + 3 * 50 + 2 * 20 + 0 * 10 + 42 * 5
3782: 0 * 200 + 1 * 100 + 3 * 50 + 2 * 20 + 1 * 10 + 40 * 5
3783: 0 * 200 + 1 * 100 + 3 * 50 + 2 * 20 + 2 * 10 + 38 * 5
3784: 0 * 200 + 1 * 100 + 3 * 50 + 2 * 20 + 3 * 10 + 36 * 5
3785: 0 * 200 + 1 * 100 + 3 * 50 + 2 * 20 + 4 * 10 + 34 * 5
3786: 0 * 200 + 1 * 100 + 3 * 50 + 2 * 20 + 5 * 10 + 32 * 5
3787: 0 * 200 + 1 * 100 + 3 * 50 + 2 * 20 + 6 * 10 + 30 * 5
3788: 0 * 200 + 1 * 100 + 3 * 50 + 2 * 20 + 7 * 10 + 28 * 5
3789: 0 * 200 + 1 * 100 + 3 * 50 + 2 * 20 + 8 * 10 + 26 * 5
3790: 0 * 200 + 1 * 100 + 3 * 50 + 2 * 20 + 9 * 10 + 24 * 5
3791: 0 * 200 + 1 * 100 + 3 * 50 + 2 * 20 + 10 * 10 + 22 * 5
3792: 0 * 200 + 1 * 100 + 3 * 50 + 2 * 20 + 11 * 10 + 20 * 5
3793: 0 * 200 + 1 * 100 + 3 * 50 + 2 * 20 + 12 * 10 + 18 * 5
3794: 0 * 200 + 1 * 100 + 3 * 50 + 2 * 20 + 13 * 10 + 16 * 5
3795: 0 * 200 + 1 * 100 + 3 * 50 + 2 * 20 + 14 * 10 + 14 * 5
3796: 0 * 200 + 1 * 100 + 3 * 50 + 2 * 20 + 15 * 10 + 12 * 5
3797: 0 * 200 + 1 * 100 + 3 * 50 + 2 * 20 + 16 * 10 + 10 * 5
3798: 0 * 200 + 1 * 100 + 3 * 50 + 2 * 20 + 17 * 10 + 8 * 5
3799: 0 * 200 + 1 * 100 + 3 * 50 + 2 * 20 + 18 * 10 + 6 * 5
3800: 0 * 200 + 1 * 100 + 3 * 50 + 2 * 20 + 19 * 10 + 4 * 5
3801: 0 * 200 + 1 * 100 + 3 * 50 + 2 * 20 + 20 * 10 + 2 * 5
3802: 0 * 200 + 1 * 100 + 3 * 50 + 2 * 20 + 21 * 10 + 0 * 5
3803: 0 * 200 + 1 * 100 + 3 * 50 + 3 * 20 + 0 * 10 + 38 * 5
3804: 0 * 200 + 1 * 100 + 3 * 50 + 3 * 20 + 1 * 10 + 36 * 5
3805: 0 * 200 + 1 * 100 + 3 * 50 + 3 * 20 + 2 * 10 + 34 * 5
3806: 0 * 200 + 1 * 100 + 3 * 50 + 3 * 20 + 3 * 10 + 32 * 5
3807: 0 * 200 + 1 * 100 + 3 * 50 + 3 * 20 + 4 * 10 + 30 * 5
3808: 0 * 200 + 1 * 100 + 3 * 50 + 3 * 20 + 5 * 10 + 28 * 5
3809: 0 * 200 + 1 * 100 + 3 * 50 + 3 * 20 + 6 * 10 + 26 * 5
3810: 0 * 200 + 1 * 100 + 3 * 50 + 3 * 20 + 7 * 10 + 24 * 5
3811: 0 * 200 + 1 * 100 + 3 * 50 + 3 * 20 + 8 * 10 + 22 * 5
3812: 0 * 200 + 1 * 100 + 3 * 50 + 3 * 20 + 9 * 10 + 20 * 5
3813: 0 * 200 + 1 * 100 + 3 * 50 + 3 * 20 + 10 * 10 + 18 * 5
3814: 0 * 200 + 1 * 100 + 3 * 50 + 3 * 20 + 11 * 10 + 16 * 5
3815: 0 * 200 + 1 * 100 + 3 * 50 + 3 * 20 + 12 * 10 + 14 * 5
3816: 0 * 200 + 1 * 100 + 3 * 50 + 3 * 20 + 13 * 10 + 12 * 5
3817: 0 * 200 + 1 * 100 + 3 * 50 + 3 * 20 + 14 * 10 + 10 * 5
3818: 0 * 200 + 1 * 100 + 3 * 50 + 3 * 20 + 15 * 10 + 8 * 5
3819: 0 * 200 + 1 * 100 + 3 * 50 + 3 * 20 + 16 * 10 + 6 * 5
3820: 0 * 200 + 1 * 100 + 3 * 50 + 3 * 20 + 17 * 10 + 4 * 5
3821: 0 * 200 + 1 * 100 + 3 * 50 + 3 * 20 + 18 * 10 + 2 * 5
3822: 0 * 200 + 1 * 100 + 3 * 50 + 3 * 20 + 19 * 10 + 0 * 5
3823: 0 * 200 + 1 * 100 + 3 * 50 + 4 * 20 + 0 * 10 + 34 * 5
3824: 0 * 200 + 1 * 100 + 3 * 50 + 4 * 20 + 1 * 10 + 32 * 5
3825: 0 * 200 + 1 * 100 + 3 * 50 + 4 * 20 + 2 * 10 + 30 * 5
3826: 0 * 200 + 1 * 100 + 3 * 50 + 4 * 20 + 3 * 10 + 28 * 5
3827: 0 * 200 + 1 * 100 + 3 * 50 + 4 * 20 + 4 * 10 + 26 * 5
3828: 0 * 200 + 1 * 100 + 3 * 50 + 4 * 20 + 5 * 10 + 24 * 5
3829: 0 * 200 + 1 * 100 + 3 * 50 + 4 * 20 + 6 * 10 + 22 * 5
3830: 0 * 200 + 1 * 100 + 3 * 50 + 4 * 20 + 7 * 10 + 20 * 5
3831: 0 * 200 + 1 * 100 + 3 * 50 + 4 * 20 + 8 * 10 + 18 * 5
3832: 0 * 200 + 1 * 100 + 3 * 50 + 4 * 20 + 9 * 10 + 16 * 5
3833: 0 * 200 + 1 * 100 + 3 * 50 + 4 * 20 + 10 * 10 + 14 * 5
3834: 0 * 200 + 1 * 100 + 3 * 50 + 4 * 20 + 11 * 10 + 12 * 5
3835: 0 * 200 + 1 * 100 + 3 * 50 + 4 * 20 + 12 * 10 + 10 * 5
3836: 0 * 200 + 1 * 100 + 3 * 50 + 4 * 20 + 13 * 10 + 8 * 5
3837: 0 * 200 + 1 * 100 + 3 * 50 + 4 * 20 + 14 * 10 + 6 * 5
3838: 0 * 200 + 1 * 100 + 3 * 50 + 4 * 20 + 15 * 10 + 4 * 5
3839: 0 * 200 + 1 * 100 + 3 * 50 + 4 * 20 + 16 * 10 + 2 * 5
3840: 0 * 200 + 1 * 100 + 3 * 50 + 4 * 20 + 17 * 10 + 0 * 5
3841: 0 * 200 + 1 * 100 + 3 * 50 + 5 * 20 + 0 * 10 + 30 * 5
3842: 0 * 200 + 1 * 100 + 3 * 50 + 5 * 20 + 1 * 10 + 28 * 5
3843: 0 * 200 + 1 * 100 + 3 * 50 + 5 * 20 + 2 * 10 + 26 * 5
3844: 0 * 200 + 1 * 100 + 3 * 50 + 5 * 20 + 3 * 10 + 24 * 5
3845: 0 * 200 + 1 * 100 + 3 * 50 + 5 * 20 + 4 * 10 + 22 * 5
3846: 0 * 200 + 1 * 100 + 3 * 50 + 5 * 20 + 5 * 10 + 20 * 5
3847: 0 * 200 + 1 * 100 + 3 * 50 + 5 * 20 + 6 * 10 + 18 * 5
3848: 0 * 200 + 1 * 100 + 3 * 50 + 5 * 20 + 7 * 10 + 16 * 5
3849: 0 * 200 + 1 * 100 + 3 * 50 + 5 * 20 + 8 * 10 + 14 * 5
3850: 0 * 200 + 1 * 100 + 3 * 50 + 5 * 20 + 9 * 10 + 12 * 5
3851: 0 * 200 + 1 * 100 + 3 * 50 + 5 * 20 + 10 * 10 + 10 * 5
3852: 0 * 200 + 1 * 100 + 3 * 50 + 5 * 20 + 11 * 10 + 8 * 5
3853: 0 * 200 + 1 * 100 + 3 * 50 + 5 * 20 + 12 * 10 + 6 * 5
3854: 0 * 200 + 1 * 100 + 3 * 50 + 5 * 20 + 13 * 10 + 4 * 5
3855: 0 * 200 + 1 * 100 + 3 * 50 + 5 * 20 + 14 * 10 + 2 * 5
3856: 0 * 200 + 1 * 100 + 3 * 50 + 5 * 20 + 15 * 10 + 0 * 5
3857: 0 * 200 + 1 * 100 + 3 * 50 + 6 * 20 + 0 * 10 + 26 * 5
3858: 0 * 200 + 1 * 100 + 3 * 50 + 6 * 20 + 1 * 10 + 24 * 5
3859: 0 * 200 + 1 * 100 + 3 * 50 + 6 * 20 + 2 * 10 + 22 * 5
3860: 0 * 200 + 1 * 100 + 3 * 50 + 6 * 20 + 3 * 10 + 20 * 5
3861: 0 * 200 + 1 * 100 + 3 * 50 + 6 * 20 + 4 * 10 + 18 * 5
3862: 0 * 200 + 1 * 100 + 3 * 50 + 6 * 20 + 5 * 10 + 16 * 5
3863: 0 * 200 + 1 * 100 + 3 * 50 + 6 * 20 + 6 * 10 + 14 * 5
3864: 0 * 200 + 1 * 100 + 3 * 50 + 6 * 20 + 7 * 10 + 12 * 5
3865: 0 * 200 + 1 * 100 + 3 * 50 + 6 * 20 + 8 * 10 + 10 * 5
3866: 0 * 200 + 1 * 100 + 3 * 50 + 6 * 20 + 9 * 10 + 8 * 5
3867: 0 * 200 + 1 * 100 + 3 * 50 + 6 * 20 + 10 * 10 + 6 * 5
3868: 0 * 200 + 1 * 100 + 3 * 50 + 6 * 20 + 11 * 10 + 4 * 5
3869: 0 * 200 + 1 * 100 + 3 * 50 + 6 * 20 + 12 * 10 + 2 * 5
3870: 0 * 200 + 1 * 100 + 3 * 50 + 6 * 20 + 13 * 10 + 0 * 5
3871: 0 * 200 + 1 * 100 + 3 * 50 + 7 * 20 + 0 * 10 + 22 * 5
3872: 0 * 200 + 1 * 100 + 3 * 50 + 7 * 20 + 1 * 10 + 20 * 5
3873: 0 * 200 + 1 * 100 + 3 * 50 + 7 * 20 + 2 * 10 + 18 * 5
3874: 0 * 200 + 1 * 100 + 3 * 50 + 7 * 20 + 3 * 10 + 16 * 5
3875: 0 * 200 + 1 * 100 + 3 * 50 + 7 * 20 + 4 * 10 + 14 * 5
3876: 0 * 200 + 1 * 100 + 3 * 50 + 7 * 20 + 5 * 10 + 12 * 5
3877: 0 * 200 + 1 * 100 + 3 * 50 + 7 * 20 + 6 * 10 + 10 * 5
3878: 0 * 200 + 1 * 100 + 3 * 50 + 7 * 20 + 7 * 10 + 8 * 5
3879: 0 * 200 + 1 * 100 + 3 * 50 + 7 * 20 + 8 * 10 + 6 * 5
3880: 0 * 200 + 1 * 100 + 3 * 50 + 7 * 20 + 9 * 10 + 4 * 5
3881: 0 * 200 + 1 * 100 + 3 * 50 + 7 * 20 + 10 * 10 + 2 * 5
3882: 0 * 200 + 1 * 100 + 3 * 50 + 7 * 20 + 11 * 10 + 0 * 5
3883: 0 * 200 + 1 * 100 + 3 * 50 + 8 * 20 + 0 * 10 + 18 * 5
3884: 0 * 200 + 1 * 100 + 3 * 50 + 8 * 20 + 1 * 10 + 16 * 5
3885: 0 * 200 + 1 * 100 + 3 * 50 + 8 * 20 + 2 * 10 + 14 * 5
3886: 0 * 200 + 1 * 100 + 3 * 50 + 8 * 20 + 3 * 10 + 12 * 5
3887: 0 * 200 + 1 * 100 + 3 * 50 + 8 * 20 + 4 * 10 + 10 * 5
3888: 0 * 200 + 1 * 100 + 3 * 50 + 8 * 20 + 5 * 10 + 8 * 5
3889: 0 * 200 + 1 * 100 + 3 * 50 + 8 * 20 + 6 * 10 + 6 * 5
3890: 0 * 200 + 1 * 100 + 3 * 50 + 8 * 20 + 7 * 10 + 4 * 5
3891: 0 * 200 + 1 * 100 + 3 * 50 + 8 * 20 + 8 * 10 + 2 * 5
3892: 0 * 200 + 1 * 100 + 3 * 50 + 8 * 20 + 9 * 10 + 0 * 5
3893: 0 * 200 + 1 * 100 + 3 * 50 + 9 * 20 + 0 * 10 + 14 * 5
3894: 0 * 200 + 1 * 100 + 3 * 50 + 9 * 20 + 1 * 10 + 12 * 5
3895: 0 * 200 + 1 * 100 + 3 * 50 + 9 * 20 + 2 * 10 + 10 * 5
3896: 0 * 200 + 1 * 100 + 3 * 50 + 9 * 20 + 3 * 10 + 8 * 5
3897: 0 * 200 + 1 * 100 + 3 * 50 + 9 * 20 + 4 * 10 + 6 * 5
3898: 0 * 200 + 1 * 100 + 3 * 50 + 9 * 20 + 5 * 10 + 4 * 5
3899: 0 * 200 + 1 * 100 + 3 * 50 + 9 * 20 + 6 * 10 + 2 * 5
3900: 0 * 200 + 1 * 100 + 3 * 50 + 9 * 20 + 7 * 10 + 0 * 5
3901: 0 * 200 + 1 * 100 + 3 * 50 + 10 * 20 + 0 * 10 + 10 * 5
3902: 0 * 200 + 1 * 100 + 3 * 50 + 10 * 20 + 1 * 10 + 8 * 5
3903: 0 * 200 + 1 * 100 + 3 * 50 + 10 * 20 + 2 * 10 + 6 * 5
3904: 0 * 200 + 1 * 100 + 3 * 50 + 10 * 20 + 3 * 10 + 4 * 5
3905: 0 * 200 + 1 * 100 + 3 * 50 + 10 * 20 + 4 * 10 + 2 * 5
3906: 0 * 200 + 1 * 100 + 3 * 50 + 10 * 20 + 5 * 10 + 0 * 5
3907: 0 * 200 + 1 * 100 + 3 * 50 + 11 * 20 + 0 * 10 + 6 * 5
3908: 0 * 200 + 1 * 100 + 3 * 50 + 11 * 20 + 1 * 10 + 4 * 5
3909: 0 * 200 + 1 * 100 + 3 * 50 + 11 * 20 + 2 * 10 + 2 * 5
3910: 0 * 200 + 1 * 100 + 3 * 50 + 11 * 20 + 3 * 10 + 0 * 5
3911: 0 * 200 + 1 * 100 + 3 * 50 + 12 * 20 + 0 * 10 + 2 * 5
3912: 0 * 200 + 1 * 100 + 3 * 50 + 12 * 20 + 1 * 10 + 0 * 5
3913: 0 * 200 + 1 * 100 + 4 * 50 + 0 * 20 + 0 * 10 + 40 * 5
3914: 0 * 200 + 1 * 100 + 4 * 50 + 0 * 20 + 1 * 10 + 38 * 5
3915: 0 * 200 + 1 * 100 + 4 * 50 + 0 * 20 + 2 * 10 + 36 * 5
3916: 0 * 200 + 1 * 100 + 4 * 50 + 0 * 20 + 3 * 10 + 34 * 5
3917: 0 * 200 + 1 * 100 + 4 * 50 + 0 * 20 + 4 * 10 + 32 * 5
3918: 0 * 200 + 1 * 100 + 4 * 50 + 0 * 20 + 5 * 10 + 30 * 5
3919: 0 * 200 + 1 * 100 + 4 * 50 + 0 * 20 + 6 * 10 + 28 * 5
3920: 0 * 200 + 1 * 100 + 4 * 50 + 0 * 20 + 7 * 10 + 26 * 5
3921: 0 * 200 + 1 * 100 + 4 * 50 + 0 * 20 + 8 * 10 + 24 * 5
3922: 0 * 200 + 1 * 100 + 4 * 50 + 0 * 20 + 9 * 10 + 22 * 5
3923: 0 * 200 + 1 * 100 + 4 * 50 + 0 * 20 + 10 * 10 + 20 * 5
3924: 0 * 200 + 1 * 100 + 4 * 50 + 0 * 20 + 11 * 10 + 18 * 5
3925: 0 * 200 + 1 * 100 + 4 * 50 + 0 * 20 + 12 * 10 + 16 * 5
3926: 0 * 200 + 1 * 100 + 4 * 50 + 0 * 20 + 13 * 10 + 14 * 5
3927: 0 * 200 + 1 * 100 + 4 * 50 + 0 * 20 + 14 * 10 + 12 * 5
3928: 0 * 200 + 1 * 100 + 4 * 50 + 0 * 20 + 15 * 10 + 10 * 5
3929: 0 * 200 + 1 * 100 + 4 * 50 + 0 * 20 + 16 * 10 + 8 * 5
3930: 0 * 200 + 1 * 100 + 4 * 50 + 0 * 20 + 17 * 10 + 6 * 5
3931: 0 * 200 + 1 * 100 + 4 * 50 + 0 * 20 + 18 * 10 + 4 * 5
3932: 0 * 200 + 1 * 100 + 4 * 50 + 0 * 20 + 19 * 10 + 2 * 5
3933: 0 * 200 + 1 * 100 + 4 * 50 + 0 * 20 + 20 * 10 + 0 * 5
3934: 0 * 200 + 1 * 100 + 4 * 50 + 1 * 20 + 0 * 10 + 36 * 5
3935: 0 * 200 + 1 * 100 + 4 * 50 + 1 * 20 + 1 * 10 + 34 * 5
3936: 0 * 200 + 1 * 100 + 4 * 50 + 1 * 20 + 2 * 10 + 32 * 5
3937: 0 * 200 + 1 * 100 + 4 * 50 + 1 * 20 + 3 * 10 + 30 * 5
3938: 0 * 200 + 1 * 100 + 4 * 50 + 1 * 20 + 4 * 10 + 28 * 5
3939: 0 * 200 + 1 * 100 + 4 * 50 + 1 * 20 + 5 * 10 + 26 * 5
3940: 0 * 200 + 1 * 100 + 4 * 50 + 1 * 20 + 6 * 10 + 24 * 5
3941: 0 * 200 + 1 * 100 + 4 * 50 + 1 * 20 + 7 * 10 + 22 * 5
3942: 0 * 200 + 1 * 100 + 4 * 50 + 1 * 20 + 8 * 10 + 20 * 5
3943: 0 * 200 + 1 * 100 + 4 * 50 + 1 * 20 + 9 * 10 + 18 * 5
3944: 0 * 200 + 1 * 100 + 4 * 50 + 1 * 20 + 10 * 10 + 16 * 5
3945: 0 * 200 + 1 * 100 + 4 * 50 + 1 * 20 + 11 * 10 + 14 * 5
3946: 0 * 200 + 1 * 100 + 4 * 50 + 1 * 20 + 12 * 10 + 12 * 5
3947: 0 * 200 + 1 * 100 + 4 * 50 + 1 * 20 + 13 * 10 + 10 * 5
3948: 0 * 200 + 1 * 100 + 4 * 50 + 1 * 20 + 14 * 10 + 8 * 5
3949: 0 * 200 + 1 * 100 + 4 * 50 + 1 * 20 + 15 * 10 + 6 * 5
3950: 0 * 200 + 1 * 100 + 4 * 50 + 1 * 20 + 16 * 10 + 4 * 5
3951: 0 * 200 + 1 * 100 + 4 * 50 + 1 * 20 + 17 * 10 + 2 * 5
3952: 0 * 200 + 1 * 100 + 4 * 50 + 1 * 20 + 18 * 10 + 0 * 5
3953: 0 * 200 + 1 * 100 + 4 * 50 + 2 * 20 + 0 * 10 + 32 * 5
3954: 0 * 200 + 1 * 100 + 4 * 50 + 2 * 20 + 1 * 10 + 30 * 5
3955: 0 * 200 + 1 * 100 + 4 * 50 + 2 * 20 + 2 * 10 + 28 * 5
3956: 0 * 200 + 1 * 100 + 4 * 50 + 2 * 20 + 3 * 10 + 26 * 5
3957: 0 * 200 + 1 * 100 + 4 * 50 + 2 * 20 + 4 * 10 + 24 * 5
3958: 0 * 200 + 1 * 100 + 4 * 50 + 2 * 20 + 5 * 10 + 22 * 5
3959: 0 * 200 + 1 * 100 + 4 * 50 + 2 * 20 + 6 * 10 + 20 * 5
3960: 0 * 200 + 1 * 100 + 4 * 50 + 2 * 20 + 7 * 10 + 18 * 5
3961: 0 * 200 + 1 * 100 + 4 * 50 + 2 * 20 + 8 * 10 + 16 * 5
3962: 0 * 200 + 1 * 100 + 4 * 50 + 2 * 20 + 9 * 10 + 14 * 5
3963: 0 * 200 + 1 * 100 + 4 * 50 + 2 * 20 + 10 * 10 + 12 * 5
3964: 0 * 200 + 1 * 100 + 4 * 50 + 2 * 20 + 11 * 10 + 10 * 5
3965: 0 * 200 + 1 * 100 + 4 * 50 + 2 * 20 + 12 * 10 + 8 * 5
3966: 0 * 200 + 1 * 100 + 4 * 50 + 2 * 20 + 13 * 10 + 6 * 5
3967: 0 * 200 + 1 * 100 + 4 * 50 + 2 * 20 + 14 * 10 + 4 * 5
3968: 0 * 200 + 1 * 100 + 4 * 50 + 2 * 20 + 15 * 10 + 2 * 5
3969: 0 * 200 + 1 * 100 + 4 * 50 + 2 * 20 + 16 * 10 + 0 * 5
3970: 0 * 200 + 1 * 100 + 4 * 50 + 3 * 20 + 0 * 10 + 28 * 5
3971: 0 * 200 + 1 * 100 + 4 * 50 + 3 * 20 + 1 * 10 + 26 * 5
3972: 0 * 200 + 1 * 100 + 4 * 50 + 3 * 20 + 2 * 10 + 24 * 5
3973: 0 * 200 + 1 * 100 + 4 * 50 + 3 * 20 + 3 * 10 + 22 * 5
3974: 0 * 200 + 1 * 100 + 4 * 50 + 3 * 20 + 4 * 10 + 20 * 5
3975: 0 * 200 + 1 * 100 + 4 * 50 + 3 * 20 + 5 * 10 + 18 * 5
3976: 0 * 200 + 1 * 100 + 4 * 50 + 3 * 20 + 6 * 10 + 16 * 5
3977: 0 * 200 + 1 * 100 + 4 * 50 + 3 * 20 + 7 * 10 + 14 * 5
3978: 0 * 200 + 1 * 100 + 4 * 50 + 3 * 20 + 8 * 10 + 12 * 5
3979: 0 * 200 + 1 * 100 + 4 * 50 + 3 * 20 + 9 * 10 + 10 * 5
3980: 0 * 200 + 1 * 100 + 4 * 50 + 3 * 20 + 10 * 10 + 8 * 5
3981: 0 * 200 + 1 * 100 + 4 * 50 + 3 * 20 + 11 * 10 + 6 * 5
3982: 0 * 200 + 1 * 100 + 4 * 50 + 3 * 20 + 12 * 10 + 4 * 5
3983: 0 * 200 + 1 * 100 + 4 * 50 + 3 * 20 + 13 * 10 + 2 * 5
3984: 0 * 200 + 1 * 100 + 4 * 50 + 3 * 20 + 14 * 10 + 0 * 5
3985: 0 * 200 + 1 * 100 + 4 * 50 + 4 * 20 + 0 * 10 + 24 * 5
3986: 0 * 200 + 1 * 100 + 4 * 50 + 4 * 20 + 1 * 10 + 22 * 5
3987: 0 * 200 + 1 * 100 + 4 * 50 + 4 * 20 + 2 * 10 + 20 * 5
3988: 0 * 200 + 1 * 100 + 4 * 50 + 4 * 20 + 3 * 10 + 18 * 5
3989: 0 * 200 + 1 * 100 + 4 * 50 + 4 * 20 + 4 * 10 + 16 * 5
3990: 0 * 200 + 1 * 100 + 4 * 50 + 4 * 20 + 5 * 10 + 14 * 5
3991: 0 * 200 + 1 * 100 + 4 * 50 + 4 * 20 + 6 * 10 + 12 * 5
3992: 0 * 200 + 1 * 100 + 4 * 50 + 4 * 20 + 7 * 10 + 10 * 5
3993: 0 * 200 + 1 * 100 + 4 * 50 + 4 * 20 + 8 * 10 + 8 * 5
3994: 0 * 200 + 1 * 100 + 4 * 50 + 4 * 20 + 9 * 10 + 6 * 5
3995: 0 * 200 + 1 * 100 + 4 * 50 + 4 * 20 + 10 * 10 + 4 * 5
3996: 0 * 200 + 1 * 100 + 4 * 50 + 4 * 20 + 11 * 10 + 2 * 5
3997: 0 * 200 + 1 * 100 + 4 * 50 + 4 * 20 + 12 * 10 + 0 * 5
3998: 0 * 200 + 1 * 100 + 4 * 50 + 5 * 20 + 0 * 10 + 20 * 5
3999: 0 * 200 + 1 * 100 + 4 * 50 + 5 * 20 + 1 * 10 + 18 * 5
4000: 0 * 200 + 1 * 100 + 4 * 50 + 5 * 20 + 2 * 10 + 16 * 5
4001: 0 * 200 + 1 * 100 + 4 * 50 + 5 * 20 + 3 * 10 + 14 * 5
4002: 0 * 200 + 1 * 100 + 4 * 50 + 5 * 20 + 4 * 10 + 12 * 5
4003: 0 * 200 + 1 * 100 + 4 * 50 + 5 * 20 + 5 * 10 + 10 * 5
4004: 0 * 200 + 1 * 100 + 4 * 50 + 5 * 20 + 6 * 10 + 8 * 5
4005: 0 * 200 + 1 * 100 + 4 * 50 + 5 * 20 + 7 * 10 + 6 * 5
4006: 0 * 200 + 1 * 100 + 4 * 50 + 5 * 20 + 8 * 10 + 4 * 5
4007: 0 * 200 + 1 * 100 + 4 * 50 + 5 * 20 + 9 * 10 + 2 * 5
4008: 0 * 200 + 1 * 100 + 4 * 50 + 5 * 20 + 10 * 10 + 0 * 5
4009: 0 * 200 + 1 * 100 + 4 * 50 + 6 * 20 + 0 * 10 + 16 * 5
4010: 0 * 200 + 1 * 100 + 4 * 50 + 6 * 20 + 1 * 10 + 14 * 5
4011: 0 * 200 + 1 * 100 + 4 * 50 + 6 * 20 + 2 * 10 + 12 * 5
4012: 0 * 200 + 1 * 100 + 4 * 50 + 6 * 20 + 3 * 10 + 10 * 5
4013: 0 * 200 + 1 * 100 + 4 * 50 + 6 * 20 + 4 * 10 + 8 * 5
4014: 0 * 200 + 1 * 100 + 4 * 50 + 6 * 20 + 5 * 10 + 6 * 5
4015: 0 * 200 + 1 * 100 + 4 * 50 + 6 * 20 + 6 * 10 + 4 * 5
4016: 0 * 200 + 1 * 100 + 4 * 50 + 6 * 20 + 7 * 10 + 2 * 5
4017: 0 * 200 + 1 * 100 + 4 * 50 + 6 * 20 + 8 * 10 + 0 * 5
4018: 0 * 200 + 1 * 100 + 4 * 50 + 7 * 20 + 0 * 10 + 12 * 5
4019: 0 * 200 + 1 * 100 + 4 * 50 + 7 * 20 + 1 * 10 + 10 * 5
4020: 0 * 200 + 1 * 100 + 4 * 50 + 7 * 20 + 2 * 10 + 8 * 5
4021: 0 * 200 + 1 * 100 + 4 * 50 + 7 * 20 + 3 * 10 + 6 * 5
4022: 0 * 200 + 1 * 100 + 4 * 50 + 7 * 20 + 4 * 10 + 4 * 5
4023: 0 * 200 + 1 * 100 + 4 * 50 + 7 * 20 + 5 * 10 + 2 * 5
4024: 0 * 200 + 1 * 100 + 4 * 50 + 7 * 20 + 6 * 10 + 0 * 5
4025: 0 * 200 + 1 * 100 + 4 * 50 + 8 * 20 + 0 * 10 + 8 * 5
4026: 0 * 200 + 1 * 100 + 4 * 50 + 8 * 20 + 1 * 10 + 6 * 5
4027: 0 * 200 + 1 * 100 + 4 * 50 + 8 * 20 + 2 * 10 + 4 * 5
4028: 0 * 200 + 1 * 100 + 4 * 50 + 8 * 20 + 3 * 10 + 2 * 5
4029: 0 * 200 + 1 * 100 + 4 * 50 + 8 * 20 + 4 * 10 + 0 * 5
4030: 0 * 200 + 1 * 100 + 4 * 50 + 9 * 20 + 0 * 10 + 4 * 5
4031: 0 * 200 + 1 * 100 + 4 * 50 + 9 * 20 + 1 * 10 + 2 * 5
4032: 0 * 200 + 1 * 100 + 4 * 50 + 9 * 20 + 2 * 10 + 0 * 5
4033: 0 * 200 + 1 * 100 + 4 * 50 + 10 * 20 + 0 * 10 + 0 * 5
4034: 0 * 200 + 1 * 100 + 5 * 50 + 0 * 20 + 0 * 10 + 30 * 5
4035: 0 * 200 + 1 * 100 + 5 * 50 + 0 * 20 + 1 * 10 + 28 * 5
4036: 0 * 200 + 1 * 100 + 5 * 50 + 0 * 20 + 2 * 10 + 26 * 5
4037: 0 * 200 + 1 * 100 + 5 * 50 + 0 * 20 + 3 * 10 + 24 * 5
4038: 0 * 200 + 1 * 100 + 5 * 50 + 0 * 20 + 4 * 10 + 22 * 5
4039: 0 * 200 + 1 * 100 + 5 * 50 + 0 * 20 + 5 * 10 + 20 * 5
4040: 0 * 200 + 1 * 100 + 5 * 50 + 0 * 20 + 6 * 10 + 18 * 5
4041: 0 * 200 + 1 * 100 + 5 * 50 + 0 * 20 + 7 * 10 + 16 * 5
4042: 0 * 200 + 1 * 100 + 5 * 50 + 0 * 20 + 8 * 10 + 14 * 5
4043: 0 * 200 + 1 * 100 + 5 * 50 + 0 * 20 + 9 * 10 + 12 * 5
4044: 0 * 200 + 1 * 100 + 5 * 50 + 0 * 20 + 10 * 10 + 10 * 5
4045: 0 * 200 + 1 * 100 + 5 * 50 + 0 * 20 + 11 * 10 + 8 * 5
4046: 0 * 200 + 1 * 100 + 5 * 50 + 0 * 20 + 12 * 10 + 6 * 5
4047: 0 * 200 + 1 * 100 + 5 * 50 + 0 * 20 + 13 * 10 + 4 * 5
4048: 0 * 200 + 1 * 100 + 5 * 50 + 0 * 20 + 14 * 10 + 2 * 5
4049: 0 * 200 + 1 * 100 + 5 * 50 + 0 * 20 + 15 * 10 + 0 * 5
4050: 0 * 200 + 1 * 100 + 5 * 50 + 1 * 20 + 0 * 10 + 26 * 5
4051: 0 * 200 + 1 * 100 + 5 * 50 + 1 * 20 + 1 * 10 + 24 * 5
4052: 0 * 200 + 1 * 100 + 5 * 50 + 1 * 20 + 2 * 10 + 22 * 5
4053: 0 * 200 + 1 * 100 + 5 * 50 + 1 * 20 + 3 * 10 + 20 * 5
4054: 0 * 200 + 1 * 100 + 5 * 50 + 1 * 20 + 4 * 10 + 18 * 5
4055: 0 * 200 + 1 * 100 + 5 * 50 + 1 * 20 + 5 * 10 + 16 * 5
4056: 0 * 200 + 1 * 100 + 5 * 50 + 1 * 20 + 6 * 10 + 14 * 5
4057: 0 * 200 + 1 * 100 + 5 * 50 + 1 * 20 + 7 * 10 + 12 * 5
4058: 0 * 200 + 1 * 100 + 5 * 50 + 1 * 20 + 8 * 10 + 10 * 5
4059: 0 * 200 + 1 * 100 + 5 * 50 + 1 * 20 + 9 * 10 + 8 * 5
4060: 0 * 200 + 1 * 100 + 5 * 50 + 1 * 20 + 10 * 10 + 6 * 5
4061: 0 * 200 + 1 * 100 + 5 * 50 + 1 * 20 + 11 * 10 + 4 * 5
4062: 0 * 200 + 1 * 100 + 5 * 50 + 1 * 20 + 12 * 10 + 2 * 5
4063: 0 * 200 + 1 * 100 + 5 * 50 + 1 * 20 + 13 * 10 + 0 * 5
4064: 0 * 200 + 1 * 100 + 5 * 50 + 2 * 20 + 0 * 10 + 22 * 5
4065: 0 * 200 + 1 * 100 + 5 * 50 + 2 * 20 + 1 * 10 + 20 * 5
4066: 0 * 200 + 1 * 100 + 5 * 50 + 2 * 20 + 2 * 10 + 18 * 5
4067: 0 * 200 + 1 * 100 + 5 * 50 + 2 * 20 + 3 * 10 + 16 * 5
4068: 0 * 200 + 1 * 100 + 5 * 50 + 2 * 20 + 4 * 10 + 14 * 5
4069: 0 * 200 + 1 * 100 + 5 * 50 + 2 * 20 + 5 * 10 + 12 * 5
4070: 0 * 200 + 1 * 100 + 5 * 50 + 2 * 20 + 6 * 10 + 10 * 5
4071: 0 * 200 + 1 * 100 + 5 * 50 + 2 * 20 + 7 * 10 + 8 * 5
4072: 0 * 200 + 1 * 100 + 5 * 50 + 2 * 20 + 8 * 10 + 6 * 5
4073: 0 * 200 + 1 * 100 + 5 * 50 + 2 * 20 + 9 * 10 + 4 * 5
4074: 0 * 200 + 1 * 100 + 5 * 50 + 2 * 20 + 10 * 10 + 2 * 5
4075: 0 * 200 + 1 * 100 + 5 * 50 + 2 * 20 + 11 * 10 + 0 * 5
4076: 0 * 200 + 1 * 100 + 5 * 50 + 3 * 20 + 0 * 10 + 18 * 5
4077: 0 * 200 + 1 * 100 + 5 * 50 + 3 * 20 + 1 * 10 + 16 * 5
4078: 0 * 200 + 1 * 100 + 5 * 50 + 3 * 20 + 2 * 10 + 14 * 5
4079: 0 * 200 + 1 * 100 + 5 * 50 + 3 * 20 + 3 * 10 + 12 * 5
4080: 0 * 200 + 1 * 100 + 5 * 50 + 3 * 20 + 4 * 10 + 10 * 5
4081: 0 * 200 + 1 * 100 + 5 * 50 + 3 * 20 + 5 * 10 + 8 * 5
4082: 0 * 200 + 1 * 100 + 5 * 50 + 3 * 20 + 6 * 10 + 6 * 5
4083: 0 * 200 + 1 * 100 + 5 * 50 + 3 * 20 + 7 * 10 + 4 * 5
4084: 0 * 200 + 1 * 100 + 5 * 50 + 3 * 20 + 8 * 10 + 2 * 5
4085: 0 * 200 + 1 * 100 + 5 * 50 + 3 * 20 + 9 * 10 + 0 * 5
4086: 0 * 200 + 1 * 100 + 5 * 50 + 4 * 20 + 0 * 10 + 14 * 5
4087: 0 * 200 + 1 * 100 + 5 * 50 + 4 * 20 + 1 * 10 + 12 * 5
4088: 0 * 200 + 1 * 100 + 5 * 50 + 4 * 20 + 2 * 10 + 10 * 5
4089: 0 * 200 + 1 * 100 + 5 * 50 + 4 * 20 + 3 * 10 + 8 * 5
4090: 0 * 200 + 1 * 100 + 5 * 50 + 4 * 20 + 4 * 10 + 6 * 5
4091: 0 * 200 + 1 * 100 + 5 * 50 + 4 * 20 + 5 * 10 + 4 * 5
4092: 0 * 200 + 1 * 100 + 5 * 50 + 4 * 20 + 6 * 10 + 2 * 5
4093: 0 * 200 + 1 * 100 + 5 * 50 + 4 * 20 + 7 * 10 + 0 * 5
4094: 0 * 200 + 1 * 100 + 5 * 50 + 5 * 20 + 0 * 10 + 10 * 5
4095: 0 * 200 + 1 * 100 + 5 * 50 + 5 * 20 + 1 * 10 + 8 * 5
4096: 0 * 200 + 1 * 100 + 5 * 50 + 5 * 20 + 2 * 10 + 6 * 5
4097: 0 * 200 + 1 * 100 + 5 * 50 + 5 * 20 + 3 * 10 + 4 * 5
4098: 0 * 200 + 1 * 100 + 5 * 50 + 5 * 20 + 4 * 10 + 2 * 5
4099: 0 * 200 + 1 * 100 + 5 * 50 + 5 * 20 + 5 * 10 + 0 * 5
4100: 0 * 200 + 1 * 100 + 5 * 50 + 6 * 20 + 0 * 10 + 6 * 5
4101: 0 * 200 + 1 * 100 + 5 * 50 + 6 * 20 + 1 * 10 + 4 * 5
4102: 0 * 200 + 1 * 100 + 5 * 50 + 6 * 20 + 2 * 10 + 2 * 5
4103: 0 * 200 + 1 * 100 + 5 * 50 + 6 * 20 + 3 * 10 + 0 * 5
4104: 0 * 200 + 1 * 100 + 5 * 50 + 7 * 20 + 0 * 10 + 2 * 5
4105: 0 * 200 + 1 * 100 + 5 * 50 + 7 * 20 + 1 * 10 + 0 * 5
4106: 0 * 200 + 1 * 100 + 6 * 50 + 0 * 20 + 0 * 10 + 20 * 5
4107: 0 * 200 + 1 * 100 + 6 * 50 + 0 * 20 + 1 * 10 + 18 * 5
4108: 0 * 200 + 1 * 100 + 6 * 50 + 0 * 20 + 2 * 10 + 16 * 5
4109: 0 * 200 + 1 * 100 + 6 * 50 + 0 * 20 + 3 * 10 + 14 * 5
4110: 0 * 200 + 1 * 100 + 6 * 50 + 0 * 20 + 4 * 10 + 12 * 5
4111: 0 * 200 + 1 * 100 + 6 * 50 + 0 * 20 + 5 * 10 + 10 * 5
4112: 0 * 200 + 1 * 100 + 6 * 50 + 0 * 20 + 6 * 10 + 8 * 5
4113: 0 * 200 + 1 * 100 + 6 * 50 + 0 * 20 + 7 * 10 + 6 * 5
4114: 0 * 200 + 1 * 100 + 6 * 50 + 0 * 20 + 8 * 10 + 4 * 5
4115: 0 * 200 + 1 * 100 + 6 * 50 + 0 * 20 + 9 * 10 + 2 * 5
4116: 0 * 200 + 1 * 100 + 6 * 50 + 0 * 20 + 10 * 10 + 0 * 5
4117: 0 * 200 + 1 * 100 + 6 * 50 + 1 * 20 + 0 * 10 + 16 * 5
4118: 0 * 200 + 1 * 100 + 6 * 50 + 1 * 20 + 1 * 10 + 14 * 5
4119: 0 * 200 + 1 * 100 + 6 * 50 + 1 * 20 + 2 * 10 + 12 * 5
4120: 0 * 200 + 1 * 100 + 6 * 50 + 1 * 20 + 3 * 10 + 10 * 5
4121: 0 * 200 + 1 * 100 + 6 * 50 + 1 * 20 + 4 * 10 + 8 * 5
4122: 0 * 200 + 1 * 100 + 6 * 50 + 1 * 20 + 5 * 10 + 6 * 5
4123: 0 * 200 + 1 * 100 + 6 * 50 + 1 * 20 + 6 * 10 + 4 * 5
4124: 0 * 200 + 1 * 100 + 6 * 50 + 1 * 20 + 7 * 10 + 2 * 5
4125: 0 * 200 + 1 * 100 + 6 * 50 + 1 * 20 + 8 * 10 + 0 * 5
4126: 0 * 200 + 1 * 100 + 6 * 50 + 2 * 20 + 0 * 10 + 12 * 5
4127: 0 * 200 + 1 * 100 + 6 * 50 + 2 * 20 + 1 * 10 + 10 * 5
4128: 0 * 200 + 1 * 100 + 6 * 50 + 2 * 20 + 2 * 10 + 8 * 5
4129: 0 * 200 + 1 * 100 + 6 * 50 + 2 * 20 + 3 * 10 + 6 * 5
4130: 0 * 200 + 1 * 100 + 6 * 50 + 2 * 20 + 4 * 10 + 4 * 5
4131: 0 * 200 + 1 * 100 + 6 * 50 + 2 * 20 + 5 * 10 + 2 * 5
4132: 0 * 200 + 1 * 100 + 6 * 50 + 2 * 20 + 6 * 10 + 0 * 5
4133: 0 * 200 + 1 * 100 + 6 * 50 + 3 * 20 + 0 * 10 + 8 * 5
4134: 0 * 200 + 1 * 100 + 6 * 50 + 3 * 20 + 1 * 10 + 6 * 5
4135: 0 * 200 + 1 * 100 + 6 * 50 + 3 * 20 + 2 * 10 + 4 * 5
4136: 0 * 200 + 1 * 100 + 6 * 50 + 3 * 20 + 3 * 10 + 2 * 5
4137: 0 * 200 + 1 * 100 + 6 * 50 + 3 * 20 + 4 * 10 + 0 * 5
4138: 0 * 200 + 1 * 100 + 6 * 50 + 4 * 20 + 0 * 10 + 4 * 5
4139: 0 * 200 + 1 * 100 + 6 * 50 + 4 * 20 + 1 * 10 + 2 * 5
4140: 0 * 200 + 1 * 100 + 6 * 50 + 4 * 20 + 2 * 10 + 0 * 5
4141: 0 * 200 + 1 * 100 + 6 * 50 + 5 * 20 + 0 * 10 + 0 * 5
4142: 0 * 200 + 1 * 100 + 7 * 50 + 0 * 20 + 0 * 10 + 10 * 5
4143: 0 * 200 + 1 * 100 + 7 * 50 + 0 * 20 + 1 * 10 + 8 * 5
4144: 0 * 200 + 1 * 100 + 7 * 50 + 0 * 20 + 2 * 10 + 6 * 5
4145: 0 * 200 + 1 * 100 + 7 * 50 + 0 * 20 + 3 * 10 + 4 * 5
4146: 0 * 200 + 1 * 100 + 7 * 50 + 0 * 20 + 4 * 10 + 2 * 5
4147: 0 * 200 + 1 * 100 + 7 * 50 + 0 * 20 + 5 * 10 + 0 * 5
4148: 0 * 200 + 1 * 100 + 7 * 50 + 1 * 20 + 0 * 10 + 6 * 5
4149: 0 * 200 + 1 * 100 + 7 * 50 + 1 * 20 + 1 * 10 + 4 * 5
4150: 0 * 200 + 1 * 100 + 7 * 50 + 1 * 20 + 2 * 10 + 2 * 5
4151: 0 * 200 + 1 * 100 + 7 * 50 + 1 * 20 + 3 * 10 + 0 * 5
4152: 0 * 200 + 1 * 100 + 7 * 50 + 2 * 20 + 0 * 10 + 2 * 5
4153: 0 * 200 + 1 * 100 + 7 * 50 + 2 * 20 + 1 * 10 + 0 * 5
4154: 0 * 200 + 1 * 100 + 8 * 50 + 0 * 20 + 0 * 10 + 0 * 5
4155: 0 * 200 + 2 * 100 + 0 * 50 + 0 * 20 + 0 * 10 + 60 * 5
4156: 0 * 200 + 2 * 100 + 0 * 50 + 0 * 20 + 1 * 10 + 58 * 5
4157: 0 * 200 + 2 * 100 + 0 * 50 + 0 * 20 + 2 * 10 + 56 * 5
4158: 0 * 200 + 2 * 100 + 0 * 50 + 0 * 20 + 3 * 10 + 54 * 5
4159: 0 * 200 + 2 * 100 + 0 * 50 + 0 * 20 + 4 * 10 + 52 * 5
4160: 0 * 200 + 2 * 100 + 0 * 50 + 0 * 20 + 5 * 10 + 50 * 5
4161: 0 * 200 + 2 * 100 + 0 * 50 + 0 * 20 + 6 * 10 + 48 * 5
4162: 0 * 200 + 2 * 100 + 0 * 50 + 0 * 20 + 7 * 10 + 46 * 5
4163: 0 * 200 + 2 * 100 + 0 * 50 + 0 * 20 + 8 * 10 + 44 * 5
4164: 0 * 200 + 2 * 100 + 0 * 50 + 0 * 20 + 9 * 10 + 42 * 5
4165: 0 * 200 + 2 * 100 + 0 * 50 + 0 * 20 + 10 * 10 + 40 * 5
4166: 0 * 200 + 2 * 100 + 0 * 50 + 0 * 20 + 11 * 10 + 38 * 5
4167: 0 * 200 + 2 * 100 + 0 * 50 + 0 * 20 + 12 * 10 + 36 * 5
4168: 0 * 200 + 2 * 100 + 0 * 50 + 0 * 20 + 13 * 10 + 34 * 5
4169: 0 * 200 + 2 * 100 + 0 * 50 + 0 * 20 + 14 * 10 + 32 * 5
4170: 0 * 200 + 2 * 100 + 0 * 50 + 0 * 20 + 15 * 10 + 30 * 5
4171: 0 * 200 + 2 * 100 + 0 * 50 + 0 * 20 + 16 * 10 + 28 * 5
4172: 0 * 200 + 2 * 100 + 0 * 50 + 0 * 20 + 17 * 10 + 26 * 5
4173: 0 * 200 + 2 * 100 + 0 * 50 + 0 * 20 + 18 * 10 + 24 * 5
4174: 0 * 200 + 2 * 100 + 0 * 50 + 0 * 20 + 19 * 10 + 22 * 5
4175: 0 * 200 + 2 * 100 + 0 * 50 + 0 * 20 + 20 * 10 + 20 * 5
4176: 0 * 200 + 2 * 100 + 0 * 50 + 0 * 20 + 21 * 10 + 18 * 5
4177: 0 * 200 + 2 * 100 + 0 * 50 + 0 * 20 + 22 * 10 + 16 * 5
4178: 0 * 200 + 2 * 100 + 0 * 50 + 0 * 20 + 23 * 10 + 14 * 5
4179: 0 * 200 + 2 * 100 + 0 * 50 + 0 * 20 + 24 * 10 + 12 * 5
4180: 0 * 200 + 2 * 100 + 0 * 50 + 0 * 20 + 25 * 10 + 10 * 5
4181: 0 * 200 + 2 * 100 + 0 * 50 + 0 * 20 + 26 * 10 + 8 * 5
4182: 0 * 200 + 2 * 100 + 0 * 50 + 0 * 20 + 27 * 10 + 6 * 5
4183: 0 * 200 + 2 * 100 + 0 * 50 + 0 * 20 + 28 * 10 + 4 * 5
4184: 0 * 200 + 2 * 100 + 0 * 50 + 0 * 20 + 29 * 10 + 2 * 5
4185: 0 * 200 + 2 * 100 + 0 * 50 + 0 * 20 + 30 * 10 + 0 * 5
4186: 0 * 200 + 2 * 100 + 0 * 50 + 1 * 20 + 0 * 10 + 56 * 5
4187: 0 * 200 + 2 * 100 + 0 * 50 + 1 * 20 + 1 * 10 + 54 * 5
4188: 0 * 200 + 2 * 100 + 0 * 50 + 1 * 20 + 2 * 10 + 52 * 5
4189: 0 * 200 + 2 * 100 + 0 * 50 + 1 * 20 + 3 * 10 + 50 * 5
4190: 0 * 200 + 2 * 100 + 0 * 50 + 1 * 20 + 4 * 10 + 48 * 5
4191: 0 * 200 + 2 * 100 + 0 * 50 + 1 * 20 + 5 * 10 + 46 * 5
4192: 0 * 200 + 2 * 100 + 0 * 50 + 1 * 20 + 6 * 10 + 44 * 5
4193: 0 * 200 + 2 * 100 + 0 * 50 + 1 * 20 + 7 * 10 + 42 * 5
4194: 0 * 200 + 2 * 100 + 0 * 50 + 1 * 20 + 8 * 10 + 40 * 5
4195: 0 * 200 + 2 * 100 + 0 * 50 + 1 * 20 + 9 * 10 + 38 * 5
4196: 0 * 200 + 2 * 100 + 0 * 50 + 1 * 20 + 10 * 10 + 36 * 5
4197: 0 * 200 + 2 * 100 + 0 * 50 + 1 * 20 + 11 * 10 + 34 * 5
4198: 0 * 200 + 2 * 100 + 0 * 50 + 1 * 20 + 12 * 10 + 32 * 5
4199: 0 * 200 + 2 * 100 + 0 * 50 + 1 * 20 + 13 * 10 + 30 * 5
4200: 0 * 200 + 2 * 100 + 0 * 50 + 1 * 20 + 14 * 10 + 28 * 5
4201: 0 * 200 + 2 * 100 + 0 * 50 + 1 * 20 + 15 * 10 + 26 * 5
4202: 0 * 200 + 2 * 100 + 0 * 50 + 1 * 20 + 16 * 10 + 24 * 5
4203: 0 * 200 + 2 * 100 + 0 * 50 + 1 * 20 + 17 * 10 + 22 * 5
4204: 0 * 200 + 2 * 100 + 0 * 50 + 1 * 20 + 18 * 10 + 20 * 5
4205: 0 * 200 + 2 * 100 + 0 * 50 + 1 * 20 + 19 * 10 + 18 * 5
4206: 0 * 200 + 2 * 100 + 0 * 50 + 1 * 20 + 20 * 10 + 16 * 5
4207: 0 * 200 + 2 * 100 + 0 * 50 + 1 * 20 + 21 * 10 + 14 * 5
4208: 0 * 200 + 2 * 100 + 0 * 50 + 1 * 20 + 22 * 10 + 12 * 5
4209: 0 * 200 + 2 * 100 + 0 * 50 + 1 * 20 + 23 * 10 + 10 * 5
4210: 0 * 200 + 2 * 100 + 0 * 50 + 1 * 20 + 24 * 10 + 8 * 5
4211: 0 * 200 + 2 * 100 + 0 * 50 + 1 * 20 + 25 * 10 + 6 * 5
4212: 0 * 200 + 2 * 100 + 0 * 50 + 1 * 20 + 26 * 10 + 4 * 5
4213: 0 * 200 + 2 * 100 + 0 * 50 + 1 * 20 + 27 * 10 + 2 * 5
4214: 0 * 200 + 2 * 100 + 0 * 50 + 1 * 20 + 28 * 10 + 0 * 5
4215: 0 * 200 + 2 * 100 + 0 * 50 + 2 * 20 + 0 * 10 + 52 * 5
4216: 0 * 200 + 2 * 100 + 0 * 50 + 2 * 20 + 1 * 10 + 50 * 5
4217: 0 * 200 + 2 * 100 + 0 * 50 + 2 * 20 + 2 * 10 + 48 * 5
4218: 0 * 200 + 2 * 100 + 0 * 50 + 2 * 20 + 3 * 10 + 46 * 5
4219: 0 * 200 + 2 * 100 + 0 * 50 + 2 * 20 + 4 * 10 + 44 * 5
4220: 0 * 200 + 2 * 100 + 0 * 50 + 2 * 20 + 5 * 10 + 42 * 5
4221: 0 * 200 + 2 * 100 + 0 * 50 + 2 * 20 + 6 * 10 + 40 * 5
4222: 0 * 200 + 2 * 100 + 0 * 50 + 2 * 20 + 7 * 10 + 38 * 5
4223: 0 * 200 + 2 * 100 + 0 * 50 + 2 * 20 + 8 * 10 + 36 * 5
4224: 0 * 200 + 2 * 100 + 0 * 50 + 2 * 20 + 9 * 10 + 34 * 5
4225: 0 * 200 + 2 * 100 + 0 * 50 + 2 * 20 + 10 * 10 + 32 * 5
4226: 0 * 200 + 2 * 100 + 0 * 50 + 2 * 20 + 11 * 10 + 30 * 5
4227: 0 * 200 + 2 * 100 + 0 * 50 + 2 * 20 + 12 * 10 + 28 * 5
4228: 0 * 200 + 2 * 100 + 0 * 50 + 2 * 20 + 13 * 10 + 26 * 5
4229: 0 * 200 + 2 * 100 + 0 * 50 + 2 * 20 + 14 * 10 + 24 * 5
4230: 0 * 200 + 2 * 100 + 0 * 50 + 2 * 20 + 15 * 10 + 22 * 5
4231: 0 * 200 + 2 * 100 + 0 * 50 + 2 * 20 + 16 * 10 + 20 * 5
4232: 0 * 200 + 2 * 100 + 0 * 50 + 2 * 20 + 17 * 10 + 18 * 5
4233: 0 * 200 + 2 * 100 + 0 * 50 + 2 * 20 + 18 * 10 + 16 * 5
4234: 0 * 200 + 2 * 100 + 0 * 50 + 2 * 20 + 19 * 10 + 14 * 5
4235: 0 * 200 + 2 * 100 + 0 * 50 + 2 * 20 + 20 * 10 + 12 * 5
4236: 0 * 200 + 2 * 100 + 0 * 50 + 2 * 20 + 21 * 10 + 10 * 5
4237: 0 * 200 + 2 * 100 + 0 * 50 + 2 * 20 + 22 * 10 + 8 * 5
4238: 0 * 200 + 2 * 100 + 0 * 50 + 2 * 20 + 23 * 10 + 6 * 5
4239: 0 * 200 + 2 * 100 + 0 * 50 + 2 * 20 + 24 * 10 + 4 * 5
4240: 0 * 200 + 2 * 100 + 0 * 50 + 2 * 20 + 25 * 10 + 2 * 5
4241: 0 * 200 + 2 * 100 + 0 * 50 + 2 * 20 + 26 * 10 + 0 * 5
4242: 0 * 200 + 2 * 100 + 0 * 50 + 3 * 20 + 0 * 10 + 48 * 5
4243: 0 * 200 + 2 * 100 + 0 * 50 + 3 * 20 + 1 * 10 + 46 * 5
4244: 0 * 200 + 2 * 100 + 0 * 50 + 3 * 20 + 2 * 10 + 44 * 5
4245: 0 * 200 + 2 * 100 + 0 * 50 + 3 * 20 + 3 * 10 + 42 * 5
4246: 0 * 200 + 2 * 100 + 0 * 50 + 3 * 20 + 4 * 10 + 40 * 5
4247: 0 * 200 + 2 * 100 + 0 * 50 + 3 * 20 + 5 * 10 + 38 * 5
4248: 0 * 200 + 2 * 100 + 0 * 50 + 3 * 20 + 6 * 10 + 36 * 5
4249: 0 * 200 + 2 * 100 + 0 * 50 + 3 * 20 + 7 * 10 + 34 * 5
4250: 0 * 200 + 2 * 100 + 0 * 50 + 3 * 20 + 8 * 10 + 32 * 5
4251: 0 * 200 + 2 * 100 + 0 * 50 + 3 * 20 + 9 * 10 + 30 * 5
4252: 0 * 200 + 2 * 100 + 0 * 50 + 3 * 20 + 10 * 10 + 28 * 5
4253: 0 * 200 + 2 * 100 + 0 * 50 + 3 * 20 + 11 * 10 + 26 * 5
4254: 0 * 200 + 2 * 100 + 0 * 50 + 3 * 20 + 12 * 10 + 24 * 5
4255: 0 * 200 + 2 * 100 + 0 * 50 + 3 * 20 + 13 * 10 + 22 * 5
4256: 0 * 200 + 2 * 100 + 0 * 50 + 3 * 20 + 14 * 10 + 20 * 5
4257: 0 * 200 + 2 * 100 + 0 * 50 + 3 * 20 + 15 * 10 + 18 * 5
4258: 0 * 200 + 2 * 100 + 0 * 50 + 3 * 20 + 16 * 10 + 16 * 5
4259: 0 * 200 + 2 * 100 + 0 * 50 + 3 * 20 + 17 * 10 + 14 * 5
4260: 0 * 200 + 2 * 100 + 0 * 50 + 3 * 20 + 18 * 10 + 12 * 5
4261: 0 * 200 + 2 * 100 + 0 * 50 + 3 * 20 + 19 * 10 + 10 * 5
4262: 0 * 200 + 2 * 100 + 0 * 50 + 3 * 20 + 20 * 10 + 8 * 5
4263: 0 * 200 + 2 * 100 + 0 * 50 + 3 * 20 + 21 * 10 + 6 * 5
4264: 0 * 200 + 2 * 100 + 0 * 50 + 3 * 20 + 22 * 10 + 4 * 5
4265: 0 * 200 + 2 * 100 + 0 * 50 + 3 * 20 + 23 * 10 + 2 * 5
4266: 0 * 200 + 2 * 100 + 0 * 50 + 3 * 20 + 24 * 10 + 0 * 5
4267: 0 * 200 + 2 * 100 + 0 * 50 + 4 * 20 + 0 * 10 + 44 * 5
4268: 0 * 200 + 2 * 100 + 0 * 50 + 4 * 20 + 1 * 10 + 42 * 5
4269: 0 * 200 + 2 * 100 + 0 * 50 + 4 * 20 + 2 * 10 + 40 * 5
4270: 0 * 200 + 2 * 100 + 0 * 50 + 4 * 20 + 3 * 10 + 38 * 5
4271: 0 * 200 + 2 * 100 + 0 * 50 + 4 * 20 + 4 * 10 + 36 * 5
4272: 0 * 200 + 2 * 100 + 0 * 50 + 4 * 20 + 5 * 10 + 34 * 5
4273: 0 * 200 + 2 * 100 + 0 * 50 + 4 * 20 + 6 * 10 + 32 * 5
4274: 0 * 200 + 2 * 100 + 0 * 50 + 4 * 20 + 7 * 10 + 30 * 5
4275: 0 * 200 + 2 * 100 + 0 * 50 + 4 * 20 + 8 * 10 + 28 * 5
4276: 0 * 200 + 2 * 100 + 0 * 50 + 4 * 20 + 9 * 10 + 26 * 5
4277: 0 * 200 + 2 * 100 + 0 * 50 + 4 * 20 + 10 * 10 + 24 * 5
4278: 0 * 200 + 2 * 100 + 0 * 50 + 4 * 20 + 11 * 10 + 22 * 5
4279: 0 * 200 + 2 * 100 + 0 * 50 + 4 * 20 + 12 * 10 + 20 * 5
4280: 0 * 200 + 2 * 100 + 0 * 50 + 4 * 20 + 13 * 10 + 18 * 5
4281: 0 * 200 + 2 * 100 + 0 * 50 + 4 * 20 + 14 * 10 + 16 * 5
4282: 0 * 200 + 2 * 100 + 0 * 50 + 4 * 20 + 15 * 10 + 14 * 5
4283: 0 * 200 + 2 * 100 + 0 * 50 + 4 * 20 + 16 * 10 + 12 * 5
4284: 0 * 200 + 2 * 100 + 0 * 50 + 4 * 20 + 17 * 10 + 10 * 5
4285: 0 * 200 + 2 * 100 + 0 * 50 + 4 * 20 + 18 * 10 + 8 * 5
4286: 0 * 200 + 2 * 100 + 0 * 50 + 4 * 20 + 19 * 10 + 6 * 5
4287: 0 * 200 + 2 * 100 + 0 * 50 + 4 * 20 + 20 * 10 + 4 * 5
4288: 0 * 200 + 2 * 100 + 0 * 50 + 4 * 20 + 21 * 10 + 2 * 5
4289: 0 * 200 + 2 * 100 + 0 * 50 + 4 * 20 + 22 * 10 + 0 * 5
4290: 0 * 200 + 2 * 100 + 0 * 50 + 5 * 20 + 0 * 10 + 40 * 5
4291: 0 * 200 + 2 * 100 + 0 * 50 + 5 * 20 + 1 * 10 + 38 * 5
4292: 0 * 200 + 2 * 100 + 0 * 50 + 5 * 20 + 2 * 10 + 36 * 5
4293: 0 * 200 + 2 * 100 + 0 * 50 + 5 * 20 + 3 * 10 + 34 * 5
4294: 0 * 200 + 2 * 100 + 0 * 50 + 5 * 20 + 4 * 10 + 32 * 5
4295: 0 * 200 + 2 * 100 + 0 * 50 + 5 * 20 + 5 * 10 + 30 * 5
4296: 0 * 200 + 2 * 100 + 0 * 50 + 5 * 20 + 6 * 10 + 28 * 5
4297: 0 * 200 + 2 * 100 + 0 * 50 + 5 * 20 + 7 * 10 + 26 * 5
4298: 0 * 200 + 2 * 100 + 0 * 50 + 5 * 20 + 8 * 10 + 24 * 5
4299: 0 * 200 + 2 * 100 + 0 * 50 + 5 * 20 + 9 * 10 + 22 * 5
4300: 0 * 200 + 2 * 100 + 0 * 50 + 5 * 20 + 10 * 10 + 20 * 5
4301: 0 * 200 + 2 * 100 + 0 * 50 + 5 * 20 + 11 * 10 + 18 * 5
4302: 0 * 200 + 2 * 100 + 0 * 50 + 5 * 20 + 12 * 10 + 16 * 5
4303: 0 * 200 + 2 * 100 + 0 * 50 + 5 * 20 + 13 * 10 + 14 * 5
4304: 0 * 200 + 2 * 100 + 0 * 50 + 5 * 20 + 14 * 10 + 12 * 5
4305: 0 * 200 + 2 * 100 + 0 * 50 + 5 * 20 + 15 * 10 + 10 * 5
4306: 0 * 200 + 2 * 100 + 0 * 50 + 5 * 20 + 16 * 10 + 8 * 5
4307: 0 * 200 + 2 * 100 + 0 * 50 + 5 * 20 + 17 * 10 + 6 * 5
4308: 0 * 200 + 2 * 100 + 0 * 50 + 5 * 20 + 18 * 10 + 4 * 5
4309: 0 * 200 + 2 * 100 + 0 * 50 + 5 * 20 + 19 * 10 + 2 * 5
4310: 0 * 200 + 2 * 100 + 0 * 50 + 5 * 20 + 20 * 10 + 0 * 5
4311: 0 * 200 + 2 * 100 + 0 * 50 + 6 * 20 + 0 * 10 + 36 * 5
4312: 0 * 200 + 2 * 100 + 0 * 50 + 6 * 20 + 1 * 10 + 34 * 5
4313: 0 * 200 + 2 * 100 + 0 * 50 + 6 * 20 + 2 * 10 + 32 * 5
4314: 0 * 200 + 2 * 100 + 0 * 50 + 6 * 20 + 3 * 10 + 30 * 5
4315: 0 * 200 + 2 * 100 + 0 * 50 + 6 * 20 + 4 * 10 + 28 * 5
4316: 0 * 200 + 2 * 100 + 0 * 50 + 6 * 20 + 5 * 10 + 26 * 5
4317: 0 * 200 + 2 * 100 + 0 * 50 + 6 * 20 + 6 * 10 + 24 * 5
4318: 0 * 200 + 2 * 100 + 0 * 50 + 6 * 20 + 7 * 10 + 22 * 5
4319: 0 * 200 + 2 * 100 + 0 * 50 + 6 * 20 + 8 * 10 + 20 * 5
4320: 0 * 200 + 2 * 100 + 0 * 50 + 6 * 20 + 9 * 10 + 18 * 5
4321: 0 * 200 + 2 * 100 + 0 * 50 + 6 * 20 + 10 * 10 + 16 * 5
4322: 0 * 200 + 2 * 100 + 0 * 50 + 6 * 20 + 11 * 10 + 14 * 5
4323: 0 * 200 + 2 * 100 + 0 * 50 + 6 * 20 + 12 * 10 + 12 * 5
4324: 0 * 200 + 2 * 100 + 0 * 50 + 6 * 20 + 13 * 10 + 10 * 5
4325: 0 * 200 + 2 * 100 + 0 * 50 + 6 * 20 + 14 * 10 + 8 * 5
4326: 0 * 200 + 2 * 100 + 0 * 50 + 6 * 20 + 15 * 10 + 6 * 5
4327: 0 * 200 + 2 * 100 + 0 * 50 + 6 * 20 + 16 * 10 + 4 * 5
4328: 0 * 200 + 2 * 100 + 0 * 50 + 6 * 20 + 17 * 10 + 2 * 5
4329: 0 * 200 + 2 * 100 + 0 * 50 + 6 * 20 + 18 * 10 + 0 * 5
4330: 0 * 200 + 2 * 100 + 0 * 50 + 7 * 20 + 0 * 10 + 32 * 5
4331: 0 * 200 + 2 * 100 + 0 * 50 + 7 * 20 + 1 * 10 + 30 * 5
4332: 0 * 200 + 2 * 100 + 0 * 50 + 7 * 20 + 2 * 10 + 28 * 5
4333: 0 * 200 + 2 * 100 + 0 * 50 + 7 * 20 + 3 * 10 + 26 * 5
4334: 0 * 200 + 2 * 100 + 0 * 50 + 7 * 20 + 4 * 10 + 24 * 5
4335: 0 * 200 + 2 * 100 + 0 * 50 + 7 * 20 + 5 * 10 + 22 * 5
4336: 0 * 200 + 2 * 100 + 0 * 50 + 7 * 20 + 6 * 10 + 20 * 5
4337: 0 * 200 + 2 * 100 + 0 * 50 + 7 * 20 + 7 * 10 + 18 * 5
4338: 0 * 200 + 2 * 100 + 0 * 50 + 7 * 20 + 8 * 10 + 16 * 5
4339: 0 * 200 + 2 * 100 + 0 * 50 + 7 * 20 + 9 * 10 + 14 * 5
4340: 0 * 200 + 2 * 100 + 0 * 50 + 7 * 20 + 10 * 10 + 12 * 5
4341: 0 * 200 + 2 * 100 + 0 * 50 + 7 * 20 + 11 * 10 + 10 * 5
4342: 0 * 200 + 2 * 100 + 0 * 50 + 7 * 20 + 12 * 10 + 8 * 5
4343: 0 * 200 + 2 * 100 + 0 * 50 + 7 * 20 + 13 * 10 + 6 * 5
4344: 0 * 200 + 2 * 100 + 0 * 50 + 7 * 20 + 14 * 10 + 4 * 5
4345: 0 * 200 + 2 * 100 + 0 * 50 + 7 * 20 + 15 * 10 + 2 * 5
4346: 0 * 200 + 2 * 100 + 0 * 50 + 7 * 20 + 16 * 10 + 0 * 5
4347: 0 * 200 + 2 * 100 + 0 * 50 + 8 * 20 + 0 * 10 + 28 * 5
4348: 0 * 200 + 2 * 100 + 0 * 50 + 8 * 20 + 1 * 10 + 26 * 5
4349: 0 * 200 + 2 * 100 + 0 * 50 + 8 * 20 + 2 * 10 + 24 * 5
4350: 0 * 200 + 2 * 100 + 0 * 50 + 8 * 20 + 3 * 10 + 22 * 5
4351: 0 * 200 + 2 * 100 + 0 * 50 + 8 * 20 + 4 * 10 + 20 * 5
4352: 0 * 200 + 2 * 100 + 0 * 50 + 8 * 20 + 5 * 10 + 18 * 5
4353: 0 * 200 + 2 * 100 + 0 * 50 + 8 * 20 + 6 * 10 + 16 * 5
4354: 0 * 200 + 2 * 100 + 0 * 50 + 8 * 20 + 7 * 10 + 14 * 5
4355: 0 * 200 + 2 * 100 + 0 * 50 + 8 * 20 + 8 * 10 + 12 * 5
4356: 0 * 200 + 2 * 100 + 0 * 50 + 8 * 20 + 9 * 10 + 10 * 5
4357: 0 * 200 + 2 * 100 + 0 * 50 + 8 * 20 + 10 * 10 + 8 * 5
4358: 0 * 200 + 2 * 100 + 0 * 50 + 8 * 20 + 11 * 10 + 6 * 5
4359: 0 * 200 + 2 * 100 + 0 * 50 + 8 * 20 + 12 * 10 + 4 * 5
4360: 0 * 200 + 2 * 100 + 0 * 50 + 8 * 20 + 13 * 10 + 2 * 5
4361: 0 * 200 + 2 * 100 + 0 * 50 + 8 * 20 + 14 * 10 + 0 * 5
4362: 0 * 200 + 2 * 100 + 0 * 50 + 9 * 20 + 0 * 10 + 24 * 5
4363: 0 * 200 + 2 * 100 + 0 * 50 + 9 * 20 + 1 * 10 + 22 * 5
4364: 0 * 200 + 2 * 100 + 0 * 50 + 9 * 20 + 2 * 10 + 20 * 5
4365: 0 * 200 + 2 * 100 + 0 * 50 + 9 * 20 + 3 * 10 + 18 * 5
4366: 0 * 200 + 2 * 100 + 0 * 50 + 9 * 20 + 4 * 10 + 16 * 5
4367: 0 * 200 + 2 * 100 + 0 * 50 + 9 * 20 + 5 * 10 + 14 * 5
4368: 0 * 200 + 2 * 100 + 0 * 50 + 9 * 20 + 6 * 10 + 12 * 5
4369: 0 * 200 + 2 * 100 + 0 * 50 + 9 * 20 + 7 * 10 + 10 * 5
4370: 0 * 200 + 2 * 100 + 0 * 50 + 9 * 20 + 8 * 10 + 8 * 5
4371: 0 * 200 + 2 * 100 + 0 * 50 + 9 * 20 + 9 * 10 + 6 * 5
4372: 0 * 200 + 2 * 100 + 0 * 50 + 9 * 20 + 10 * 10 + 4 * 5
4373: 0 * 200 + 2 * 100 + 0 * 50 + 9 * 20 + 11 * 10 + 2 * 5
4374: 0 * 200 + 2 * 100 + 0 * 50 + 9 * 20 + 12 * 10 + 0 * 5
4375: 0 * 200 + 2 * 100 + 0 * 50 + 10 * 20 + 0 * 10 + 20 * 5
4376: 0 * 200 + 2 * 100 + 0 * 50 + 10 * 20 + 1 * 10 + 18 * 5
4377: 0 * 200 + 2 * 100 + 0 * 50 + 10 * 20 + 2 * 10 + 16 * 5
4378: 0 * 200 + 2 * 100 + 0 * 50 + 10 * 20 + 3 * 10 + 14 * 5
4379: 0 * 200 + 2 * 100 + 0 * 50 + 10 * 20 + 4 * 10 + 12 * 5
4380: 0 * 200 + 2 * 100 + 0 * 50 + 10 * 20 + 5 * 10 + 10 * 5
4381: 0 * 200 + 2 * 100 + 0 * 50 + 10 * 20 + 6 * 10 + 8 * 5
4382: 0 * 200 + 2 * 100 + 0 * 50 + 10 * 20 + 7 * 10 + 6 * 5
4383: 0 * 200 + 2 * 100 + 0 * 50 + 10 * 20 + 8 * 10 + 4 * 5
4384: 0 * 200 + 2 * 100 + 0 * 50 + 10 * 20 + 9 * 10 + 2 * 5
4385: 0 * 200 + 2 * 100 + 0 * 50 + 10 * 20 + 10 * 10 + 0 * 5
4386: 0 * 200 + 2 * 100 + 0 * 50 + 11 * 20 + 0 * 10 + 16 * 5
4387: 0 * 200 + 2 * 100 + 0 * 50 + 11 * 20 + 1 * 10 + 14 * 5
4388: 0 * 200 + 2 * 100 + 0 * 50 + 11 * 20 + 2 * 10 + 12 * 5
4389: 0 * 200 + 2 * 100 + 0 * 50 + 11 * 20 + 3 * 10 + 10 * 5
4390: 0 * 200 + 2 * 100 + 0 * 50 + 11 * 20 + 4 * 10 + 8 * 5
4391: 0 * 200 + 2 * 100 + 0 * 50 + 11 * 20 + 5 * 10 + 6 * 5
4392: 0 * 200 + 2 * 100 + 0 * 50 + 11 * 20 + 6 * 10 + 4 * 5
4393: 0 * 200 + 2 * 100 + 0 * 50 + 11 * 20 + 7 * 10 + 2 * 5
4394: 0 * 200 + 2 * 100 + 0 * 50 + 11 * 20 + 8 * 10 + 0 * 5
4395: 0 * 200 + 2 * 100 + 0 * 50 + 12 * 20 + 0 * 10 + 12 * 5
4396: 0 * 200 + 2 * 100 + 0 * 50 + 12 * 20 + 1 * 10 + 10 * 5
4397: 0 * 200 + 2 * 100 + 0 * 50 + 12 * 20 + 2 * 10 + 8 * 5
4398: 0 * 200 + 2 * 100 + 0 * 50 + 12 * 20 + 3 * 10 + 6 * 5
4399: 0 * 200 + 2 * 100 + 0 * 50 + 12 * 20 + 4 * 10 + 4 * 5
4400: 0 * 200 + 2 * 100 + 0 * 50 + 12 * 20 + 5 * 10 + 2 * 5
4401: 0 * 200 + 2 * 100 + 0 * 50 + 12 * 20 + 6 * 10 + 0 * 5
4402: 0 * 200 + 2 * 100 + 0 * 50 + 13 * 20 + 0 * 10 + 8 * 5
4403: 0 * 200 + 2 * 100 + 0 * 50 + 13 * 20 + 1 * 10 + 6 * 5
4404: 0 * 200 + 2 * 100 + 0 * 50 + 13 * 20 + 2 * 10 + 4 * 5
4405: 0 * 200 + 2 * 100 + 0 * 50 + 13 * 20 + 3 * 10 + 2 * 5
4406: 0 * 200 + 2 * 100 + 0 * 50 + 13 * 20 + 4 * 10 + 0 * 5
4407: 0 * 200 + 2 * 100 + 0 * 50 + 14 * 20 + 0 * 10 + 4 * 5
4408: 0 * 200 + 2 * 100 + 0 * 50 + 14 * 20 + 1 * 10 + 2 * 5
4409: 0 * 200 + 2 * 100 + 0 * 50 + 14 * 20 + 2 * 10 + 0 * 5
4410: 0 * 200 + 2 * 100 + 0 * 50 + 15 * 20 + 0 * 10 + 0 * 5
4411: 0 * 200 + 2 * 100 + 1 * 50 + 0 * 20 + 0 * 10 + 50 * 5
4412: 0 * 200 + 2 * 100 + 1 * 50 + 0 * 20 + 1 * 10 + 48 * 5
4413: 0 * 200 + 2 * 100 + 1 * 50 + 0 * 20 + 2 * 10 + 46 * 5
4414: 0 * 200 + 2 * 100 + 1 * 50 + 0 * 20 + 3 * 10 + 44 * 5
4415: 0 * 200 + 2 * 100 + 1 * 50 + 0 * 20 + 4 * 10 + 42 * 5
4416: 0 * 200 + 2 * 100 + 1 * 50 + 0 * 20 + 5 * 10 + 40 * 5
4417: 0 * 200 + 2 * 100 + 1 * 50 + 0 * 20 + 6 * 10 + 38 * 5
4418: 0 * 200 + 2 * 100 + 1 * 50 + 0 * 20 + 7 * 10 + 36 * 5
4419: 0 * 200 + 2 * 100 + 1 * 50 + 0 * 20 + 8 * 10 + 34 * 5
4420: 0 * 200 + 2 * 100 + 1 * 50 + 0 * 20 + 9 * 10 + 32 * 5
4421: 0 * 200 + 2 * 100 + 1 * 50 + 0 * 20 + 10 * 10 + 30 * 5
4422: 0 * 200 + 2 * 100 + 1 * 50 + 0 * 20 + 11 * 10 + 28 * 5
4423: 0 * 200 + 2 * 100 + 1 * 50 + 0 * 20 + 12 * 10 + 26 * 5
4424: 0 * 200 + 2 * 100 + 1 * 50 + 0 * 20 + 13 * 10 + 24 * 5
4425: 0 * 200 + 2 * 100 + 1 * 50 + 0 * 20 + 14 * 10 + 22 * 5
4426: 0 * 200 + 2 * 100 + 1 * 50 + 0 * 20 + 15 * 10 + 20 * 5
4427: 0 * 200 + 2 * 100 + 1 * 50 + 0 * 20 + 16 * 10 + 18 * 5
4428: 0 * 200 + 2 * 100 + 1 * 50 + 0 * 20 + 17 * 10 + 16 * 5
4429: 0 * 200 + 2 * 100 + 1 * 50 + 0 * 20 + 18 * 10 + 14 * 5
4430: 0 * 200 + 2 * 100 + 1 * 50 + 0 * 20 + 19 * 10 + 12 * 5
4431: 0 * 200 + 2 * 100 + 1 * 50 + 0 * 20 + 20 * 10 + 10 * 5
4432: 0 * 200 + 2 * 100 + 1 * 50 + 0 * 20 + 21 * 10 + 8 * 5
4433: 0 * 200 + 2 * 100 + 1 * 50 + 0 * 20 + 22 * 10 + 6 * 5
4434: 0 * 200 + 2 * 100 + 1 * 50 + 0 * 20 + 23 * 10 + 4 * 5
4435: 0 * 200 + 2 * 100 + 1 * 50 + 0 * 20 + 24 * 10 + 2 * 5
4436: 0 * 200 + 2 * 100 + 1 * 50 + 0 * 20 + 25 * 10 + 0 * 5
4437: 0 * 200 + 2 * 100 + 1 * 50 + 1 * 20 + 0 * 10 + 46 * 5
4438: 0 * 200 + 2 * 100 + 1 * 50 + 1 * 20 + 1 * 10 + 44 * 5
4439: 0 * 200 + 2 * 100 + 1 * 50 + 1 * 20 + 2 * 10 + 42 * 5
4440: 0 * 200 + 2 * 100 + 1 * 50 + 1 * 20 + 3 * 10 + 40 * 5
4441: 0 * 200 + 2 * 100 + 1 * 50 + 1 * 20 + 4 * 10 + 38 * 5
4442: 0 * 200 + 2 * 100 + 1 * 50 + 1 * 20 + 5 * 10 + 36 * 5
4443: 0 * 200 + 2 * 100 + 1 * 50 + 1 * 20 + 6 * 10 + 34 * 5
4444: 0 * 200 + 2 * 100 + 1 * 50 + 1 * 20 + 7 * 10 + 32 * 5
4445: 0 * 200 + 2 * 100 + 1 * 50 + 1 * 20 + 8 * 10 + 30 * 5
4446: 0 * 200 + 2 * 100 + 1 * 50 + 1 * 20 + 9 * 10 + 28 * 5
4447: 0 * 200 + 2 * 100 + 1 * 50 + 1 * 20 + 10 * 10 + 26 * 5
4448: 0 * 200 + 2 * 100 + 1 * 50 + 1 * 20 + 11 * 10 + 24 * 5
4449: 0 * 200 + 2 * 100 + 1 * 50 + 1 * 20 + 12 * 10 + 22 * 5
4450: 0 * 200 + 2 * 100 + 1 * 50 + 1 * 20 + 13 * 10 + 20 * 5
4451: 0 * 200 + 2 * 100 + 1 * 50 + 1 * 20 + 14 * 10 + 18 * 5
4452: 0 * 200 + 2 * 100 + 1 * 50 + 1 * 20 + 15 * 10 + 16 * 5
4453: 0 * 200 + 2 * 100 + 1 * 50 + 1 * 20 + 16 * 10 + 14 * 5
4454: 0 * 200 + 2 * 100 + 1 * 50 + 1 * 20 + 17 * 10 + 12 * 5
4455: 0 * 200 + 2 * 100 + 1 * 50 + 1 * 20 + 18 * 10 + 10 * 5
4456: 0 * 200 + 2 * 100 + 1 * 50 + 1 * 20 + 19 * 10 + 8 * 5
4457: 0 * 200 + 2 * 100 + 1 * 50 + 1 * 20 + 20 * 10 + 6 * 5
4458: 0 * 200 + 2 * 100 + 1 * 50 + 1 * 20 + 21 * 10 + 4 * 5
4459: 0 * 200 + 2 * 100 + 1 * 50 + 1 * 20 + 22 * 10 + 2 * 5
4460: 0 * 200 + 2 * 100 + 1 * 50 + 1 * 20 + 23 * 10 + 0 * 5
4461: 0 * 200 + 2 * 100 + 1 * 50 + 2 * 20 + 0 * 10 + 42 * 5
4462: 0 * 200 + 2 * 100 + 1 * 50 + 2 * 20 + 1 * 10 + 40 * 5
4463: 0 * 200 + 2 * 100 + 1 * 50 + 2 * 20 + 2 * 10 + 38 * 5
4464: 0 * 200 + 2 * 100 + 1 * 50 + 2 * 20 + 3 * 10 + 36 * 5
4465: 0 * 200 + 2 * 100 + 1 * 50 + 2 * 20 + 4 * 10 + 34 * 5
4466: 0 * 200 + 2 * 100 + 1 * 50 + 2 * 20 + 5 * 10 + 32 * 5
4467: 0 * 200 + 2 * 100 + 1 * 50 + 2 * 20 + 6 * 10 + 30 * 5
4468: 0 * 200 + 2 * 100 + 1 * 50 + 2 * 20 + 7 * 10 + 28 * 5
4469: 0 * 200 + 2 * 100 + 1 * 50 + 2 * 20 + 8 * 10 + 26 * 5
4470: 0 * 200 + 2 * 100 + 1 * 50 + 2 * 20 + 9 * 10 + 24 * 5
4471: 0 * 200 + 2 * 100 + 1 * 50 + 2 * 20 + 10 * 10 + 22 * 5
4472: 0 * 200 + 2 * 100 + 1 * 50 + 2 * 20 + 11 * 10 + 20 * 5
4473: 0 * 200 + 2 * 100 + 1 * 50 + 2 * 20 + 12 * 10 + 18 * 5
4474: 0 * 200 + 2 * 100 + 1 * 50 + 2 * 20 + 13 * 10 + 16 * 5
4475: 0 * 200 + 2 * 100 + 1 * 50 + 2 * 20 + 14 * 10 + 14 * 5
4476: 0 * 200 + 2 * 100 + 1 * 50 + 2 * 20 + 15 * 10 + 12 * 5
4477: 0 * 200 + 2 * 100 + 1 * 50 + 2 * 20 + 16 * 10 + 10 * 5
4478: 0 * 200 + 2 * 100 + 1 * 50 + 2 * 20 + 17 * 10 + 8 * 5
4479: 0 * 200 + 2 * 100 + 1 * 50 + 2 * 20 + 18 * 10 + 6 * 5
4480: 0 * 200 + 2 * 100 + 1 * 50 + 2 * 20 + 19 * 10 + 4 * 5
4481: 0 * 200 + 2 * 100 + 1 * 50 + 2 * 20 + 20 * 10 + 2 * 5
4482: 0 * 200 + 2 * 100 + 1 * 50 + 2 * 20 + 21 * 10 + 0 * 5
4483: 0 * 200 + 2 * 100 + 1 * 50 + 3 * 20 + 0 * 10 + 38 * 5
4484: 0 * 200 + 2 * 100 + 1 * 50 + 3 * 20 + 1 * 10 + 36 * 5
4485: 0 * 200 + 2 * 100 + 1 * 50 + 3 * 20 + 2 * 10 + 34 * 5
4486: 0 * 200 + 2 * 100 + 1 * 50 + 3 * 20 + 3 * 10 + 32 * 5
4487: 0 * 200 + 2 * 100 + 1 * 50 + 3 * 20 + 4 * 10 + 30 * 5
4488: 0 * 200 + 2 * 100 + 1 * 50 + 3 * 20 + 5 * 10 + 28 * 5
4489: 0 * 200 + 2 * 100 + 1 * 50 + 3 * 20 + 6 * 10 + 26 * 5
4490: 0 * 200 + 2 * 100 + 1 * 50 + 3 * 20 + 7 * 10 + 24 * 5
4491: 0 * 200 + 2 * 100 + 1 * 50 + 3 * 20 + 8 * 10 + 22 * 5
4492: 0 * 200 + 2 * 100 + 1 * 50 + 3 * 20 + 9 * 10 + 20 * 5
4493: 0 * 200 + 2 * 100 + 1 * 50 + 3 * 20 + 10 * 10 + 18 * 5
4494: 0 * 200 + 2 * 100 + 1 * 50 + 3 * 20 + 11 * 10 + 16 * 5
4495: 0 * 200 + 2 * 100 + 1 * 50 + 3 * 20 + 12 * 10 + 14 * 5
4496: 0 * 200 + 2 * 100 + 1 * 50 + 3 * 20 + 13 * 10 + 12 * 5
4497: 0 * 200 + 2 * 100 + 1 * 50 + 3 * 20 + 14 * 10 + 10 * 5
4498: 0 * 200 + 2 * 100 + 1 * 50 + 3 * 20 + 15 * 10 + 8 * 5
4499: 0 * 200 + 2 * 100 + 1 * 50 + 3 * 20 + 16 * 10 + 6 * 5
4500: 0 * 200 + 2 * 100 + 1 * 50 + 3 * 20 + 17 * 10 + 4 * 5
4501: 0 * 200 + 2 * 100 + 1 * 50 + 3 * 20 + 18 * 10 + 2 * 5
4502: 0 * 200 + 2 * 100 + 1 * 50 + 3 * 20 + 19 * 10 + 0 * 5
4503: 0 * 200 + 2 * 100 + 1 * 50 + 4 * 20 + 0 * 10 + 34 * 5
4504: 0 * 200 + 2 * 100 + 1 * 50 + 4 * 20 + 1 * 10 + 32 * 5
4505: 0 * 200 + 2 * 100 + 1 * 50 + 4 * 20 + 2 * 10 + 30 * 5
4506: 0 * 200 + 2 * 100 + 1 * 50 + 4 * 20 + 3 * 10 + 28 * 5
4507: 0 * 200 + 2 * 100 + 1 * 50 + 4 * 20 + 4 * 10 + 26 * 5
4508: 0 * 200 + 2 * 100 + 1 * 50 + 4 * 20 + 5 * 10 + 24 * 5
4509: 0 * 200 + 2 * 100 + 1 * 50 + 4 * 20 + 6 * 10 + 22 * 5
4510: 0 * 200 + 2 * 100 + 1 * 50 + 4 * 20 + 7 * 10 + 20 * 5
4511: 0 * 200 + 2 * 100 + 1 * 50 + 4 * 20 + 8 * 10 + 18 * 5
4512: 0 * 200 + 2 * 100 + 1 * 50 + 4 * 20 + 9 * 10 + 16 * 5
4513: 0 * 200 + 2 * 100 + 1 * 50 + 4 * 20 + 10 * 10 + 14 * 5
4514: 0 * 200 + 2 * 100 + 1 * 50 + 4 * 20 + 11 * 10 + 12 * 5
4515: 0 * 200 + 2 * 100 + 1 * 50 + 4 * 20 + 12 * 10 + 10 * 5
4516: 0 * 200 + 2 * 100 + 1 * 50 + 4 * 20 + 13 * 10 + 8 * 5
4517: 0 * 200 + 2 * 100 + 1 * 50 + 4 * 20 + 14 * 10 + 6 * 5
4518: 0 * 200 + 2 * 100 + 1 * 50 + 4 * 20 + 15 * 10 + 4 * 5
4519: 0 * 200 + 2 * 100 + 1 * 50 + 4 * 20 + 16 * 10 + 2 * 5
4520: 0 * 200 + 2 * 100 + 1 * 50 + 4 * 20 + 17 * 10 + 0 * 5
4521: 0 * 200 + 2 * 100 + 1 * 50 + 5 * 20 + 0 * 10 + 30 * 5
4522: 0 * 200 + 2 * 100 + 1 * 50 + 5 * 20 + 1 * 10 + 28 * 5
4523: 0 * 200 + 2 * 100 + 1 * 50 + 5 * 20 + 2 * 10 + 26 * 5
4524: 0 * 200 + 2 * 100 + 1 * 50 + 5 * 20 + 3 * 10 + 24 * 5
4525: 0 * 200 + 2 * 100 + 1 * 50 + 5 * 20 + 4 * 10 + 22 * 5
4526: 0 * 200 + 2 * 100 + 1 * 50 + 5 * 20 + 5 * 10 + 20 * 5
4527: 0 * 200 + 2 * 100 + 1 * 50 + 5 * 20 + 6 * 10 + 18 * 5
4528: 0 * 200 + 2 * 100 + 1 * 50 + 5 * 20 + 7 * 10 + 16 * 5
4529: 0 * 200 + 2 * 100 + 1 * 50 + 5 * 20 + 8 * 10 + 14 * 5
4530: 0 * 200 + 2 * 100 + 1 * 50 + 5 * 20 + 9 * 10 + 12 * 5
4531: 0 * 200 + 2 * 100 + 1 * 50 + 5 * 20 + 10 * 10 + 10 * 5
4532: 0 * 200 + 2 * 100 + 1 * 50 + 5 * 20 + 11 * 10 + 8 * 5
4533: 0 * 200 + 2 * 100 + 1 * 50 + 5 * 20 + 12 * 10 + 6 * 5
4534: 0 * 200 + 2 * 100 + 1 * 50 + 5 * 20 + 13 * 10 + 4 * 5
4535: 0 * 200 + 2 * 100 + 1 * 50 + 5 * 20 + 14 * 10 + 2 * 5
4536: 0 * 200 + 2 * 100 + 1 * 50 + 5 * 20 + 15 * 10 + 0 * 5
4537: 0 * 200 + 2 * 100 + 1 * 50 + 6 * 20 + 0 * 10 + 26 * 5
4538: 0 * 200 + 2 * 100 + 1 * 50 + 6 * 20 + 1 * 10 + 24 * 5
4539: 0 * 200 + 2 * 100 + 1 * 50 + 6 * 20 + 2 * 10 + 22 * 5
4540: 0 * 200 + 2 * 100 + 1 * 50 + 6 * 20 + 3 * 10 + 20 * 5
4541: 0 * 200 + 2 * 100 + 1 * 50 + 6 * 20 + 4 * 10 + 18 * 5
4542: 0 * 200 + 2 * 100 + 1 * 50 + 6 * 20 + 5 * 10 + 16 * 5
4543: 0 * 200 + 2 * 100 + 1 * 50 + 6 * 20 + 6 * 10 + 14 * 5
4544: 0 * 200 + 2 * 100 + 1 * 50 + 6 * 20 + 7 * 10 + 12 * 5
4545: 0 * 200 + 2 * 100 + 1 * 50 + 6 * 20 + 8 * 10 + 10 * 5
4546: 0 * 200 + 2 * 100 + 1 * 50 + 6 * 20 + 9 * 10 + 8 * 5
4547: 0 * 200 + 2 * 100 + 1 * 50 + 6 * 20 + 10 * 10 + 6 * 5
4548: 0 * 200 + 2 * 100 + 1 * 50 + 6 * 20 + 11 * 10 + 4 * 5
4549: 0 * 200 + 2 * 100 + 1 * 50 + 6 * 20 + 12 * 10 + 2 * 5
4550: 0 * 200 + 2 * 100 + 1 * 50 + 6 * 20 + 13 * 10 + 0 * 5
4551: 0 * 200 + 2 * 100 + 1 * 50 + 7 * 20 + 0 * 10 + 22 * 5
4552: 0 * 200 + 2 * 100 + 1 * 50 + 7 * 20 + 1 * 10 + 20 * 5
4553: 0 * 200 + 2 * 100 + 1 * 50 + 7 * 20 + 2 * 10 + 18 * 5
4554: 0 * 200 + 2 * 100 + 1 * 50 + 7 * 20 + 3 * 10 + 16 * 5
4555: 0 * 200 + 2 * 100 + 1 * 50 + 7 * 20 + 4 * 10 + 14 * 5
4556: 0 * 200 + 2 * 100 + 1 * 50 + 7 * 20 + 5 * 10 + 12 * 5
4557: 0 * 200 + 2 * 100 + 1 * 50 + 7 * 20 + 6 * 10 + 10 * 5
4558: 0 * 200 + 2 * 100 + 1 * 50 + 7 * 20 + 7 * 10 + 8 * 5
4559: 0 * 200 + 2 * 100 + 1 * 50 + 7 * 20 + 8 * 10 + 6 * 5
4560: 0 * 200 + 2 * 100 + 1 * 50 + 7 * 20 + 9 * 10 + 4 * 5
4561: 0 * 200 + 2 * 100 + 1 * 50 + 7 * 20 + 10 * 10 + 2 * 5
4562: 0 * 200 + 2 * 100 + 1 * 50 + 7 * 20 + 11 * 10 + 0 * 5
4563: 0 * 200 + 2 * 100 + 1 * 50 + 8 * 20 + 0 * 10 + 18 * 5
4564: 0 * 200 + 2 * 100 + 1 * 50 + 8 * 20 + 1 * 10 + 16 * 5
4565: 0 * 200 + 2 * 100 + 1 * 50 + 8 * 20 + 2 * 10 + 14 * 5
4566: 0 * 200 + 2 * 100 + 1 * 50 + 8 * 20 + 3 * 10 + 12 * 5
4567: 0 * 200 + 2 * 100 + 1 * 50 + 8 * 20 + 4 * 10 + 10 * 5
4568: 0 * 200 + 2 * 100 + 1 * 50 + 8 * 20 + 5 * 10 + 8 * 5
4569: 0 * 200 + 2 * 100 + 1 * 50 + 8 * 20 + 6 * 10 + 6 * 5
4570: 0 * 200 + 2 * 100 + 1 * 50 + 8 * 20 + 7 * 10 + 4 * 5
4571: 0 * 200 + 2 * 100 + 1 * 50 + 8 * 20 + 8 * 10 + 2 * 5
4572: 0 * 200 + 2 * 100 + 1 * 50 + 8 * 20 + 9 * 10 + 0 * 5
4573: 0 * 200 + 2 * 100 + 1 * 50 + 9 * 20 + 0 * 10 + 14 * 5
4574: 0 * 200 + 2 * 100 + 1 * 50 + 9 * 20 + 1 * 10 + 12 * 5
4575: 0 * 200 + 2 * 100 + 1 * 50 + 9 * 20 + 2 * 10 + 10 * 5
4576: 0 * 200 + 2 * 100 + 1 * 50 + 9 * 20 + 3 * 10 + 8 * 5
4577: 0 * 200 + 2 * 100 + 1 * 50 + 9 * 20 + 4 * 10 + 6 * 5
4578: 0 * 200 + 2 * 100 + 1 * 50 + 9 * 20 + 5 * 10 + 4 * 5
4579: 0 * 200 + 2 * 100 + 1 * 50 + 9 * 20 + 6 * 10 + 2 * 5
4580: 0 * 200 + 2 * 100 + 1 * 50 + 9 * 20 + 7 * 10 + 0 * 5
4581: 0 * 200 + 2 * 100 + 1 * 50 + 10 * 20 + 0 * 10 + 10 * 5
4582: 0 * 200 + 2 * 100 + 1 * 50 + 10 * 20 + 1 * 10 + 8 * 5
4583: 0 * 200 + 2 * 100 + 1 * 50 + 10 * 20 + 2 * 10 + 6 * 5
4584: 0 * 200 + 2 * 100 + 1 * 50 + 10 * 20 + 3 * 10 + 4 * 5
4585: 0 * 200 + 2 * 100 + 1 * 50 + 10 * 20 + 4 * 10 + 2 * 5
4586: 0 * 200 + 2 * 100 + 1 * 50 + 10 * 20 + 5 * 10 + 0 * 5
4587: 0 * 200 + 2 * 100 + 1 * 50 + 11 * 20 + 0 * 10 + 6 * 5
4588: 0 * 200 + 2 * 100 + 1 * 50 + 11 * 20 + 1 * 10 + 4 * 5
4589: 0 * 200 + 2 * 100 + 1 * 50 + 11 * 20 + 2 * 10 + 2 * 5
4590: 0 * 200 + 2 * 100 + 1 * 50 + 11 * 20 + 3 * 10 + 0 * 5
4591: 0 * 200 + 2 * 100 + 1 * 50 + 12 * 20 + 0 * 10 + 2 * 5
4592: 0 * 200 + 2 * 100 + 1 * 50 + 12 * 20 + 1 * 10 + 0 * 5
4593: 0 * 200 + 2 * 100 + 2 * 50 + 0 * 20 + 0 * 10 + 40 * 5
4594: 0 * 200 + 2 * 100 + 2 * 50 + 0 * 20 + 1 * 10 + 38 * 5
4595: 0 * 200 + 2 * 100 + 2 * 50 + 0 * 20 + 2 * 10 + 36 * 5
4596: 0 * 200 + 2 * 100 + 2 * 50 + 0 * 20 + 3 * 10 + 34 * 5
4597: 0 * 200 + 2 * 100 + 2 * 50 + 0 * 20 + 4 * 10 + 32 * 5
4598: 0 * 200 + 2 * 100 + 2 * 50 + 0 * 20 + 5 * 10 + 30 * 5
4599: 0 * 200 + 2 * 100 + 2 * 50 + 0 * 20 + 6 * 10 + 28 * 5
4600: 0 * 200 + 2 * 100 + 2 * 50 + 0 * 20 + 7 * 10 + 26 * 5
4601: 0 * 200 + 2 * 100 + 2 * 50 + 0 * 20 + 8 * 10 + 24 * 5
4602: 0 * 200 + 2 * 100 + 2 * 50 + 0 * 20 + 9 * 10 + 22 * 5
4603: 0 * 200 + 2 * 100 + 2 * 50 + 0 * 20 + 10 * 10 + 20 * 5
4604: 0 * 200 + 2 * 100 + 2 * 50 + 0 * 20 + 11 * 10 + 18 * 5
4605: 0 * 200 + 2 * 100 + 2 * 50 + 0 * 20 + 12 * 10 + 16 * 5
4606: 0 * 200 + 2 * 100 + 2 * 50 + 0 * 20 + 13 * 10 + 14 * 5
4607: 0 * 200 + 2 * 100 + 2 * 50 + 0 * 20 + 14 * 10 + 12 * 5
4608: 0 * 200 + 2 * 100 + 2 * 50 + 0 * 20 + 15 * 10 + 10 * 5
4609: 0 * 200 + 2 * 100 + 2 * 50 + 0 * 20 + 16 * 10 + 8 * 5
4610: 0 * 200 + 2 * 100 + 2 * 50 + 0 * 20 + 17 * 10 + 6 * 5
4611: 0 * 200 + 2 * 100 + 2 * 50 + 0 * 20 + 18 * 10 + 4 * 5
4612: 0 * 200 + 2 * 100 + 2 * 50 + 0 * 20 + 19 * 10 + 2 * 5
4613: 0 * 200 + 2 * 100 + 2 * 50 + 0 * 20 + 20 * 10 + 0 * 5
4614: 0 * 200 + 2 * 100 + 2 * 50 + 1 * 20 + 0 * 10 + 36 * 5
4615: 0 * 200 + 2 * 100 + 2 * 50 + 1 * 20 + 1 * 10 + 34 * 5
4616: 0 * 200 + 2 * 100 + 2 * 50 + 1 * 20 + 2 * 10 + 32 * 5
4617: 0 * 200 + 2 * 100 + 2 * 50 + 1 * 20 + 3 * 10 + 30 * 5
4618: 0 * 200 + 2 * 100 + 2 * 50 + 1 * 20 + 4 * 10 + 28 * 5
4619: 0 * 200 + 2 * 100 + 2 * 50 + 1 * 20 + 5 * 10 + 26 * 5
4620: 0 * 200 + 2 * 100 + 2 * 50 + 1 * 20 + 6 * 10 + 24 * 5
4621: 0 * 200 + 2 * 100 + 2 * 50 + 1 * 20 + 7 * 10 + 22 * 5
4622: 0 * 200 + 2 * 100 + 2 * 50 + 1 * 20 + 8 * 10 + 20 * 5
4623: 0 * 200 + 2 * 100 + 2 * 50 + 1 * 20 + 9 * 10 + 18 * 5
4624: 0 * 200 + 2 * 100 + 2 * 50 + 1 * 20 + 10 * 10 + 16 * 5
4625: 0 * 200 + 2 * 100 + 2 * 50 + 1 * 20 + 11 * 10 + 14 * 5
4626: 0 * 200 + 2 * 100 + 2 * 50 + 1 * 20 + 12 * 10 + 12 * 5
4627: 0 * 200 + 2 * 100 + 2 * 50 + 1 * 20 + 13 * 10 + 10 * 5
4628: 0 * 200 + 2 * 100 + 2 * 50 + 1 * 20 + 14 * 10 + 8 * 5
4629: 0 * 200 + 2 * 100 + 2 * 50 + 1 * 20 + 15 * 10 + 6 * 5
4630: 0 * 200 + 2 * 100 + 2 * 50 + 1 * 20 + 16 * 10 + 4 * 5
4631: 0 * 200 + 2 * 100 + 2 * 50 + 1 * 20 + 17 * 10 + 2 * 5
4632: 0 * 200 + 2 * 100 + 2 * 50 + 1 * 20 + 18 * 10 + 0 * 5
4633: 0 * 200 + 2 * 100 + 2 * 50 + 2 * 20 + 0 * 10 + 32 * 5
4634: 0 * 200 + 2 * 100 + 2 * 50 + 2 * 20 + 1 * 10 + 30 * 5
4635: 0 * 200 + 2 * 100 + 2 * 50 + 2 * 20 + 2 * 10 + 28 * 5
4636: 0 * 200 + 2 * 100 + 2 * 50 + 2 * 20 + 3 * 10 + 26 * 5
4637: 0 * 200 + 2 * 100 + 2 * 50 + 2 * 20 + 4 * 10 + 24 * 5
4638: 0 * 200 + 2 * 100 + 2 * 50 + 2 * 20 + 5 * 10 + 22 * 5
4639: 0 * 200 + 2 * 100 + 2 * 50 + 2 * 20 + 6 * 10 + 20 * 5
4640: 0 * 200 + 2 * 100 + 2 * 50 + 2 * 20 + 7 * 10 + 18 * 5
4641: 0 * 200 + 2 * 100 + 2 * 50 + 2 * 20 + 8 * 10 + 16 * 5
4642: 0 * 200 + 2 * 100 + 2 * 50 + 2 * 20 + 9 * 10 + 14 * 5
4643: 0 * 200 + 2 * 100 + 2 * 50 + 2 * 20 + 10 * 10 + 12 * 5
4644: 0 * 200 + 2 * 100 + 2 * 50 + 2 * 20 + 11 * 10 + 10 * 5
4645: 0 * 200 + 2 * 100 + 2 * 50 + 2 * 20 + 12 * 10 + 8 * 5
4646: 0 * 200 + 2 * 100 + 2 * 50 + 2 * 20 + 13 * 10 + 6 * 5
4647: 0 * 200 + 2 * 100 + 2 * 50 + 2 * 20 + 14 * 10 + 4 * 5
4648: 0 * 200 + 2 * 100 + 2 * 50 + 2 * 20 + 15 * 10 + 2 * 5
4649: 0 * 200 + 2 * 100 + 2 * 50 + 2 * 20 + 16 * 10 + 0 * 5
4650: 0 * 200 + 2 * 100 + 2 * 50 + 3 * 20 + 0 * 10 + 28 * 5
4651: 0 * 200 + 2 * 100 + 2 * 50 + 3 * 20 + 1 * 10 + 26 * 5
4652: 0 * 200 + 2 * 100 + 2 * 50 + 3 * 20 + 2 * 10 + 24 * 5
4653: 0 * 200 + 2 * 100 + 2 * 50 + 3 * 20 + 3 * 10 + 22 * 5
4654: 0 * 200 + 2 * 100 + 2 * 50 + 3 * 20 + 4 * 10 + 20 * 5
4655: 0 * 200 + 2 * 100 + 2 * 50 + 3 * 20 + 5 * 10 + 18 * 5
4656: 0 * 200 + 2 * 100 + 2 * 50 + 3 * 20 + 6 * 10 + 16 * 5
4657: 0 * 200 + 2 * 100 + 2 * 50 + 3 * 20 + 7 * 10 + 14 * 5
4658: 0 * 200 + 2 * 100 + 2 * 50 + 3 * 20 + 8 * 10 + 12 * 5
4659: 0 * 200 + 2 * 100 + 2 * 50 + 3 * 20 + 9 * 10 + 10 * 5
4660: 0 * 200 + 2 * 100 + 2 * 50 + 3 * 20 + 10 * 10 + 8 * 5
4661: 0 * 200 + 2 * 100 + 2 * 50 + 3 * 20 + 11 * 10 + 6 * 5
4662: 0 * 200 + 2 * 100 + 2 * 50 + 3 * 20 + 12 * 10 + 4 * 5
4663: 0 * 200 + 2 * 100 + 2 * 50 + 3 * 20 + 13 * 10 + 2 * 5
4664: 0 * 200 + 2 * 100 + 2 * 50 + 3 * 20 + 14 * 10 + 0 * 5
4665: 0 * 200 + 2 * 100 + 2 * 50 + 4 * 20 + 0 * 10 + 24 * 5
4666: 0 * 200 + 2 * 100 + 2 * 50 + 4 * 20 + 1 * 10 + 22 * 5
4667: 0 * 200 + 2 * 100 + 2 * 50 + 4 * 20 + 2 * 10 + 20 * 5
4668: 0 * 200 + 2 * 100 + 2 * 50 + 4 * 20 + 3 * 10 + 18 * 5
4669: 0 * 200 + 2 * 100 + 2 * 50 + 4 * 20 + 4 * 10 + 16 * 5
4670: 0 * 200 + 2 * 100 + 2 * 50 + 4 * 20 + 5 * 10 + 14 * 5
4671: 0 * 200 + 2 * 100 + 2 * 50 + 4 * 20 + 6 * 10 + 12 * 5
4672: 0 * 200 + 2 * 100 + 2 * 50 + 4 * 20 + 7 * 10 + 10 * 5
4673: 0 * 200 + 2 * 100 + 2 * 50 + 4 * 20 + 8 * 10 + 8 * 5
4674: 0 * 200 + 2 * 100 + 2 * 50 + 4 * 20 + 9 * 10 + 6 * 5
4675: 0 * 200 + 2 * 100 + 2 * 50 + 4 * 20 + 10 * 10 + 4 * 5
4676: 0 * 200 + 2 * 100 + 2 * 50 + 4 * 20 + 11 * 10 + 2 * 5
4677: 0 * 200 + 2 * 100 + 2 * 50 + 4 * 20 + 12 * 10 + 0 * 5
4678: 0 * 200 + 2 * 100 + 2 * 50 + 5 * 20 + 0 * 10 + 20 * 5
4679: 0 * 200 + 2 * 100 + 2 * 50 + 5 * 20 + 1 * 10 + 18 * 5
4680: 0 * 200 + 2 * 100 + 2 * 50 + 5 * 20 + 2 * 10 + 16 * 5
4681: 0 * 200 + 2 * 100 + 2 * 50 + 5 * 20 + 3 * 10 + 14 * 5
4682: 0 * 200 + 2 * 100 + 2 * 50 + 5 * 20 + 4 * 10 + 12 * 5
4683: 0 * 200 + 2 * 100 + 2 * 50 + 5 * 20 + 5 * 10 + 10 * 5
4684: 0 * 200 + 2 * 100 + 2 * 50 + 5 * 20 + 6 * 10 + 8 * 5
4685: 0 * 200 + 2 * 100 + 2 * 50 + 5 * 20 + 7 * 10 + 6 * 5
4686: 0 * 200 + 2 * 100 + 2 * 50 + 5 * 20 + 8 * 10 + 4 * 5
4687: 0 * 200 + 2 * 100 + 2 * 50 + 5 * 20 + 9 * 10 + 2 * 5
4688: 0 * 200 + 2 * 100 + 2 * 50 + 5 * 20 + 10 * 10 + 0 * 5
4689: 0 * 200 + 2 * 100 + 2 * 50 + 6 * 20 + 0 * 10 + 16 * 5
4690: 0 * 200 + 2 * 100 + 2 * 50 + 6 * 20 + 1 * 10 + 14 * 5
4691: 0 * 200 + 2 * 100 + 2 * 50 + 6 * 20 + 2 * 10 + 12 * 5
4692: 0 * 200 + 2 * 100 + 2 * 50 + 6 * 20 + 3 * 10 + 10 * 5
4693: 0 * 200 + 2 * 100 + 2 * 50 + 6 * 20 + 4 * 10 + 8 * 5
4694: 0 * 200 + 2 * 100 + 2 * 50 + 6 * 20 + 5 * 10 + 6 * 5
4695: 0 * 200 + 2 * 100 + 2 * 50 + 6 * 20 + 6 * 10 + 4 * 5
4696: 0 * 200 + 2 * 100 + 2 * 50 + 6 * 20 + 7 * 10 + 2 * 5
4697: 0 * 200 + 2 * 100 + 2 * 50 + 6 * 20 + 8 * 10 + 0 * 5
4698: 0 * 200 + 2 * 100 + 2 * 50 + 7 * 20 + 0 * 10 + 12 * 5
4699: 0 * 200 + 2 * 100 + 2 * 50 + 7 * 20 + 1 * 10 + 10 * 5
4700: 0 * 200 + 2 * 100 + 2 * 50 + 7 * 20 + 2 * 10 + 8 * 5
4701: 0 * 200 + 2 * 100 + 2 * 50 + 7 * 20 + 3 * 10 + 6 * 5
4702: 0 * 200 + 2 * 100 + 2 * 50 + 7 * 20 + 4 * 10 + 4 * 5
4703: 0 * 200 + 2 * 100 + 2 * 50 + 7 * 20 + 5 * 10 + 2 * 5
4704: 0 * 200 + 2 * 100 + 2 * 50 + 7 * 20 + 6 * 10 + 0 * 5
4705: 0 * 200 + 2 * 100 + 2 * 50 + 8 * 20 + 0 * 10 + 8 * 5
4706: 0 * 200 + 2 * 100 + 2 * 50 + 8 * 20 + 1 * 10 + 6 * 5
4707: 0 * 200 + 2 * 100 + 2 * 50 + 8 * 20 + 2 * 10 + 4 * 5
4708: 0 * 200 + 2 * 100 + 2 * 50 + 8 * 20 + 3 * 10 + 2 * 5
4709: 0 * 200 + 2 * 100 + 2 * 50 + 8 * 20 + 4 * 10 + 0 * 5
4710: 0 * 200 + 2 * 100 + 2 * 50 + 9 * 20 + 0 * 10 + 4 * 5
4711: 0 * 200 + 2 * 100 + 2 * 50 + 9 * 20 + 1 * 10 + 2 * 5
4712: 0 * 200 + 2 * 100 + 2 * 50 + 9 * 20 + 2 * 10 + 0 * 5
4713: 0 * 200 + 2 * 100 + 2 * 50 + 10 * 20 + 0 * 10 + 0 * 5
4714: 0 * 200 + 2 * 100 + 3 * 50 + 0 * 20 + 0 * 10 + 30 * 5
4715: 0 * 200 + 2 * 100 + 3 * 50 + 0 * 20 + 1 * 10 + 28 * 5
4716: 0 * 200 + 2 * 100 + 3 * 50 + 0 * 20 + 2 * 10 + 26 * 5
4717: 0 * 200 + 2 * 100 + 3 * 50 + 0 * 20 + 3 * 10 + 24 * 5
4718: 0 * 200 + 2 * 100 + 3 * 50 + 0 * 20 + 4 * 10 + 22 * 5
4719: 0 * 200 + 2 * 100 + 3 * 50 + 0 * 20 + 5 * 10 + 20 * 5
4720: 0 * 200 + 2 * 100 + 3 * 50 + 0 * 20 + 6 * 10 + 18 * 5
4721: 0 * 200 + 2 * 100 + 3 * 50 + 0 * 20 + 7 * 10 + 16 * 5
4722: 0 * 200 + 2 * 100 + 3 * 50 + 0 * 20 + 8 * 10 + 14 * 5
4723: 0 * 200 + 2 * 100 + 3 * 50 + 0 * 20 + 9 * 10 + 12 * 5
4724: 0 * 200 + 2 * 100 + 3 * 50 + 0 * 20 + 10 * 10 + 10 * 5
4725: 0 * 200 + 2 * 100 + 3 * 50 + 0 * 20 + 11 * 10 + 8 * 5
4726: 0 * 200 + 2 * 100 + 3 * 50 + 0 * 20 + 12 * 10 + 6 * 5
4727: 0 * 200 + 2 * 100 + 3 * 50 + 0 * 20 + 13 * 10 + 4 * 5
4728: 0 * 200 + 2 * 100 + 3 * 50 + 0 * 20 + 14 * 10 + 2 * 5
4729: 0 * 200 + 2 * 100 + 3 * 50 + 0 * 20 + 15 * 10 + 0 * 5
4730: 0 * 200 + 2 * 100 + 3 * 50 + 1 * 20 + 0 * 10 + 26 * 5
4731: 0 * 200 + 2 * 100 + 3 * 50 + 1 * 20 + 1 * 10 + 24 * 5
4732: 0 * 200 + 2 * 100 + 3 * 50 + 1 * 20 + 2 * 10 + 22 * 5
4733: 0 * 200 + 2 * 100 + 3 * 50 + 1 * 20 + 3 * 10 + 20 * 5
4734: 0 * 200 + 2 * 100 + 3 * 50 + 1 * 20 + 4 * 10 + 18 * 5
4735: 0 * 200 + 2 * 100 + 3 * 50 + 1 * 20 + 5 * 10 + 16 * 5
4736: 0 * 200 + 2 * 100 + 3 * 50 + 1 * 20 + 6 * 10 + 14 * 5
4737: 0 * 200 + 2 * 100 + 3 * 50 + 1 * 20 + 7 * 10 + 12 * 5
4738: 0 * 200 + 2 * 100 + 3 * 50 + 1 * 20 + 8 * 10 + 10 * 5
4739: 0 * 200 + 2 * 100 + 3 * 50 + 1 * 20 + 9 * 10 + 8 * 5
4740: 0 * 200 + 2 * 100 + 3 * 50 + 1 * 20 + 10 * 10 + 6 * 5
4741: 0 * 200 + 2 * 100 + 3 * 50 + 1 * 20 + 11 * 10 + 4 * 5
4742: 0 * 200 + 2 * 100 + 3 * 50 + 1 * 20 + 12 * 10 + 2 * 5
4743: 0 * 200 + 2 * 100 + 3 * 50 + 1 * 20 + 13 * 10 + 0 * 5
4744: 0 * 200 + 2 * 100 + 3 * 50 + 2 * 20 + 0 * 10 + 22 * 5
4745: 0 * 200 + 2 * 100 + 3 * 50 + 2 * 20 + 1 * 10 + 20 * 5
4746: 0 * 200 + 2 * 100 + 3 * 50 + 2 * 20 + 2 * 10 + 18 * 5
4747: 0 * 200 + 2 * 100 + 3 * 50 + 2 * 20 + 3 * 10 + 16 * 5
4748: 0 * 200 + 2 * 100 + 3 * 50 + 2 * 20 + 4 * 10 + 14 * 5
4749: 0 * 200 + 2 * 100 + 3 * 50 + 2 * 20 + 5 * 10 + 12 * 5
4750: 0 * 200 + 2 * 100 + 3 * 50 + 2 * 20 + 6 * 10 + 10 * 5
4751: 0 * 200 + 2 * 100 + 3 * 50 + 2 * 20 + 7 * 10 + 8 * 5
4752: 0 * 200 + 2 * 100 + 3 * 50 + 2 * 20 + 8 * 10 + 6 * 5
4753: 0 * 200 + 2 * 100 + 3 * 50 + 2 * 20 + 9 * 10 + 4 * 5
4754: 0 * 200 + 2 * 100 + 3 * 50 + 2 * 20 + 10 * 10 + 2 * 5
4755: 0 * 200 + 2 * 100 + 3 * 50 + 2 * 20 + 11 * 10 + 0 * 5
4756: 0 * 200 + 2 * 100 + 3 * 50 + 3 * 20 + 0 * 10 + 18 * 5
4757: 0 * 200 + 2 * 100 + 3 * 50 + 3 * 20 + 1 * 10 + 16 * 5
4758: 0 * 200 + 2 * 100 + 3 * 50 + 3 * 20 + 2 * 10 + 14 * 5
4759: 0 * 200 + 2 * 100 + 3 * 50 + 3 * 20 + 3 * 10 + 12 * 5
4760: 0 * 200 + 2 * 100 + 3 * 50 + 3 * 20 + 4 * 10 + 10 * 5
4761: 0 * 200 + 2 * 100 + 3 * 50 + 3 * 20 + 5 * 10 + 8 * 5
4762: 0 * 200 + 2 * 100 + 3 * 50 + 3 * 20 + 6 * 10 + 6 * 5
4763: 0 * 200 + 2 * 100 + 3 * 50 + 3 * 20 + 7 * 10 + 4 * 5
4764: 0 * 200 + 2 * 100 + 3 * 50 + 3 * 20 + 8 * 10 + 2 * 5
4765: 0 * 200 + 2 * 100 + 3 * 50 + 3 * 20 + 9 * 10 + 0 * 5
4766: 0 * 200 + 2 * 100 + 3 * 50 + 4 * 20 + 0 * 10 + 14 * 5
4767: 0 * 200 + 2 * 100 + 3 * 50 + 4 * 20 + 1 * 10 + 12 * 5
4768: 0 * 200 + 2 * 100 + 3 * 50 + 4 * 20 + 2 * 10 + 10 * 5
4769: 0 * 200 + 2 * 100 + 3 * 50 + 4 * 20 + 3 * 10 + 8 * 5
4770: 0 * 200 + 2 * 100 + 3 * 50 + 4 * 20 + 4 * 10 + 6 * 5
4771: 0 * 200 + 2 * 100 + 3 * 50 + 4 * 20 + 5 * 10 + 4 * 5
4772: 0 * 200 + 2 * 100 + 3 * 50 + 4 * 20 + 6 * 10 + 2 * 5
4773: 0 * 200 + 2 * 100 + 3 * 50 + 4 * 20 + 7 * 10 + 0 * 5
4774: 0 * 200 + 2 * 100 + 3 * 50 + 5 * 20 + 0 * 10 + 10 * 5
4775: 0 * 200 + 2 * 100 + 3 * 50 + 5 * 20 + 1 * 10 + 8 * 5
4776: 0 * 200 + 2 * 100 + 3 * 50 + 5 * 20 + 2 * 10 + 6 * 5
4777: 0 * 200 + 2 * 100 + 3 * 50 + 5 * 20 + 3 * 10 + 4 * 5
4778: 0 * 200 + 2 * 100 + 3 * 50 + 5 * 20 + 4 * 10 + 2 * 5
4779: 0 * 200 + 2 * 100 + 3 * 50 + 5 * 20 + 5 * 10 + 0 * 5
4780: 0 * 200 + 2 * 100 + 3 * 50 + 6 * 20 + 0 * 10 + 6 * 5
4781: 0 * 200 + 2 * 100 + 3 * 50 + 6 * 20 + 1 * 10 + 4 * 5
4782: 0 * 200 + 2 * 100 + 3 * 50 + 6 * 20 + 2 * 10 + 2 * 5
4783: 0 * 200 + 2 * 100 + 3 * 50 + 6 * 20 + 3 * 10 + 0 * 5
4784: 0 * 200 + 2 * 100 + 3 * 50 + 7 * 20 + 0 * 10 + 2 * 5
4785: 0 * 200 + 2 * 100 + 3 * 50 + 7 * 20 + 1 * 10 + 0 * 5
4786: 0 * 200 + 2 * 100 + 4 * 50 + 0 * 20 + 0 * 10 + 20 * 5
4787: 0 * 200 + 2 * 100 + 4 * 50 + 0 * 20 + 1 * 10 + 18 * 5
4788: 0 * 200 + 2 * 100 + 4 * 50 + 0 * 20 + 2 * 10 + 16 * 5
4789: 0 * 200 + 2 * 100 + 4 * 50 + 0 * 20 + 3 * 10 + 14 * 5
4790: 0 * 200 + 2 * 100 + 4 * 50 + 0 * 20 + 4 * 10 + 12 * 5
4791: 0 * 200 + 2 * 100 + 4 * 50 + 0 * 20 + 5 * 10 + 10 * 5
4792: 0 * 200 + 2 * 100 + 4 * 50 + 0 * 20 + 6 * 10 + 8 * 5
4793: 0 * 200 + 2 * 100 + 4 * 50 + 0 * 20 + 7 * 10 + 6 * 5
4794: 0 * 200 + 2 * 100 + 4 * 50 + 0 * 20 + 8 * 10 + 4 * 5
4795: 0 * 200 + 2 * 100 + 4 * 50 + 0 * 20 + 9 * 10 + 2 * 5
4796: 0 * 200 + 2 * 100 + 4 * 50 + 0 * 20 + 10 * 10 + 0 * 5
4797: 0 * 200 + 2 * 100 + 4 * 50 + 1 * 20 + 0 * 10 + 16 * 5
4798: 0 * 200 + 2 * 100 + 4 * 50 + 1 * 20 + 1 * 10 + 14 * 5
4799: 0 * 200 + 2 * 100 + 4 * 50 + 1 * 20 + 2 * 10 + 12 * 5
4800: 0 * 200 + 2 * 100 + 4 * 50 + 1 * 20 + 3 * 10 + 10 * 5
4801: 0 * 200 + 2 * 100 + 4 * 50 + 1 * 20 + 4 * 10 + 8 * 5
4802: 0 * 200 + 2 * 100 + 4 * 50 + 1 * 20 + 5 * 10 + 6 * 5
4803: 0 * 200 + 2 * 100 + 4 * 50 + 1 * 20 + 6 * 10 + 4 * 5
4804: 0 * 200 + 2 * 100 + 4 * 50 + 1 * 20 + 7 * 10 + 2 * 5
4805: 0 * 200 + 2 * 100 + 4 * 50 + 1 * 20 + 8 * 10 + 0 * 5
4806: 0 * 200 + 2 * 100 + 4 * 50 + 2 * 20 + 0 * 10 + 12 * 5
4807: 0 * 200 + 2 * 100 + 4 * 50 + 2 * 20 + 1 * 10 + 10 * 5
4808: 0 * 200 + 2 * 100 + 4 * 50 + 2 * 20 + 2 * 10 + 8 * 5
4809: 0 * 200 + 2 * 100 + 4 * 50 + 2 * 20 + 3 * 10 + 6 * 5
4810: 0 * 200 + 2 * 100 + 4 * 50 + 2 * 20 + 4 * 10 + 4 * 5
4811: 0 * 200 + 2 * 100 + 4 * 50 + 2 * 20 + 5 * 10 + 2 * 5
4812: 0 * 200 + 2 * 100 + 4 * 50 + 2 * 20 + 6 * 10 + 0 * 5
4813: 0 * 200 + 2 * 100 + 4 * 50 + 3 * 20 + 0 * 10 + 8 * 5
4814: 0 * 200 + 2 * 100 + 4 * 50 + 3 * 20 + 1 * 10 + 6 * 5
4815: 0 * 200 + 2 * 100 + 4 * 50 + 3 * 20 + 2 * 10 + 4 * 5
4816: 0 * 200 + 2 * 100 + 4 * 50 + 3 * 20 + 3 * 10 + 2 * 5
4817: 0 * 200 + 2 * 100 + 4 * 50 + 3 * 20 + 4 * 10 + 0 * 5
4818: 0 * 200 + 2 * 100 + 4 * 50 + 4 * 20 + 0 * 10 + 4 * 5
4819: 0 * 200 + 2 * 100 + 4 * 50 + 4 * 20 + 1 * 10 + 2 * 5
4820: 0 * 200 + 2 * 100 + 4 * 50 + 4 * 20 + 2 * 10 + 0 * 5
4821: 0 * 200 + 2 * 100 + 4 * 50 + 5 * 20 + 0 * 10 + 0 * 5
4822: 0 * 200 + 2 * 100 + 5 * 50 + 0 * 20 + 0 * 10 + 10 * 5
4823: 0 * 200 + 2 * 100 + 5 * 50 + 0 * 20 + 1 * 10 + 8 * 5
4824: 0 * 200 + 2 * 100 + 5 * 50 + 0 * 20 + 2 * 10 + 6 * 5
4825: 0 * 200 + 2 * 100 + 5 * 50 + 0 * 20 + 3 * 10 + 4 * 5
4826: 0 * 200 + 2 * 100 + 5 * 50 + 0 * 20 + 4 * 10 + 2 * 5
4827: 0 * 200 + 2 * 100 + 5 * 50 + 0 * 20 + 5 * 10 + 0 * 5
4828: 0 * 200 + 2 * 100 + 5 * 50 + 1 * 20 + 0 * 10 + 6 * 5
4829: 0 * 200 + 2 * 100 + 5 * 50 + 1 * 20 + 1 * 10 + 4 * 5
4830: 0 * 200 + 2 * 100 + 5 * 50 + 1 * 20 + 2 * 10 + 2 * 5
4831: 0 * 200 + 2 * 100 + 5 * 50 + 1 * 20 + 3 * 10 + 0 * 5
4832: 0 * 200 + 2 * 100 + 5 * 50 + 2 * 20 + 0 * 10 + 2 * 5
4833: 0 * 200 + 2 * 100 + 5 * 50 + 2 * 20 + 1 * 10 + 0 * 5
4834: 0 * 200 + 2 * 100 + 6 * 50 + 0 * 20 + 0 * 10 + 0 * 5
4835: 0 * 200 + 3 * 100 + 0 * 50 + 0 * 20 + 0 * 10 + 40 * 5
4836: 0 * 200 + 3 * 100 + 0 * 50 + 0 * 20 + 1 * 10 + 38 * 5
4837: 0 * 200 + 3 * 100 + 0 * 50 + 0 * 20 + 2 * 10 + 36 * 5
4838: 0 * 200 + 3 * 100 + 0 * 50 + 0 * 20 + 3 * 10 + 34 * 5
4839: 0 * 200 + 3 * 100 + 0 * 50 + 0 * 20 + 4 * 10 + 32 * 5
4840: 0 * 200 + 3 * 100 + 0 * 50 + 0 * 20 + 5 * 10 + 30 * 5
4841: 0 * 200 + 3 * 100 + 0 * 50 + 0 * 20 + 6 * 10 + 28 * 5
4842: 0 * 200 + 3 * 100 + 0 * 50 + 0 * 20 + 7 * 10 + 26 * 5
4843: 0 * 200 + 3 * 100 + 0 * 50 + 0 * 20 + 8 * 10 + 24 * 5
4844: 0 * 200 + 3 * 100 + 0 * 50 + 0 * 20 + 9 * 10 + 22 * 5
4845: 0 * 200 + 3 * 100 + 0 * 50 + 0 * 20 + 10 * 10 + 20 * 5
4846: 0 * 200 + 3 * 100 + 0 * 50 + 0 * 20 + 11 * 10 + 18 * 5
4847: 0 * 200 + 3 * 100 + 0 * 50 + 0 * 20 + 12 * 10 + 16 * 5
4848: 0 * 200 + 3 * 100 + 0 * 50 + 0 * 20 + 13 * 10 + 14 * 5
4849: 0 * 200 + 3 * 100 + 0 * 50 + 0 * 20 + 14 * 10 + 12 * 5
4850: 0 * 200 + 3 * 100 + 0 * 50 + 0 * 20 + 15 * 10 + 10 * 5
4851: 0 * 200 + 3 * 100 + 0 * 50 + 0 * 20 + 16 * 10 + 8 * 5
4852: 0 * 200 + 3 * 100 + 0 * 50 + 0 * 20 + 17 * 10 + 6 * 5
4853: 0 * 200 + 3 * 100 + 0 * 50 + 0 * 20 + 18 * 10 + 4 * 5
4854: 0 * 200 + 3 * 100 + 0 * 50 + 0 * 20 + 19 * 10 + 2 * 5
4855: 0 * 200 + 3 * 100 + 0 * 50 + 0 * 20 + 20 * 10 + 0 * 5
4856: 0 * 200 + 3 * 100 + 0 * 50 + 1 * 20 + 0 * 10 + 36 * 5
4857: 0 * 200 + 3 * 100 + 0 * 50 + 1 * 20 + 1 * 10 + 34 * 5
4858: 0 * 200 + 3 * 100 + 0 * 50 + 1 * 20 + 2 * 10 + 32 * 5
4859: 0 * 200 + 3 * 100 + 0 * 50 + 1 * 20 + 3 * 10 + 30 * 5
4860: 0 * 200 + 3 * 100 + 0 * 50 + 1 * 20 + 4 * 10 + 28 * 5
4861: 0 * 200 + 3 * 100 + 0 * 50 + 1 * 20 + 5 * 10 + 26 * 5
4862: 0 * 200 + 3 * 100 + 0 * 50 + 1 * 20 + 6 * 10 + 24 * 5
4863: 0 * 200 + 3 * 100 + 0 * 50 + 1 * 20 + 7 * 10 + 22 * 5
4864: 0 * 200 + 3 * 100 + 0 * 50 + 1 * 20 + 8 * 10 + 20 * 5
4865: 0 * 200 + 3 * 100 + 0 * 50 + 1 * 20 + 9 * 10 + 18 * 5
4866: 0 * 200 + 3 * 100 + 0 * 50 + 1 * 20 + 10 * 10 + 16 * 5
4867: 0 * 200 + 3 * 100 + 0 * 50 + 1 * 20 + 11 * 10 + 14 * 5
4868: 0 * 200 + 3 * 100 + 0 * 50 + 1 * 20 + 12 * 10 + 12 * 5
4869: 0 * 200 + 3 * 100 + 0 * 50 + 1 * 20 + 13 * 10 + 10 * 5
4870: 0 * 200 + 3 * 100 + 0 * 50 + 1 * 20 + 14 * 10 + 8 * 5
4871: 0 * 200 + 3 * 100 + 0 * 50 + 1 * 20 + 15 * 10 + 6 * 5
4872: 0 * 200 + 3 * 100 + 0 * 50 + 1 * 20 + 16 * 10 + 4 * 5
4873: 0 * 200 + 3 * 100 + 0 * 50 + 1 * 20 + 17 * 10 + 2 * 5
4874: 0 * 200 + 3 * 100 + 0 * 50 + 1 * 20 + 18 * 10 + 0 * 5
4875: 0 * 200 + 3 * 100 + 0 * 50 + 2 * 20 + 0 * 10 + 32 * 5
4876: 0 * 200 + 3 * 100 + 0 * 50 + 2 * 20 + 1 * 10 + 30 * 5
4877: 0 * 200 + 3 * 100 + 0 * 50 + 2 * 20 + 2 * 10 + 28 * 5
4878: 0 * 200 + 3 * 100 + 0 * 50 + 2 * 20 + 3 * 10 + 26 * 5
4879: 0 * 200 + 3 * 100 + 0 * 50 + 2 * 20 + 4 * 10 + 24 * 5
4880: 0 * 200 + 3 * 100 + 0 * 50 + 2 * 20 + 5 * 10 + 22 * 5
4881: 0 * 200 + 3 * 100 + 0 * 50 + 2 * 20 + 6 * 10 + 20 * 5
4882: 0 * 200 + 3 * 100 + 0 * 50 + 2 * 20 + 7 * 10 + 18 * 5
4883: 0 * 200 + 3 * 100 + 0 * 50 + 2 * 20 + 8 * 10 + 16 * 5
4884: 0 * 200 + 3 * 100 + 0 * 50 + 2 * 20 + 9 * 10 + 14 * 5
4885: 0 * 200 + 3 * 100 + 0 * 50 + 2 * 20 + 10 * 10 + 12 * 5
4886: 0 * 200 + 3 * 100 + 0 * 50 + 2 * 20 + 11 * 10 + 10 * 5
4887: 0 * 200 + 3 * 100 + 0 * 50 + 2 * 20 + 12 * 10 + 8 * 5
4888: 0 * 200 + 3 * 100 + 0 * 50 + 2 * 20 + 13 * 10 + 6 * 5
4889: 0 * 200 + 3 * 100 + 0 * 50 + 2 * 20 + 14 * 10 + 4 * 5
4890: 0 * 200 + 3 * 100 + 0 * 50 + 2 * 20 + 15 * 10 + 2 * 5
4891: 0 * 200 + 3 * 100 + 0 * 50 + 2 * 20 + 16 * 10 + 0 * 5
4892: 0 * 200 + 3 * 100 + 0 * 50 + 3 * 20 + 0 * 10 + 28 * 5
4893: 0 * 200 + 3 * 100 + 0 * 50 + 3 * 20 + 1 * 10 + 26 * 5
4894: 0 * 200 + 3 * 100 + 0 * 50 + 3 * 20 + 2 * 10 + 24 * 5
4895: 0 * 200 + 3 * 100 + 0 * 50 + 3 * 20 + 3 * 10 + 22 * 5
4896: 0 * 200 + 3 * 100 + 0 * 50 + 3 * 20 + 4 * 10 + 20 * 5
4897: 0 * 200 + 3 * 100 + 0 * 50 + 3 * 20 + 5 * 10 + 18 * 5
4898: 0 * 200 + 3 * 100 + 0 * 50 + 3 * 20 + 6 * 10 + 16 * 5
4899: 0 * 200 + 3 * 100 + 0 * 50 + 3 * 20 + 7 * 10 + 14 * 5
4900: 0 * 200 + 3 * 100 + 0 * 50 + 3 * 20 + 8 * 10 + 12 * 5
4901: 0 * 200 + 3 * 100 + 0 * 50 + 3 * 20 + 9 * 10 + 10 * 5
4902: 0 * 200 + 3 * 100 + 0 * 50 + 3 * 20 + 10 * 10 + 8 * 5
4903: 0 * 200 + 3 * 100 + 0 * 50 + 3 * 20 + 11 * 10 + 6 * 5
4904: 0 * 200 + 3 * 100 + 0 * 50 + 3 * 20 + 12 * 10 + 4 * 5
4905: 0 * 200 + 3 * 100 + 0 * 50 + 3 * 20 + 13 * 10 + 2 * 5
4906: 0 * 200 + 3 * 100 + 0 * 50 + 3 * 20 + 14 * 10 + 0 * 5
4907: 0 * 200 + 3 * 100 + 0 * 50 + 4 * 20 + 0 * 10 + 24 * 5
4908: 0 * 200 + 3 * 100 + 0 * 50 + 4 * 20 + 1 * 10 + 22 * 5
4909: 0 * 200 + 3 * 100 + 0 * 50 + 4 * 20 + 2 * 10 + 20 * 5
4910: 0 * 200 + 3 * 100 + 0 * 50 + 4 * 20 + 3 * 10 + 18 * 5
4911: 0 * 200 + 3 * 100 + 0 * 50 + 4 * 20 + 4 * 10 + 16 * 5
4912: 0 * 200 + 3 * 100 + 0 * 50 + 4 * 20 + 5 * 10 + 14 * 5
4913: 0 * 200 + 3 * 100 + 0 * 50 + 4 * 20 + 6 * 10 + 12 * 5
4914: 0 * 200 + 3 * 100 + 0 * 50 + 4 * 20 + 7 * 10 + 10 * 5
4915: 0 * 200 + 3 * 100 + 0 * 50 + 4 * 20 + 8 * 10 + 8 * 5
4916: 0 * 200 + 3 * 100 + 0 * 50 + 4 * 20 + 9 * 10 + 6 * 5
4917: 0 * 200 + 3 * 100 + 0 * 50 + 4 * 20 + 10 * 10 + 4 * 5
4918: 0 * 200 + 3 * 100 + 0 * 50 + 4 * 20 + 11 * 10 + 2 * 5
4919: 0 * 200 + 3 * 100 + 0 * 50 + 4 * 20 + 12 * 10 + 0 * 5
4920: 0 * 200 + 3 * 100 + 0 * 50 + 5 * 20 + 0 * 10 + 20 * 5
4921: 0 * 200 + 3 * 100 + 0 * 50 + 5 * 20 + 1 * 10 + 18 * 5
4922: 0 * 200 + 3 * 100 + 0 * 50 + 5 * 20 + 2 * 10 + 16 * 5
4923: 0 * 200 + 3 * 100 + 0 * 50 + 5 * 20 + 3 * 10 + 14 * 5
4924: 0 * 200 + 3 * 100 + 0 * 50 + 5 * 20 + 4 * 10 + 12 * 5
4925: 0 * 200 + 3 * 100 + 0 * 50 + 5 * 20 + 5 * 10 + 10 * 5
4926: 0 * 200 + 3 * 100 + 0 * 50 + 5 * 20 + 6 * 10 + 8 * 5
4927: 0 * 200 + 3 * 100 + 0 * 50 + 5 * 20 + 7 * 10 + 6 * 5
4928: 0 * 200 + 3 * 100 + 0 * 50 + 5 * 20 + 8 * 10 + 4 * 5
4929: 0 * 200 + 3 * 100 + 0 * 50 + 5 * 20 + 9 * 10 + 2 * 5
4930: 0 * 200 + 3 * 100 + 0 * 50 + 5 * 20 + 10 * 10 + 0 * 5
4931: 0 * 200 + 3 * 100 + 0 * 50 + 6 * 20 + 0 * 10 + 16 * 5
4932: 0 * 200 + 3 * 100 + 0 * 50 + 6 * 20 + 1 * 10 + 14 * 5
4933: 0 * 200 + 3 * 100 + 0 * 50 + 6 * 20 + 2 * 10 + 12 * 5
4934: 0 * 200 + 3 * 100 + 0 * 50 + 6 * 20 + 3 * 10 + 10 * 5
4935: 0 * 200 + 3 * 100 + 0 * 50 + 6 * 20 + 4 * 10 + 8 * 5
4936: 0 * 200 + 3 * 100 + 0 * 50 + 6 * 20 + 5 * 10 + 6 * 5
4937: 0 * 200 + 3 * 100 + 0 * 50 + 6 * 20 + 6 * 10 + 4 * 5
4938: 0 * 200 + 3 * 100 + 0 * 50 + 6 * 20 + 7 * 10 + 2 * 5
4939: 0 * 200 + 3 * 100 + 0 * 50 + 6 * 20 + 8 * 10 + 0 * 5
4940: 0 * 200 + 3 * 100 + 0 * 50 + 7 * 20 + 0 * 10 + 12 * 5
4941: 0 * 200 + 3 * 100 + 0 * 50 + 7 * 20 + 1 * 10 + 10 * 5
4942: 0 * 200 + 3 * 100 + 0 * 50 + 7 * 20 + 2 * 10 + 8 * 5
4943: 0 * 200 + 3 * 100 + 0 * 50 + 7 * 20 + 3 * 10 + 6 * 5
4944: 0 * 200 + 3 * 100 + 0 * 50 + 7 * 20 + 4 * 10 + 4 * 5
4945: 0 * 200 + 3 * 100 + 0 * 50 + 7 * 20 + 5 * 10 + 2 * 5
4946: 0 * 200 + 3 * 100 + 0 * 50 + 7 * 20 + 6 * 10 + 0 * 5
4947: 0 * 200 + 3 * 100 + 0 * 50 + 8 * 20 + 0 * 10 + 8 * 5
4948: 0 * 200 + 3 * 100 + 0 * 50 + 8 * 20 + 1 * 10 + 6 * 5
4949: 0 * 200 + 3 * 100 + 0 * 50 + 8 * 20 + 2 * 10 + 4 * 5
4950: 0 * 200 + 3 * 100 + 0 * 50 + 8 * 20 + 3 * 10 + 2 * 5
4951: 0 * 200 + 3 * 100 + 0 * 50 + 8 * 20 + 4 * 10 + 0 * 5
4952: 0 * 200 + 3 * 100 + 0 * 50 + 9 * 20 + 0 * 10 + 4 * 5
4953: 0 * 200 + 3 * 100 + 0 * 50 + 9 * 20 + 1 * 10 + 2 * 5
4954: 0 * 200 + 3 * 100 + 0 * 50 + 9 * 20 + 2 * 10 + 0 * 5
4955: 0 * 200 + 3 * 100 + 0 * 50 + 10 * 20 + 0 * 10 + 0 * 5
4956: 0 * 200 + 3 * 100 + 1 * 50 + 0 * 20 + 0 * 10 + 30 * 5
4957: 0 * 200 + 3 * 100 + 1 * 50 + 0 * 20 + 1 * 10 + 28 * 5
4958: 0 * 200 + 3 * 100 + 1 * 50 + 0 * 20 + 2 * 10 + 26 * 5
4959: 0 * 200 + 3 * 100 + 1 * 50 + 0 * 20 + 3 * 10 + 24 * 5
4960: 0 * 200 + 3 * 100 + 1 * 50 + 0 * 20 + 4 * 10 + 22 * 5
4961: 0 * 200 + 3 * 100 + 1 * 50 + 0 * 20 + 5 * 10 + 20 * 5
4962: 0 * 200 + 3 * 100 + 1 * 50 + 0 * 20 + 6 * 10 + 18 * 5
4963: 0 * 200 + 3 * 100 + 1 * 50 + 0 * 20 + 7 * 10 + 16 * 5
4964: 0 * 200 + 3 * 100 + 1 * 50 + 0 * 20 + 8 * 10 + 14 * 5
4965: 0 * 200 + 3 * 100 + 1 * 50 + 0 * 20 + 9 * 10 + 12 * 5
4966: 0 * 200 + 3 * 100 + 1 * 50 + 0 * 20 + 10 * 10 + 10 * 5
4967: 0 * 200 + 3 * 100 + 1 * 50 + 0 * 20 + 11 * 10 + 8 * 5
4968: 0 * 200 + 3 * 100 + 1 * 50 + 0 * 20 + 12 * 10 + 6 * 5
4969: 0 * 200 + 3 * 100 + 1 * 50 + 0 * 20 + 13 * 10 + 4 * 5
4970: 0 * 200 + 3 * 100 + 1 * 50 + 0 * 20 + 14 * 10 + 2 * 5
4971: 0 * 200 + 3 * 100 + 1 * 50 + 0 * 20 + 15 * 10 + 0 * 5
4972: 0 * 200 + 3 * 100 + 1 * 50 + 1 * 20 + 0 * 10 + 26 * 5
4973: 0 * 200 + 3 * 100 + 1 * 50 + 1 * 20 + 1 * 10 + 24 * 5
4974: 0 * 200 + 3 * 100 + 1 * 50 + 1 * 20 + 2 * 10 + 22 * 5
4975: 0 * 200 + 3 * 100 + 1 * 50 + 1 * 20 + 3 * 10 + 20 * 5
4976: 0 * 200 + 3 * 100 + 1 * 50 + 1 * 20 + 4 * 10 + 18 * 5
4977: 0 * 200 + 3 * 100 + 1 * 50 + 1 * 20 + 5 * 10 + 16 * 5
4978: 0 * 200 + 3 * 100 + 1 * 50 + 1 * 20 + 6 * 10 + 14 * 5
4979: 0 * 200 + 3 * 100 + 1 * 50 + 1 * 20 + 7 * 10 + 12 * 5
4980: 0 * 200 + 3 * 100 + 1 * 50 + 1 * 20 + 8 * 10 + 10 * 5
4981: 0 * 200 + 3 * 100 + 1 * 50 + 1 * 20 + 9 * 10 + 8 * 5
4982: 0 * 200 + 3 * 100 + 1 * 50 + 1 * 20 + 10 * 10 + 6 * 5
4983: 0 * 200 + 3 * 100 + 1 * 50 + 1 * 20 + 11 * 10 + 4 * 5
4984: 0 * 200 + 3 * 100 + 1 * 50 + 1 * 20 + 12 * 10 + 2 * 5
4985: 0 * 200 + 3 * 100 + 1 * 50 + 1 * 20 + 13 * 10 + 0 * 5
4986: 0 * 200 + 3 * 100 + 1 * 50 + 2 * 20 + 0 * 10 + 22 * 5
4987: 0 * 200 + 3 * 100 + 1 * 50 + 2 * 20 + 1 * 10 + 20 * 5
4988: 0 * 200 + 3 * 100 + 1 * 50 + 2 * 20 + 2 * 10 + 18 * 5
4989: 0 * 200 + 3 * 100 + 1 * 50 + 2 * 20 + 3 * 10 + 16 * 5
4990: 0 * 200 + 3 * 100 + 1 * 50 + 2 * 20 + 4 * 10 + 14 * 5
4991: 0 * 200 + 3 * 100 + 1 * 50 + 2 * 20 + 5 * 10 + 12 * 5
4992: 0 * 200 + 3 * 100 + 1 * 50 + 2 * 20 + 6 * 10 + 10 * 5
4993: 0 * 200 + 3 * 100 + 1 * 50 + 2 * 20 + 7 * 10 + 8 * 5
4994: 0 * 200 + 3 * 100 + 1 * 50 + 2 * 20 + 8 * 10 + 6 * 5
4995: 0 * 200 + 3 * 100 + 1 * 50 + 2 * 20 + 9 * 10 + 4 * 5
4996: 0 * 200 + 3 * 100 + 1 * 50 + 2 * 20 + 10 * 10 + 2 * 5
4997: 0 * 200 + 3 * 100 + 1 * 50 + 2 * 20 + 11 * 10 + 0 * 5
4998: 0 * 200 + 3 * 100 + 1 * 50 + 3 * 20 + 0 * 10 + 18 * 5
4999: 0 * 200 + 3 * 100 + 1 * 50 + 3 * 20 + 1 * 10 + 16 * 5
5000: 0 * 200 + 3 * 100 + 1 * 50 + 3 * 20 + 2 * 10 + 14 * 5
5001: 0 * 200 + 3 * 100 + 1 * 50 + 3 * 20 + 3 * 10 + 12 * 5
5002: 0 * 200 + 3 * 100 + 1 * 50 + 3 * 20 + 4 * 10 + 10 * 5
5003: 0 * 200 + 3 * 100 + 1 * 50 + 3 * 20 + 5 * 10 + 8 * 5
5004: 0 * 200 + 3 * 100 + 1 * 50 + 3 * 20 + 6 * 10 + 6 * 5
5005: 0 * 200 + 3 * 100 + 1 * 50 + 3 * 20 + 7 * 10 + 4 * 5
5006: 0 * 200 + 3 * 100 + 1 * 50 + 3 * 20 + 8 * 10 + 2 * 5
5007: 0 * 200 + 3 * 100 + 1 * 50 + 3 * 20 + 9 * 10 + 0 * 5
5008: 0 * 200 + 3 * 100 + 1 * 50 + 4 * 20 + 0 * 10 + 14 * 5
5009: 0 * 200 + 3 * 100 + 1 * 50 + 4 * 20 + 1 * 10 + 12 * 5
5010: 0 * 200 + 3 * 100 + 1 * 50 + 4 * 20 + 2 * 10 + 10 * 5
5011: 0 * 200 + 3 * 100 + 1 * 50 + 4 * 20 + 3 * 10 + 8 * 5
5012: 0 * 200 + 3 * 100 + 1 * 50 + 4 * 20 + 4 * 10 + 6 * 5
5013: 0 * 200 + 3 * 100 + 1 * 50 + 4 * 20 + 5 * 10 + 4 * 5
5014: 0 * 200 + 3 * 100 + 1 * 50 + 4 * 20 + 6 * 10 + 2 * 5
5015: 0 * 200 + 3 * 100 + 1 * 50 + 4 * 20 + 7 * 10 + 0 * 5
5016: 0 * 200 + 3 * 100 + 1 * 50 + 5 * 20 + 0 * 10 + 10 * 5
5017: 0 * 200 + 3 * 100 + 1 * 50 + 5 * 20 + 1 * 10 + 8 * 5
5018: 0 * 200 + 3 * 100 + 1 * 50 + 5 * 20 + 2 * 10 + 6 * 5
5019: 0 * 200 + 3 * 100 + 1 * 50 + 5 * 20 + 3 * 10 + 4 * 5
5020: 0 * 200 + 3 * 100 + 1 * 50 + 5 * 20 + 4 * 10 + 2 * 5
5021: 0 * 200 + 3 * 100 + 1 * 50 + 5 * 20 + 5 * 10 + 0 * 5
5022: 0 * 200 + 3 * 100 + 1 * 50 + 6 * 20 + 0 * 10 + 6 * 5
5023: 0 * 200 + 3 * 100 + 1 * 50 + 6 * 20 + 1 * 10 + 4 * 5
5024: 0 * 200 + 3 * 100 + 1 * 50 + 6 * 20 + 2 * 10 + 2 * 5
5025: 0 * 200 + 3 * 100 + 1 * 50 + 6 * 20 + 3 * 10 + 0 * 5
5026: 0 * 200 + 3 * 100 + 1 * 50 + 7 * 20 + 0 * 10 + 2 * 5
5027: 0 * 200 + 3 * 100 + 1 * 50 + 7 * 20 + 1 * 10 + 0 * 5
5028: 0 * 200 + 3 * 100 + 2 * 50 + 0 * 20 + 0 * 10 + 20 * 5
5029: 0 * 200 + 3 * 100 + 2 * 50 + 0 * 20 + 1 * 10 + 18 * 5
5030: 0 * 200 + 3 * 100 + 2 * 50 + 0 * 20 + 2 * 10 + 16 * 5
5031: 0 * 200 + 3 * 100 + 2 * 50 + 0 * 20 + 3 * 10 + 14 * 5
5032: 0 * 200 + 3 * 100 + 2 * 50 + 0 * 20 + 4 * 10 + 12 * 5
5033: 0 * 200 + 3 * 100 + 2 * 50 + 0 * 20 + 5 * 10 + 10 * 5
5034: 0 * 200 + 3 * 100 + 2 * 50 + 0 * 20 + 6 * 10 + 8 * 5
5035: 0 * 200 + 3 * 100 + 2 * 50 + 0 * 20 + 7 * 10 + 6 * 5
5036: 0 * 200 + 3 * 100 + 2 * 50 + 0 * 20 + 8 * 10 + 4 * 5
5037: 0 * 200 + 3 * 100 + 2 * 50 + 0 * 20 + 9 * 10 + 2 * 5
5038: 0 * 200 + 3 * 100 + 2 * 50 + 0 * 20 + 10 * 10 + 0 * 5
5039: 0 * 200 + 3 * 100 + 2 * 50 + 1 * 20 + 0 * 10 + 16 * 5
5040: 0 * 200 + 3 * 100 + 2 * 50 + 1 * 20 + 1 * 10 + 14 * 5
5041: 0 * 200 + 3 * 100 + 2 * 50 + 1 * 20 + 2 * 10 + 12 * 5
5042: 0 * 200 + 3 * 100 + 2 * 50 + 1 * 20 + 3 * 10 + 10 * 5
5043: 0 * 200 + 3 * 100 + 2 * 50 + 1 * 20 + 4 * 10 + 8 * 5
5044: 0 * 200 + 3 * 100 + 2 * 50 + 1 * 20 + 5 * 10 + 6 * 5
5045: 0 * 200 + 3 * 100 + 2 * 50 + 1 * 20 + 6 * 10 + 4 * 5
5046: 0 * 200 + 3 * 100 + 2 * 50 + 1 * 20 + 7 * 10 + 2 * 5
5047: 0 * 200 + 3 * 100 + 2 * 50 + 1 * 20 + 8 * 10 + 0 * 5
5048: 0 * 200 + 3 * 100 + 2 * 50 + 2 * 20 + 0 * 10 + 12 * 5
5049: 0 * 200 + 3 * 100 + 2 * 50 + 2 * 20 + 1 * 10 + 10 * 5
5050: 0 * 200 + 3 * 100 + 2 * 50 + 2 * 20 + 2 * 10 + 8 * 5
5051: 0 * 200 + 3 * 100 + 2 * 50 + 2 * 20 + 3 * 10 + 6 * 5
5052: 0 * 200 + 3 * 100 + 2 * 50 + 2 * 20 + 4 * 10 + 4 * 5
5053: 0 * 200 + 3 * 100 + 2 * 50 + 2 * 20 + 5 * 10 + 2 * 5
5054: 0 * 200 + 3 * 100 + 2 * 50 + 2 * 20 + 6 * 10 + 0 * 5
5055: 0 * 200 + 3 * 100 + 2 * 50 + 3 * 20 + 0 * 10 + 8 * 5
5056: 0 * 200 + 3 * 100 + 2 * 50 + 3 * 20 + 1 * 10 + 6 * 5
5057: 0 * 200 + 3 * 100 + 2 * 50 + 3 * 20 + 2 * 10 + 4 * 5
5058: 0 * 200 + 3 * 100 + 2 * 50 + 3 * 20 + 3 * 10 + 2 * 5
5059: 0 * 200 + 3 * 100 + 2 * 50 + 3 * 20 + 4 * 10 + 0 * 5
5060: 0 * 200 + 3 * 100 + 2 * 50 + 4 * 20 + 0 * 10 + 4 * 5
5061: 0 * 200 + 3 * 100 + 2 * 50 + 4 * 20 + 1 * 10 + 2 * 5
5062: 0 * 200 + 3 * 100 + 2 * 50 + 4 * 20 + 2 * 10 + 0 * 5
5063: 0 * 200 + 3 * 100 + 2 * 50 + 5 * 20 + 0 * 10 + 0 * 5
5064: 0 * 200 + 3 * 100 + 3 * 50 + 0 * 20 + 0 * 10 + 10 * 5
5065: 0 * 200 + 3 * 100 + 3 * 50 + 0 * 20 + 1 * 10 + 8 * 5
5066: 0 * 200 + 3 * 100 + 3 * 50 + 0 * 20 + 2 * 10 + 6 * 5
5067: 0 * 200 + 3 * 100 + 3 * 50 + 0 * 20 + 3 * 10 + 4 * 5
5068: 0 * 200 + 3 * 100 + 3 * 50 + 0 * 20 + 4 * 10 + 2 * 5
5069: 0 * 200 + 3 * 100 + 3 * 50 + 0 * 20 + 5 * 10 + 0 * 5
5070: 0 * 200 + 3 * 100 + 3 * 50 + 1 * 20 + 0 * 10 + 6 * 5
5071: 0 * 200 + 3 * 100 + 3 * 50 + 1 * 20 + 1 * 10 + 4 * 5
5072: 0 * 200 + 3 * 100 + 3 * 50 + 1 * 20 + 2 * 10 + 2 * 5
5073: 0 * 200 + 3 * 100 + 3 * 50 + 1 * 20 + 3 * 10 + 0 * 5
5074: 0 * 200 + 3 * 100 + 3 * 50 + 2 * 20 + 0 * 10 + 2 * 5
5075: 0 * 200 + 3 * 100 + 3 * 50 + 2 * 20 + 1 * 10 + 0 * 5
5076: 0 * 200 + 3 * 100 + 4 * 50 + 0 * 20 + 0 * 10 + 0 * 5
5077: 0 * 200 + 4 * 100 + 0 * 50 + 0 * 20 + 0 * 10 + 20 * 5
5078: 0 * 200 + 4 * 100 + 0 * 50 + 0 * 20 + 1 * 10 + 18 * 5
5079: 0 * 200 + 4 * 100 + 0 * 50 + 0 * 20 + 2 * 10 + 16 * 5
5080: 0 * 200 + 4 * 100 + 0 * 50 + 0 * 20 + 3 * 10 + 14 * 5
5081: 0 * 200 + 4 * 100 + 0 * 50 + 0 * 20 + 4 * 10 + 12 * 5
5082: 0 * 200 + 4 * 100 + 0 * 50 + 0 * 20 + 5 * 10 + 10 * 5
5083: 0 * 200 + 4 * 100 + 0 * 50 + 0 * 20 + 6 * 10 + 8 * 5
5084: 0 * 200 + 4 * 100 + 0 * 50 + 0 * 20 + 7 * 10 + 6 * 5
5085: 0 * 200 + 4 * 100 + 0 * 50 + 0 * 20 + 8 * 10 + 4 * 5
5086: 0 * 200 + 4 * 100 + 0 * 50 + 0 * 20 + 9 * 10 + 2 * 5
5087: 0 * 200 + 4 * 100 + 0 * 50 + 0 * 20 + 10 * 10 + 0 * 5
5088: 0 * 200 + 4 * 100 + 0 * 50 + 1 * 20 + 0 * 10 + 16 * 5
5089: 0 * 200 + 4 * 100 + 0 * 50 + 1 * 20 + 1 * 10 + 14 * 5
5090: 0 * 200 + 4 * 100 + 0 * 50 + 1 * 20 + 2 * 10 + 12 * 5
5091: 0 * 200 + 4 * 100 + 0 * 50 + 1 * 20 + 3 * 10 + 10 * 5
5092: 0 * 200 + 4 * 100 + 0 * 50 + 1 * 20 + 4 * 10 + 8 * 5
5093: 0 * 200 + 4 * 100 + 0 * 50 + 1 * 20 + 5 * 10 + 6 * 5
5094: 0 * 200 + 4 * 100 + 0 * 50 + 1 * 20 + 6 * 10 + 4 * 5
5095: 0 * 200 + 4 * 100 + 0 * 50 + 1 * 20 + 7 * 10 + 2 * 5
5096: 0 * 200 + 4 * 100 + 0 * 50 + 1 * 20 + 8 * 10 + 0 * 5
5097: 0 * 200 + 4 * 100 + 0 * 50 + 2 * 20 + 0 * 10 + 12 * 5
5098: 0 * 200 + 4 * 100 + 0 * 50 + 2 * 20 + 1 * 10 + 10 * 5
5099: 0 * 200 + 4 * 100 + 0 * 50 + 2 * 20 + 2 * 10 + 8 * 5
5100: 0 * 200 + 4 * 100 + 0 * 50 + 2 * 20 + 3 * 10 + 6 * 5
5101: 0 * 200 + 4 * 100 + 0 * 50 + 2 * 20 + 4 * 10 + 4 * 5
5102: 0 * 200 + 4 * 100 + 0 * 50 + 2 * 20 + 5 * 10 + 2 * 5
5103: 0 * 200 + 4 * 100 + 0 * 50 + 2 * 20 + 6 * 10 + 0 * 5
5104: 0 * 200 + 4 * 100 + 0 * 50 + 3 * 20 + 0 * 10 + 8 * 5
5105: 0 * 200 + 4 * 100 + 0 * 50 + 3 * 20 + 1 * 10 + 6 * 5
5106: 0 * 200 + 4 * 100 + 0 * 50 + 3 * 20 + 2 * 10 + 4 * 5
5107: 0 * 200 + 4 * 100 + 0 * 50 + 3 * 20 + 3 * 10 + 2 * 5
5108: 0 * 200 + 4 * 100 + 0 * 50 + 3 * 20 + 4 * 10 + 0 * 5
5109: 0 * 200 + 4 * 100 + 0 * 50 + 4 * 20 + 0 * 10 + 4 * 5
5110: 0 * 200 + 4 * 100 + 0 * 50 + 4 * 20 + 1 * 10 + 2 * 5
5111: 0 * 200 + 4 * 100 + 0 * 50 + 4 * 20 + 2 * 10 + 0 * 5
5112: 0 * 200 + 4 * 100 + 0 * 50 + 5 * 20 + 0 * 10 + 0 * 5
5113: 0 * 200 + 4 * 100 + 1 * 50 + 0 * 20 + 0 * 10 + 10 * 5
5114: 0 * 200 + 4 * 100 + 1 * 50 + 0 * 20 + 1 * 10 + 8 * 5
5115: 0 * 200 + 4 * 100 + 1 * 50 + 0 * 20 + 2 * 10 + 6 * 5
5116: 0 * 200 + 4 * 100 + 1 * 50 + 0 * 20 + 3 * 10 + 4 * 5
5117: 0 * 200 + 4 * 100 + 1 * 50 + 0 * 20 + 4 * 10 + 2 * 5
5118: 0 * 200 + 4 * 100 + 1 * 50 + 0 * 20 + 5 * 10 + 0 * 5
5119: 0 * 200 + 4 * 100 + 1 * 50 + 1 * 20 + 0 * 10 + 6 * 5
5120: 0 * 200 + 4 * 100 + 1 * 50 + 1 * 20 + 1 * 10 + 4 * 5
5121: 0 * 200 + 4 * 100 + 1 * 50 + 1 * 20 + 2 * 10 + 2 * 5
5122: 0 * 200 + 4 * 100 + 1 * 50 + 1 * 20 + 3 * 10 + 0 * 5
5123: 0 * 200 + 4 * 100 + 1 * 50 + 2 * 20 + 0 * 10 + 2 * 5
5124: 0 * 200 + 4 * 100 + 1 * 50 + 2 * 20 + 1 * 10 + 0 * 5
5125: 0 * 200 + 4 * 100 + 2 * 50 + 0 * 20 + 0 * 10 + 0 * 5
5126: 0 * 200 + 5 * 100 + 0 * 50 + 0 * 20 + 0 * 10 + 0 * 5
5127: 1 * 200 + 0 * 100 + 0 * 50 + 0 * 20 + 0 * 10 + 60 * 5
5128: 1 * 200 + 0 * 100 + 0 * 50 + 0 * 20 + 1 * 10 + 58 * 5
5129: 1 * 200 + 0 * 100 + 0 * 50 + 0 * 20 + 2 * 10 + 56 * 5
5130: 1 * 200 + 0 * 100 + 0 * 50 + 0 * 20 + 3 * 10 + 54 * 5
5131: 1 * 200 + 0 * 100 + 0 * 50 + 0 * 20 + 4 * 10 + 52 * 5
5132: 1 * 200 + 0 * 100 + 0 * 50 + 0 * 20 + 5 * 10 + 50 * 5
5133: 1 * 200 + 0 * 100 + 0 * 50 + 0 * 20 + 6 * 10 + 48 * 5
5134: 1 * 200 + 0 * 100 + 0 * 50 + 0 * 20 + 7 * 10 + 46 * 5
5135: 1 * 200 + 0 * 100 + 0 * 50 + 0 * 20 + 8 * 10 + 44 * 5
5136: 1 * 200 + 0 * 100 + 0 * 50 + 0 * 20 + 9 * 10 + 42 * 5
5137: 1 * 200 + 0 * 100 + 0 * 50 + 0 * 20 + 10 * 10 + 40 * 5
5138: 1 * 200 + 0 * 100 + 0 * 50 + 0 * 20 + 11 * 10 + 38 * 5
5139: 1 * 200 + 0 * 100 + 0 * 50 + 0 * 20 + 12 * 10 + 36 * 5
5140: 1 * 200 + 0 * 100 + 0 * 50 + 0 * 20 + 13 * 10 + 34 * 5
5141: 1 * 200 + 0 * 100 + 0 * 50 + 0 * 20 + 14 * 10 + 32 * 5
5142: 1 * 200 + 0 * 100 + 0 * 50 + 0 * 20 + 15 * 10 + 30 * 5
5143: 1 * 200 + 0 * 100 + 0 * 50 + 0 * 20 + 16 * 10 + 28 * 5
5144: 1 * 200 + 0 * 100 + 0 * 50 + 0 * 20 + 17 * 10 + 26 * 5
5145: 1 * 200 + 0 * 100 + 0 * 50 + 0 * 20 + 18 * 10 + 24 * 5
5146: 1 * 200 + 0 * 100 + 0 * 50 + 0 * 20 + 19 * 10 + 22 * 5
5147: 1 * 200 + 0 * 100 + 0 * 50 + 0 * 20 + 20 * 10 + 20 * 5
5148: 1 * 200 + 0 * 100 + 0 * 50 + 0 * 20 + 21 * 10 + 18 * 5
5149: 1 * 200 + 0 * 100 + 0 * 50 + 0 * 20 + 22 * 10 + 16 * 5
5150: 1 * 200 + 0 * 100 + 0 * 50 + 0 * 20 + 23 * 10 + 14 * 5
5151: 1 * 200 + 0 * 100 + 0 * 50 + 0 * 20 + 24 * 10 + 12 * 5
5152: 1 * 200 + 0 * 100 + 0 * 50 + 0 * 20 + 25 * 10 + 10 * 5
5153: 1 * 200 + 0 * 100 + 0 * 50 + 0 * 20 + 26 * 10 + 8 * 5
5154: 1 * 200 + 0 * 100 + 0 * 50 + 0 * 20 + 27 * 10 + 6 * 5
5155: 1 * 200 + 0 * 100 + 0 * 50 + 0 * 20 + 28 * 10 + 4 * 5
5156: 1 * 200 + 0 * 100 + 0 * 50 + 0 * 20 + 29 * 10 + 2 * 5
5157: 1 * 200 + 0 * 100 + 0 * 50 + 0 * 20 + 30 * 10 + 0 * 5
5158: 1 * 200 + 0 * 100 + 0 * 50 + 1 * 20 + 0 * 10 + 56 * 5
5159: 1 * 200 + 0 * 100 + 0 * 50 + 1 * 20 + 1 * 10 + 54 * 5
5160: 1 * 200 + 0 * 100 + 0 * 50 + 1 * 20 + 2 * 10 + 52 * 5
5161: 1 * 200 + 0 * 100 + 0 * 50 + 1 * 20 + 3 * 10 + 50 * 5
5162: 1 * 200 + 0 * 100 + 0 * 50 + 1 * 20 + 4 * 10 + 48 * 5
5163: 1 * 200 + 0 * 100 + 0 * 50 + 1 * 20 + 5 * 10 + 46 * 5
5164: 1 * 200 + 0 * 100 + 0 * 50 + 1 * 20 + 6 * 10 + 44 * 5
5165: 1 * 200 + 0 * 100 + 0 * 50 + 1 * 20 + 7 * 10 + 42 * 5
5166: 1 * 200 + 0 * 100 + 0 * 50 + 1 * 20 + 8 * 10 + 40 * 5
5167: 1 * 200 + 0 * 100 + 0 * 50 + 1 * 20 + 9 * 10 + 38 * 5
5168: 1 * 200 + 0 * 100 + 0 * 50 + 1 * 20 + 10 * 10 + 36 * 5
5169: 1 * 200 + 0 * 100 + 0 * 50 + 1 * 20 + 11 * 10 + 34 * 5
5170: 1 * 200 + 0 * 100 + 0 * 50 + 1 * 20 + 12 * 10 + 32 * 5
5171: 1 * 200 + 0 * 100 + 0 * 50 + 1 * 20 + 13 * 10 + 30 * 5
5172: 1 * 200 + 0 * 100 + 0 * 50 + 1 * 20 + 14 * 10 + 28 * 5
5173: 1 * 200 + 0 * 100 + 0 * 50 + 1 * 20 + 15 * 10 + 26 * 5
5174: 1 * 200 + 0 * 100 + 0 * 50 + 1 * 20 + 16 * 10 + 24 * 5
5175: 1 * 200 + 0 * 100 + 0 * 50 + 1 * 20 + 17 * 10 + 22 * 5
5176: 1 * 200 + 0 * 100 + 0 * 50 + 1 * 20 + 18 * 10 + 20 * 5
5177: 1 * 200 + 0 * 100 + 0 * 50 + 1 * 20 + 19 * 10 + 18 * 5
5178: 1 * 200 + 0 * 100 + 0 * 50 + 1 * 20 + 20 * 10 + 16 * 5
5179: 1 * 200 + 0 * 100 + 0 * 50 + 1 * 20 + 21 * 10 + 14 * 5
5180: 1 * 200 + 0 * 100 + 0 * 50 + 1 * 20 + 22 * 10 + 12 * 5
5181: 1 * 200 + 0 * 100 + 0 * 50 + 1 * 20 + 23 * 10 + 10 * 5
5182: 1 * 200 + 0 * 100 + 0 * 50 + 1 * 20 + 24 * 10 + 8 * 5
5183: 1 * 200 + 0 * 100 + 0 * 50 + 1 * 20 + 25 * 10 + 6 * 5
5184: 1 * 200 + 0 * 100 + 0 * 50 + 1 * 20 + 26 * 10 + 4 * 5
5185: 1 * 200 + 0 * 100 + 0 * 50 + 1 * 20 + 27 * 10 + 2 * 5
5186: 1 * 200 + 0 * 100 + 0 * 50 + 1 * 20 + 28 * 10 + 0 * 5
5187: 1 * 200 + 0 * 100 + 0 * 50 + 2 * 20 + 0 * 10 + 52 * 5
5188: 1 * 200 + 0 * 100 + 0 * 50 + 2 * 20 + 1 * 10 + 50 * 5
5189: 1 * 200 + 0 * 100 + 0 * 50 + 2 * 20 + 2 * 10 + 48 * 5
5190: 1 * 200 + 0 * 100 + 0 * 50 + 2 * 20 + 3 * 10 + 46 * 5
5191: 1 * 200 + 0 * 100 + 0 * 50 + 2 * 20 + 4 * 10 + 44 * 5
5192: 1 * 200 + 0 * 100 + 0 * 50 + 2 * 20 + 5 * 10 + 42 * 5
5193: 1 * 200 + 0 * 100 + 0 * 50 + 2 * 20 + 6 * 10 + 40 * 5
5194: 1 * 200 + 0 * 100 + 0 * 50 + 2 * 20 + 7 * 10 + 38 * 5
5195: 1 * 200 + 0 * 100 + 0 * 50 + 2 * 20 + 8 * 10 + 36 * 5
5196: 1 * 200 + 0 * 100 + 0 * 50 + 2 * 20 + 9 * 10 + 34 * 5
5197: 1 * 200 + 0 * 100 + 0 * 50 + 2 * 20 + 10 * 10 + 32 * 5
5198: 1 * 200 + 0 * 100 + 0 * 50 + 2 * 20 + 11 * 10 + 30 * 5
5199: 1 * 200 + 0 * 100 + 0 * 50 + 2 * 20 + 12 * 10 + 28 * 5
5200: 1 * 200 + 0 * 100 + 0 * 50 + 2 * 20 + 13 * 10 + 26 * 5
5201: 1 * 200 + 0 * 100 + 0 * 50 + 2 * 20 + 14 * 10 + 24 * 5
5202: 1 * 200 + 0 * 100 + 0 * 50 + 2 * 20 + 15 * 10 + 22 * 5
5203: 1 * 200 + 0 * 100 + 0 * 50 + 2 * 20 + 16 * 10 + 20 * 5
5204: 1 * 200 + 0 * 100 + 0 * 50 + 2 * 20 + 17 * 10 + 18 * 5
5205: 1 * 200 + 0 * 100 + 0 * 50 + 2 * 20 + 18 * 10 + 16 * 5
5206: 1 * 200 + 0 * 100 + 0 * 50 + 2 * 20 + 19 * 10 + 14 * 5
5207: 1 * 200 + 0 * 100 + 0 * 50 + 2 * 20 + 20 * 10 + 12 * 5
5208: 1 * 200 + 0 * 100 + 0 * 50 + 2 * 20 + 21 * 10 + 10 * 5
5209: 1 * 200 + 0 * 100 + 0 * 50 + 2 * 20 + 22 * 10 + 8 * 5
5210: 1 * 200 + 0 * 100 + 0 * 50 + 2 * 20 + 23 * 10 + 6 * 5
5211: 1 * 200 + 0 * 100 + 0 * 50 + 2 * 20 + 24 * 10 + 4 * 5
5212: 1 * 200 + 0 * 100 + 0 * 50 + 2 * 20 + 25 * 10 + 2 * 5
5213: 1 * 200 + 0 * 100 + 0 * 50 + 2 * 20 + 26 * 10 + 0 * 5
5214: 1 * 200 + 0 * 100 + 0 * 50 + 3 * 20 + 0 * 10 + 48 * 5
5215: 1 * 200 + 0 * 100 + 0 * 50 + 3 * 20 + 1 * 10 + 46 * 5
5216: 1 * 200 + 0 * 100 + 0 * 50 + 3 * 20 + 2 * 10 + 44 * 5
5217: 1 * 200 + 0 * 100 + 0 * 50 + 3 * 20 + 3 * 10 + 42 * 5
5218: 1 * 200 + 0 * 100 + 0 * 50 + 3 * 20 + 4 * 10 + 40 * 5
5219: 1 * 200 + 0 * 100 + 0 * 50 + 3 * 20 + 5 * 10 + 38 * 5
5220: 1 * 200 + 0 * 100 + 0 * 50 + 3 * 20 + 6 * 10 + 36 * 5
5221: 1 * 200 + 0 * 100 + 0 * 50 + 3 * 20 + 7 * 10 + 34 * 5
5222: 1 * 200 + 0 * 100 + 0 * 50 + 3 * 20 + 8 * 10 + 32 * 5
5223: 1 * 200 + 0 * 100 + 0 * 50 + 3 * 20 + 9 * 10 + 30 * 5
5224: 1 * 200 + 0 * 100 + 0 * 50 + 3 * 20 + 10 * 10 + 28 * 5
5225: 1 * 200 + 0 * 100 + 0 * 50 + 3 * 20 + 11 * 10 + 26 * 5
5226: 1 * 200 + 0 * 100 + 0 * 50 + 3 * 20 + 12 * 10 + 24 * 5
5227: 1 * 200 + 0 * 100 + 0 * 50 + 3 * 20 + 13 * 10 + 22 * 5
5228: 1 * 200 + 0 * 100 + 0 * 50 + 3 * 20 + 14 * 10 + 20 * 5
5229: 1 * 200 + 0 * 100 + 0 * 50 + 3 * 20 + 15 * 10 + 18 * 5
5230: 1 * 200 + 0 * 100 + 0 * 50 + 3 * 20 + 16 * 10 + 16 * 5
5231: 1 * 200 + 0 * 100 + 0 * 50 + 3 * 20 + 17 * 10 + 14 * 5
5232: 1 * 200 + 0 * 100 + 0 * 50 + 3 * 20 + 18 * 10 + 12 * 5
5233: 1 * 200 + 0 * 100 + 0 * 50 + 3 * 20 + 19 * 10 + 10 * 5
5234: 1 * 200 + 0 * 100 + 0 * 50 + 3 * 20 + 20 * 10 + 8 * 5
5235: 1 * 200 + 0 * 100 + 0 * 50 + 3 * 20 + 21 * 10 + 6 * 5
5236: 1 * 200 + 0 * 100 + 0 * 50 + 3 * 20 + 22 * 10 + 4 * 5
5237: 1 * 200 + 0 * 100 + 0 * 50 + 3 * 20 + 23 * 10 + 2 * 5
5238: 1 * 200 + 0 * 100 + 0 * 50 + 3 * 20 + 24 * 10 + 0 * 5
5239: 1 * 200 + 0 * 100 + 0 * 50 + 4 * 20 + 0 * 10 + 44 * 5
5240: 1 * 200 + 0 * 100 + 0 * 50 + 4 * 20 + 1 * 10 + 42 * 5
5241: 1 * 200 + 0 * 100 + 0 * 50 + 4 * 20 + 2 * 10 + 40 * 5
5242: 1 * 200 + 0 * 100 + 0 * 50 + 4 * 20 + 3 * 10 + 38 * 5
5243: 1 * 200 + 0 * 100 + 0 * 50 + 4 * 20 + 4 * 10 + 36 * 5
5244: 1 * 200 + 0 * 100 + 0 * 50 + 4 * 20 + 5 * 10 + 34 * 5
5245: 1 * 200 + 0 * 100 + 0 * 50 + 4 * 20 + 6 * 10 + 32 * 5
5246: 1 * 200 + 0 * 100 + 0 * 50 + 4 * 20 + 7 * 10 + 30 * 5
5247: 1 * 200 + 0 * 100 + 0 * 50 + 4 * 20 + 8 * 10 + 28 * 5
5248: 1 * 200 + 0 * 100 + 0 * 50 + 4 * 20 + 9 * 10 + 26 * 5
5249: 1 * 200 + 0 * 100 + 0 * 50 + 4 * 20 + 10 * 10 + 24 * 5
5250: 1 * 200 + 0 * 100 + 0 * 50 + 4 * 20 + 11 * 10 + 22 * 5
5251: 1 * 200 + 0 * 100 + 0 * 50 + 4 * 20 + 12 * 10 + 20 * 5
5252: 1 * 200 + 0 * 100 + 0 * 50 + 4 * 20 + 13 * 10 + 18 * 5
5253: 1 * 200 + 0 * 100 + 0 * 50 + 4 * 20 + 14 * 10 + 16 * 5
5254: 1 * 200 + 0 * 100 + 0 * 50 + 4 * 20 + 15 * 10 + 14 * 5
5255: 1 * 200 + 0 * 100 + 0 * 50 + 4 * 20 + 16 * 10 + 12 * 5
5256: 1 * 200 + 0 * 100 + 0 * 50 + 4 * 20 + 17 * 10 + 10 * 5
5257: 1 * 200 + 0 * 100 + 0 * 50 + 4 * 20 + 18 * 10 + 8 * 5
5258: 1 * 200 + 0 * 100 + 0 * 50 + 4 * 20 + 19 * 10 + 6 * 5
5259: 1 * 200 + 0 * 100 + 0 * 50 + 4 * 20 + 20 * 10 + 4 * 5
5260: 1 * 200 + 0 * 100 + 0 * 50 + 4 * 20 + 21 * 10 + 2 * 5
5261: 1 * 200 + 0 * 100 + 0 * 50 + 4 * 20 + 22 * 10 + 0 * 5
5262: 1 * 200 + 0 * 100 + 0 * 50 + 5 * 20 + 0 * 10 + 40 * 5
5263: 1 * 200 + 0 * 100 + 0 * 50 + 5 * 20 + 1 * 10 + 38 * 5
5264: 1 * 200 + 0 * 100 + 0 * 50 + 5 * 20 + 2 * 10 + 36 * 5
5265: 1 * 200 + 0 * 100 + 0 * 50 + 5 * 20 + 3 * 10 + 34 * 5
5266: 1 * 200 + 0 * 100 + 0 * 50 + 5 * 20 + 4 * 10 + 32 * 5
5267: 1 * 200 + 0 * 100 + 0 * 50 + 5 * 20 + 5 * 10 + 30 * 5
5268: 1 * 200 + 0 * 100 + 0 * 50 + 5 * 20 + 6 * 10 + 28 * 5
5269: 1 * 200 + 0 * 100 + 0 * 50 + 5 * 20 + 7 * 10 + 26 * 5
5270: 1 * 200 + 0 * 100 + 0 * 50 + 5 * 20 + 8 * 10 + 24 * 5
5271: 1 * 200 + 0 * 100 + 0 * 50 + 5 * 20 + 9 * 10 + 22 * 5
5272: 1 * 200 + 0 * 100 + 0 * 50 + 5 * 20 + 10 * 10 + 20 * 5
5273: 1 * 200 + 0 * 100 + 0 * 50 + 5 * 20 + 11 * 10 + 18 * 5
5274: 1 * 200 + 0 * 100 + 0 * 50 + 5 * 20 + 12 * 10 + 16 * 5
5275: 1 * 200 + 0 * 100 + 0 * 50 + 5 * 20 + 13 * 10 + 14 * 5
5276: 1 * 200 + 0 * 100 + 0 * 50 + 5 * 20 + 14 * 10 + 12 * 5
5277: 1 * 200 + 0 * 100 + 0 * 50 + 5 * 20 + 15 * 10 + 10 * 5
5278: 1 * 200 + 0 * 100 + 0 * 50 + 5 * 20 + 16 * 10 + 8 * 5
5279: 1 * 200 + 0 * 100 + 0 * 50 + 5 * 20 + 17 * 10 + 6 * 5
5280: 1 * 200 + 0 * 100 + 0 * 50 + 5 * 20 + 18 * 10 + 4 * 5
5281: 1 * 200 + 0 * 100 + 0 * 50 + 5 * 20 + 19 * 10 + 2 * 5
5282: 1 * 200 + 0 * 100 + 0 * 50 + 5 * 20 + 20 * 10 + 0 * 5
5283: 1 * 200 + 0 * 100 + 0 * 50 + 6 * 20 + 0 * 10 + 36 * 5
5284: 1 * 200 + 0 * 100 + 0 * 50 + 6 * 20 + 1 * 10 + 34 * 5
5285: 1 * 200 + 0 * 100 + 0 * 50 + 6 * 20 + 2 * 10 + 32 * 5
5286: 1 * 200 + 0 * 100 + 0 * 50 + 6 * 20 + 3 * 10 + 30 * 5
5287: 1 * 200 + 0 * 100 + 0 * 50 + 6 * 20 + 4 * 10 + 28 * 5
5288: 1 * 200 + 0 * 100 + 0 * 50 + 6 * 20 + 5 * 10 + 26 * 5
5289: 1 * 200 + 0 * 100 + 0 * 50 + 6 * 20 + 6 * 10 + 24 * 5
5290: 1 * 200 + 0 * 100 + 0 * 50 + 6 * 20 + 7 * 10 + 22 * 5
5291: 1 * 200 + 0 * 100 + 0 * 50 + 6 * 20 + 8 * 10 + 20 * 5
5292: 1 * 200 + 0 * 100 + 0 * 50 + 6 * 20 + 9 * 10 + 18 * 5
5293: 1 * 200 + 0 * 100 + 0 * 50 + 6 * 20 + 10 * 10 + 16 * 5
5294: 1 * 200 + 0 * 100 + 0 * 50 + 6 * 20 + 11 * 10 + 14 * 5
5295: 1 * 200 + 0 * 100 + 0 * 50 + 6 * 20 + 12 * 10 + 12 * 5
5296: 1 * 200 + 0 * 100 + 0 * 50 + 6 * 20 + 13 * 10 + 10 * 5
5297: 1 * 200 + 0 * 100 + 0 * 50 + 6 * 20 + 14 * 10 + 8 * 5
5298: 1 * 200 + 0 * 100 + 0 * 50 + 6 * 20 + 15 * 10 + 6 * 5
5299: 1 * 200 + 0 * 100 + 0 * 50 + 6 * 20 + 16 * 10 + 4 * 5
5300: 1 * 200 + 0 * 100 + 0 * 50 + 6 * 20 + 17 * 10 + 2 * 5
5301: 1 * 200 + 0 * 100 + 0 * 50 + 6 * 20 + 18 * 10 + 0 * 5
5302: 1 * 200 + 0 * 100 + 0 * 50 + 7 * 20 + 0 * 10 + 32 * 5
5303: 1 * 200 + 0 * 100 + 0 * 50 + 7 * 20 + 1 * 10 + 30 * 5
5304: 1 * 200 + 0 * 100 + 0 * 50 + 7 * 20 + 2 * 10 + 28 * 5
5305: 1 * 200 + 0 * 100 + 0 * 50 + 7 * 20 + 3 * 10 + 26 * 5
5306: 1 * 200 + 0 * 100 + 0 * 50 + 7 * 20 + 4 * 10 + 24 * 5
5307: 1 * 200 + 0 * 100 + 0 * 50 + 7 * 20 + 5 * 10 + 22 * 5
5308: 1 * 200 + 0 * 100 + 0 * 50 + 7 * 20 + 6 * 10 + 20 * 5
5309: 1 * 200 + 0 * 100 + 0 * 50 + 7 * 20 + 7 * 10 + 18 * 5
5310: 1 * 200 + 0 * 100 + 0 * 50 + 7 * 20 + 8 * 10 + 16 * 5
5311: 1 * 200 + 0 * 100 + 0 * 50 + 7 * 20 + 9 * 10 + 14 * 5
5312: 1 * 200 + 0 * 100 + 0 * 50 + 7 * 20 + 10 * 10 + 12 * 5
5313: 1 * 200 + 0 * 100 + 0 * 50 + 7 * 20 + 11 * 10 + 10 * 5
5314: 1 * 200 + 0 * 100 + 0 * 50 + 7 * 20 + 12 * 10 + 8 * 5
5315: 1 * 200 + 0 * 100 + 0 * 50 + 7 * 20 + 13 * 10 + 6 * 5
5316: 1 * 200 + 0 * 100 + 0 * 50 + 7 * 20 + 14 * 10 + 4 * 5
5317: 1 * 200 + 0 * 100 + 0 * 50 + 7 * 20 + 15 * 10 + 2 * 5
5318: 1 * 200 + 0 * 100 + 0 * 50 + 7 * 20 + 16 * 10 + 0 * 5
5319: 1 * 200 + 0 * 100 + 0 * 50 + 8 * 20 + 0 * 10 + 28 * 5
5320: 1 * 200 + 0 * 100 + 0 * 50 + 8 * 20 + 1 * 10 + 26 * 5
5321: 1 * 200 + 0 * 100 + 0 * 50 + 8 * 20 + 2 * 10 + 24 * 5
5322: 1 * 200 + 0 * 100 + 0 * 50 + 8 * 20 + 3 * 10 + 22 * 5
5323: 1 * 200 + 0 * 100 + 0 * 50 + 8 * 20 + 4 * 10 + 20 * 5
5324: 1 * 200 + 0 * 100 + 0 * 50 + 8 * 20 + 5 * 10 + 18 * 5
5325: 1 * 200 + 0 * 100 + 0 * 50 + 8 * 20 + 6 * 10 + 16 * 5
5326: 1 * 200 + 0 * 100 + 0 * 50 + 8 * 20 + 7 * 10 + 14 * 5
5327: 1 * 200 + 0 * 100 + 0 * 50 + 8 * 20 + 8 * 10 + 12 * 5
5328: 1 * 200 + 0 * 100 + 0 * 50 + 8 * 20 + 9 * 10 + 10 * 5
5329: 1 * 200 + 0 * 100 + 0 * 50 + 8 * 20 + 10 * 10 + 8 * 5
5330: 1 * 200 + 0 * 100 + 0 * 50 + 8 * 20 + 11 * 10 + 6 * 5
5331: 1 * 200 + 0 * 100 + 0 * 50 + 8 * 20 + 12 * 10 + 4 * 5
5332: 1 * 200 + 0 * 100 + 0 * 50 + 8 * 20 + 13 * 10 + 2 * 5
5333: 1 * 200 + 0 * 100 + 0 * 50 + 8 * 20 + 14 * 10 + 0 * 5
5334: 1 * 200 + 0 * 100 + 0 * 50 + 9 * 20 + 0 * 10 + 24 * 5
5335: 1 * 200 + 0 * 100 + 0 * 50 + 9 * 20 + 1 * 10 + 22 * 5
5336: 1 * 200 + 0 * 100 + 0 * 50 + 9 * 20 + 2 * 10 + 20 * 5
5337: 1 * 200 + 0 * 100 + 0 * 50 + 9 * 20 + 3 * 10 + 18 * 5
5338: 1 * 200 + 0 * 100 + 0 * 50 + 9 * 20 + 4 * 10 + 16 * 5
5339: 1 * 200 + 0 * 100 + 0 * 50 + 9 * 20 + 5 * 10 + 14 * 5
5340: 1 * 200 + 0 * 100 + 0 * 50 + 9 * 20 + 6 * 10 + 12 * 5
5341: 1 * 200 + 0 * 100 + 0 * 50 + 9 * 20 + 7 * 10 + 10 * 5
5342: 1 * 200 + 0 * 100 + 0 * 50 + 9 * 20 + 8 * 10 + 8 * 5
5343: 1 * 200 + 0 * 100 + 0 * 50 + 9 * 20 + 9 * 10 + 6 * 5
5344: 1 * 200 + 0 * 100 + 0 * 50 + 9 * 20 + 10 * 10 + 4 * 5
5345: 1 * 200 + 0 * 100 + 0 * 50 + 9 * 20 + 11 * 10 + 2 * 5
5346: 1 * 200 + 0 * 100 + 0 * 50 + 9 * 20 + 12 * 10 + 0 * 5
5347: 1 * 200 + 0 * 100 + 0 * 50 + 10 * 20 + 0 * 10 + 20 * 5
5348: 1 * 200 + 0 * 100 + 0 * 50 + 10 * 20 + 1 * 10 + 18 * 5
5349: 1 * 200 + 0 * 100 + 0 * 50 + 10 * 20 + 2 * 10 + 16 * 5
5350: 1 * 200 + 0 * 100 + 0 * 50 + 10 * 20 + 3 * 10 + 14 * 5
5351: 1 * 200 + 0 * 100 + 0 * 50 + 10 * 20 + 4 * 10 + 12 * 5
5352: 1 * 200 + 0 * 100 + 0 * 50 + 10 * 20 + 5 * 10 + 10 * 5
5353: 1 * 200 + 0 * 100 + 0 * 50 + 10 * 20 + 6 * 10 + 8 * 5
5354: 1 * 200 + 0 * 100 + 0 * 50 + 10 * 20 + 7 * 10 + 6 * 5
5355: 1 * 200 + 0 * 100 + 0 * 50 + 10 * 20 + 8 * 10 + 4 * 5
5356: 1 * 200 + 0 * 100 + 0 * 50 + 10 * 20 + 9 * 10 + 2 * 5
5357: 1 * 200 + 0 * 100 + 0 * 50 + 10 * 20 + 10 * 10 + 0 * 5
5358: 1 * 200 + 0 * 100 + 0 * 50 + 11 * 20 + 0 * 10 + 16 * 5
5359: 1 * 200 + 0 * 100 + 0 * 50 + 11 * 20 + 1 * 10 + 14 * 5
5360: 1 * 200 + 0 * 100 + 0 * 50 + 11 * 20 + 2 * 10 + 12 * 5
5361: 1 * 200 + 0 * 100 + 0 * 50 + 11 * 20 + 3 * 10 + 10 * 5
5362: 1 * 200 + 0 * 100 + 0 * 50 + 11 * 20 + 4 * 10 + 8 * 5
5363: 1 * 200 + 0 * 100 + 0 * 50 + 11 * 20 + 5 * 10 + 6 * 5
5364: 1 * 200 + 0 * 100 + 0 * 50 + 11 * 20 + 6 * 10 + 4 * 5
5365: 1 * 200 + 0 * 100 + 0 * 50 + 11 * 20 + 7 * 10 + 2 * 5
5366: 1 * 200 + 0 * 100 + 0 * 50 + 11 * 20 + 8 * 10 + 0 * 5
5367: 1 * 200 + 0 * 100 + 0 * 50 + 12 * 20 + 0 * 10 + 12 * 5
5368: 1 * 200 + 0 * 100 + 0 * 50 + 12 * 20 + 1 * 10 + 10 * 5
5369: 1 * 200 + 0 * 100 + 0 * 50 + 12 * 20 + 2 * 10 + 8 * 5
5370: 1 * 200 + 0 * 100 + 0 * 50 + 12 * 20 + 3 * 10 + 6 * 5
5371: 1 * 200 + 0 * 100 + 0 * 50 + 12 * 20 + 4 * 10 + 4 * 5
5372: 1 * 200 + 0 * 100 + 0 * 50 + 12 * 20 + 5 * 10 + 2 * 5
5373: 1 * 200 + 0 * 100 + 0 * 50 + 12 * 20 + 6 * 10 + 0 * 5
5374: 1 * 200 + 0 * 100 + 0 * 50 + 13 * 20 + 0 * 10 + 8 * 5
5375: 1 * 200 + 0 * 100 + 0 * 50 + 13 * 20 + 1 * 10 + 6 * 5
5376: 1 * 200 + 0 * 100 + 0 * 50 + 13 * 20 + 2 * 10 + 4 * 5
5377: 1 * 200 + 0 * 100 + 0 * 50 + 13 * 20 + 3 * 10 + 2 * 5
5378: 1 * 200 + 0 * 100 + 0 * 50 + 13 * 20 + 4 * 10 + 0 * 5
5379: 1 * 200 + 0 * 100 + 0 * 50 + 14 * 20 + 0 * 10 + 4 * 5
5380: 1 * 200 + 0 * 100 + 0 * 50 + 14 * 20 + 1 * 10 + 2 * 5
5381: 1 * 200 + 0 * 100 + 0 * 50 + 14 * 20 + 2 * 10 + 0 * 5
5382: 1 * 200 + 0 * 100 + 0 * 50 + 15 * 20 + 0 * 10 + 0 * 5
5383: 1 * 200 + 0 * 100 + 1 * 50 + 0 * 20 + 0 * 10 + 50 * 5
5384: 1 * 200 + 0 * 100 + 1 * 50 + 0 * 20 + 1 * 10 + 48 * 5
5385: 1 * 200 + 0 * 100 + 1 * 50 + 0 * 20 + 2 * 10 + 46 * 5
5386: 1 * 200 + 0 * 100 + 1 * 50 + 0 * 20 + 3 * 10 + 44 * 5
5387: 1 * 200 + 0 * 100 + 1 * 50 + 0 * 20 + 4 * 10 + 42 * 5
5388: 1 * 200 + 0 * 100 + 1 * 50 + 0 * 20 + 5 * 10 + 40 * 5
5389: 1 * 200 + 0 * 100 + 1 * 50 + 0 * 20 + 6 * 10 + 38 * 5
5390: 1 * 200 + 0 * 100 + 1 * 50 + 0 * 20 + 7 * 10 + 36 * 5
5391: 1 * 200 + 0 * 100 + 1 * 50 + 0 * 20 + 8 * 10 + 34 * 5
5392: 1 * 200 + 0 * 100 + 1 * 50 + 0 * 20 + 9 * 10 + 32 * 5
5393: 1 * 200 + 0 * 100 + 1 * 50 + 0 * 20 + 10 * 10 + 30 * 5
5394: 1 * 200 + 0 * 100 + 1 * 50 + 0 * 20 + 11 * 10 + 28 * 5
5395: 1 * 200 + 0 * 100 + 1 * 50 + 0 * 20 + 12 * 10 + 26 * 5
5396: 1 * 200 + 0 * 100 + 1 * 50 + 0 * 20 + 13 * 10 + 24 * 5
5397: 1 * 200 + 0 * 100 + 1 * 50 + 0 * 20 + 14 * 10 + 22 * 5
5398: 1 * 200 + 0 * 100 + 1 * 50 + 0 * 20 + 15 * 10 + 20 * 5
5399: 1 * 200 + 0 * 100 + 1 * 50 + 0 * 20 + 16 * 10 + 18 * 5
5400: 1 * 200 + 0 * 100 + 1 * 50 + 0 * 20 + 17 * 10 + 16 * 5
5401: 1 * 200 + 0 * 100 + 1 * 50 + 0 * 20 + 18 * 10 + 14 * 5
5402: 1 * 200 + 0 * 100 + 1 * 50 + 0 * 20 + 19 * 10 + 12 * 5
5403: 1 * 200 + 0 * 100 + 1 * 50 + 0 * 20 + 20 * 10 + 10 * 5
5404: 1 * 200 + 0 * 100 + 1 * 50 + 0 * 20 + 21 * 10 + 8 * 5
5405: 1 * 200 + 0 * 100 + 1 * 50 + 0 * 20 + 22 * 10 + 6 * 5
5406: 1 * 200 + 0 * 100 + 1 * 50 + 0 * 20 + 23 * 10 + 4 * 5
5407: 1 * 200 + 0 * 100 + 1 * 50 + 0 * 20 + 24 * 10 + 2 * 5
5408: 1 * 200 + 0 * 100 + 1 * 50 + 0 * 20 + 25 * 10 + 0 * 5
5409: 1 * 200 + 0 * 100 + 1 * 50 + 1 * 20 + 0 * 10 + 46 * 5
5410: 1 * 200 + 0 * 100 + 1 * 50 + 1 * 20 + 1 * 10 + 44 * 5
5411: 1 * 200 + 0 * 100 + 1 * 50 + 1 * 20 + 2 * 10 + 42 * 5
5412: 1 * 200 + 0 * 100 + 1 * 50 + 1 * 20 + 3 * 10 + 40 * 5
5413: 1 * 200 + 0 * 100 + 1 * 50 + 1 * 20 + 4 * 10 + 38 * 5
5414: 1 * 200 + 0 * 100 + 1 * 50 + 1 * 20 + 5 * 10 + 36 * 5
5415: 1 * 200 + 0 * 100 + 1 * 50 + 1 * 20 + 6 * 10 + 34 * 5
5416: 1 * 200 + 0 * 100 + 1 * 50 + 1 * 20 + 7 * 10 + 32 * 5
5417: 1 * 200 + 0 * 100 + 1 * 50 + 1 * 20 + 8 * 10 + 30 * 5
5418: 1 * 200 + 0 * 100 + 1 * 50 + 1 * 20 + 9 * 10 + 28 * 5
5419: 1 * 200 + 0 * 100 + 1 * 50 + 1 * 20 + 10 * 10 + 26 * 5
5420: 1 * 200 + 0 * 100 + 1 * 50 + 1 * 20 + 11 * 10 + 24 * 5
5421: 1 * 200 + 0 * 100 + 1 * 50 + 1 * 20 + 12 * 10 + 22 * 5
5422: 1 * 200 + 0 * 100 + 1 * 50 + 1 * 20 + 13 * 10 + 20 * 5
5423: 1 * 200 + 0 * 100 + 1 * 50 + 1 * 20 + 14 * 10 + 18 * 5
5424: 1 * 200 + 0 * 100 + 1 * 50 + 1 * 20 + 15 * 10 + 16 * 5
5425: 1 * 200 + 0 * 100 + 1 * 50 + 1 * 20 + 16 * 10 + 14 * 5
5426: 1 * 200 + 0 * 100 + 1 * 50 + 1 * 20 + 17 * 10 + 12 * 5
5427: 1 * 200 + 0 * 100 + 1 * 50 + 1 * 20 + 18 * 10 + 10 * 5
5428: 1 * 200 + 0 * 100 + 1 * 50 + 1 * 20 + 19 * 10 + 8 * 5
5429: 1 * 200 + 0 * 100 + 1 * 50 + 1 * 20 + 20 * 10 + 6 * 5
5430: 1 * 200 + 0 * 100 + 1 * 50 + 1 * 20 + 21 * 10 + 4 * 5
5431: 1 * 200 + 0 * 100 + 1 * 50 + 1 * 20 + 22 * 10 + 2 * 5
5432: 1 * 200 + 0 * 100 + 1 * 50 + 1 * 20 + 23 * 10 + 0 * 5
5433: 1 * 200 + 0 * 100 + 1 * 50 + 2 * 20 + 0 * 10 + 42 * 5
5434: 1 * 200 + 0 * 100 + 1 * 50 + 2 * 20 + 1 * 10 + 40 * 5
5435: 1 * 200 + 0 * 100 + 1 * 50 + 2 * 20 + 2 * 10 + 38 * 5
5436: 1 * 200 + 0 * 100 + 1 * 50 + 2 * 20 + 3 * 10 + 36 * 5
5437: 1 * 200 + 0 * 100 + 1 * 50 + 2 * 20 + 4 * 10 + 34 * 5
5438: 1 * 200 + 0 * 100 + 1 * 50 + 2 * 20 + 5 * 10 + 32 * 5
5439: 1 * 200 + 0 * 100 + 1 * 50 + 2 * 20 + 6 * 10 + 30 * 5
5440: 1 * 200 + 0 * 100 + 1 * 50 + 2 * 20 + 7 * 10 + 28 * 5
5441: 1 * 200 + 0 * 100 + 1 * 50 + 2 * 20 + 8 * 10 + 26 * 5
5442: 1 * 200 + 0 * 100 + 1 * 50 + 2 * 20 + 9 * 10 + 24 * 5
5443: 1 * 200 + 0 * 100 + 1 * 50 + 2 * 20 + 10 * 10 + 22 * 5
5444: 1 * 200 + 0 * 100 + 1 * 50 + 2 * 20 + 11 * 10 + 20 * 5
5445: 1 * 200 + 0 * 100 + 1 * 50 + 2 * 20 + 12 * 10 + 18 * 5
5446: 1 * 200 + 0 * 100 + 1 * 50 + 2 * 20 + 13 * 10 + 16 * 5
5447: 1 * 200 + 0 * 100 + 1 * 50 + 2 * 20 + 14 * 10 + 14 * 5
5448: 1 * 200 + 0 * 100 + 1 * 50 + 2 * 20 + 15 * 10 + 12 * 5
5449: 1 * 200 + 0 * 100 + 1 * 50 + 2 * 20 + 16 * 10 + 10 * 5
5450: 1 * 200 + 0 * 100 + 1 * 50 + 2 * 20 + 17 * 10 + 8 * 5
5451: 1 * 200 + 0 * 100 + 1 * 50 + 2 * 20 + 18 * 10 + 6 * 5
5452: 1 * 200 + 0 * 100 + 1 * 50 + 2 * 20 + 19 * 10 + 4 * 5
5453: 1 * 200 + 0 * 100 + 1 * 50 + 2 * 20 + 20 * 10 + 2 * 5
5454: 1 * 200 + 0 * 100 + 1 * 50 + 2 * 20 + 21 * 10 + 0 * 5
5455: 1 * 200 + 0 * 100 + 1 * 50 + 3 * 20 + 0 * 10 + 38 * 5
5456: 1 * 200 + 0 * 100 + 1 * 50 + 3 * 20 + 1 * 10 + 36 * 5
5457: 1 * 200 + 0 * 100 + 1 * 50 + 3 * 20 + 2 * 10 + 34 * 5
5458: 1 * 200 + 0 * 100 + 1 * 50 + 3 * 20 + 3 * 10 + 32 * 5
5459: 1 * 200 + 0 * 100 + 1 * 50 + 3 * 20 + 4 * 10 + 30 * 5
5460: 1 * 200 + 0 * 100 + 1 * 50 + 3 * 20 + 5 * 10 + 28 * 5
5461: 1 * 200 + 0 * 100 + 1 * 50 + 3 * 20 + 6 * 10 + 26 * 5
5462: 1 * 200 + 0 * 100 + 1 * 50 + 3 * 20 + 7 * 10 + 24 * 5
5463: 1 * 200 + 0 * 100 + 1 * 50 + 3 * 20 + 8 * 10 + 22 * 5
5464: 1 * 200 + 0 * 100 + 1 * 50 + 3 * 20 + 9 * 10 + 20 * 5
5465: 1 * 200 + 0 * 100 + 1 * 50 + 3 * 20 + 10 * 10 + 18 * 5
5466: 1 * 200 + 0 * 100 + 1 * 50 + 3 * 20 + 11 * 10 + 16 * 5
5467: 1 * 200 + 0 * 100 + 1 * 50 + 3 * 20 + 12 * 10 + 14 * 5
5468: 1 * 200 + 0 * 100 + 1 * 50 + 3 * 20 + 13 * 10 + 12 * 5
5469: 1 * 200 + 0 * 100 + 1 * 50 + 3 * 20 + 14 * 10 + 10 * 5
5470: 1 * 200 + 0 * 100 + 1 * 50 + 3 * 20 + 15 * 10 + 8 * 5
5471: 1 * 200 + 0 * 100 + 1 * 50 + 3 * 20 + 16 * 10 + 6 * 5
5472: 1 * 200 + 0 * 100 + 1 * 50 + 3 * 20 + 17 * 10 + 4 * 5
5473: 1 * 200 + 0 * 100 + 1 * 50 + 3 * 20 + 18 * 10 + 2 * 5
5474: 1 * 200 + 0 * 100 + 1 * 50 + 3 * 20 + 19 * 10 + 0 * 5
5475: 1 * 200 + 0 * 100 + 1 * 50 + 4 * 20 + 0 * 10 + 34 * 5
5476: 1 * 200 + 0 * 100 + 1 * 50 + 4 * 20 + 1 * 10 + 32 * 5
5477: 1 * 200 + 0 * 100 + 1 * 50 + 4 * 20 + 2 * 10 + 30 * 5
5478: 1 * 200 + 0 * 100 + 1 * 50 + 4 * 20 + 3 * 10 + 28 * 5
5479: 1 * 200 + 0 * 100 + 1 * 50 + 4 * 20 + 4 * 10 + 26 * 5
5480: 1 * 200 + 0 * 100 + 1 * 50 + 4 * 20 + 5 * 10 + 24 * 5
5481: 1 * 200 + 0 * 100 + 1 * 50 + 4 * 20 + 6 * 10 + 22 * 5
5482: 1 * 200 + 0 * 100 + 1 * 50 + 4 * 20 + 7 * 10 + 20 * 5
5483: 1 * 200 + 0 * 100 + 1 * 50 + 4 * 20 + 8 * 10 + 18 * 5
5484: 1 * 200 + 0 * 100 + 1 * 50 + 4 * 20 + 9 * 10 + 16 * 5
5485: 1 * 200 + 0 * 100 + 1 * 50 + 4 * 20 + 10 * 10 + 14 * 5
5486: 1 * 200 + 0 * 100 + 1 * 50 + 4 * 20 + 11 * 10 + 12 * 5
5487: 1 * 200 + 0 * 100 + 1 * 50 + 4 * 20 + 12 * 10 + 10 * 5
5488: 1 * 200 + 0 * 100 + 1 * 50 + 4 * 20 + 13 * 10 + 8 * 5
5489: 1 * 200 + 0 * 100 + 1 * 50 + 4 * 20 + 14 * 10 + 6 * 5
5490: 1 * 200 + 0 * 100 + 1 * 50 + 4 * 20 + 15 * 10 + 4 * 5
5491: 1 * 200 + 0 * 100 + 1 * 50 + 4 * 20 + 16 * 10 + 2 * 5
5492: 1 * 200 + 0 * 100 + 1 * 50 + 4 * 20 + 17 * 10 + 0 * 5
5493: 1 * 200 + 0 * 100 + 1 * 50 + 5 * 20 + 0 * 10 + 30 * 5
5494: 1 * 200 + 0 * 100 + 1 * 50 + 5 * 20 + 1 * 10 + 28 * 5
5495: 1 * 200 + 0 * 100 + 1 * 50 + 5 * 20 + 2 * 10 + 26 * 5
5496: 1 * 200 + 0 * 100 + 1 * 50 + 5 * 20 + 3 * 10 + 24 * 5
5497: 1 * 200 + 0 * 100 + 1 * 50 + 5 * 20 + 4 * 10 + 22 * 5
5498: 1 * 200 + 0 * 100 + 1 * 50 + 5 * 20 + 5 * 10 + 20 * 5
5499: 1 * 200 + 0 * 100 + 1 * 50 + 5 * 20 + 6 * 10 + 18 * 5
5500: 1 * 200 + 0 * 100 + 1 * 50 + 5 * 20 + 7 * 10 + 16 * 5
5501: 1 * 200 + 0 * 100 + 1 * 50 + 5 * 20 + 8 * 10 + 14 * 5
5502: 1 * 200 + 0 * 100 + 1 * 50 + 5 * 20 + 9 * 10 + 12 * 5
5503: 1 * 200 + 0 * 100 + 1 * 50 + 5 * 20 + 10 * 10 + 10 * 5
5504: 1 * 200 + 0 * 100 + 1 * 50 + 5 * 20 + 11 * 10 + 8 * 5
5505: 1 * 200 + 0 * 100 + 1 * 50 + 5 * 20 + 12 * 10 + 6 * 5
5506: 1 * 200 + 0 * 100 + 1 * 50 + 5 * 20 + 13 * 10 + 4 * 5
5507: 1 * 200 + 0 * 100 + 1 * 50 + 5 * 20 + 14 * 10 + 2 * 5
5508: 1 * 200 + 0 * 100 + 1 * 50 + 5 * 20 + 15 * 10 + 0 * 5
5509: 1 * 200 + 0 * 100 + 1 * 50 + 6 * 20 + 0 * 10 + 26 * 5
5510: 1 * 200 + 0 * 100 + 1 * 50 + 6 * 20 + 1 * 10 + 24 * 5
5511: 1 * 200 + 0 * 100 + 1 * 50 + 6 * 20 + 2 * 10 + 22 * 5
5512: 1 * 200 + 0 * 100 + 1 * 50 + 6 * 20 + 3 * 10 + 20 * 5
5513: 1 * 200 + 0 * 100 + 1 * 50 + 6 * 20 + 4 * 10 + 18 * 5
5514: 1 * 200 + 0 * 100 + 1 * 50 + 6 * 20 + 5 * 10 + 16 * 5
5515: 1 * 200 + 0 * 100 + 1 * 50 + 6 * 20 + 6 * 10 + 14 * 5
5516: 1 * 200 + 0 * 100 + 1 * 50 + 6 * 20 + 7 * 10 + 12 * 5
5517: 1 * 200 + 0 * 100 + 1 * 50 + 6 * 20 + 8 * 10 + 10 * 5
5518: 1 * 200 + 0 * 100 + 1 * 50 + 6 * 20 + 9 * 10 + 8 * 5
5519: 1 * 200 + 0 * 100 + 1 * 50 + 6 * 20 + 10 * 10 + 6 * 5
5520: 1 * 200 + 0 * 100 + 1 * 50 + 6 * 20 + 11 * 10 + 4 * 5
5521: 1 * 200 + 0 * 100 + 1 * 50 + 6 * 20 + 12 * 10 + 2 * 5
5522: 1 * 200 + 0 * 100 + 1 * 50 + 6 * 20 + 13 * 10 + 0 * 5
5523: 1 * 200 + 0 * 100 + 1 * 50 + 7 * 20 + 0 * 10 + 22 * 5
5524: 1 * 200 + 0 * 100 + 1 * 50 + 7 * 20 + 1 * 10 + 20 * 5
5525: 1 * 200 + 0 * 100 + 1 * 50 + 7 * 20 + 2 * 10 + 18 * 5
5526: 1 * 200 + 0 * 100 + 1 * 50 + 7 * 20 + 3 * 10 + 16 * 5
5527: 1 * 200 + 0 * 100 + 1 * 50 + 7 * 20 + 4 * 10 + 14 * 5
5528: 1 * 200 + 0 * 100 + 1 * 50 + 7 * 20 + 5 * 10 + 12 * 5
5529: 1 * 200 + 0 * 100 + 1 * 50 + 7 * 20 + 6 * 10 + 10 * 5
5530: 1 * 200 + 0 * 100 + 1 * 50 + 7 * 20 + 7 * 10 + 8 * 5
5531: 1 * 200 + 0 * 100 + 1 * 50 + 7 * 20 + 8 * 10 + 6 * 5
5532: 1 * 200 + 0 * 100 + 1 * 50 + 7 * 20 + 9 * 10 + 4 * 5
5533: 1 * 200 + 0 * 100 + 1 * 50 + 7 * 20 + 10 * 10 + 2 * 5
5534: 1 * 200 + 0 * 100 + 1 * 50 + 7 * 20 + 11 * 10 + 0 * 5
5535: 1 * 200 + 0 * 100 + 1 * 50 + 8 * 20 + 0 * 10 + 18 * 5
5536: 1 * 200 + 0 * 100 + 1 * 50 + 8 * 20 + 1 * 10 + 16 * 5
5537: 1 * 200 + 0 * 100 + 1 * 50 + 8 * 20 + 2 * 10 + 14 * 5
5538: 1 * 200 + 0 * 100 + 1 * 50 + 8 * 20 + 3 * 10 + 12 * 5
5539: 1 * 200 + 0 * 100 + 1 * 50 + 8 * 20 + 4 * 10 + 10 * 5
5540: 1 * 200 + 0 * 100 + 1 * 50 + 8 * 20 + 5 * 10 + 8 * 5
5541: 1 * 200 + 0 * 100 + 1 * 50 + 8 * 20 + 6 * 10 + 6 * 5
5542: 1 * 200 + 0 * 100 + 1 * 50 + 8 * 20 + 7 * 10 + 4 * 5
5543: 1 * 200 + 0 * 100 + 1 * 50 + 8 * 20 + 8 * 10 + 2 * 5
5544: 1 * 200 + 0 * 100 + 1 * 50 + 8 * 20 + 9 * 10 + 0 * 5
5545: 1 * 200 + 0 * 100 + 1 * 50 + 9 * 20 + 0 * 10 + 14 * 5
5546: 1 * 200 + 0 * 100 + 1 * 50 + 9 * 20 + 1 * 10 + 12 * 5
5547: 1 * 200 + 0 * 100 + 1 * 50 + 9 * 20 + 2 * 10 + 10 * 5
5548: 1 * 200 + 0 * 100 + 1 * 50 + 9 * 20 + 3 * 10 + 8 * 5
5549: 1 * 200 + 0 * 100 + 1 * 50 + 9 * 20 + 4 * 10 + 6 * 5
5550: 1 * 200 + 0 * 100 + 1 * 50 + 9 * 20 + 5 * 10 + 4 * 5
5551: 1 * 200 + 0 * 100 + 1 * 50 + 9 * 20 + 6 * 10 + 2 * 5
5552: 1 * 200 + 0 * 100 + 1 * 50 + 9 * 20 + 7 * 10 + 0 * 5
5553: 1 * 200 + 0 * 100 + 1 * 50 + 10 * 20 + 0 * 10 + 10 * 5
5554: 1 * 200 + 0 * 100 + 1 * 50 + 10 * 20 + 1 * 10 + 8 * 5
5555: 1 * 200 + 0 * 100 + 1 * 50 + 10 * 20 + 2 * 10 + 6 * 5
5556: 1 * 200 + 0 * 100 + 1 * 50 + 10 * 20 + 3 * 10 + 4 * 5
5557: 1 * 200 + 0 * 100 + 1 * 50 + 10 * 20 + 4 * 10 + 2 * 5
5558: 1 * 200 + 0 * 100 + 1 * 50 + 10 * 20 + 5 * 10 + 0 * 5
5559: 1 * 200 + 0 * 100 + 1 * 50 + 11 * 20 + 0 * 10 + 6 * 5
5560: 1 * 200 + 0 * 100 + 1 * 50 + 11 * 20 + 1 * 10 + 4 * 5
5561: 1 * 200 + 0 * 100 + 1 * 50 + 11 * 20 + 2 * 10 + 2 * 5
5562: 1 * 200 + 0 * 100 + 1 * 50 + 11 * 20 + 3 * 10 + 0 * 5
5563: 1 * 200 + 0 * 100 + 1 * 50 + 12 * 20 + 0 * 10 + 2 * 5
5564: 1 * 200 + 0 * 100 + 1 * 50 + 12 * 20 + 1 * 10 + 0 * 5
5565: 1 * 200 + 0 * 100 + 2 * 50 + 0 * 20 + 0 * 10 + 40 * 5
5566: 1 * 200 + 0 * 100 + 2 * 50 + 0 * 20 + 1 * 10 + 38 * 5
5567: 1 * 200 + 0 * 100 + 2 * 50 + 0 * 20 + 2 * 10 + 36 * 5
5568: 1 * 200 + 0 * 100 + 2 * 50 + 0 * 20 + 3 * 10 + 34 * 5
5569: 1 * 200 + 0 * 100 + 2 * 50 + 0 * 20 + 4 * 10 + 32 * 5
5570: 1 * 200 + 0 * 100 + 2 * 50 + 0 * 20 + 5 * 10 + 30 * 5
5571: 1 * 200 + 0 * 100 + 2 * 50 + 0 * 20 + 6 * 10 + 28 * 5
5572: 1 * 200 + 0 * 100 + 2 * 50 + 0 * 20 + 7 * 10 + 26 * 5
5573: 1 * 200 + 0 * 100 + 2 * 50 + 0 * 20 + 8 * 10 + 24 * 5
5574: 1 * 200 + 0 * 100 + 2 * 50 + 0 * 20 + 9 * 10 + 22 * 5
5575: 1 * 200 + 0 * 100 + 2 * 50 + 0 * 20 + 10 * 10 + 20 * 5
5576: 1 * 200 + 0 * 100 + 2 * 50 + 0 * 20 + 11 * 10 + 18 * 5
5577: 1 * 200 + 0 * 100 + 2 * 50 + 0 * 20 + 12 * 10 + 16 * 5
5578: 1 * 200 + 0 * 100 + 2 * 50 + 0 * 20 + 13 * 10 + 14 * 5
5579: 1 * 200 + 0 * 100 + 2 * 50 + 0 * 20 + 14 * 10 + 12 * 5
5580: 1 * 200 + 0 * 100 + 2 * 50 + 0 * 20 + 15 * 10 + 10 * 5
5581: 1 * 200 + 0 * 100 + 2 * 50 + 0 * 20 + 16 * 10 + 8 * 5
5582: 1 * 200 + 0 * 100 + 2 * 50 + 0 * 20 + 17 * 10 + 6 * 5
5583: 1 * 200 + 0 * 100 + 2 * 50 + 0 * 20 + 18 * 10 + 4 * 5
5584: 1 * 200 + 0 * 100 + 2 * 50 + 0 * 20 + 19 * 10 + 2 * 5
5585: 1 * 200 + 0 * 100 + 2 * 50 + 0 * 20 + 20 * 10 + 0 * 5
5586: 1 * 200 + 0 * 100 + 2 * 50 + 1 * 20 + 0 * 10 + 36 * 5
5587: 1 * 200 + 0 * 100 + 2 * 50 + 1 * 20 + 1 * 10 + 34 * 5
5588: 1 * 200 + 0 * 100 + 2 * 50 + 1 * 20 + 2 * 10 + 32 * 5
5589: 1 * 200 + 0 * 100 + 2 * 50 + 1 * 20 + 3 * 10 + 30 * 5
5590: 1 * 200 + 0 * 100 + 2 * 50 + 1 * 20 + 4 * 10 + 28 * 5
5591: 1 * 200 + 0 * 100 + 2 * 50 + 1 * 20 + 5 * 10 + 26 * 5
5592: 1 * 200 + 0 * 100 + 2 * 50 + 1 * 20 + 6 * 10 + 24 * 5
5593: 1 * 200 + 0 * 100 + 2 * 50 + 1 * 20 + 7 * 10 + 22 * 5
5594: 1 * 200 + 0 * 100 + 2 * 50 + 1 * 20 + 8 * 10 + 20 * 5
5595: 1 * 200 + 0 * 100 + 2 * 50 + 1 * 20 + 9 * 10 + 18 * 5
5596: 1 * 200 + 0 * 100 + 2 * 50 + 1 * 20 + 10 * 10 + 16 * 5
5597: 1 * 200 + 0 * 100 + 2 * 50 + 1 * 20 + 11 * 10 + 14 * 5
5598: 1 * 200 + 0 * 100 + 2 * 50 + 1 * 20 + 12 * 10 + 12 * 5
5599: 1 * 200 + 0 * 100 + 2 * 50 + 1 * 20 + 13 * 10 + 10 * 5
5600: 1 * 200 + 0 * 100 + 2 * 50 + 1 * 20 + 14 * 10 + 8 * 5
5601: 1 * 200 + 0 * 100 + 2 * 50 + 1 * 20 + 15 * 10 + 6 * 5
5602: 1 * 200 + 0 * 100 + 2 * 50 + 1 * 20 + 16 * 10 + 4 * 5
5603: 1 * 200 + 0 * 100 + 2 * 50 + 1 * 20 + 17 * 10 + 2 * 5
5604: 1 * 200 + 0 * 100 + 2 * 50 + 1 * 20 + 18 * 10 + 0 * 5
5605: 1 * 200 + 0 * 100 + 2 * 50 + 2 * 20 + 0 * 10 + 32 * 5
5606: 1 * 200 + 0 * 100 + 2 * 50 + 2 * 20 + 1 * 10 + 30 * 5
5607: 1 * 200 + 0 * 100 + 2 * 50 + 2 * 20 + 2 * 10 + 28 * 5
5608: 1 * 200 + 0 * 100 + 2 * 50 + 2 * 20 + 3 * 10 + 26 * 5
5609: 1 * 200 + 0 * 100 + 2 * 50 + 2 * 20 + 4 * 10 + 24 * 5
5610: 1 * 200 + 0 * 100 + 2 * 50 + 2 * 20 + 5 * 10 + 22 * 5
5611: 1 * 200 + 0 * 100 + 2 * 50 + 2 * 20 + 6 * 10 + 20 * 5
5612: 1 * 200 + 0 * 100 + 2 * 50 + 2 * 20 + 7 * 10 + 18 * 5
5613: 1 * 200 + 0 * 100 + 2 * 50 + 2 * 20 + 8 * 10 + 16 * 5
5614: 1 * 200 + 0 * 100 + 2 * 50 + 2 * 20 + 9 * 10 + 14 * 5
5615: 1 * 200 + 0 * 100 + 2 * 50 + 2 * 20 + 10 * 10 + 12 * 5
5616: 1 * 200 + 0 * 100 + 2 * 50 + 2 * 20 + 11 * 10 + 10 * 5
5617: 1 * 200 + 0 * 100 + 2 * 50 + 2 * 20 + 12 * 10 + 8 * 5
5618: 1 * 200 + 0 * 100 + 2 * 50 + 2 * 20 + 13 * 10 + 6 * 5
5619: 1 * 200 + 0 * 100 + 2 * 50 + 2 * 20 + 14 * 10 + 4 * 5
5620: 1 * 200 + 0 * 100 + 2 * 50 + 2 * 20 + 15 * 10 + 2 * 5
5621: 1 * 200 + 0 * 100 + 2 * 50 + 2 * 20 + 16 * 10 + 0 * 5
5622: 1 * 200 + 0 * 100 + 2 * 50 + 3 * 20 + 0 * 10 + 28 * 5
5623: 1 * 200 + 0 * 100 + 2 * 50 + 3 * 20 + 1 * 10 + 26 * 5
5624: 1 * 200 + 0 * 100 + 2 * 50 + 3 * 20 + 2 * 10 + 24 * 5
5625: 1 * 200 + 0 * 100 + 2 * 50 + 3 * 20 + 3 * 10 + 22 * 5
5626: 1 * 200 + 0 * 100 + 2 * 50 + 3 * 20 + 4 * 10 + 20 * 5
5627: 1 * 200 + 0 * 100 + 2 * 50 + 3 * 20 + 5 * 10 + 18 * 5
5628: 1 * 200 + 0 * 100 + 2 * 50 + 3 * 20 + 6 * 10 + 16 * 5
5629: 1 * 200 + 0 * 100 + 2 * 50 + 3 * 20 + 7 * 10 + 14 * 5
5630: 1 * 200 + 0 * 100 + 2 * 50 + 3 * 20 + 8 * 10 + 12 * 5
5631: 1 * 200 + 0 * 100 + 2 * 50 + 3 * 20 + 9 * 10 + 10 * 5
5632: 1 * 200 + 0 * 100 + 2 * 50 + 3 * 20 + 10 * 10 + 8 * 5
5633: 1 * 200 + 0 * 100 + 2 * 50 + 3 * 20 + 11 * 10 + 6 * 5
5634: 1 * 200 + 0 * 100 + 2 * 50 + 3 * 20 + 12 * 10 + 4 * 5
5635: 1 * 200 + 0 * 100 + 2 * 50 + 3 * 20 + 13 * 10 + 2 * 5
5636: 1 * 200 + 0 * 100 + 2 * 50 + 3 * 20 + 14 * 10 + 0 * 5
5637: 1 * 200 + 0 * 100 + 2 * 50 + 4 * 20 + 0 * 10 + 24 * 5
5638: 1 * 200 + 0 * 100 + 2 * 50 + 4 * 20 + 1 * 10 + 22 * 5
5639: 1 * 200 + 0 * 100 + 2 * 50 + 4 * 20 + 2 * 10 + 20 * 5
5640: 1 * 200 + 0 * 100 + 2 * 50 + 4 * 20 + 3 * 10 + 18 * 5
5641: 1 * 200 + 0 * 100 + 2 * 50 + 4 * 20 + 4 * 10 + 16 * 5
5642: 1 * 200 + 0 * 100 + 2 * 50 + 4 * 20 + 5 * 10 + 14 * 5
5643: 1 * 200 + 0 * 100 + 2 * 50 + 4 * 20 + 6 * 10 + 12 * 5
5644: 1 * 200 + 0 * 100 + 2 * 50 + 4 * 20 + 7 * 10 + 10 * 5
5645: 1 * 200 + 0 * 100 + 2 * 50 + 4 * 20 + 8 * 10 + 8 * 5
5646: 1 * 200 + 0 * 100 + 2 * 50 + 4 * 20 + 9 * 10 + 6 * 5
5647: 1 * 200 + 0 * 100 + 2 * 50 + 4 * 20 + 10 * 10 + 4 * 5
5648: 1 * 200 + 0 * 100 + 2 * 50 + 4 * 20 + 11 * 10 + 2 * 5
5649: 1 * 200 + 0 * 100 + 2 * 50 + 4 * 20 + 12 * 10 + 0 * 5
5650: 1 * 200 + 0 * 100 + 2 * 50 + 5 * 20 + 0 * 10 + 20 * 5
5651: 1 * 200 + 0 * 100 + 2 * 50 + 5 * 20 + 1 * 10 + 18 * 5
5652: 1 * 200 + 0 * 100 + 2 * 50 + 5 * 20 + 2 * 10 + 16 * 5
5653: 1 * 200 + 0 * 100 + 2 * 50 + 5 * 20 + 3 * 10 + 14 * 5
5654: 1 * 200 + 0 * 100 + 2 * 50 + 5 * 20 + 4 * 10 + 12 * 5
5655: 1 * 200 + 0 * 100 + 2 * 50 + 5 * 20 + 5 * 10 + 10 * 5
5656: 1 * 200 + 0 * 100 + 2 * 50 + 5 * 20 + 6 * 10 + 8 * 5
5657: 1 * 200 + 0 * 100 + 2 * 50 + 5 * 20 + 7 * 10 + 6 * 5
5658: 1 * 200 + 0 * 100 + 2 * 50 + 5 * 20 + 8 * 10 + 4 * 5
5659: 1 * 200 + 0 * 100 + 2 * 50 + 5 * 20 + 9 * 10 + 2 * 5
5660: 1 * 200 + 0 * 100 + 2 * 50 + 5 * 20 + 10 * 10 + 0 * 5
5661: 1 * 200 + 0 * 100 + 2 * 50 + 6 * 20 + 0 * 10 + 16 * 5
5662: 1 * 200 + 0 * 100 + 2 * 50 + 6 * 20 + 1 * 10 + 14 * 5
5663: 1 * 200 + 0 * 100 + 2 * 50 + 6 * 20 + 2 * 10 + 12 * 5
5664: 1 * 200 + 0 * 100 + 2 * 50 + 6 * 20 + 3 * 10 + 10 * 5
5665: 1 * 200 + 0 * 100 + 2 * 50 + 6 * 20 + 4 * 10 + 8 * 5
5666: 1 * 200 + 0 * 100 + 2 * 50 + 6 * 20 + 5 * 10 + 6 * 5
5667: 1 * 200 + 0 * 100 + 2 * 50 + 6 * 20 + 6 * 10 + 4 * 5
5668: 1 * 200 + 0 * 100 + 2 * 50 + 6 * 20 + 7 * 10 + 2 * 5
5669: 1 * 200 + 0 * 100 + 2 * 50 + 6 * 20 + 8 * 10 + 0 * 5
5670: 1 * 200 + 0 * 100 + 2 * 50 + 7 * 20 + 0 * 10 + 12 * 5
5671: 1 * 200 + 0 * 100 + 2 * 50 + 7 * 20 + 1 * 10 + 10 * 5
5672: 1 * 200 + 0 * 100 + 2 * 50 + 7 * 20 + 2 * 10 + 8 * 5
5673: 1 * 200 + 0 * 100 + 2 * 50 + 7 * 20 + 3 * 10 + 6 * 5
5674: 1 * 200 + 0 * 100 + 2 * 50 + 7 * 20 + 4 * 10 + 4 * 5
5675: 1 * 200 + 0 * 100 + 2 * 50 + 7 * 20 + 5 * 10 + 2 * 5
5676: 1 * 200 + 0 * 100 + 2 * 50 + 7 * 20 + 6 * 10 + 0 * 5
5677: 1 * 200 + 0 * 100 + 2 * 50 + 8 * 20 + 0 * 10 + 8 * 5
5678: 1 * 200 + 0 * 100 + 2 * 50 + 8 * 20 + 1 * 10 + 6 * 5
5679: 1 * 200 + 0 * 100 + 2 * 50 + 8 * 20 + 2 * 10 + 4 * 5
5680: 1 * 200 + 0 * 100 + 2 * 50 + 8 * 20 + 3 * 10 + 2 * 5
5681: 1 * 200 + 0 * 100 + 2 * 50 + 8 * 20 + 4 * 10 + 0 * 5
5682: 1 * 200 + 0 * 100 + 2 * 50 + 9 * 20 + 0 * 10 + 4 * 5
5683: 1 * 200 + 0 * 100 + 2 * 50 + 9 * 20 + 1 * 10 + 2 * 5
5684: 1 * 200 + 0 * 100 + 2 * 50 + 9 * 20 + 2 * 10 + 0 * 5
5685: 1 * 200 + 0 * 100 + 2 * 50 + 10 * 20 + 0 * 10 + 0 * 5
5686: 1 * 200 + 0 * 100 + 3 * 50 + 0 * 20 + 0 * 10 + 30 * 5
5687: 1 * 200 + 0 * 100 + 3 * 50 + 0 * 20 + 1 * 10 + 28 * 5
5688: 1 * 200 + 0 * 100 + 3 * 50 + 0 * 20 + 2 * 10 + 26 * 5
5689: 1 * 200 + 0 * 100 + 3 * 50 + 0 * 20 + 3 * 10 + 24 * 5
5690: 1 * 200 + 0 * 100 + 3 * 50 + 0 * 20 + 4 * 10 + 22 * 5
5691: 1 * 200 + 0 * 100 + 3 * 50 + 0 * 20 + 5 * 10 + 20 * 5
5692: 1 * 200 + 0 * 100 + 3 * 50 + 0 * 20 + 6 * 10 + 18 * 5
5693: 1 * 200 + 0 * 100 + 3 * 50 + 0 * 20 + 7 * 10 + 16 * 5
5694: 1 * 200 + 0 * 100 + 3 * 50 + 0 * 20 + 8 * 10 + 14 * 5
5695: 1 * 200 + 0 * 100 + 3 * 50 + 0 * 20 + 9 * 10 + 12 * 5
5696: 1 * 200 + 0 * 100 + 3 * 50 + 0 * 20 + 10 * 10 + 10 * 5
5697: 1 * 200 + 0 * 100 + 3 * 50 + 0 * 20 + 11 * 10 + 8 * 5
5698: 1 * 200 + 0 * 100 + 3 * 50 + 0 * 20 + 12 * 10 + 6 * 5
5699: 1 * 200 + 0 * 100 + 3 * 50 + 0 * 20 + 13 * 10 + 4 * 5
5700: 1 * 200 + 0 * 100 + 3 * 50 + 0 * 20 + 14 * 10 + 2 * 5
5701: 1 * 200 + 0 * 100 + 3 * 50 + 0 * 20 + 15 * 10 + 0 * 5
5702: 1 * 200 + 0 * 100 + 3 * 50 + 1 * 20 + 0 * 10 + 26 * 5
5703: 1 * 200 + 0 * 100 + 3 * 50 + 1 * 20 + 1 * 10 + 24 * 5
5704: 1 * 200 + 0 * 100 + 3 * 50 + 1 * 20 + 2 * 10 + 22 * 5
5705: 1 * 200 + 0 * 100 + 3 * 50 + 1 * 20 + 3 * 10 + 20 * 5
5706: 1 * 200 + 0 * 100 + 3 * 50 + 1 * 20 + 4 * 10 + 18 * 5
5707: 1 * 200 + 0 * 100 + 3 * 50 + 1 * 20 + 5 * 10 + 16 * 5
5708: 1 * 200 + 0 * 100 + 3 * 50 + 1 * 20 + 6 * 10 + 14 * 5
5709: 1 * 200 + 0 * 100 + 3 * 50 + 1 * 20 + 7 * 10 + 12 * 5
5710: 1 * 200 + 0 * 100 + 3 * 50 + 1 * 20 + 8 * 10 + 10 * 5
5711: 1 * 200 + 0 * 100 + 3 * 50 + 1 * 20 + 9 * 10 + 8 * 5
5712: 1 * 200 + 0 * 100 + 3 * 50 + 1 * 20 + 10 * 10 + 6 * 5
5713: 1 * 200 + 0 * 100 + 3 * 50 + 1 * 20 + 11 * 10 + 4 * 5
5714: 1 * 200 + 0 * 100 + 3 * 50 + 1 * 20 + 12 * 10 + 2 * 5
5715: 1 * 200 + 0 * 100 + 3 * 50 + 1 * 20 + 13 * 10 + 0 * 5
5716: 1 * 200 + 0 * 100 + 3 * 50 + 2 * 20 + 0 * 10 + 22 * 5
5717: 1 * 200 + 0 * 100 + 3 * 50 + 2 * 20 + 1 * 10 + 20 * 5
5718: 1 * 200 + 0 * 100 + 3 * 50 + 2 * 20 + 2 * 10 + 18 * 5
5719: 1 * 200 + 0 * 100 + 3 * 50 + 2 * 20 + 3 * 10 + 16 * 5
5720: 1 * 200 + 0 * 100 + 3 * 50 + 2 * 20 + 4 * 10 + 14 * 5
5721: 1 * 200 + 0 * 100 + 3 * 50 + 2 * 20 + 5 * 10 + 12 * 5
5722: 1 * 200 + 0 * 100 + 3 * 50 + 2 * 20 + 6 * 10 + 10 * 5
5723: 1 * 200 + 0 * 100 + 3 * 50 + 2 * 20 + 7 * 10 + 8 * 5
5724: 1 * 200 + 0 * 100 + 3 * 50 + 2 * 20 + 8 * 10 + 6 * 5
5725: 1 * 200 + 0 * 100 + 3 * 50 + 2 * 20 + 9 * 10 + 4 * 5
5726: 1 * 200 + 0 * 100 + 3 * 50 + 2 * 20 + 10 * 10 + 2 * 5
5727: 1 * 200 + 0 * 100 + 3 * 50 + 2 * 20 + 11 * 10 + 0 * 5
5728: 1 * 200 + 0 * 100 + 3 * 50 + 3 * 20 + 0 * 10 + 18 * 5
5729: 1 * 200 + 0 * 100 + 3 * 50 + 3 * 20 + 1 * 10 + 16 * 5
5730: 1 * 200 + 0 * 100 + 3 * 50 + 3 * 20 + 2 * 10 + 14 * 5
5731: 1 * 200 + 0 * 100 + 3 * 50 + 3 * 20 + 3 * 10 + 12 * 5
5732: 1 * 200 + 0 * 100 + 3 * 50 + 3 * 20 + 4 * 10 + 10 * 5
5733: 1 * 200 + 0 * 100 + 3 * 50 + 3 * 20 + 5 * 10 + 8 * 5
5734: 1 * 200 + 0 * 100 + 3 * 50 + 3 * 20 + 6 * 10 + 6 * 5
5735: 1 * 200 + 0 * 100 + 3 * 50 + 3 * 20 + 7 * 10 + 4 * 5
5736: 1 * 200 + 0 * 100 + 3 * 50 + 3 * 20 + 8 * 10 + 2 * 5
5737: 1 * 200 + 0 * 100 + 3 * 50 + 3 * 20 + 9 * 10 + 0 * 5
5738: 1 * 200 + 0 * 100 + 3 * 50 + 4 * 20 + 0 * 10 + 14 * 5
5739: 1 * 200 + 0 * 100 + 3 * 50 + 4 * 20 + 1 * 10 + 12 * 5
5740: 1 * 200 + 0 * 100 + 3 * 50 + 4 * 20 + 2 * 10 + 10 * 5
5741: 1 * 200 + 0 * 100 + 3 * 50 + 4 * 20 + 3 * 10 + 8 * 5
5742: 1 * 200 + 0 * 100 + 3 * 50 + 4 * 20 + 4 * 10 + 6 * 5
5743: 1 * 200 + 0 * 100 + 3 * 50 + 4 * 20 + 5 * 10 + 4 * 5
5744: 1 * 200 + 0 * 100 + 3 * 50 + 4 * 20 + 6 * 10 + 2 * 5
5745: 1 * 200 + 0 * 100 + 3 * 50 + 4 * 20 + 7 * 10 + 0 * 5
5746: 1 * 200 + 0 * 100 + 3 * 50 + 5 * 20 + 0 * 10 + 10 * 5
5747: 1 * 200 + 0 * 100 + 3 * 50 + 5 * 20 + 1 * 10 + 8 * 5
5748: 1 * 200 + 0 * 100 + 3 * 50 + 5 * 20 + 2 * 10 + 6 * 5
5749: 1 * 200 + 0 * 100 + 3 * 50 + 5 * 20 + 3 * 10 + 4 * 5
5750: 1 * 200 + 0 * 100 + 3 * 50 + 5 * 20 + 4 * 10 + 2 * 5
5751: 1 * 200 + 0 * 100 + 3 * 50 + 5 * 20 + 5 * 10 + 0 * 5
5752: 1 * 200 + 0 * 100 + 3 * 50 + 6 * 20 + 0 * 10 + 6 * 5
5753: 1 * 200 + 0 * 100 + 3 * 50 + 6 * 20 + 1 * 10 + 4 * 5
5754: 1 * 200 + 0 * 100 + 3 * 50 + 6 * 20 + 2 * 10 + 2 * 5
5755: 1 * 200 + 0 * 100 + 3 * 50 + 6 * 20 + 3 * 10 + 0 * 5
5756: 1 * 200 + 0 * 100 + 3 * 50 + 7 * 20 + 0 * 10 + 2 * 5
5757: 1 * 200 + 0 * 100 + 3 * 50 + 7 * 20 + 1 * 10 + 0 * 5
5758: 1 * 200 + 0 * 100 + 4 * 50 + 0 * 20 + 0 * 10 + 20 * 5
5759: 1 * 200 + 0 * 100 + 4 * 50 + 0 * 20 + 1 * 10 + 18 * 5
5760: 1 * 200 + 0 * 100 + 4 * 50 + 0 * 20 + 2 * 10 + 16 * 5
5761: 1 * 200 + 0 * 100 + 4 * 50 + 0 * 20 + 3 * 10 + 14 * 5
5762: 1 * 200 + 0 * 100 + 4 * 50 + 0 * 20 + 4 * 10 + 12 * 5
5763: 1 * 200 + 0 * 100 + 4 * 50 + 0 * 20 + 5 * 10 + 10 * 5
5764: 1 * 200 + 0 * 100 + 4 * 50 + 0 * 20 + 6 * 10 + 8 * 5
5765: 1 * 200 + 0 * 100 + 4 * 50 + 0 * 20 + 7 * 10 + 6 * 5
5766: 1 * 200 + 0 * 100 + 4 * 50 + 0 * 20 + 8 * 10 + 4 * 5
5767: 1 * 200 + 0 * 100 + 4 * 50 + 0 * 20 + 9 * 10 + 2 * 5
5768: 1 * 200 + 0 * 100 + 4 * 50 + 0 * 20 + 10 * 10 + 0 * 5
5769: 1 * 200 + 0 * 100 + 4 * 50 + 1 * 20 + 0 * 10 + 16 * 5
5770: 1 * 200 + 0 * 100 + 4 * 50 + 1 * 20 + 1 * 10 + 14 * 5
5771: 1 * 200 + 0 * 100 + 4 * 50 + 1 * 20 + 2 * 10 + 12 * 5
5772: 1 * 200 + 0 * 100 + 4 * 50 + 1 * 20 + 3 * 10 + 10 * 5
5773: 1 * 200 + 0 * 100 + 4 * 50 + 1 * 20 + 4 * 10 + 8 * 5
5774: 1 * 200 + 0 * 100 + 4 * 50 + 1 * 20 + 5 * 10 + 6 * 5
5775: 1 * 200 + 0 * 100 + 4 * 50 + 1 * 20 + 6 * 10 + 4 * 5
5776: 1 * 200 + 0 * 100 + 4 * 50 + 1 * 20 + 7 * 10 + 2 * 5
5777: 1 * 200 + 0 * 100 + 4 * 50 + 1 * 20 + 8 * 10 + 0 * 5
5778: 1 * 200 + 0 * 100 + 4 * 50 + 2 * 20 + 0 * 10 + 12 * 5
5779: 1 * 200 + 0 * 100 + 4 * 50 + 2 * 20 + 1 * 10 + 10 * 5
5780: 1 * 200 + 0 * 100 + 4 * 50 + 2 * 20 + 2 * 10 + 8 * 5
5781: 1 * 200 + 0 * 100 + 4 * 50 + 2 * 20 + 3 * 10 + 6 * 5
5782: 1 * 200 + 0 * 100 + 4 * 50 + 2 * 20 + 4 * 10 + 4 * 5
5783: 1 * 200 + 0 * 100 + 4 * 50 + 2 * 20 + 5 * 10 + 2 * 5
5784: 1 * 200 + 0 * 100 + 4 * 50 + 2 * 20 + 6 * 10 + 0 * 5
5785: 1 * 200 + 0 * 100 + 4 * 50 + 3 * 20 + 0 * 10 + 8 * 5
5786: 1 * 200 + 0 * 100 + 4 * 50 + 3 * 20 + 1 * 10 + 6 * 5
5787: 1 * 200 + 0 * 100 + 4 * 50 + 3 * 20 + 2 * 10 + 4 * 5
5788: 1 * 200 + 0 * 100 + 4 * 50 + 3 * 20 + 3 * 10 + 2 * 5
5789: 1 * 200 + 0 * 100 + 4 * 50 + 3 * 20 + 4 * 10 + 0 * 5
5790: 1 * 200 + 0 * 100 + 4 * 50 + 4 * 20 + 0 * 10 + 4 * 5
5791: 1 * 200 + 0 * 100 + 4 * 50 + 4 * 20 + 1 * 10 + 2 * 5
5792: 1 * 200 + 0 * 100 + 4 * 50 + 4 * 20 + 2 * 10 + 0 * 5
5793: 1 * 200 + 0 * 100 + 4 * 50 + 5 * 20 + 0 * 10 + 0 * 5
5794: 1 * 200 + 0 * 100 + 5 * 50 + 0 * 20 + 0 * 10 + 10 * 5
5795: 1 * 200 + 0 * 100 + 5 * 50 + 0 * 20 + 1 * 10 + 8 * 5
5796: 1 * 200 + 0 * 100 + 5 * 50 + 0 * 20 + 2 * 10 + 6 * 5
5797: 1 * 200 + 0 * 100 + 5 * 50 + 0 * 20 + 3 * 10 + 4 * 5
5798: 1 * 200 + 0 * 100 + 5 * 50 + 0 * 20 + 4 * 10 + 2 * 5
5799: 1 * 200 + 0 * 100 + 5 * 50 + 0 * 20 + 5 * 10 + 0 * 5
5800: 1 * 200 + 0 * 100 + 5 * 50 + 1 * 20 + 0 * 10 + 6 * 5
5801: 1 * 200 + 0 * 100 + 5 * 50 + 1 * 20 + 1 * 10 + 4 * 5
5802: 1 * 200 + 0 * 100 + 5 * 50 + 1 * 20 + 2 * 10 + 2 * 5
5803: 1 * 200 + 0 * 100 + 5 * 50 + 1 * 20 + 3 * 10 + 0 * 5
5804: 1 * 200 + 0 * 100 + 5 * 50 + 2 * 20 + 0 * 10 + 2 * 5
5805: 1 * 200 + 0 * 100 + 5 * 50 + 2 * 20 + 1 * 10 + 0 * 5
5806: 1 * 200 + 0 * 100 + 6 * 50 + 0 * 20 + 0 * 10 + 0 * 5
5807: 1 * 200 + 1 * 100 + 0 * 50 + 0 * 20 + 0 * 10 + 40 * 5
5808: 1 * 200 + 1 * 100 + 0 * 50 + 0 * 20 + 1 * 10 + 38 * 5
5809: 1 * 200 + 1 * 100 + 0 * 50 + 0 * 20 + 2 * 10 + 36 * 5
5810: 1 * 200 + 1 * 100 + 0 * 50 + 0 * 20 + 3 * 10 + 34 * 5
5811: 1 * 200 + 1 * 100 + 0 * 50 + 0 * 20 + 4 * 10 + 32 * 5
5812: 1 * 200 + 1 * 100 + 0 * 50 + 0 * 20 + 5 * 10 + 30 * 5
5813: 1 * 200 + 1 * 100 + 0 * 50 + 0 * 20 + 6 * 10 + 28 * 5
5814: 1 * 200 + 1 * 100 + 0 * 50 + 0 * 20 + 7 * 10 + 26 * 5
5815: 1 * 200 + 1 * 100 + 0 * 50 + 0 * 20 + 8 * 10 + 24 * 5
5816: 1 * 200 + 1 * 100 + 0 * 50 + 0 * 20 + 9 * 10 + 22 * 5
5817: 1 * 200 + 1 * 100 + 0 * 50 + 0 * 20 + 10 * 10 + 20 * 5
5818: 1 * 200 + 1 * 100 + 0 * 50 + 0 * 20 + 11 * 10 + 18 * 5
5819: 1 * 200 + 1 * 100 + 0 * 50 + 0 * 20 + 12 * 10 + 16 * 5
5820: 1 * 200 + 1 * 100 + 0 * 50 + 0 * 20 + 13 * 10 + 14 * 5
5821: 1 * 200 + 1 * 100 + 0 * 50 + 0 * 20 + 14 * 10 + 12 * 5
5822: 1 * 200 + 1 * 100 + 0 * 50 + 0 * 20 + 15 * 10 + 10 * 5
5823: 1 * 200 + 1 * 100 + 0 * 50 + 0 * 20 + 16 * 10 + 8 * 5
5824: 1 * 200 + 1 * 100 + 0 * 50 + 0 * 20 + 17 * 10 + 6 * 5
5825: 1 * 200 + 1 * 100 + 0 * 50 + 0 * 20 + 18 * 10 + 4 * 5
5826: 1 * 200 + 1 * 100 + 0 * 50 + 0 * 20 + 19 * 10 + 2 * 5
5827: 1 * 200 + 1 * 100 + 0 * 50 + 0 * 20 + 20 * 10 + 0 * 5
5828: 1 * 200 + 1 * 100 + 0 * 50 + 1 * 20 + 0 * 10 + 36 * 5
5829: 1 * 200 + 1 * 100 + 0 * 50 + 1 * 20 + 1 * 10 + 34 * 5
5830: 1 * 200 + 1 * 100 + 0 * 50 + 1 * 20 + 2 * 10 + 32 * 5
5831: 1 * 200 + 1 * 100 + 0 * 50 + 1 * 20 + 3 * 10 + 30 * 5
5832: 1 * 200 + 1 * 100 + 0 * 50 + 1 * 20 + 4 * 10 + 28 * 5
5833: 1 * 200 + 1 * 100 + 0 * 50 + 1 * 20 + 5 * 10 + 26 * 5
5834: 1 * 200 + 1 * 100 + 0 * 50 + 1 * 20 + 6 * 10 + 24 * 5
5835: 1 * 200 + 1 * 100 + 0 * 50 + 1 * 20 + 7 * 10 + 22 * 5
5836: 1 * 200 + 1 * 100 + 0 * 50 + 1 * 20 + 8 * 10 + 20 * 5
5837: 1 * 200 + 1 * 100 + 0 * 50 + 1 * 20 + 9 * 10 + 18 * 5
5838: 1 * 200 + 1 * 100 + 0 * 50 + 1 * 20 + 10 * 10 + 16 * 5
5839: 1 * 200 + 1 * 100 + 0 * 50 + 1 * 20 + 11 * 10 + 14 * 5
5840: 1 * 200 + 1 * 100 + 0 * 50 + 1 * 20 + 12 * 10 + 12 * 5
5841: 1 * 200 + 1 * 100 + 0 * 50 + 1 * 20 + 13 * 10 + 10 * 5
5842: 1 * 200 + 1 * 100 + 0 * 50 + 1 * 20 + 14 * 10 + 8 * 5
5843: 1 * 200 + 1 * 100 + 0 * 50 + 1 * 20 + 15 * 10 + 6 * 5
5844: 1 * 200 + 1 * 100 + 0 * 50 + 1 * 20 + 16 * 10 + 4 * 5
5845: 1 * 200 + 1 * 100 + 0 * 50 + 1 * 20 + 17 * 10 + 2 * 5
5846: 1 * 200 + 1 * 100 + 0 * 50 + 1 * 20 + 18 * 10 + 0 * 5
5847: 1 * 200 + 1 * 100 + 0 * 50 + 2 * 20 + 0 * 10 + 32 * 5
5848: 1 * 200 + 1 * 100 + 0 * 50 + 2 * 20 + 1 * 10 + 30 * 5
5849: 1 * 200 + 1 * 100 + 0 * 50 + 2 * 20 + 2 * 10 + 28 * 5
5850: 1 * 200 + 1 * 100 + 0 * 50 + 2 * 20 + 3 * 10 + 26 * 5
5851: 1 * 200 + 1 * 100 + 0 * 50 + 2 * 20 + 4 * 10 + 24 * 5
5852: 1 * 200 + 1 * 100 + 0 * 50 + 2 * 20 + 5 * 10 + 22 * 5
5853: 1 * 200 + 1 * 100 + 0 * 50 + 2 * 20 + 6 * 10 + 20 * 5
5854: 1 * 200 + 1 * 100 + 0 * 50 + 2 * 20 + 7 * 10 + 18 * 5
5855: 1 * 200 + 1 * 100 + 0 * 50 + 2 * 20 + 8 * 10 + 16 * 5
5856: 1 * 200 + 1 * 100 + 0 * 50 + 2 * 20 + 9 * 10 + 14 * 5
5857: 1 * 200 + 1 * 100 + 0 * 50 + 2 * 20 + 10 * 10 + 12 * 5
5858: 1 * 200 + 1 * 100 + 0 * 50 + 2 * 20 + 11 * 10 + 10 * 5
5859: 1 * 200 + 1 * 100 + 0 * 50 + 2 * 20 + 12 * 10 + 8 * 5
5860: 1 * 200 + 1 * 100 + 0 * 50 + 2 * 20 + 13 * 10 + 6 * 5
5861: 1 * 200 + 1 * 100 + 0 * 50 + 2 * 20 + 14 * 10 + 4 * 5
5862: 1 * 200 + 1 * 100 + 0 * 50 + 2 * 20 + 15 * 10 + 2 * 5
5863: 1 * 200 + 1 * 100 + 0 * 50 + 2 * 20 + 16 * 10 + 0 * 5
5864: 1 * 200 + 1 * 100 + 0 * 50 + 3 * 20 + 0 * 10 + 28 * 5
5865: 1 * 200 + 1 * 100 + 0 * 50 + 3 * 20 + 1 * 10 + 26 * 5
5866: 1 * 200 + 1 * 100 + 0 * 50 + 3 * 20 + 2 * 10 + 24 * 5
5867: 1 * 200 + 1 * 100 + 0 * 50 + 3 * 20 + 3 * 10 + 22 * 5
5868: 1 * 200 + 1 * 100 + 0 * 50 + 3 * 20 + 4 * 10 + 20 * 5
5869: 1 * 200 + 1 * 100 + 0 * 50 + 3 * 20 + 5 * 10 + 18 * 5
5870: 1 * 200 + 1 * 100 + 0 * 50 + 3 * 20 + 6 * 10 + 16 * 5
5871: 1 * 200 + 1 * 100 + 0 * 50 + 3 * 20 + 7 * 10 + 14 * 5
5872: 1 * 200 + 1 * 100 + 0 * 50 + 3 * 20 + 8 * 10 + 12 * 5
5873: 1 * 200 + 1 * 100 + 0 * 50 + 3 * 20 + 9 * 10 + 10 * 5
5874: 1 * 200 + 1 * 100 + 0 * 50 + 3 * 20 + 10 * 10 + 8 * 5
5875: 1 * 200 + 1 * 100 + 0 * 50 + 3 * 20 + 11 * 10 + 6 * 5
5876: 1 * 200 + 1 * 100 + 0 * 50 + 3 * 20 + 12 * 10 + 4 * 5
5877: 1 * 200 + 1 * 100 + 0 * 50 + 3 * 20 + 13 * 10 + 2 * 5
5878: 1 * 200 + 1 * 100 + 0 * 50 + 3 * 20 + 14 * 10 + 0 * 5
5879: 1 * 200 + 1 * 100 + 0 * 50 + 4 * 20 + 0 * 10 + 24 * 5
5880: 1 * 200 + 1 * 100 + 0 * 50 + 4 * 20 + 1 * 10 + 22 * 5
5881: 1 * 200 + 1 * 100 + 0 * 50 + 4 * 20 + 2 * 10 + 20 * 5
5882: 1 * 200 + 1 * 100 + 0 * 50 + 4 * 20 + 3 * 10 + 18 * 5
5883: 1 * 200 + 1 * 100 + 0 * 50 + 4 * 20 + 4 * 10 + 16 * 5
5884: 1 * 200 + 1 * 100 + 0 * 50 + 4 * 20 + 5 * 10 + 14 * 5
5885: 1 * 200 + 1 * 100 + 0 * 50 + 4 * 20 + 6 * 10 + 12 * 5
5886: 1 * 200 + 1 * 100 + 0 * 50 + 4 * 20 + 7 * 10 + 10 * 5
5887: 1 * 200 + 1 * 100 + 0 * 50 + 4 * 20 + 8 * 10 + 8 * 5
5888: 1 * 200 + 1 * 100 + 0 * 50 + 4 * 20 + 9 * 10 + 6 * 5
5889: 1 * 200 + 1 * 100 + 0 * 50 + 4 * 20 + 10 * 10 + 4 * 5
5890: 1 * 200 + 1 * 100 + 0 * 50 + 4 * 20 + 11 * 10 + 2 * 5
5891: 1 * 200 + 1 * 100 + 0 * 50 + 4 * 20 + 12 * 10 + 0 * 5
5892: 1 * 200 + 1 * 100 + 0 * 50 + 5 * 20 + 0 * 10 + 20 * 5
5893: 1 * 200 + 1 * 100 + 0 * 50 + 5 * 20 + 1 * 10 + 18 * 5
5894: 1 * 200 + 1 * 100 + 0 * 50 + 5 * 20 + 2 * 10 + 16 * 5
5895: 1 * 200 + 1 * 100 + 0 * 50 + 5 * 20 + 3 * 10 + 14 * 5
5896: 1 * 200 + 1 * 100 + 0 * 50 + 5 * 20 + 4 * 10 + 12 * 5
5897: 1 * 200 + 1 * 100 + 0 * 50 + 5 * 20 + 5 * 10 + 10 * 5
5898: 1 * 200 + 1 * 100 + 0 * 50 + 5 * 20 + 6 * 10 + 8 * 5
5899: 1 * 200 + 1 * 100 + 0 * 50 + 5 * 20 + 7 * 10 + 6 * 5
5900: 1 * 200 + 1 * 100 + 0 * 50 + 5 * 20 + 8 * 10 + 4 * 5
5901: 1 * 200 + 1 * 100 + 0 * 50 + 5 * 20 + 9 * 10 + 2 * 5
5902: 1 * 200 + 1 * 100 + 0 * 50 + 5 * 20 + 10 * 10 + 0 * 5
5903: 1 * 200 + 1 * 100 + 0 * 50 + 6 * 20 + 0 * 10 + 16 * 5
5904: 1 * 200 + 1 * 100 + 0 * 50 + 6 * 20 + 1 * 10 + 14 * 5
5905: 1 * 200 + 1 * 100 + 0 * 50 + 6 * 20 + 2 * 10 + 12 * 5
5906: 1 * 200 + 1 * 100 + 0 * 50 + 6 * 20 + 3 * 10 + 10 * 5
5907: 1 * 200 + 1 * 100 + 0 * 50 + 6 * 20 + 4 * 10 + 8 * 5
5908: 1 * 200 + 1 * 100 + 0 * 50 + 6 * 20 + 5 * 10 + 6 * 5
5909: 1 * 200 + 1 * 100 + 0 * 50 + 6 * 20 + 6 * 10 + 4 * 5
5910: 1 * 200 + 1 * 100 + 0 * 50 + 6 * 20 + 7 * 10 + 2 * 5
5911: 1 * 200 + 1 * 100 + 0 * 50 + 6 * 20 + 8 * 10 + 0 * 5
5912: 1 * 200 + 1 * 100 + 0 * 50 + 7 * 20 + 0 * 10 + 12 * 5
5913: 1 * 200 + 1 * 100 + 0 * 50 + 7 * 20 + 1 * 10 + 10 * 5
5914: 1 * 200 + 1 * 100 + 0 * 50 + 7 * 20 + 2 * 10 + 8 * 5
5915: 1 * 200 + 1 * 100 + 0 * 50 + 7 * 20 + 3 * 10 + 6 * 5
5916: 1 * 200 + 1 * 100 + 0 * 50 + 7 * 20 + 4 * 10 + 4 * 5
5917: 1 * 200 + 1 * 100 + 0 * 50 + 7 * 20 + 5 * 10 + 2 * 5
5918: 1 * 200 + 1 * 100 + 0 * 50 + 7 * 20 + 6 * 10 + 0 * 5
5919: 1 * 200 + 1 * 100 + 0 * 50 + 8 * 20 + 0 * 10 + 8 * 5
5920: 1 * 200 + 1 * 100 + 0 * 50 + 8 * 20 + 1 * 10 + 6 * 5
5921: 1 * 200 + 1 * 100 + 0 * 50 + 8 * 20 + 2 * 10 + 4 * 5
5922: 1 * 200 + 1 * 100 + 0 * 50 + 8 * 20 + 3 * 10 + 2 * 5
5923: 1 * 200 + 1 * 100 + 0 * 50 + 8 * 20 + 4 * 10 + 0 * 5
5924: 1 * 200 + 1 * 100 + 0 * 50 + 9 * 20 + 0 * 10 + 4 * 5
5925: 1 * 200 + 1 * 100 + 0 * 50 + 9 * 20 + 1 * 10 + 2 * 5
5926: 1 * 200 + 1 * 100 + 0 * 50 + 9 * 20 + 2 * 10 + 0 * 5
5927: 1 * 200 + 1 * 100 + 0 * 50 + 10 * 20 + 0 * 10 + 0 * 5
5928: 1 * 200 + 1 * 100 + 1 * 50 + 0 * 20 + 0 * 10 + 30 * 5
5929: 1 * 200 + 1 * 100 + 1 * 50 + 0 * 20 + 1 * 10 + 28 * 5
5930: 1 * 200 + 1 * 100 + 1 * 50 + 0 * 20 + 2 * 10 + 26 * 5
5931: 1 * 200 + 1 * 100 + 1 * 50 + 0 * 20 + 3 * 10 + 24 * 5
5932: 1 * 200 + 1 * 100 + 1 * 50 + 0 * 20 + 4 * 10 + 22 * 5
5933: 1 * 200 + 1 * 100 + 1 * 50 + 0 * 20 + 5 * 10 + 20 * 5
5934: 1 * 200 + 1 * 100 + 1 * 50 + 0 * 20 + 6 * 10 + 18 * 5
5935: 1 * 200 + 1 * 100 + 1 * 50 + 0 * 20 + 7 * 10 + 16 * 5
5936: 1 * 200 + 1 * 100 + 1 * 50 + 0 * 20 + 8 * 10 + 14 * 5
5937: 1 * 200 + 1 * 100 + 1 * 50 + 0 * 20 + 9 * 10 + 12 * 5
5938: 1 * 200 + 1 * 100 + 1 * 50 + 0 * 20 + 10 * 10 + 10 * 5
5939: 1 * 200 + 1 * 100 + 1 * 50 + 0 * 20 + 11 * 10 + 8 * 5
5940: 1 * 200 + 1 * 100 + 1 * 50 + 0 * 20 + 12 * 10 + 6 * 5
5941: 1 * 200 + 1 * 100 + 1 * 50 + 0 * 20 + 13 * 10 + 4 * 5
5942: 1 * 200 + 1 * 100 + 1 * 50 + 0 * 20 + 14 * 10 + 2 * 5
5943: 1 * 200 + 1 * 100 + 1 * 50 + 0 * 20 + 15 * 10 + 0 * 5
5944: 1 * 200 + 1 * 100 + 1 * 50 + 1 * 20 + 0 * 10 + 26 * 5
5945: 1 * 200 + 1 * 100 + 1 * 50 + 1 * 20 + 1 * 10 + 24 * 5
5946: 1 * 200 + 1 * 100 + 1 * 50 + 1 * 20 + 2 * 10 + 22 * 5
5947: 1 * 200 + 1 * 100 + 1 * 50 + 1 * 20 + 3 * 10 + 20 * 5
5948: 1 * 200 + 1 * 100 + 1 * 50 + 1 * 20 + 4 * 10 + 18 * 5
5949: 1 * 200 + 1 * 100 + 1 * 50 + 1 * 20 + 5 * 10 + 16 * 5
5950: 1 * 200 + 1 * 100 + 1 * 50 + 1 * 20 + 6 * 10 + 14 * 5
5951: 1 * 200 + 1 * 100 + 1 * 50 + 1 * 20 + 7 * 10 + 12 * 5
5952: 1 * 200 + 1 * 100 + 1 * 50 + 1 * 20 + 8 * 10 + 10 * 5
5953: 1 * 200 + 1 * 100 + 1 * 50 + 1 * 20 + 9 * 10 + 8 * 5
5954: 1 * 200 + 1 * 100 + 1 * 50 + 1 * 20 + 10 * 10 + 6 * 5
5955: 1 * 200 + 1 * 100 + 1 * 50 + 1 * 20 + 11 * 10 + 4 * 5
5956: 1 * 200 + 1 * 100 + 1 * 50 + 1 * 20 + 12 * 10 + 2 * 5
5957: 1 * 200 + 1 * 100 + 1 * 50 + 1 * 20 + 13 * 10 + 0 * 5
5958: 1 * 200 + 1 * 100 + 1 * 50 + 2 * 20 + 0 * 10 + 22 * 5
5959: 1 * 200 + 1 * 100 + 1 * 50 + 2 * 20 + 1 * 10 + 20 * 5
5960: 1 * 200 + 1 * 100 + 1 * 50 + 2 * 20 + 2 * 10 + 18 * 5
5961: 1 * 200 + 1 * 100 + 1 * 50 + 2 * 20 + 3 * 10 + 16 * 5
5962: 1 * 200 + 1 * 100 + 1 * 50 + 2 * 20 + 4 * 10 + 14 * 5
5963: 1 * 200 + 1 * 100 + 1 * 50 + 2 * 20 + 5 * 10 + 12 * 5
5964: 1 * 200 + 1 * 100 + 1 * 50 + 2 * 20 + 6 * 10 + 10 * 5
5965: 1 * 200 + 1 * 100 + 1 * 50 + 2 * 20 + 7 * 10 + 8 * 5
5966: 1 * 200 + 1 * 100 + 1 * 50 + 2 * 20 + 8 * 10 + 6 * 5
5967: 1 * 200 + 1 * 100 + 1 * 50 + 2 * 20 + 9 * 10 + 4 * 5
5968: 1 * 200 + 1 * 100 + 1 * 50 + 2 * 20 + 10 * 10 + 2 * 5
5969: 1 * 200 + 1 * 100 + 1 * 50 + 2 * 20 + 11 * 10 + 0 * 5
5970: 1 * 200 + 1 * 100 + 1 * 50 + 3 * 20 + 0 * 10 + 18 * 5
5971: 1 * 200 + 1 * 100 + 1 * 50 + 3 * 20 + 1 * 10 + 16 * 5
5972: 1 * 200 + 1 * 100 + 1 * 50 + 3 * 20 + 2 * 10 + 14 * 5
5973: 1 * 200 + 1 * 100 + 1 * 50 + 3 * 20 + 3 * 10 + 12 * 5
5974: 1 * 200 + 1 * 100 + 1 * 50 + 3 * 20 + 4 * 10 + 10 * 5
5975: 1 * 200 + 1 * 100 + 1 * 50 + 3 * 20 + 5 * 10 + 8 * 5
5976: 1 * 200 + 1 * 100 + 1 * 50 + 3 * 20 + 6 * 10 + 6 * 5
5977: 1 * 200 + 1 * 100 + 1 * 50 + 3 * 20 + 7 * 10 + 4 * 5
5978: 1 * 200 + 1 * 100 + 1 * 50 + 3 * 20 + 8 * 10 + 2 * 5
5979: 1 * 200 + 1 * 100 + 1 * 50 + 3 * 20 + 9 * 10 + 0 * 5
5980: 1 * 200 + 1 * 100 + 1 * 50 + 4 * 20 + 0 * 10 + 14 * 5
5981: 1 * 200 + 1 * 100 + 1 * 50 + 4 * 20 + 1 * 10 + 12 * 5
5982: 1 * 200 + 1 * 100 + 1 * 50 + 4 * 20 + 2 * 10 + 10 * 5
5983: 1 * 200 + 1 * 100 + 1 * 50 + 4 * 20 + 3 * 10 + 8 * 5
5984: 1 * 200 + 1 * 100 + 1 * 50 + 4 * 20 + 4 * 10 + 6 * 5
5985: 1 * 200 + 1 * 100 + 1 * 50 + 4 * 20 + 5 * 10 + 4 * 5
5986: 1 * 200 + 1 * 100 + 1 * 50 + 4 * 20 + 6 * 10 + 2 * 5
5987: 1 * 200 + 1 * 100 + 1 * 50 + 4 * 20 + 7 * 10 + 0 * 5
5988: 1 * 200 + 1 * 100 + 1 * 50 + 5 * 20 + 0 * 10 + 10 * 5
5989: 1 * 200 + 1 * 100 + 1 * 50 + 5 * 20 + 1 * 10 + 8 * 5
5990: 1 * 200 + 1 * 100 + 1 * 50 + 5 * 20 + 2 * 10 + 6 * 5
5991: 1 * 200 + 1 * 100 + 1 * 50 + 5 * 20 + 3 * 10 + 4 * 5
5992: 1 * 200 + 1 * 100 + 1 * 50 + 5 * 20 + 4 * 10 + 2 * 5
5993: 1 * 200 + 1 * 100 + 1 * 50 + 5 * 20 + 5 * 10 + 0 * 5
5994: 1 * 200 + 1 * 100 + 1 * 50 + 6 * 20 + 0 * 10 + 6 * 5
5995: 1 * 200 + 1 * 100 + 1 * 50 + 6 * 20 + 1 * 10 + 4 * 5
5996: 1 * 200 + 1 * 100 + 1 * 50 + 6 * 20 + 2 * 10 + 2 * 5
5997: 1 * 200 + 1 * 100 + 1 * 50 + 6 * 20 + 3 * 10 + 0 * 5
5998: 1 * 200 + 1 * 100 + 1 * 50 + 7 * 20 + 0 * 10 + 2 * 5
5999: 1 * 200 + 1 * 100 + 1 * 50 + 7 * 20 + 1 * 10 + 0 * 5
6000: 1 * 200 + 1 * 100 + 2 * 50 + 0 * 20 + 0 * 10 + 20 * 5
6001: 1 * 200 + 1 * 100 + 2 * 50 + 0 * 20 + 1 * 10 + 18 * 5
6002: 1 * 200 + 1 * 100 + 2 * 50 + 0 * 20 + 2 * 10 + 16 * 5
6003: 1 * 200 + 1 * 100 + 2 * 50 + 0 * 20 + 3 * 10 + 14 * 5
6004: 1 * 200 + 1 * 100 + 2 * 50 + 0 * 20 + 4 * 10 + 12 * 5
6005: 1 * 200 + 1 * 100 + 2 * 50 + 0 * 20 + 5 * 10 + 10 * 5
6006: 1 * 200 + 1 * 100 + 2 * 50 + 0 * 20 + 6 * 10 + 8 * 5
6007: 1 * 200 + 1 * 100 + 2 * 50 + 0 * 20 + 7 * 10 + 6 * 5
6008: 1 * 200 + 1 * 100 + 2 * 50 + 0 * 20 + 8 * 10 + 4 * 5
6009: 1 * 200 + 1 * 100 + 2 * 50 + 0 * 20 + 9 * 10 + 2 * 5
6010: 1 * 200 + 1 * 100 + 2 * 50 + 0 * 20 + 10 * 10 + 0 * 5
6011: 1 * 200 + 1 * 100 + 2 * 50 + 1 * 20 + 0 * 10 + 16 * 5
6012: 1 * 200 + 1 * 100 + 2 * 50 + 1 * 20 + 1 * 10 + 14 * 5
6013: 1 * 200 + 1 * 100 + 2 * 50 + 1 * 20 + 2 * 10 + 12 * 5
6014: 1 * 200 + 1 * 100 + 2 * 50 + 1 * 20 + 3 * 10 + 10 * 5
6015: 1 * 200 + 1 * 100 + 2 * 50 + 1 * 20 + 4 * 10 + 8 * 5
6016: 1 * 200 + 1 * 100 + 2 * 50 + 1 * 20 + 5 * 10 + 6 * 5
6017: 1 * 200 + 1 * 100 + 2 * 50 + 1 * 20 + 6 * 10 + 4 * 5
6018: 1 * 200 + 1 * 100 + 2 * 50 + 1 * 20 + 7 * 10 + 2 * 5
6019: 1 * 200 + 1 * 100 + 2 * 50 + 1 * 20 + 8 * 10 + 0 * 5
6020: 1 * 200 + 1 * 100 + 2 * 50 + 2 * 20 + 0 * 10 + 12 * 5
6021: 1 * 200 + 1 * 100 + 2 * 50 + 2 * 20 + 1 * 10 + 10 * 5
6022: 1 * 200 + 1 * 100 + 2 * 50 + 2 * 20 + 2 * 10 + 8 * 5
6023: 1 * 200 + 1 * 100 + 2 * 50 + 2 * 20 + 3 * 10 + 6 * 5
6024: 1 * 200 + 1 * 100 + 2 * 50 + 2 * 20 + 4 * 10 + 4 * 5
6025: 1 * 200 + 1 * 100 + 2 * 50 + 2 * 20 + 5 * 10 + 2 * 5
6026: 1 * 200 + 1 * 100 + 2 * 50 + 2 * 20 + 6 * 10 + 0 * 5
6027: 1 * 200 + 1 * 100 + 2 * 50 + 3 * 20 + 0 * 10 + 8 * 5
6028: 1 * 200 + 1 * 100 + 2 * 50 + 3 * 20 + 1 * 10 + 6 * 5
6029: 1 * 200 + 1 * 100 + 2 * 50 + 3 * 20 + 2 * 10 + 4 * 5
6030: 1 * 200 + 1 * 100 + 2 * 50 + 3 * 20 + 3 * 10 + 2 * 5
6031: 1 * 200 + 1 * 100 + 2 * 50 + 3 * 20 + 4 * 10 + 0 * 5
6032: 1 * 200 + 1 * 100 + 2 * 50 + 4 * 20 + 0 * 10 + 4 * 5
6033: 1 * 200 + 1 * 100 + 2 * 50 + 4 * 20 + 1 * 10 + 2 * 5
6034: 1 * 200 + 1 * 100 + 2 * 50 + 4 * 20 + 2 * 10 + 0 * 5
6035: 1 * 200 + 1 * 100 + 2 * 50 + 5 * 20 + 0 * 10 + 0 * 5
6036: 1 * 200 + 1 * 100 + 3 * 50 + 0 * 20 + 0 * 10 + 10 * 5
6037: 1 * 200 + 1 * 100 + 3 * 50 + 0 * 20 + 1 * 10 + 8 * 5
6038: 1 * 200 + 1 * 100 + 3 * 50 + 0 * 20 + 2 * 10 + 6 * 5
6039: 1 * 200 + 1 * 100 + 3 * 50 + 0 * 20 + 3 * 10 + 4 * 5
6040: 1 * 200 + 1 * 100 + 3 * 50 + 0 * 20 + 4 * 10 + 2 * 5
6041: 1 * 200 + 1 * 100 + 3 * 50 + 0 * 20 + 5 * 10 + 0 * 5
6042: 1 * 200 + 1 * 100 + 3 * 50 + 1 * 20 + 0 * 10 + 6 * 5
6043: 1 * 200 + 1 * 100 + 3 * 50 + 1 * 20 + 1 * 10 + 4 * 5
6044: 1 * 200 + 1 * 100 + 3 * 50 + 1 * 20 + 2 * 10 + 2 * 5
6045: 1 * 200 + 1 * 100 + 3 * 50 + 1 * 20 + 3 * 10 + 0 * 5
6046: 1 * 200 + 1 * 100 + 3 * 50 + 2 * 20 + 0 * 10 + 2 * 5
6047: 1 * 200 + 1 * 100 + 3 * 50 + 2 * 20 + 1 * 10 + 0 * 5
6048: 1 * 200 + 1 * 100 + 4 * 50 + 0 * 20 + 0 * 10 + 0 * 5
6049: 1 * 200 + 2 * 100 + 0 * 50 + 0 * 20 + 0 * 10 + 20 * 5
6050: 1 * 200 + 2 * 100 + 0 * 50 + 0 * 20 + 1 * 10 + 18 * 5
6051: 1 * 200 + 2 * 100 + 0 * 50 + 0 * 20 + 2 * 10 + 16 * 5
6052: 1 * 200 + 2 * 100 + 0 * 50 + 0 * 20 + 3 * 10 + 14 * 5
6053: 1 * 200 + 2 * 100 + 0 * 50 + 0 * 20 + 4 * 10 + 12 * 5
6054: 1 * 200 + 2 * 100 + 0 * 50 + 0 * 20 + 5 * 10 + 10 * 5
6055: 1 * 200 + 2 * 100 + 0 * 50 + 0 * 20 + 6 * 10 + 8 * 5
6056: 1 * 200 + 2 * 100 + 0 * 50 + 0 * 20 + 7 * 10 + 6 * 5
6057: 1 * 200 + 2 * 100 + 0 * 50 + 0 * 20 + 8 * 10 + 4 * 5
6058: 1 * 200 + 2 * 100 + 0 * 50 + 0 * 20 + 9 * 10 + 2 * 5
6059: 1 * 200 + 2 * 100 + 0 * 50 + 0 * 20 + 10 * 10 + 0 * 5
6060: 1 * 200 + 2 * 100 + 0 * 50 + 1 * 20 + 0 * 10 + 16 * 5
6061: 1 * 200 + 2 * 100 + 0 * 50 + 1 * 20 + 1 * 10 + 14 * 5
6062: 1 * 200 + 2 * 100 + 0 * 50 + 1 * 20 + 2 * 10 + 12 * 5
6063: 1 * 200 + 2 * 100 + 0 * 50 + 1 * 20 + 3 * 10 + 10 * 5
6064: 1 * 200 + 2 * 100 + 0 * 50 + 1 * 20 + 4 * 10 + 8 * 5
6065: 1 * 200 + 2 * 100 + 0 * 50 + 1 * 20 + 5 * 10 + 6 * 5
6066: 1 * 200 + 2 * 100 + 0 * 50 + 1 * 20 + 6 * 10 + 4 * 5
6067: 1 * 200 + 2 * 100 + 0 * 50 + 1 * 20 + 7 * 10 + 2 * 5
6068: 1 * 200 + 2 * 100 + 0 * 50 + 1 * 20 + 8 * 10 + 0 * 5
6069: 1 * 200 + 2 * 100 + 0 * 50 + 2 * 20 + 0 * 10 + 12 * 5
6070: 1 * 200 + 2 * 100 + 0 * 50 + 2 * 20 + 1 * 10 + 10 * 5
6071: 1 * 200 + 2 * 100 + 0 * 50 + 2 * 20 + 2 * 10 + 8 * 5
6072: 1 * 200 + 2 * 100 + 0 * 50 + 2 * 20 + 3 * 10 + 6 * 5
6073: 1 * 200 + 2 * 100 + 0 * 50 + 2 * 20 + 4 * 10 + 4 * 5
6074: 1 * 200 + 2 * 100 + 0 * 50 + 2 * 20 + 5 * 10 + 2 * 5
6075: 1 * 200 + 2 * 100 + 0 * 50 + 2 * 20 + 6 * 10 + 0 * 5
6076: 1 * 200 + 2 * 100 + 0 * 50 + 3 * 20 + 0 * 10 + 8 * 5
6077: 1 * 200 + 2 * 100 + 0 * 50 + 3 * 20 + 1 * 10 + 6 * 5
6078: 1 * 200 + 2 * 100 + 0 * 50 + 3 * 20 + 2 * 10 + 4 * 5
6079: 1 * 200 + 2 * 100 + 0 * 50 + 3 * 20 + 3 * 10 + 2 * 5
6080: 1 * 200 + 2 * 100 + 0 * 50 + 3 * 20 + 4 * 10 + 0 * 5
6081: 1 * 200 + 2 * 100 + 0 * 50 + 4 * 20 + 0 * 10 + 4 * 5
6082: 1 * 200 + 2 * 100 + 0 * 50 + 4 * 20 + 1 * 10 + 2 * 5
6083: 1 * 200 + 2 * 100 + 0 * 50 + 4 * 20 + 2 * 10 + 0 * 5
6084: 1 * 200 + 2 * 100 + 0 * 50 + 5 * 20 + 0 * 10 + 0 * 5
6085: 1 * 200 + 2 * 100 + 1 * 50 + 0 * 20 + 0 * 10 + 10 * 5
6086: 1 * 200 + 2 * 100 + 1 * 50 + 0 * 20 + 1 * 10 + 8 * 5
6087: 1 * 200 + 2 * 100 + 1 * 50 + 0 * 20 + 2 * 10 + 6 * 5
6088: 1 * 200 + 2 * 100 + 1 * 50 + 0 * 20 + 3 * 10 + 4 * 5
6089: 1 * 200 + 2 * 100 + 1 * 50 + 0 * 20 + 4 * 10 + 2 * 5
6090: 1 * 200 + 2 * 100 + 1 * 50 + 0 * 20 + 5 * 10 + 0 * 5
6091: 1 * 200 + 2 * 100 + 1 * 50 + 1 * 20 + 0 * 10 + 6 * 5
6092: 1 * 200 + 2 * 100 + 1 * 50 + 1 * 20 + 1 * 10 + 4 * 5
6093: 1 * 200 + 2 * 100 + 1 * 50 + 1 * 20 + 2 * 10 + 2 * 5
6094: 1 * 200 + 2 * 100 + 1 * 50 + 1 * 20 + 3 * 10 + 0 * 5
6095: 1 * 200 + 2 * 100 + 1 * 50 + 2 * 20 + 0 * 10 + 2 * 5
6096: 1 * 200 + 2 * 100 + 1 * 50 + 2 * 20 + 1 * 10 + 0 * 5
6097: 1 * 200 + 2 * 100 + 2 * 50 + 0 * 20 + 0 * 10 + 0 * 5
6098: 1 * 200 + 3 * 100 + 0 * 50 + 0 * 20 + 0 * 10 + 0 * 5
6099: 2 * 200 + 0 * 100 + 0 * 50 + 0 * 20 + 0 * 10 + 20 * 5
6100: 2 * 200 + 0 * 100 + 0 * 50 + 0 * 20 + 1 * 10 + 18 * 5
6101: 2 * 200 + 0 * 100 + 0 * 50 + 0 * 20 + 2 * 10 + 16 * 5
6102: 2 * 200 + 0 * 100 + 0 * 50 + 0 * 20 + 3 * 10 + 14 * 5
6103: 2 * 200 + 0 * 100 + 0 * 50 + 0 * 20 + 4 * 10 + 12 * 5
6104: 2 * 200 + 0 * 100 + 0 * 50 + 0 * 20 + 5 * 10 + 10 * 5
6105: 2 * 200 + 0 * 100 + 0 * 50 + 0 * 20 + 6 * 10 + 8 * 5
6106: 2 * 200 + 0 * 100 + 0 * 50 + 0 * 20 + 7 * 10 + 6 * 5
6107: 2 * 200 + 0 * 100 + 0 * 50 + 0 * 20 + 8 * 10 + 4 * 5
6108: 2 * 200 + 0 * 100 + 0 * 50 + 0 * 20 + 9 * 10 + 2 * 5
6109: 2 * 200 + 0 * 100 + 0 * 50 + 0 * 20 + 10 * 10 + 0 * 5
6110: 2 * 200 + 0 * 100 + 0 * 50 + 1 * 20 + 0 * 10 + 16 * 5
6111: 2 * 200 + 0 * 100 + 0 * 50 + 1 * 20 + 1 * 10 + 14 * 5
6112: 2 * 200 + 0 * 100 + 0 * 50 + 1 * 20 + 2 * 10 + 12 * 5
6113: 2 * 200 + 0 * 100 + 0 * 50 + 1 * 20 + 3 * 10 + 10 * 5
6114: 2 * 200 + 0 * 100 + 0 * 50 + 1 * 20 + 4 * 10 + 8 * 5
6115: 2 * 200 + 0 * 100 + 0 * 50 + 1 * 20 + 5 * 10 + 6 * 5
6116: 2 * 200 + 0 * 100 + 0 * 50 + 1 * 20 + 6 * 10 + 4 * 5
6117: 2 * 200 + 0 * 100 + 0 * 50 + 1 * 20 + 7 * 10 + 2 * 5
6118: 2 * 200 + 0 * 100 + 0 * 50 + 1 * 20 + 8 * 10 + 0 * 5
6119: 2 * 200 + 0 * 100 + 0 * 50 + 2 * 20 + 0 * 10 + 12 * 5
6120: 2 * 200 + 0 * 100 + 0 * 50 + 2 * 20 + 1 * 10 + 10 * 5
6121: 2 * 200 + 0 * 100 + 0 * 50 + 2 * 20 + 2 * 10 + 8 * 5
6122: 2 * 200 + 0 * 100 + 0 * 50 + 2 * 20 + 3 * 10 + 6 * 5
6123: 2 * 200 + 0 * 100 + 0 * 50 + 2 * 20 + 4 * 10 + 4 * 5
6124: 2 * 200 + 0 * 100 + 0 * 50 + 2 * 20 + 5 * 10 + 2 * 5
6125: 2 * 200 + 0 * 100 + 0 * 50 + 2 * 20 + 6 * 10 + 0 * 5
6126: 2 * 200 + 0 * 100 + 0 * 50 + 3 * 20 + 0 * 10 + 8 * 5
6127: 2 * 200 + 0 * 100 + 0 * 50 + 3 * 20 + 1 * 10 + 6 * 5
6128: 2 * 200 + 0 * 100 + 0 * 50 + 3 * 20 + 2 * 10 + 4 * 5
6129: 2 * 200 + 0 * 100 + 0 * 50 + 3 * 20 + 3 * 10 + 2 * 5
6130: 2 * 200 + 0 * 100 + 0 * 50 + 3 * 20 + 4 * 10 + 0 * 5
6131: 2 * 200 + 0 * 100 + 0 * 50 + 4 * 20 + 0 * 10 + 4 * 5
6132: 2 * 200 + 0 * 100 + 0 * 50 + 4 * 20 + 1 * 10 + 2 * 5
6133: 2 * 200 + 0 * 100 + 0 * 50 + 4 * 20 + 2 * 10 + 0 * 5
6134: 2 * 200 + 0 * 100 + 0 * 50 + 5 * 20 + 0 * 10 + 0 * 5
6135: 2 * 200 + 0 * 100 + 1 * 50 + 0 * 20 + 0 * 10 + 10 * 5
6136: 2 * 200 + 0 * 100 + 1 * 50 + 0 * 20 + 1 * 10 + 8 * 5
6137: 2 * 200 + 0 * 100 + 1 * 50 + 0 * 20 + 2 * 10 + 6 * 5
6138: 2 * 200 + 0 * 100 + 1 * 50 + 0 * 20 + 3 * 10 + 4 * 5
6139: 2 * 200 + 0 * 100 + 1 * 50 + 0 * 20 + 4 * 10 + 2 * 5
6140: 2 * 200 + 0 * 100 + 1 * 50 + 0 * 20 + 5 * 10 + 0 * 5
6141: 2 * 200 + 0 * 100 + 1 * 50 + 1 * 20 + 0 * 10 + 6 * 5
6142: 2 * 200 + 0 * 100 + 1 * 50 + 1 * 20 + 1 * 10 + 4 * 5
6143: 2 * 200 + 0 * 100 + 1 * 50 + 1 * 20 + 2 * 10 + 2 * 5
6144: 2 * 200 + 0 * 100 + 1 * 50 + 1 * 20 + 3 * 10 + 0 * 5
6145: 2 * 200 + 0 * 100 + 1 * 50 + 2 * 20 + 0 * 10 + 2 * 5
6146: 2 * 200 + 0 * 100 + 1 * 50 + 2 * 20 + 1 * 10 + 0 * 5
6147: 2 * 200 + 0 * 100 + 2 * 50 + 0 * 20 + 0 * 10 + 0 * 5
6148: 2 * 200 + 1 * 100 + 0 * 50 + 0 * 20 + 0 * 10 + 0 * 5

Nimmt man noch 1 und zwei Eurostücke hinzu, dann sind 52525 Möglichkeiten, ohne 1-Eurostücke immerhin noch 29962.

This week's maths problem

Problem of the week

exercice de maths de la semaine, math problem of the week, problema di matematica della settimana, सप्ताह के गणित समस्या, математическая задача недели, Ejercicio de matemáticas semanal, 今週の数学問題, בעיה מתמטית של השבוע, مشكلة الرياضيات الأسبوع, 这个周的数学问题, Haftanın matematik problemi, temporäre Problem vun der Woch, μαθηματικό πρόβλημα της εβδομάδας, math tatizo la wiki,

On Friday of each week we will post a new maths challenge.
You may submit your solution by the following Thursday.
Each problem contains two different levels of difficulty. You will be awarded from 2 to 12 points for a full answer.
Each series contains 12 problems before the stage winner is determined.
Your score will be published here.

In each series we will give away 3 books as prices. The prices will be drawn by lot among the best ten participants of the series. The books are kindly sponsored by Buchdienst Rattei of Chemnitz.

Suggestions for problems are welcome.

Deadline is 18th. April 2024.


German version - Italian version - French version - Spanish version  Hungarian Version Russian version  --> 中文/Chinese <-- --> Ελληνικά   <--

Series 66

problem 783

 

Maria and Bernd have been given two bars of chocolate, which can easily be divided into 24 pieces each.
"Let's work out how many times we have to break the bar until we have all 24 pieces individually," said Maria to Bernd. "But as good mathematicians, we should divide it perfectly!"

783

Broken pieces must not be placed on top of each other or next to each other on the first board.
Bernd notes an example:
First broken edge vertically between 2 and 3
Second broken edge horizontally between 7 and 13
Bernd now has three square pieces.
Third break edge vertical between 4 and 5.
Break the fourth and fifth edges to make 6 equal small squares with 4 pieces of chocolate each. From the small squares you can obtain the individual pieces with 3 fractions each. Bernd therefore needed 1+1+ 1 +1 +1 +1 +6*3 = 23 divisions.
Surely that's better, right? How do you get by with fewer fractions? There are 4 blue points for finding a path with fewer than 23 fractions or showing that there must always be 23.
On the second board, overlapping and adjacent lines are allowed. Bernd uses a hot and very sharp knife to help with the breaking.
How many divisions can you manage with this type? Do you get 4 red points for finding the smallest possible number of divisions?

Deadline for solution is the 18th. April 2024.

 

 


-> Enigma <--

https://www.schulmodell.eu/images/stories/mathe/horst/raetsel.php

 

Send your solutions to Diese E-Mail-Adresse ist vor Spambots geschützt! Zur Anzeige muss JavaScript eingeschaltet sein!, if your answer contains attachments.

For text versions you can also use this form

--> here <--

Please give your full name so your points can be added to the score. If you would like to receive our weekly maths problem automatically you can

--> subscribe to our newsletter <-- .

Presently this newsletter is received by more than 2000 subscibers.

You can also send a paper copy of your solution as long as it is postmarked on or before the deadline.
adress:
Thomas Jahre
Paul-Jäkel-Straße 60
09113 Chemnitz
Germany


Links:

http://www.wurzel.org/

https://www.facebook.com/ArchimedesLab

http://people.missouristate.edu/lesreid/potw.html

post address:

Thomas Jahre
Chemnitzer Schulmodell
Stollberger Straße 25
09119 Chemnitz
Deutschland/Germany
QR-Code for this site
qrcodewochen

Serie-28

Serie 28

Aufgabe 1

325. Wertungsaufgabe
Bernd sitzt am Schreibtisch und studiert eine Karte. "Was hast du denn da?", fragt Mike. "Wir haben vor ein paar Wochen mit der Mathegruppe ein Suchspiel auf dem großen Waldspielspielplatz veranstaltet.Und das ist die "Schatzkarte". Hier nun die Zusammenfassung:"
Beim Verstecken haben Arno, Brit, Cecil und Dirk geholfen. Die 4 versteckten Gegenstände waren jeweils von einem der vier abgelegt worden. Je einer im Norden, Süden, Osten und Westen des Spielplatzes. Weil der Spielplatz so groß ist, waren die vier mit ihrem Fahrrad unterwegs. Es gab ein gelbes, rotes, blaues und ein silbernes Fahrrad. Versteckt wurden ein Messer, ein Buch, eine Lampe und ein Schuh.
Brit war im Süden, aber nicht mit dem Messer.
Das Messer wurde nicht von dem Fahrer/in des silbernen Rades versteckt.
Der Fahrer/in des gelben Rades versteckte das Buch.
Im Westen war die Lampe versteckt
Der Fahrer/in des roten Rades fuhr nach Norden, aber nicht mit dem Messer.
Dirk hat kein blaues Fahrrad.
Den Schuh versteckte Cecil.
Wer hat wo was versteckt und wem gehört welches Fahrrad? - 4 blaue Punkte.
Auf der Rückseite der Schatzkarte klebte noch ein Foto, auf dem 10 Würfel (mit 1, 2, .. 6 Punkten -- normale Würfel eines Spieles eben) nebeneinander zu sehen waren. (Von links nach rechts Würfel 1, Würfel 2, ..., Würfel 10)
Würfel 8 hatte weniger Punkte als Würfel 10, wobei Würfel 3 noch mal eine andere Punktzahl aufwies.
Nimmt man die Würfel 6 bis 10, so lassen sich vier davon so anordnen, das vier aufeinander folgende Zahlen zu finden sind.
Bei den Würfeln 1 bis 5 gibt es einen Pasch (zwei gleiche) mit Punktzahlen, die nicht bei den Würfeln 6 bis 10 zu finden sind.
Es gibt genau einmal die Punktzahl 3. Auch die Punktzahl 2 gibt es nur einmal, die ist auf dem Würfel Nummer 4. Auf den Würfeln 9, 7, 5 und 3 ist die Punktzahl 6 nicht zu finden. Betrachtet man die Würfel 5, 2 und 3 in dieser Reihenfolge, so nimmt deren Punktzahl immer zu, wobei die Punktezahl von Nummer 5 und Nummer 2 zusammen gerade die von der Nummer 3 ergibt. Erstaunlicherweise sind auf der Nummer 1 eine 1 und auf der Nummer 6 eine 6 zu sehen, bei den anderen Würfeln gibt es so eine Übereinstimmung nicht. Rechnet man die Punktezahl der ersten fünf Würfel zusammen, so ergeben sich genau 10 Punkte weniger als bei den Würfeln 6 bis 10.
Was zeigen die Würfel 1 bis 10 jeweils für eine Punktezahl? - 4 rote Punkte.

Lösung:
Hier die Lösungsvariante von Doreen N, danke:
blau:
-Eintragen der eindeutig zutreffenden Dinge mit einem + und der
eindeutig nicht zutreffenden Dinge mit einem - in einem Lösungsgitter
wie in der P.M.
-durch log. Kombination ergeben sich schnell weitere + und -
-die Lösung:
Arno-Ost-blaues Fahrrad-Messer
Brit-Süd-gelbes Fahrrad-Buch
Cecil-Nord-rotes Fahrrad-Schuh
Dirk-West-silbernes Fahrrad-Lampe

rot:
-hier habe ich mir eine Tabelle gemacht, wo ich links die möglichen
Punkte 1-6 und oben die Würfel 1-10 hingeschrieben und dann mit + und -
die einzelnen Varianten markiert habe
-Würfel 8 weniger Punkte als Würfel 10 -> Würfel 8 nicht 6 Punkte,
Würfel 10 nicht 1 Punkt
-auf Würfel 4 sind 2 Punkte, alle anderen Würfel haben keine 2 Punkte
-auf Würfel 3, 5, 7 und 9 keine 6 Punkte
-auf Würfel 1 ist 1 Punkt, auf Würfel 6 sind 6 Punkte, doch auf Würfel 3
keine 3, auf Würfel 4 keine 4 und auf
Würfel 5 keine 5 Punkte
-auf den Würfeln 2 und 3 mehr als 1 Punkt, da Würfel 5 noch eine
kleinere Punktzahl hat
->2 Möglichkeiten für Würfel 5, 2 und 3: 1+3=4  und 1+4=5
->Würfel 2 hat nicht 5 oder 6 Punkte, da Würfel 3 noch mehr Punkte hat
->Würfel 5 muss 1 Punkt haben ->das ist der gesuchte Pasch der Würfel
1-5 (mit der 1 von Würfel 1), da Würfel 2 und 3 keine gleiche Punktzahl
haben können ->die Würfel 7-9 haben nicht 1 Punkt
-Würfel 8 hat mind. 3 Punkte, Würfel 10 mind. 4 Punkte
-es soll genau 1x die Zahl 3 auftreten-> muss bei Würfel 7-9 sein, da
sich Würfel 6-10  zu 4 aufeinanderfolgenden Zahlen anordnen lassen
sollen und die Punkte 1 und 2 für die Würfel 6-10 wegfallen
->Würfel 2 keine 3 Punkte ->Würfel 2 hat 4 Punkte, Würfel 3 hat 5
Punkte (1+4=5)
->Würfel 8 und 10 keine 5 Punkte
-die 4 aufeinanderfolgenden Zahlen sind 3, 4, 5 und 6
-Summe Würfel 1-5: 1+4+5+2+1=13 -> Summe Würfel 6-10 muss 23 sein (13+10=23)
-2 Möglichkeiten: -Würfel 8 hat 3 Punkte- dann hat Würfel 10 4 oder 6 Punkte
-Würfel 8 hat 4 Punkte- dann hat Würfel
10 6 Punkte
-Würfel 10 kann nicht 6 Punkte haben, da dann die Summe 24 wäre
(3+4+5+6+6)-> Würfel 8 muss 3 und Würfel 10 muss 4 Punkte haben
-von den Würfeln 7 und 9 hat einer 5 Punkte->23-3-4-5-6=5
-> beide Würfel haben 5 Punkte
=>Lösung:
Würfel  1= 1 Punkt
Würfel  2= 4 Punkte
Würfel  3= 5 Punkte
Würfel  4= 2 Punkte
Würfel  5= 1 Punkt
Würfel  6= 6 Punkte
Würfel  7= 5 Punkte
Würfel  8= 3 Punkte
Würfel  9= 5 Punkte
Würfel10= 4 Punkte


 

Aufgabe 2

326. Wertungsaufgabe

"Hallo Mike, wie ich sehe, hast du die Mikadostäbe wieder mal aus dem Schrank geholt. Wollen wir eine Runde spielen?"; fragt Bernd. "Nachher sicher, jetzt bin ich gerade dabei zu zählen." Wie das?" "Nun, ich habe dieses quadratische Spielfeld (10 cm). Die 19 cm langen Mikadostäbe ragen also darüber hinaus. Wenn ich nun zwei Mikadostäbe nehme, wird das Quadrat in maximal 4 Flächen zerlegt. Auf den Mikadostäben selbst entstehen 4 Strecken innerhalb des Quadrates." "Alles klar".
Wie viele Flächen und Strecken erhält man maximal, wenn man 5 Mikadostäbe nimmt. (Besser gesagt, man nimmt einfach Geraden.) - 4 blaue Punkte. Wie lässt sich die maximale Zahl von Flächen und Strecken ausrechnen, wenn man n Geraden nimmt? - 4 rote Punkte

 

“Hi Mike, I see you got your Mikado pick-up-sticks out of the cabinet. Do you want to play?”, Bernd asks. “Maybe later. Right now I'm counting.” “What are you counting?” “Well, there's this square game board that measures 10cm. These 19 cm long pick-up-sticks extend over the edges. With two pick-up-sticks I can divide the board into no more than 4 areas. The pick-up-sticks would make 4 line segments within the square, right?” “Right”
How many areas and line segments would you get at most if you used 5 pick-up-sticks? - 4 blue points.
A general solution for n pick-up-sticks? - 4 red points

Lösung:

blau/rot:

Am besten man geht es systematisch an.

Zuerst hat man das leere Quadrat: 0 Geraden, 0 Streckenabschnitte und 1 Fläche

Nun eine Gerade: 1 Streckenabschnitt und zwei Fflächen

Die zweite Gerade muss die erste schneiden (darf nicht parallel sein), damit die Streckenabschnitts- und Flächenzahlen maximal werden.

2  Geraden, 4 Streckenabschnitte und 4 Flächen.

3 Geraden (dritte Gerade schneidet die beiden schon vorhandenen, aber nicht in deren Schnittpunkt) 9 Abschnitte und 7 Flächen.

4 Geraden (Anmerkung s. o.) 16 Abschnitte und 11 Flächen

5 Geraden (...) 25 Abschnitte und 16 Flächen

...

n Geraden. n² Abschnitte und (1  + 2 + 3 + 4 + ... + n) +1 Flächen =  \frac {n(n+1)}{2} + 1

 


Aufgabe 3

327. Wertungsaufgabe

"Schau mal, ich habe hier ein zauberhaftes gleichschenkliges Trapez ABCD.", sagte Maria zu Bernd. "Was ist daran zauberhaft?" "Dieses Trapez wird von der Diagonalen AC genau in zwei gleichschenklige Dreiecke zerlegt." "Cool". Wie groß sind die Winkel in diesem Trapez (AB > CD, AC || CD)? - 4 blaue Punkte. Wie groß ist der Flächeninhalt des Trapezes, wenn die Diagonale 5,0 cm groß ist? - 4 rote Punkte.

Bild eines "passenden" Trapezes:

327

 

english version

„Look at my magical isosceles trapezoid ABCD.“, Maria says to Bernd. “Why magical?” “Well this trapezoid is divided into two isosceles triangles by its diagonal AC.” “Cool” What measure do the angles in this trapezoid have (AB > CD, AC || CD)? - 4 blue points. What is the area of this trapezoid if the diagonal is 5,0 cm? - 4 red points.

Lösung:

Trapez Aufgabe 327 1
das Bild in groß

Da das Trapez ABCD gleichschenklig sein soll gilt α + β = γ. Wegen CD kürzer AB muss ζ ein stumpfer Winkel sein. Daraus folgt CD = DA (gleichschenkliges Dreieck). Dann muss entsprechend AC = AB sein. Wenn aber AC = AB, dann ist  γ = δ. Ebenso gilt dann  β =  ε.  Zugleich ist aber α ein gleichgroßer Wechselwinkel zu  ε. Also sind α und  β gleich groß und damit halb so groß wie γ bzw. 2α = γ. Im Dreieck ABC wird dann aus α + γ + δ = 180°

5α = 180° also α = 36° Nun ist der Rest einfach α + β = 72°, γ = 72°, δ + ε = 108° und ζ = 108°

Kennt man die Winkel und AC = e mit 5,0 cm lässt sich mittels Sinussatz die Seite c zu 3,09 cm ausrechnen. ( d = c = b s.o.)

A = 0,5 ab sin γ + 0,5 cd sin ζ = 19,9 cm²

Für jüngere Schüler: Es darf auch die Höhe und c des Trapezes "gemessen" werden  und dann die Flächeninhaltsformel für das Trapez Anwendung finden, wenn die Konstruktion des Trapezes gelungen ist.

 


Aufgabe 4

328. Wertungsaufgabe

"Also, auf die Lösung der Aufgabe der letzten Woche zu kommen, war gar nicht so leicht", meinte Bernds Opa. "Das stimmt schon", gab Bernd zu. "Nun, dann probiert mal das aus".
Es wurden viele Schüler befragt. Genau 3,4 % von denen sagten, dass Grün ihre Lieblingsfarbe sei. Wie viele Schüler wurden mindestens befragt? - 3 blaue Punkte.
Bei einer anderen Umfrage sagten genau 4,7029 der befragten Schüler, dass Blau die Lieblingsfarbe sei. Wie viele Schüler waren es in diesem Falle mindestens? 4 rote Punkte.

 

english version

 

“Well, the solution of last week's problem wasn't all that easy, was it?” Bernd's granddad said.

“No, it wasn't”, Bernd agreed. “But what do you think about this one here?”

A lot of students had responded to a survey. Exactly 3,4% stated that their favourite colour was green. How many students had at least taken part in the survey? - 3 blue points

In another survey exactly 4,7029 of the students answered that blue was their favourite colour. How many students took part in this survey at the minimum? - 4 red points

 

Lösung/solution:

blau 3,4 %, das heißt  \frac{3,4}{100} = \frac{34}{1000} Es könnte also 34 von 1000 Personen gewesen sein. Da aber nach der Minimalzahl gefragt war muss der bruch gekürzt werden.  \frac{34}{1000} = \frac{17}{500} mehr kürzen geht nicht, also waren 500 Personen an der Umfrage beteiligt, von denen sich 17 für grün entschieden hatten.

rot: Hier gilt es einen periodischen Dezimalbruch auf einen gemeinen Bruch zuführen. Dafür gibt es einen schönen "Trick". Der Dezimalbruch wird mit 10n multipliziert, wobei n die Länge der Periode ist. In unserem Fall wäre das 10 000.

x =0,047029   die 0,04 ist wegen der Prozentangabe. (s.o. 3,4 % --> 0,034

10 000x = 470,297029s   von dieser Zahl wird das x abgezogen, was nun geht, da ab 3. Stelle nach dem Komma immer die gleichen Ziffern untereinander stehen (- wenn man sich die Subtraktion schriftlich vor Augen führt)

Es bleiben 9999x = 470,25 Jetzt geht es wie bei blau weiter:

 \frac{470,25}{9999} = \frac{47025}{999900}

Konsequentes Kürzen führt zur Antwort: Es waren 19 von 404, die sich für die Lieblingsfarbe blau entschieden hatten.

 

 


Aufgabe 5

329. Wertungsaufgabe

Aufgabe 329

329 kBernd hat im Urlaub eine interessante Pflasterung gesehen und sogar ein Bild davon gemacht. „Die Pflastersteine sehen aus, als wären sie mit dem Zirkel gemacht“, meinte Lisa. „Das denke ich auch,“ fand Mike. Wie lässt sich ein solcher Stein – eine solche Figur - konstruieren, wenn nur ein Zirkel (Zirkelspanne 5 cm) verwendet werden darf. Ein Lineal darf genommen werden, aber nicht zum Abmessen. 4 blaue Punkte für eine komplette Konstruktionsbeschreibung. 5 rote Punkte für die Berechnung von Umfang und Flächeninhalt einer solcher Figur.

großes Bild

 

 

 

 

english version

329 kWhile on holiday Bernd found this interesting kind of paving and even
took a picture of it.
“These paving stones look as if drawn with a compass”, Lisa observed. “I
think so, too,” Mike replies.
How can you construct such a stone – such a plane figure – if you may
only use a compass at a radius of 5 cm? 4 blue points for a complete
description of the construction. 5 red points for calculating
circumference and area of such a figure.

click to enlarge

 

 

 

Lösung/solution:

Hier die Lösung von Linus-Valentin, danke:

als pdf


 

Aufgabe 6

330. Wertungsaufgabe
„Bernd, stell dir vor, ich habe heute zum ersten Mal einen Fünfhundert-Euroschein in der Hand gehabt“; sagte Mike. „Cool, wie kam das?“ Wir hatten heute Hauswirtschaftsunterricht und unser Schulleiter holte beim Thema Geld plötzlich so einen Schein aus der Tasche .“ „Nicht schlecht, aber bevor man damit einkauft, wird man den wohl eher wechseln müssen.“ „Bestimmt.“
Wie viele Möglichkeiten gibt es, einen 500 Euroschein in Scheine zu wechseln, wenn  keine 5 und 10 Euro Scheine dabei sind? - 6 blaue Punkte. Wie viel Geld kann man maximal in der Geldbörse haben, wenn man  keinen von zwei 500 Euroscheinen (oder beide) genau wechseln kann. Bei diesem Teil der Aufgabe gibt es keine Einschränkung bei den Scheinen und Stücken. - 5 rote Punkte.

english version

“Bernd, guess what, I got hold of a 500 Euro banknote for the first time today”, Mike said.
“Great, how did that come about?” “We talked about money in our home economics lesson today. Our head teacher took the banknote out of his pocket just like that.”
“Wow, but before you could go shopping with this one you'd have to make change for it, wouldn't you?”
“I guess so.” How many possibilities are there to change a 500 Euro note into smaller notes except 5 Euro and 10 Euro notes? - 6 blue points. How much money would be in your purse at the maximum if you weren't able to exactly make change for one or both of two 500 Euro notes? You'd be able to use any common note or coin. - 5 red points.

 

Lösung/solution:

Hier Lösung von Elisa und Uwe, danke.

blau:
durch systematisches Ausprobieren gefunden
Scheine mit jeweiliger Anzahl (34 Varianten)
200|100| 50| 20
  2|  1|   |   
  2|   |  2|   
  2|   |   |  5
  1|  3|   |   
  1|  2|  2|   
  1|  2|  2|  5
  1|  1|  4|   
  1|  1|  2|  5
  1|  1|   | 10
  1|   |  6|   
  1|   |  4|  5
  1|   |  2| 10
  1|   |   | 15
   |  5|   |   
   |  4|  2|   
   |  4|   |  5
   |  3|  4|   
   |  3|  2|  5
   |  3|   | 10
   |  2|  6|   
   |  2|  4|  5
   |  2|  2| 10
   |  2|   | 15
   |  1|  8|   
   |  1|  6|  5
   |  1|  4| 10
   |  1|  2| 15
   |  1|   | 20
   |   | 10|   
   |   |  8|  5
   |   |  6| 10
   |   |  4| 15
   |   |  2| 20
   |   |   | 25

Werden auch 5 und 10 Euroscheine zugelassen, so gibt es 1648 Möglichkeiten einen 500 Euroschein zu wechseln.

--> komplette Übersicht <--

rot:
100 10 und 1 € sowie 10 und 1 Cent können nicht dabeisein, da es sonst mit den 2-ern und 5-ern glatte 100 10 oder 1 ergeben. Somit bleiben die folgenden max. Beträge in der Geldbörse.
4*200,00
0*100,00
1* 50,00
4* 20,00
0* 10,00
1*  5,00
4*  2,00
0*  1,00
1*  0,50
4*  0,20
0*  0,10
1*  0,05
4*  0,02
0*  0,01
--------
  944,43 Euro


Aufgabe 7

331. Wochenaufgabe

„Dass es so viel Geld ist, hätte ich auf Anhieb nicht vermutet“, meinte Bernds Vater, als er von der roten Aufgabe der letzten Woche hörte. „Aber schaut euch mal diese Zeichnung an. Was seht Ihr?“ „Ein gleichschenkliges Dreieck auf dessen Basislänge (6 cm) ein Rechteck steht. Außerdem liegen die Ecken des Rechtecks auch auf den Schenkeln (8 cm) des Dreiecks.“ „Stimmt, außerdem ist der Flächeninhalt des Rechtecks genau halb so groß wie der des Dreiecks.“  4 blaue Punkte gibt es für die Konstruktionsbeschreibung und -begründung für ein solches Bild, wie es Bernds Vater in der Hand hält. Für 5 rote Punkte soll überlegt werden, wie groß das Volumen eines Körpers ist, wenn das Bild aus dem blauen Teil den Aufriss eines Körpers darstellt, wobei des „Rechteck“ gestrichelt zu zeichnen ist. (Eine Lösung für rot reicht.)

english version
“I wouldn't have thought it such a lot of money”, Bernd's father said when he learned about last weeks maths problem.
“But take a look at this drawing. What do you see?”
“An isosceles triangle whose base (6cm) is also a side of a rectangle. Furthermore the vertices of the rectangle lie on the two equal sides of
the triangle (8 cm).” “Right, plus the area of the rectangle is exactly half as large as the  triangles' area.” -4 four blue points for explaining how to construct
the figure that Bernd's father is holding. For five red points find out the volume of a solid figure of which the drawing above is the front view in a first angle projection. The sides of the rectangle would be drawn in a dashed line as they are hidden. (one solution sufficient)

 

Lösung/solution:

Konstruktion: Zuerst zeichne ich die Strecke AB mit 6 cm. Dann wird von A 331 und B jeweils ein Kreisbogen mit dem Radius 8 cm gezogen. Die Kreisbögen schneiden sich in zwei Punkten, einen davon bezeichne ich mit C und erhalte so das gesuchte Dreieck ABC. Die beiden Schenkel des Dreiecks werden halbiert. (Grundkonstruktion). Die Halbierungspunkte bezeichne ich mit E bzw. F. EF ist eine Seite des gesuchten Rechtecks, dessen Punkte mittels Lotfällung auf AB gefunden werden (Grundkonstruktion).

Begründung: Das Dreieck EFC ist zum Dreieck ABC ähnlich und halb so groß wie ABC. (Hauptähnlichkeitssatz). Der Flächeninhalt von EFC ist demzufolge 1/4 so groß wie der vom Dreieck ABC. Die beiden Dreiecke sind jeweils kongruent zu den Teildreiecken des Dreiecks EFC (Kongruenzsatz sws). Damit haben die vier Teildrecke den halben Flächeninhalt des Dreiecks ABC - der verbleibende Rest - das Rechteck  hat dann auch den halben Flächeninhalt.

rot: Hier gibt es viele Varianten, wie der Körper aussehen kann, der einen solchen Aufriss hat.

Bei den eingereichten Lösungen waren folgende Varianten dabei:

"Liegendes" dreiseitiges Prisma hinter dem ein Quader steht.

Eine Pyramide bzw. Kegel hinter dem ein Prisma steht.

Ein Kegel mit zylindrischer Bohrung ( V in dem Fall rund 69,8 cm³).

Eine quadratische Pyramide mit - gemäß Aufriss maximal großem - harausgefrästem Quader - (V rund 88,9 cm³.)

Weitere Lösungen sind denkbar.

 


Aufgabe 8

332. Wertungsaufgabe

„Hallo Mike, hast du schon mal was von Teilermonstern gehört?“ „Nein. Was soll denn das sein?“, fragte Bernd zurück. „Teilermonster sind natürliche Zahlen, die kleiner als 100 sind und die mehr als 10 echte Teiler haben.“ „Echte Teiler?“ „Die echten Teiler einer natürlichen Zahl sind die Zahlen, durch die sich die Zahl ohne Rest teilen lässt und die kleiner sind als die Zahl selbst.“ „Ich verstehe, hast du mal ein Beispiel für ein solches Teilermonster?“ „Ich denke, die findest du selber heraus.“ Für jedes gefundene Teilermonster gibt es 2 blaue Punkte. Für den Nachweis, dass es nicht mehr geben kann, gibt es noch einmal 2 blaue Punkte dazu. Wie kann man Zahlen, die genau 1 Million echte Teiler haben erhalten? 4 rote Punkte. Welche der Zahlen davon ist die kleinste, noch mal 4 rote Punkte. (Anmerkung in beiden Aufgabenteilen sind auch bei den Teilern nur natürliche Zahlen zugelassen.)

english version

"Hi Mike, have you ever heard of a divisor monsters?" "No I haven't. What are they supposed to be?", Bernd asked back.  "Well, divisor monsters are natural numbers smaller than 100 that have more than 10 proper divisors." "Proper divisors?" "Proper divisors are divisors of our natural number except the number
itself." "I got it. Can you give an example for such a divisor monster?" "I think you'll find out yourself." Two blue points awarded for each divisor monster. Two extra blue points for proving that there cannot be more. How can you obtain numbers that have exactly a million proper divisors?  - 4 red points. Which of them is the smallest? - another 4 red points.

 

Lösung/solution:

Die angestellten Überlegungen beziehen sich auch natüliche Zahlen - die zu Teilenden und deren Teiler.

 blau: Die Anzahl der Teiler einer Zahl zu finden ist --> hier <-- möglich. Die gesuchte "Teilermonster" sind die 60; 72; 84; 90 und 96. Es waren also 10 + 2 Punkte möglich. Im Fall der roten Aufgabe hilft ein Probieren nicht wirklich. Es gilt also zu überlegen (oder das Internet zu durchforsten) wie man die Anzahl der Teiler einer Zahl ermittelt.

Zunächst werden mal alle Teiler betrachtet:

8 - die Teiler sind 1; 2; 4 und 8, die 9 - die Teiler sind 1; 3 und 9 und nun die 72 = 8 * 9 - die Teiler sind 1; 2; 3; 4; 6; 8; 9; 12; 18; 24 und 72. Wie die 72 zeigt, ergibt sich deren Teilerzahl als Produkt der Teilerzahlen der Teiler von 8 und 9. Wie ist das nun bei 8 und 9 selber? 8 = 2³, 9 = 3². Die Teilerzahlen sind also gerade um eines größer wie die Exponenten. Für die 72 gilt 72 = 2³ * 3² die Anzahl der aller Teiler ist (3+1) * (2+1). Untersucht man nun die Primfaktorenzerleung von weiteren Beispielen so wird schnell die Vermutung "sichtbar", dass sich aus den Exponenten der der Primfaktoren, die Anzahl der Teiler ergibt sich durch Multiplation der um 1 vergrößerten Exponeten der Primfaktoren. Den Beweis dafür zu führen versage ich mir hier an dieser Stelle - soltte  mir jemand einen  Beweis schicken, dann veröffentliche ich den gern..

Zurück zu rot:. 21 000 00, 31 000 000 , p1 000 000 (mit p - Primzahl) haben 1 000001 Teiler bzw. 1 000 000 echte Teiler. Ist aber die erste Potenz die gesuchte kleinste Zahl? Dazu wird die 1 000 001 selbst auf Teiler untersucht. Es gilt nun, dass sich 1 000 001 als Produkt zweier Primzahlen zerlegen lässt. 1 000 001 = 9901 * 301. Daraus lässt sich eines Primfaktorenzerlegung ableiten, wobei die kleinsten Primfaktoren verwendet werden. ==>

x= 29900 * 3300 hat (9901*301) = 1 000 001 Teiler also genau 1 000 000 echte Teiler. Kleiner geht es nicht mit 3026 Stellen ist diese Zahl deutlich kleiner als 21 000 000.

Hier die Zahl: (Wenn der Rechner sich nicht vertan hat)

1292661278285391124272494567449911759342464444

0739149688243384201912861092532479344764283716

8968041067932109434442887935225466936757703492

6112322219975060263711523401505257782740801461

4258111456459865865506197255696626543735016697

5827896227422242038893557334263031267060476985

4075546746515244172682062225441966202985937865

3099231518144964820788023271879376709229714001

3764084809867978658529510006739111260756994408

9753390582057475682412280783083768946807490929

9000664612677438957122792816864210497114924506

0089883426395487923407270960045255641647938032

9208748907908718846547490771883119854009123766

6815094391582099789787493869000841436243301869

6421302399983783308291748517878603797508280810

3592342664629707478277022594516669131826353275

0459721128588809917629235359648280312833018171

0005794933671775247756138188251624261970375922

6988699534948582076989827395743394552711940761

1791835113914989049445355770027730619497767298

0260669670504016837925740693887617569601454449

9276972581093665191312721052278115251917840412

4367673524404737384465224825341893626881230552

7328546968260714278168002228798074070244850752

1448037763512227361867983577393931218817871606

8573514747014611783070636603674328894476364087

7212374315815637219043197462389200952885926657

9294886672056642598330791102060763880145716884

6817829240151748449185016464622353545085833144

8232450831171270512658011301624047707179100907

8863684723118799209868590058339658522283599533

2634099521419107989219435595978464547519372674

3976428276818052570708793901794521381482542498

4879429123871955274287260074066384765768153095

3571237381992576851971755679864442236932315574

3553964179274470995613358968675803185263137800

6009494042638737851900602957054452092374106763

3812502161463633563226859497489016352558021557

6342219705779937785183739035135418519564473397

6995481399575426692932647049279758753841383324

8534404118621763044825149248093010681184506625

3906677940390586129860662663261838257031821879

1829855829360446772254199838552816041125544691

2079419423629436335127647492268709248230959027

1987318290500539130336892390963045258893769021

0288681252528543920038283845127657376576107630

7645851486729818280282002790779126589001089780

5211540898818980390450620707432895768493996506

2592176539488428059880715870138305953273031677

07066112523842723172130044052316518822861726721

73627053374078772975094492118247356337837787247

41146321329452330740862897780387121956885498221

78320460297300210866001754782938760060670651245

24816066220978249238575937850061273219687200337

89035477866076008916400687505888171614425501441

82085475396350323652648187055770228433908438167

36586314253203879525356975185649877821479286178

705928865041066448124318543086610072164875721440

480793606958451301165327463838036868796492090594

156317822009033686099313564426117237155623275759

0929894032650950005407047923775494731793964405265

2721638169573371657537839181682610205505569577900

793414578928661323091913547582724636229556907345

107444682183191447782123143587839071886838105212

403280892251314180176077121173328454494010902214

0862493152232879649101234124719725439336735732918

022273963249675195006138159820649149726668829546

544948576256

Ob Mersenne oder seine Mitstreiter diese Zahl wirklich ausgerechnet haben oder ob sie sich mit der Primfaktorendarstellung "zufrieden" gegeben haben, ist mir nicht bekannt.


Aufgabe 9

333. Wertungsaufgabe

„Bernd, ist dir eigentlich schon aufgefallen, dass es diese Aufgaben der Woche schon mehr als 10 Jahre gibt?“, fragte Mike. „Na logisch.“ „Apropos logisch, hast du eine Ahnung wie das gehen soll?“
Dr. Bunt aus Dresden hat einen Bruder in Chemnitz, der Professor für Informatik ist. Der Professor aber hat keinen Bruder in Dresden, der Doktor ist. Wie das wohl geht? – 2 blaue Punkte.
In Lisas Klasse kamen zu Beginn des Schuljahres zwei neue Schülerinnen, die sich zum Verwechseln ähnlich sahen. Lisa war sich sicher, dass es sich um Zwillinge handeln musste, aber das war nicht so. Die beiden neuen Schülerinnen haben die selben Eltern, sind gleich alt und haben am gleichen Tag Geburtstag. Wie geht das? - 2 rote Punkte
english version
“Bernd, do you know that the weekly maths challenge has already been running for over 10 years?”, Mike asked. “Of course I know that.”
“Speaking of your knowledge, you wouldn't be able to help me with this one, would you?”
Dr. Bunt from Dresden has a brother in Chemnitz who is a professor of Computer Science. This professor however, hasn't got a brother in Dresden who is a doctor. How can that be? - Two blue points.
At the beginning of the new school year two there were two new girls in Lisa's form who looked very much alike. Lisa was sure they had to be twins, however, they were not. The new girls have the same parents, are of the same age and their birthday is at the same day. How can that be? - two red points.

Lösung/solution:

 blau: einfachste Lösung Dr. Bunt ist eine Fau.

rot: Für etwas Verwirrung hat die Formulierung die gleichen bzw. die selben Eltern gesorgt. Also im Falle der selben Eltern ist die einfachste Lösung: Es handelt sich nicht um Zwilinnge, sondern um Drillinge (Vierlinge, ...), von denen eben nur zwei in der Klasse sind, warum auch immer. (Krank, andere Klasse, andere Schule, ...)


Aufgabe 10:

334. Wertungsaufgabe

„Eigentlich war ja die letzte Aufgabe nicht schwer, aber man muss eben darauf kommen“, sagte Lisa. „Da wird es mit der Aufgabe für euren Mathematikclub sicher einfacher sein.“ Lass hören:
Wie viele dreistellige Zahlen gibt es, deren Ziffern alle verschieden sind. Außerdem sollen sich benachbarte Ziffern der Zahl um 1 unterscheiden. (Ausnahme: Es darf aber eine Neun neben einer Null stehen.) 4 blaue Punkte. Wie viele solcher Zahlen mit beliebiger Stellenzahl gibt es? 8 rote Punkte.

english version:
Last week's maths problem wasn't exactly difficult if you know how to solve it”, Lisa said. “I guess the problem of your Maths Club will be more challenging. Let me hear.”

How many three-digit numbers are there whose digits are all different and differ by 1 in adjacent pairs (with the exception of 9 and 0)? - 4 blue points. How many of these numbers of any length are there? - 8 red points.

Lösung/solution:

Am besten man macht es gleich systemtisch:

1. Man nimmt einstellige Zahlen dann erhält man natürlich 10. (Einige Teilnehmer haben die weggelassen, weil von Ziffern die Rede war, aber kein Problem)

2. Nun betrachtet man zweistellige Zahlen und nimmt dazu, damit man nichts vergisst die 10 Ziffern aus 1. als "letzte" Stelle, denn die Null als erste geht eigentlich nicht. Beginnend mit der Null als letzte Ziffer ergeben:

10; 90; 21; 12; 32; 23; 43; 34; 54; 45; 65; 56; 76; 67; 87; 78; 98; 89; Es sind 18 Möglichkeiten. 

3. dreistellige Zahlen (blau): Das Verfahren von der letzten Ziffer auszugehen kann auch hier wieder angewandt werden.

Damit kommt man natürlich auch auf die zweistelligen, wobei die 2 und die 8 am Ende nur jeweils einmal genommen werden können, dafür aber jetzt die 1 und die 9 je zweimal an der letzten Stelle Verwendung finden. Es sind also wieder 18 Zahlen.

210; 890; 321; 901; 012; 432; 123; 543; 234; 654; 345; 765; 456; 876; 567; 987; 678; 098; 789; 109;

4. Die Ersetzungsmöglichkeiten sind bei vierstelligen Zahlen analog --> wieder 18 Möglichkeiten. Damit sind es insgesamt 172 = 10 + 9*18 Zahlen, die so gebildet werden können.

 

 


Aufgabe 11

335. Wertungsaufgabe

„Gestern haben wir eine merkwürdige Runde Bücherdrehen spielen müssen.“ „Bücherdrehen?“ „Wir sind 25 Schüler in der Klasse. Der Mathematiklehrer hatte unsere 25 Mathebücher mit der Rückseite nach oben in eine Reihe gelegt (Buch 1 bis 25). Wir stellten uns auch in eine Reihe. Die Aufgabe war nicht schwer. Der erste Schüler drehte alle Bücher nach oben. Der zweite drehte alle zweiten Bücher herum. Der dritte drehte alle Bücher herum, deren Nummer durch 3 teilbar war. Na ja bis eben der letzte Schüler noch das 25. herumdrehte. Allerdings mussten wir das zweimal machen, denn der Lehrer sah am Ende mit einem Blick, dass sich welche nicht konzentriert der Aufgabe gestellt hatten.“ Welche Bücher müssen am Ende mit der Seite nach oben liegen? – 4 blaue Punkte. Wenn sich alle 400 Schüler der Schule beteiligen, welche Bücher sind es dann und warum?– 6 rote Punkte

english version
“Yesterday we had to play a strange game of book turning.” “Book turning?” “We're 25 students in class. Our Maths teacher put our 25 Maths books face down in a row (book 1 to 25). We stood in a row, too. The task wasn't too difficult. The fist student turned all books face up. The second turned every second book. The third turned all books whose number could be divided by 3. And so on until the last student turned the 25th book. We had to go through that twice, though, because in the end our teacher instantly saw that not everyone had worked with the necessary concentration.”
How many books should be face up in the end? - 4 blue points. How many books should be face up if all 400 students of the school took part. With explanation – 6 red points.

Lösung/solution:

Hier die Lösung von Doreen N., danke:

blau:
-Buch 1 wird  1x umgedreht (nur bei Schüler 1), liegt also  mit der Vorderseite nach oben
-die Bücher 2,3,5,7,11,13,17,19 und 23 werden 2x umgedreht (bei Schüler 1 und der jeweiligen Buch-Nr.), liegen also wieder mit der Rückseite
nach oben
-die Bücher 4,9 und 25 werden 3x umgedreht, liegen also  mit der Vorderseite nach oben
-die Bücher 6,8,10,14,15,21 und 22 werden 4x umgedreht, liegen also wieder mit der Rückseite nach oben
-das Buch 16 wird 5x umgedreht (bei Schüler 1,2,4,8,16), liegt also mit der Vorderseite nach oben
-die Bücher 12,18 und 20 werden  6x umgedreht, liegen also wieder mit der Rückseite nach oben
-das Buch 24 wird  8x umgedreht (bei Schüler 1,2,3,4,6,8,12,24), liegt also wieder mit der Rückseite nach oben
->nur die Bücher 1, 4, 9, 16 und 25 haben die Vorderseite oben

rot:
Man erkennt bei blau schon, nach welchem System dieses "Spiel" funktioniert:
-wenn das Buch eine gerade Anzahl mal umgedreht wird, liegt es am Ende wieder mit der Rückseite nach oben, bei einer ungeraden Anzahl ist die
Vorderseite oben
-Buch 1 wird immer nur 1x umgedreht, hat also die Vorderseite oben
-die Buchnummern, die Primzahlen sind, haben immer 2 Teiler (die 1 und die Zahl selbst), werden also 2x umgedreht und haben die Rückseite oben
-bei den anderen Büchern geht es nach der Anzahl der Teiler, die meisten haben eine gerade Anzahl an Teilern, werden also eine gerade Zahl mal
umgedreht und haben die Rückseite oben
-Ausnahme sind die Buchnummern, die eine Quadratzahl darstellen, z.B. 4=2² oder 64=8², diese haben eine ungerade Anzahl an Teilern, werden
also eine ungerade Zahl mal umgedreht und haben deshalb die Vorderseite oben
-für unsere 400 Bücher bedeutet das: die Bücher 1,4,9,16,25,36,49,64,81,100,121,144,169,196,225,256,289,324,361 und 400
liegen mit der Vorderseite nach oben, alle anderen Bücher mit der Rückseite
-übrigens: wenn man 2 Durchgänge direkt hintereinander macht, ohne nach dem 1.Durchgang alle Bücher wieder auf die gleiche Seite zu drehen,
zeigen nach dem 2.Durchgang wieder alle Bücher mit der Rückseite nach oben, da man sie auf alle Fälle eine gerade Anzahl mal gedreht hat


Aufgabe 12

336. Wertungsaufgabe
Bernd sitzt in seinem Zimmer und hat 5 gleichgroße Würfel. Sehr gespannt und konzentriert schiebt er sie hin und her als Mike dazu kommt. „Was machst du da?“ „Ich versuche herauszufinden, wie viele Möglichkeiten es gibt, die fünf Würfel anzuordnen.“ Wie viele Möglichkeiten gibt es? Die Würfel werden passend aneinander gelegt. (Seite eines Würfels genau an eine Seite eines anderen Würfels). Nur eine Schicht von Würfeln nutzen. Drehung und Spiegelung einer Anordnung zählen nur einmal. Es sind immer alle 5 Würfel zu nehmen. Für je zwei Formen gibt es einen blauen Punkt.
Bei rot darf mit „Klebstoff gearbeitet“ werden. Wie viele Möglichkeiten gibt es noch, die 5 Würfel anzuordnen, wenn die Bedingung nur eine Schicht an Würfeln zu nutzen, nicht zutreffen soll. Für je zwei Formen gibt es einen roten Punkt, am Ende runde ich auf.

english version:

Bernd is sitting in his room with 5 cubes of identical size. He is absorbedly pushing them here and there when Mike enters. “What are you doing?” “I'm trying to figure out how many ways there are to arrange 5 cubes.” How many ways are there? The cubes must be arranged side by side in just
one layer. Rotations and reflections of the same arrangement count as one. Use all 5 cubes each time. - one blue point for every two arrangements. How many arrangements are the when you can use more than one layer? - one red point for every two figures.

Lösung/solution:

blau: 12 Möglichkeiten, rot: 17 Möglichkeiten

Eine schöne Spielvariante, die alle insgesamt 29 Möglichkeiten beinhaltet findet sich --> hier <--

Bilder zur Lösung gibt es viele. Das Suchwort heißt dann Pentacuben bzw. PentaKuben. (Penta - Fünf, Kuben (cuben (Würfel))

Alle Varianten für blau zu finden ist nicht sooooooooooo schwer. Am besten natürlich wieder systematisch:

1. 5 Würfel hintereiander --> eine Möglichkeit

2. 4 Würfel hintereinander --> der verbleibende 5. Würfel kann nun nur an den ersten oder zweiten der Viererreihe gelegt werden --> zwei Möglichkeiten.

3. 3 Würfel aneinander

3.1 Die verbleibenden Würfel liegen aneinander. Dieser "Zweierblock" "abstehend" an den Anfang oder in die Mitte des Dreieses gelegt werden oder schmeigt sich an den Dreier driekt oder überstehend an --> vier Möglichkeiten

3.2 Die verbleibenden zwei Würfel sind einzeln. Auf der gleichen Seite des Dreiers eine Möglichkeit. Auf verschieden Seiten (Aufpassen, dass nicht ein weiter oben beschriebener fall auftritt) Einer am Rand des Dreiers, der anders auch am Rand (am anderen Rand) oder in der Mitte --> zwei Möglichkeiten und schließlich die beiden mittig --> eine Möglichkeit.

4. keine 3 Würfel aneinander (etwas hin und her geschoben ...) --> ein Möglichkeit.

1. bis 4. zusammen ergibt genau 12 Möglichkeiten.

Für die rote Aufgabe funktioniert das entsprechend, wobei an sehr aufpassen muss, das sich nichts doppelt.


 Auswertung Serie 28 Anmerkung Punkte der 336, die noch einzutragen wären, weil Lösungen in der Schule liegen, gehen nicht verloren, werden aber hier nicht nachgetragen.

Auswertung Serie 28 (blaue Liste)

Platz Name Ort Summe Aufgabe
  325 326 327 328 329 330 331 332 333 334 335 336
1. Rafael Seidel Chemnitz 57 4 4 4 3 4 6 4 12 2 4 4 6
2. Linus-Valentin Lohs Chemnitz 56 4 4 3 3 4 6 4 12 2 4 4 6
3. Jürgen Urbig Chemnitz 55 4 4 4 3 4 5 4 12 2 4 4 5
4. Sabine Fischbach Hessen 51 4 3 - 3 3 6 4 12 2 4 4 6
5. Uwe Parsche Chemnitz 49 4 - - 3 4 6 4 12 2 4 4 6
6. Doreen Naumann Duisburg 48 4 3 2 3 4 6 4 12 2 4 4 -
7. Paula Hartmannsdorf 45 - - 4 3 4 6 4 12 2 - 4 6
8. Valentin Grundmann Chemnitz 36 - 4 - 2 - 6 4 6 - 4 4 6
9. Lukas Thieme Chemnitz 35 - - 2 3 4 - 4 10 2 - 4 6
9. Lene Haag Chemnitz 35 - - - 3 4 4 1 8 2 - 4 6
10. Tobias Morgenstern Chemnitz 34 - - 2 3 4 5 2 10 - - 4 4
11. Gunnar Reinelt Chemnitz 33 - 4 3 - - 6 - 6 - 4 4 6
12. Luis Raupach Chemnitz 32 - 4 4 - - - 4 10 - - 4 6
12. Helene Fischer Chemnitz 32 - - 1 3 4 5 3 4 2 - 4 6
13. Celestina Montero Perez Chemnitz 31 - - - 3 4 5 3 4 2 - 4 6
13. Arne Weißbach Chemnitz 31 3 - - - 4 6 4 12 2 - - -
14. Emily Neuwirth Chemnitz 28 - 4 - - 4 - 4 6 - - - 6
14. Henrike Grundmann Chemnitz 28 - 4 - - 4 - 4 6 - 4 - 6
15. Tom Straßer Chemnitz 27 - 4 4 3 4 6 - - - - - 6
15. Theresa Jänich Chemnitz 27 - 3 - - 4 5 - 6 - 4 - 5
16. Frederike Meiser Chemnitz 26 - - - 3 4 5 2 - 2 - 4 6
17. Elisa Parsche Chemnitz 24 - - - - - 6 4 12 2 - - -
17. Felix Taubert Chemnitz 24 - 4 4 3 - - - 8 - - - 5
17. Simon Anders Chemnitz 24 - - 4 3 - 5 - 12 - - - -
17. Hannah Gebhardt Chemnitz 24 - 4 4 - - - 4 6 - - - 6
17. XXX ??? 24 - - - 3 - - - 12 - - 4 5
18. Lena Elisa Penzlin Chemnitz 23 - 4 - - 4 - 4 6 - - - 5
18. Hannes Hohmann Chemnitz 23 - - - - - 3 1 10 - 3 - 6
18. Valentin Sellin Chemnitz 23 - 3 4 - - - - 6 - - 4 6
18. Simon Winger Chemnitz 23 - 4 - 3 - - 4 12 - - - -
18. Heinrich Grossinger Chemnitz 23 - 4 - 3 4 6 - - - - - 6
19. Moritz Duderstadt Chemnitz 22 - 4 - - 4 - - 8 - - - 6
20. Tobias Richter Chemnitz 21 - - 2 3 2 - 4 8 2 - - -
20. Leon Hoppe Chemnitz 21 - - 1 3 2 - 1 12 - - 2 -
21. Willy Stöckel Chemnitz 20 - - 4 - 4 - - 6 - - - 6
21. Emilie Grossinger Chemnitz 20 - 4 - - 4 - - 6 - - - 6
22. Lukas Kirchberg Chemnitz 19 - 4 4 - - - - 6 - - - 5
22. Lisanne Brinkel Chemnitz 19 - 2 - 3 - 5 - 4 - - - 5
22. Katharina Zweiniger Chemnitz 19 - - - 1 2 4 1 4 2 - - 5
23. Marcel Reichelt Chemnitz 18 - 3 - 2 - - - 8 - - - 5
24. Ole Koelb Chemnitz 17 - 3 4 3 - 5 - 2 - - - -
25. Melanie Petz Chemnitz 16 - 3 4 - 4 5 - - - - - -
25. Felicitas Güra Chemnitz 16 - 4 - 2 - 6 4 - - - - -
25. Friederike Lenk Chemnitz 16 - 4 - - - - - 6 - - - 6
26. Lilli Weiß Chemnitz 15 - 2 - - - 4 - 4 - - - 5
26. Paula Mühlmann Dittersdorf 15 - - - - 4 3 - - 2 - - 6
26. Ellen Wilde Chemnitz 15 - 3 - 3 - 4 - 2 - - - 3
26. Ernesto Uhlmann Chemnitz 15 - 4 - - - - - 6 - - - 5
26. Nele Mäding Chemnitz 15 - 2 - 3 - 6 4 - - - - -
27. Franz Kemter Chemnitz 14 - - 1 - - - 2 8 - 3 - -
27. Felicitas Hastedt Chemnitz 14 - 4 4 - - 6 - - - - - -
27. Julia Voigt Chemnitz 14 - - 4 - - - 4 - - - - 6
27. Karl Herrmann Chemnitz 14 - 2 - 3 - 5 4 - - - - -
27. Adrian Schlegel Chemnitz 14 - 4 - - - 5 - - - - - 5
28. Andree Dammann München 13 - - - 3 - - 4 - - - - 6
28. Marvin Gülden Chemnitz 13 - - 4 - - 5 4 - - - - -
28. Nicklas Reichert Chemnitz 13 - - - - - - - 4 - 3 - 6
28. Elena Oelschlägel Chemnitz 13 - 4 - 3 - 6 - - - - - -
29. Elina Rech Chemnitz 12 - - - 3 4 5 - - - - - -
29. Amarin Roßberg Chemnitz 12 - 4 4 - - - - 4 - - - -
29. Ulrike Böhme Chemnitz 12 - 3 - - 4 5 - - - - - -
30. Felix Karu Innsbruck 10 - - - - - - - - 2 4 4 -
30. Jessica Ritter Chemnitz 10 - 3 2 - - 5 - - - - - -
30. Florian A. Schönherr Chemnitz 10 - - - - 4 6 - - - - - -
30. Cynthia Raschkowsky Chemnitz 10 - 3 - - - 3 4 - - - - -
31. Kai-Lutz Wagner Chemnitz 9 - - - - - 5 4 - - - - -
31. Tim Jechorek Chemnitz 9 - - - 3 - - - 2 - - - 4
32. Kevin Ngyen Chemnitz 8 - - - - - - - 8 - - - -
32. Saskia Schlosser Chemnitz 8 - - - - 4 4 - - - - - -
32. Jonas Frederik Otto Lichtenwalde 8 - - - - 4 - 4 - - - - -
33. Tom Hartig Chemnitz 7 - 3 4 - - - - - - - - -
34. Moritz Weber Chemnitz 6 - 2 - - - - 3 - - - - -
34. Carl Geißler Chemnitz 6 - - - - - 6 - - - - - -
34. Hannah-Sophie Schubert Chemnitz 6 - - - - - - - 6 - - - -
35. Marie Juhran Chemnitz 5 - - - 3 - - - 2 - - - -
35. Joel Magyar Chemnitz 5 - - - - - 4 1 - - - - -
36. Niels Steinert Chemnitz 4 4 - - - - - - - - - - -
36. Albin Uhlig Chemnitz 4 - - - - - - 1 2 - - 1 -
36. Melina Seerig Chemnitz 4 4 - - - - - - - - - - -
36. Hermann Thum Chemnitz 4 - 4 - - - - - - - - - -
36. Lisa Grassmann Chemnitz 4 - - - - 4 - - - - - - -
36. Christian Wagner Bamberg 4 4 - - - - - - - - - - -
36. Helen Taubman Berlin 4 4 - - - - - - - - - - -
36. Elisa Bolte Chemnitz 4 - 4 - - - - - - - - - -
36. Julian Vass Chemnitz 4 - 4 - - - - - - - - - -
36. Sophie Kalmer Chemnitz 4 - - - - 4 - - - - - - -
36. Shari Schmidt Chemnitz 4 - - - - - 1 1 2 - - - -
36. Anna Grünert Chemnitz 4 - 3 1 - - - - - - - - -
36. Lena Rabbeau Chemnitz 4 - - - - 4 - - - - - - -
36. Camilla Schreiter Chemnitz 4 - - - - 4 - - - - - - -
36. Anna Georgi Chemnitz 4 - - - - - - - 4 - - - -
36. Pauline Marschk Chemnitz 4 - - - - 4 - - - - - - -
37. Katrin Posselt Chemnitz 3 - 3 - - - - - - - - - -
37. Ida Heuschkel Chemnitz 3 - - 1 - 2 - - - - - - -
38. Astrid Fischer Chemnitz 2 - - - - - - - - 2 - - -
38. Leonie Freiherr Chemnitz 2 - - - - - - - 2 - - - -
38. Tim Schiefer Chemnitz 2 - - - - - - - 2 - - - -
38. Emilia Lesselt Chemnitz 2 - - - - - - - 2 - - - -
38. PC Zerbe Leipzig 2 - - - - - - - - 2 - - -
38. Friederike-Charlotte Wolf Chemnitz 2 - - - - - - - 2 - - - -
38. Jessica Spindler Chemnitz 2 - - - - 2 - - - - - - -
38. Marie Berger Chemnitz 2 - - - - - - - - 2 - - -
38. Laura Schlosser Chemnitz 2 - - - - - 2 - - - - - -
38. Tonio Drechsler Chemnitz 2 - - - - - - - 2 - - - -
38. Annika Theumer Chemnitz 2 - - - - - - - - 2 - - -
38. Nathalie Müller Chemnitz 2 - - - - - - - 2 - - - -

Auswertung Serie 28 (rote Liste)

Platz Name Ort Summe Aufgabe
  325 326 327 328 329 330 331 332 333 334 335 336
1. Uwe Parsche Chemnitz 53 4 - - 4 5 5 5 8 2 8 6 6
2. Jürgen Urbig Chemnitz 49 4 4 4 4 5 5 5 - 1 8 4 5
3. Rafael Seidel Chemnitz 47 - 4 4 4 5 5 5 - 2 8 4 6
4. Sabine Fischbach Hessen 45 4 - - 4 2 - 5 8 - 8 6 8
4. Doreen Naumann Duisburg 45 4 3 3 4 5 5 5 - 2 8 6 -
5. XXX ??? 24 - - - 4 - - - 8 - - 6 6
6. Linus-Valentin Lohs Chemnitz 23 - - 3 4 5 1 - 4 2 - 4 -
7. Paula Hartmannsdorf 20 - - 4 - 5 5 4 - 2 - - -
8. Arne Weißbach Chemnitz 16 4 - - - 1 5 4 - 2 - - -
8. Felix Karu Innsbruck 16 - - - - - - - - 2 8 6 -
9. Andree Dammann München 14 - - - 4 - - - 4 - - - 6
10. Melanie Petz Chemnitz 11 - 4 4 - 3 - - - - - - -
11. Elisa Parsche Chemnitz 10 - - - - - 5 5 - - - - -
12. Valentin Grundmann Chemnitz 8 - - - - - - - - - - - 8
12. Valentin Sellin Chemnitz 8 - - 4 - - - - - - - 4 -
13. Tom Straßer Chemnitz 7 - - - - - - - - - - - 7
14. Julia Voigt Chemnitz 6 - - - - - - - - - - - 6
14. Heinrich Grossinger Chemnitz 6 - - - - - - - - - - - 6
15. Elisa Bolte Chemnitz 4 - 4 - - - - - - - - - -
15. Helen Taubman Berlin 4 4 - - - - - - - - - - -
15. Christian Wagner Bamberg 4 4 - - - - - - - - - - -
15. Tom Hartig Chemnitz 4 - - 4 - - - - - - - - -
16. Kevin Ngyen Chemnitz 3 - - - - - 3 - - - - - -
16. Nicklas Reichert Chemnitz 3 - - - - - 3 - - - - - -
17. Lukas Kirchberg Chemnitz 2 - 2 - - - - - - - - - -
17. Hannah Gebhardt Chemnitz 2 - - - - - - - - - - - 2
17. Ulrike Böhme Chemnitz 2 - 2 - - - - - - - - - -
17. Gunnar Reinelt Chemnitz 2 - - 2 - - - - - - - - -
17. Jessica Ritter Chemnitz 2 - - 2 - - - - - - - - -
17. Annika Theumer Chemnitz 2 - - - - - - - - 2 - - -
17. Emily Neuwirth Chemnitz 2 - - - - - - - - - - - 2
17. Astrid Fischer Chemnitz 2 - - - - - - - - 2 - - -
17. Celestina Montero Perez Chemnitz 2 - - - - - - - - 2 - - -
17. Helene Fischer Chemnitz 2 - - - - - - - - 2 - - -
17. PC Zerbe Leipzig 2 - - - - - - - - 2 - - -
17. Amarin Roßberg Chemnitz 2 - 2 - - - - - - - - - -

 

Serie-27

Serie 27


Aufgabe 1

313. Wertungsaufgabe

Zu Opas Geburtstagsfeier hat Bernds Mutti die lange nicht benutzten Zauberschachteln geputzt und mit Kugeln gefüllt. "Zauberschachteln?", fragt Lisa, die diese zum ersten Mal sieht, nach. "Die sollten besser Logikschachteln heißen", schmunzelte Bernds Mutti, "denn ähnlich dem Mastermind gilt es herauszufinden, was in den Schachteln ist."
Es sind fünf Schachteln von links nach rechts eine weiße mit einer 11 drauf, eine schwarze mit einer 4, eine rote mit einer 7, eine blaue mit einer 16 und eine grüne mit einer 17.
In jeder Schachtel sind genau zwei Kugeln drin, die von 1 bis 10 durchnummeriert sind. Die Zahl auf der Schachtel gibt die Summe der Zahlen an, die auf den in den Schachteln versteckten Kugeln stehen. Wie lauten die Nummern in den einzelnen Schachteln? 4 blaue Punkte.
Es geht noch weiter. Die Kugeln gibt es in den selben Farben wie sie auch die Schachteln haben -- von jeder Sorte genau zwei.
1. In keinem Fall stimmen Schachtelfarbe und Kugelfarbe überein.
2. In der weißen oder der schwarzen Schachtel sind eine rote und eine grüne Kugel zusammen drin.
3. In der blauen Schachtel ist eine schwarze Kugel drin.
4. Es gibt eine Schachtel mit einer weißen und einer blauen Kugel.
5. In der schwarzen Schachtel findet man eine grüne und eine blaue Kugel.
6. In der roten Schachtel ist keine blaue Kugel.
Wo ist was drin? 4 rote Punkte.

Lösung:
Hier die Lösungsvariante von Doreen N., danke.
blau:
-in schwarzer Schachtel mit der Zahl 4 müssen Kugeln 1 und 3 sein, da keine andere Zahlenkombination möglich
-in roter Schachtel mit der Zahl 7 müssen Kugeln 2 und 5 sein, da sonst nur noch 3+4 möglich und die 3 ist bereits für schwarz benutzt
-in weißer Schachtel mit der Zahl 11 müssen Kugeln 4 und 7 sein, da 1, 2, 3 und 5 schon anderweitig benutzt wurden
-übrig bleiben die Kugeln 6 und 10 für die blaue und die Kugeln 8 und 9 für die grüne Schachtel
=> weiß: 11=4+7
schwarz: 4=1+3
rot: 7=2+5
blau: 16=6+10
grün: 17=8+9
rot:
-nach 3. in blauer Schachtel 1 schwarze Kugel und nach 5. in schwarzer Schachtel  1 grüne+1 blaue Kugel
-nach 2. muss in weißer Schachtel 1 rote+1 grüne Kugel, da der Inhalt der schwarzen Schachtel bekannt ist
-nach 6. muss die zweite blaue Kugel in grüner Schachtel sein, da sie nicht in roter (nach 6.)und blauer (nach1.) ist
-nach 4. kommt in die grüne Schachtel zu der blauen eine weiße Kugel
-die zweite rote Kugel muss in blauer Schachtel sein, da sonst nur noch rote Schachtel übrig (nach 1. nicht erlaubt)
-übrig bleiben 1 weiße + 1schwarze Kugel für die rote Schachtel
=> weiße Schachtel: 1 rote + 1 grüne Kugel
schwarze Schachtel: 1 blaue + 1 grüne Kugel
rote Schachtel: 1 weiße + 1 schwarze Kugel
blaue Schachtel: 1 rote + 1 schwarze Kugel
grüne Schachtel: 1 blaue + 1 weiße Kugel



Aufgabe 2


314. Wertungsaufgabe
"Hallo Mike, schau mal, ich habe hier einen recht ungewöhnlichen Bruch:  \frac{503}{504}". "Was ist an dem ungewöhnlich?" Nun, er ist die Summe von drei Brüchen, deren Nenner alle teilerfremd sind und die Zähler einen "direkten Bezug" zur Aufgabennummer haben. Wie heißen die drei Brüche? 3 blaue Punkte. "Kannst du eigentlich noch die Stammbruchumwandlung?" Du meinst diese ägyptische Bruchrechnung, die auf der Homepage beschrieben ist?". "Genau." Wie sieht die Zerlegung des Bruches  \frac{503}{504} in eine Summe von Stammbrüchen aus? - 4 rote Punkte.

Lösung:
blau: Die drei gesuchten Brüche sollen teilerfremd sein. Damit ist die 504 also das kleinste gemeinsame Vielfache von drei teilerfremden Zahlen. Die Primfaktorenzerlegung von 504 ist 2*2*2*3*3*7. Diese Zerlegung enthält drei verschieden Primzahlen, die wegen der Teilerfremdheit nicht "gemischt werden dürfen. Als sind die gesuchten Nenner 2³ = 8, 3² = 9 und 7. Geordnet 7, 8 und 9. Es gilt also zu folgendes untersuchen:  \frac{x}{7} + \frac {y}{8} + \frac {z}{9} = \frac{503}{504}
Macht man die Brüche gleichnamig so erhält man:  \frac{72x}{504} + \frac {63y}{504} + \frac {56z}{504} = \frac{503}{504}.
Gesucht wären also ganzzahlige Lösungen der Gleichung 72x + 63y + 56z = 503. Da gibt es letztlich unendlich viele. Nun gibt es aber noch den Hinweis mit dem Bezug Nenner und Aufgabenzahl 314 - eine kleine Hommage an \Pi. Die Zahlen 3 1 und 4 den Nenner zuzuordnen geht auf 6 verschiedende Arten. Probiert man die durch, so verbleibt: \frac{3}{7} + \frac {1}{8} + \frac {4}{9} = \frac{503}{504}
rot: Den Bruch in eine Summe von Stammbrüchen zu zerlegen, ist nicht nur auf eine Art und Weise möglich. Das System der ägytischen Bruchrechnung leifert folgende Variante:

\begin{array}{rcl}\frac{503}{504}&=&\frac{503}{1006} + \frac{503}{504} - \frac{503}{1006}\\
&=&\frac{1}{2} + \frac{251}{504} \\
&=&\frac{1}{2} + \frac{251}{753} + \frac{251}{504} - \frac{251}{753}\\
&=&\frac{1}{2} + \frac{1}{3} + \frac{83}{504}\\
&=&\frac{1}{2} + \frac{1}{3} + \frac{83}{581} + \frac{83}{504} - \frac {83}{581}\\
&=&\frac{1}{2} + \frac{1}{3} + \frac{1}{7} + \frac{11}{504}\\
&=&\frac{1}{2} + \frac{1}{3} + \frac{1}{7} + \frac{11}{506}+ \frac{11}{504} - \frac{11}{506}\\
&=&\frac{1}{2} + \frac{1}{3} + \frac{1}{7} + \frac{1}{46}+ \frac{1}{11592}

\end{array}
Zerlegung in Stammbrüche gefunden.
Den Nachweis, dass diese Methode der Zerlegung endlich ist, überlasse ich dem geneigten Leser.




Aufgabe 3

315. Wertungsaufgabe

"Hallo Mike, was hast du denn da?", fragt Bernd. "Das sind Denksportkarten. Die hat mir mein Lehrer mal ausgeliehen. Es gibt verschiedene Schwierigkeitsstufen." "Dürfen wir mal ein Beispiel zeigen?" "Ich denke schon." Nehmen wir ein Tierrätsel: Unter einem Adler steht die Zahl 0104120518.
Welche Zahl steht dann unter dem Pferd. Kleiner Tipp die Zahl ist genau so lang wie beim Adler und beginnt mit einer 1 und endet auf 4. (4 blaue Punkte)
Hier noch ein zweites Beispiel: In der Zeit von vorgestern bis übermorgen liegen so viele Tage wie von Sonntag bis zum gesuchten Tag. Welcher Tag ist morgen, wenn es in drei Tagen fünf Tage nach dem gesuchten Tag sein wird? (4 rote Punkte)

Lösung:
blau: Dem Adler wird eine Zahl zugeordnet. Es könnte also eine "Chiffre" sein. Das einfachste A = 1, B = 2, ... Cool, das passt. Es werden lediglich die führenden Nullen mit geschrieben. Beim Pferd ist der letzte Buchstabe d - das wäre also 04. Auch dass passt mit dem Hinweis Pferd endet auf 4 überein. Zählt man das Alphabet bis zum jeweiligen Buchstaben durch wird Pferd zu 1606051804.
rot: Vorgestern bis übermorgen = 5 Tage Fünf Tage von Sonntag ...... Donnerstag.
Zweiter Tei ähnlich einfach Morgen ist dann Sonntag.
Quelle der Aufgaben Denksportkarten TOPASS giga IQ




Aufgabe 4

316. Wertungsaufgabe

Als Mike zu Bernd ins Zimmer tritt, sieht er 4 Holzquader auf dem Tisch. "Was willst du denn damit?" "Diese Quader -- wie du siehst, sind alle gleich groß (4 cm x 6 cm x 10 cm) und wiegen jeweils 192 Gramm. In drei der Quader soll ein "quadratisches" Loch durchgebohrt werden - zentriert und jeweils 2 cm groß." "Ach, ich verstehe, ich vermute mal, dass die Löcher jeweils durch die unterschiedlich großen Seiten verlaufen sollen." "Genau." Wie schwer sind die jeweils herausgebohrten Teile? (3x2=6 blaue Punkte.) Wie schwer ist der Abfall, wenn man in dem vierten Quader alle 3 Bohrungen vornimmt. (3 rote Punkte) Noch mehr rote Punkte kann man erhalten, wenn man statt der rausgebohrten Quader drei Zylinder mit d = 2 cm verwendet, die alle aus einem 5. Holzquader gebohrt werden sollen.

Lösung:
Hier die Lösung von Jürgen Urbig, danke.
als pdf



Aufgabe 5

317. Wertungsaufgabe
"Lisa, ist das dein Spickzettel für deinen Vortrag, der ist so winzig?", fragte Maria. "Nicht wirklich, der ist so klein, weil dort die größte Zahl drauf steht, die mit drei Ziffern geschrieben werden kann." Welche Zahl steht auf dem Zettel von Lisa? 2 blaue Punkte. Wie viele Stellen hat wohl diese Zahl drei rote Punkte.  Noch mehr rote Punkte gibt es, wenn  jemand die letzten 10 Ziffern der Zahl angibt.

Lösung:
Die Ziffern der gesuchten Zahl  sind 9  9  9, aber die Zahl 999 ist es nicht, sondern: 9^{9^9}
Um Missverständnisse zu vermeiden setze ich mal noch Klammern 9^(9^9).
Gibt man 9^9^9 in den Taschenrechner ein, so rechnet der allerdings (9^9)^9
Ein paar Größenordnungen als Vergleich:
(99)9 = 1,966 .. * 1077
999 = 2,9512 * 1094
Bei der größten Zahl aber wird der Faktor 9 sagenhafte 387420489 mal miteinander multipliziert.
Die Zahl hat 369.693.100 Ziffern - lässt mittels Logarithmengesetz ermitteln.
In dem in der nächsten Aufgabe genannten Buch kann man noch die ersten und letzten Stellen finden:
Die ersten Ziffern: 428124773175747028036987115930563521339055...
die letzten Ziffern: ...681422627177289
Die Quelle dafür ist (also wie man darauf kommt) Chr. Weiss, "Hu", Tallet 9^(9^9) og Endecifrene i Potenser af 9, Matematisk Tidsskrift A 1941, S. 63 ff.
Falls jemand diese Zeitschrift hat - ich würde mich über eine Kopie freuen. Winken
Eine englische Version des Artikel gefunden von J. Urbig, danke --> als pdf <--
 



Aufgabe 6

318. Wertungsaufgabe
Lisa hat das Büchlein " Riesen und Zwerge im Zahlenreich" vor sich liegen. "Ach, daher war wohl deine Aufgabe der letzten Woche?", fragte Mike. "Stimmt. Aus diesem Buch stammt -- leicht abgewandelt - auch diese Problemstellung: Mal angenommen, der Bodensee friere komplett zu und alle Menschen Europas -- Russland komplett gerechnet -- würden dieses Ereignis sehen wollen und dorthin fahren. Passen alle Menschen auf das Eis, wenn man bequem steht, also mit 3 Leuten pro Quadratmeter rechnet. (5 blaue Punkte wegen der notwendigen Recherche) Bleiben wir mal noch bei großen Zahlen. Wie groß wäre ein Goldwürfel, dessen Masse genau so groß wäre wie die Masse der Weltbevölkerung (rund 7 Mrd. Menschen), wenn -- wegen der vielen Kinder -- von 48 Kilogramm pro Person ausgegangen wird? 3 rote Punkte.

Lösung:
Blau. Es gibt verschiedene Ansätze. So haben viele Einsender die Einwohnerzahl(en) ermittelt. Diese liegt bei rund 770.000.000. Der Bodensee hat eine ein Fläche von ca. 536 km². Umgerechnte in m² sind das 536.000.000 m³. Wenn man mit 3 Personen pro m² rechnet, dann passen mehr als die doppelte Anzahl aller Europäer auf den Bodensee - das ist schon erstaunlich.
rot: Masse der Erdbevölkerung: 48 * 7 Mrd = 336.000.000.000 kg. Die Dichte des Goldes beträgt 19,32 g/cm³ = 19,32 kg/dm³ = 19320 kg/m³
Das Volumen des Würfels ergibt sich dann mit V = Masse : Dichte mit 17391304,347... m³. Die dritte Wurzel aus diesem Wert liefert dann die Kantenlänge des Würfels 259,08 m.
 



Aufgabe 7

319 . Wertungsaufgabe
319"Wir hatten doch vor drei Wochen die Holzquader. Hast du vielleicht noch einen?", fragte Mike. "Aber klar doch, was willst du denn damit?". "Lisa will in Ihrer Mathegruppe herausfinden lassen, wie viele verschiedene Wege es gibt, um von A nach G zu gelangen, wenn man nur die Kanten entlang gehen darf, aber keine Kante und keinen Punkt doppelt nutzen darf." "Alles klar" (4 blaue Punkte) Wie lang ist der kürzeste Weg von A nach G, wenn man nicht auf den Kanten des massiven Holzquaders bleiben muss? (4 rote Punkte) Die Maße des Quaders waren 4 cm x 6 cm x 10 cm. (Das Bild des Würfels ist nur zur Orientierung.)

Lösung:
Für den blauen Teil ist es möglicherweise am sinnvollsten sytematisch vorzugehen, damit kein Weg übersehen wird.  So gibt es von A aus drei Möglichkeiten A --> B, A --> D und A --> E.
Betrachtet man nun B so geht B --> C und  B --> F (zurück nach A ist nicht zulässig, auf die nicht zulässigen wird ab jetzt nicht weiter verwiesen.)
Weiter im Alphabet C --> G . 1. Weg A --> B --> C --> G
C --> D --> H --> G  2. Weg A --> B --> C --> D --> H --> G
Aber halt:  Von H geht es auch noch so weiter H --> E --> F --> G 3. Weg A --> B --> C --> D --> H --> E --> F --> G
Betrachtet man nun die "offene" Variante von B --> F ergibt das wieder drei Wege. Es gibt also 6 verschiedene Wege A --> --> ...
Damit sind es auch 6 Wege mit A --> D --> ... und letztlich noch 6 Wege mit mit A --> E --> ...
Insgesamt also 18 Wege. Hier noch die Zeichnung von U. Parsche --> als pdf  <--, danke.
Die rote Aufgabe erwies sich als recht knifflig, aber wenn man auf den Trick mit dem Netz gekommen war, ging es dann doch.
Die Angaben für die Längen des Quaders waren nicht ganz eindeutig, so dass es durchaus unterschiedliche Ergebnisse geben konnte.
Eine vollständige Beschreibung ist der Lösung von Uwe Parsche entnehmbar, danke. Hier --> als pdf <--.




Aufgabe 8

320. Wertungsaufgabe
Bernd hat auf dem Schreibtisch einen Ausdruck der Eiskarte seiner Lieblingseisdiele. Mike liest.
Fruchteisbecher: 3 Kugeln Erdbeer, 1 Kugel Vanille mit Früchten 4,10 Euro
Der Fruchteisbecher mit Sahne 5,30 Euro
Kalter Genuss: 2 x Erdbeer, 2 Vanille und Sahne 3 Euro
Kalter Genuss mit Früchten 5,20 Euro
Wie viel kostet eine Portion Sahne mit Früchten? - 2 blaue Punkte
Wie viel kostet "Eistraum" (3 x Vanille, 1 x Erdbeer mit Sahne und Früchten)? 3 rote Punkte

Lösung:

Eine sehr ausführliche und nachvollziehbare Lösung hat Linus (Klasse 6) beigesteuert, danke.
als pdf




Aufgabe 9

321. Wertungsaufgabe

Lisa sitzt und träumt. Vor ihr auf dem Tisch steht ein Holzwürfel: "Der Zauberer Maths teilt den Holzwürfel mit einem ebenen Schnitt -- parallel zu einer der Seiten -- glatt in der Mitte durch." Als Lisa erwacht, sind wirklich zwei solche halben Würfel auf dem Tisch. (Mike hatte sich reingeschlichen). Ob nun so ein volumenmäßig halbierter "Würfel" auch eine halb so große Oberfläche wie der ursprüngliche Würfel hat? 3 blaue Punkte. Wie erreicht man, dass mit zwei ebenen Schnitten durch einen Würfel ein Restkörper entsteht, dessen Oberfläche maximal wird, wenn dessen Volumen halb so groß ist wie das des ursprünglichen Würfels? (6 rote Punkte)

Lösung:
blau: Der Restkörper hat als Seiten zwei Quadrate der Kantenlänge a und vier Rechtecke mit den Kantenlängen a und a/2. Rechnet man den Flächeninhalt dieser 6 Seiten zusammen, so erhält man für die Oberfläche 4a². Die Oberfläche des Ausgangswürfels aber war 6a². Der Oberflächeninhalt hat sich also nicht halbiert, im "Gegensatz" zum Volumen.
rot:
Schon mal im Voraus - eine abschließende Lösung - der Nachweis für die Maximierung des Oberflächeninhaltes steht noch aus.
Ich stelle hier mal meine Variante vor, die einen Oberflächeninhalt hat, der alle bisher eingegangen Werte für den Oberflächeninhalt überschreitet.
321
Au verflixt, da hat sich ein Fehler eingeschlichen, sorry, wird noch korrigiert.
Mit rot sind hier zwei Ebenen eingezeichnet, die den Würfel vom Volumen her gesehen halbieren. Die Fläche EFYX ist halb so groß wie EFGH. HX ist mit a/4 gewählt, damit muss dann GY a/3. sein. --> Fläche EXH = a*a/4 = a²/4 und Fläche YGX = a/3*3/4a = a²/4 zusammen also a²/2.
Die Oberfläche des entstandenen Prismas ergibt sich dann aus. 2* Grundfläche (= a²) + Fläche ABFE (= a²) + 2/3a² (= Fläche BFY?) + XY*a + EX*a
EX =  \frac {\sqrt {5}}{2} \cdot a ebenso ergibt sich mittels Satz des Pythagoras für XY =  \frac {\sqrt {97}}{12} \cdot a
Setzt man alle Teilergebnisse ein, so ergibt sich die Oberfläche zu 4,605 a².
Wer mag, kann ja diesen Ansatz auf eine optimale Lage von X und damit von Y untersuchen, über eine Zusendung würde ich mich freuen.
Anmerkungen: Wählt man Y=F so erhält man ein dreiseitiges Prisma, welches unanhängig von der Lage von X ein volumenmäßig halbiertes Volumen liefert. Halbiert X die Strecke GH so hat dieses Prisma, die kleinste aller Oberflächen mit 4,23 ... a². Das Maximum läge bei X= H oder X = G mit 4,414 .. a². (Das lässt sich dann aber schon mit einem Schnitt bewältigen.)



Aufgabe 10

322. Wertungsaufgabe
Maria und Lisa haben ein Quadrat (4 cm) und ein Rechteck (5 x 6 cm). Während sie arbeiten, rutscht das Quadrat über das Rechteck. Mike kommt dazu und sagt: "Cool, es sieht so aus, als ob das Quadrat die halbe Fläche des Rechtecks verdeckt. Schiebt mal nicht weiter, ich will mal nachmessen. Ja, stimmt genau." Wie könnte das Quadrat auf dem Rechteck gelegen haben? (Eine Möglichkeit "zeigen" - 3 blaue Punkte)
Wie müsste man zwei gleichgroße Kreise übereinander schieben, so dass der untere zur Hälfte verdeckt wird -- 6 rote Punkte.

Lösung:
blau: Das Rechteckeck hat einen Flächeninhalt von 30 cm². Die Hälfte davon soll - also 15 cm² - sollen bedeckt sein. Das Quadrat hat einen Flächeninhalt von 16 cm², also muss das Quadrat so auf das Rechteck gelegt werden, dass ein Quadratzentimer "übersteht". Dafür gibt es viele Möglichkeiten. Zum Beispiel wird das Quadrat so geschoben, dass ein rechteckiger Streifen von 4cm x 0,5 cm "übersteht".
rot: Zur Illustration verwende ich hier das Bild von Uwe Parsche, danke.
322Die überdeckte Fläche  des blauen Kreises, besteht aus zwei gleich großen Kreisabschnitten (Kreissegmenten). ZU sehen ist das Kreisgegment des rechten Kreises, das vom blauen Kreis liegt darunter.
Die Fläche der beiden Segmente
(Formel für ein Segment, wenn der Winkel Alpha im Bogenmaß verwendet wird:  A_s = \frac{r^2}{2} (\alpha - sin \alpha)
soll genau so groß sein wie die halbe Fläche  eines Kreises.
Damit ergibt sich:
 \frac {\pi r^2}{2} = r^2 (\alpha - sin \alpha)
Wird durch r² dividiert und anschließend die Klammer aufgelöst so ergibt sich:
\frac{\pi}{2} = \alpha - sin \alpha bzw.
 sin \alpha = \alpha - \frac{\pi}{2}
Es handelt sich hier um eine transzendente Gleichung. Die zu lösen, nun ja. Hier kann man nun zum einen mit einer Tabellenkalkulation arbeiten oder auch graphisch, in dem der Schnittpunkt der beiden Funktionen  y = sin \alpha und  y = \alpha - \frac{\pi}{2} mit x = \alpha gesucht wird.
Der Winkel der so ermittelt, wird liegt bei 2,30988.... Nun wird noch die Höhe des (bzw. der Segmente) gebraucht. Wer es bis hierhin geschafft hat, schafft den Rest auch allein, so dass letztlich die beiden Mittelpunkte der sich überlagernden Kreise einen Abstand von 0,80794 r haben müssen.



Aufgabe 11

322. Wertungsaufgabe

Bernd stapelt Würfel übereinander und zeichnet verschiedene Ansichten seines „Bauwerkes“.

Von vorn:

323-von-vorn

von rechts:

323-von-rechts

von oben:

323-von-oben

Wie viele Würfel hat Bernd mindestens verbaut? 3 blaue Punkte

Wie groß ist die Oberfläche des Gebildes –  bei minimaler Würfelzahl, wenn jeder Würfel 2 cm groß ist? 4 rote Punkte

Lösung:

Für die Anordnung der Würfel gibt es zwei Möglichkeiten. Es können 14 oder 13 Würfel sein. Die Aufgabenstellung aber verlangte die Minimalzahl der Würfel.

323-14323-13

Da bei der Nutzung der Minimalzahl von Würfel keine Lücken im "Bauwerk" entstehen, ist die Zahl der Quadrate schnell ermittelt. Es ist einfach die doppelte Zahl von Quadraten, die bei der Aufgabenstellung zu sehen sind. Also 2 mal 25 ==> 50 Quadrate mit einem Flächeninhalt von je 4 cm². Damit hat das "Bauwerk" eine Oberfläche von 200 cm².


 

Aufgabe 12

324. Wertungsaufgabe

Bernd hat aus seinem Metallbaukasten eine Balkenwaage gebaut und probiert sie gerade aus. Mike kommt ins Zimmer und fängt an zu grübeln. "Was hast du?". "Ich überlege gerade, wie das ging, dass man mit höchstens zwei Wägeversuchen die falsche Münze entdeckt. Es gibt sieben Münzen, die bis auf eine, alle gleich schwer sind. Eine der sieben Münzen ist leichter als die anderen, ohne dass man es sieht." Wie schafft man das mit Bernds Balkenwaage? - 3 blaue Punkte. Bernds Vater hatte das Gespräch gehört und wandelte die Aufgabe ab. "Es gibt vier Münzen, drei davon wiegen 10 Gramm. Bei der vierten Münze weiß man nicht, ob sie schwerer oder leichter ist. Es gibt aber auch ein 10-Gramm-Stück." Wie lässt sich mit maximal zwei Versuchen auf der Balkenwaage herausfinden, welches die "falsche" Münze ist und auch, ob sie mehr oder eben weniger als 10 Gramm wiegt? - 4 rote Punkte.

Lösung:

Es mehrere sehr überzeugende Darstellungen der Lösungen. Ich habe mich für die Veröffentlichung der Lösung von J. Urbig entscheiden, danke.

--> als pdf <--

 

Auswertung der Serie 27

Auswertung Serie 27 (blaue Liste)

Platz Name Ort Summe Aufgabe

313 314 315 316 317 318 319 320 321 322 323 324
1. Rafael Seidel Chemnitz 42 4 3 4 6 2 5 4 2 3 3 3 3
2. Jürgen Urbig Chemnitz 41 4 3 4 6 2 5 3 2 3 3 3 3
2. Sabine Fischbach Hessen 41 4 3 4 6 1 5 4 2 3 3 3 3
3. Doreen Naumann Duisburg 40 4 3 4 5 1 5 4 2 3 3 3 3
4. Uwe Parsche Chemnitz 39 4 3 4 6 2 5 4 2 - 3 3 3
5. Linus-Valentin Lohs Chemnitz 38 3 3 4 6 2 5 1 2 3 3 3 3
6. Arne Weißbach Chemnitz 37 3 - 4 6 1 5 4 2 3 3 3 3
7. Andree Dammann München 35 4 3 3 6 1 5 4 - 3 3 3 -
8. Felix Haase Chemnitz 28 4 3 4 6 - 5 4 2 - - - -
9. Richard Hahmann Chemnitz 26 4 3 4 6 - 3 4 2 - - - -
10. Jamila Wähner Chemnitz 24 4 3 4 6 2 5 - - - - - -
10. Loise Reichmann Chemnitz 24 3 3 4 6 - 3 3 2 - - - -
11. Anja Posselt Chemnitz 23 4 3 4 6 1 5 - - - - - -
12. Marie Sophie Roß Chemnitz 21 3 - 4 6 - 5 3 - - - - -
13. Nina Zschätzsch Chemnitz 20 4 3 3 6 - - 4 - - - - -
13. Marion Sarah Zenk Chemnitz 20 3 3 4 6 - - 4 - - - - -
14. Hermann Thum Chemnitz 19 4 3 4 6 - - - 2 - - - -
14. Robin König Chemnitz 19 4 3 4 - - 3 3 1 - - - 1
14. Ellen Richter Chemnitz 19 3 3 3 6 1 - 3 - - - - -
15. Johanna Ranft Chemnitz 18 4 - 4 6 - 3 1 - - - - -
15. Ria Hopke Chemnitz 18 3 3 3 6 - - 3 - - - - -
15. Elisa Parsche Chemnitz 18 4 - 4 - - 5 - 2 - - - 3
16. Karolin Schuricht Chemnitz 17 3 - 3 - - 5 4 2 - - - -
17. Stephanie Dani Chemnitz 16 3 3 4 - - - 4 2 - - - -
17. Josephine Pallus Chemnitz 16 4 3 3 6 - - - - - - - -
18. Duncan Mahlendorff Chemnitz 15 - 3 - 6 - - 4 2 - - - -
19. Jonathan Kässler Chemnitz 14 3 - 3 6 - - - 2 - - - -
20. Lisa Grassmann Chemnitz 13 4 3 4 - - - - 2 - - - -
21. Paula Hartmannsdorf 11 - 3 4 - - - 4 - - - - -
21. Mara Neudert Chemnitz 11 3 - 3 - - 5 - - - - - -
21. Emily Neuwirth Chemnitz 11 4 - 4 - - - - - 3 - - -
21. Kai-Lutz Wagner Chemnitz 11 3 - 3 - - 5 - - - - - -
22. Lucas Steinke Chemnitz 10 3 3 - - - - 2 2 - - - -
22. Philipp Fürstenberg Chemnitz 10 3 - 3 - - - 2 2 - - - -
22. Ellen Wilde Chemnitz 10 4 - - - - 5 - - 1 - - -
22. Rebecca Wagner Oberwiesenthal 10 3 - - 6 - - 1 - - - - -
23. Felix Brinkel Chemnitz 9 4 3 - - - - - 2 - - - -
23. Adrian Schlegel Chemnitz 9 - - - - - 3 1 - - 3 2 -
24. Willy Stöckel Chemnitz 8 4 - 4 - - - - - - - - -
25. Marcel Reichelt Chemnitz 7 3 - 4 - - - - - - - - -
25. Theresa Jänich Chemnitz 7 - - 4 - - - - - 3 - - -
25. Moritz Duderstadt Chemnitz 7 3 - 4 - - - - - - - - -
25. Ingmar Richter Chemnitz 7 3 - - - - - 2 2 - - - -
25. Hannah Gebhardt Chemnitz 7 4 - - - - - - - 3 - - -
25. Lena Elisa Penzlin Chemnitz 7 - - 4 - - - - - 3 - - -
25. zge Chemnitz 7 - - - - - 5 2 - - - - -
26. XXX ??? 6 - - - - - - 3 - - - - 3
26. Ole Koelb Chemnitz 6 3 - - - - - - - 3 - - -
26. Laura Schlosser Chemnitz 6 - - - - - 5 1 - - - - -
26. Luisa Schlosser Chemnitz 6 - - - - - 5 1 - - - - -
26. Saskia Schlosser Chemnitz 6 - - - - - 5 1 - - - - -
26. Astrid Fischer Chemnitz 6 3 - - - - - 3 - - - - -
27. Felicitas Güra Chemnitz 5 - - - - - - - - - 3 - -
27. Franz Artur Chemnitz 5 - - - - - 5 - - - - - -
27. Helene Fischer Chemnitz 5 - - - 5 - - - - - - - -
27. Felix Taubert Chemnitz 5 - - - - - - 2 - 3 - - -
27. Leon Hoppe Chemnitz 5 - - - 5 - - - - - - - -
27. Vincent Baessler Chemnitz 5 - - - - - 5 - - - - - -
27. Julia Ritter Chemnitz 5 - - - - - 5 - - - - - -
27. Tobias Morgenstern Chemnitz 5 - - - 5 - - - - - - - -
27. Lisa Berger Chemnitz 5 - - - - - 5 - - - - - -
27. Florian A. Schönherr Chemnitz 5 - - - - - 5 - - - - - -
27. Wim Winter Chemnitz 5 3 - 2 - - - - - - - - -
28. Ida Heuschkel Chemnitz 4 - - - 4 - - - - - - - -
28. Christian Wagner Bamberg 4 4 - - - - - - - - - - -
28. Luis Raupach Chemnitz 4 - - 4 - - - - - - - - -
28. Lewis Knittel Chemnitz 4 - - - - - 4 - - - - - -
28. Lukas Kirchberg Chemnitz 4 - - 4 - - - - - - - - -
28. Lukas Thieme Chemnitz 4 - - - 4 - - - - - - - -
28. Andreas M. Dittersdorf 4 4 - - - - - - - - - - -
29. Celestina Montero Perez Chemnitz 3 - - - 3 - - - - - - - -
29. Lisanne Brinkel Chemnitz 3 - - - - - - - - - 3 - -
29. Emma Irmscher Eibenberg 3 - - - - - - 3 - - - - -
29. Hannes Hohmann Chemnitz 3 - - - 3 - - - - - - - -
29. Daniel Hufenbach Leipzig 3 - 3 - - - - - - - - - -
29. Nicklas Reichert Chemnitz 3 - - - 3 - - - - - - - -
29. Svenja Reinelt Chemnitz 3 - - 3 - - - - - - - - -
29. Albin Uhlig Chemnitz 3 - - - 3 - - - - - - - -
29. Ole Weiß Chemnitz 3 - - - - - - - - - - 3 -
29. Janosch Fiebig Chemnitz 3 - - - - - - - - - - 3 -
29. Elena Oelschlägel Chemnitz 3 - - - - - - - - - 3 - -
29. Heinrich Grossinger Chemnitz 3 - - - - - - - - - 3 - -
29. Selma Juhran Chemnitz 3 - - - 3 - - - - - - - -
29. Pauline Marschk Chemnitz 3 - - 3 - - - - - - - - -
29. Tim Jechorek Chemnitz 3 - - - - - - - - 3 - - -
29. Frederike Meiser Chemnitz 3 - - - 3 - - - - - - - -
29. Marie Berger Chemnitz 3 - - - 3 - - - - - - - -
29. Paula Mühlmann Dittersdorf 3 - - - 3 - - - - - - - -
29. Jessica Ritter Chemnitz 3 - - - - - - - - - 3 - -
29. Simon Winger Chemnitz 3 - - - - - - - - - 3 - -
29. Valentin Sellin Chemnitz 3 - - - - - - - - - 3 - -
29. Tom Straßer Chemnitz 3 - - - - - - - - - 3 - -
29. Charline Patzelt Chemnitz 3 - - - 3 - - - - - - - -
29. Tobias Richter Chemnitz 3 - - - 3 - - - - - - - -
29. Katharina Zweiniger Chemnitz 3 - - - 3 - - - - - - - -
29. Lene Haag Chemnitz 3 - - - 3 - - - - - - - -
29. Moritz Weber Chemnitz 3 - - - - - - - - - 3 - -
29. Gunnar Reinelt Chemnitz 3 - - - - - - - - - 3 - -
29. Karl Herrmann Chemnitz 3 - - - - - - - - - 3 - -
29. Marvin Köllner Chemnitz 3 - - - - - - - - - 3 - -
29. Melanie Petz Chemnitz 3 - - - - - - - - - 3 - -
29. Elina Rech Chemnitz 3 - - - - - - - - - 3 - -
29. Cynthia Raschkowsky Chemnitz 3 - - - - - - - - - 3 - -
29. Simon Anders Chemnitz 3 - - - - - - - - - 3 - -
29. Felicitas Hastedt Chemnitz 3 - - - - - - - - - 3 - -
29. Lilli Weiß Chemnitz 3 - - - - - - - - - 3 - -
29. Valentin Grundmann Chemnitz 3 - - - - - - - - - 3 - -
29. Ulrike Böhme Chemnitz 3 - - - - - - - - - 3 - -
29. Anna Georgi Chemnitz 3 - - - - - - - - - 1 - -
30. Alex Gähler Chemnitz 2 - - - - - - - 2 - - - -
30. Anna Grünert Chemnitz 2 - - - - - - - - - 2 - -
30. Marie Juhran Chemnitz 2 - - - 2 - - - - - - - -
30. Carlo Klemm Chemnitz 2 - - - 2 - - - - - - - -
30. Agnieszka Urban Chemnitz 2 - - - - - - - 2 - - - -
30. Joel Magyar Chemnitz 2 - - - 2 - - - - - - - -
30. Marvin Gülden Chemnitz 2 - - - - - - - - - 2 - -
30. Franz Kemter Chemnitz 2 - - - 2 - - - - - - - -
30. Nele Mäding Chemnitz 2 - - - - - - - - - 2 - -
30. Shari Schmidt Chemnitz 2 - - - 2 - - - - - - - -
30. Josephine Klotz Chemnitz 2 - - - - - - - - - 2 - -
30. Kevin Ngyen Chemnitz 2 - - - 2 - - - - - - - -
30. Jessica Spindler Chemnitz 2 - - - 2 - - - - - - - -
31. Justine Schlächter Chemnitz 1 - - - - - - - - - 1 - -
31. Erik Walther Chemnitz 1 - - - - - - - - - 1 - -
31. Luisa Franke Chemnitz 1 - - - - - - - - - 1 - -
31. Clara Stöckel Chemnitz 1 - - - - - - - - - 1 - -
31. Hanna Kallenbach Chemnitz 1 - - - - - - - - - 1 - -
31. Joshua May Chemnitz 1 - - - - - - - - - 1 - -
31. Emmely Schöne Chemnitz 1 - - - - - - - - - 1 - -
31. Maxi John Chemnitz 1 - - - - - - - - - 1 - -
31. Malte Gebhardt Chemnitz 1 - - - - - - - - - 1 - -
31. Alina Berger Chemnitz 1 - - - - - - - - - 1 - -
31. Tim Missullis Chemnitz 1 - - - - - - - - - 1 - -
31. Jule Irmscher Eibenberg 1 - - - - - - - - - 1 - -
31. Eva-Lotta Rümmler Chemnitz 1 - - - - - - - - - 1 - -
31. Lina Krug Chemnitz 1 - - - - - - - - - 1 - -
31. Tim Sigmund Chemnitz 1 - - - - - - - - - 1 - -
31. Laurin Roßberg Chemnitz 1 - - - - - - - - - 1 - -
31. Michelle Wade Chemnitz 1 - - - - - - - - - 1 - -

Auswertung Serie 27 (rote Liste)
Platz Name Ort Summe Aufgabe

313 314 315 316 317 318 319 320 321 322 323 324
1. Jürgen Urbig Chemnitz 52 4 4 4 8 5 3 4 2 4 6 4 4
2. Uwe Parsche Chemnitz 48 4 4 4 8 5 2 4 3 - 6 4 4
3. Doreen Naumann Duisburg 44 4 4 4 4 3 3 4 3 2 5 4 4
4. Sabine Fischbach Hessen 37 4 2 4 5 3 3 2 3 - 3 4 4
5. Arne Weißbach Chemnitz 31 3 - 4 5 - 3 4 2 2 - 4 4
6. Andree Dammann München 30 4 4 3 3 3 2 4 - 3 - 4 -
7. Linus-Valentin Lohs Chemnitz 26 3 3 3 - 4 - 2 3 - - 4 4
8. Felix Haase Chemnitz 24 4 4 4 5 - 1 4 2 - - - -
9. Rafael Seidel Chemnitz 23 - - 4 - 5 3 - 3 - - 4 4
10. Jamila Wähner Chemnitz 21 4 4 4 3 3 3 - - - - - -
11. Richard Hahmann Chemnitz 20 4 4 - 4 - 3 3 2 - - - -
11. Anja Posselt Chemnitz 20 4 4 4 3 2 3 - - - - - -
12. Elisa Parsche Chemnitz 18 4 - 4 - - 3 - 3 - - - 4
12. Hermann Thum Chemnitz 18 3 4 4 4 - - - 3 - - - -
13. Duncan Mahlendorff Chemnitz 17 - 4 - 4 - 3 3 3 - - - -
13. Robin König Chemnitz 17 4 4 4 - - 1 3 - - - - 1
14. Loise Reichmann Chemnitz 15 4 4 4 2 - - 1 - - - - -
15. Stephanie Dani Chemnitz 14 4 4 4 - - - - 2 - - - -
16. Marie Sophie Roß Chemnitz 13 4 4 3 2 - - - - - - - -
17. Ellen Richter Chemnitz 12 3 - 4 2 2 - 1 - - - - -
17. Karolin Schuricht Chemnitz 12 3 - 1 - - 3 2 3 - - - -
17. Paula Hartmannsdorf 12 - 4 4 - - - 4 - - - - -
18. Lisa Grassmann Chemnitz 11 4 3 4 - - - - - - - - -
19. Johanna Ranft Chemnitz 10 4 - 4 2 - - - - - - - -
19. Jonathan Kässler Chemnitz 10 4 - 4 - - - - 2 - - - -
19. Philipp Fürstenberg Chemnitz 10 4 - 4 - - - - 2 - - - -
19. Ria Hopke Chemnitz 10 3 - 4 2 - - 1 - - - - -
20. Nina Zschätzsch Chemnitz 9 4 - 4 1 - - - - - - - -
20. Emily Neuwirth Chemnitz 9 4 - 4 - - - - - 1 - - -
21. XXX ??? 8 - - - - - - 4 - - - - 4
21. Marion Sarah Zenk Chemnitz 8 3 1 4 - - - - - - - - -
22. Astrid Fischer Chemnitz 7 3 - - - - - 4 - - - - -
22. Josephine Pallus Chemnitz 7 3 - - 4 - - - - - - - -
22. Lucas Steinke Chemnitz 7 4 - - - - - 1 2 - - - -
23. Luisa Schlosser Chemnitz 5 - - - - - 3 2 - - - - -
23. Laura Schlosser Chemnitz 5 - - - - - 3 2 - - - - -
23. Lena Elisa Penzlin Chemnitz 5 - - 4 - - - - - 1 - - -
24. Andreas M. Dittersdorf 4 4 - - - - - - - - - - -
24. Ingmar Richter Chemnitz 4 3 - - - - - 1 - - - - -
24. Christian Wagner Bamberg 4 4 - - - - - - - - - - -
24. Daniel Hufenbach Leipzig 4 - 4 - - - - - - - - - -
25. Pauline Marschk Chemnitz 3 - - 3 - - - - - - - - -
25. Kai-Lutz Wagner Chemnitz 3 3 - - - - - - - - - - -
25. Mara Neudert Chemnitz 3 - - - - - 3 - - - - - -
25. Saskia Schlosser Chemnitz 3 - - - - - 3 0 - - - - -
25. Svenja Reinelt Chemnitz 3 - - 3 - - - - - - - - -
25. Rebecca Wagner Oberwiesenthal 3 - - - 3 - - - - - - - -
25. zge Chemnitz 3 - - - - - 3 - - - - - -
25. Lisa Berger Chemnitz 3 - - - - - 3 - - - - - -
25. Janosch Fiebig Chemnitz 3 - - - - - - - - - - 3 -
25. Ole Weiß Chemnitz 3 - - - - - - - - - - 3 -
26. Lisanne Brinkel Chemnitz 2 - - - - - - - - - 2 - -
26. Gunnar Reinelt Chemnitz 2 - - - - - - - - - 2 - -
26. Heinrich Grossinger Chemnitz 2 - - - - - - - - - 2 - -
26. Simon Winger Chemnitz 2 - - - - - - - - - 2 - -
26. Alex Gähler Chemnitz 2 - - - - - - - 2 - - - -
26. Anna Grünert Chemnitz 2 - - - - - - - - - 2 - -
26. Marvin Gülden Chemnitz 2 - - - - - - - - - 2 - -
26. Valentin Grundmann Chemnitz 2 - - - - - - - - - 2 - -
26. Felicitas Hastedt Chemnitz 2 - - - - - - - - - 2 - -
26. Cynthia Raschkowsky Chemnitz 2 - - - - - - - - - 2 - -
26. Nele Mäding Chemnitz 2 - - - - - - - - - 2 - -
26. Anna Georgi Chemnitz 2 - - - - - - - - - 2 - -
26. Jessica Ritter Chemnitz 2 - - - - - - - - - 2 - -
27. Simon Anders Chemnitz 1 - - - - - - - - - 1 - -
27. Adrian Schlegel Chemnitz 1 - - - - - - - - - 1 - -
27. Tom Straßer Chemnitz 1 - - - - - - - - - 1 - -
27. Melanie Petz Chemnitz 1 - - - - - - - - - 1 - -
27. Elina Rech Chemnitz 1 - - - - - - - - - 1 - -
27. Elena Oelschlägel Chemnitz 1 - - - - - - - - - 1 - -
27. Lilli Weiß Chemnitz 1 - - - - - - - - - 1 - -
27. Moritz Weber Chemnitz 1 - - - - - - - - - 1 - -
27. Theresa Jänich Chemnitz 1 - - - - - - - - 1 - - -
27. Karl Herrmann Chemnitz 1 - - - - - - - - - 1 - -
27. Valentin Sellin Chemnitz 1 - - - - - - - - - 1 - -
27. Ulrike Böhme Chemnitz 1 - - - - - - - - - 1 - -

Serie-26

Serie 26

Aufgaben und Lösungen


Aufgabe 1
301. Wertungsaufgabe


"Hallo Mike, du warst doch letzte Woche zu dem Treffen der Wochenaufgabenentwerfer in Potsdam. War das interessant?" "Aber selbstverständlich." Na dann erzähl doch mal," drängelte Lisa.
"Hier lest einfach meine Notizen."
Es sind 5 Schüler, die haben  alle in diesem Jahr begonnen, Wochenaufgaben ins Netz zu stellen. Start war Januar, Februar, April, Mai bzw. Juli. Es sind zwei Jungs, Bert und Helmut, sowie die Mädchen Elke, Gina und Melanie. Angereist kamen sie aus Freiberg, Berlin, Zwickau, Köln und München. Keiner kam aus seiner Heimatstadt (Hannover, Dresden, Hamburg, Leipzig und Rostock), in denen sie ihre Wochenaufgaben veröffentlichen.
1..Einen Monat bevor Bert begann, seine Wochenaufgaben zu veröffentlichen, begann der/die Leipziger/Leipzigerin, der/die aus Freiberg angereist kam, mit seinen Aufgaben.
2..Den Start der Wochenaufgaben im Februar unternahm nicht der/die aus Köln angereiste und nicht der/die in Hannover wohnende.
3..Der Start der Veröffentlichung des (oder der) aus München angereisten war eher wie  des Hamburgers (oder Hamburgerin) und genau drei Monate früher als der Start durch die Rostockerin Gina.
4..Melanies Veröffentlichung war einen Monat eher als der Start von Elke, die  aus Berlin anreiste.
Wer kam aus welcher Stadt nach Potsdam? Wo wohnen die Leute? Wann begannen die Veröffentlichungen? 6 blaue Punkte
Begleitet wurden die Schüler durch ihre Mathelehrer, die alle verschiedene Augenfarben -- blau, grün, grau, braun und rötlich- hatten. Sie hielten alle einen kurzen Vortrag, diese begannen 9.50 Uhr, 10.00 Uhr, 10.20 Uhr 10.40 Uhr und 10.50 Uhr. Auf dem Rednerpult stand jedesmal etwas anderes. Saft, Milch, Tonic, Limo und Wasser. Ich schätzte die Lehrer  auf 25 , 40, 50, 55 und 60 Jahre.
1.Der Lehrer mit den brauen Augen und dem Saft ist 10 Jahre älter als der, welcher eine halbe Stunde eher mit seinem Vortrag begann.
2. Der 55-jährige begann 20 Minuten später als der Milchtrinker, der wiederum älter ist als der Limotrinker mit den grünen Augen.
3. Der Wassertrinker begann seinen Vortrag 20 Minuten eher als der mit den rötlichen Augen.
4. Der mit den blauen Augen war der Jüngste.
Wer (Augenfarbe) hielt wann seinen Vortag und trank dazu welches Getränk? 6 rote Punkte
"Sag mal, war da der Mathelehrer aus Chemnitz, der sich schon so viele Aufgaben ausgedacht hat, auch dabei?" "Nö, das ist doch auch wieder nur so eine Aufgabe von dem."

Lösung:
Die Tabellen im Sinne der Aufgaben des Logiktrainers sind Uwe, danke.
Januar Februar April Mai Juli Hannover Düsseldorf Hamburg Leipzig Rostock Freiberg Berlin Zwickau Köln München
Bert - + - - - - + - - - - - + - -
Helmut + - - - - - - - + - + - - - -
Elke - - - + - - - + - - - + - - -
Gina - - - - + - - - - + - - - + -
Melanie - - + - - + - - - - - - - - +
Freiberg + - - - - - - - + -
Berlin - - - + - - - + - -
Zwickau - + - - - - + - - -
Köln - - - - + - - - - +
München - - + - - + - - - -
Hannover - - + - -
Düsseldorf - + - - -
Hamburg - - - + -
Leipzig + - - - -
Rostock - - - - +
Daraus lässt sich ablesen: Wann, wer, wohnt in, kommt aus:
Januar - Helmut - Leipzig - Freiberg
Februar - Bert - Dresden - Zwickau
April - Melanie - Hannover - München
Juli - Gina - Rostock - Köln
rote Aufgabe:
Januar Februar April Mai Juli 25 Jahre 40 Jahre 50 Jahre 55 Jahre 60 Jahre Saft Milch Tonic Limo Wasser
blau - + - - - + - - - - - - - - +
grün + - - - - - + - - - - - - + -
grau - - - + - - - - + - - - + - -
braun - - - - + - - - - + + - - - -
rötlich - - + - - - - + - - - + - - -
Saft - - - - + - - - - +
Milch - - + - - - - + - -
Tonic - - - + - - - - + -
Limo + - - - - - + - - -
Wasser - + - - - + - - - -
25 Jahre - + - - -
40 Jahre + - - - -
50 Jahre - - + - -
55 Jahre - - - + -
60 Jahre - - - - +

Daraus lässt sich ablesen: Vortragsbeginn, Augenfarbe, Getränk, Alter:
9.50 Uhr - grün - Limo - 40
10.00 Uhr - blau - Wasser - 25
10.20 Uhr - rötlich - Milch - 50
10.40 Uhr - grau - Tonic - 55
10.50 Uhr - braun - Saft - 60



Aufgabe 2

302. Wertungsaufgabe
"Hallo Lisa." Hallo Mike, was machst du denn mit dem Zettel zum kleinen 1 x 1.?" Das ist nicht das ganze 1  x 1, sondern es sind  nur die zweistelligen Ergebnisse davon." Wie viele solcher Ergebnisse gibt es eigentlich und lässt sich daraus die Anzahl der Primzahlen zwischen 10 und 100 ermitteln? (Kleines Einmaleins bedeutet beide Faktoren sind einstellig.) 3 blaue Punkte. Aus den Ergebnissen der blauen Aufgabe soll eine 9-stellige Zahl zusammengestellt werden, deren Ziffern alle verschieden sind (keine Null). Je zwei Ziffern, die nebeneinander stehen, bilden ein Produkt des kleinen Einmaleins. 5 rote Punkte. Beispiel für 5-stellig 32481 -- 32; 24; 48 und 81 sind Produkte des kleinen Einmaleins.

Lösung:
Blau: Alle Ergebnisse des Kleinen 1 x 1:
10,12,14,15,16,18, 20, 21, 24, 25, 27, 28, 30, 32 ,35, 36, 40, 42, 45, 48, 49, 54, 56, 63, 64,72, 81 ==> 27 Ergebnisse. Es gibt natürlich Ergebnisse, die doppelt vorkommen (18 = 2*9 = 9*2, aber danach war nicht gefragt. Auch das häufig auch die 10-er Reihe mit zum kleinen 1 x1 gezählt wird ändert nicht am zweiten Teil der Aussage ...-->)  Streicht man aus einer Tabelle der Zahlen von 10 bis 99 die Ergebnisse heraus, so bleiben auch Zahlen Stehen, die keine Primzahlen sind, z.B. 91 = 9*13. Damit lässt sich die Anzahl der Primzahlen also so nicht ermitteln.
rot: 23 Ergebnisse aus blau enthalten keine Null und sind Kandidaten für die gesuchte 9-stellige Zahl. Die Ziffer 9 kommt nur einmal vor, also muss die gesuchte Zahl xxxxxxx49 heißen. Zwei Ziffern geschafft.
Ziffer 7 gibt es in 27 und 72. Würde die 7 irgendwo in der Mitte der Zahl stehen, so wäre sie von zwei Zweien eingeschlossen, was aber nicht sein darf, damit muss die 7 an den Anfang. ==> 72xxxxx49. Was kommt vor die 4. Da wären 72xxxx149, 72xxxx549 oder 72xxxx649.
72xxxx149 ==> 72xxx8149 ==> 72xx18149 nicht zulässig.
72xxxx649 ==> 72xxx1649 oder 72xxx3649 oder 72xxx5649 ==> ... weiteres systematisches Einsetzen führt in jedem Fall auf eine nicht zulässige Zahl, damit bleibt nur
72xxxx549
==> 72xxx1549 oder 72xxx3549 ==> Untersuchung erste Variante
72xx81549 ==> Widerspruch, denn die auf 8 endenden Produkte fürfen nicht genommen werden werden (wegen 1 oder 2 doppelt) ==>
es bleibt 72xxx3549 ==>
72xx63549 ==>72x163549 ==> 728163549. Die systemtische Untersuchung zeigt es gibt genau eine solche gesuchte Zahl.


Aufgabe 3

303. Wertungsaufgabe
"Lisa, du hast ja schon wieder die Schere in der Hand. Willst du wieder Schachbretter zerschneiden?, grinste Bernd. "Nein, ich habe regelmäßige Vielecke ausgeschnitten, die alle die gleiche Kantenlänge haben. So kann ich sowohl die gleiche Art auf Kante legen, aber eben auch verschiedene." "Auf Kante legen?" "Kante an Kante, ohne das was übersteht."  Bernd spielt gleich mal ein wenig. "Wenn ich nur die Dreiecke nehme dann passen diese super zusammen, aber auch die Sechsecke. Nehme ich allerdings Achtecke, dann bleiben Lücken." Was passt in die Lücken hinein und warum? (2 + 3 blaue Punkte). Rot Gesucht sind 3 verschiedene regelmäßige Vielecke mit jeweils gleicher Kantenlänge, die paarweise auf Kante liegen, aber die gemeinsame Ecke komplett ausfüllen. (Bei blau sind es aber eben zwei Achtecke -- also leider nicht verschieden -- und das gesuchte n-Eck, was so eine gemeinsame Ecke komplett ausfüllt. Pro Tripel je zwei Punkte.

Lösung:
blau: Die Innenwinkel eines regelmäßigen Achtecks sind 135° groß - Begründung bei rot. Legt man zwei solche Achtecke auf Kante, so ergeben sich 2*135° = 270° an den Eckpunkten der gemeinsamen Kante. Um diese Lücke zu schließen, muss die gesuchte Fläche (360° - 270° = ) 90° große Innenwinkel besitzen. Da die Kanten alle gleichlang sein sollen, handelt es sich also um ein Quadrat.
rot: Die Innenwinkelsumme eines n-Ecks lässt sich (n - 2 ) * 180° berechnen. (Für die ganz genau Lesenden - ebenes konvexes n-Eck).  Da es sich bei der Aufgaben Stellung um regelmäßige n-Ecke handeln soll ergibt sich für den einzelnen Innenwinkel  \frac {(n-2) * 180^\circ} n. Um eine Ecke mit drei verschiedenen n-Ecken auszufüllen muss also  \frac {(n_1-2) * 180^\circ} {n_1} + \frac {(n_2-2) * 180^\circ} {n_2} + \frac {(n_3-2) * 180^\circ} {n_3} = 360^\circ gelten. Dabei sollen die einzelnen Werte für n alle verschieden sein. Wird die Gleichung durch 180° dividiert, dann erhält man:
 \frac {(n_1-2)} {n_1} + \frac {(n_2-2)} {n_2} + \frac {(n_3-2)} {n_3} = 2
Jetzt gibt es verschiedene Möglichkeiten die Überlegungen fortzusetzen. So lässt sich z.B. die Gleichung nach n3 umstellen und dann systematisch probieren.  Oder man rechnet per Tabellenkalkulation durch, ... Wie auch immer.
Es gibt genau 6 verschiedene Lösungen - den Nachweis, dass dies alle sind, überlasse ich dem geneigten Leser.
Die Zahlen geben die Anzahl der Ecken an:
(3; 7; 42) - krumme Gradgrößen, deshalb nur mit Probieren ganzahliger Gradzahlen nicht zu finden
(3; 8; 24)
(3; 9; 18)
(3; 10; 15)
(4; 5; 20)
(4; 6; 12)





Aufgabe 4

304. Wertungsaufgabe
Maria konstruiert mit dem Zirkel ein regelmäßiges Achteck. Dies hat eine Kantenlänge von 6 cm. Die Bezeichnung der Eckpunkte ist wie üblich ABCDEFGH. Nun aber sitzt sie grübelnd vor ihrem Blatt. Bernd kommt ins Zimmer und fragt, was denn sei. "Ich möchte ein  weiteres Achteck in das Achteckeck hinein konstruieren. Der Punkt E soll auch Eckpunkt des neuen Achtecks sein. Zwei Seiten sollen auf den Seiten d und e liegen und eine dritte Seite auf der Diagonalen AD." "Ich verstehe," meint Bernd, nach dem er sich eine Skizze angefertigt hat. 6 blaue Punkte für eine begründete Konstruktionsbeschreibung. 5 rote Punkte für die Berechnung der Kantenlänge des kleinen Achtecks. (Anmerkung: Es ist immer ein regelmäßiges Achteck gemeint.)
Lösung:
Das regelmäßige Achteck bei vorgebener Seitenlänge zu konstruieren, ist auf mehrere Arten möglich. Hier eine Variante.
Zeichne eine Strecke AB mit 6 cm. Die Strecke wird über den Punkt B hinaus verlängert. Nun wird in B eine Senkrechte errichtet (Grundkonstruktion). Der rechte Winkel, der nicht die Strecke AB einschließt, wird halbiert (Grundkonstruktion). Der Winkel zwischen AB und der Winkelhalbierenden ist somit 135° groß. Genau das aber ist die Größe der Innenwinkel eines regelmäßigen Achtecks. Es wird nun von B aus 6 cm auf der Winkelhalbierenden abgetragen und man erhält den Punkt C des Achtecks. Eine Wiederholung der Schritte führt zum gesuchten großen Achteck.
304
Jetzt wird die Diagonale AD eigezeichnet. Siehe Bild von Uwe, danke.
Das Prinzip ist jetzt erkennbar. Das kleine Achteck lässt sich als Bild einer zentrischen Streckung des großen Achtecks mit dem Streckungszentrum E auffassen, wobei B' und C' auf der Diagonalen liegen müssen. Damit liegt aber zugleich die Seitenlänge des kleinen Achtecks fest. Die Lage der anderen Punkte lassen sich durch Parallelverschiebungen ermitteln.
rot: Die Seitenlänge des kleinen Achtecks lässt sich nun mittels Pythagoras und Strahlensatz ermitteln. Gegebene Seitenlange sei a
EB' / EB = B'C'/ a
(EB-a) /EB = B'C'/a
Die Diagonale EB ist Wurzel (2)*a + a lang.
Wird die Länge der Diagonalen in die zweite Gleichung eingesetzt, so ergibt sich:
B'C' = a*  Wurzel (2)/  (Wurzel (2) + 1) das sind rund 0,5857864 * a. Das bei 6 cm Ausgangswert 3,5147186 cm für das kleine Achteck.
Sollte es Nachfragen geben, so können diese gern im Forum gestellt werden.


Aufgabe 5

305. Wertungsaufgabe
Bernd fährt auf dem Weg von Leipzig nach Chemnitz durch den Ort Narsdorf. Der Name des Ortes besteht aus 8 verschiedenen Buchstaben. Wie viele Orte mit dorf am Ende ließen sich mit den Buchstaben N A R und S bilden. 2 blaue Punkte. Wie viele echt verschiedene Wörter lassen sich aus den 8 Buchstaben bilden? Die Wörter haben die Längen 1 bis 8 Buchstaben. 5 rote Punkte.

Lösung:
Blau: Die ersten vier Buchstaben waren auf alle möglichen Arten anzuordnen, dies nennt man Permutation. Für die erste Stelle habe ich vier Möglichkeiten. Für jede der 4 Möglichkeiten. verbleiben für die 2. Stelle 3 Buchstaben. Das sind 4 * 3 Möglichkeiten. Für jede dieser Möglichkeiten kann ich aus zwei verbleiben Buchstaben auswählen. Das ergibt also 4*3*2 Möglichkeiten. Bei jeder dieser 24 Möglichkeiten kann ich den verbleiben 4. Buchstaben nur noch ergänzen. Es sind also 24 Möglichkeiten. Kurze Schreibweise 4! (sprich  vier Fakultät). n! (n>1) steht für 1*2*3* ... * n. siehe --> Mathelexikon <--
Anmerkung. Die "Namen" des zu bildenden Dorfes muss nicht lesbar bzw. aussprechbar sein.
rot: Hier sind nun für die 1 bis 8 Buchstaben langen "Worte"  von den acht Buchstaben des Wortes NARSDORF 1, zwei, drei, .... Buchstaben zu wählen.
Der Fachbegriff dafür ist Kombination, aber und jetzt kommt es, die bekannten Formeln für die  Kombination greifen nur, wenn alle Elemente (alle Buchstaben) verschieden sind. NARSDORF aber enthält das R eben zwei mal. Okay, wenn man NARSdorf schreiben, könnte man R und r unterscheiden, aber ...
Hier nun die Betrachtungen von XXX einmal NARSdorf und einmal NARSDORF, danke.
als pdf




Aufgabe 6

306. Wertungsaufgabe
Maria sitzt am Computer als Bernd in ihr Zimmer kommt. „Das sieht aber cool aus. Was ist das?“ „Ich lese auf der Schulhomepage gerade den Artikel zur Sierpinskipyramide. --> Zum Nachlesen<--  Dieses Modell wurde in der letzten Projektwoche gebaut.“ „Aus wie vielen kleinen Tetraedern mag wohl dieses Modell bestehen?“ „Das lässt sich ausrechnen. Pass auf. Ich gehe die Beschreibung noch mal durch. Man nimmt ein Ausgangstetraeder – Stufe 0. Nun werden alle Kanten des Tetraeders halbiert. Danach lassen sich also ausgehend von den Ecken des Ausgangstetraeders vier halb so große finden. Alles, was in der Mitte ist, kommt weg. Fertig ist die Stufe 1. Nun wird mit den verbleibenden Stufen-1-Tetraedern der Vorgang – Halbieren …  – wiederholt.  Das gebaute Sierpinskitetraeder entspricht der Stufe 5.“ Aus wie vielen kleinen Tetraedern besteht das Modell und was für ein Körper passt in die Lücke der Stufe 1? – 3+3 blaue Punkte (Nur Anzahl oder Name des Körpers bringt nicht die volle Punktzahl). Wie groß sind Oberfläche und Volumen der Stufe 5 im Vergleich zur Stufe 0, wenn die Kantenlänge in Stufe 0 bei 96 cm liegt? (8 rote Punkte)
Lösung:
blau:
Stufe 0 - 1 Tetraeder = 40
Stufe 1 - 4 Tetraeder = 41
Stufe 2 - 16 Tetraeder = 42
...
Stufe 5 - 1024 Tetraeder = 45
allgemein: Stufe n 4n Tetreder
Betrachtet man das Bild aus Aufgabe 307, so ist zu erkennen, dass die Figur, die in die Lücke passt 6 Ecken hat und 8 kongruente gleichseitige Dreiecke als Flächen besitzt ein Oktaeder ist.
rot: Betrachten wir zuerst den Übergang von Stufe 0 zur Stufe 1: Volumen. Die Stufe 1 besteht aus 4 Tetredern, die die halb so groß sind wie das Teraeder der Stufe 0. Damit ist das Volumen eines Tetraeders 1/8 des Ausgangsvolumens. Da es vier Tetraeder sind 4/8 oder 1/2 des Ausgangsvolumens. Oberfläche der Stufe 0 besteht aus vier zueinander kongruenten  gleichseitigen Dreiecken 4*A0. Die Stufe 1 besteht aus 16 halb so großen Dreiecken 16*A1. A1 ist aber 1/4 von A0. 16*1/4*A0 = 4A0.
Zusammengefasst: Beim Übergang von Stufe 0 zur Stufe 1 wird das Volumen halbiert, aber die Oberfläche bleibt. Wie man leicht nachvollziehen kann, passiert beim Übergang von Stufe 1 zur Stufe 2 das Gleiche:  das Volumen wird halbiert, aber die Oberfläche bleibt.
Allgemein: Vn = (1/2)n * V0 und An = A0
Setzt man die 96 cm der Stufe 0 in die Tetraederformel und dann in die obige Formel ein ergeben sich:
V5 = 3258 cm3 und A5 = 15963 cm2
Hier mal noch ein Bild einer passenden Mathematikbriefmarke:
Noch mehr Mathemarken --> hier <-- sierpinski





Aufgabe 7

307. Wertungsaufgabe
"Das Modell der letzten Woche würde ich am liebsten mit GEOMAG – Teilen nachbauen“, sagte Mike, als er sich das Bild des Modells ebenfalls im Internet angeschaut hatte. „GEOMAG, das ist doch der Magnetbaukasten, oder?“ „Aber ja doch. Du hast gleichlange magnetische Stäbe. Als Ecken nimmt man Stahlkugeln. Wenn du ein Modell der Stufe 1 baust – siehe Aufgabe 306 – dann sieht man die Lücken nicht wirklich. Es sei denn, du verwendest dreieckige Panele als Seiten für die äußeren kleinen Tetraeder. Wie viele Stäbe und  Kugeln werden für die Stufe 2 benötigt? 4 blaue Punkte – kleine Herleitung nicht vergessen. 4 rote Punkte für die Anzahl von Stäben und Kugeln der Stufe n, n – beliebige natürliche  Zahl.
Bild der Stufe 1:307

Lösung:
blau:
Stufe 0: 1*6 Stäbe + 4 Kugeln.
Das Bild zeigt die Stufe 1.
Es sind 4*6 Stäbe (24) und 4*4- 6 (10) Kugeln.
Für die Stufe zwei müssen nun vier solche Tetraeder benutzt werden, wobei die 3 oberen Kugeln der "unten stehenden" Tetraeder doppelt verwendet werden, aber die drei Kugeln, wo sich die unteren Tetraeder berühren.
24 * 6 = 96 Stäbe und 4 *10 - 6 = 34 Kugeln.
rot: n - die Stufe Anzahl der Stäbe 4n * 6 das ist leicht zu sehen.
Für die Zahl der Kugeln gab es mehere Ansätze und so waren die Formeln letztlich auch unterschiedlich "kompliziert". Richtig allerdings waren die alle.
Einen interessanten Ansatz fand Rafael.
Für die Anzahl der Kugeln multiplizierte der die Anzahl der Stangen mit 2 - macht Sinn. Um nun auf die obigen (blauen) Zahlen zu kommen wurde das Produkt durch 6 dividiert und dann kamen zwei Kugeln dazu. Setzt man nun die Anzahl der Stäbe mit 4n * 6 ein, dann ergibt sich nach wenigen Schritten: Anzahl der Kugeln K = 2(4n+1).
Wer das Ganze mal größer bauen will, der sollte folgende Anzahl Kugeln und Stäbe sich besorgen.
Stufe Stäbe Kugeln
0 6 4
1 24 10
2 96 34
3 384 130
4 1536 514
5 6144 2050
6 24576 8194
7 98304 32770
8 393216 131074
9 1572864 524290
10 6291456 2097154
11 25165824 8388610
12 100663296 33554434
13 402653184 134217730
14 1610612736 536870914
15 6442450944 2147483650
16 25769803776 8589934594
17 103079215104 34359738370
18 412316860416 137438953474
19 1649267441664 549755813890
20 6597069766656 2199023255554
21 26388279066624 8796093022210
22 105553116266496 35184372088834
23 422212465065984 140737488355330




Aufgabe 8

308. Wertungsaufgabe
"Was machst du Schönes?", fragte Bernd seine Schwester. "Ich habe das Geheimnis der Zahl 7 entdeckt und werde in meiner Spezialistengruppe diesen Mythos untersuchen." Klingt interessant, lass hören." "Hier die Aufgabe:"1234567654321 *(1+2+3+4+5+6+7+6+5+4+3+2+1) = 7 777 777 * 7 777 777
"Und das stimmt?" Formale Überprüfung durch Nachrechnen 3 blaue, Herleitung durch andere Überlegungen  noch mal 3 blaue Punkte.
"Kennst du ein anderes Geheimnis der 7?", fragte Maria ihren Bruder. "Ein Geheimnis ist es vielleicht nicht, aber die 7 soll dabei sein. Die Zahlen 5, 6 und 7 folgen ja aufeinander. Gesucht sind drei aufeinander folgende natürliche Zahlen (größer als 10), wo die erste durch 5, die zweite durch 6 und die dritte durch 7 teilbar sein soll und drei andere aufeinander folgende natürliche Zahlen (größer als 10), wo die erste durch 7, die zweite durch 6 und die dritte durch 5 teilbar sein soll." "Da gibt es doch sicher mehr als eine Lösung." "Aber klar doch." (je ein Tripel mit der geforderten Eigenschaft ist zu finden, je 2 rote Punkte, wer viele rote Punkte will -- 8 -- der sollte  drei andere aufeinander folgende natürliche Zahlen finden, wo die erste durch 307, die zweite durch 308 und die dritte durch 309 teilbar ist.)

Lösung:
blau: Mit einem "normalen" Taschenrechner funktioniert das Nachrechnen meist nicht, da es zu viele Ziffern sind, die es anzuzeigen gilt.
Das Ergebnis lautet für beide Seiten:  60 493 815 061 729
Die linke Seite 1234567654321 *(1+2+3+4+5+6+7+6+5+4+3+2+1) lässt sich zu 1234567654321 * 49 umwandeln. Die Linke Zahl hat eine einfache Struktur und wird nun untersucht:
11 * 11 = 121
111 * 111 = 12321
1111 * 1111 = 1234321 ...
1234567654321 * 49 = 1111111 * 1111111 * 7 * 7 = 7 * 1111111 * 1111111 * 7 = 7777777 * 7777777
rot: die Grundlage für diese Aufgabe ist der chinesische Restsatz. Ohne diesen zu erwähnen geht es auch. Hier die Lösung von Uwe Parsche, danke
1. Tripel teilbar durch 5, 6 bzw. 7: 215, 216, 217
weitere Tripel ergeben sich aus {5, 6, 7} + n*5*6*7 mit n .. natürliche Zahlen
z.B.: mit n = 6: --> 1265; 1266; 1267
z.B.: mit n = 7: --> 1475; 1476; 1477
z.B.: mit n = 8: --> 1685; 1686; 1687
z.B.: mit n = 10: --> 1895; 1896; 1897
z.B.: mit n = 10: --> 2105; 2106; 2107
z.B.: mit n = 37: --> 7775; 7776; 7777 --> Geheimnis der 7 ???
2. Tripel teilbar durch 7, 6 bzw. 5: 203, 204, 205
weitere Tripel ergeben sich aus {-7, -6, -5} + n*5*6*7 mit n .. natürliche Zahlen
z.B.: mit n = 6: --> 1253; 1254; 1255
z.B.: mit n = 7: --> 1463; 1464; 1465
z.B.: mit n = 8: --> 1673; 1674; 1675
z.B.: mit n = 9: --> 1883; 1884; 1885
z.B.: mit n = 10: --> 2093; 2094; 2095

3. Tripel teilbar durch 307, 308 bzw. 309: 29218111, 29218112, 29218113

weitere Tripel ergeben sich aus {307, 308, 309} + n*307*308*309 mit n .. natürliche Zahlen

3. Tripel teilbar durch 307, 308 bzw. 309: 29218111, 29218112, 29218113

weitere Tripel ergeben sich aus {307, 308, 309} + n*307*308*309 mit n .. natürliche Zahlen

z.B.: mit n = 2: --> 58435915; 58435916; 58435917

z.B.: mit n = 3: --> 87653719; 87653720; 87653721

z.B.: mit n = 4: --> 116871523; 1168715234; 1168715235

z.B.: mit n = 5: --> 146089327; 146089328; 146089329

z.B.: mit n = 6: --> 175307131; 175307132; 175307133

z.B.: mit n = 73: --> 2132899999; 2132900000; 2132900001

z.B.: mit n = 242: --> 7070708875; 7070708876; 7070708877

allgemein gilt:

- zerlege die 3 oder k Grundzahlen in ihre Primzahlen

z.B.: 14; 15; 16 --> 2*7; 3*5; 2*2*2*2

- multipliziere alle Primzahlen

(evtl. kommen in der Zerlegung mehrere Primzahlen doppelt vor, diese nicht doppelt zählen)

z.B.: 2*2*2*2*3*5*7 = 1680

falls die Zahlen aufsteigend sind gilt:

{14, 15, 16} + n * 1680 mit n .. natürliche Zahlen

falls die Zahlen abfallend sind gilt:

{-16, -15, -14} + n * 1680 mit n .. natürliche Zahlen

 






Aufgabe 9

309. Wertungsaufgabe
Bernd stöbert in den Mathematikbriefmarken, die auf den Seiten des Chemnitzer Schulmodells zu finden sind. Bei Mathematik querbeet ist die vom Mathematikerkongress aus dem Jahr 1998 zu sehen. "Schau mal, die 11 kleinen Quadrate bilden (fast genau) wieder ein Quadrat. Aber interessant ist auch der Wert -- die 110." Wieso?", fragt Mike nach. "Nun, die 110 lässt sich auf drei unterschiedliche Arten als Summe von 3 verschiedenen Quadratzahlen bilden." 6 blaue Punkte (Nur das Vertauschen von Summanden zählt nicht als andere Lösung.)
kongress1998Es gibt genau eine natürliche Zahl, die kleiner ist als 110, die ebenfalls diese Eigenschaft hat. 3 rote Punkte. Für die Zahlenspezialisten: Gesucht ist eine Zahl, die sich auf vier verschiedene Arten als Summe von drei Quadratzahlen darstellen lässt. Dafür gibt es extra rote Punkte.

Lösung:
blau: Es gibt ja nicht so viele Quadratzahlen, die als Summanden in Frage kommen:
0; 1; 4; 9; 16; 25; 36; 49; 64; 81 und 100.
Beginnt mann mit der 100, so wird klar da geht nur 110 = 100 + 9 + 1
Die nächste große Quadratzahl ist ist 81: Hier gilt 81 + 29 = 110. Die 29 aber lässt sich durch 25 + 4  ersetzen, also gefunden 110 = 81 + 25 + 4.
Nun wird die 64 Verwendet, 110 = 64 +  46, aber die passende Zerlegung der 46 geht leider nicht.
Also mal noch die 49: 110 = 49 + 61 = 49 + 36 + 25 geschafft.
rot: Da die gesuchte Zahl kleiner als 110 sein soll, sind es erst einmal die gleichen Quadratzahlen wie oben. Mit Geduld und Ausdauer wird man (endlich) bei der 101 fündig:
12 + 62 + 82 = 101
22 + 42 + 92 = 101
42 + 62 + 72 = 101
Der Nachweis, dass es kleiner nicht geht, war nicht verlangt.
Extrarot: Eigentlich hat man ja mit der 101 eine solche gesuchte Zahl gefunden, denn 02 + 12 + 102 = 101, wenn man die die Null mit dazu nimmt.
Hier mal noch die Erweiterung der Lösung durch Uwe Parche (danke), der sein mathlab- Programm entsprechend hat laufen lassen: als pdf
Interessant aber auch der Ansatz von XXX, danke:
Interessant ist eine Zahl, die sich auf [mindestens] vier verschiedene Arten als Summe dreier verschiedener Quadrate ...
Wenn ich vier pythagoreische Tripel nehme, etwa (3,4,5), (5,12,13), (7,24,25), (8,15,17),
dann hat die Zahl 5*13*25*17 die geforderte Eigenschaft:
(5*13*25*17)² = (3*13*25*17)² +(4*13*25*17)²
... = (5*5*25*17)² + (5*12*25*17)²
... = (5*13*7*17)² + (5*13*24*17)²
... = (5*13*25*8)² + (5*13*25*15)²
So könnte man viele Beispiele konstruieren.




Aufgabe 10

310. Wertungsaufgabe

Maria und Lisa bereiten ihren Kurs vor. Bernd, der ins Zimmer schaut, sieht die beiden bei Ausschneiden. "Was macht ihr denn da?" "Wir bereiten verschiedene Kreisausschnitte vor, die dann zu Mantelflächen eines Kegels genutzt werden sollen. Unsere Vorlagen haben alle den gleichen Radius vom 8 cm. Dann wollen wir untersuchen, wie groß das Volumen des Kegels wird, welcher sich aus so einem Stück jeweils formen lässt." "Fehlt da nicht die Grundfläche der Kegel", fragt Bernd nach. "Das stimmt schon, aber für die Ermittlung des Volumens ist das vielleicht nicht so wild", antwortete Lisa. "Wie können eigentlich die Schüler aus der 5. und 6. Klasse, die in eurem Kurs sind, das Volumen ermitteln?". "Nun, die basteln den Zylinder, messen die Höhe und den Radius aus und setzten die Werte in die Volumenformel ein. Die älteren sollen aus den Vorgaben die notwendigen Stücke ausrechnen." "Das sollte schaffbar sein", meinte Mike, der inzwischen auch eingetroffen war.
Wie groß ist das Volumen, wenn aus einem Kreis (r = 8,0 cm) ein 90° - Stück herausgeschnitten und  der "Rest" als Mantel für einen Kegel genutzt wird -- 4 blaue Punkte. Wie groß muss das Stück sein,  damit das Volumen maximal wird? - 4 rote Punkte.

Für die Berechnung des Volumens nutzt man diese Formel: V=\frac{1}{3} \cdot \Pi \cdot r^2 \cdot h, wobei r der Radius des beim Falten entstehenden Kreises ist  und der griechische Buchstabe die Zahl 3,1415.... darstellt.

Lösung:
blau: Es haben einige Schüler den Kegel gebastelt und mit der dabei erreichten Messgenauigkeit ein Volumen von rund 202 cm3 erzielt.
310Auf dem Bildern erkennt man noch einmal die Zusammenhänge. Der Rand des "Tortenstücks" wird zum Umfang des Kreises mit dem Radius r, der die Grundfläche bildet.
Es gelten dann folgende Beziehungen:
\frac {\alpha}{360^\circ} = \frac {r}{s} ==>
1. r = \frac {\alpha \cdot s}{360^\circ}
Nach dem Satz des Pythagoras gilt dann noch: h2 = s2 - r2 ==>
2.  h = \sqrt{s^2 - r^2}
Setzt man s = 8 cm in 1. so erhält man r = 6 cm und mit Hilfe der 2. Gleichung dann h = 5,291 cm. Das Ergibt ein Volumen von V = 199,485 cm3.
rot: Einige Schüler der Klasse haben nun den Winkel in 30° verändert, gemessen und gerechnet und so herausgefunden, dass wenn aus dem Vollkreis  60° ausgeschnitten werden, das größte Volumen entsteht. - Gute Näherung, alle Achtung.
Mit Hilfe der obigen Formeln kann man natürlich auch systematisch probieren und so auf die Suche nach dem größten Volumen gehen, das war der Weg von Doreen N., die auf diesem Wege gefunden hat, dass der "auszuschneidende Winkel" bei 66,06° liegen muss. Uwe Parsche und Rafael (mit Papas Hilfe?) haben die obigen Formeln in die Volumenformel eingesetzt, so dass dieses nur noch s und \alpha enthält. Letztlich also eine Funktion von \alpha, davon wurde die erste Ableitung auf "Null" gesetzt <-> Suche nach dem Maximum und so erhält man ein maximales Volumen für \alpha = \sqrt{ \frac{2}{3}} \cdot 360^\circ = 293,938^\circ Das führt auf einen Ausschneidewinkel von 66,0612°.




Aufgabe 11

311. Wochenaufgabe
Bernds Vater klopft an die Tür zu Marias Zimmer und als er nach dem "Herein" ins Zimmer geht, steht er im Dunkeln. Die vier Freunde untersuchen nämlich mit dem alten Optikbaukasten vom Opa verschiedene Varianten der Entstehung von Kern und Halbschatten mit 2 bis 4 Lampen und verschiedenen Hindernissen. Anschließend übertragen Sie ihre Ergebnisse in Koordinatensysteme. Ein Experiment ist schon komplett. Den Zettel nimmt Bernds Vater mit aus dem Zimmer und sieht: Zwei punktförmige Lichtquellen in A (0; 0) und B (0; 5) beleuchten ein Hindernis  - dieses entspricht der Strecke von (5; 1) nach (5; 2). Blau: Bei welchem Punkt endet der Kernschatten -- kann auch konstruiert werden (3 Punkte). Rot: Wie groß ist der Flächeninhalt des Kernschattens. Berechnung basierend  auf der Verwendung der gegebenen Koordinaten. (4 Punkte)

Lösung:
311_k Auf dem Bild (auf das Bild klicken zum Vergrößern) sieht man die Umsetzung der Aufgabe. Der Kernschatten endet beim Schnittpunkt der "roten" und "grünen" Funktion. Der Kernschattenendet also beim Punkt  (6,25; 1,25).
Diesen Schnittpunkt kann man auch rechnerisch ermitteln.
Aus den Punkten (0;5) und (5; 2) (rot) ergibt sich die Funktionsgleichung: y = f(x) = -0,6 x + 5 und aus den Punkten (0; 0) und (5; 1) (grün) ergibt sich y = g(x) = 0,2x.
Schnittpunktberechnung:
-0,6 xs + 5 = 0,2 xs | + 0,6 xs
5 = 0,8 xs
xs = 6,25 Einsetzten in f(x) ergibt ys = 1,25.
Nimmt man als Grundseite für das Dreieck die Strecke von (5; 1) nach (5; 2), dann ist die dazu gehörige Höhe 1,25 (6,25 - 5) Mit  A = \frac {g \cdot h_g}{2} ergibt sich der Flächeninhalt zu 0,625 Flächeninhalten. (Wird als Längeneinheit 1 cm gewählt, so sind das 0,625 cm².  )





Aufgabe 12

312. Wochenaufgabe
"Hallo Bernd, was hast du denn da?", fragt Maria. "Nun, das ist eine coole Uhr, die mir mein Mathematiklehrer mal geborgt hat. Hier hast du auch noch eine Beschreibung dazu."
312Der Grundkörper der Uhr ist (war) ein 10 cm großer Würfel. Dieser wurde abgeschrägt und zwar so, dass die Kanten der schrägen Fläche ein Dreieck bilden, die den Diagonalen der Deckfläche der vorderen Fläche und der rechten Seitenfläche entsprechen. Man betrachte dazu das Bild. Die drei Pyramiden haben jeweils die gleiche Höhe. Damit sie sich drehen können, sind kleine Abstände zwischen ihnen bzw. dem Würfelrestkörper von je 3 mm. Die untere Pyramidenscheibe zeigt die Stunden, die mittlere Scheibe die Minuten und die kleine Pyramide steht für die Sekunden. Wenn man die Uhr um 12 Uhr startet, dann bilden Würfelrestkörper und die drei Pyramiden genau wieder den Urprungswürfel. Wann bilden dann die drei drehbaren Teile zum ersten Mal wieder eine "richtige" Pyramide? 5 blaue Punkte (Achtung, wie das ganze in Bezug auf den Würfelrest aussieht, ist egal.) Wie groß sind die Volumina der drei drehbaren Teile? 6 rote Punkte

Lösung:
blau: Das Problem lässt sich auf die Frage zurückführen, wann stehen bei einer Uhr die zeiger übereineinander. Für den Minutenzeiger gilt, dass er 12 mal schneller ist als der Stundenzeiger. Der Minutenzeiger bewegt sich mit der Zeit t so, dass er 360°*t zurücklegt, der Stundenzeiger nur 30°*t. Übereinander liegen die genau dann, wenn der Unterschied zwischen den beiden Werten für das gleichte t bei einem Vielfachen von 360° Grad liegt. 360°*t - 30°*t = n* 360° Das nach t umgestellt, führt auf t = 12/11 *n.  Nun haben wir es bei der Uhr aber damit zu tun, dass jede Scheibe ja schon nach 120° wieder in den °Ausgangszustand" kommt. Wegen 360= 3* 120  Wird demzufolge t = 4/11*n.
Die untere Scheibe (Stunden) und die mittlere Scheibe liegen also nach je 4/11 Stunden genau übereinander.
Eine vollständige Pyramide aber ist ja erst erreicht, wenn auch die Sekundepyramide "richtig" über den beiden Scheiben dreht. Wendet man das obige Prinzip noch einmmal an, so erkennt man, dass dies Übereinstimmung zum ersten Mal erst nach 4 Stunden erreicht werden kann. (Analog bei den Zeigern einer "normalen" Uhr heißt dass nur um 12.00 Uhr liegen alle Zeiger übereinander.) Auch wenn es bei beim Betrachten der eben fast so ausssieht als ob es zwischen durch klappt, ist es eben nur dann der Fall, wenn der "Ausgangswürfel" wieder komplett ist . Technisch kommt noch hinzu, dass sich die Sekundenpyramide nicht kontinuierlich dreht, sondern "Sekundensprünge" macht, aber das war für die Aufgabenstellung nicht zu berücksichtigen.
rot: Zuerst kann man die Pyramide betrachten, die vom Würfelabgeschnitten und zur Uhr umfunktioniert wird. Die komplette Pyramide (ohne Abstände) hat ein  \frac{1}{6} des Volumens des Würfels.  V = \frac {1}{3} \cdot A_G \cdot h AG ist in dem Fall eines halbe Quadratfläche und h entspricht der Kantenlänge. Das Volumen der Pyramide ist also 1000/6 cm³ = 166,666 ... cm³. Für die Weitere Berechnung gehe ich nun zur Grundfläche über, die durch das gleichseitige Dreieck gebildet wird. Die Seitenlängen entsprechen der Diagonalen der Quadratflächen (14,142 ... cm), somit ergibt sich für diese Grundfläche Ag eine Grundfläche von 86,60 ... cm².
Mit der obigen Volumenformel  h = \frac {3V}{A_G} ergibt sich eine Höhe (von der Schnittfläche zur Ecke) von 5,77 cm. Daraus lassen sich die Höhen für die einzelnen Scheiben ermitteln (5,77 - 0,9) : 3 = 1,62 cm.
Berechnung des Volumens für die Sekundenpyramide. Es wird noch die Kantenlänge für deren Grundfläche gebraucht. Diese lässt sich mit Hilfe des 2. Teils des Strahlensatzes ermitteln.  \frac {a_1}{14,142 cm} = \frac {1,62 cm}{5,77 cm} ==> a1 = 3,97 cm. Mit der obigen Formel  ergibt sich VS = 3,68 cm³.
Die Minuten- und die Stundenscheibe sind Pyramidenstümpfe. Dafür gibt es diese Formel V = \frac{1}{3} h \cdot (A_G + sqrt{A_G A_D} + A_D) D steht dabei bei für die Deckfläche und G für die Grundfläche. UnterAusnutzung der Formel für das gleichseitige Dreieck verändert sich die Formel zu V = \frac{1}{12} h \cdot sqrt {3} \cdot ({a_G}^2 + {a_G \cdot a_D} + {a_D}^2)
Für h ist jeweils 1,62 cm einzusetzen, das jeweilige aG bzw. aD ist mit dem Strahlensatz ermittelbar. Für die Minutenscheibe sind das: aD = 4,706 cm und aG = 8,676 cm. In die Formel eingesetzt ergibt das: VM = 32,32 cm³.
Entsprechend ergibt sich für die Stundenscheibe: VH = 92,02  cm³.



Die Auswertung der Serie 26

Auswertung Serie 26 (blaue Liste)

Platz Name Ort Summe Aufgabe
301 302 303 304 305 306 307 308 309 310 311 312
1. Rafael Seidel Chemnitz 56 6 3 5 6 2 6 4 6 6 4 3 5
2. Uwe Parsche Chemnitz 53 6 3 5 6 - 6 4 6 6 4 3 4
3. Doreen Naumann Duisburg 52 6 3 5 5 2 6 4 3 6 4 3 5
4. Linus-Valentin Lohs Chemnitz 46 6 3 2 6 2 6 - 3 6 4 3 5
5. Elisa Parsche Chemnitz 39 - 3 5 6 - 6 4 6 6 - 3 -
6. Felix Haase Chemnitz 30 6 3 5 6 - - 4 3 - - - 3
7. Hermann Thum Chemnitz 28 6 - - 6 2 - 2 - 6 - 3 3
8. Richard Hahmann Chemnitz 26 - 2 - 5 2 - 4 1 6 - 3 3
9. Loise Reichmann Chemnitz 25 - - 5 5 2 - 4 3 - - 3 3
10. Jamila Wähner Chemnitz 24 - 3 - - 2 6 4 6 - - - 3
11. Stephanie Dani Chemnitz 22 - 1 5 5 2 - - 3 - - 3 3
12. Philipp Fürstenberg Chemnitz 21 - 2 5 5 2 - 4 - - - 3 -
13. Ellen Richter Chemnitz 20 - - - 6 1 - 4 3 - - 3 3
13. Anja Posselt Chemnitz 20 - - - - 2 6 - - 6 - 3 3
13. Ria Hopke Chemnitz 20 - 2 - - - 6 - - 6 - 3 3
14. Lisa Grassmann Chemnitz 19 5 - - - - - - - 6 - 3 5
15. Ingmar Richter Chemnitz 18 6 - - - - - - - 6 - 3 3
16. XXX ??? 17 - 3 - 6 2 - - - 6 - - -
16. Marie Sophie Roß Chemnitz 17 - 2 5 - - - 4 - - - 3 3
17. Sabine Fischbach Hessen 15 4 3 - - 2 - - - 6 - - -
18. Karolin Schuricht Chemnitz 14 6 - - - - 4 - - - - - -
18. Arne Weißbach Chemnitz 14 - 2 - - - - - 6 6 - - -
18. Felix Taubert Chemnitz 14 - 3 - - 2 - - - 6 - 3 -
19. Astrid Fischer Chemnitz 13 5 3 5 - - - - - - - - -
20. Rebecca Wagner Oberwiesenthal 12 6 3 - - - - - - - 3 - -
21. Luis Raupach Chemnitz 11 - - - - 2 3 - - - - 3 -
21. Nina Zätsch Chemnitz 11 - 2 - - - - - 3 - - 3 3
22. Marion Sarah Zenk Chemnitz 10 - - - - - - - 3 - - 3 4
22. Josephine Pallus Chemnitz 10 4 - - - - - - - - - 3 3
23. Felix Brinkel Chemnitz 9 6 - - - - - - - - - 3 -
23. Ellen Wilde Chemnitz 9 - - - - 2 2 - - - - - 5
24. Jonathan Kässler Chemnitz 8 - 2 - - - - - - - - 3 3
24. Hannah-Sophie Schubert Chemnitz 8 - 2 - - - - - - 6 - - -
25. Melina Seerig Chemnitz 6 - - - - - - - - 6 - - -
25. Lucas Steinke Chemnitz 6 - - - - - - - - - - 3 3
25. Wim Winter Chemnitz 6 - 1 - - - - 2 - - 3 - -
25. Christian Wagner Bamberg 6 6 - - - - - - - - - - -
25. Robin König Chemnitz 6 - - - - - - - - - - 3 3
25. Andreas M. Dittersdorf 6 - - - - - - - - 6 - - -
25. Johanna Ranft Chemnitz 6 - - - - - - - - - - 3 3
25. Duncan Mahlendorff Chemnitz 6 - - - - - - - - - - 3 3
26. Hannah Gebhardt Chemnitz 5 - - - - 2 - - - - - 3 -
26. Ole Koelb Chemnitz 5 - - - - 2 - - - - - 3 -
26. Theresa Jänich Chemnitz 5 - - - - 2 - - - - - 3 -
26. Helene Fischer Chemnitz 5 - 3 - - 2 - - - - - - -
26. Kai-Lutz Wagner Chemnitz 5 - - - - 2 - - 3 - - - -
26. Laura Schlosser Chemnitz 5 - 3 2 - - - - - - - - -
26. Marcel Reichelt Chemnitz 5 - - - - 1 1 - - - - 3 -
27. Emilie Grossinger Chemnitz 4 - - - - 1 - - - - - 3 -
27. Henrike Grundmann Chemnitz 4 - - - - 1 - - - - - 3 -
27. Hannes Eltner ???? 4 4 - - - - - - - - - - -
28. Lukas Kirchberg Chemnitz 3 - - - - - - - - - - 3 -
28. Julia Ritter Chemnitz 3 - - - - - - - - - - 3 -
28. Tim Jechorek Chemnitz 3 - - - - 2 - - - - - 1 -
28. Willy Stöckel Chemnitz 3 - - - - - - - - - - 3 -
28. Amarin Roßberg Chemnitz 3 - - - - - - - - - - 3 -
28. Carl Geißler Chemnitz 3 - - - - 1 2 - - - - - -
28. Daniel Hufenbach Leipzig 3 - - - - - - - - - - 3 -
28. Julia Voigt Chemnitz 3 - - - - - - - - - - 3 -
29. Agnieszka Urban Chemnitz 2 - - - - - - - - - - 2 -
29. Niels Steinert Chemnitz 2 - 2 - - - - - - - - - -
29. Jule Schwalbe Chemnitz 2 - 2 - - - - - - - - - -
29. Matthias Engewald Erfurt 2 - - - - 2 - - - - - - -
29. Pauline Marschk Chemnitz 2 - - - - 2 - - - - - - -
29. Paula Hartmannsdorf 2 - - - - 2 - - - - - - -
29. Andree Dammann München 2 - - - - 2 - - - - - - -
29. Lukas Thieme Chemnitz 2 - - - - 2 - - - - - - -
30. Gwendolin Eichler Chemnitz 1 - - - - - 1 - - - - - -
30. Hannes Langenstraß Chemnitz 1 - 1 - - - - - - - - - -
30. Mara Neudert Chemnitz 1 - - - - 1 - - - - - - -
30. Jonas Frederik Otto Lichtenwalde 1 - - - - 1 - - - - - - -
30. Vincent Baessler Chemnitz 1 - - - - - 1 - - - - - -
30. Emma Irmscher Eibenberg 1 - 1 - - - - - - - - - -

 

Auswertung Serie 26 (rote Liste)

Platz Name Ort Summe Aufgabe
301 302 303 304 305 306 307 308 309 310 311 312
1. Uwe Parsche Chemnitz 75 6 5 12 5 - 8 4 12 10 4 4 5
2. Doreen Naumann Duisburg 58 6 5 - - 4 8 4 12 6 4 4 5
3. Jamila Wähner Chemnitz 29 - 5 - - 4 8 - 12 - - - -
4. Rafael Seidel Chemnitz 24 - - 12 - - - 4 - - 4 4 -
5. Astrid Fischer Chemnitz 21 6 5 10 - - - - - - - - -
6. XXX ??? 20 - 5 - 5 5 - - - 5 - - -
7. Felix Haase Chemnitz 18 6 5 - 2 - - - - - - - 5
8. Linus-Valentin Lohs Chemnitz 16 - - - - 4 - - 4 6 - - 2
9. Richard Hahmann Chemnitz 14 - - - - - - - 4 5 - 1 4
10. Elisa Parsche Chemnitz 12 - 5 - - - - 3 - - - 4 -
10. Sabine Fischbach Hessen 12 - 5 - - 4 - - - 3 - - -
11. Hannah-Sophie Schubert Chemnitz 10 - 5 - - - - - 2 3 - - -
12. Anja Posselt Chemnitz 8 - - - - - 8 - - - - - -
12. Ria Hopke Chemnitz 8 - - - - - 8 - - - - - -
13. Marion Sarah Zenk Chemnitz 7 - - - - - - - 4 - - - 3
13. Ingmar Richter Chemnitz 7 - - - - - - - - 5 - - 2
14. Marie Sophie Roß Chemnitz 6 - - - - - - - 4 - - - 2
14. Christian Wagner Bamberg 6 6 - - - - - - - - - - -
15. Lisa Grassmann Chemnitz 5 - - - - - - - - - - - 5
15. Matthias Engewald Erfurt 5 - - - - 5 - - - - - - -
15. Niels Steinert Chemnitz 5 - 5 - - - - - - - - - -
16. Hannes Langenstraß Chemnitz 4 - 4 - - - - - - - - - -
16. Hannah Gebhardt Chemnitz 4 - - - - 4 - - - - - - -
16. Daniel Hufenbach Leipzig 4 - - - - - - - - - - 4 -
16. Felix Taubert Chemnitz 4 - - - - 4 - - - - - - -
16. Andree Dammann München 4 - - - - 4 - - - - - - -
17. Melina Seerig Chemnitz 3 - - - - - - - - 3 - - -
17. Rebecca Wagner Oberwiesenthal 3 - - - - - - - - - 3 - -
17. Hannes Eltner ???? 3 3 - - - - - - - - - - -
17. Jonathan Kässler Chemnitz 3 - - - - - - - - - - - 3
17. Andreas M. Dittersdorf 3 - - - - - - - - 3 - - -
17. Jule Schwalbe Chemnitz 3 - 3 - - - - - - - - - -
17. Ellen Richter Chemnitz 3 - - - - - - - - - - - 3
17. Arne Weißbach Chemnitz 3 - 3 - - - - - - - - - -
17. Paula Hartmannsdorf 3 - - - - 3 - - - - - - -
18. Stephanie Dani Chemnitz 2 - - - - - - - 2 - - - -
18. Nina Zätsch Chemnitz 2 - - - - - - - - - - - 2
18. Johanna Ranft Chemnitz 2 - - - - - - - - - - - 2
18. Loise Reichmann Chemnitz 2 - - - - - - - 2 - - - -
19. Henrike Grundmann Chemnitz 1 - - - - - - - - - - 1 -
19. Ole Koelb Chemnitz 1 - - - - - - - - - - 1 -
19. Robin König Chemnitz 1 - - - - - - - - - - - 1
19. Marcel Reichelt Chemnitz 1 - - - - 1 - - - - - - -


Serie-25


Serie 25
Aufgabe und Lösungen

Aufgabe 1

289. Wertungsaufgabe

„Wie war eigentlich eure Radtour, die ihr vom Donnerstag bis zum Sonntag unternommen habt?“, fragte Bernd. „Wir – Lisa und ich – haben  wegen des nicht so tollen Wetters nur 4 Tagestouren von Chemnitz aus unternommen, nach Limbach, Hohnstein, Freiberg und Thum.“, sagte Mike. „Na, es war eine Pizzatour“, ergänzte Lisa. „Wie das?“ „Wir haben uns jeden Tag in den Orten in eine Pizzeria gesetzt, bevor wir wieder zurückgefahren sind.“ „Aber jeder hatte jeden Tag eine andere“, knurrte Mike.
Es gab Margarita, Diavolo, Cipola, Funghi, Bolognese, Calzone, Salamie und al Tonno.
1. Am Freitag aß Mike eine Salamiepizza, aber das war nicht in Thum.
2. Zwei Tage nach der Funghi-Pizza, die nicht in Hohnstein serviert wurde, aß Lisa die Cipola, während Mike sich für die al Tonno entschieden hatte.
3. Am Samstag, gönnte sich Lisa eine Diavola, das war weder in Thum noch in Freiberg.
4. In Limbach hatte Mike Appetit auf seine Lieblingspizza – Pizza Bolognese.

"Mike, da ich dich schon lange kenne, bin ich mir sicher, dass du auch noch die Calzone auf deinem Teller hattest", sagte Bernd. Mike grinste bestätigend.
Wer hat wann, wo was gegessen? 6 blaue Punkte
Zwischendurch kamen Sie an einer neugebauten Siedlung vorbei. Es handelte sich um sieben Häuser (Haus 1 ganz links , ..., Haus 7 ganz rechts) und einen großen Spielplatz. In der Pizzaria kamen sie zufälligerweise mit dem dem Makler ins Gespräch, der vor lauter Stolz ins Reden kam, hatte er doch alle Häuser verkauft und so zugleich für den Bestand der Schule im Ort gesorgt, da in jedes Haus nicht nur jeweils ein Ehepaar, sondern auch deren Kinder mit eingezogen waren. Als er hörte, dass Lisa und Mike zwei von denen waren, die immer bei der Aufgabe der Woche dabei sind, gab er seine Informationen in Form eines Rätsels preis.
1. Zelia wohnt im Haus 6.
2. Solveig wohnt zwei Häuser weiter links als Peter.
3. Jule ist nicht mit Benno, aber auch nicht mit Mirko verheiratet.
4. Olaf wohnt mit seiner Familie in Haus 2.
5. In Haus 4 – es wohnt nicht Jonas dort, sind genau zwei Jungen zu Hause, aber kein Mädchen.
6. Peter hat 3 Söhne.
7. Linda wohnt nicht ganz links und hat eine Tochter und einen Sohn.
8. Das Haus von Linda steht drei Häuser weiter links als das von Ria, die wiederum direkt links neben der Familie mit den zwei Töchtern und dem einem Sohn leben.
9. Elias hat zwei Töchter, aber keinen Sohn und er wohnt – genau wie Benno- nicht in Haus 7. Benno wohnt allerdings drei Häuser weiter rechts als Mirandas Familie.
10. Drei Häuser weiter links als Jonas wohnt die Familie, die eine Tochter und zwei Söhne hat.  

Alles klar, oder? Na gut,hier noch mal die Namen, und die Kinderzahlen:
Anja, Jule, Linda, Miranda, Ria, Solveig und Zelia.
Benno, Elias, Jonas, Mirko, Olaf, Peter und Uwe.
Die Kinderkonstellation ist immer anders: drei Jungen, zwei Jungen, ein Mädchen und zwei Jungen,  zwei Mädchen und ein Junge, ein Mädchen und ein Junge, zwei Mädchen und dann noch ein  einzelnes Mädchen.
Wer wohnt mit wem in welchem Haus? 10 rote Punkte

Lösung:
bei blau gab es drei Lösungen, sollte nicht sein, aber passiert Traurig
Hier die Notierungen von Elisa, danke.
1. Variante:
Am Donnerstag in Limbach gab's Bolognese (Mike) und Margarita (Lisa).
Am Freitag in Freiberg gab's Salami (Mike) und Funghi (Lisa).
Am Sonnabend in Hohenstein gab's Calzone (Mike) und Diavolo (Lisa).
Am Sonntag in Thum gab's al Tonno (Mike) und Cipola (Lisa).
2. Variante:
Am Donnerstag in Hohenstein gab's Calzone (Mike) und Margarita (Lisa).
Am Freitag in Freiberg gab's Salami (Mike) und Funghi (Lisa).
Am Sonnabend in Limbach gab's Bolognese (Mike) und Diavolo (Lisa).
Am Sonntag in Thum gab's al Tonno (Mike) und Cipola (Lisa).
3. Variante:
Am Donnerstag in Thum gab's Calzone (Mike) und Margarita (Lisa).
Am Freitag in Freiberg gab's Salami (Mike) und Funghi (Lisa).
Am Sonnabend in Limbach gab's Bolognese (Mike) und Diavolo (Lisa).
Am Sonntag in Hohenstein gab's al Tonno (Mike) und Cipola (Lisa).

rot:
eines sehr schöne Variante der Herltung und des Aufschreibens von mawi, danke.
Ich führe Variablen für die Namen ein:
Anja = a
Jule = b
Linda = c
Miranda = d
Ria = e
Solveig = f
Zelia = g

Benno = u
Elias = v
Jonas = w
Mirko = x
Olaf = y
Peter = z
Uwe = t

3 Jungs = k1
2 Jungs = k2
2 Jungen, 1 Mädchen = k3
1 Junge, 2 Mädchen = k4
1 Junge, 1 Mädchen = k5
2 Mädchen = k6
1 Mädchen = k7

In einem Haus i (i = 1..7) wohnt eine Mutter (a..g), ein Vater (t..z)
und Kinder (k1..k7). Die eingeführten Variablen haben den Wert der
Hausnummer. Ganz links = 1, ganz rechts = 7. Damit gibt es also immer
eine Konstellation Variable der Mutter = Variable des Vaters = Variable
der Kinder = {1..7}.

Man kann dann die getroffenen Aussagen in Form dieser Un-/Gleichungen
schreiben:
1) g=6
2) f+2=z
3.1) b != u
3.2) b != x
4) y=2
5.1) w != 4
5.2) k2 = 4
6) z=k1
7.1) c>1
7.2) c= k5
8.1) c+3 = e
8.2) e+1 = k4
9.1) v=k6
9.2) v != 7
9.3) u != 7
9.4) u = d+3
10) w = 3+k3

Damit gilt un-/mittelbar:
11) y=2 aus 4)
12) g=6 aus 1)
13) k2 = 4 aus 5.2)
14) e = 5, weil: e != {1,2,3} aus 8.1); e!=4 aus 8.1) und 7.1); e!=6 aus
12); e!=7 aus 8.2)
15) k4 = 6 aus 14) und 8.2)
16) c = 2 aus 8.1) und 14)
17) k5 = 2 aus 16) und 7.2)
18) k3 = 3 aus [k3 != 1 aus 5.1) und 10)] und [k3!=2 aus 17)] und [k3!=4
aus 12)] und [k3!={5,6,7} aus 10)]
19) w = 6 aus 19) und 10)
20) k1 = 5 aus [k1 != {1,2} aus 2) und 6)] und [k1 != 3 aus 18)] und [k1
!= 4 aus 13)] und [k1 != 6 aus 15)] und [k1 != 7 aus 2) und 6) und 14)]
21) k7 = 7 aus [k1 != 7 aus 20)] und [k2 != 7 aus 13)] und [k3 != 7 aus
18)] und k4 != 7 aus 15)] und [k5 != 7 aus 17)] und [k6 != 7 aus 9.1)
und 9.2)]
22) k6 = 1 (alle anderen ki sind schon belegt)
23) v = 1 aus 9.1)
24) z = 5 aus 20) und 6)
25) u = 4 aus [u != 1 aus 23)] und [u != 2 aus 11)] und [u != 3 aus
9.4)] und [u != 5 aus 24)] und [u != 6 aus 19)] und [u != 7 aus 9.3)]
26) d = 1 aus 9.4) und 25)
27) f = 3 aus 24) und 2)
28) a = 4, da nur a =4 oder b = 4 übrig ist und mit 25) und 3.1) b != 4
sein muß
29) b = 7, da alle anderen Mütter bereits festgelegt sind
30) t  = 7, da nur t und x als Väter noch nicht vergeben sind und 29)
mit 3.2)
31) x = 3, da dies der einzige übrige Vater ist

Damit heißt dies:
Im Haus 1 wohnen Miranda mit Elias und 2 Mädchen.
Im Haus 2 wohnen Linda und Olaf mit 1 Junge und 1 Mädchen.
Im Haus 3 wohnen Solveig und Mirko mit 2 Jungen und 1 Mädchen.
Im Haus 4 wohnen Anja und Benno mit 2 Jungen.
Im Haus 5 wohnen Ria und Peter mit 3 Jungen.
Im Haus 6 wohnen Zelia und Jonas mit 1 Junge und 2 Mädchen.
Im Haus 7 wohnen Jule und Uwe mit 1 Mädchen.

Nun noch die Probe...
1) ok
2) ok
3) ok
4) ok
5) ok
6) ok
7) ok
8) ok
9) ok
10) ok

Der Lösungsweg ist eindeutig, die Probe bestätigt die Lösung, also ist  die gefundene Lösung tatsächlich richtig und die einzige Lösung.



Aufgabe 2


290.  Wertungsaufgabe

„Ihr habt aber viele Murmeln gesammelt“, sagte Bernd zu Maria. „Stimmt und die sehen auch noch richtig schön aus.“ „Wie viele sind das denn?“ „Pass auf.“ Du Bernd könntest dir eine wegnehmen und dann ein Viertel der Verbliebenen. Danach kommt Mike, nimmt eine Murmel weg und anschließend ein Viertel. So verfahren auch Lisa und ich. Jetzt sind immer noch welche da, von denen wir eine beiseite legen und dann können wir vier uns den Rest gerecht teilen.“ „Insgesamt hat Bernd dann aber mehr als du.“ „Das stimmt, aber ihr könnt ausrechnen, wie viele Murmeln (mindestens) am Anfang da waren.“ - 6 blaue Punkte. Wie sieht die Mindestzahl auf dem Kugelhaufen aus, wenn es n Leute sind (n>1), die jeweils eine Kugel weglegen und vom verbleibenden Rest 1/n wegnehmen. Nachdem sich jeder so bedient hat, lässt sich, nach dem nochmaligen Weglegen einer Kugel der verbliebene Haufen gleichmäßig unter allen aufteilen. 6 rote Punkte

Lösung:
blau: die Aufgabe durch Probieren herauszubekommen war nicht gerade einfach, aber versucht wurde es doch. Da es zwischenzeitlich einen Newsletter gab, dass die kleinste Zahl etwas über 1000 liegt, gab es dann noch ein paar Lösungszuschriften mehr. Viele habe ide Aufgabe von hinten her aufgezäumt und dann systematisch probiert, zum Teil mittels Tabellenkalkulation. Das ist natürlich zulässig.
Hier die Überlegung: Bekommt am Ende jeder der 4 noch n Kugeln, dann waren es vor dem Verteilen und Weglegen 4n + 1 Kugeln. Das hatte Maria übrig gelassen, nach dem sie sich 1/4 genommen hatte, Es waren 3/4 des Haufens, also hatte Maria 4/3(4n +1) plus die eine Kugel vorgefunden: 4/3(4n+1)+1 war also die Situation von Lisa, die demzufolge vor dem Weglegen der einen Kugel 4/3(4/3(4n+1)+1)+1 vorgefunden hatte. Entsprechend für Mike 4/3(4/3(4/3(4n+1)+1)+1) +1 und damit zu Beginn für Bernd 4/3(4/3(4/3(4/3(4N+1)+1)+1) +1)+1. Durch den Faktor 4/3 ist die "Chance", dass da am Ende eine natürliche Zahl herauskommt oder auch zwischendurch nicht so groß. Ja beim systematischen Probieren findet man erst mit N = 80 die erste Lösung und somit als Kugelzahl am Anfang 1021. Schaut man sich den Probierausdruck an, so ist es ja so dass die 4 bei 4/3 von der Anzahl der 4 Leute (n)  kommt, die 3 um eins kleiner ist als die 4 und insgesamt 5 mal (also einmal mehr wie Leute ) geteilt wird. Mit etwas mehr oder weniger Spielerei erhält man nun die 1021 auch mit 45 - 3. Jede Anzahl von Kugeln mit m*45 - 3 (m - natürliche Zahl >0) liefert eine Lösung für die Aufgabenstellung:
rot: Ist die Anzahl der Leute n, wird vor jeder Teilung eine Kugel weggenommen und bei der Teilung wird 1/n genommen bzw. aufgeteilt, dann braucht man:
m(n(n+1) - (n-1) Kugeln. Die sehr ausführliche Herleitung inklusive des Beweises mittels vollständiger Induktion von mawi, wird zu einem späteren Zeitpunkt noch ergänzt, danke an mawi schon mal vorab. Mit m = 1 erhält man immer die Minimalzahl, wenn man für n= 2 davon ausgeht, dass es bei der letzten Teilung auch mal Null Kugeln sein dürfen, ansonsten nimmt man dort m = 2.



Aufgabe 3


291.  Wertungsaufgabe

„Wir haben uns in unserer Gruppe über rechtwinklige Dreiecke unterhalten. Der Linus aus der 5. Klasse kannte sogar schon den Satz des Pythagoras und wusste, dass der Flächeninhalt dieser Dreiecksart sich auch einfach ausrechnen lässt. (Sind a und b die kürzeren Seiten und c die längste, so gilt a² + b² = c² und A = a*b/2)
Mit etwas Mühe haben wir zwei verschiedene solche Dreiecke gefunden, deren Seitenlängen ganzzahlig waren und das bei gleichem Flächeninhalt.“ 8 blaue Punkte – verschieden heißt, die Dreiecke sind natürlich nicht kongruent zueinander.
Richtig hammerhart ist die rote Aufgabe. Es sind vier verschiedene rechtwinklige Dreiecke zu finden, deren Seitenlängen ganzzahlig sind und deren Flächeninhalt gleich ist. 12 rote Punkte. Anmerkung zu rot, der kleinstmögliche Flächeninhalt hat mehr als 100 000 Einheiten.
Lösung:
Im Newsletter zu dieser Aufgabe wurde ein Hinweis auf die Serie 13 Aufgabe 5 gegeben. Dort ist ein Verfahren zur Erzeugung pythagoräischer Zahlentripel bewiesen worden. Man nehme zwei verschiedene natürliche Zahlen (>0) m und n mit m > 0. Mit a = m² - n², b = 2mn und c = m² + n² erhält man dann pythagoräisches Tripel.
Nun braucht man nur noch die Flächeninhalte zu vergleichen und sieht ob die Flächeninhalte gleich sind (oder eben nicht).
Viele haben (zum Teil sehr lange) systematisch probiert und so ein oder mehrer Lösungen gefunden:
Blau: a = 20, b = 21 und c = 29 liefern den Flächeninhalt 210, aber auch (12; 35; 37) führt auf den Flächeninhalt 210. Darauf lassen sich durch ganzzahliges Vervielfachen weitere Lösungen finden.
Lösungen für rot und andere Anzahlen sind der Tabelle entnehmbar. (Danke Elisa und ihrem Papa.)









kleinste Flächeninhalte


a b c A
n x y k k*|x²-y²| k*2*x*y k*(x²+y²) k²*x*y*|x-y|*(x+y)
1 1 2 1 3 4 5 6
2 3 2 1 20
21 29 210

1 6 1 35 12 37 210
3 3 7 1 40 42 58 840

5 7 1 24 70 74 840

7 8 1 15 112 113 840
4 7 37 1 1320 518 1418 341880

33 37 1 280 2442 2458 341880

37 40 1 231 2960 2969 341880

55 56 1 111 6160 6161 341880
5 11 35 13 14352 10010 17498 71831760

23 33 13 7280 19734 21034 71831760

15 169 1 28336 5070 28786 71831760

161 169 1 2640 54418 54482 71831760

169 176 1 2415 59488 59537 71831760
6 189 64 13 411125 314496 517621 64648584000

11 35 390 430560 300300 524940 64648584000

23 33 390 218400 592020 631020 64648584000

15 169 30 850080 152100 863580 64648584000

161 169 30 79200 1632540 1634460 64648584000

7 345 15 1784640 72450 1786110 64648584000
7 65 133 138 1858032 2386020 3024132 2216650756320

77 247 46 2533680 1749748 3079148 2216650756320

117 187 69 1468320 3019302 3357402 2216650756320

45 221 69 3230304 1372410 3509754 2216650756320

133 153 138 789360 5616324 5671524 2216650756320

1045 1071 6 330096 13430340 13434396 2216650756320

2907 2912 3 87285 50791104 50791179 2216650756320
weitere Varianten






1 3 2 1 5 12 13 30
2 3 8 1 55 48 73 1320

1 11 1 120 22 122 1320
3 7 13 1 120 182 218 10920

13 8 1 105 208 233 10920

13 15 1 56 390 394 10920
4 13 55 2 5712 2860 6388 8168160

11 91 1 8160 2002 8402 8168160

85 91 1 1056 15470 15506 8168160

91 96 1 935 17472 17497 8168160
5 13 55 6 17136 8580 19164 73513440

11 91 3 24480 6006 25206 73513440

11 119 2 28080 5236 28564 73513440

85 91 3 3168 46410 46518 73513440

91 96 3 2805 52416 52491 73513440
6 7 37 4830 6375600 2501940 6848940 7975684332000

375 32 69 9632469 1656000 9773781 7975684332000

33 37 4830 1352400 11794860 11872140 7975684332000

3 77 2415 14296800 1115730 14340270 7975684332000

11 529 70 19580400 814660 19597340 7975684332000

1 111 2415 29752800 536130 29757630 7975684332000

Der "Erfinder" dieses Aufgabe - Henry Ernest Dudeney - hatte allerdings keinen Computer.
Hier die Angabe der Lösung aus: The Canterbury Puzzles nr. 107

The Four Princes.

When Montucla, in his edition of Ozanam's Recreations in Mathematics, declared that "No more than three right-angled triangles, equal to each other, can be found in whole numbers, but we may find as many as we choose in fractions," he curiously overlooked the obvious fact that if you give all your sides a common denominator and then cancel that denominator you have the required answer in integers!

Every reader should know that if we take any two numbers, m and n, then m2 + n2, m2 - n2, and 2mn will be the three sides of a rational right-angled triangle. Here m and n are called generating numbers. To form three such triangles of equal area, we use the following simple formula, where m is the greater number:—

mn + m2 + n2 = a
m2 - n2 = b
2mn + n2 = c

Now, if we form three triangles from the following pairs of generators, a and b, a and c, a and b + c, they will all be of equal area. This is the little problem respecting which Lewis Carroll says in his diary (see his Life and Letters by Collingwood, p. 343), "Sat up last night till 4 a.m., over a tempting problem, sent me from New York, 'to find three equal rational-sided right-angled triangles.' I found two ... but could not find three!"

The following is a subtle formula by means of which we may always find a R.A.T. equal in area to any given R.A.T. Let z = hypotenuse, b = base, h = height, a = area of the given triangle; then[Pg 247] all we have to do is to form a R.A.T. from the generators z2 and 4a, and give each side the denominator 2z (b2 - h2), and we get the required answer in fractions. If we multiply all three sides of the original triangle by the denominator, we shall get at once a solution in whole numbers.

The answer to our puzzle in smallest possible numbers is as follows:—

First Prince 518 1320 1418
Second Prince 280 2442 2458
Third Prince 231 2960 2969
Fourth Prince 111 6160 6161

The area in every case is 341,880 square furlongs. I must here refrain from showing fully how I get these figures. I will explain, however, that the first three triangles are obtained, in the manner shown, from the numbers 3 and 4, which give the generators 37, 7; 37, 33; 37, 40. These three pairs of numbers solve the indeterminate equation, a3b - b3a = 341,880. If we can find another pair of values, the thing is done. These values are 56, 55, which generators give the last triangle. The next best answer that I have found is derived from 5 and 6, which give the generators 91, 11; 91, 85; 91, 96. The fourth pair of values is 63, 42.

The reader will understand from what I have written above that there is no limit to the number of rational-sided R.A.T.'s of equal area that may be found in whole numbers.





Aufgabe 4

292. Wertungsaufgabe

"Hallo Mike, bei dir sieht es ja aus. Der Computer ist an, aber den siehst du ja kaum, wegen des großen Büchstapels", sagte Bernd vorwurfsvoll schmunzelnd - bei ihm war es ja kaum anders - als er Mike das erste Mal nach dessen Urlaub sah. "Na schau doch mal, auf der neuen Homepage des Chemnitzer Schulmodells habe ich einen Briefmarkenzusammendruck aus Ungarn entdeckt - anlicken -. Der gefällt mir sehr, aber erstaunlicherweise hat der Gestalter der Marken bei der Nummerierung einen Fehler gemacht." "Stimmt", meinte Bernd, nach dem er sich die Marken genau betrachtet hatte. (Wo liegt der Fehler? - 2 blaue Punkte). "Den Stapel mit den Krimis räume ich gleich weg, aber das Zitat will ich dir mal schnell noch vorlesen:" ... Gleichungen, also Gleichungen, für die es ??? Lösung gibt, heißen absurd. Ein Beispiel dafür ist (a + b)(a -b)  = a² – b² +1" "Das steht in einem Krimi?" "Aber klar doch, schau her". Aufgabe rot: Was steht im Zitat statt ??? und wie heißt der Krimi (2 + 1) Versiegelt

Lösung:

blau: Die Briefmarken sind durchnummeriert. 1; 2; ...; 30. Allerdings sind diese Zahlen in der Primfaktorenzerlegung notiert. 1 (ist keine Primzahl aber) 2; 3; 2*2, 5; 2*3; 7; 2³, ...17; 2*9; 2²*5; 3*7; ... Der Fehler ist also bei der 18, die mit 2*9 angegeben ist, statt richtigerweise 2*3². Übrigens einige Paare der Marken bilden Stereogramme, so die 29 und 30 - pardon 2*3*5.
rot: "Unwahre Gleichungen, also Gleichungen, für die es keine Lösung gibt, heißen absurd." Zitat aus Stieg Larsson - Verdammnis.





Aufgabe 5



293. Wertungsaufgabe

"Na, den komischen Fehler auf den Marken zu finden, war ja nicht so schwer, wenn man einmal erkannt hat, dass es sich um die Primzahlzerlegung handelte", meinte Lisa. "Das ist schon richtig, aber trotzdem muss man darauf kommen. Ich habe auch noch ein kleines Primzahlrätsel." "Lass hören."
"Gesucht sind Paare von Primzahlen, deren Differenz eine zweistellige Primzahl ist. Zu finden sind alle Paare von Primzahlen, wobei die Differenz kleiner als 30 sein soll?" 4 blaue Punkte
"Aber pass mal auf, ob du dieses verblüffende Primzahlrätsel auch lösen kannst. Es sind alle Zahlen zu untersuchen, deren Ziffern alle verschieden sind -- wobei die Null nicht vorkommen soll." "Da gibt es doch unheimlich viele." "Man kann diese aber systematisch finden, mittels Permutation, also so zum Beispiel die 137, dann gibt es noch die 173, 317, 371, 713 und 731." "Ja, ich erinnere mich, aber wie lautet denn nun die Aufgabe?" "Es ist die größte Zahl zu finden, bei der alle Permutationen  eine Primzahl darstellen. Bei meinem Beispiel sind drei der Zahlen Primzahlen, drei aber eben auch nicht. Das Ergebnis wird dich überraschen." 8 rote Punkte

Lösung:
Bei der blauen Aufgabe haben sich einige etwas schwer getan oder auch was vergessen, weil der Text nicht genau gelesen wurde, aber okay.
Wenn die Differenz zweier Primzahlen eine zweistellige Primzahl sein soll, muss eine der Primzahlen die 2 sein. Bei allen anderen Primzahlen ergibt sich ja eine gerade Zahl als Differnz und somit keine Primzahl. Zweistellige Primzahlzahldifferenzen kleiner als 30 könnten sein 11, 13, 17, 19, 23 und 29. Das ergäbe die Paare:
(2; 13), (2; 15), (2; 19), (2;25) und (2;31). Davon sind allerdings nur diese Primzahlpaare: (2; 13), (2; 19) und (2;31).
Rot. Hier möchte ich den von XXX geschriebenen Dialog als Lösung anbieten, danke sehr.:

„Also Mike“, sagte Bernd, „das mit den großen Zahlen, die Primzahlen sind oder nicht, und dann noch permutiert, da sollten wir uns einen schnellen Rechner suchen!“ Und Mike rechnet mit: „Die größte Zahl ist 987654321 und deren Ziffern haben 9! (Fakultät) Permutationen der Reihenfolge. Ich mag so große Zahlen nicht teilen!“

„Okay!“ grinst Bernd, „bei der Größten helfe ich dir, die hat Quersumme 45 und ist damit eine Neunerzahl! Schon 9! Probleme gelöst!“

Mike: „Nun mal systematisch: Welche Teilbarkeitsregeln kennen wir? Da sind die Teiler der Stufenzahl, der 10. Zweier- und Fünferzahlen erkennen wir an der letzten Stelle.“ „Ja“, meint Bernd, „und durchs Permutieren kommt jede Ziffer mal nach hinten, wir können also alle Zahlen weglassen mit einer geraden Ziffer oder mit einer 5.“ Da ist Mike aber erleichtert: „9731 statt 987654321 als größte Zahl, da sind wir gut voran gekommen.“ „Aber mit der Quersumme geht nun nur noch die 93 und die 39 zu knacken. Oder gibt es noch weitere Teilbarkeitsregeln? Opa kennt da sicher ein paar alte Tricks.“

„Eine Quersummenregel gibt es für die Teiler der Zahlen neben der Stufenzahl, und zwar die normale Quersumme für die Zahl davor, bei unserem Zehnersystem also für die 9.“ „Im Sechzehnersystem für die 5, weil sie die 15 teilt?“ fragt Bernd erstaunt. „Ja! Und für die Zahl nach der Stufenzahl, also unsere 11 gibt es die alternierende Quersummenregel, also einmal zuzählen, einmal abziehen.“ „Warum denn das?“ fragt Mike. „11 ist 1 mehr als 10, 99 ist 1 weniger als 100, 1001 wieder 1 mehr, usw …“ „ Nicht so allgemein! Bei den vierstelligen Zahlen zähle ich die 1. und 3. Ziffer zusammen und ebenfalls die 2. und 4. Wenn die Summen gleich sind [oder sich um 11 unterscheiden] ist es eine Elferzahl?“ „Ja! 9731 haben wir noch:
9 + 1 = 7 + 3 = 10. Dann sind 9713, 9317, 7931, 7139 alles Elferzahlen?“ „Claro – und rückwärts gelesen auch.“

„Mensch!“ ruft Mike, „mit der einen Beobachtung sind nun alle vierstelligen Zahlen auch noch weg, dann ist die Größte ja nur dreistellig!“ „Wie heißt denn die vereinfachte Elferregel für dreistellige Zahlen?“ fragt Bernd. „Wir haben das so gelernt“, sagt Opa: „Elferzahlen, da ist die mittlere Ziffer Summe der äußeren wie bei 253 = 11*23 oder die beiden äußeren sind um 11 größer als die mittlere wie bei 407 = 11*37.“ „Da finde ich nur noch die 913 mit 9+3 = 1+11.“

„Toll wie das mit den Teilbarkeitsregeln ging. Die fehlenden dreistelligen Zahlen 973, 971 und 731 schaffen wir jetzt auch ohne Regeln.“ „Die kleinste Primzahl, die wir noch nicht getestet haben, ist die 7. Da kann man auch Regeln basteln, aber für unsere kleinen Zahlen bringt das nichts mehr!“ Und Opa hat noch einen letzten Tipp: „Mit euren Teilbarkeitsregeln für 2, 3 und 5 könnt ihr bis 100 jede Zahl zerlegen außer 49 = 7*7, 77 = 7*11 und 91 = 7*13. Jetzt reicht euer Handwerkzeug aber …“

Mike zeigt gleich, dass er gut zugehört hat:

„Wenn 91 eine Siebenerzahl ist, dann auch 910 + 7 = 917. Wieder eine weg!“ Bernd legt sofort nach: „Nimm statt 910 die 903 und finde 903+70 = 973.“

„Die letzte Dreistellige, die 731 erwähnt Thomas ja schon, also 713 = 690 + 23 = 31*23 noch der Vollständigkeit halber aufgeschrieben.“

Die größte Zweistellige von den untersuchen Zahlen tut es nun aber doch noch! 97 und 79 sind Primzahlen!



Aufgabe 6


294. Wertungsaufgabe

16

3

2

13

5

10

11

8

9

6

7

12

4

15

14

1

"Hallo Lisa, wie ich sehe hast du dir das magische Quadrat von Albrecht Dürer in deine 16 Felder eingetragen."
"Stimmt genau. In unserer Gruppe werden wir nächste Woche dieses 4 x 4 Quadrat genau untersuchen. Es ist fantastisch, welche der 2 x 2 Teilquadrate man auch untersucht, es kommt als Summe immer 34 heraus."  Mike schaut noch mal ganz genau hin und meint. "Na bei allen stimmt das aber nicht." "Du hast Recht." Wie viele solche  2 x 2 Quadrate gibt es und warum muss die Summe 34 sein? (2 + 3 blaue Punkte). Für 5 blaue Punkte ist ein "perfektes" 4 x 4 Quadrat zu finden. Das heißt, alle bildbaren Teilquadrate haben die gleiche Summe 34.


Lösung:

blau: Es gibt insgesamt neun 2 x 2 Quadrate. Drei lassen sich finden, wenn man die oberste und zweite Zeile nimmt, dann drei aus zweiter und dritter Zeile. Dazu kommen noch drei in dritter mit vierten Zeile kombiniert. Allerdings sind es "nur" 5 bei denen die geforderte Summe 34 herauskommt. Warum nun 34?

Addiert man die Zahlen von 1 bis 16, so ergiebt sich 136. Soll in allen Zeilen die gleiche Summe herauskommen, so ergibt das für jede Zeile 34. Die Summe von vier Zahlen sollte also 34 sein. Noch mal gezeigt für den Fall alle 2 x 2 Quadrate in der Summe gleich, dann gilt diese natürlich auch, in den die 2 x 2 Quadrate nebeneineinander liegen. Somit umfassen solche zwei 2 x 2-Quadrate genau die 8 Zahlen zweier Zeilen, deren Summe ja 2 x 34 = 68 (Zeilensummen) muss. Somit bleibt für die Quadrate jeweils 34.

rot: Das perfekte Quadrat musste laut Aufgabenstellung nicht notwendigerweise magisch sein und so gab es eine Reihe von Lösungen, bei denen alle Teilquadrate die 34 aufwiesen - ohne magisch zu sein. Das war in Ordnung. Magische Quadrate, die die geforderte Bedingung erfüllen, gibt es auch. Echt verschieden - im Sinne der nächsten Aufgabe - sind genau 48 mag. Quadrate, die die Bedingung erfüllen.

Ein ganz besonders davon ist:

1

12

13

8

14

7

2

11

4

9

16

5

15

6

3

10

Es ist magisch und alle Teilquadrate ergeben 34. Aber schneidet man man dieses Quadrat aus, dann kann man aus diesem Papierstück auf zwei Arten einen Zylinder formen. Und auch hier hat jedes Teilquadrat die Summe 34, egal wie man hinschaut - ist doch cool, oder?


Aufgabe 7


295. Wertungsaufgabe

"Die perfekten Quadrate, man nennt sie auch diabolische  Quadrate, der letzten Woche haben mir sehr gefallen", meinte Bernds Vater. Es gibt davon 48 echt verschiedene. Von den 4x4 Quadraten sind 880 magisch, die echt verschieden sind." "Echt verschieden?", fragte Maria nach. "Nun, das heißt, dass die nicht durch Spiegelung oder Drehung auseinander hervorgehen." "Ach, ich verstehe." "Da stellt sich doch die Frage, wie viele echt verschiedene 4 x 4 Quadrate (gebildet aus den Zahlen 1 bis 16 -- jeweils einmal verwendet) gibt es überhaupt, egal ob die magisch sind oder nicht?" (5 rote Punkte)
Wie viele echt verschiedene 2 x 2 Quadrate gibt es (Zahlen 1 bis 4 jeweils einmal) (Sind zwar nie magisch, aber trotzdem kann man die ja untersuchen.) Punktzahl gleich Anzahl + die selbe Punktzahl dazu, wenn man notiert, warum es nicht mehr sein können.

 

Lösung:

Bei beiden Aufgabenteilen gilt die Anzahl der Anordnungen der Zahlen von 1 bis n liegt bei n! Ebenso gilt, dass je 8 Quadrate durch Spiegeln und Drehen eigentlich gleich sind. Echt verscheiden sind also n!/8.

Das bedeutet für blau es gibt nur drei echt verschiedene Quadrate.

Beispiele von echt verschiedenen Quadraten:

2 1   3 1 4 1
3 4   4 2   2 3

Bei rot sind es 16!/8 = 2615348736000 echt verschiedene und nur 880 davon sind (echt verschieden) magisch.

 


Aufgabe 8


296. Wertungsaufgabe:


"Bei den magischen Quadraten gibt es noch viel zu entdecken", sagte Bernds Opa, als die vier Freunde ihm von der letzten Aufgabe erzählten. "Schaut mal her, ich schreibe die natürlichen Zahlen etwas pyramidenförmig auf."

 

 

 

 

 

1

2

3

 

 

 

 

 

 

 

4

5

6

7

8

 

 

 

 

 

9

10

11

12

13

14

15

 

 

 

16

17

18

19

20

21

22

23

24

 

25

26

27

28

29

30

31

32

33

34

35


"Und so weiter. Wenn ich jetzt an einer Stelle quer durch die Pyramide einen senkrechten Strich ziehe, so ist die Summe der vor dem Strich stehenden Zahlen immer genau so groß wie die Summe der Zahlen hinter dem Strich und das Zeile für Zeile." Wo muss der Strich hin (3 blaue Punkte -Rechnung nicht vergessen-) und warum funktioniert das (6 rote Punkte)?


Lösung:

Die Pyramide wurde um eine Zeile "nach oben" ergänzt. Hinzu kommt noch eine Zeilennummer:

Blau. Der gesuchte Strich verläuft nach der 2 bzw. 6; 12 ...

Als Summen ergeben sich: 1 + 2 = 3

4 + 5 + 6 = 7 + 8 = 15

9 + 10 + 11 + 12 = 13 + 14 + 15 = 42

16 + 17 + 18 + 19 + 20 = 21 + 22 + 23 + 24 = 90 ...

0

 

 

 

 

 

0 |

 

 

 

 

 

 

1

 

 

 

 

1

2 |

3

 

 

 

 

 

2

 

 

 

4

5

6 |

7

8

 

 

 

 

3

 

 

9

10

11

12|

13

14

15

 

 

 

4

 

16

17

18

19

20|

21

22

23

24

 

 

 

 

 

 

 

 

 

 

 

 

 

 

 

n

...

 

 

 

n² + (n - 1)

n² + n |

n² + (n +1)

 

 

 

 

 

 

 

 

 

 

 

 

 

 

 

 

 

 

rot : Die Struktur der Pyramide zeigt folgendes:

Die n-te Zeile beginnt mit n².

Behauptet wird, dass die Summe von n² bis (n² + n) genau so groß ist wie die Summe von (n² + (n+1)) bis (n² + 2n).

Linke Seite: n² + (n² + 1) + ... + (n² + n) ==> n² + n * n² + (1 + 2 + ... + n) ==> n³ + n² + (1 + 2 + ... + n)

 

Rechte Seite (n² + (n+1)) + (n² + (n+2)) + ... + (n² + 2n) ==> n * n² + n * n + (1 + 2 + ... + n) ==> n³ + n² + (1 + 2 + ... + n)

Die Summen links und rechts des Striches sind also gleich.


 

Aufgabe 9

297. Wertungsaufgabe

"Hallo Maria. Pendelst du jetzt die Aufgaben für eure Gruppe aus?" "Nein, natürlich nicht", erwiderte sie. "Ich versuche die Erdrotation nachzuweisen", gab sie Mike zu verstehen. "Und wie soll das gehen? "Stell dir vor, du bist am Nordpol und baust dort ein großes Pendel auf. Wenn du es anschiebst, dann behält es seine Schwingungsebene bei und die Erde dreht sich in 24 Stunden einmal unter dem Pendel durch. Du kannst also eine richtige Uhr in den Schnee zeichnen und das Pendel zeigt die Zeit." "Warum muss ich da bis zum Nordpol? Das geht in Chemnitz doch sicher auch." "Ja und nein. So geht es zum Beispiel am Äquator gar nicht mit so einer "Uhr", weil das Pendel ja so schwingt, wie die Erde sich bewegt. In Chemnitz, welches sich auf 50° 50' nördlicher Breite befindet, schafft das Pendel in 24 Stunden auch nur 279,11°."
Wie lange braucht das Pendel in Chemnitz für einen Vollkreis?(360°) 2 blaue Punkte.
Wie lässt sich für jeden Ort der Erde ausrechnen, wie viel Grad das Pendel an einem Tag schafft? 2 rote Punkte für die Formel + 6 Punkte für eine gute Herleitung der Formel.

Lösung:

Blau: Nun, hier kann man mit einer einfachen Verhältnisgleichung weiterkommen:

x die Zeit für die 360°:

x : 24 h = 360° : 279,11° | * 24 h

x = 24 h * 360°: 279,11°

x = 30,9555372 h, also rund 31 Stunden oder eben genauer: 30 Stunden 57 Minuten und 19,9340762 Sekunden.

rot: Die Formel basiert nicht auf einer solchen Verhältnisgleichung. Um die Zeit für einen Ort müssen die 24 h mit dem Sinus des Breitengrades multipliziert werden.

Mit dem Stichworten Foucaultsches Pendel und Breitengrad ist ein Herleitung schnell zu finden. Bitte mal damit vorlieb nehmen, danke.


Aufgabe 10

298. Wertungsaufgabe

"Du testest wohl neue Spielregeln für Schach?, schmunzelte Mike, als er Lisa mit einer Schere vor einem Schachplan erblickte. "Nein, natürlich nicht. Ich bin den Gedanken bloß mal durchgegangen, wie viele verschiedene Rechteckformen sich aus so einem 8 x 8 "Schachbrett" ausschneiden lassen." "Wie jetzt - Rechteckformen?" Nun, du siehst doch auf dem Spielbrett die Quadrate. Entlang der Kanten dieser Quadrate wird vorsichtig geschnitten, so dass die ausgeschnittenen Teile rechteckig (Quadrate eingeschlossen) sind." Ah ja, jetzt verstehe ich." Wie viele Arten von Rechteckformen lassen sich auf dem Schachbrett finden (sehr wahrscheinlich bräuchte mehr als ein Brett um die alle auszuschneiden) - die Farbe der Felder ist egal. 4 blaue Punkte.
"Wenn man von jeder dieser Formen genau eine nimmt, so kann man aus einem Teil von ihnen ja vielleicht auch wieder ein 8x8-Feld zusammensetzen, natürlich so, dass sich keine Teile überlagern." "Meinst du, dass dies lückenlos geht?" "Ich weiß es noch nicht, ich habe es noch nicht getestet.." Rote Punkte: Für jede verwendete Form gibt es einen Punkt. Für jedes Feld, welches frei bleibt wird ein Punkt abgezogen.

Lösung:

blau:

Rechtecke, die ein "Quadrat breit" sind: 1*1, 1*2, 1*3, 1*4, 1*5, 1*6, 1*7, 1*8.

Rechtecke, die "zwei Quadrate, aber nicht 1 Quadrat breit" sind: 2*2, 2*3, 2*4, 2*5, 2*6, 2*7, 2*8;

Entsprechend: 3*3, ..., 3*8, 4*4, ..., 4*8, 5*5, ..., 8*5, 6*6, 6*7, 6*8, 7*7, 7*8, 8*8

Es sind also insgesamt 36 Möglichkeiten. Natürlich kann man diese nicht alle aus einem Brett schneiden. Rechnet man alle Teilquadrate zusammen kommt man auf auf über 700 Felder.

rot: Aus der letzten Bemerkung bei blau wird klar, soll genau ein Schachbrett (64 Teilquadrate) aus möglichst vielen verschiedenen Stücken zusammen gesetzt werden, so dürfen aus möglichst wenigen Teilquadraten bestehen. Die kleinen sind 1*1, 1*2, 1*3, 1*4, 2*2, 1*5, 1*6, 2*3, 1*7, so jetzt sind es schon 38 Teilquadrate. 1*8, 2*4 dazu führt auf 54 Teilquadrate. 1*9 gibt es nicht, also kommt als nächst größeres die 2*5. Damit sind es maximal zwölf Stücke die die 64 Stücke eines Schachbrettes bilden können. Sind die Teilstücke größer, wird die zulässige Anzahl kleiner. Es wurden mehrere Versionen gefunden - am schnellsten war XXX, so dass dessen Beispiel hier gezeigt wird, danke.

 

1x8

 

 

 

 

 

 

 

 

2x5

 

 

 

2x2

 

1x7

 

 

 

 

 

 

 

 

 

2x3

 

 

2x4

 

 

 

 

 

 

 

 

 

 

 

 

1x6

 

 

 

 

 

 

 

1x5

 

 

 

 

1x3

 

 

1x4

 

 

1x1

1x2

 

 Anmerkung: rote Punkte hätte natürlich jeder haben können, mindestens 1, wenn er sagt ich nehme das 8*8, aber nun ja.


 

Aufgabe 11

299. Wochenaufgabe

"Die Aufgabe von letzter Woche gefiel mir richtig gut, hinzu kam, dass die größte rote Punktzahl auch gerade noch die 12 war, welche als Maximalpunktzahl bei der Wochenaufgaben üblicherweise vorkommen kann", meinte Bernds Opa. "Aber habt ihr euch auch mal über die Restschachbretter Gedanken gemacht.?" "Wie meinst du das?", hakte Maria nach. "Pass mal auf. Eine der Formen von letzter Woche war das 2x2 Quadrat. Das kannst du von einer Ecke ausschneiden, aber auch irgendwo anders. Der übrig bleibende Rest des Brettes sieht dann natürlich  anders aus." "Stimmt.". Wie viele verschiedene Reste gibt es beim Ausschneiden von 2 x2 Quadraten. 6 blaue Punkte. Die Farben der Felder oder auch die Betrachtung von der Rückseite des Brettes aus begründen keine Verschiedenheit. Wie viele Restformen gäbe es beim 10 x  10 internationalen Damefeld, wenn man dort diese 2 x 2 Felder ausschneiden würde? 6 rote Punkte. Für die Herleitung einer Formel für ein n x n -Feld gibt es noch mal extra Punkte.

Lösung:

Als Lösungsbeispiel die Variante von XXX, danke:

als pdf


 

Aufgabe 12

300. Wochenaufgabe

"Findest du nicht auch, dass 300 eine besondere Zahl ist?", fragte Bernd. "Ich weiß nicht, wobei, 300 Wochenaufgaben sind schon eine Menge, wobei wir ja nicht von Anfang an dabei waren." "Das stimmt, aber egal, da wollen wir mal nicht so sein." "Dann lass uns die 300 mal genauer anschauen, einverstanden?"
Wie viele Multplikationsaufgaben, die das Ergebnis 300 haben, gibt es? (natürliche Zahlen (größer 1) sollen verwendet werden, Vertauschung der Faktoren führt zu keiner neuen Aufgabe.) 8 blaue Punkte.
Wie viele Additionsaufgaben gibt es, so dass das Ergebnis 300 ist. Bedingung: die Summanden sollen (maximal 3) Primzahlen sein. (Vertauschung der Summanden zählt nicht als verschieden) 6 rote Punkte, wenn man sicher nachweist, dass man alle gefunden hat.

 

Lösung:

Blau: Zur Vereinfachung wird verzichtet immer wieder auf die Nichtzulassung der Vertauschung hinzuweisen.
Zerlegung in Primfaktoren führt auf 2*2*3*5*5  - also 1 Möglichkeit mit 5 Faktoren.
Für vier Faktoren müssen zwei der Primfaktoren zu einem Faktor zusammengefasst werden. ==> Daraus ergeben sich 5 Möglichkeiten. Für drei Faktoren sind es dann 11 Möglichkeiten und für zwei Faktoren noch einmal 8. Die letzt genannten wurden von den meisten Teilnehmern notiert, mehr als Faktoren wurden releativ selten notiert, nun ja. Eine Auswahl von Ergebnissen: 3*4*5*5; 2*2*75; 3*4*25; 5*5*12; 10*30; 5*60; ...

Rot: Mit Hilfe der Primzahlen im Bereich bis zur 300 (Tabelle - letzte Spalte) lassen sich die passenden Summen aus zwei Summanden recht schnell finden. Es sind 21, da ja auch hier die Vertauschung nicht weiterführt. Für drei Summanden gilt, dass einer der Summanden die "2" sein muss. Die systematische Suche erfolgt nun wieder und führt siehe Tabelle auf 11 Möglichkeiten. Es sind also insgesamt genau 32 Möglichkeiten die 300 in 2 bzw. 3 "Primsummanden" zu zerlegen. Hinweis: lässt man 5 Summanden zu, so gibt es rund 2.500 Möglichkeiten.

 





2, 3, 5, 7, 11, 13, 17, 19, 23, 29, 31, 37, 41, 43,
3 297 - - 47, 53, 59, 61, 67, 71, 73, 79, 83, 89, 97, 101, 103, 107,
5 295 - 5+293+2 109, 113, 127, 131, 137, 139, 149, 151, 157, 163, 167, 173, 179, 181,
7 293 7+293 2+5+293 D 191, 193, 197, 199, 211, 223, 227, 229, 233, 239, 241, 251, 257, 263,
9 291 - - 269, 271, 277, 281, 283, 293,
11 289 - -
13 287 - -
15 285 -

17 283 17+283 17+281+2
19 281 19+281 17+2+281 D
21 279 - -
23 277 23+277 -
25 275 - -
27 273 - -
29 271 29+271 29+269+2
31 269 31+269 29+2+269 D
33 267 - -
35 265 - -
37 263 37+263 -
39 261 - -
41 259 - 41+257+2
43 257 43+257 41+2+257 D
45 255 - -
47 253 - 47+251+2
49 251 - 47+2+251 D
51 249 - -
53 247 - -
55 245 - -
57 243 - -
59 241 59+241 59+239+2
61 239 61+239 59+2+239 D
63 237 - -
65 235 - -
67 233 67+233 -
69 231 -

71 229 71+229 71+227+2
73 227 73+227 71+2+227 D
75 225 - -
77 223 - -
79 221 - -
81 219 - -
83 217 - -
85 215 - -
87 213 - -
89 211 89+211 -
91 209 - -
93 207 - -
95 205 -

97 203 - -
99 201 - -
101 199 101+199 101+197+2
103 197 103+197 101+2-197 D
105 195 - -
107 193 107+193 107+191+2
109 191 109+191 107+2+191 D
111 189 - -
113 187 - -
115 185 - -
117 183 - -
119 181 - -
121 179 - -
123 177 - -
125 175 - -
127 173 127+173 -
129 171 - -
131 169 - 131+167+2
133 167 - 131+2+167 D
135 165 - -
137 163 137+163 -
139 161 - -
141 159 - -
143 157 - -
145 155 - -
147 153 - -
149 151 149+151 149+2+149




Auswertung Serie 25 (blaue Liste)

Platz Name Ort Summe Aufgabe

289 290 291 292 293 294 295 296 297 298 299 300
1. Rafael Seidel Chemnitz 64 10 6 8 2 4 5 6 3 2 4 6 8
2. Sabine Fischbach Hessen 63 10 6 8 2 4 5 6 3 2 4 5 8
3. Doreen Naumann Duisburg 62 10 6 8 - 4 5 6 3 2 4 6 8
4. Elisa Parsche Chemnitz 60 10 6 8 2 4 5 6 3 2 - 6 8
5. Astrid Fischer Chemnitz 40 10 - - 2 4 5 1 - - 4 6 8
6. Linus-Valentin Lohs Chemnitz 35 - - - 2 4 2 6 3 - 4 6 8
7. Hermann Thum Chemnitz 34 5 - 8 2 - - 6 3 - 4 6 -
7. XXX ??? 34 - 6 - - 4 - 6 - - 4 6 8
8. Richard Hahmann Chemnitz 29 - - 8 - - 5 - - - 4 6 6
8. Uwe Parsche Chemnitz 29 - - - - - - 6 3 2 4 6 8
9. Felix Haase Chemnitz 27 - - 8 2 - 5 - 3 2 4 - 3
10. Kai-Lutz Wagner Chemnitz 25 8 - - 2 - - - 3 - - 4 8
11. Nina Zätsch Chemnitz 24 5 - 8 2 - - 1 3 - 2 - 3
11. Mawi Dresden 24 10 6 8 - - - - - - - - -
11. Jamila Wähner Chemnitz 24 6 - 8 - - - 3 3 - 4 - -
11. Loise Reichmann Chemnitz 24 - - 8 2 - 5 - 3 - - 3 3
12. Lisa Grassmann Chemnitz 23 - 4 8 - 4 - - - - - - 7
13. Anja Posselt Chemnitz 22 - - 8 2 - 5 - - - 4 - 3
14. Marie Sophie Roß Chemnitz 21 - - 8 2 - 5 - 3 - - - 3
15. Pauline Marschk Chemnitz 20 - - - - 3 4 - 3 2 - - 8
15. Felix Taubert Chemnitz 20 - - - - 3 5 - - 2 4 6 -
16. Paula Hartmannsdorf 17 - - - - - - - - - 4 5 8
17. Jonathan Kässler Chemnitz 16 - - 8 - - - 2 3 - - - 3
17. Ellen Richter Chemnitz 16 - - 4 - - 5 2 - - 3 - 2
18. Andree Dammann München 15 - 6 - - 3 - - 3 - - - -
19. Ellen Wilde Chemnitz 14 - - - - - 5 - - - 3 6 -
19. Daniel Hufenbach Leipzig 14 - - 8 2 4 - - - - - - -
19. Johanna Ranft Chemnitz 14 6 - - 2 - - - 3 - - - 3
20. Laura Schlosser Chemnitz 13 - - - - - 5 - 3 2 - - 3
20. Philipp Fürstenberg Chemnitz 13 - - 8 - - 5 - - - - - -
21. Hannah Gebhardt Chemnitz 12 - - - - - - 2 - - 4 6 -
22. Duncan Mahlendorff Chemnitz 11 - - 8 - - - - 3 - - - -
22. Clara Stöckel Chemnitz 11 - - - - - - - - - 3 - 8
22. Theresa Jänich Chemnitz 11 - - - - - 4 - - 1 - 6 -
22. Ernesto Uhlmann Chemnitz 11 - - - - - 5 - - - - 6 -
23. Ole Koelb Chemnitz 10 - - - - - 3 - - - - 4 3
23. Emilie Grossinger Chemnitz 10 - - - - - 4 - - - - 6 -
23. Julia Voigt Chemnitz 10 - - - - - 4 - - - - 6 -
23. Jule Irmscher Eibenberg 10 - - - - - - - - - 2 - 8
23. Jonas Frederik Otto Lichtenwalde 10 - - - 2 - - - - - - - 8
23. Ria Hopke Chemnitz 10 - - - 1 - - - 3 - - - 3
24. Michelle Wade Chemnitz 9 - - - - - - - - - 2 - 7
24. Henrike Grundmann Chemnitz 9 - - - - - 3 - - - - 6 -
24. Helene Fischer Chemnitz 9 - - - - - - 1 - - - - 8
25. Pit Hopke Chemnitz 8 - - - - - - - - - 1 - 7
25. Anne Haag Chemnitz 8 - - - - - - - - - 1 - 7
25. Joshua May Chemnitz 8 - - - - - - - - - 1 - 7
25. Laurin Roßberg Chemnitz 8 - - - - - - - - - 2 - 6
25. Celestina Montero Perez Chemnitz 8 - - - - - - - - - - - 8
25. Christian Wagner Bamberg 8 8 - - - - - - - - - - -
25. Justine Schlächter Chemnitz 8 - - - - - - - - - 1 - 7
25. Tobias Morgenstern Chemnitz 8 - - - - - - - - - - - 8
25. Tim Sigmund Chemnitz 8 - - - - - - - - - 1 - 7
25. Stephanie Dani Chemnitz 8 - - 8 - - - - - - - - -
25. Carl Geißler Chemnitz 8 - - - - - - - - - - - 8
26. Charline Patzelt Chemnitz 7 - - - - - - - - - - - 7
26. Marvin Köllner Chemnitz 7 - - - - - - - - - 1 - 6
26. Erik Walther Chemnitz 7 - - - - - - - - - 3 - 4
26. Paula Mühlmann Dittersdorf 7 - - - - - - - - - - - 7
26. Willy Stöckel Chemnitz 7 - - - - - 4 - 3 - - - -
26. Katrin Posselt Chemnitz 7 - - - 2 - 5 - - - - - -
26. Josephine Klotz Chemnitz 7 - - - - - - - - - 2 - 5
26. Ingmar Richter Chemnitz 7 4 - - - - - - - - - - 3
26. Malte Gebhardt Chemnitz 7 - - - - - - - - - - - 7
26. Lisa Berger Chemnitz 7 - - - - - - - - - - - 7
26. Frederike Meiser Chemnitz 7 - - - - - - - - - - - 7
26. Saskia Schlosser Chemnitz 7 - - - - - - - - - - - 7
27. Felix Brinkel Chemnitz 6 - - - - - - 6 - - - - -
27. Ida Heuschkel Chemnitz 6 - - - - - - - - - - - 6
27. Joel Magyar Chemnitz 6 - - - - - - - - - - - 6
27. Nicklas Reichert Chemnitz 6 - - - - - - - - - - - 6
27. Tobias Richter Chemnitz 6 - - - - - - - - - - - 6
27. Kevin Ngyen Chemnitz 6 - - - - - - - - - - - 6
27. Lene Haag Chemnitz 6 - - - - - - - - - - - 6
27. Robin König Chemnitz 6 - - - - - - - 3 - - 3 -
27. Katharina Zweiniger Chemnitz 6 - - - - - - - - - - - 6
27. Paula Geißler Chemnitz 6 - - - - - - - - - - - 6
27. Leon Hopke Chemnitz 6 - - - - - - - - - - - 6
28. Pascal Graupner Chemnitz 5 - - - - - - - 2 - - - 3
28. Marion Sarah Zenk Chemnitz 5 - - - - - - - 3 2 - - -
28. Lina Krug Chemnitz 5 - - - - - - - - - 2 - 3
28. Maria Dreßler Chemnitz 5 - - - - - - - - - 2 - 3
28. Jonas Kirchberg Chemnitz 5 - - - - - - - - - - - 5
28. Lena Elisa Penzlin Chemnitz 5 - - - - - - - 2 - - - 3
28. Marie Berger Chemnitz 5 - - - - - - - - - - - 5
28. Selma Juhran Chemnitz 5 - - - - - - - - - - - 5
28. Gwendolin Eichler Chemnitz 5 - - - - - - - 2 - - - 3
28. Florian A. Schönherr Chemnitz 5 - - - - - - - 2 - - - 3
28. Alina Berger Chemnitz 5 - - - - - - - - - 3 - 2
28. Eva-Lotta Rümmler Chemnitz 5 - - - - - - - - - 1 - 4
29. Shari Schmidt Chemnitz 4 - - - - - - - - - - - 4
29. Jessica Spindler Chemnitz 4 - - - - - - - - - - - 4
29. Dr. Frank Göring Chemnitz 4 - - - - 4 - - - - - - -
29. Annika Theumer Chemnitz 4 - - - - - - - - - 4 - -
29. Hannes Hohmann Chemnitz 4 - - - - - - - - - - - 4
29. Hanna Kallenbach Chemnitz 4 - - - - - - - - - 1 - 3
29. Camilla Schreiter Chemnitz 4 - - - - - 4 - - - - - -
29. Luis Raupach Chemnitz 4 - - - - - - - 3 1 - - -
29. Emily Neuwirth Chemnitz 4 - - - - 4 - - - - - - -
29. Amarin Roßberg Chemnitz 4 - - - - 4 - - - - - - -
29. Emmely Schöne Chemnitz 4 - - - - - - - - - 1 - 3
29. Amerin Roß Chemnitz 4 - - - - - 2 - - - - 2 -
29. Luisa Franke Chemnitz 4 - - - - - - - - - 1 - 3
29. Lucas Steinke Chemnitz 4 - - - - - 4 - - - - - -
29. Celine Strumpf Chemnitz 4 - - - - - - - - - - - 4
30. Gunnar Reinelt Chemnitz 3 - - - - - - - - - - - 3
30. Mara Neudert Chemnitz 3 - - - - - - - - - - - 3
30. zge Chemnitz 3 - - - - - - - 3 - - - -
30. Friederike Lenk Chemnitz 3 - - - - - 3 - - - - - -
30. Rebecca Wagner Oberwiesenthal 3 - - - - - - - - - - - 3
30. Arne Weißbach Chemnitz 3 - - - - - - - - - - - 3
30. Anna Grünert Chemnitz 3 - - - - - - - - - - - 3
30. Heinrich Grossinger Chemnitz 3 - - - - - - - - - - - 3
30. Moritz Weber Chemnitz 3 - - - - - - - - - - - 3
30. Karl Herrmann Chemnitz 3 - - - - - - - - - - - 3
30. Lisanne Brinkel Chemnitz 3 - - - - - - - - - - - 3
30. Lena Rabbeau Chemnitz 3 - - - - - - - - - - - 3
30. Josephine Pallus Chemnitz 3 - - - - - - - 3 - - - -
30. Felicitas Hastedt Chemnitz 3 - - - - - - - - - - - 3
30. Adrian Schlegel Chemnitz 3 - - - - - - - - - - - 3
30. Simon Winger Chemnitz 3 - - - - - - - - - - - 3
31. Ulrike Böhme Chemnitz 2 - - - - - - - - - - - 2
31. Franz Kemter Chemnitz 2 - - - - - - - - - - - 2
31. Tom Straßer Chemnitz 2 - - - - - - - - - - - 2
31. Emma Irmscher Eibenberg 2 - - - 0 - - - - - - 2 -
31. Valentin Sellin Chemnitz 2 - - - - - - - - - - - 2
31. Marvin Gülden Chemnitz 2 - - - - - - - - - - - 2
31. Elena Oelschlägel Chemnitz 2 - - - - - - - - - - - 2
31. Johannes Allert Chemnitz 2 - - - - - - - - - 2 - -
31. Simon Anders Chemnitz 2 - - - - - - - - - - - 2
31. Valentin Grundmann Chemnitz 2 - - - - - - - - - - - 2
31. Melanie Petz Chemnitz 2 - - - - - - - - - - - 2
31. Karolin Schuricht Chemnitz 2 - - - - - - - - 2 - - -
31. Elina Rech Chemnitz 2 - - - - - - - - - - - 2
31. Nele Mäding Chemnitz 2 - - - - - - - - - - - 2
31. Anna Georgi Chemnitz 2 - - - - - - - - - - - 2
31. Lilli Weiß Chemnitz 2 - - - - - - - - - - - 2
31. Cynthia Raschkowsky Chemnitz 2 - - - - - - - - - - - 2
32. Jessica Ritter Chemnitz 1 - - - - - - - - - - - 1
32. Christian Winkler Chemnitz 1 - - - - - - - - - - - 1
32.

1 - - - - - - - - - 1 - -


Auswertung Serie 25 (rote Liste)

Platz Name Ort Summe Aufgabe

289 290 291 292 293 294 295 296 297 298 299 300
1. Doreen Naumann Duisburg 75 10 3 12 1 8 5 2 4 2 12 10 6
2. Sabine Fischbach Hessen 69 10 6 12 2 8 5 1 2 - 10 7 6
3. Elisa Parsche Chemnitz 58 10 6 12 3 8 5 - 4 - - 10 -
4. Astrid Fischer Chemnitz 53 10 - - 3 8 5 - - - 12 9 6
5. Uwe Parsche Chemnitz 47 - - - - - - 5 6 8 12 10 6
6. XXX ??? 46 - 5 - - 8 - 5 - - 12 10 6
7. Mawi Dresden 30 10 6 12 2 - - - - - - - -
8. Paula Hartmannsdorf 27 - - - - - - - - - 11 10 6
9. Richard Hahmann Chemnitz 24 - - - - - 5 - - - 8 6 5
10. Linus-Valentin Lohs Chemnitz 23 - - - 2 - - - - - 11 10 -
11. Daniel Hufenbach Leipzig 22 - - 12 2 8 - - - - - - -
12. Andree Dammann München 19 - - - - 6 - - 6 - - - -
13. Felix Haase Chemnitz 16 - - - - - 5 - 3 - 8 - -
14. Rafael Seidel Chemnitz 12 - - 12 - - - - - - - - -
14. Annika Theumer Chemnitz 12 - - - - - - - - - 12 - -
15. Josephine Klotz Chemnitz 11 - - - - - - - - - 11 - -
16. Michelle Wade Chemnitz 10 - - - - - - - - - 10 - -
16. Christian Wagner Bamberg 10 10 - - - - - - - - - - -
17. Maria Dreßler Chemnitz 9 - - - - - - - - - 9 - -
17. Marvin Köllner Chemnitz 9 - - - - - - - - - 9 - -
18. Hanna Kallenbach Chemnitz 8 - - - - - - - - - 8 - -
18. Dr. Frank Göring Chemnitz 8 - - - - 8 - - - - - - -
18. Jule Irmscher Eibenberg 8 - - - - - - - - - 8 - -
18. Eva-Lotta Rümmler Chemnitz 8 - - - - - - - - - 8 - -
19. Marie Sophie Roß Chemnitz 7 - - - - - 5 - 2 - - - -
19. Loise Reichmann Chemnitz 7 - - - - - 5 - 2 - - - -
20. Anne Haag Chemnitz 6 - - - - - - - - - 6 - -
20. Pit Hopke Chemnitz 6 - - - - - - - - - 6 - -
20. Emmely Schöne Chemnitz 6 - - - - - - - - - 6 - -
21. Jamila Wähner Chemnitz 5 - - - - - - 5 - - - - -
21. Justine Schlächter Chemnitz 5 - - - - - - - - - 5 - -
21. Philipp Fürstenberg Chemnitz 5 - - - - - 5 - - - - - -
21. Julia Voigt Chemnitz 5 - - - - - 5 - - - - - -
21. Katrin Posselt Chemnitz 5 - - - - - 5 - - - - - -
21. Ria Hopke Chemnitz 5 - - - - - - 5 - - - - -
21. Alina Berger Chemnitz 5 - - - - - - - - - 5 - -
22. Robin König Chemnitz 4 - - - - - - - 4 - - - -
22. Luisa Franke Chemnitz 4 - - - - - - - - - 4 - -
22. Kai-Lutz Wagner Chemnitz 4 - - - 1 - - - 3 - - - -
23. Joshua May Chemnitz 3 - - - - - - - - - 3 - -
23. Luis Raupach Chemnitz 3 - - - - - - - 2 1 - - -
23. Felix Taubert Chemnitz 3 - - - - - 1 - - 2 - - -
24. Lisa Grassmann Chemnitz 2 - - - 2 - - - - - - - -
24. Willy Stöckel Chemnitz 2 - - - 2 - - - - - - - -
24. Marion Sarah Zenk Chemnitz 2 - - - - - - - 2 - - - -
24. Tim Sigmund Chemnitz 2 - - - - - - - - - 2 - -
24. Malte Gebhardt Chemnitz 2 - - - - - - - - - 2 - -
24.

2 - - - - - - - - - 2 - -
24. Theresa Jänich Chemnitz 2 - - - - - 2 - - - - - -
24. Pascal Graupner Chemnitz 2 - - - - - - - 2 - - - -
24. Jonathan Kässler Chemnitz 2 - - - - - - - 2 - - - -
24. Pauline Marschk Chemnitz 2 - - - - 2 - - - - - - -
24. Laura Schlosser Chemnitz 2 - - - - - - - - 2 - - -

Valid XHTML 1.0 Transitional

Serie-24

Serie 24
Aufgaben und Lösungen

Aufgabe 1

277. Wertungsaufgabe

277. Wertungsaufgabe
"Ich freue mich schon auf den Faschingsauftritt unseres Mathezirkels. Es werden 4 unserer Teilnehmer in ihrer Verkleidung als Gauß, Newton, Einstein und Galilei, mit ihrem persönlichen Buch in der Hand auftreten. Leider kann Paula nicht dabei sein, aber Alexandra, Corinna, Anja und Zeta." Klingt doch gut", meinte Bernd, "gib es noch mehr Informationen?" "Aber klar doch", erwiderte Lisa. Die Mathematiker kommen jeweils in einem der Büchern vor, die mit Band 1 bis 4 nummeriert sind.
Sie kamen nacheinander auf die Bühne.

  1. Die als Zweite auf die Bühne kommt, hat den Band 4 in der Hand.
  2. Anja hat einen Band, dessen Nummer direkt nach nach dem Buch mit Einstein kommt.
  3. Zeta kommt später auf die Bühne als Galilei.
  4. Alexandra als Gauß hat weder den Band 1 noch Band 4
  5. Corinna kommt später als die "Mathematikerin", die in Band 1 vorkommt.

Wer kommt wann (Reihenfolge) in welcher Verkleidung und mit welchem persönlichen Buch auf die Bühne? 6 blaue Punkte
Inzwischen war Maria noch ins Zimmer gekommen und hatte den Rest von Lisas Information gehört. "Weißt du noch wie wir die erste Probe hatten?" "Aber ja doch, das war ja auch noch die Paula mit dabei". "Stimmt, alle kamen im Abstand von 15 Minuten ins Zimmer, hat jeweils ein anders farbiges T-Shirt an, auf denen jeweils ein Musikinstrument aufgedruckt war." "Okay", meinte Mike, "dann gebt mal die genaueren Informationen preis."
Die Zeiten waren 14.45 Uhr bis 15.45 Uhr. Die Farben der T-Shirts waren rot, blau, grün, weiß und schwarz. Auf den T-Shirts waren:
Cello, Harfe, Klavier, eine Tube und – echt stark – eine Triangel.
  1. Anja kam zu erst.
  2. Das Mädchen, die direkt nach Cello-Corinna kam, hatte ein weißes T-Shirt.
  3. 15.15 Uhr trat das Mädchen mit dem Triangel-T-Shirt ins Zimmer.
  4. Direkt nach dem Mädchen mit dem Klavier-T-Shirt, kam das Mädchen - es war nicht Alexandra - mit dem roten Tuba-T-Shirt.
  5. Um 15.00 Uhr kam das Mädchen mit dem schwarzen T-Shirt.
  6. Paula hatte ein grünes T-Shirt.
Zu erreichen sind 6 rote Punkte

Lösung

Auf die genaue Dastellung des Lösungsweges wird dieses mal verzichtet.
blau:

Auftrittsreihenfolge Name Mathematiker Nummer des Bandes
1 Alexandra Gauß 2
2 Anja Galilei 4
3 Zeta Newton 1
4 Corinna Einstein 3
rot:
Uhrzeit Name Instrument Farbe des Shirts
14.45 Uhr Anja Harfe blau
15.00 Uhr Corinna Cello schwarz
15.15 Uhr Alexandra Triangel weiß
15.30 Uhr Paula Klavier grün
15.45 Uhr Zeta Tuba rot



Aufgabe 2

278. Wertungsaufgabe
"In unserem Mathezirkel untersuchen wir große und kleine natürliche Zahlen, erzählte Maria am Abendbrotstisch. "Erzähl mal weiter," forderte ihr Vater sie interessiert auf. "Als erstes suchten wir die kleinste natürliche Zahl (größer als Null), die sich ohne Rest durch alle natürlichen Zahlen von 1 bis 10 teilen lässt. (2 blaue Punkte)
Die andere Aufgabe war dann schon was für unsere richtigen Experten. Welche ist die kleinste natürliche Zahl (größer 1) aus der sich die 2., die 3. , .. und die 10. Wurzel ziehen lässt, so dass die Ergebnisse natürlich auch wieder natürliche Zahlen sind." - 3 rote Punkte

Lösung

blau: Mal ohne Primfaktorenzerlegung. Die Zahl muss durch 10 teilbar und durch 9, also durch 90. Auch der Faktor (Teiler) sieben wird gebraucht. Das aber geht nur, wenn die gesuchte Zahl 7 · 90 = 630 enthält. So muss die gesuchte Zahl ein Vielfaches von 630 sein. 1260 - aber das geht nicht durch 8. 1890 geht auch nicht, aber 2520 enhählt allen gesuchten Teiler. (Die Primzahlzerlegung beweist das auch noch mal: 2520 = 2³ · 3² · 5 · 7)
rot: Wegen > 1 muss es eine Potenz 2n sein. Jetzt wird zum Beispiel die 5. Wurzel gesucht, dann nutze ich die Exponentialschreibweise. 5. Wurzel aus 2n= 2n/5. n muss also durch 5 teilbar sein. Das gleiche gilt entsprechend 4. Wurzel also n muss durch 4 teilaber sein usw. n ist letztlich die bei blau gesuchte Zahl. 22520 ist die gesuchte Zahl. 3,9408425 · 10758Die Zahl also insgesamt 759 Stellen.
39408424552214162695348543183638915172819172249751642655322154182349336765880096
10655644786388200003560563883371670355420740089454019139502362143605063997052312
03021164366069389563701733455174652493802096528279659381259483508916176782516892
61632215488187059650565457777432980818725650237046825687537631627813593798578816
08885188091378378731800863271837927577487029464607207207704361774773772297845000
22657580657233628383930137914619684009220791267089768552182903618603146950084219
24278007257807164800126572667987375177230234311435842855213499193805644680391721
69626202673688062730898676596391772134889601552116981492110306817797885781410543
59274289556411400436598704927821275214881488970218576557325551889577507340928956
338410400961096026352642413831783448576
Die Potenzen der 2 von 21 bis 22520 hier mal klicken



Aufgabe 3

279. Wertungsaufgabe
"Ich habe heute eine besondere Variante einer Quadratkonstruktion entdeckt", sagte Maria. "Wie meinst du das?", fragte Bernd. "Pass auf.  Man nimmt eine Gerade a und legt auf ihr einen Punkt A fest. Dann legst du einen Punkt X fest, der nicht auf der Geraden liegt, aber so, dass AX nicht senkrecht zur Geraden a liegt. Das gesuchte Quadrat ABCD liegt mit einer Seite auf a. Der Punkte D des gesuchten Quadrates soll von X genauso weit entfernt sein, wie von A. Wie lässt sich nun ein solches Quadrat konstruieren?" (Konstruktionsbeschreibung 5 blaue Punkte.)
Bernd und Mike grübelten eine Weile, dann hatten sie die Konstruktion gefunden. Mike machte zur Übung, die Konstruktion gleich mehrfach. Lisa sah dies und probierte es ebenfalls, allerdings so, dass sie bei ihren Konstruktionen nur den Punkt A auf der Geraden a immer mal anders einzeichnete. Die Lage von X und a blieb also. Als sich Bernd später die Konstruktionen ansah machte er bei der Betrachtung der Lage der jeweils konstruierten Punkte D eine erstaunliche Beobachtung. Als er die Punkte miteinander "elegant" verband, sah er eine klassische Kurve. Welche? - 3 rote Punkte (Name und kurze Begründung)

Lösung

279 Auf dem Bild sind die Konstruktionen für den Punkt D bzw. D1 - Punkt auf A zu erkennen. m ist die Mittelsenkrechte von AX. Deshalb sind die Punkte D bzw. D1 gleichweit von A und X entfernt. (D liegt senkrecht über A bzgl. a, damit eine der Quadratseiten von ABCD auf a liegt.
Da die Quadratseiten AD bzw. AD1 nun konstruiert sind, lassen sich daruas jeweils zwei Quadate konstruieren ABCD (mathematisch positiver bzw. negativer Umlaufsinn) und ABCD1 (mathematisch positiver bzw. negativer Umlaufsinn) Anmerkung: Wird AX senkrecht a zugelassen, so gibt es einen Punkt D mit den geforderten Eigenschaften, aber nicht D1. rot: Die Punkte D liegen auf einer Parabel. Mögliche Definition einer Parabel:
Eine Parabel ist die Menge aller Punkte X, deren Abstand zu einem speziellen festen Punkt – dem Brennpunkt F – und einer speziellen Geraden – der Leitgeraden l – gleich ist.
X wurde durch D ersetzt, F durch X und l durch a. Quelle
Die Punkte D1 " bewegen sich auf a. (letzlich ein Spezialfall einer Parabel.)



Aufgabe 4

280. Wertungsaufgabe
"Die Parabel von letzter Woche hat mir gefallen. Ich hatte das zwar mal gelesen, aber leider auch wieder vergessen", sagte Lisa. "Dafür habe ich hier eine einfache Bastelei. Ich nehme mir ein beliebiges spitzwinkliges Dreieck und halbiere dessen Seiten. Wenn ich diese Halbierungspunkte miteinander verbinde. Entstehen im Inneren des Dreiecks vier Dreiecke." "Du ich habe das mal probiert mit einem gleichseitigen und einem spitzwinklig gleichschenkligen Dreieck. Da sind die vier Dreiecke kongruent zueinander, stimmt 's". "Aber ja, die Frage ist, gilt das aber auch für den allgemeinen Fall?" - 5 blaue Punkte für einen überzeugenden Beweis. Wird der Beweis für das gleichschenklige Dreieck probiert, so gibt es immerhin auch 3 Punkte. (Aber nicht zusammen 8)
"Ich habe deine Vorschrift mal auf ein Dreieck mit 8 cm, 9 cm und 10 cm angewendet. Nun kann ich die Figur zu einer Pyramide falten, deren Dreiecksflächen kongruent sind. Die Oberfläche dieses komischen Tetraeders ist relativ schnell gefunden, aber wie groß ist dessen Volumen?" - 12 rote Punkte.

Lösung

Die Lösung dieser Aufgabe von Dr. F. Göring und XXX, danke.
280.pdf



Aufgabe 5

281. Wertungsaufgabe
"Ich habe neulich wieder mal "Per Anhalter durch die Galaxis" gelesen", sagte Bernd. "Ja, das kenne ich auch," meinte Lisa. "Die berühmte Zahl daraus ist bekanntlich die 42 - die Antwort auf alles." "Nun, das hat sich mir nicht so erschlossen, aber die 42 ist trotzdem interessant, denn sie ist das Produkt aus genau drei verschiedenen Primzahlen. Gibt es eigentlich noch mehr solche Zahlen?. Ich denke schon." Für je zwei solcher Zahlen (kleiner als 100) gibt es einen blauen Punkt. (Achtung die Primzahlen dürfen nicht potenziert auftreten, denn sonst würde die 60 = 2²*3*5 auch die Bedingung erfüllen)
"Wie viele Teiler mögen solche "Primelzahlen" wohl haben", fragte sich Mike. "Du meinst, egal wie groß die auch sind?" "Genau!" 3 rote Punkte, 4 rote Punkte gibt es noch dazu, wer ein "Primelzahlzwillingspärchen" findet - also zwei direkt aufeinander folgende Zahlen, die die Bedingung "Primelzahl" zu sein, erfüllen.

Lösung

Der wissenschaftliche Name der "Primelzahlen" lautet sphenische Zahl infos bei wikipedia
blau die gesuchte Zahlen sind recht einfach zu finden:
Die kleinst mögliche ist: 2*3*5 = 30
2*3*7 = 42 - war schon gegeben.
2*3*11 = 66
2*3*13 = 78
2*3*17 > 100 also 2*3 abgearbeitet
2*5*7 = 70
2*5*11 > 100 also 2*5 abgearbeitet
2*7*11 > 100 also 2 abgearbeitet
3*5*7 > 100 also mit drei geht nichts mehr fertig. rot: Die sphenische Zahl ensteht durch a*b*c (Primzahlen alle verschieden: Diese Produkt lässt sich durch 1, a, b, c, a*b, a*c, b*c und a*b*c teilen. Es sind also genau 8 Teiler.
Die kleinsten Zwillinge sind 230 = 2*5*23 und 231 = 3*7*11
Drillinge gibt es auch: kleinstes Tripel: 1309 (7*11*17), 1310 (2*5*131) und 1311 (3*19*23)
Vierlinge gibt es nicht, denn dann würde eine der Zahlen den Primfaktor 2 in der Form 2*2 enthalten.


Aufgabe 6

282. Wertungsaufgabe
"Die blaue Aufgabe der letzten Woche fand ich aufwändiger als den roten Teil", meinte Mike, der sich die doch recht vielen Varianten anschaute. "Na ja, es musste mehr geschrieben werden, aber bei systematischer Betrachtung war das doch nicht so schlimm", gab Lisa zu bedenken.
" Da nun das Osterfest vor der Tür steht, kam mir die klassische Eieraufgabe wieder in den Sinn." "Lass hören." Ein Osterhase ist auf dem Weg um Ostereier zu verstecken. Er trifft auf einen Kollegen, dessen Korb leer ist. Er gibt ihm die Hälfte seiner Eier und ein halbes. Wenig später teilt er den verbliebenen Rest wieder mit einem Kollegen, dem er die Hälfte und ein halbes Ei gibt. So geht das noch zwei mal. Kurz vor dem Ziel angelangt, hat er so nur noch genau ein Ei im Korb. Das isst er selbst und geht nach Hause. Wie viele Eier hatte er zu Beginn im Korb? - 4 blaue Punkte (natürlich hat er nicht wirklich irgendeines der Eier geteilt.)
Wie viele Eier müsste der Osterhase in seinem nichtleeren Korb mindestens haben, um, ohne eines der Eier kaputtzumachen, folgende Teilungsvarianten vorzunehmen:

Der erste Kollege bekommt die Hälfte aller Eier und ein halbes Ei, der zweite ein Drittel des Restes und ein Drittel-Ei. Dann gibt es ein Viertel und ein Viertel-Ei und zum Schluss ein Fünftel und ein Fünftel-Ei. 4 rote Punkte (Die im Korb verbleibenden Eier sollte er diesmal besser verteilen und nicht aufessen.)

 

Lösung:

Es gab eine recht große Beteiligung, aber dabei waren leider auch eine Reihe falscher Angaben. Es besonders die Angabe, dass es nicht wirklich kaputte Eier geben darf, wurde missachtet. Etwas schade, dass die schöne rote Aufgabe nur von wenigen in Angriff genommen wurde, vor allem, wenn doch das Prinzip bei blau deutlich wurde, aber okay.

blau: Eine von vielen Varianten. Wenn der Hase am Ende ein Ei hat, dann waren es vorher x, so dass x - x/2 -1/2 = 1 gilt. (Für Nichtgleichungsleser: Er hatte x Eier, von denen die Hälfte ab und ein halbes gab er auch noch weg. Umstellen oder Probieren führt auf x = 3. Vorher hatte er y Eier mit y - y/2 -1/2 = 3. Das führt auf y = 7. Die nächste Überlegung führt auf 15 und schließlich auf 31 Eier.

rot: Bei blau sieht man dass die Eierzahl ungerade sein muss. Will man am ein drittel Ei zu einem Rest von Eiern ergänzen, so geht das bei 2/3 bei 5/3, bei 8/3 ... Also bei einer Zahl die 1 kleiner ist als eine durch drei teilbare Zahl. Für das 1/4 Ei gilt entsprechend. Es muss sich um eine Zahl handeln, die um 1 kleiner ist als eine durch 4 teilbare Zahl. ... Die gesuchte Startzahl ist also um 1 kleiner als eine Zahl, die durch 2; 3; 4 und 5 teilbar ist. Eine solche Zahl ist das KgV der Zahlen (oder ein Vielfaches davon). Das kgV ist 60, die Startzahl also 59. (Die Teilungsvarianten gehen aber mit mit jedem Vielfachen von 60, wenn man dann noch 1 subtrahiert. so auch mit 5! -1 = 119)

der Test:

59 - (59/2 + 1/2) = 59 - 30 = 29

29 - (29/3 + 1/3) = 29 - 10 = 19

19 - (19/4 + 1/4) = 19 - 5 = 14

14 - (14/5 + 1/5) = 14 - 3 = 11

(Man könnte sogar noch weiter machen: 11 - (11/6 + 1/6) = 11 - 2 = 9, weil die 6 im obigen kgV als Teiler drin steckt.)
Interessant auch die Aufgabenstellung von XXX 1. die Hälfte und 1/2 Ei  , dann zwei Drittel und ein 2/3 Ei, ... Eine Lösung dafür habe ich noch nicht.


Aufgabe 7

283. Wertungsaufgabe

Additionsmauer „Was zeichnest du denn? Das erinnert mich an die Klasse 3.“,sagte Maria.
„Du hast schon Recht. Es soll eine „Additionsmauer“sein. Man schreibt vier Zahlen (1a, 1b, 1c und 1d) in die Kästchen der unteren Zeile hinein. Dann werden die direkt nebeneinander stehenden Zahlen addiert und das Ergebnis in das obere Kästchen geschrieben (z.B 1a + 1b = 2a). Schritt für Schritt wird nun die Mauer voll.“ „Ach ja, jetzt weiß ich es wieder.“ Um die Mauer eindeutig lösbar zu machen, müssen die Zahlen aber nicht notwendigerweise in der unteren Zeile stehen. Es können auch Ergebnisse der oberen Reihen genutzt werden, um den Rest der Mauer eindeutig zu bestimmen. Für vier blaue Punkte sind alle Varianten anzugeben, wo mindestens Zahlen stehen müssen, um die Mauer eindeutig berechnen zu können
Eine Variante ist schon mal kurz notiert: 1a, 1b, 1c, 1d Für zwei rote Punkte ist die Mauer so auszufüllen, dass alle Beträge der Zahlen der Zeile 1 verschieden sind und ganz oben eine Null steht. - Beispiel angeben reicht.

Lösung:

blau: Leider wurde die Aufgabe nicht von allen Teilnehmern erfasst. Es ging ja darum, die Kombinationen von Feldern zu finden, die mindestens ausgefüllt sein müssen, um die Additionsmauer eindeutig (und widerspruchsfrei ausfüllen zu können. Es leicht nachvollziehbar, dass es 4 Felder sein müssen. Nun um 4 Felder von 10 auszuwählen, gibt es 210 Möglichkeiten. Allerdings gibt es auch "verbotene" Vierer bzw. "unvollständige" Vierer. Verboten: Beispiel: 1a, 1d, 2b und 4. Hier ist nur die Summe S von 1b + 1c ermittelbar. Mit 1b = S - 1c gibt es unendlich viele Lösungen. Unvollständig: Beispiel: 1a, 1b, 1c und 2a. 2a stellt keine neue Information her. sondern ist aus 1a und 1b zwingend.
Rafael hat 120 Möglichkeiten gefunden, Glückwunsch.
Es wäre spannend zu wissen, ob es eine Formel für den Fall einer n-stufigen Additionsmauer gibt.
rot: Beispiel: 1a = -17, 1b = 4, 1c = -1 und 1d = 8
Es kam also darauf an, die Grundschulaufgaben auf negative Zahlen auszudehnen.


Aufgabe 8

284. Wertungsaufgabe

„Du mit deinen „Primelzahlen“, da hast du mich neulich ganz schön auf Trab gehalten,“ knurrte Lisa. „Aber egal, schau mal. Beim letzten Treffen unseres Mathematikzirkels hat uns Alfred eine Zeichnung gezeigt, auf der ein gleichschenklig rechtwinkliges Dreieck zu sehen war. Er hatte einen Punkt X in das Innere gezeichnet und diesen mit den Eckpunkten des Dreiecks verbunden. Die drei den Punkt umgebenden Winkel waren „zufälligerweise“ alle gleich groß. Er hatte alle in der Figur vorkommenden Winkel schon ausgerechnet.“ Wie groß sind die in der Figur vorkommenden Winkel? - 4 blaue Punkte.
„Aber es war ihm bisher noch nicht möglich, eine exakte Konstruktionsvorschrift – und Begründung – anzugeben, wie man den Punkt X bei gegebenen Dreieck durch eine Konstruktion findet.“ 6 rote Punkte.

 

Lösung:

284

Dem Bild kann man die blaue und rote Lösung entnehmen. Der gesuchte Punkt um den es geht, heißt in der Zeichnung H. Es ist der sogenante "innere" Fermatpunkt, der für jedes Dreieick, dessen größer Winkel < als 120° ist existiert. Die drei Winkel, die den Punkt H als Schteilpunkt haben, sollen laut Aufgabenstellung gleich groß sein. Damit sind diese je 120° groß. Allerdings werden ja auch die Winkel bei den Eckpunkten geteilt. Im Dreieck ABH sind die Winkel 30°, 30° und 120° groß. Im Dreieck BCH gibt es 15°, 45° und 120°, genau wie im Dreieck ACH.

Die Konstruktion des Punktes H ist leider nur einmal komplett gelöst worden - Elisa (mit Papa?) aus Klasse 7. Hier wurde die spezielle Form des Dreiecks genutzt, nämlich, dass es gleichschenklig war, bei der Aufgabenstellung vollständig korrekt.

Hier nun die allgemeinere Variante.

Über den Seiten a und b des Dreiecks ABC werden gleichseitige Dreiecke konstruiert. - rötliche Hilfskreise. ZU diesen werden jeweils die Umkreise konstruiert - blaue Kreise. Diese schneiden sich im Punkt H.

Warum ist nun der Winkel CHB 120° groß? Nun die Punkte HBDund C liegen auf einem Kreis. Damit handelt es sich, bei dem aus den Punkten gebildeten Viereck um ein Sehnenviereck. In einem solchen gilt, dass die Summe der gegenüberliegenden Winkel 180° beträgt. Winkel BDC = 60° (gleichseitiges Dreieck) als Winkel CHB = 180° - 60° = 120°.

Entsprechend gilt die Überlegung für den Winkel CHA ==> 120° und so mit alle Winkel bei H 120°.

Praktische Bedeutung des Punktes H. Sind A, B, und C drei Orte, die durch Straßen verbunden werden sollen, die insgesamt möglich kurz sein sollen, so geht dies mit Hilfe des Punktes H.

Zum selberprobieren (Achtung, Java wird gebraucht und es kann zum Absturz des Browsers kommen) < -- hier -->

 



Aufgabe 8

285. Wertungsaufgabe

„Der Punkt aus eurer letzten Aufgabe wird auch Fermatpunkt genannt.“, sagte Bernds Opa, der den Zettel auf dem Tisch hatte liegen sehen. „Ich habe hier zwei ganz besondere Zahlen für euch: 1827 und  2187. Die haben nicht nur die Ziffern gemeinsam, sondern lassen sich jeweils als Produkt zweier zweistelliger Zahlen schreiben, die auch aus genau den Ziffern gebildet werden.“ „Mit etwas Taschenrechnerarbeit habe ich die Lösungen gefunden“, verkündete Mike nach nur 15 Minuten. - 4 blaue Punkte.
„Na gut, dann bekommst du noch die hier: 125460. Die lässt sich auf zwei verschiedene Arten als Produkt zweier dreistelliger Zahlen schreiben, die auch aus genau den Ziffern gebildet werden.“ Für den Fleiß 6 rote Punkte

Lösung:

blau: man kann probieren - geht ja auch schnell, aber es darf auch etwas systematischer sein.

Zum Beispiel 1827 Primfaktorenzerlegung 1827 = 3 * 3 * 7 * 29

Die gesuchten Faktoren (Ziffern 1 8 2 7) müssen sich aus den Primfaktoren bilden lassen und zweistellig sein.

Einer der Fakoren muss die 29 enthalten

==> Faktor 1: 3 *29 = 87 mit 1827 : 87 = 21 ist eine Lösung gefunden.

==> Faktor 1: 7 *29 = 203 zu groß

==> Faktor 1: 3 *3 * 29 = 261 zu groß

So mit die Lösung gefunden und zu gleich gezeigt, dass es nur diese eine gibt.

Mitt dieser Methode lassen sich auch die anderen Zahlen untersuchen:

2187 = 27 * 81

rot: 125460 = 204 * 615 = 246 * 510

Die Zahlen um die es hier geht, heißen auch Vampirzahlen, die Faktoren sind die "Zähne" der Vampire.

Quelle: http://en.wikipedia.org/wiki/Vampire_number

 

 


Aufgabe 10

286. Wertungsaufgabe

„Mir sind neulich zwei alte Umfragen, die an unserer Schule durchgeführt wurden und in der Schülerzeitung standen, wieder in die Hände gefallen“, sagte Mike. „Lass hören“, meinte Bernd. „Bei der ersten wurden alle Schüler befragt. Das waren 650. 500 davon gaben an, sich sportlich zu betätigen. 400 antworteten auf die Frage, ob sie mehr als zwei Stunden pro Woche am Computer verbringen,mit ja. Für genau 100 traf keines der beiden zu.“ „Da lässt sich doch herausfinden, wie viele Sport treiben und trotzdem die Zeit für den Computer haben“, war sich Lisa sicher, die mit im Zimmer war. (4 blaue Punkte - !Begründung nicht vergessen!)

„Hier noch die Angaben zur zweiten Umfrage“, sagte Mike.
Reiseziele: A – Polen B – Frankreich und C – Dänemark
50 Schüler nahmen teil und kreuzten bei A, B und C entweder ja oder nein an – Enthaltungen gab es keine.
Bei der Auswertung ergaben sich folgende Ja-Anteile.
1. 20 A
2. 25 B
3. 30 C
4. 8 A und B
5. 12 B und C
6. 10 A und C
7. 3 A und B und C
Wie viele Schüler waren in keinem der Länder? Wie viele waren in genau einem der Länder? Wie viele waren in Polen, aber nicht in Frankreich? (3*3 rote Punkte)

 

Lösung:

blau: Es gab verschiedene Argumantationsvarianten. Ich greife mal die am häufigsten genutzte auf.

Es sind 650 Schüler, 100 treiben werder Sport, noch sitzen sie am Computer. Es bleiben also 550 Schüler. Addiert man die 500 Sportantworten und die 400 Computerangaben so ergibt sich 900. Da es nur 550 Teilnehmer sind müssen es 900 - 550 doppelte Antworten sein. Es sind also 350, die beides machen. (Nun sind es also 150 mit nur Sport, und 50 mit nur Computer.)

rot: Die Überlegungen laufen, ähnlich, zwecks kurzer Darstellung hier die Variante von mawi, danke:

2) S() heiße Schnittmenge von ()

alle = 50

A = 20

B = 25

C = 30

S(AB) = 8

S(BC) = 12

S(AC) = 10

S(ABC) = 3

Es gilt: alle = keins + A + B - S(AB) + C - S(BC) - ( S(AC) - S(ABC) )

=> 50 = keins + 20 + 25 - 8 + 30 - 12 - (10 - 3) = keins + 48

=> keins = 2 => in keinem dieser Länder waren 2 Schüler

nur in A = A - S(AB) - ( S(AC) - S(ABC) ) = 20 - 8 - (10-3) = 5

nur in B = B - S(AB) - ( S(BC) - S(ABC) ) = 25 - 8 - (12-3) = 8

nur in C = C - S(AC) - ( S(BC) - S(ABC) ) = 30 - 10 - (12-3) = 11

=> Nur in Polen waren 5, nur in Frankreich 8 und nur in Dänemark waren 11 Kinder. Zusammen sind das 24 Kinder, die nur in einem der Länder waren. Sollte nur die Summe gesucht gewesen sein, so kann man auch so darauf kommen:

nur ein Land = alle - keins - S(AB) - ( S(BC) - S(ABC) ) - ( S(AC) - S(ABC) ) = 50 - 2 - 8 - (12-3) - (10-3) = 24

in A aber nicht in B = A - S(AB) = 20 - 8 = 12

=> 12 Kinder waren in Polen aber nicht in Frankreich.

 


Aufgabe 11

287. Wertungsaufgabe

„Ein Teilnehmer an unserem Mathematikzirkel ist am 16.05.2010 Bruder einer Schwester geworden, ein Sonntagskind“, sagte Maria etwas aufgeregt. „Nun da wird sie also an einem Montag ihren ersten Geburtstag feiern, aber wie alt wird sie werden, wenn sie zum ersten Mal wieder an einem Sonntag Geburtstag hat?“ 3 blaue Punkte.
Warum ist der 28. Geburtstag immer der gleiche Wochentag wie der Tag der Geburt? - 2 rote Punkte.

 

Lösung:

blau:

2011 - Montag, 2012 - Mittwoch (Schaltjahr) 2013 - Donnerstag, 2014 - Freitag,

2015 - Samstag, 2016 - Montag (Schaltjahr), 2017 - Dienstag, 2018 - Mittwoch,

2019 - Donnerstag, 2020 - Samstag (Schaltjahr) 2021 - Sonntag (endlich am 11. Geburtstag ist es wieder so weit.

rot: 28 = 4 * 7 Das kgV von Schaltjahrregelung und Verschiebung um eine Woche durch "normale" Jahre.

Einige Beantworter haben auf die Jahrhundertregelung hingewiesen. 1700, 1800, 1900, 2100, 2200, 2300, 2500 sind keine Schaltjahre, deshalb greift beim Überschreiten die 28-er Regel nicht.

Da das Jahr 2000 ein Schaltjahr war, sind alle aktuellen Leser nicht von dieser Ausnahme betroffen.

 


 

Aufgabe 12

288. Wertungsaufgabe


„In unserer Gruppe bereiten wir gerade einen kleinen Schnellrechenwettbewerb vor“, sagte Maria. „Sag doch mal ein Beispiel.“  „Bei einer dreistelligen Zahl ist der Einer sechsmal so groß wie die Hunderterziffer, und die Zehnerziffer ist die Differenz der beiden anderen Ziffern.“ „Na, das ist doch nicht schwer“, meinte Mike.  Stimmt sicher – 2 blaue Punkte.
„Die Aufgabe kann man auch allgemeiner formulieren,“ sagte Bernd, „passt auf“.
Die Einerziffer ist ein ganzzahliges Vielfaches der Hunderterziffer und die Zehnerziffer ergibt sich aus der Differenz der beiden anderen Ziffern.
Für wie viele (und welche) dreistelligen Zahlen trifft die Verallgemeinerung der Aufgabe zu? 5 rote Punkte
Anmerkung. In jedem Fall gelte, die Hunderterziffer ist größer als Null.

Lösung:

blau: kleinste Hunderter-Stelle das Sechsfache da von ist 6 Differenz ist 5 also gesuchte Zahl 156. Hunderterstelle 2 das Sechfache ist 12 - zu groß für eine Ziffernstelle, also ist 156 auch die einzige Zahl.

rot: Antwort von XXX, danke:

Die Einerziffer ist ein ganzzahliges Vielfaches der Hunderterziffer und die Zehnerziffer ergibt sich aus der Differenz der beiden anderen Ziffern.

Dann habe ich Zahlen der Form (x) (nx-x) (nx), wobei n die Art des Vielfachen angibt, also von 0 bis 9 geht.

x wiederum ist eine positive Ziffer und da nx auch eine Ziffer sein soll, beschränkt durch nx<10.
Das ergibt folgende Familien:
n=0: 110, 220, 330, 440, ..., 990, also 9
n=1: 101, 202, 303, ... 909        , also 9
n=2: 112; 224; 336; 448; also 4
n=3; 123;246;369; also 3
n=4; 134;268; also 2
n=5: 145
n=6: 156
n=7: 167
n=8: 178
n=9: 189




Auswertung Serie 24 (blaue Liste)

Platz Name Ort Summe Aufgabe

277 278 279 280 281 282 283 284 285 286 287 288
1. Rafael Seidel Chemnitz 53 6 2 5 5 2 4 12 4 4 4 3 2
2. Doreen Naumann Duisburg 42 6 2 3 5 2 4 3 4 4 4 3 2
2. Elisa Parsche Chemnitz 42 6 2 5 5 2 4 1 4 4 4 3 2
3. Sabine Fischbach Hessen 39 6 2 3 4 2 4 2 3 4 4 3 2
4. Linus-Valentin Lohs Chemnitz 32 - 2 5 5 2 2 1 4 4 4 3 -
5. Kai-Lutz Wagner Chemnitz 27 4 1 4 - 2 4 - 3 - 4 3 2
6. Ellen Richter Chemnitz 20 6 - 4 2 - - - - 4 - 2 2
7. Hermann Thum Chemnitz 19 - - 3 - 2 - - 4 4 4 - 2
8. Pit Hopke Chemnitz 18 6 - - - 2 4 - - - - - 2
9. Ria Hopke Chemnitz 17 6 - 3 2 - - - - 4 - - 2
10. Nina Zätsch Chemnitz 16 - - - - 2 4 - - 4 4 2 -
11. Lucas Steinke Chemnitz 15 - - 4 2 2 2 - 3 - - - 2
11. XXX ??? 15 - 2 - 5 2 4 - - - - - 2
11. Mawi Dresden 15 - - - - - - - - 4 6 3 2
11. Loise Reichmann Chemnitz 15 6 - - - - 4 - - - - 3 2
12. Marie Sophie Roß Chemnitz 14 - - 4 - 2 4 - - - - 2 2
13. Jamila Wähner Chemnitz 12 - - - - 2 4 - - 4 - - 2
13. Stephanie Dani Chemnitz 12 6 - - 2 - 4 - - - - - -
14. Anja Posselt Chemnitz 11 - - - - 2 - - - 4 - 3 2
14. Carl Gei Chemnitz 11 6 - - - - 3 - - - - - 2
14. Jule Irmscher Eibenberg 11 - 1 1 - 2 1 - - 4 - 2 -
14. Paula Hartmannsdorf 11 - 2 5 - 2 - - - - - - 2
14. Lisa Grassmann Chemnitz 11 - - - - 2 3 - - 4 - - 2
15. Dr. Frank Göring Chemnitz 10 - - 5 5 - - - - - - - -
15. Jonathan Kässler Chemnitz 10 6 - - - 2 - - - - - - 2
15. Robin K Chemnitz 10 6 - - - 2 - - - - - - 2
16. Andree Dammann M 9 - - - - - - - - 4 - 3 2
16. Christian Wagner Bamberg 9 6 - - - - - - - - - 3 -
16. Astrid Fischer Chemnitz 9 - - - - - - - - - 4 3 2
17. Vincent Baessler Chemnitz 8 6 - - - 2 - - - - - - -
17. Karolin Schuricht Chemnitz 8 - - - - - 4 - - 4 - - -
18. Richard Hahmann Chemnitz 7 5 - - - - - - - - - - 2
18. Alina Berger Chemnitz 7 6 1 - - - - - - - - - -
18. Adrian Schlegel Chemnitz 7 - 2 - - 1 3 - 1 - - - -
18. Pauline Marschk Chemnitz 7 6 1 - - - - - - - - - -
19. Felix Brinkel Chemnitz 6 - - - - 2 - - - - 2 2 -
19. Luis Raupach Chemnitz 6 - - - - 2 - - 4 - - - -
19. Philipp Fürstenberg Chemnitz 6 6 - - - - - - - - - - -
19. Samuel Kilian Stuttgart 6 6 - - - - - - - - - - -
19. Ole Koelb Chemnitz 6 - 2 - - - - - - 4 - - -
20. Ingmar Richter Chemnitz 5 - - - - 1 - - - 4 - - -
20. Felix Taubert Chemnitz 5 - - - - 1 - - 4 - - - -
20. Duncan Mahlendorff Chemnitz 5 - - - - - - - - - - 3 2
20. Emma Irmscher Eibenberg 5 - - - - - 3 - - - 2 - -
20. Theresa J Chemnitz 5 - - - - 1 - - - 4 - - -
20. Tim Sigmund Chemnitz 5 - 1 - - - 4 - - - - - -
21. Friederike Lenk Chemnitz 4 - - - - - - - - - 4 - -
21. Josephine Pallus Chemnitz 4 - - - - - - - - 4 - - -
21. Jana (+S) Schneider Lugau 4 - - - - - 4 - - - - - -
21. Elias Schmidt Chemnitz 4 - - - - - 2 - - - - - 2
21. Luisa Schlosser Chemnitz 4 - - 2 - - - - 2 - - - -
22. Ida Gwendolin Eichler Chemnitz 3 - - - - - 3 - - - - - -
22. Viola (KATI-Team) Berlin 3 - - - - - 3 - - - - - -
22. Daniel Hufenbach Chemnitz 3 - - - - - - - - - - 3 -
23. Luise Adam Chemnitz 2 - - - - 2 - - - - - - -
23. Julia Voigt Chemnitz 2 - - - - 2 - - - - - - -
23. Jonas Frederik Otto Lichtenwalde 2 - - - - 2 - - - - - - -
23. Marion Sarah Zenk Chemnitz 2 - - - - 2 - - - - - - -
23. Lisanne Brinkel Chemnitz 2 - - - - - - - - - - 2 -
23. Malte Gebhardt Chemnitz 2 - 2 - - - - - - - - - -
23. Erik Walther Chemnitz 2 - - - - 2 - - - - - - -
23. Laura Schlosser Chemnitz 2 - - - - - - - - - 2 - -
23. Emilie Grossinger Chemnitz 2 - 2 - - - - - - - - - -
23. Ernesto Uhlmann Chemnitz 2 - 2 - - - - - - - - - -
23. Henrike Grundmann Chemnitz 2 - - - - 2 - - - - - - -
23. Marcel Seerig Chemnitz 2 - - - - 2 - - - - - - -
24. Johannes Allert Chemnitz 1 - 1 - - - - - - - - - -
24. Anne Haag Chemnitz 1 - 1 - - - - - - - - - -
24. Tim Jechorek Chemnitz 1 - - - - - 1 - - - - - -
24. Amerin Ro Chemnitz 1 - - - - 1 - - - - - - -
24. Pascal Graupner Chemnitz 1 - - - - - 1 - - - - - -

 

Auswertung Serie 24 (rote Liste)

Platz Name Ort Summe Aufgabe

277 278 279 280 281 282 283 284 285 286 287 288
1. Elisa Parsche Chemnitz 62 6 1 3 12 6 4 2 6 6 9 2 5
2. Doreen Naumann Duisburg 52 6 1 3 6 5 4 2 3 6 9 2 5
3. Sabine Fischbach Hessen 41 6 1 - 6 1 4 2 - 6 9 2 4
4. XXX ??? 30 - 3 - 12 7 3 - - - - - 5
5. Mawi Dresden 22 - - - - - - - - 6 9 2 5
6. Paula Hartmannsdorf 18 - 3 3 - 7 - - - - - - 5
7. Dr. Frank Göring Chemnitz 15 - - 3 12 - - - - - - - -
7. Astrid Fischer Chemnitz 15 - - - - - - - - - 9 2 4
8. Rafael Seidel Chemnitz 12 - - - - 2 - 2 - 6 - 2 -
9. Andree Dammann M 10 - - - - - - - - 3 - 2 5
10. Karolin Schuricht Chemnitz 9 - - - - - 3 - - 6 - - -
11. Annika Theumer Chemnitz 8 - - - 8 - - - - - - - -
12. Jamila Wähner Chemnitz 7 - - - - - - - 3 - - - 4
12. Lisa Grassmann Chemnitz 7 - - - - 3 - - - - - - 4
12. Marion Sarah Zenk Chemnitz 7 - - - - 7 - - - - - - -
13. Samuel Kilian Stuttgart 6 6 - - - - - - - - - - -
13. Ellen Richter Chemnitz 6 - - - - - - 3 - 3 - - -
13. Felix Taubert Chemnitz 6 - - - - 4 - - 2 - - - -
13. Stephanie Dani Chemnitz 6 6 - - - - - - - - - - -
13. Pauline Marschk Chemnitz 6 6 - - - - - - - - - - -
13. Christian Wagner Bamberg 6 6 - - - - - - - - - - -
14. Anja Posselt Chemnitz 5 - - - - - - - 3 - - 2 -
15. Jana (+S) Schneider Lugau 4 - - - - - 4 - - - - - -
15. Elias Schmidt Chemnitz 4 - - - - - - - - - - - 4
16. Richard Hahmann Chemnitz 3 - - - - - - - - - - - 3
16. Josephine Pallus Chemnitz 3 - - - - - - - - 3 - - -
16. Julia Voigt Chemnitz 3 - - - - 3 - - - - - - -
17. Kai-Lutz Wagner Chemnitz 2 - - 2 - - - - - - - - -
17. Daniel Hufenbach Chemnitz 2 - - - - - - - - - - 2 -
17. Nina Zätsch Chemnitz 2 - - - - - - - - - - 2 -
17. Lucas Steinke Chemnitz 2 - - 2 - - - - - - - - -
17. Marie Sophie Roß Chemnitz 2 - - - - - - 2 - - - - -
17. Loise Reichmann Chemnitz 2 - - - - - - 2 - - - - -
18. Luis Raupach Chemnitz 1 - - - - - - - 1 - - - -
18. Pit Hopke Chemnitz 1 - - - - 1 - - - - - - -

 


Serie-23

Serie 23
Aufgaben und Lösungen

Aufgabe 1

265. Wertungsaufgabe

Quadrate"Es gibt doch sicher noch viel mehr Mathematisches bei den Spielen zu finden", meinte Mike. "Aber bestimmt, da wäre ja noch die sogenannte Spieltheorie (manchmal klang die natürlich durch) selbst, Überlegungen bei Computerspielen und so weiter. Nun aber muss ich mich erst mal mit dieser Aufgabe beschäftigen". "Zeig mal". "Auf dem Bild siehst du die zwei Quadrate ABCD und BEFG. Das rechts liegende soll für die Überlegungen der roten Aufgabenstellung immer kleiner sein als das andere. Auf der Strecke AE ist ein Punkt X zu finden. Der Punkt X soll mit F bzw. mit D verbunden werden.  Der Punkt X soll folgende Eigenschaften haben: Wird die Figur entlang der Linien DX und XF geteilt, so lassen sich die Teile zu einem Quadrat zusammenlegen.   (6 rote Punkte)
Das geht? , fragte Lisa etwas zweifelnd, die sich die Figur betrachtete.  Aber ja doch und deine Spezialistengruppe könnte sich ja mal eine Formel überlegen, wie man den Umfang der Figur elegant ausrechnet, wenn die Längen der Seiten der Quadrate a bzw. b sein sollen (a>b). 2 blaue Punkte.

Lösung

blau: Ich gehe mal von C im mathematisch positiven Sinn um die Figur:
a + a + a + b + b + b +(a - b) = a + a + a + b + b + b + a - b = 4a + 2b
rot:
Der Punkte X muss b cm von A entfernt sein - Die Aufgabe zum Spielen kann man sich in der Mathelandausstellung in Dresden anschauen.
Die komplette Lösung von Stefan G. aus Dresden, danke.
Die Seitenlänge des Quadrates ABCD sei a und die Seitenlänge des Quadrates BEFG sei b.
O.b.d.A. sei a>b. (Spiegele an BC falls b>a und benenne um.)
Durch die Schnitte entsteht stets ein Fünfeck mit konkavem, rechtem Winkel CGF. In diesen muss zur Konstruktion eines Quadrates eines der entstehenden Dreiecke mit rechtem Winkel DAB bzw. BEF eingfügt werden. Dabei muss eine Kathete die Länge b aufweisen und der Winkel den sie mit der Hypothenuse einschließt muss sich mit dem Winkel XFG zu neunzig Grad ergänzen.
Da der Winkel XDC kleiner als neunzig Grad ist, muss auch er durch einen weiteren Winkel ergänzt werden, d.h. das zweite Dreieck muss auf der Seite DC angebracht werden. Ist die Kathete auf DC kleiner als a bzw. die Kathete auf FG kleiner als b, so verbleibt eine Kante an C, die nicht mehr aufgefüllt werden kann. Für längere Katheten auf diesen Seiten, entstehen nicht zu kompensierende Spitzen.
Um ein Quadrat zu ergeben müssen die Katheten, die auf der Geraden BC liegen so lang sein, dass sie einen gemeinsamen Eckpunkt des neuen Quadrates bilden. Da die Dreiecksseiten alle größer als Null sind (X=A bzw. X=E ergeben nur ein echtes Dreieck, dass nicht zur Vervollständigung zu einem Quadrat taugt, bspw.da die Strecken DX und XF nicht gleich lang sind) muss die Kathete des Dreiecks im Winkel CGF, die die Strecke CG enthält größer als b sein und also über den Punkt C hinausragen. Zugleich müssen sich die Winkel im neu gebildeten Quadratpunkt zu neunzig Grad ergänzen. Damit haben die beiden Dreiecke neben dem rechten Winkel auch einen weiteren Winkel gemein, da sich die neunzig Grad jeweils mit dem Winkel der sich im anderen Dreieck der gemeinsamen Quadratspitze findet zu hundertachtzig Grad ergänzen. Damit sind die beiden Dreiecke ähnlich.
Außerdem bilden XF und XD Seiten des neuen Quadrates und müssen daher gleich lang sein. Dies sind aber die Hypothenusen der beiden Dreiecke, die folglich kongruent sind.
Daraus folgt, dass AX = b und BX = a.
Selbiges lässt sich alternativ auch ohne Betrachtung der Winkel zeigen. Dafür sei zunächst AX = q und BX = p. Durch die Gleichheit der Hypothenusen, sprich XF = XD, gilt dann (1) b^2+p^2 = a^2 + q^2
Zudem muss für die aus den Dreiecken gebildete Spitze des Quadrates gelten, dass die eine Kathete gleich der anderen Kathetenlänge zuzüglich der Differenz aus a und b, sprich CG, ist, also (2) p = q + (a-b)
Die Lösung des Gleichungssystems führt auf p = a und q = b und damit ebenfalls auf die oben angegebene Lösung.
Hier noch ein Bild von Jana & S. aus Lugau, danke.
Lösung 265



Aufgabe 2

266. Wertungsaufgabe

"Heute haben wir die Grundkonstruktionen wiederholt und sind dabei auf die Aufgabe gestoßen, wie groß wohl der größte Kreis ist, der in ein gleichseitiges Dreieck von 1 m Kantenlänge  hineinpasst.", berichtete Lisa. (3 blaue Punkte für eine passende konstruktive Lösung, 3 noch dazu, wenn die Formel zur Berechnung hergeleitet wird.)
"Nun das ist ja nicht so schwierig", meinte Mike. "Wie wäre es denn damit? Wie groß ist der größte Halbkreis, der in das 1 m große gleichseitige Dreieck hineinpasst?" - 5 rote Punkte

Lösung

Hier noch ein passendes Bild von Jana & S. aus Lugau, danke.
Lösung 265 blau: Der größte Kreis, der in ein Dreieck, hinein passt ist der Inkreis. Ein konstruktive Lösung geht natürlich mittels einer maßstabsgerechten Zeichnung.
Rechnerisch: Mit den Mitteln der Klasse 9: tan α = r/(a/2) mit α = 30° a/2 = 0,5 m
Damit ergibt sich r =0,2886751 m, der Flächeninhalt mit A = πr² = 0,2617994 m² = 0,26 m²
rot: Der Halbkreis liegt auf einer der Seite, die beiden anderen Seiten sind Tangenten.
Wieder mit den Mitteln der Klasse 9 ergibt sich: sin α = r/(a/2) mit α = 60° a/2 = 0,5 m (Achtung es sind nicht die selben Winkel gemeint.)
Damit ergibt sich r = 0,4330127 m, der Flächeninhalt mit A = πr²/2 = 0,2945243 m² = 0,29 m².
So mit ist erstaunlicherweise der Halbkreis größer als der Kreis - Radius so wie so, aber auch vom Flächeninhalt her.



Aufgabe 3

267. Wertungsaufgabe

"Das war ja eine geniale Schnippelei bei der vorletzten Aufgabe. Ich hatte schon so eine Ahnung, dass dies mit dem Satz des Pythagoras zu tun hat, wegen der Quadrate und so.", meinte Maria, die gerade die Ergebnisse der Spezialistengruppe präsentierte. "Apropos Pythagoras, wir hatten mal so eine Aufgabe bekommen, dass man mit einem der Sätze aus der Satzgruppe des Pythagoras eine Strecke mit der Länge von Wurzel aus 3 konstruieren konnte." "Da hast du Recht Lisa, ich kann mich erinnern." "Na siehst du und nun sollen wir sogar die Strecke konstruieren, die gleich der 4. Wurzel aus 3 entspricht, da stehe ich noch auf dem Schlauch." - 4 rote Punkte für die Konstruktionsbeschreibung (mit Begründung).
"In unserer Gruppe haben wir neulich Quadrate untersucht. Leicht war zu sehen, dass die Diagonalen immer länger waren als die Quadratseiten. Da taucht die Frage auf, ob es Rhomben gäbe, deren eine oder gar beide Diagonalen so lang wären, wie die Seiten."(3 + 2 blaue Punkte)

Lösung

blau: Ich entscheide mich dafür, mit der Untersuchung der Diagonale BC zu beginnen. Die Seiten AB und AD (= a) sind gleich lang, also ist das Dreieck ABD gleichschenklig. Wird die Länge von BC gleich a gewählt, so ist ein Rhombus gefunden, dessen eine Diagonale so lang ist wie die Seiten. Es gibt aber zugleich auch nur dieses eine Rhombus mit BC = a. Dessen zweite Diagonale (e) ist dann automatisch länger. Dies lässt sich mittels Kosinussatz oder der Formel für die Höhe im gleichseitigen Dreieck (= Hälfte der anderen Diagonale e).
e = a² · √ 3
Wird mit der andren Diagoanle begonnen gilt das oben geschriebene analog.
rot: sehr ausführliche Lösung von Samuel Kilian, danke
Man nehme eine Strecke SB der Länge 1 und verlängere diese. Mit dem Zirkel kann man die Strecke noch 3 mal um die Länge 1 verlängern.
Man hat dann eine Strecke AB mit der Länge 4, die geviertelt ist. Der Punkt S liegt auf dieser Strecke, 1 von B entfernt und 3 von A. Dann zeichne man einen Halbkreis mit dem Mittelpunkt M in der Mitte der Strecke AB mit dem Radius der Hälfte der Länge der Strecke AB. Der Radius ist dann 2.
Nun konstruiert man eine Senkrechte zu AB, die AB im Punkt S schneidet. Der Schnittpunkt der Senkrechte mit dem Halbkreis ist der Punkt C. Nach dem Satz des Thales liegt jetzt ein rechtwinkliges Dreieck mit c=4, p=1 und q=3 vor.
Die Strecke SC ist die Höhe und hat die Länge Wurzel aus 3, da h*h=p*q=1*3 und somit h*h=3 (Höhensatz des Euklid).
Jetzt greift man mit dem Zirkel die Strecke SB ab, diese hat die Länge 1. Man kann somit auf der Strecke SC den Punkt P ermitteln. Die Strecke SP hat die Länge 1. Jetzt konstruiert man eine Senkrechte zu SC, die SC in P schneidet. Außerdem ermittelt man den Mittelpunkt der Strecke SC und zeichnet mit diesem Punkt als Mittelpunkt einen Halbkreis mit dem Radius (SC)/2. Der Schnittpunkt dieses Kreis mit der Senkrechten ergibt den Punkt Q, der nach dem Satz des Thales ein rechtwinkliges Dreieck mit S und C bildet mit den Längen von (SC) = c' =√(3), p'=1 und q'=√(3)-1.
Da nach dem Kathetensatz des Euklid gilt: a*a=p*c, also hier a' * a' = p' + c' = 1*√(3) = √(3), ist a= "vierte Wurzel aus 3".
Anmerkungen von Thomas: 1. Es ist auch möglich nur mit dem Höhensatz oder dem Kathetensatz zu arbeiten. 2. Diese Konstruktion wird u.a. genutzt, um Teilung eines gleichseitigen Dreiecks zu ermöglichen, aus dessen Teilen sich dann ein Quadrat zusammensetzen lässt. siehe hier



Aufgabe 4

268. Wertungsaufgabe

„Die letzte rote Aufgabe führt letztlich auf das Dudeney Puzzle, das fand ich verblüffend“, gab Bernds Opa verwundert zu, als er die Aufgabe mit den vier Experten durchgegangen war.  „Denkt aber auch daran, dass die Mitglieder eurer Mathematikgruppe mal etwas körperliche Bewegung gebrauchen können. Wie viele sind denn in eurer Gruppe?“ „Das wechselt immer mal, aber so zwischen 8 und 12 sind es schon“, sagte Lisa. „Alle klar. Also, wenn es z.B. 8 sind, dann sollen die sich (durchnummeriert) in einem Kreis aufstellen und im Uhrzeigersinn sich nacheinander immer einen Ball zuwerfen. Ein paar Runden, so dass immer der nächste den Ball bekommt. Dann wieder Start bei Nummer 1, der Ballwurf so, dass jeder zweite den Ball bekommt. Neues Spiel - nun ist es jeder dritte, na und so weiter bis zur sieben.“ „Verstanden, aber da gibt es doch sicher auch Runden, wo mal einige (oder mindestens einer) den Ball gar nicht bekommen.“ „Das stimmt schon, aber Spaß macht es trotzdem.“ Es gibt so viele blaue Punkte wie es Runden gibt, wo jeder Teilnehmer mal den Ball bekommt. Bei welcher der möglichen Teilnehmerzahlen von 8 bis 12 passiert es nicht, dass, egal wie die Wurfregel lautet, mal irgendwelche Leute in der Runde den Ball nicht bekommen können. Bei welcher Art von Teilnehmerzahlen tritt dieses „Alle machen immer mit“-Phänomen auf? 4 rote Punkte. (Anmerkung: Jede mögliche Spielsituation kann beliebig lange dauern, mindestens so lange bis klar ist, ob die Teilnehmer den Ball bekommen können oder eben auch nicht.)

Lösung

blau: hier kann man die Varianten durchaus schnell notieren:
Anzahl der Teilnehmer t ist 8
Wurfmodus 1: 1 - 2 - 3- 4- -5 -6 - 7 - 8 - fertig jeder hatte mal den Ball
Wurfmodus 2: 1 - 3 - 5 - 7 - 1 - 3 - ... es ist leicht zu sehen, dass die Spieler mit den geraden Nummern den Ball nicht bekommen.
Wurfmodus 3: 1 - 4 - 7 - 2 - 5 - 8 - 3 - 6 - 1 - fertig jeder hatte mal den Ball
Wurfmodus 4: 1 - 5 - 1 - 5 - ... es ist leicht zu sehen, dass nur zwei Spieler den Ball bekommen.
Wurfmodus 5: 1 - 6 - 3 - 8 - 5 - 2 - 7 - 4 - 1 - fertig jeder hatte mal den Ball
Wurfmodus 6: 1 - 7 - 5 - 3 - 1 - ... es ist leicht zu sehen, dass die Spieler mit den geraden Nummern den Ball nicht bekommen.
Wurfmodus 7: 1 - 8 - 7 - 6 - 5 - 4 - 3 - 2 - 1 - fertig jeder hatte mal den Ball
Es sind also 4 Wurfmodi, bei denen jeder Teilnehmer den Ball bekommt. Also gibt es maximal 4 blaue Punkte
Anmerkungen:
Es ist aber noch mehr erkennbar. Wurfmodi (wm), die sich zu 8 addieren sind, entsprechen einander, nur das die Fängerfolge umgekehrt ist.
Sind t und wm teilerfremd sind alle beteiligt. Die Zahl der Beteiligten (b) lässt sich als b = ggT (t;wm) beschreiben - ggT - größter gemeinsamer Teiler
rot: Mit den obigen Anmerkungen ist die Frage schon fast beantwortet. Bei 11 Teilnehmern gilt für alle Wurfmodi ggT (11; wm) = 1. Dies gilt entsprechend für alle Teilnehmerzahlen, wenn diese eine Primzahl sind.
Den zugrunde liegenden zahlentheoretischen Bezug üerlasse ich dem geneigten Leser.



Aufgabe 5

269. Wertungsaufgabe

"Immer wieder muss man Quadrieren und Wurzelziehen und das schon sehr lange im Verlaufe der Mathematikgeschichte", sagte Bernds Opa, nachdem er sich die letzten Aufgaben angeschaut hatte. "Das Quadrieren ist ja recht einfach. Zahl a mal Zahl a und fertig, aber wenn man das gemacht hat, ist die nächste Quadratzahl - also das Quadrat von (a+1) sogar ohne Multiplikation ermittelbar. Das bekommen (für 3 blaue Punkte) sicher auch die Mitglieder eures Mathezirkels heraus."
"Na Opa, so wie du guckst, hast du doch sicher auch einen Tipp für das Wurzelziehen", feixte Bernd. "Aber klar doch, zumindest eine gute Näherungslösung. Ich betrachte mal den Bereich bis zur 1000. Die Quadratzahlen zwischen 100 und 1000 kennt ihr doch sicherlich." "Die haben wir mal gelernt," sagte Maria, "aber  für die kennen wir ja dann auch die Wurzel schon, interessanter wäre ja dann beispielsweise die Wurzel aus 230." "Na selbstverständlich.
Dann nehme ich doch gleich mal die 230. Die nächstkleinere Quadratzahl ist 225. Die Wurzel daraus ist 15. Also ist Wurzel 230 = 15,???. Für die Stellen nach dem Komma rechne ich nun die Differenz von 230 bis 225 aus, also 5. Dieses Ergebnis teile ich durch das Doppelte von 15. Dieses neue Ergebnis ersetzt die Fragezeichen." "Das probiere ich mal aus," meinte Mike. "Cool, Wurzel aus 230 mit dem Taschenrechner ist 15,16575... und die Näherung ergibt 15,1666... Damit unterscheiden sich die Ergebnisse nur um etwa 1/1000." Warum funktioniert das Verfahren so gut und wird es für Zahlen über 1000 eher besser oder eher schlechter? - 4 rote Punkte.

Lösung

blau: ganz ohne Formel:

Zahl nächste Zahl Quadratzahl nächste Quadratzahl Differenz
10 11 100 121 21
11 12 121 144 23
12 13 144 169 25
... ... ... ... ...
100 101 10 000 10 201 201
Es ist leicht zu sehen, dass sich die nächste Quadratzahl dadurch ergibt, dass zur Quadratzahl das Doppelte der Basis und noch 1 addiert wird.
12² = 144 ⇒ 13² = 144 + 2*12 + 1 = 169
mit Formel:
(a+1)²= a² + 2*a +1 (binomische Formel)
rot: Lösung Samuel Kilian, danke.
Anmerkung: SQR - Quadrat einer Zahl, SQRT - Quadratwurzel aus einer Zahl
Näherung der Wurzel aus x:
Der Näherungswert für die Wurzel aus x heißt im Folgenden f(x) . Die in der Aufgabe beschriebene Näherung für die Wurzel aus x sieht aus wie folgt:
f(x) = n + ( x-SQR(n) ) / (2n) wobei SQR(n) die nächstkleinere Quadratzahl ist.
Um als Mensch die Näherung zu nutzen, muss n eine natürliche Zahl sein. Für den Beweis spielt es allerdings keine Rolle, ob n natürlich oder nur reell ist.
Ich nenne die Differenz zwischen SQR(n) und x im Folgenden e, so dass gilt SQR(n) =x - e und für die Näherung entsprechend f(x) = n + e / (2n).
e liegt zwischen 0 und 2n+1, ich betrachte also die Abweichung der Näherung vom tatsächlichen Wert für diese beiden Fälle. Ist e=0, so ist SQR(n)=x, so dass sich für die Näherung ergibt: f (x) = n + 0 / 2n = n = SQRT(x) . Wie zu erwarten war, stimmt die Näherung hier genau.
Ist e=2n+1, dann ist x=SQR(n+1) und n=SQRT(x) -1 , so dass die Näherung f(x)= SQRT(x) -1 + (x- SQR(SQRT(x) -1) ) / ( 2*(SQRT(x) -1) ) .
Will man die relative Abweichung A dieser Näherung für den echten Wert berechnen, muss man A = ( f(x) - SQRT(x) ) / SQRT(x) bilden.
Nach ein wenig Umformung ergibt sich dadurch für A: A = 1 / ( SQRT (x) -1) - 1 / SQRT(x) - 1 / (2*SQRT(x)*(SQRT(x) -1) )
Für einigermaßen große Werte von x strebt 1/(SQRT(x)-1) - 1/SQRT (x) von oben gegen 0 und -1/(2*SQRT(x)*(SQRT(x)-1) strebt von unten gegen 0.
Die Abweichung ist also sehr gering, schon für x=100 ist A = 1/180 = 0,556 % und das im Fall der größten Abweichung. Diese wird für große Werte von x sehr klein, für 1024 ist sie beispielsweise A = 1 / 1984 = 0,0504 % .
1. Durch oben gezeigte geringe Abweichung funktioniert die Näherung so gut.
2. Für große Werte (also Werte über 1000) funktioniert die Näherung besser als für kleine (x ≤000).
Das Bild zeigt die sich ergebenden Abweichungen:
269

Aufgabe 6

270. Wertungsaufgabe

„Wir hatten doch vor kurzem die Umkreiskonstruktion bei gleichseitigen Dreiecken mit unserer Mathematikgruppe besprochen, erinnert ihr euch?“, fragte Maria. „Aber klar.“ „Jetzt hat einer unserer Spezialisten festgestellt, dass bei seiner Konstruktion der Umkreisradius und der  Inkreisradius in einem sehr einfachen Verhältnis stehen.“  (2 blaue Punkte für ein konstruktiv gelöstes Beispiel oder 4 blaue Punkte für einen  Beweis.)
„Nun hat ja bekanntlich jedes Dreieck einen In- und einen Umkreis, deren Radien in einem Verhältnis stehen. ?? Mit dem Verhältnis der blauen Aufgabe habe ich es mit rechtwinkligen Dreiecken versucht??, doch das hat erwartungsgemäß nicht geklappt. Aber ich habe rechtwinklige Dreiecke gefunden, wo das Verhältnis von ru zu ri genau 2,5 war.“,  berichtete Bernd.  In welchem Verhältnis müssen die Katheten a und b stehen (a < b), damit das ru zu ri -Verhältnis genau 2,5 beträgt? - 6 rote Punkte

Lösung

Aufgabe 270 In dem Dreieck sind Um- und Inkreis eingetragen. Das es sich um ein gleichseitiges Dreieck handelt, sind die „besonderen Linien“ identisch. - die Höhen, Mittelsenkrechten, Winkelhalbierenden und die Seitenhalbierenden. Alle schneiden sich in einem Punkt, dem Mittelpunkt der beiden Kreise. Dieser Punkt teilt die Linien im Verhältnis 2 zu 1 - als Eigenschaft des Schnittpunktes der Seitenhalbierenden in jedem Dreieck. Damit ist der Umkreisradius genau doppelt so groß wie der Inkreisradius.
Es gab auch spannende Überlegungen, die auf dem Satz des Pythagoras beruhten bzw. die Eigenschaft sin 30° = 0,5 nutzten.
rot: a,b Katheten, c- Hypotenuse, A - Flächeninhalt
Eigenschaft 1: ru = c/2
Eigenschaft 2: A = ab/2
allgemeine Eigenschaft von Dreiecken: ru = abc/4A (hier steckt Eigenschaft 1 drin) und ri = 2A/(a + b + c)
Es soll ru =2,5 ri sein. ∗
c/2 = 2,5 · ab/(a + b + c)
c = 5 · ab/(a + b + c), und c = √ a² + b² liefert mit einer recht komplizierten Umformungen das gewünschte Ergebnis.
Ein Einsetzen des bekannten 3 - 4 - 5 Dreiecks ergibt, dass dieses die Bedingungen erfüllt, so lässt dann mittels genauer Betrachtungen zur Ähnlichkeit zeigen, dass das Verhältnis a:b=3:4 das Gesuchte ist.
Aufgabe 270 Teile 2Ein Weg, den auch die Lösung von Samuel K aufzeigte, sei noch vorgestellt.
Die Dreiecke AZM und AMY bzw. die die Dreiecke BMZ und BXM sind nach dem Kongruenzsatz sSw jeweils kongruent zueinander. Es sind aus dem gleichen Grund die Dreiecke MCY und MXC kongruent zueinander. Damit ist MXCY ein Quadrat mit der Kantenlänge r (=ri). Weiterhin folgt: b1 und c1 sind gleichlang und c2 und a1 sind ebenfalls gleich lang.
r = b - b1 (mit b1 = c -c2) ⇒
r = b - (c - c2) (mit c2 =a1 = a - r) ⇒
r = b - (c - (a - r)) ⇒
r = b - c + a -r ⇒
2r = b - c + a ⇒ r = 0,5(b - c + a)
Wegen ∗ ⇒ r/ru= 1/2,5 = 0,4 = (0,5(b - c + a))/(0,5c) = (b - c + a)/c = (a +b )/c -1 ⇒
0,4 = (a +b )/c -1 | + 1 ⇒
1,4 = (a +b )/c | · c ⇒
1,4c = a + b mit c = √ a² + b² ⇒
1,4√ a² + b² = a + b |² ⇒
1,96 a² + 1,96b² = a² + 2ab + b² | -(a² + 2ab + b²) ⇒
0,96 a² - 2ab + 0,96 b² = 0 | : (96/100) ⇒
a² - 50/24 ab + b² = 0 | Lösungsformel für quadratische Gleichungen ⇒
a1,2 = 25/24 b ± √(625/576)b² - b²
a1,2 = 25/24 b ± √(49/576)b²
a1,2 = 25/24 b ± 7/24 b ⇒
a1 = 18/24 b = 3/4 b und a2 = 32/24 b = 4/3 b ⇒
a und b stehen im Verhältnis 3 : 4 bzw. 4 : 3.



Aufgabe 7

271. Wertungsaufgabe

„Schaut mal, was ich gefunden habe, ein Bild vom Weihnachtsmanntreffen im letzten Jahr“, strahlte Maria. „Hier seht ihr sie alle nebeneinander in der Reihe stehen.“ „Ich kann sie gar nicht so genau erkennen mit dem Kostüm und dem Bart“, meinte Bernd.
„Nun die Namen, warte mal ach ja. Es waren Ines, Andrea, Carla, Marla, Fred, Peter, Tom und Eric dabei.“ „Stimmt genau“. „Hier noch ein paar Tipps:“
Tom steht zwischen Carla und Peter.
Eric steht nicht neben Andrea.
Marla steht zwischen Eric und Fred.
Zwischen Carla und Ines stehen zwei Personen.
Andrea steht rechts neben Carla.
Wer steht wo? - 4 blaue Punkte - Es könnte sein, dass es sogar noch Zusatzpunkte gibt.
„Und was ist auf dem anderen Foto zu sehen?“, fragte Mike. „Nun das ist ein Foto von den Fotobüchern der letzten Urlaubsfahrten, pro Jahr eins. Ihr seht, da stehen die 6 Bücher nebeneinander (links die 1 und rechts die 6). Oh, die stehen ja etwas durcheinander, aber Ihr bekommt sicher heraus, an welchem Platz welches Buch steht - Urlaubsjahr (2003 - 2008, Urlaubsziel und Tage des Aufenthaltes vor Ort (14, 15, 16, 17, 18, bzw. 19).“ 8 rote Punkte
Das Buch über Polen steht an der Stelle 4 und ist nicht von 2005
Ganz links steht wirklich das Buch von 2003, das Buch an der Stelle zwei umfasst 15 Urlaubstage.
2006 waren wir in Italien.
Das Norwegenbuch steht zwischen dem Buch aus dem Jahr 2008 und dem 18-Tage-Urlaub.
Neben Norwegen steht nicht Deutschland.
2007 waren wir 14 Tage vor Ort, das Buch dazu steht nicht ganz rechts.
Das Buch über Deutschland ist direkt rechts neben dem 17-Tage-Urlaub.
Der 16Tage-Urlaub in Frankreich steht direkt neben dem Spanienbuch.
8 rote Punkte

Lösung

blau: die Beschreibung lässt mehrere Lösungen zu. Dies trifft sowohl für die Überlegung zu dem Begriff rechts - aus der Sicht des Betrachters oder der Personen auf dem Bild, als auch auf die Betrachtungen selbst. Die Aussage - Zwischen Carla und Ines stehen zwei Personen - ist im streng mathematischen Sinne als mindestens zwei Personen auffassbar. - Die Aufgabenstellung impliziert eher die Variante genau 2. Rechts neben ist meiner Auffassung nach ein Hinweis auf direkter Nachbar, anders als bei rechts von oder nur rechts.
Es waren Andrea, Carla, Eric, Fred, Ines, Marla, Peter, Tom . - alphabetisch geordnet.
Tom steht zwischen Carla und Peter.
1. Tom steht zwischen Carla und Peter.
2. Eric steht nicht neben Andrea.
3. Marla steht zwischen Eric und Fred.
4. Zwischen Carla und Ines stehen zwei Personen.
5. Andrea steht rechts neben Carla.
⇒ a. Carla-Tom-Peter oder Peter-Tom-Carla (1.)
⇒ b. Andrea-Carla-Tom-Peter (aus der Sicht der Personen) oder Peter-Tom-Carla-Andrea (aus der Sicht des Betrachters)(5.)
⇒ c. Eric-Marla-Fred oder Fred-Marla-Eric (3.) ⇒ d. die Dreiergruppe aus c. passt nur vor oder eben nach die Vierergruppe aus b (es gibt keine Überschneidungen der Namen)
⇒ e. Es gibt zwei Vierer-Konstellationen(b), dazu zwei Dreier-Konstellationen(c) und Einzeln die Ines
⇒ Es gibt die Möglichkeiten: 431; 413; 341; 314; 143 und 134, wo bei 3 und 4 jeweils dopplet belegt sind.
⇒e1: 431
Andrea-Carla-Tom-Peter+Fred-Marla-Eric+Ines - (nur) streng mathematisch richtig (4.), 2. erfüllt.
Andrea-Carla-Tom-Peter+Eric-Marla-Fred+Ines - (nur) streng mathematisch richtig (4.), 2. erfüllt.
Peter-Tom-Carla-Andrea+Fred-Marla-Eric+Ines - (nur) streng mathematisch richtig (4.), 2. erfüllt.
Peter-Tom-Carla-Andrea+Eric-Marla-Fred+Ines - geht nicht - 2. nicht erfüllt.
⇒e2: 413
Andrea-Carla-Tom-Peter+Ines+Fred-Marla-Eric - 4. im Sinne von genau 2 Personen und 2. erfüllt.
Andrea-Carla-Tom-Peter+Ines+Eric-Marla-Fred - 4. im Sinne von genau 2 Personen und 2. erfüllt.
Peter-Tom-Carla-Andrea+Ines+Fred-Marla-Eric - geht nicht - 4. nicht erfüllt.
Peter-Tom-Carla-Andrea+Ines+Eric-Marla-Fred - geht nicht - 4. nicht erfüllt.
⇒e3: 341
Fred-Marla-Eric+Andrea-Carla-Tom-Peter+Ines - geht nicht - 2. nicht erfüllt.
Fred-Marla-Eric+Peter-Tom-Carla-Andrea+Ines - geht nicht - 4. nicht erfüllt.
Eric-Marla-Fred+Andrea-Carla-Tom-Peter+Ines - 4. im Sinne von genau 2 Personen und 2. erfüllt.
Eric-Marla-Fred+Peter-Tom-Carla-Andrea+Ines - geht nicht - 4. nicht erfüllt.
⇒e4: 314
Fred-Marla-Eric+Ines+Andrea-Carla-Tom-Peter - geht nicht - 4. nicht erfüllt.
Fred-Marla-Eric+Ines+Peter-Tom-Carla-Andrea - 4. im Sinne von genau 2 Personen und 2. erfüllt.
Eric-Marla-Fred+Ines+Andrea-Carla-Tom-Peter - geht nicht - 4. nicht erfüllt.
Eric-Marla-Fred+Ines+Peter-Tom-Carla-Andrea - 4. im Sinne von genau 2 Personen und 2. erfüllt.
⇒e5: 143
Ines+Andrea-Carla-Tom-Peter+Fred-Marla-Eric - geht nicht - 4. nicht erfüllt.
Ines+Andrea-Carla-Tom-Peter+Eric-Marla-Fred - geht nicht - 4. nicht erfüllt.
Ines+Peter-Tom-Carla-Andrea+Fred-Marla-Eric - 4. im Sinne von genau 2 Personen und 2. erfüllt.
Ines+Peter-Tom-Carla-Andrea+Eric-Marla-Fred - geht nicht - 2. nicht erfüllt. ⇒e6: 134
Ines+Fred-Marla-Eric+Andrea-Carla-Tom-Peter - geht nicht - 2. nicht erfüllt. Ines+Fred-Marla-Eric+Peter-Tom-Carla-Andrea - (nur) streng mathematisch richtig (4.), 2. erfüllt.
Ines+Eric-Marla-Fred+Andrea-Carla-Tom-Peter - (nur) streng mathematisch richtig (4.), 2. erfüllt.
Ines+Eric-Marla-Fred+Peter-Tom-Carla-Andrea - (nur) streng mathematisch richtig (4.), 2. erfüllt.

Bild von Josephine Pallus, danke
271 große Ansicht
rot: nach der sehr ausführlichen Lösung von blau, hier erst mal nur kurz die Variante rot:
von links nach rechts:
Frankreich 16 Seiten 2003
Spanien 15 Seiten 2008
Norwegen 14 Seiten 2007
Polen 18 Seiten 2004
Italien 17 Seiten 2006
Deutschland 19 Seiten 2005
Aber nach meiner Schreiberei, kam noch die Lösung von Sabine, danke:
270 - Rote Aufgabe
„Und was ist auf dem anderen Foto zu sehen?“, fragte Mike. „Nun das ist ein Foto von den Fotobüchern der letzten Urlaubsfahrten, pro Jahr eins.
Ihr seht, da stehen die 6 Bücher nebeneinander (links die 1 und rechts die 6). Oh, die stehen ja etwas durcheinander, aber Ihr bekommt sicher heraus, an welchem Platz welches Buch steht“ - Urlaubsjahr (2003 - 2008, Urlaubsziel und Tage des Aufenthaltes vor Ort (14, 15, 16, 17, 18, bzw. 19). 8 rote Punkte
Aussagen:
1)Das Buch über Polen steht an der Stelle 4 und ist nicht von 2005 .
2)Ganz links steht wirklich das Buch von 2003.
3)Das Buch an der Stelle zwei umfasst 15 Urlaubstage.
4)2006 waren wir in Italien.
5)Das Norwegenbuch steht zwischen dem Buch aus dem Jahr 2008 und dem 18-Tage-Urlaub.
6)Neben Norwegen steht nicht Deutschland.
7)2007 waren wir 14 Tage vor Ort, das Buch dazu steht nicht ganz rechts.
8)Das Buch über Deutschland ist direkt rechts neben dem 17-Tage-Urlaub.
9)Der 16Tage-Urlaub in Frankreich steht direkt neben dem Spanienbuch.
Zunächst trage ich die Aussagen in meine Tabelle (+) ein.
(Anmerkung Thomas Tabelle analog Logiktrainer)
Es ergibt sich weiter:
Platz 1 (ganz links): 2003 (Aussage 2)
es kann hier nicht Deutschland sein (Aussage 8 - D steht direkt rechts neben ….)
nicht Polen (Platz Nr. 4)
nicht Norwegen (N steht zwischen ….. - Aussage 5)
nicht Italien (Aussage 4)
nicht Spanien (Aussage 9 - an Platz 2 kann nicht 16 Tage sein, vergl. Aussage 3)
also muss hier Frankreich sein
daraus ergibt sich, dass an Platz 2 Spanien steht (Aussage 9).
1- F - 2003 - 16 Tage
2- S - 15 Tage
Daraus ergibt sich, dass Deutschland nicht an Platz 3 sein kann (Aussage 8).
Der 18 Tage Urlaub kann ebenfalls nicht an 3 sein (Aussage 5).
Norwegen entweder 3 oder 5
Deutschland 5 oder 6
Da Deutschland und Norwegen nicht nebeneinander sind (Aussage 6) muss Norwegen an Platz 3 sein. Folgt man dann Aussage 5, muss dann Spanien 2008 und Platz 4 der 18 Tage Urlaub sein. Da Deutschland rechts neben dem 17 Tage Urlaub ist (Aussage 8), muss Deutschland auf Platz 6 und der 17 Tage Urlaub auf Platz 5 sein. Italien, 2006, also an 5.
2007 nicht ganz rechts, also Deutschland nicht 2007 und nicht 14 Tage. Daraus folgt Deutschland - 19 Tage.
Platz 3 = 14 Tage
Norwegen - 14 Tage - 2007
2005 = Deutschland
2004 = Polen
Ich habe meine Tabelle nach und nach vervollständigt. Hier die Übersicht:
1 - Frankreich - 2003 - 16 Tage
2 - Spanien - 2008 - 15 Tage
3 - Norwegen - 2007 - 14 Tage
4 - Polen - 2004 - 18 Tage
5 - Italien - 2006 - 17 Tage
6 - Deutschland - 2005 - 19 Tage



Aufgabe 8

272. Wertungsaufgabe

„Die Aufgabe mit dem Zerschneiden haben wir in unserer Mathematikgruppe noch mal diskutiert. Dabei kamen wir auf ein anderes Problem.“ „Lass mal hören“, sagte Mike zu Lisa. „In ein Rechteck ABCD wird ein Dreieck ABX gezeichnet. X liegt dabei auf der Seite c des Rechtecks. Dabei entstehen auch die Dreiecke AXD und BCX.  Wo muss der Punkt X liegen, so dass der Flächeninhalt des Dreiecks AXD genau doppelt so groß ist wie der Flächeninhalt des Dreiecks BCX?“ „Haben das die Mitglieder der Mathematikgruppe herausgefunden?“, fragte Bernd nachdenklich. „Es hat einen kleinen Moment gedauert, aber dann stellte sich ja heraus, das war gar nicht so schwer.“ 6 blaue Punkte, wenn es gut und vollständig begründet wird.
„Ich habe hier noch so etwas Ähnliches“, sagte Maria. „Wieder geht es um ein Rechteck ABCD und um ein Dreieck im Inneren. Das Dreieck AXY hat die Eigenschaften, dass X auf der Seite b und Y auf der Seite c liegt. Es entstehen nun noch drei Dreiecke. Wie müssen X und Y gewählt werden, so dass die drei Dreiecke AYD, CYX und ABX alle den gleichen Flächeninhalt haben?“ - 8 rote Punkte

Lösung

Aufgabe 272 blau blau:
Der Flächeninhalt des Dreiecks AXD ist gleich 1/2 · c1d.
Der Flächeninhalt des Dreiecks BCX ist gleich 1/2 · c2b.
Der erste Flächeninhalt soll doppelt so groß sein ⇒ 1/2 · c1d= c2b.
Wegen b=d ⇒ 1/2 · c1b= c2b. | :b
1/2 · c1 = c2
c2 muss also halb so groß sein wie c1 bzw. die Strecke c wird im Verhältnis 1 zu 2 geteilt.
Aufgabe 272 rot rot:
Für die Flächeninhalte soll gelten:
1/2 · c1d = 1/2 · c2b1= 1/2 · b2a | · 2
c1d = c2b1 = b2a
Werden nun die Teilstrecken nicht absolut, sondern als Anteile von c bzw. b aufgefasst, lässt sich das so schreiben.
c1cd = c2cb1b = b2ba
(1-c2)cd = c2cb1b = (1-b1)ba
Wegen c = a und und b = d wird aus (1-c2)cd = (1-b1)ba
(1-c2)ab = (1-b1)ba und damit folgt (1-c2)=(1-b1) ⇒c2 = b1
Aus (1-c2)cd= c2cb1b wird nun (1-c2)ab = c2b1ab ⇒ (1-c2)ab = c2c2ab | : ab
(1-c2) = c2² Diese Gleichung lässt sich leicht ausrechnen: c2(und auch b1) = 1/2 + 1/2 √5 = 0,618... (1/2 - 1/2 √5 entfällt.)
Die Gleichung (1-c2) = c2² lässt aber auch so schreiben: (1-c2)/c2 = c2/1 und wird damit als Gleichung des goldenen Schnitts erkennbar.



Aufgabe 9

273. Wertungsaufgabe

„Zeig mal deine Zeichnung“, sagte Mike zu Lisa. „Da staunst du was?. Ich habe an jeder Seite des Quadrats ABCD (Kantenlänge a) jeweils ein gleichseitiges Dreieck hinein gezeichnet. Diese haben auch die Kantenlänge a. Zwei der Punkte eines jeden Dreiecks stimmen also mit den Punkten des Quadrats überein. Die anderen vier Eckpunkte bilden im Inneren des Quadrates wiederum ein Quadrat.“ - Wie lang ist die „kleine“ Quadratseite, wenn das Ausgangsquadrat eine Kantenlänge von 10 cm hat? (3 blaue Punkte für eine konstruktive Lösung + 6 blaue Punkte für eine Berechnung)
„Sag mal, deine Figur kann man doch auch mit nach außen zeigenden Dreiecken versehen, oder?“, fragte Bernd nachdenklich. „Aber klar doch. Das sieht auch nicht schlecht aus. Die außen liegenden Ecken der Dreiecke bilden ja dann auch wieder ein Quadrat“ meinte Lisa. In welchem Verhältnis stehen die Flächeninhalte des „äußeren“ und des „inneren“ Quadrates? - 6 rote Punkte

Lösung

Aufgabe 273 blau:
Die Konstruktion eines gleichseitigen Dreiecks ist ja nicht schwer. Misst man die Seiten des innen entstandenen Quadrates ab so erhält man etwas 5,2 cm.
Na und der Flächeninhalt ist dann auch kein Problem, oder?
Die Berechnung: DM ist gleich Höhe des gleichseitigen Dreiecks minus der halben Seite.
DM und BM bilden gleichschenklig rechtwinkliges Dreieck. Mit der Anwendung des Satz des Pythagoras (oder der Formel für eine Diagonale im Quadrat) ergibt sich:
kleine Quadratseite ak=(a/2√3-a/2)√2 mit a = 10 cm ergibt sich ak = 5,176 ... cm
rot:
Die Berechnung der äußeren Quadratseite aa ist nun nicht schwer.
CM ist gleich Höhe des gleichseitigen Dreiecks plus der halben Seite.
CM und AM bilden gleichschenklig rechtwinkliges Dreieck. Mit der Anwendung des Satz des Pythagoras (oder der Formel für eine Diagonale im Quadrat) ergibt sich:
Quadratseite aa=(a/2√3+a/2)√2 mit a = 10 cm ergibt sich ak = 19,318 ... cm
Das Verhätnis der Quadratseiten ist dann 3,73 .. und das der Flächeninhalte 3,73² = 13,93.
Nur mit Formel:
((a/2√3+a/2)√2)/((a/2√3-a/2)√2)
⇒ ((√3+1)√2)/((√3-1)√2) ⇒ (√3+1)/(√3-1)
erweitert mit (√3+1) ⇒
Das Verhältnis der Seiten ist 2 + √3 ⇒
Das Verhältnis der Flächeninhalte 7 + 4√3



Aufgabe 10

274. Wertungsaufgabe

„Was ist denn das für ein interessantes Gebilde?“, fragte Mike, als er Lisa beim Basteln eines durchlöcherten Würfels überraschte. „Nun zu erst hatte ich einen Würfel von 27 cm Kantenlänge. Dann habe ich den aus 9 cm großen Würfeln noch einmal gebaut. Das war aber etwas langweilig und so habe ich vorsichtig immer die Würfel heraus genommen, die mit Ihren Flächen mindestens 4 andere berührten. Den Rest habe ich dann verleimt, damit das Gebilde nicht verrutscht.“ Wie viele der 9 cm großen Würfel wurden entfernt? - 2 blaue Punkte. Wie groß sind Oberfläche und Volumen des so entstandenen Gebildes? - 8 blaue Punkte dazu.
Auch Bernd staunte. „Ich stelle mir gerade vor, wir würden nun mit 3 cm großen Würfeln Lisas „Lochwürfel“ noch mal bauen. Dann nehmen wir nach der gleichen Vorschrift (bezogen auf die vorher noch vollen Würfel)* wieder Würfel heraus. Das sieht bestimmt cool aus. Vielleicht solltet ihr das in eurer Mathegruppe mal basteln.“ „Das machen wir bestimmt mal,“ Wie viele der 3 cm großen Würfel wurden entfernt? - 1 roter Punkt. Wie groß sind Oberfläche und Volumen des so entstandenen Gebildes? - 7+4 rote Punkte dazu.

Lösung

Hier gab es etwas Verwirrung, da es einen "Druckfehler" in der ersten Version der Aufgabe gab. Hier nun aber die Lösung:
Aufgabe 274 blau:
Die Anzahl der heraus zunehmenden Würfel ist 7.
Kleine Überlegung. Jeder Würfel hat sechs Seiten, also maximal 6 Nachbarn. Zieht man die sichtbaren Seiten von 6 ab, so erhählt man die Zahl der Nachbarn. ...
Der große Würfel wurde durch 27 kleine ersetzt.
7 Würfel kommen weg ⇒ V = 20/27 des Ausgangsvolumens V0 = 27³ cm³ = 19683 cm³. Vblau= 14580 cm³
Die Ausgangsoberfläche A0 (6 &middat; 27² = 4374 cm²) wird durch 54 kleine Flächen ersetzt. Davon kommen 6 weg, aber innen 6 x 4 = 24 hinzu.
Es sind also 54 + (24 - 6) = 72 kleine Flächen oder 72/54 A0 = 4/3 A0 = 4/3 · 4374 cm² = 5832 cm²
Aufgabe 274 rot: Auch hier gab es Irritationen, wegen der Nachbarschaftsregel. *(bezogen auf die vorher noch vollen Würfel) sollte heißen, dass nur die zu ersetzenden großen Würfel einzeln zu betrachten waren. Dass die kleinen dann Nachbarn bekommen, die vorher zu einem anderen großen Würfel gehörten, sollte nicht ber¨cksichtigt werden. Damit entsteht dann dieses Bild. (Nun ja, der sogenannte "Menger-Schwamm", um den es sich hier handelt, lässt sich eindeutiger und kürzer beschreiben, aber dann wäre eben auch ein Teil der Aufgabenstellung nicht mehr vorhanden gewesen. Punkte auf Grund der Irritationen gab es trotzdem.)
Jeder der 20 Würfel wurde durch 27 kleinere Würfel ersetzt. Es werden also 20 x 7 = 140 Würfel entfernt.
Das Volumen ist denkbar einfach: Jeweils 7 der 27 Würfel werden entfernt, also bleibt 20/27 Vblau und das sind 10800 cm³ (oder 20 x 27 - 140 = 400 Würfel ⇒ 400 · 27 cm³ = 10800 cm³)
Oberfläche: Die 72 Flächen aus dem blauen Teil werden durch 72 x 9 = 648 ersetzt. davon werden in den "Eckbereichen" je 3 also 8 x 3 = 24 entfernt. In den "Mittelstücken je 4 also 12 x 4 = 48. Hinzu kommt, dass an jeder Stelle, wo ein Würfel (außer dem in der Mitte) entfernt wurde 4 neue Flächen auftauchen. 20 x 6 x 4 = 480 kommen hinzu.
Es sind also 648 - 24 - 48 + 480 = 1056 Flächen. Diese sind je 9 cm² groß also ergibt sich eine Oberfläche 9504 cm²
Auf die Herleitung von allgemeinen Formeln wird hier verzichtet. Zu beachten ist, dass einige Formeln im Internet die ursprünglichen Außenflächenteile vernachlässigen.



Aufgabe 11

275. Wertungsaufgabe

"Der "Schwamm" vom letzten Mal hatte es in sich, da wurde das Volumen immer kleiner und der Flächeninhalt wuchs immer mehr. "Schwamm" ist sogar richtig, denn das Gebilde heißt Menger-Schwamm.", sagte Bernd stolz. "Aber was machst du mit dem Schachbrett", fragte er Maria. "Ich untersuche Primzahlen." Nicht nur Bernd staunte. "Ich habe Zettel mit den Zahlen 1 bis 64. Die 64 kommt auf das Feld a8 (oben links). Dann lege ich die 63 rechts daneben, dann die 62 usw. immer am Rand entlang. Bin ich beim Feld a7, geht es wieder nach rechts weiter. So entsteht letztlich eine Zahlenspirale. Die 1 und alle anderen Nichtprimzahlen drehe ich herum. Jetzt geht es ans Zählen." Ermittle die maximale Anzahl von Primfeldern, die ein Läufer in einem "Zug" (Zick-Zack erlaubt) betreten kann, 3 blaue Punkte
"Es gibt aber auch eine zauberhafte Spirale von Primzahlen, die sich sogar über das Brett hinaus fortsetzen lässt. Zu einer solchen Spirale gehören die Primzahlen, deren Abstand von einer zur nächsten immer um 2 wieder zunimmt." Es sind möglichst alle Spiralen zu finden, die auf dem Brett beginnen und mindestens 3 Primzahlen enhalten, die auch auf dem Brett liegen. - pro Spirale gibt es einen roten Punkt

Lösung

Aufgabe 275 Die Lösung für blau lässt sich schnell ablesen. Die unterschiedlichen Schraffuren dienen nur der Kennzeichnung "Farbigkeit" der Schachbrettfelder.
Der längste Läferzug geht von der 37 aus, wenn man hin und zurück auch noch zuläst. Bei nur Zick-Zack ist der Zug von der 59 zur 53 auch 7 Felder lang.
rot: eine Variante bei einer Primzahl starten und dann systematisch suchen:
2 - 4 entfällt
3 - 5 - 9 entfällt
5 - 7 - 11 - 17 - 25 kurz aber geht.
7 - 9 entfällt
11 - 13 - 17 - 23 - 31 -41 - 53- (65) .
13 - 15 entfällt
17 - 19 - 23 - 29 - 37 - 47 - 59 - 73 - ... - 257 Superspirale
19 - 21 entfällt
23 - 25 entfällt
29 - 31 - 35 entfällt
31 - 33 entfällt
37 - 39 entfällt
41 - 43 - 47 - 53 - 61 - ... - 1601 Superspirale
43 - 45 entfällt
47 - 49 entfällt
53 - 55 entfällt
Es sind also 4 solche Spiralen zu entdecken.



Aufgabe 12

276. Wertungsaufgabe

Aufgabe 276 "Hallo Lisa, was machst du denn mit dem Schachbrettmuster?", fragte Bernd. "Ich bereite unseren Mathezirkel vor. Das Schachbrettpapier (8x8 - wie üblich) soll durch Zerschneiden entlang der Kanten in vier gleichgroße und zueinander kongruente Stücke zerteilt werden." "Da fallen mir sofort zwei ganz einfache Varianten ein," meinte Mike, der gerade vom Sport gekommen war. "Mir auch, meinte Bernd, "aber ich bin mir sicher, es gibt noch mehr Möglichkeiten." Für jede echt unterschiedliche Variante gibt es einen blauen Punkt - für besonders originelle auch 2.
"Für meine Aufgabe eignet sich das Schachbrett nicht", grübelte Bernd. "Lass hören". "Ihr sehr auf dem Bild das große Quadrat ABCD mit der Seitenlänge a. In diesem befindet sich das Quadrat EFGH. Die Seite EF liegt auf der Seite AB. Die Punkte GCDH bilden ein gleichschenkliges Trapez.
Wie groß muss das innere Quadrat sein, wenn dessen Flächeninhalt genau so groß sein soll wie der des Trapezes GCDH?" - 5 rote Punkte

Lösung

Für die blaue Aufgabe werden hier nach und nach die eingesandten Lösungen vorgestellt, ob das alle sind, möge die Zukunft zeigen. Danke u.a. an Elisa, Rafael und Carl.
Entscheidend für die Verschiedenheit der Lösungen wird die Form der Teile angesehen, da nach dem Ausschneiden nur die Form entscheidet.
Wegen der angestellten Überlegungen der Teilnehmer, sind allerdings teilweise verschiedene Muster, die auf die gleiche Lösung führen, mit dargestellt. Z.B. Bild 5 und 6 in der dritten Zeile

276_1 276_1_1
276_1_2 276_1_3
276_1_4 276_1_5
276_1_6 276_1_7

Hier die Varianten (109 !!!) von Elisa, danke. 276.pdf
Jetzt wird es ganz verückt: 734 (wahrscheinlich alle) Lösungen: 276_1.pdf vielen Dank an den nicht genannt sein wollenden.
Einfacher geht die Aufgabe möglicherweise, wenn das Quadrat einfach an den linken "Rand" geschoben wird. Die Flächeninhalte ändern sich dadurch nicht.
Die Seitenlänge des großen Quadrates ist a. Die Seitenlänge des kleinen Quadrates sei x.
Der Flächeninhalt des Trapezes lässt berechnen: (a+x)/2 · (a-x) . Dies soll gleich x² sein.
(a+x)/2 · (a-x) = x² | · 2
a² - x² = 2 x² | + x²
3 x² = a² | : 3
x² = a²/3 | √ 
etwas aufgehübscht
x = 1/3 · √3 · a ≈ 0,5773 · a


Hier fehlen noch einige Punkte, aber in der ewigen Liste sind sie erfasst.

Auswertung Serie 23 (blaue Liste)

Platz

Name

Ort

Summe

Aufgabe

 

265

266

267

268

269

270

271

272

273

274

275

276

1.

Rafael Seidel

Chemnitz

120

2

6

5

4

3

6

6

6

9

10

3

60

2.

Elisa Parsche

Chemnitz

112

-

-

-

-

-

-

-

-

-

-

3

109

3.

Samuel Kilian

Stuttgart

78

-

-

5

4

3

4

6

6

-

9

3

38

4.

Doreen Naumann

Duisburg

64

2

6

5

4

3

4

6

6

9

10

3

6

5.

Sabine Fischbach

Hessen

62

2

6

5

4

3

6

6

6

9

9

3

3

6.

Linus-Valentin Lohs

Chemnitz

51

-

-

-

4

-

4

6

-

-

10

3

24

7.

Paula

Hartmannsdorf

30

-

-

-

-

-

-

-

-

-

10

-

20

8.

Jamila Wähner

Chemnitz

20

-

3

-

-

-

4

-

-

-

10

-

3

9.

Jonathan Käßler

Chemnitz

18

-

-

-

-

3

2

-

-

3

10

-

-

9.

Adrian Schlegel

Chemnitz

18

-

-

-

-

-

-

4

-

2

-

2

10

9.

Jana (+S) Schneider

Lugau

18

-

6

5

4

3

-

-

-

-

-

-

-

10.

Lisa Grassmann

Chemnitz

17

-

-

5

4

-

-

-

-

2

-

3

3

10.

Hermann Thum

Chemnitz

17

1

3

-

4

-

2

-

-

3

-

-

4

10.

Andreas M.

Dittersdorf

17

-

6

5

3

3

-

-

-

-

-

-

-

11.

Thomas Klotz

Chemnitz

16

-

-

-

-

-

-

-

6

-

8

2

-

11.

Marcel Reichelt

Chemnitz

16

-

-

-

3

-

-

4

-

3

6

-

-

11.

Josephine Klotz

Chemnitz

16

-

-

-

-

-

-

-

6

-

8

2

-

12.

Katrin Posselt

Chemnitz

13

-

-

-

-

3

-

-

-

-

10

-

-

12.

Nina Zätsch

Chemnitz

13

-

3

-

-

-

-

-

-

-

9

1

-

13.

Anja Posselt

Chemnitz

12

-

3

-

-

3

-

-

-

-

3

-

3

14.

Ellen Richter

Chemnitz

11

-

3

-

4

-

-

-

-

2

2

-

-

15.

Tim Jechorek

Chemnitz

10

-

-

-

3

-

-

4

-

3

-

-

-

15.

Julian Vass

Chemnitz

10

-

-

-

-

-

-

-

-

-

10

-

-

15.

Jonas Frederik Otto

Lichtenwalde

10

-

-

-

-

-

-

-

-

-

10

-

-

15.

Felix Taubert

Chemnitz

10

-

-

-

3

-

-

4

-

3

-

-

-

15.

Marie Sophie Roß

Chemnitz

10

-

-

-

3

3

-

-

2

-

2

-

-

15.

Marcel Seerig

Chemnitz

10

-

-

-

-

-

-

-

-

-

10

-

-

16.

Ria Hopke

Chemnitz

9

-

3

4

-

-

-

-

-

2

-

-

-

16.

Duncan Mahlendorff

Chemnitz

9

-

-

-

-

-

2

3

2

-

2

-

-

16.

Marion Sarah Zenk

Chemnitz

9

-

6

-

3

-

-

-

-

-

-

-

-

16.

Anne Haag

Chemnitz

9

-

-

-

-

-

-

-

-

-

6

3

-

16.

Daniel Hufenbach

Chemnitz

9

-

-

-

-

-

-

-

-

9

-

-

-

17.

Kai-Lutz Wagner

Chemnitz

8

-

-

-

-

-

-

-

-

-

8

-

-

17.

Lukas Kirchberg

Chemnitz

8

-

-

-

3

-

2

-

-

3

-

-

-

17.

Hannah Gebhardt

Chemnitz

8

-

-

-

-

-

-

-

-

-

4

-

-

18.

Loise Reichmann

Chemnitz

7

-

3

-

1

-

-

-

-

-

3

-

-

18.

Ellen Wilde

Chemnitz

7

-

-

-

-

-

-

4

-

-

-

3

-

18.

Josephine Pallus

Chemnitz

7

-

-

-

-

-

-

4

-

-

-

3

-

18.

Emily Neuwirth

Chemnitz

7

-

-

-

3

-

-

4

-

-

-

-

-

19.

Robin König

Chemnitz

6

-

-

-

3

-

3

-

-

-

-

-

-

19.

Simon Anders

Chemnitz

6

-

-

-

-

-

-

6

-

-

-

-

-

19.

Camilla Schreiter

Chemnitz

6

-

-

-

-

-

-

-

-

-

6

-

-

19.

Lina Krug

Chemnitz

6

-

-

-

-

-

-

-

-

-

-

2

4

19.

Felix Brinkel

Chemnitz

6

-

-

-

-

-

-

4

-

2

-

-

-

19.

Tim Missullis

Chemnitz

6

-

-

-

-

-

-

-

-

-

6

-

-

19.

Stephanie Dani

Chemnitz

6

-

-

-

-

-

3

-

2

-

1

-

-

19.

Ole Koelb

Chemnitz

6

-

-

-

3

-

-

-

-

3

-

-

-

19.

Jule Irmscher

Eibenberg

6

-

-

-

-

-

-

-

-

-

2

2

2

19.

Pauline Marschk

Chemnitz

6

-

-

-

-

-

-

-

-

-

6

-

-

19.

XXX

???

6

-

6

-

-

-

-

-

-

-

-

-

-

19.

Johanna Ranft

Chemnitz

6

-

-

-

3

-

3

-

-

-

-

-

-

20.

Philipp Fürstenberg

Chemnitz

5

-

-

-

2

3

-

-

-

-

-

-

-

20.

Melanie Petz

Chemnitz

5

-

-

-

-

-

-

5

-

-

-

-

-

20.

Gunnar Reindt

Chemnitz

5

-

-

-

-

-

-

5

-

-

-

-

-

20.

Moritz Weber

Chemnitz

5

-

-

-

-

-

-

5

-

-

-

-

-

20.

Laurin Roßberg

Chemnitz

5

-

-

-

-

-

-

-

-

-

2

3

-

20.

Richard Hahmann

Chemnitz

5

-

-

-

2

3

-

-

-

-

-

-

-

20.

Felicitas Hastedt

Chemnitz

5

-

-

-

-

-

-

5

-

-

-

-

-

20.

Felicitas Güra

Chemnitz

5

-

-

-

-

-

-

5

-

-

-

-

-

20.

Carl Geißler

Chemnitz

5

-

-

-

-

-

-

-

-

-

-

-

5

21.

Charline Patelt

Chemnitz

4

-

-

-

-

-

-

4

-

-

-

-

-

21.

Theresa Jänich

Chemnitz

4

-

-

-

-

-

-

-

-

-

4

-

-

21.

Doreen Taubert

Chemnitz

4

-

-

-

-

-

-

4

-

-

-

-

-

21.

Karl Herrmann

Chemnitz

4

-

-

-

-

-

-

4

-

-

-

-

-

21.

Nele Mäding

Chemnitz

4

-

-

-

-

-

-

4

-

-

-

-

-

21.

Lilli Weiß

Chemnitz

4

-

-

-

-

-

-

4

-

-

-

-

-

21.

Luise Adam

Chemnitz

4

-

-

-

-

-

-

-

-

-

1

-

3

21.

Lene Haag

Chemnitz

4

-

-

-

-

-

-

4

-

-

-

-

-

21.

Cynthia Raschkowsky

Chemnitz

4

-

-

-

-

-

-

4

-

-

-

-

-

21.

Helene Fischer

Chemnitz

4

-

-

-

-

-

-

4

-

-

-

-

-

21.

Marvin Gülden

Chemnitz

4

-

-

-

-

-

-

4

-

-

-

-

-

21.

Charline Patzelt

Chemnitz

4

-

-

-

-

-

-

4

-

-

-

-

-

21.

Elina Rech

Chemnitz

4

-

-

-

-

-

-

4

-

-

-

-

-

21.

Jessica Ritter

Chemnitz

4

-

-

-

-

-

-

4

-

-

-

-

-

21.

Tobias Morgenstern

Chemnitz

4

-

-

-

-

-

-

4

-

-

-

-

-

21.

Valentin Grundmann

Chemnitz

4

-

-

-

-

-

-

4

-

-

-

-

-

21.

Ulrike Böhme

Chemnitz

4

-

-

-

-

-

-

4

-

-

-

-

-

21.

Lucas Steinke

Chemnitz

4

-

-

-

-

-

-

-

4

-

-

-

-

21.

Simon Winger

Chemnitz

4

-

-

-

-

-

-

4

-

-

-

-

-

21.

Valentin Sellin

Chemnitz

4

-

-

-

-

-

-

4

-

-

-

-

-

21.

Christian Wagner

Bamberg

4

-

-

-

-

-

-

4

-

-

-

-

-

21.

Anna Georgi

Chemnitz

4

-

-

-

-

-

-

4

-

-

-

-

-

22.

Ingmar Richter

Chemnitz

3

-

-

-

1

-

-

-

-

-

2

-

-

22.

Johannes Allert

Chemnitz

3

-

-

-

-

-

-

-

-

-

-

3

-

22.

Tom Hartig

Chemnitz

3

-

-

-

3

-

-

-

-

-

-

-

-

22.

Agnieszka Urban

Chemnitz

3

-

3

-

-

-

-

-

-

-

-

-

-

22.

Theresa G.

Chemnitz

3

-

-

-

-

-

-

-

-

-

-

3

-

22.

Tom Straßer

Chemnitz

3

-

-

-

-

-

-

3

-

-

-

-

-

22.

Julia Ritter

Chemnitz

3

-

-

-

-

-

-

-

-

-

-

3

-

22.

Henrike Grundmann

Chemnitz

3

-

-

-

-

-

-

-

-

-

-

3

-

22.

Nina Fischer

???

3

-

-

-

-

-

3

-

-

-

-

-

-

22.

Anja Ruf

???

3

-

-

-

-

-

-

3

-

-

-

-

-

22.

Ernesto Uhlmann

Chemnitz

3

-

-

-

-

-

-

3

-

-

-

-

-

23.

Phillpp Schleupner

Chemnitz

2

-

-

-

-

-

2

-

-

-

-

-

-

23.

Karolin Schuricht

Chemnitz

2

-

-

-

-

-

-

-

-

-

-

2

-

23.

Torsten Zwingelberg

Pritzwalk

2

-

-

-

-

-

2

-

-

-

-

-

-

23.

Pit Hopke

Chemnitz

2

-

-

-

-

-

-

-

-

-

2

-

-

23.

Katharina Zweiniger

Chemnitz

2

-

-

-

-

-

-

-

-

-

2

-

-

23.

Tim Sigmund

Chemnitz

2

-

-

-

-

-

-

-

-

-

2

-

-

23.

Alina Berger

Chemnitz

2

-

-

-

-

-

-

-

-

-

2

-

-

23.

Michelle Wade

Chemnitz

2

-

-

-

-

-

-

-

-

-

2

-

-

23.

Florian A. Schönherr

Chemnitz

2

-

-

-

-

-

-

-

-

-

-

2

-

23.

Emma Irmscher

Eibenberg

2

-

-

-

-

-

-

-

-

-

-

2

-

23.

Amarin Roßberg

Chemnitz

2

-

-

-

2

-

-

-

-

-

-

-

-

23.

Eva-Lotta Rümmler

Chemnitz

2

-

-

-

-

-

-

-

-

-

2

-

-

23.

Toni Krug

Chemnitz

2

-

-

-

-

-

-

-

-

-

2

-

-

23.

Ida Gwendolin Eichler

Chemnitz

2

-

-

-

-

-

-

-

-

-

2

-

-



Auswertung Serie 23 (rote Liste)

Platz

Name

Ort

Summe

Aufgabe

 

265

266

267

268

269

270

271

272

273

274

275

276

1.

Doreen Naumann

Duisburg

54

6

5

-

4

3

2

8

5

6

6

4

5

2.

Samuel Kilian

Stuttgart

53

-

-

4

4

4

6

7

8

-

12

3

5

3.

Sabine Fischbach

Hessen

46

-

3

-

4

2

5

8

5

6

8

-

5

4.

Jana (+S) Schneider

Lugau

20

6

5

4

2

3

-

-

-

-

-

-

-

5.

Paula

Hartmannsdorf

15

-

-

-

-

-

-

-

-

-

10

-

5

6.

Andreas M.

Dittersdorf

13

-

5

4

2

2

-

-

-

-

-

-

-

7.

Jonathan Käßler

Chemnitz

10

-

-

-

-

-

-

-

-

-

10

-

-

7.

Linus-Valentin Lohs

Chemnitz

10

6

-

-

4

-

-

-

-

-

-

-

-

7.

XXX

???

10

-

2

-

-

-

-

-

8

-

-

-

-

8.

Elisa Parsche

Chemnitz

9

-

-

-

-

-

-

-

-

-

-

4

5

9.

Christian Wagner

Bamberg

8

-

-

-

-

-

-

8

-

-

-

-

-

10.

Daniel Hufenbach

Chemnitz

6

-

-

-

-

-

-

-

-

6

-

-

-

10.

Stefan Giron

Dresden

6

6

-

-

-

-

-

-

-

-

-

-

-

11.

Josephine Pallus

Chemnitz

5

-

-

-

-

-

-

-

-

-

-

-

5

11.

Andreas Walter

Bautzen

5

-

-

-

-

-

-

-

-

-

-

-

5

11.

Marion Sarah Zenk

Chemnitz

5

-

5

-

-

-

-

-

-

-

-

-

-

12.

Lisa Grassmann

Chemnitz

4

-

-

-

4

-

-

-

-

-

-

-

-

13.

Ole Koelb

Chemnitz

3

-

-

-

-

-

-

-

-

3

-

-

-

13.

Katrin Posselt

Chemnitz

3

-

-

-

-

-

-

-

-

-

3

-

-

13.

Tim Jechorek

Chemnitz

3

-

-

-

-

-

-

-

-

3

-

-

-

14.

Rafael Seidel

Chemnitz

2

-

-

-

2

-

-

-

-

-

-

-

-

14.

Jamila Wähner

Chemnitz

2

-

-

-

-

-

-

-

-

-

2

-

-

Valid XHTML 1.0  Transitional

Serie-22

Serie 22
Aufgaben und Lösungen

Spiele und Mathematik

Aufgabe 1

253. Wertungsaufgabe

Hallo Mike, hast du schon von dem Skandal im Vorfeld der Monopoly-WM gehört?" Mike schaut Bernd etwas zweifelnd an. "Das Team von Otto Bitter, dem Mathematiker aus C. wurde durch einen Detektiv ausspioniert, welcher von einem Teamchef aus L. engagiert wurde. Der berichtete seinem Auftraggeber in einem geheimen Chat, der aber geknackt wurde. Das kannst du hier nachlesen." "Zeig mal."
Zum Team von Otto Bitter gehören 5 Leute, diese fahren jeweils eines der Fahrzeuge Moped, Fahrrad, Motorrad, Auto und man glaubt es kaum, auch ein altes Skateboard ist dabei.
Die Fahrzeuge sind zwischen 8 und 12 Jahren alt.
Die Familiennamen der Teammitglieder lauten Hoppe, Kalt, Holz, Meier und Siebert.
Das Fahrrad ist älter als Dieter Meiers Fahrzeug.
Der Hoppe, welcher nicht Emil heißt, hat ein nicht 10 Jahre altes Gefährt.
Das Moped ist ein Jahr älter als das Auto von dem Siebert.
Bert hat ein 9 Jahre altes Fahrzeug. Das Motorrad ist hingegen 11 Jahre alt.
Carlos Skateboard ist älter als das Fahrzeug von Mister Kalt.
Zu Arno kann ich noch nichts sagen.
"Etwas verworren, was der Detektiv so schreibt," meinte Bernd. "Stimmt, aber immerhin kann man schon jetzt herausbekommen, wie die 5 Teammitglieder von Otto Bitter mit vollständigem Namen heißen, welches Fahrzeug sie fahren und wie alt die Fahrzeuge sind." - 5 blaue Punkte
Weiter im Chatprotokoll:
Zur Absprache trafen sich die 5 Teammitglieder jeweils an einem anderen Tag der letzten Woche einzeln mit Otto Bitter (Mo. -- Fr.).
Mal dauerte es recht lange, dann wieder ging es recht schnell. Was besprochen wurde, war nicht zu erfahren. Es waren Zeiten von 1 Stunde, 1,5 Stunden, aber auch 2,5 bzw. 3,5 Stunden, einmal sogar 4 Stunden.
Die Treffen begannen 17.00 Uhr, 17.10 Uhr, 17.20 Uhr, 17.30 Uhr und einmal gar erst um 17.40 Uhr. Jedes Teammitglied hat einen anderen Beruf.
Der Dachdecker kam nicht am Montag. Er blieb eine Stunde weniger lange als der, dessen Treffzeit 10 Minuten nach der des Dachdeckers begann.
Der Fleischer kam an einem späteren Wochentag als der, der nur eine Stunde blieb und an einem früherem Tag als der, der um 17.10 Uhr kam.
Der Dachdecker kam nicht am Mittwoch, er kam an einem früherem Tag als der, welcher um 17.30 Uhr kam, wobei dieser wiederum eine Stunde länger blieb als der Frisör, der nicht am Montag kam.
Derjenige, welcher 3,5 Stunden blieb, traf zu einer früheren Uhrzeit ein als der Maler, der nicht am Montag kam und nicht der Rekordhalter der Aufenthaltsdauer war.
Zu dem Team gehört auch ein Bäcker.
"Uff, ich kann hier nicht den Beruf und den passenden Namen zusammenbringen.", meinte Bernd. "Stimmt, aber Beruf, Tag, Eintreffzeit und Beratungszeit lassen sich zuordnen." 5 rote Punkte

Lösung

Die Lösung von Sabine, - besser geht es nicht, danke
Lösung Aufgabe 253



Aufgabe 2

254. Wertungsaufgabe

Kugelwürfel"Von dem Skandal der letzten Woche habe ich erst einmal genug." "Das geht mit genau so", gab Mike Bernd Recht."Hast du schon Marias neue Würfelkreation gesehen?" "Zeig mal". Staunend begutachtet Mike den Würfel, an dessen 8 Ecken sich jeweils eine kleine Kugel befindet. Auf zwei der Kugeln stehen die Zahlen 1 und 3. Auf den anderen Kugeln sind mit Bleistift die Buchstaben a bis f zu erkennen. "Was hat es mit den Kugeln und Buchstaben auf sich?" "Maria und Lisa sind noch dabei, die Buchstaben durch die Zahlen 2; 4; 5; 6; 7 und 8 zu ersetzen -- keine Dopplungen, so dass die Zahlen auf jeder der Seiten in der Summe 18 ergeben." "Ich verstehe."
Wie können die Zahlen verteilt werden -- Angabe der Seite und "Kontrollrechnung" - 4 blaue Punkte.
Ist es möglich, dass die Seitensumme der Kugelzahlen andere Werte annimmt -- je eine  der Zahlen 1 bis 8 an den Ecken soll bleiben und die Summe soll natürlich auf allen Seiten gleich sein. - 2 rote Punkte.

Lösung

Kugelwürfel Lösung Lösungen von Andree D. (linkes Bild) und Felix K. (rechtes Bild), danke
Kugelwürfel Lösung Es wurden noch drei weitere Varianten gefunden.
zur roten Aufgabe: Die Summe der Zahlen eins bis acht ist 36. Betrachtet man alle Seiten des Würfels, so wird jede Zahl drei mal eingesetzt. Die Summe aller Seiten zusammen ist also 3 · 36 = 108. Verteilt auf die 6 Seiten des Würfels ergibt das eine Seitensumme von 18. Es ist also mit diesen Zahlen nur die 18 möglich.



Aufgabe 3

255. Wertungsaufgabe

Dominoaufgaben"Da hatte ich ja Glück, dass ich bei dem Kugelwürfel letzte Woche schon richtig angefangen hatte", meinte Maria. "Vielleicht hilft deine mathematische Intuition auch hierbei?", fragte Lisa. "Lass sehen." Ich habe mir aus einem Dominospiel (0-0 bis 6-6, also das mit den 28 Steinen) vier Steine genommen und das gelegt. Was siehst du?" "Hm, ach ja. Die Steine lassen sich als 3 Zahlen auffassen. Eine dreistellige Zahl a oben, eine einstellige Zahl b in der zweite "Reihe" und eine vierstellige Zahl c unten. Es gilt nun noch a mal b = c." " Ist ja cool. Gibt es noch mehr solcher Aufgaben?" "Ich denke schon." Für jede gefundene Aufgabenstellung gibt es zwei Punkte, aber wenn man eine solche gefunden hat, dürfen die Steine nicht noch einmal für eine weitere Aufgabe genommen werden. (Die Steine der Beispielaufgabe sind also auch nicht noch mal verwendbar. Mit den verbleibenden 24 Steinen wären also theoretisch noch 6 Aufgaben legbar, aber auch praktisch? - keine "0" am Anfang der Zahl.)
Wie viele Steine hat ein 12er-Superdomino? Es gibt also die Steine 0-0 bis 12-12, wobei auch hier gilt, jede Zahl darf immer nur einmal mit jeder der Zahlen kombiniert werden. (2 rote Punkte, wie viele Punkte müsste der höchste Paschstein mindestens haben, wenn man ein Dominospiel mit mehr als 1 000 Steinen haben möchte -- noch einmal 2 rote Punkte.)

Lösung

Es gibt zum Beispiel diese Variante alle 28 Steine zu verwenden:
Vorgabe war: 223 * 5 = 1115 s. Bild
500*2=1000; 401*3=1203; 415*4=1660
633*4=2532; 664*4=2656; 555*6=3324
Es wurden bis zu 5 Lösungen gefunden, herzlichen Glückwunsch, hier noch ein paar Beispielaufgaben:
300*4=1200; 661*4=2644; 33*5= 1610;
602*6=3612; 256*4=1024; 444*3=1332
611*4=2444; 465*5=2325; 600*5=3000
rot. Es viel Wege die nach Rom führen, so auch hier. Alle Steine xy eines Spieles werden erfasst, wenn die Bedingung x<=y gefordert wird. (oder alternativ x>=y) - so werden Doppelzählungen vermieden. Die Höchstpunktzahl auf einem Stein sei n (Normaldomino n=6). Also x; y <=n.
Dann wird schnell klar: Es gibt n+1 Steine 0y, n Steine 1y, n-1 Steine 2y, ... 1 Stein ny, (nämmlich nn). Daraus folgt: Die Summe der Steine ist:
(n+1) + n + ... 3 + 2 + 1 bzw. 1 + 2 + 3 + ... + n + (n+1) (Beispiel n= 6 ==> 1+2+3+4+5+6+7=28)
Nun lassen sich die gesuchten Ergebnisse durch Auszählen finden oder mittels der Summenformel (n+1)(n+2)/2.
Superdomino: n = 12 also sind es 91 Steine.
n=43 ==> 990 Steine, n = 44 ==> 1035 Steine. Also müsste der gesuchte höchste Paschstein 44|44 sein, um mindestens ein 1000-er Spiel zu haben.



Aufgabe 4

256. Wertungsaufgabe

Dominoaufgaben Rahmen"Das mit dem Dominospiel hat mir gefallen und ich hätte nicht vermutet, dass man wirklich sieben verschiedene solche Multiplikationsaufgaben legen kann", meinte Bernds Opa, der sich mal wieder von seinem Garten losreißen konnte und zum Kaffeetrinken gekommen war. "Da habe ich auch gleich eine solche Aufgabe für euch. Schaut euch die vier Steine an, die ich zu einem Quadratrahmen zusammengelegt habe. Die Summe der Punkte auf jeder Seite des Rahmens ist gleich. Nehmt nun die verbleibenden Steine und bildet weitere solche Rahmen." Die blaue Punktzahl für jeden gefundenen Rahmen ist zwei. (Die Steine der Beispielaufgabe sind also nicht noch mal verwendbar. Mit den verbleibenden 24 Steinen wären also theoretisch noch 6 Rahmen legbar, aber auch praktisch? - Die Gleichheit der Seitensumme gilt immer nur für einen Rahmen. Im Beispiel war die Summe 9, es kann aber genau so auch mehr oder weniger pro Quadrat sein.)
Man kann natürlich auch versuchen aus allen 28 Steinen einen solchen Quadratrahmen zu legen. Welche Varianten für je gleiche Seitensummen gibt es? 4 rote Punkte

Lösung

blau:
Lösung Teil 1
Lösung Teil 2
Die Summe für eine solche Seite eines Superrahmens ergibt sich aus einem 1/4 der Summe aller Punkte auf dem Domino und 1/4 der Summe aller Punkte, die die Ecken des Rahmens bilden. (Das kann man für das obige Bild ganz gut nachvollziehen.
Jeder der Werte von 0 bis 6 ist 8 mal auf dem Domino vertreten, also 8*(0+1+2+3+4+5+6) = 168. werden "leere" Ecken verwendet, so ist die minimale Seitesumme 168/4 = 42 (ist ja so wie so die Antwort auf alles ;-) ) Ist in jeder Ecke eine "6", so ist nach obiger Überlegung, die gesuchte maximale Seitesumme 48.



Aufgabe 5

257. Wertungsaufgabe

"Mit den Dominosteinen lässt sich noch mehr anstellen, aber ich muss mich erst einmal mit dem Testen meiner gleichseitigen Dreiecke befassen", meinte Lisa. "Wir bereiten eine kleine Geometrieausstellung mit unserer Spezialistengruppe vor." "Die Dreiecke sind ja auch alle gleich groß und aus weißem Karton. Soll ich dir helfen, noch welche auszuschneiden? Wie viele brauchst du denn?" "Das weiß ich noch nicht genau, ich bin noch am Überlegen. Ich will Figuren aus diesen Dreiecken zusammenstellen. Die Dreiecke werden genau auf Kante gelegt. Wenn ich nur zwei Dreiecke verwende, gibt es letztlich nur eine Möglichkeit, eine solche Figur zu legen. Würde ich zwei Zweier legen, so lassen sich die durch Drehung, Spiegelung oder Verschiebung aufeinander abbilden. Damit sind sie nicht wirklich verschieden von einander." "Ach ich verstehe," meinte Mike, der Feuer und Flamme war, "das ist wie bei der Frage nach der Anzahl von möglichen Würfelnetzen, wo es letztlich auch nur 11 verschiedene gibt." "Genau".
Wie viele Dreiecke braucht Lisa, wenn sie alle Möglichkeiten für je 4 bzw. 5 Dreiecke legen und zusammenkleben will. (Es gibt je einen blauen Punkt für die Varianten, wenn was doppelt ist, werden die Punkte wieder abgezogen.) Rote Punkte gibt es für die 6-Dreieckvariante. Interessant wäre sicher auch, die Frage wie viele Möglichkeiten es bei n Dreiecken gibt -- dafür habe ich bisher noch keine Formel entdecken können.

Lösung

Ein Dreieck - eine Möglichkeit
Zwei Dreiecke - eine Möglichkeit
Drei Dreiecke - eine Möglichkeit der Dreier
blaue Aufgabe:
Vier Dreiecke - drei Möglichkeiten der Vierer 1der Vierer 2der Vierer 3 Es werden also 12 einzelne Dreiecke gebraucht.
Fünf Dreiecke - vier Möglichkeiten der Fünfer 1der Fünfer 2der Fünfer 3der Fünfer 4 Es werden also 20 einzelne Dreiecke gebraucht.
Damit waren also sieben blaue Punkte möglich.



rote Aufgabe: Sechs Dreiecke - zwölf Möglichkeiten:
1.der Sechser 1 2.der Sechser 2
3.der Sechser 3 4.der Sechser 4
5.der Sechser 5 6.der Sechser 6
7.der Sechser 7 8.der Sechser 8
9.der Sechser 9 10.der Sechser 10
11.der Sechser 11 12.der Sechser 12
Es werden also 72 einzelne Dreiecke gebraucht.
Anmerkungen:
Verwendet man 7 Dreiecke gibt 24 Möglichkeiten
Verwendet man 8 Dreiecke gibt 66 Möglichkeiten
Verwendet man 9 Dreiecke gibt 160 Möglichkeiten
Verwendet man 10 Dreiecke gibt 448 Möglichkeiten
Verwendet man 11 Dreiecke gibt 1186 Möglichkeiten


Aufgabe 6

258. Wertungsaufgabe

"Mit den in der letzten Woche gefundenen Dreiecksteilen lassen sich viele Figuren legen", meinte Mike. (Zu benutzen sind die Ergebnisse der roten Aufgabe.) "Dann mach doch mal einen Vorschlag." " Nun, Du siehst doch, dass eines der 12 Teile wie ein regelmäßiges Sechseck aussieht. Mit welchen der 12 Teile, lässt sich wohl ein doppelt so großes Sechseck legen?" (4 blaue Punkte). "Mich würde ja mal interessieren, ob man aus allen 12 Teilen auch schöne Figuren legen kann," fragte Maria ihren Bruder.
"Sicher, aber wie groß mag wohl der kleinstmögliche Umfang einer solchen Figur sein, wenn eine Dreiecksseite drei Zentimeter groß ist?" 6 rote Punkte.

Lösung

Die Aufgaben 257 und 258 basieren auf dem Spiel "Verhext" nach Professor Haber. Die aus Dreiecken zusammengesetzten Teile heißen auch Polyiamonds. zum Weiterlesen
Blaue Aufgabe:
der Doppel-Sechser Es gibt eine Vielzahl von Möglichkeiten solche ein Sechseck zu legen. Es ist auch möglich aus 9 Figuren ein entsprechend drei mal so großes Sechseck zu legen.
Legt eine Figur aus allen Teilen, so besteht diese - sofern man keine Lücken im Inneren hat aus 72 Dreiecken. Durch die Beschränkung auf die gleichseitigen Dreiecke lässt sich recht leicht herleiten, dass die Zerlegung von 72 in 8*9 Dreiecke ein "kompaktes" Parallelogramm entsteht. Wie man ein solches legen kann, ist in dem obigen Hinweis entnehmbar.
Dort kann man auch die Länge des Umfangs auszählen. Allerings ergeben sich für das 6x12 Parallogramm 24x3 cm Randlinie, für ein 8X9 26x3 cm. Umso bemerkenswerter, dass diese legbare Figur nur 22x3cm Randlinie aufweist (stammt aus der Spielanleitung):
der Kurzrand
Ich lass eine endgültige Entscheidung mal noch offen.



Aufgabe 7

259. Wertungsaufgabe

TIC-TAC-TOE "Ich habe heute den Film Wargames angeschaut und dort ging es unter anderem um das bekannte Spiel Tic-Tac-Toe. Es wurde behauptet, dieses Spiel geht immer unentschieden aus." "Den Film kenne ich auch", sagte Lisa, "aber verlieren kann man da schon, wenn man nicht aufpasst." "Na, das ist ja klar. Die Frage, die ich für 4 rote Punkte stelle, ist:   Wohin darf der Spieler B seinen ersten Zug nicht machen, wenn A auf das Feld 3 gesetzt hat, weil sonst der Sieg von A nicht zu verhindern ist?" "Du meinst, so einen Fehler gibt es", gab Mike zu bedenken.
"Meistens startet A ja genau in der Mitte," war Bernds Meinung. Welchen "ersten Fehler" darf B nicht machen, weil sonst A mit Sicherheit gewinnt -- 4 blaue Punkte.
TIC-TAC-TOE - online zum Üben und Probieren

Lösung

Da das Spielfeld symmetrisch ist, wird hier jeweils nur eine der möglichen Varianten besprochen. Auch heißt ja die Aufgabe, welchen Zug darf B nicht machen, ob es außer der eingesandten bzw. hier dargestellten Variante noch mehr Möglichkeiten gibt war, ja nicht gefragt. ;-)
Blau: Setzt B auf Feld 2, so setzt A auf 3. Nun muss Spieler B auf Feld 7 setzen. Jetzt setzt Spieler A auf das Feld 6 und erreicht damit zwei Linien, von denen er auf jeden Fall eine komplettieren, den B kann nur eine Vervollständigung verhindern.
Rot: Setzt Spieler B auf 2, so kontert Spieler A mit Feld 5. Nun muss Spieler B auf Feld 7 setzen. Jetzt setzt Spieler A auf das Feld 6 und erreicht damit zwei Linien, von denen er auf jeden Fall eine komplettieren, den B kann nur eine Vervollständigung verhindern.



Aufgabe 8

260. Wertungsaufgabe

"Hallo Mike, du magst doch Logikrätsel?", fragte Maria. "Aber klar doch, so was hatten wir schon am Anfang dieser Serie." "Ja, das weiß ich, aber ich meine Mastermind". "Ach so." "Na klar, du weißt schon, einer versteckt vier Farbstifte und der andere muss die Kombination erraten. Wir haben das mit unserer Spezialistengruppe zuerst in einer vereinfachten Würfelvariante gespielt." "Erzähl mal."
"Also, ich nehme 5 Würfel (für nur 5 Farben). Ich verstecke eine Würfelkombination aus 4 verschiedenen Zahlen -- ohne die 6. Mein Gegenspieler dreht vier Würfel so, wie er glaubt, was meine Kombination wäre. Ist die Zahl dabei, aber an der falschen Stelle, dann schreibe ich ein kleines r, ist die Zahl richtig und an der richtigen Position, dann bekommt er ein großes R. Pass auf, ein Beispiel: Meine Kombination sei 3415, der Gegenspieler legt 4213, jetzt bekommt er ein rrR. Er versucht durch eine neue Kombination sich meiner Variante zu nähern und das so lange, bis er von mir ein RRRR bekommt." "Das habe ich verstanden, lass es uns probieren."
1234 rrr
5123 rrR
4152 rrrR
2451 rrrR
4125 rrrr
"Na komm, jetzt solltest du aber die richtige Kombination finden, außerdem war auch was nicht ganz logisch" - 4 blaue Punkte (Begründung nicht vergessen)
Das richtige Mastermind hat 6 verschiedene Farben, von denen vier versteckt werden. Wie viele Möglichkeiten gibt es für die Farbkombinationen, wenn die versteckten Farben alle verschieden sein müssen bzw. wenn die Farben beliebig oft benutzt werden dürfen? (2+2 rote Punkte)

Lösung

blau: Aus dem Vergleich von letzter Zeile - Zahlen richtig aber alle an der falschen Stelle - und der zweiten Zeile folgt 5 kommt auf Position 1. (unlogisch in den Zeilen die gleich 12 Position zu verwenden, denn in Zeile 2 gibt es nur ein R.
Die 2 kann nicht an Position 2 sein (1. Zeile) und nicht an Position 3 - letzte Zeile, aber auch nicht Pos. 1, denn da ist die 5. also ist die 2 an der letzten Stelle.
5XX2
Wegen der letzten Zeile folgt nun 5X12, letztlich 5412.
rot: alle Farben verschieden: 1. Stelle 6 Möglichkeiten, verbleiben für die 2. Stelle noch 5 Möglichkeiten, jetzt kann an an der dritten Stelle aus noch 4 Möglichkeiten wählen und die letzte Stelle erlaubt noch 3 Möglichkeiten. Es sind also 6*5*4*3 = 360 Varianten.
die Farben beliebig oft führt auf 6 Möglichkeiten an jeder Stelle also 6*6*6*6= 1296 Varianten.



Aufgabe 9

261. Wertungsaufgabe

"Nach dem Mastermind ist uns Maria aber noch so ein richtiges Logikrätsel schuldig oder was meinst du Lisa?" "Fragen wir wir sie doch einfach, da kommt sie gerade." "Ist ja schon gut Mike, ich habe sogar zwei, lies sie dir durch."
Für 6 blaue Punkte soll der schüchterne Schachspieler Robert Bootsbauer sein Geheimnis lüften. Dieser Robert reiste vor einem großen Turnier in Sachsen herum und wollte nicht gesehen werden. Der gerissene Reporter N. F. war ihm auf der Spur, aber am Ende fand er nur heraus, wann Robert in welchem der fünf Orten war, was für eine Krawatte er an dem Tag trug und von wie vielen Personen (2 bis 6 -- jedes Mal anders) das bestätigt werden konnte. Hier nun sein Bericht:
1. Die Anzahl der Personen, die den Schachspieler mit einer blauen Krawatte sahen, ist zwei kleiner als die der Personen vom 6. November, wo er aber nicht in Zwickau war.
2. Am 3. November ist Robert von weniger als 5 Leuten gesehen worden.
3. Hingegen gab es in Leipzig - am 17. -, mehr Personen als in der Stadt, in der er mit einer roten Krawatte erspäht wurde.
4. Es waren genau vier Personen, die ihn mit seiner grünen Krawatte sahen.
5. Genau 5 Leute sahen ihn am 21. November.
6. Seine schwarze Krawatte wurde von einer Person mehr gesehen als in Dresden.
7. Ach ja, und am 12. November wurde er von weniger Personen gesehen als in Chemnitz, dort trug er eine furchtbare lila Krawatte.
8. Was er in Döbeln wollte, war nicht herauszufinden.
Rot -- 8 Punkte
Zur Chemnitzer Spielenacht, die immer in der neuen Mensa stattfindet, gibt es auch Turniere: Monopoly, Mensch ärgere dich nicht, aber auch Siedler von Catan. Diese Turniere werden über mehrere Runden durchgeführt und sind ziemlich spannend. Die Siedler wurden an sechs Tischen mit jeweils 6 Teilnehmern gespielt. ("Der" Spieler ist kein Hinweis auf das Geschlecht). Es gab also die Startnummern 1 bis 36. In der vorletzten Runde lagen die Startnummern 8, 16, 18, 32, 34 und 36 vorn. Ina, Christian, Anja, Tom, Birgit und Marlon waren ganz aussichtsreich. Wer hatte welche Startnummer, wie war die Platzierung in der vorletzten Runde und wie am Schluss? - keine Spieler teilten sich einen Platz, sondern waren auf Platz 1, 2 ... oder 6.
1. Christians Startnummer war kleiner als die von Marlon und er - Christian  - wurde am Ende nicht Letzter.
2. Der Spieler mit der vorletzten Startnummer aus der Gruppe war in der Vorrunde auf Platz 6.
3. Inas Startnummer war doppelt so groß wie von dem, der am Ende Vierter wurde.
4. Anja wurde am Ende Fünfte, in der vorletzten Runde war sie eine Position hinter Marlon.
5. Der Bronzegewinner hatte sich um einen Platz verbessert.
6. Birgit, sie hatte nicht die 16, war in der vorletzten Runde die Erste. Der zu  dem Zeitpunkt auf Rang drei liegende, ist auch am Ende zwei Plätze hinter ihr.
7. Tom hat die Nummer 8 und kommt am Ende fast noch (eins dahinter) an den Spieler heran, der in der vorletzten Runde noch an zweiter Position lag.

Lösung

Auf eine Herleitung der Lösung wird dieses Mal verzichtet, die Teilnehmer haben meist die PM-Kreuzgittermethode gewählt.
Blau:
3.11, Chemnitz, 3 Leute, lila
6.11, Döbeln, 4 Leute, grün
12.11, Zwickau, 2 Leute, blau
17.11, Leipzig 6 Leute, schwarz
21.11, Dresden, 5 Leute, rot
Rot:
Name, Platz, vorletzte Runde, letzte Runde, Startnummer:
Birgit, 1, 4, 18
Chris, 2, 2, 16
Ina, 3, 6, 36
Tom, 4, 3, 8
Marlon, 5, 1, 32
Anja, 6, 5, 34



Aufgabe 10

262. Wertungsaufgabe
"Also ich fand die Logikaufgabe der letzten Woche ganz schön knifflig, um so schöner, dass ich sie dann doch noch heraus bekam," meinte Bernds Opa, der wieder mal zur Tüftlerrunde gestoßen war." In einem älterem Spielebuch habe ich verschiedene Mühlespielvariationen entdeckt. Einfach, aber genial." Lass hören."
"Bei einer der Varianten ist es so: Man konstruiert ein regelmäßiges Fünfeck. Die Mittelpunkte der Seiten werden ermittelt. Die fünf Mittelpunkte werden zu einem Fünfeck verbunden. Nun wiederholt sich das Ganze. Mittelpunkte -- neues Fünfeck usw. Bis zum Schluss in dem ersten Fünfeck vier weitere Fünfecke drin sind. Die Mittelpunkte des äußeren Fünfeckes werden mit dem jeweils direkt innen liegenden Mittelpunkt der Seite des kleinsten Fünfeckes verbunden. Für fünf blaue Punkte soll die Zahl der Spielpunkte ermittelt werden. (Spielpunkte sind die Stellen, wo eine Mühle gebildet werden darf, also 3 Punkte, die durch eine Linie miteinander verbunden sind. 2x5 rote Punkte soll es für die Gesamtlänge aller Linien geben, die auf dem Spielbrett sind. (Das erste Fünfeck hat eine Kantenlänge von 20 cm. 5 rote Punkte, wenn die Aufgabe sauber konstruktiv gelöst wird.)"

Lösung

Fünfeck-mühle Links sieht man eine Zeichnung des Spielfeldes. Zählt man die Punkte aus, so erkennt man, dass es genau 30 Punkte sind, die zur Mühlebildung geeignet sind.
Die Fünfecke sollen regelmäßig sein, daraus folgt, dass der Winkel AMD (360° : 5) 72° groß ist. (M – Mittelpunkt des Umkreises.) Der Winkel BAC ist dann als Basiswinkel des Dreiecks AMD 54° groß. (54° + 54° + 72° = 180°). MC ist die Höhe des gleichschenkligen Dreiecks AMD. (Höhe über der Basis teilt die diese und das soll C ja machen.) Damit ist der Winkel AMC 36° groß. Daraus folgt Winkel BCA ist gleich (180° - 90° -54°) = 36°.
cos BCA = BC/AC also BC = AC *cos BCA.
Auf Grund der Ähnlichkeit der Figuren setzt sich die Berechnung der halben Fünfeckseiten fort.
u = 10 * AC + 10 * AC*cos BCA + 10 *AC*cos BCA*cos BCA + 10 * AC*cos BCA*cos BCA*cos BCA+ 10 * AC*cosBCA*cos BCA*cos BCA*cos BCA
u= 10 * AC(1+ cos BCA + cos BCA*cosBCA + cos BCA*cos BCA*cos BCA + cos BCA*cos BCA*cos BCA*cos BCA)
Dazu kommen noch die nach innen führenden Längen l:
Es gilt hier:
AB = AC* sin BCA, wieder kommt die Ähnlichkeit ins Spiel, denn die Verkleinerung setzt sich nach innen noch drei mal fort.
l = 5* AC* sin BCA + 5* AC* cos BCA*sin BCA + 5* AC* cos BCA* cos BCA* sin BCA + 5* AC* cos BCA* cosBCA* cos BCA* sin BCA
l = 5*AC( sin BCA + cos BCA* sin BCA + cos BCA* cos BCA* sin BCA + cos BCA* cos BCA* cos BCA* sin BCA)
l und u müssen noch addiert werden.
342, 14 cm + 48,77 cm = 390,91 cm



Aufgabe 11

263. Wertungsaufgabe

"Nimm 3", hörte Mike als er Bernds Zimmer betrat. "Hallo, was spielt ihr denn da?" "Nimm drei. Und das geht so: Wir haben zu Beginn einen Haufen mit 21 Streichhölzern. Es gibt den Spieler A und den Spieler B. Ein Spielzug besteht darin, dass abwechselnd 1, 2 oder 3 Streichhölzer von dem Haufen entfernt werden. Es gewinnt derjenige, der das bzw. die letzten Hölzer nehmen darf." "Ich verstehe", sagte Mike. "Aber ob das Spiel gerecht ist? Ich habe da so meine Zweifel." "Wie meinst du das?", fragte Bernd. "Nun, ich vermute, dass einer der Spieler den Sieg erzwingen kann."  Eine Bestätigung oder Widerlegung der Vermutung bringt 4 blaue Punkte. Gibt es eine solche Strategie für einen Haufen mit 100 Streichhölzern, wenn man bis zu 7 Hölzer nehmen darf? - 4 rote Punkte

Lösung

Rot und blau betrachtet man am besten vom Ende her.
blau: Will man das letzte nehmen, so geht das, wenn es 18, 19 oder 20 Streichhölzer sind. In diesen Dreierschriiten muss man weiter denken.
Nachzulesen in der Lösung von Rafael, danke. Lösung 263 als pdf



Aufgabe 12

264. Wertungsaufgabe

Mensch ärgere dich nicht "Lass uns zum Schluss der Serie noch einen Blick auf das Mensch ärgere dich nicht Spielfeld werfen." "Na, dann mal los", gab Mike an Bernd zurück. "Wie oft muss ein Spieler mindestens würfeln,um alle Spielfiguren ins Ziel zu bringen, wenn er nicht raus geworfen wird?" "Na gut, aber so viele  Sechsen nacheinander gibt es ja nicht." Klar ist das sehr unwahrscheinlich, aber trotzdem. Also der Spieler Gelb soll es sein. Sein Startfeld hat die Nummer Null. Er muss den Startplatz frei räumen, wenn noch eine Figur draußen ist.
Innerhalb seines Zielbereiches darf er seine Figuren nicht überspringen." "Das wird doch schnell unübersichtlich oder?", gab Mike zu bedenken. "Das stimmt, da sollte am besten mit einer Tabelle gearbeitet werden." 12 blaue Punkte für den Fleiß.
Glück für rot. Der rote Spieler hat eine Figur auf Feld 5. Der Spieler Gelb ist am Zug. Er hat eine Figur auf Feld 1, eine Figur auf Feld 14 und zwei Figuren, die er noch einsetzen kann (muss). Mehr als dreimal eine Sechs soll nicht auftreten. Wie hoch ist die Wahrscheinlichkeit, dass die rote Spielfigur nicht so kurz vor dem Ende bei diesem Spielzug rausgeworfen wird? - 6 rote Punkte.

Lösung


blau: Der Tabelle kann man die Zugfolge entnehmen. Die Zahlen in der Tabelle geben die Stellung auf dem Spielfeld an. Drei Runden muss der Spieler spielen. Na ja so viel Glück wird wohl kaum eintreffen. Die Wahrscheinlichkeit für diesen Verlauf mögen Kenner selber ausrechnen.
Die Zahlen in Klammern sind die von einer Sechs abweichenden Würfelzahl

Wurf Figur 1 Figur 2 Figur 3 Figur 4
1 0      
2 6      
3 6 0    
4 12 0    
5 12 6    
6 12 6 0  
7 18 6 0  
8 18 12 0  
9 18 12 6  
10 18 12 6 0
11 24 12 6 0
12 30 12 6 0
13 36 12 6 0
14 z2 12 6 0
15 Z1 (1) 12 6 0
16   18 6 0
17   24 6 0
18   30 6 0
19   36 6 0
20   z2 6 0
21     18 0
23     24 0
24     30 0
25     36 0
26     z3 (5) 0
27       6
28       12
29       18
30       24
31       30
32       36
33       z4 (4)

Um das Glück für die rote Figur ist bei dieser Aufgabenstellung gemeint, wie groß die Wahrscheinlichkeit, dass Gelb ihn bei dem gerade dran seienden Spielzug nicht raus wirft. Was nach dem Überstehen der ersten Atacke passiert, also in einer nächsten Runde für den Fall, dass rot sich mit einer 5 oder 6 in Zielfeld nicht rettet wird hier nicht untersucht.
Die Möglichkeiten rausgeworfen zu werden,sind :
1. würfeln einer 4
2. würfeln von 6 und 5
3. würfeln von 6,6,6 und 5
Die Wahrscheinlichkeiten addieren sich: 1/6 + 1/6*1/6 + 1/6*1/6*1/6*1/6=253/1296.
Die Wahrscheinlichkeit nicht hinaus geworfen zu werden, ist demzufolge 1 - 253/1296 = 1043/1296 also 80,4 %. Das sieht doch gut aus.

Auswertung Serie 22 (blaue Liste)

Ria Hopke

Platz

Name

Ort

Summe

Aufgabe

 

253

254

255

256

257

258

259

260

261

262

263

264

1.

Rafael Seidel

Chemnitz

75

5

4

10

11

7

5

2

4

6

5

4

12

2.

Doreen Naumann

Duisburg

73

5

4

11

11

5

5

4

4

6

4

2

12

3.

Sabine Fischbach

Hessen

56

5

4

8

6

-

4

-

4

6

5

2

12

4.

Andree Dammann

München

24

-

4

8

8

-

-

-

4

-

-

-

-

5.

Richard Hahmann

Chemnitz

21

-

-

-

8

3

4

4

2

-

-

-

-

6.

Hermann Thum

Chemnitz

20

-

4

4

-

6

4

-

2

-

-

-

-

7.

Loise Reichmann

Chemnitz

19

4

-

-

6

4

4

1

-

-

-

-

-

8.

Linus-Valentin Lohs

Chemnitz

17

-

4

8

-

-

-

-

-

-

5

-

-

8.

Ingmar Richter

Chemnitz

17

-

4

-

2

7

4

-

-

-

-

-

-

8.

Jamila Wähner

Chemnitz

17

-

-

-

10

3

4

-

-

-

-

-

-

9.

Ellen Richter

Chemnitz

16

-

-

-

4

4

4

4

-

-

-

-

-

10.

Anja Posselt

Chemnitz

15

-

-

-

6

3

4

2

-

-

-

-

-

10.

Chemnitz

15

-

-

-

4

3

4

4

-

-

-

-

-

10.

Stephanie Dani

Chemnitz

15

-

-

-

6

4

4

1

-

-

-

-

-

11.

Katrin Posselt

Chemnitz

14

4

4

-

6

-

-

-

-

-

-

-

-

12.

Luise Adam

Chemnitz

13

-

-

-

6

3

4

-

-

-

-

-

-

12.

Marie Sophie Roß

Chemnitz

13

-

-

-

6

3

4

-

-

-

-

-

-

13.

Lisa Grassmann

Chemnitz

11

-

-

-

4

3

-

4

-

-

-

-

-

13.

Christian Wagner

Bamberg

11

5

-

-

-

-

-

-

-

6

-

-

-

14.

Lucas Steinke

Chemnitz

10

-

-

-

6

-

-

-

-

-

4

-

-

15.

Nina Zätsch

Chemnitz

9

-

-

-

6

3

-

-

-

-

-

-

-

16.

Camilla Schreiter

Chemnitz

8

-

4

-

-

-

4

-

-

-

-

-

-

16.

Philipp Fürstenberg

Chemnitz

8

-

-

-

-

4

4

-

-

-

-

-

-

16.

Kai-Lutz Wagner

Chemnitz

8

-

4

-

-

-

-

4

-

-

-

-

-

16.

Carl Geißler

Chemnitz

8

-

4

-

-

-

4

-

-

-

-

-

-

16.

Pauline Marschk

Chemnitz

8

-

4

-

-

-

4

-

-

-

-

-

-

16.

Emma Irmscher

Eibenberg

8

-

4

-

-

-

4

-

-

-

-

-

-

16.

Laura Schlosser

Chemnitz

8

-

4

-

-

-

4

-

-

-

-

-

-

16.

Vincent Baessler

Chemnitz

8

-

4

-

-

-

4

-

-

-

-

-

-

16.

Jonas Frederik Otto

Lichtenwalde

8

-

4

-

-

-

4

-

-

-

-

-

-

16.

Paula Geißler

Chemnitz

8

-

4

-

-

-

4

-

-

-

-

-

-

17.

Andreas M.

Dittersdorf

7

-

-

-

-

-

4

-

-

3

-

-

-

17.

Marion Sarah Zenk

Chemnitz

7

-

-

-

-

3

-

-

4

-

-

-

-

18.

Duncan Mahlendorff

Chemnitz

6

-

-

-

-

2

4

-

-

-

-

-

-

19.

Tim Jechorek

Chemnitz

5

-

-

-

-

-

5

-

-

-

-

-

-

19.

Jana (+S) Schneider

Lugau

5

-

-

-

-

-

-

-

-

-

-

-

5

20.

Pascal Graupner

Chemnitz

4

-

4

-

-

-

-

-

-

-

-

-

-

20.

Lisanne Brinkel

Chemnitz

4

-

4

-

-

-

-

-

-

-

-

-

-

20.

Lena Rabbeau

Chemnitz

4

-

4

-

-

-

-

-

-

-

-

-

-

20.

Sophie Kalmer

Chemnitz

4

-

4

-

-

-

-

-

-

-

-

-

-

20.

Felix Brinkel

Chemnitz

4

-

-

-

-

-

-

-

4

-

-

-

-

20.

Celine Strumpf

Chemnitz

4

-

4

-

-

-

-

-

-

-

-

-

-

20.

Elisa Parsche

Chemnitz

4

-

4

-

-

-

-

-

-

-

-

-

-

20.

Karolin Schuricht

Chemnitz

4

-

4

-

-

-

-

-

-

-

-

-

-

20.

Max Beuckert

Chemnitz

4

-

-

-

-

-

4

-

-

-

-

-

-

20.

Saskia Schlosser

Chemnitz

4

-

-

-

-

-

4

-

-

-

-

-

-

20.

XXX

???

4

-

-

-

-

-

-

4

-

-

-

-

-

20.

Vincent Hartig

Chemnitz

4

-

-

-

-

-

4

-

-

-

-

-

-

20.

Emily Neuwirth

Chemnitz

4

-

-

-

-

-

4

-

-

-

-

-

-

20.

Amarin Roßberg

Chemnitz

4

-

-

-

-

-

4

-

-

-

-

-

-

20.

Felix Karu

Innsbruck

4

-

4

-

-

-

-

-

-

-

-

-

-

20.

Luisa Schlosser

Chemnitz

4

-

4

-

-

-

-

-

-

-

-

-

-

20.

Marcel Seerig

Chemnitz

4

-

-

-

-

-

4

-

-

-

-

-

-

20.

Ernesto Uhlmann

Chemnitz

4

-

4

-

-

-

-

-

-

-

-

-

-

20.

Mara Neudert

Chemnitz

4

-

4

-

-

-

-

-

-

-

-

-

-

20.

Henrike Grundmann

Chemnitz

4

-

-

-

-

-

4

-

-

-

-

-

-

20.

Julia Ritter

Chemnitz

4

-

-

-

-

-

4

-

-

-

-

-

-

20.

Robin König

Chemnitz

4

-

-

-

-

-

-

-

4

-

-

-

-

20.

Tom Hartig

Chemnitz

4

-

-

-

-

-

4

-

-

-

-

-

-

20.

Paula

Hartmannsdorf

4

-

-

-

-

-

4

-

-

-

-

-

-

20.

Jonathan Käßler

Chemnitz

4

-

-

-

-

-

4

-

-

-

-

-

-

20.

Willy Stöckel

Chemnitz

4

-

4

-

-

-

-

-

-

-

-

-

-

20.

Jakob Lenk

Chemnitz

4

-

4

-

-

-

-

-

-

-

-

-

-

20.

Lukas Kirchberg

Chemnitz

4

-

-

-

-

-

4

-

-

-

-

-

-

20.

Max Steinert

Chemnitz

4

-

4

-

-

-

-

-

-

-

-

-

-

20.

Benedikt Hastedt

Chemnitz

4

-

4

-

-

-

-

-

-

-

-

-

-

20.

David Kahle

Chemnitz

4

-

4

-

-

-

-

-

-

-

-

-

-

21.

Ellen Wilde

Chemnitz

1

-

1

-

-

-

-

-

-

-

-

-

-

Auswertung Serie 22 (rote Liste)

Platz

Name

Ort

Summe

Aufgabe

 

253

254

255

256

257

258

259

260

261

262

263

264

1.

Doreen Naumann

Duisburg

60

5

2

4

4

6

5

4

4

8

10

2

6

2.

Sabine Fischbach

Hessen

42

5

2

4

3

-

4

4

4

6

8

2

-

3.

Christian Wagner

Bamberg

13

5

-

-

-

-

-

-

-

8

-

-

-

4.

Andree Dammann

München

9

-

2

3

-

-

-

-

4

-

-

-

-

5.

Rafael Seidel

Chemnitz

8

-

-

-

-

-

-

-

-

-

4

4

-

6.

Ingmar Richter

Chemnitz

7

-

-

-

-

5

-

-

-

2

-

-

-

7.

Marion Sarah Zenk

Chemnitz

6

-

-

-

-

-

-

2

4

-

-

-

-

8.

Marie Sophie Roß

Chemnitz

5

-

-

-

-

1

-

4

-

-

-

-

-

8.

Lisa Grassmann

Chemnitz

5

-

-

-

-

1

-

4

-

-

-

-

-

8.

Jana (+S) Schneider

Lugau

5

-

-

-

-

-

-

-

-

-

-

-

5

8.

Richard Hahmann

Chemnitz

5

-

-

-

-

1

-

4

-

-

-

-

-

9.

Loise Reichmann

Chemnitz

4

-

-

-

-

-

4

-

-

-

-

-

-

9.

Stephanie Dani

Chemnitz

4

-

-

-

-

-

4

-

-

-

-

-

-

9.

Kai-Lutz Wagner

Chemnitz

4

-

-

-

-

-

-

4

-

-

-

-

-

9.

XXX

???

4

-

-

-

-

-

-

4

-

-

-

-

-

9.

Katrin Posselt

Chemnitz

4

-

2

-

2

-

-

-

-

-

-

-

-

10.

Hermann Thum

Chemnitz

3

-

-

-

-

-

-

3

-

-

-

-

-

10.

Jamila Wähner

Chemnitz

3

-

-

-

2

1

-

-

-

-

-

-

-

11.

Felix Karu

Innsbruck

2

-

2

-

-

-

-

-

-

-

-

-

-

11.

Julia Ritter

Chemnitz

2

-

-

2

-

-

-

-

-

-

-

-

-

11.

Emily Neuwirth

Chemnitz

2

-

-

2

-

-

-

-

-

-

-

-

-

11.

Tim Jechorek

Chemnitz

2

-

-

2

-

-

-

-

-

-

-

-

-

11.

Anja Posselt

Chemnitz

2

-

-

-

2

-

-

-

-

-

-

-

-

12.

Nina Zätsch

Chemnitz

1

-

-

-

-

1

-

-

-

-

-

-

-

12.

Carl Geißler

Chemnitz

1

-

-

1

-

-

-

-

-

-

-

-

-

12.

Philipp Fürstenberg

Chemnitz

1

-

-

-

-

1

-

-

-

-

-

-

-

12.

Luise Adam

Chemnitz

1

-

-

-

-

1

-

-

-

-

-

-

-

Valid XHTML 1.0 Transitional

Serie-21

Serie 21
Aufgaben und Lösungen

Aufgaben und Lösungen Serie 21

Mathematik und Sport

Aufgabe 1

241. Wertungsaufgabe

"Das war letzte Woche eine recht sportliche Aufgabe", meinte Maria. "Das stimmt." "In diesem Jahr ist ja die Leichtathletik-WM und so wird in unserer Gruppe das Thema Mathematik und Sport aufgegriffen werden", sagte Maria. "Klingt richtig gut, da will ich mal hoffen, dass sich unser Ausdenker mal eine ganze Serie zu dem Thema überlegt - „geht klar", murmelte Mike vor sich hin. Das hörte Bernd, der gerade ins Zimmer kam und rief ganz aufgeregt: "Dann lasst uns diese sportliche Logelei angehen, die sich aus einem Interview mit Karl Berg ergibt." Karl Berg fuhr im Jahr 2008 zu den olympischen Spielen und traf dort vier Freunde, die er aus verschiedenen Trainingslagern kannte. In dem Interview ging es mit den Vor- und Nachnamen, sportlichen Disziplinen (jeder hatte eine andere) und sonstigen Informationen etwas durcheinander. Sportfreund Beier heißt nicht Emil. Emil ist Speerwerfer. Dann gibt es noch den Sportler Monk. Der Stabhochspringer ist der Herr Jung. Es gibt einen Zehnkämpfer. Arno Meyer ist kein Hochspringer. Einer heißt Fred und Bert ist Weitspringer. (Für 6 blaue Punkte ist herauszufinden, wie die Sportler mit vollständigem Namen heißen und welche Sportart sie betreiben.)
Jeder der Sportler ist in einem anderen Haus untergebracht (1 bis 5 -- von links nach rechts). Hinzukommt, dass jeder der Sportler in verschiedenen Jahren eine Sportschulausbildung beendete (2003, 04, 05, 06 und 07). Der Sportler aus Haus 1 beendete die Sportschule eher als der, der direkt neben ihm untergebracht ist. Emil war genau ein Jahr eher fertig als der, der direkt rechts neben ihm wohnt. Bert war ein Jahr eher fertig als Fred, allerdings später als der Zehnkämpfer, der wiederum direkt links neben dem wohnt, der seine Ausbildung 2005 beendete. Arno war eher fertig als Fred, jedoch genau ein Jahr eher als der Nachbar aus Haus 4. - Wer wohnt in welchem Haus und beendete wann seine Ausbildung an einer Sportschule? - 6 rote Punkte.
Anmerkung, diese Logikaufgabe ist im Stil von Einstein's Weihnachtsrätsel: die Aufgabe

Lösung

Die Lösung von Doreen, danke
blaue Punkte:
-Tabelle erstellen mit Vorname, Familienname, Sportart (Tabellenkopf: Vornamen: Arno, Bert, Emil, Fred, Karl)
-Eintragen der gegebenen Dinge: Karl-Berg, Emil-Speerwerfen, Arno-Meyer, Bert-Weitsprung
-Angabe Hr.Jung+Stabhochsprung->kann nur Fred sein, da bei allen anderen Familienname oder Sportart schon bekannt
-Karl Berg muss Hochsprung sein, da Arno Meyer nicht Hochsprung und sonst alle anderen Sportarten bekannt
-letzte Sportart Zehnkampf->nur noch Arno Meyer frei
-Emil heißt nicht Beier->Bert muss Beier heißen und Emil Monk
=>Arno-Meyer-Zehnkampf
Bert-Beier-Weitsprung
Emil-Monk-Speerwerfen
Fred-Jung-Stabhochsprung
Karl-Berg-Hochsprung

rote Punkte:
-Tabelle erstellen mit Vorname, Jahr und Hausnummer (Tabellenkopf: Jahr 2003-2007)
-Aufschreiben der bekannten Angaben:
-Haus 1 nicht 2007 -Bert nicht 2007
-Haus 2 nicht 2003 -Bert nicht 2003
-Haus 5 nicht 2003 -Fred nicht 2003
-Haus 4 nicht 2003 -Arno(Zehnkämpfer) nicht 2007
-Emil nicht 2007 -Arno nicht Haus 2
-Emil nicht Haus 5 -Arno nicht Haus 4
-Aufschreiben der Reihenfolge(nach Jahren):
-Arno-Bert-Fred (zwischen Bert+Fred 1Jahr)
-Haus1-Haus2
-Arno-2005
-Arno-Haus4-Fred (zwischen Arno+Haus4 1Jahr)
-Arno muss 2003 oder 2004 sein, da nicht 2005 und auch nicht später(siehe Reihenfolge)
nun habe ich solange die Annahme Arno+2003 überprüft, bis sich ein Widerspruch ergab->damit konnte Arno nur 2004 sein
->Tüfteln an den übrigen Angaben bis zum Ergebnis:
2003-Karl-Haus1
2004-Arno-Haus3
2005-Emil-Haus4
2006-Bert-Haus5
2007-Fred-Haus2
noch einmal eine Zusammenfassung von Christian (Bamberg), danke
Haus 1 Karl Berg, Hochsprung, 2003
Haus 2 Fred Jung, Stabhochsprung, 2007
Haus 3 Arno Meyer, Zehnkampf, 2004
Haus 4 Emil Monk, Speerwerfer, 2005
Haus 5 Bert Beier, Weitsprung, 2006



Aufgabe 2

242. Wertungsaufgabe

"Ich habe mir mal die Wertung in der Fußballbundesliga der letzten Saison angeschaut und überlegt, ob wohl die Punkteverteilung einen Einfluss auf den Tabellenplatz hat," fragte Bernd. "Fußball ist nicht so meins", meinte Mike, "aber gut lass uns das doch mal durchgehen. Es ist ja so, dass eine Mannschaft für einen Sieg (G) 3 Punkte für ein Unentschieden (U) 1 und bei Verloren (V) keinen Punkt erhält."
8 blaue Punkte für die Untersuchungen zur Platzierung bei einer Verteilung GUV mit 2;1;0 bzw. 3;2;0.

 

Verein

Sp

G

U

V

1.

FC Bayern München

34

22

10

2

2.

Werder Bremen

34

20

6

8

3.

FC Schalke 04

34

18

10

6

4.

Hamburger SV

34

14

12

8

5.

VfL Wolfsburg

34

15

9

10

6.

VfB Stuttgart (M)

34

16

4

14

7.

Bayer 04 Leverkusen

34

15

6

13

8.

Hannover 96

34

13

10

11

9.

Eintracht Frankfurt

34

12

10

12

10.

Hertha BSC

34

12

8

14

11.

Karlsruher SC (N)

34

11

10

13

12.

VfL Bochum

34

10

11

13

13.

Borussia Dortmund

34

10

10

14

14.

Energie Cottbus

34

9

9

16

15.

Arminia Bielefeld

34

8

10

16

16.

1. FC Nürnberg (P)

34

7

10

17

17.

Hansa Rostock (N)

34

8

6

20

18.

MSV Duisburg (N)

34

8

5

21


"Elfmetertore zählen in der Bilanz nicht extra, sind aber sehr wichtig," sagte Maria, die gerade mit Lisa zu Bernd und Mike ins Zimmer trat. "Ich glaube, so ein Torwart hat eigentlich keine Chance, den Ball zu bekommen, wenn ein Spieler den in die obere linke oder rechte Ecke zirkelt", warf Lisa ein. "Tja, dann lass uns das mal überlegen. Der Fußball hat einen Durchmesser von etwa 23 cm. Beim Strafstoß ist der Schütze vom 11 (10,92) m von der Torlinie entfernt. Das Tor hat als Innenmaße etwa 7,32 m Breite und 2,2 m Höhe. Der Torwart braucht etwa 1 Sekunde (oder mehr) bis in so eine Ecke. Die Geschwindigkeit des Balls liegt bei 72 bis 100 km/h." 4 rote Punkte um zu entscheiden, ob da eigentlich eine Chance besteht oder nicht.

Lösung

Bei der blauen Aufgabe war dieses mal ja lediglich nachzurechnen, ob sich bei der veränderten Punkteverteilung eine andere Platzierung ergibt. Dies ist durchaus der Fall. Zum Selberrechnen mit einer beliebigen Punktevorgabe bitte das script nutzen:
Bundesliga
Wie man leicht sieht, ändern sich nur die mittleren Plätze. Allerdings wird bei 2;1,0 das Torverhältnis deutlich entscheidender.
Für den roten Teil die Lösung von Doreen, danke
Bildung eines Quaders mit den Seiten a (halbe Torbreite), b (Entfernung vom Elfmeterpunkt bis zum Tornetz) und c (Torhöhe)
a=7,32m/2=3,66m
b=10,92m+2m=12,92m
c=2,2m
Berechnung der Quaderdiagonale e
e=Wurzel(a²+b²+c²)
e=13,607424m
das Ergebnis runden wir großzügig auf 13m, da der Ball ja nicht genau in die Torecke passt mit seinen 23cm Durchmesser
-Berechnung der Zeit, die der Ball für die Entfernung e braucht (bei 72km/h)
x=e*3600s:72000m/s
x=0,65s
=>Bei einer Reaktionszeit von 1s hat der Torwart bei einem Schuss in die oberen Ecken keine Chance, den Ball zu fangen.
Ich habe auch die Zeit berechnet, wenn der Ball auf einer Seite knapp hinter der Latte ins Netz geht(also nicht bis nach hinten in die Ecke) - bis dahin vergehen ca.0,55s also auch (rein rechnerisch) keine Chance.
Anmerkung: Alle Lösung, die zeigen, dass es keine eigentlich Chance gibt, gelten als richtig. Es zeigt sich also, dass es nur am Können (Wollen?) der Spieler liegt, ob es klappt, den Ball in eine der oberen Ecken zu befördern und so den Elfmeter erfolgreich abzuschließen.



Aufgabe 3

243. Wertungsaufgabe

Siegerpodest"Wenn ich mir die Siegerehrungen so anschaue, so bin ich immer wieder verblüfft wie viele Varianten von Siegerpodesten es gibt," meinte Mike. "Da hast du Recht. Es gibt runde, längliche, welche aus Holz, aus Metall usw.," erinnerte sich Maria, als sie an die letzten Olympischen Spiele dachte. Auf dem Bild ist ein recht einfaches zu sehen. Die Flächen oben sind quadratisch (40 x 40 cm). Die Höhe liegen bei 20, 40 bzw. 60 cm. Wie groß sind Oberfläche und Volumen dieses Podestes? (5 blaue Punkte).
Siegerpodest 2 "Schaut mal meine Skizze an. Ich habe aus den Quadraten möglichst große regelmäßige Achtecke gemacht. So gefällt mir das Siegerpodest besser," sagte Lisa. "Nicht schlecht", staunte Bernd. Wie groß sind Oberfläche und Volumen dieses Podestes? (5 rote Punkte).

Lösung

Die blaue Aufgabe:
Oberfläche: Hier bietet sich ein "Flug" um das Podest an.
Von oben und unten ist letztlich jeweils ein Rechteck zu sehen (40 x 120) x 2 = 9600 cm².
Von links und rechts ist letztlich jeweils ein Rechteck zu sehen (40 x 60) x2 = 4800 cm².
Von vorn und hinten ist letztlich jeweils ein Rechteck zu sehen, wenn man einfach den Platz 3 auf den Platz 2 "stellt" (80 x 60) x 2 = 9600 cm².
Das sind zusammen 24000 cm² bzw. 2,4 m².
Beim Volumen ist auch das "Stellen" von Platz 3 auf 2. angebracht. Dann hat man einen Quader V = a x b x c = 80 cm x 40 cm x 60 cm = 192000 cm³ = 0,192 m².
Die rote Aufgabe:
Zuerst wird die Kantenlänge s der Achtecke gebraucht. Von den 40 cm des Quadrates müssen zwei Stücke der Länge x abgeschnitten werden.
s = 40 - 2x
x = (40 - s)/2 An den Ecken ist aber auch der Satz des Pythagoras anwendbar s² = x² + x² = 2x²
Die obige Formel wird in die zweite eingesetzt.
s² = 2* ((40 - s)/2)²
...
s² = 800 - 40s s²/2
...
0 = s² + 80 - 1600
s 1,2 = - 40 ± Wurzel ((1600 + 1600))
negatives Ergebnis entfällt.
s = 16,5685425 cm ≈ 16,57 cm.
Formel 243Aus der allgemeinen Flächeninhaltsformel für regelmäße n-Ecke (n > 2)lässt sich das Achteck schnell finden. (a - Seitenlänge)
Die Herleitung ist mittels Inkreisradius, Satz des Pythagoras und Tangens relativ einfach.
Das a in der Formel ist das oben ausgerechnete s.
A = 1325,48 cm²
Volumen (Stellen" von Platz 3 auf 2) ⇒ V = 2 · A · h = 2 · 1325,48 cm² · 60 cm = 159058 cm³ = 0,159 m³
Interessant wird nun wieder die Oberfläche. Oben und unten zusammen sind das 6 Achtecke - kein Problem.
Nun die Rechtecke: Platz zwei 7 Rechtecke (40 cm hoch) Platz drei 7 Rechtecke (20 cm hoch) ⇒ 7 Rechtecke (60 cm hoch)
Platz eins 6 Rechtecke (60 cm hoch) + links 1 Rechteck 20 cm + rechts 1 Rechteck 40 cm ⇒ 7 Rechtecke (60 cm hoch)
Alles zusammen AO = 6 · A + 14 · s · h = 6 · 1325,48 cm² + 14 · 16,57 cm · 60 cm = 21871,7 cm² = 2,18 m²



Aufgabe 4

244. Wertungsaufgabe

"Wir hatten uns doch letzte Woche  über die Siegerpodeste unterhalten. In dem Zusammenhang sind mir noch folgende Fragen eingefallen", sagte Mike, der gerade mit Lisa bei Maria und Bernd war. "Lass hören." "Bei einem 400 m Endlauf gibt es meistens 8 Teilnehmer. Wie viele verschiedene Startaufstellungen gäbe es, wenn man die Bahnen per Los verteilen würde? (3 blaue Punkte). Wie viele verschiedene Siegerehrungen könnte es bei so einem Rennen geben, wenn es keine Doppelplatzierungen gibt. (2 rote Punkte)?"

Lösung

Der Fachbegriff für die blaue Aufgabe ist Permutation ohne Wiederholung.
Für die Bahn 1 stehen 8 Starter zur Verfügung. (8x) Wenn der Starter feststeht, dann habe ich noch 7 Starter (7x), usw.
Es gibt also 8*7*6*5*4*3*2*1 = 40320 Startaufstellungen. (mathematische Kurzform 8!)
Der Fachbegriff für die rote Aufgabe ist Variation (ohne Zurücklegen) - eine Auswahl unter Berücksichtigung der Reihefolge.
Die Überlegung ist wie bei der blauen Aufgabe. jeder der 8 Starter kann Sieger sein. Dann verbleiben für Platz noch 7 und Platz 3 sind es noch sechs Starter.
Es gibt also 8*7*6 = 336 verschiedene Siegerehrungen.
Der fehlende Fachbegriff ist die Kombination. Eine Auswahl von drei Sportlern von insgesamt acht, wobei es auf die Reihenfolge nicht ankommt.
Bei der Siegerehrung stellt Starter 1 , Starter 2 und Starter 3 eine andere Reihefolgen dar wie Starter 2 , Starter 1 und Starter 3. Wenn ich hingegen sage, drei Leute treffen sich bei der Dopingprobe ist die Auswahl 1;2;3 die gleiche wie 2;1;3. Somit gibt es (8*7*6)/(3*2*1)= 56 Kombinationen.
Die Kombination und die Variation wurde von einigen Teilnehmern verwechselt.



Aufgabe 5

245. Wertungsaufgabe

"Ich möchte den 400 m Lauf noch einmal unter die Lupe nehmen." "Na dann mal los," meinte Bernd zu Mike. "Wie schafft man es, dass alle Teilnehmer wirklich 400 m laufen?. Dazu musst du Folgendes wissen: Die Laufbahn besteht aus zwei parallelen Geraden zu je 84,40 m Länge und zwei verbindenden Halbkreiskurven mit je 36,50 m Radius. Das bezieht sich auf die innere Kante." "Warte mal, wenn ich da mal schnell mit dem Taschenrechner nachrechne, komme ich aber nicht auf 400 m." (Wie viel m sind das? - 3 blaue Punkte). "Du musst bedenken, dass die Sportler nicht genau auf der Innenkante laufen, sondern in einem gewissen Abstand." Wie groß (in vollen cm) muss der Abstand sein, damit ziemlich genau 400 m herauskommen? -( noch mal 3 blaue Punkte.)
"Alles klar, das war die Innenbahn, aber wie sieht es für den Läufer auf Bahn 2 aus. Hier wieder ein Ausschnitt aus der Vorschrift: Die Bahnen sind 1,22 m breit anzulegen einschließlich der 5 cm breiten rechten Grenzlinie. Ab Bahn 2 wird die Nennmesslänge mit 20 cm von der linken Begrenzung bestimmt, da man an die Strichmarkierung dichter heran laufen kann als an die Innenkante." Wie viel Meter Vorsprung bekommt der Läufer auf Bahn 2, damit er den Kurvennachteil ausgleichen kann. (4 rote Punkte - Wer die Werte für die restlichen Bahnen ausrechnet, bekommt jeweils noch zwei Punkte dazu)

Lösung

Die Lösung von Sabine Fischbach, danke.
Zur Berechnung der Strecke benötigt man den Kreisumfang U
U = 2 Pi mal r
U = 229,32 m
Zuüglich der beiden Geraden kommt man auf insgesamt 398,14 m, also fehlen 1,86 m (oder 186 cm) an 400m.
Rechnet man also rückwärts und zieht von den 400 m die beiden Geraden (zusammen 168,80 m) ab, verbleiben 231,20 m
Das teilt man durch 2 Pi (6,2831), um den Kreisradius zu errechnen. Als Ergebnis bekommt man hier 36,80 m.
Der Läufer muss also einen Abstand von etwa 30 cm zur Innenkante haben, um pro Runde volle 400 m zu laufen.
Den Nachteil für die Außenbahnen zu berechnen ist ja so schwer nicht. Die geraden Strecken ändern sich ja nicht. Nur die zwei Kurven - bei den 400 m - also ein voller Kreis. Was der zu lang ist, muss ausgeglichen werden.
Bahn 2 Radius 36,50 m + 1,22 m (Bahnbreite) + 0,2 m (Abstand zur Bahnkante) => 407,04 m. Also bekommt der Starter auf Bahn 2 einen Kurvennachteilsausgleich von 7,05 m.
Bahn 3 Radius 36,50 m + 2*1,22 m (Bahnbreite) + 0,2 m (Abstand zur Bahnkante) => 414,71 m. Also bekommt der Starter auf Bahn 2 einen Kurvennachteilsausgleich von 14,7 m.
Mit jeder weiteren Bahn nimmt der Kurvennachteilsausgleich um 2*π*1,22 m = 7,665 m zu.
Damit kommt man ganz schnell auch zur DLV-Tabelle:
Bahn 4: 22,37 m
Bahn 5: 30,03 m
Bahn 6: 37,70 m
Bahn 7: 45,36 m
Bahn 8: 53,03 m
Anmerkungen: Beim 800 m - Lauf (4 Kurven) ist zu beachten, dass nur eine Kurve zählt, weil der nach der ersten Kurven, alle Starter auf die Innenbahn wechseln. Bei der 4 x 400 m Staffel fallen 3 Kurven ins Gewicht. Der Wechsel nach innen ist dann mittels Pythagoras nachvollziehbar.



Aufgabe 6

246. Wertungsaufgabe

Handball"Nach so viel Rennerei im Stadion habe ich wieder mal den Handballern zugesehen. Da geht es flott zur Sache, wobei es meiner Mannschaft zur Zeit nicht so gut geht," sagte Bernds Opa, der wieder mal zu Besuch war. Aber der Rechtsaußen von denen ist einfach Spitze." "Was ist denn ein Rechtaußen?", fragte Maria. "Hier schau mal auf das Bild des Spielfeldes. Als Verteidiger vor seinem Tor steht er etwa da, wo ich den schwarzen Punkt gesetzt habe. Ist er in der angreifenden Mannschaft, entspricht das dem roten Punkt." "Alles klar." "Sag mal Opa, der Bereich wo nur der Torwart hin darf, das ist doch gar kein Halbkreis, oder?," fragte Bernd nach. "Da hast du Recht, da es Mindestabstände zum Tor gibt, ist das keine Kreislinie, auch wenn die Sportreporter häufig von Kreisspielern sprechen - die Trainer aber auch." (Wie groß ist der Torraum (in m²) - 6 blaue Punkte. Wie groß ist die Fläche zwischen Torraumlinie und Freiwurflinie - 8 rote Punkte)

Lösung

blau: Der Toraum setzt sich aus zwei Viertelkreisen (r = 6 m und einem Rechteck (a = 3 m; b = 6 m) zusammen.
A = π/2 · r² + a · b = 74,5 m²
rot: die Fläche zwischen Torraumlinie und Freiwurflinie setzt sich zusammen aus einem Rechteck A1(vor dem Tor) a = 3 m und b = 9 m. Dazu kommen zwei Halbkreise A2(r = 9 m), von diesen aber muss links und rechts jeweils ein halber Kreisabschnitt A3 (Höhe h = 0,5 m), auch Kreissegment genannt, abgezogen werden. Schließlich muss noch das Ergebnis der blauen Aufgabe A4 abgezogen werden.
A1 = π/2 · r² = 127,234 m²
A2 = a · b = 27 m²
A3 = r² · arccos (1 - h/r) - (r-h) · Wurzel (2rh - h²) = 1,98 m²
Es ist mit A3 ein ganzes Segment ausgerechnet worden, da ja links und recht je ein halbes vorkommt.
A4 = 74,5 m²
A = A1 + A2 - A3 - A4 = 77,7 m²



Aufgabe 7

247. Wertungsaufgabe

"Hallo Mike, hast du gestern den Dartwettbewerb gesehen?", fragte Bernd. "Nein, das ist nicht so mein Ding, wobei die Geometrie der Dartscheibe an sich schon recht interessant ist." "Dann lass uns mal den Aufbau und die Abmessungen betrachten."
Dartscheibe Double- und Triple-Ring (Innenmaß) 8 mm
Durchmesser des Bull's Eye (Innenmaß) 12,7 mm
Größe des gesamten Bull (Innenmaß) 31,8 mm
Entfernung vom äußeren Draht des Doublerings zum Bull 170 mm
Entfernung vom äußeren Draht des Triplerings zum Bull 107 mm
Durchmesser des äußeren Rings des Doubledrahts 340 mm
Die Stärke des Drahtes liegt zwischen 1,22 mm und 1,63 mm
"Macht die Einbeziehung der Drahtstärke unsere gewohnten Berechnungen nicht recht kompliziert?", fragte Lisa, als sie die beiden über der Dartscheibe grübeln sah. "Das stimmt, lass uns aber erst einmal was anderes überlegen."
Die Variante 301.
Bei dem Spiel 301 muss ein Spieler mit entweder 3 oder 6 oder 9, 12, ... Würfen die Zahl 301 erreichen. Hat er die Zahl nicht erreicht, muss er noch drei Würfe machen und die nächsten Zahlen dazu addieren. Wirft er daneben oder überschreitet er die 301, so ist die Runde verloren.
Für vier blaue Punkte ist eine Trefferfolge zu finden, mit der genau 301 Punkte erreicht werden, wobei keines der Felder doppelt genutzt werden darf, für die "5" heißt dies, es gibt von innen nach außen 4 unterscheidbare Felder. 5 innen, Treble 5 (=15 Punkte) 5 außen und 5 double (=10 Punkte). (Achtung, es muss eine durch drei teilbare Zahl von Treffern sein.)
Wie viele Treffer muss ein Spieler mindestens ins Ziel bringen, um auf die 301 zu kommen? (mehrfache "Nutzung" eines Feldes ist möglich, die Bedingung der Teilbarkeit besteht natürlich weiterhin) - 3 rote Punkte

Lösung

rot: Die höchste Punktzahl, die in einenm Feld erreicht werden kann, triple 20 = 60. Mit dreimal 60 erreicht man die geforderten 301 nicht. Es werden also noch drei Würfe benötigt.
Lösung von Felix Karu, danke: 60+57+54+51+45+34=301
Lösung von Doreen N., danke: 5xTriple 20 + 1x1außen=5x60+1=301
Lösung von Sabine F., danke: 1) Innerbull - 50 2) 19 (x2) = 38 3) 20 (x3) = 60 4) 18 (x3) = 54 5) 17 (x3) = 51 6) 16 (x3) = 48 ==> 301
Lösung von Juliane H., danke: 60 (Treble 20, 60) + 50 (inner Bull, 110) + 45 (Treble 15, 155) + 60 (Treble 20, 215) + 50 (inner Bull, 265) + 36 (Treble 12, 301)
...
blau: Es gibt sehr viele Möglichkeiten mit einer durch drei teilbaren Zahl von Treffern, die 301 zu erreichen.
Das es mit der Minmalmalzahl von 6 verschiedenen Würfen geht zeigt die obige Lösung von Felix K, danke.
weitere:
Juliane H., danke: 50 (inner Bull, 50) + 25 (outer Bull, 75) + 20 (inneres Feld 20, 95) + 60 (Treble 20, 155) + 20 (äußeres Feld 20, 175) + 40 (Double 20, 215) + 10 (inneres Feld 10, 225) + 30 (Treble 10, 255) + 10 (äußereseres Feld 10, 265) + 20 (Double 10, 285) + 15 (inneres Feld 15, 200) + 1 (inneres Feld 1, 301)
Tim J., danke: erst habe ich alle möglichen treffer der 20 adiert was 140 ergibt. Dann habe ich alle möglichen Treffer der 19 addiert was 133 sind dann habe ich beides zusammen gerechnet was 273 ergibt. Dann muss mann die Doppel 10 treffen danach die 5 dann die 2 dann die 1 und alles zusammen sind 301 Also ist die Trefferfolge:
20 - 20 - Double 20 - Trible 20 - 19 - 19 - Double 19 - Trible 19 - Double 10 - 5 - 2 - 1 = 12 Treffer und 301 Punkte
Andree D., danke:
1. Innerbull ==> 50 ==> Summe 50
2. Outerbull ==> 25 ==> Summe 75
3. Triple 20 ==> 60 ==> Summe 135
4. Double 20 ==> 40 ==> Summe 175
5. Triple 19 ==> 57 ==> Summe 232
6. Triple 18 ==> 54 ==> Summe 286
7. 1 ==> 1 ==> Summe 287
8. 4 ==> 4 ==> Summe 291
9. 10 ==> 10 ==> Summe 301

... und noch mehr Varianten gab es ...



Aufgabe 8

248. Wertungsaufgabe

Dartscheibe "Nachdem wir die Zahlen auf der Dartscheibe unter die Lupe genommen haben, könnten wir ja mal noch die Scheibe geometrisch untersuchen", meinte Mike. "Na dann mach mal einen Vorschlag." "Damit die unterschiedlich großen Drahtstärken nicht die Berechnung verkomplizieren, soll nach den Innenmaßen (s. Aufgabe 247) sofort das nächste Feld beginnen, analog gilt das für die Außenmaßangaben. Wie groß ist der prozentuale Anteil von Feldern (bezogen auf die  Fläche der gesamten Scheibe, wo eine durch 5 teilbare Punktzahl erreicht werden kann." "Oh das ist aber nicht ganz einfach", gab Maria zu bedenken. "Ich denke aber, dass es trotzdem möglich ist, dass eure Spezialgruppe dies herausbekommt". 8 blaue Punkte.
"Ich habe mir mal die Verdopplungs- und Verdreifachungsbereiche angeschaut und finde, so ganz gerecht ist das nicht," sagte Lisa. "Wie meinst du das?", fragte Bernd zurück. "Na, schau mal, die Felder für die einfache Punktzahl sind einfach zu groß. Wenn sich die Punkte eines Sektors (z.B. für die 20) wie 1:2:3 verhalten, so sollten sich die Trefferflächen wie 3:2:1 verhalten." "Das würde ich auch gut finden", bemerkte Maria. Wie würde die Geometrie eines Sektors aussehen, wenn die geforderte Gerechtigkeit durch die Veränderung der Double- und Triple-Ring (Innenmaß), aber nicht durch die Veränderung der sonstigen Abmessungen erreicht werden soll. 8 rote Punkte.

Lösung

blaue Aufgabe: Durch 5 teilbar sind alle Sektoren, in denen die Zahlen 5, 10, 15 und 20 vorkommen, inklusive der darin enthaltenen Triple und Double. Hinzu kommen Inner- und Outerbull. Alle anderen Sektoren entfallen, da diese Grundzahlen enthalten, die nicht durch 5 teilbar sind und die Verdopplung bzw. Verdreifachung bringt keine Teilbarkeit durch 5.
Die Sektorenfläche ist 1/5 aller Sektoren (Sektoren im Sinne eines Kreisringes um die Bulls).
1/5 · π · (ra² -ri²) = 1/5 · π · (170² mm² - 15,9 ² mm²) = 17999,5 mm²
zu diesem Werte kommt noch die Gesamtfläche der Bulls. π · ri² = 794,2 mm²
Die durch 5 teilbare Fläche ist insgesamt 18793,78 groß. Die Fläche der gesamten Scheibe ergibt sich aus: π · ra²
Der Anteil ist dann elementar berechenbar und und liegt bei 20,69 %.
rote Aufgabe. Hier ist die Aufgabenstellung möglicherweise nicht ganz eindeutig. Deshalb wird hier so vorgegangen.:
Bulls bleiben. Damit ist die einzuteilende Fläche der Kreisring um das Bull.
π · (ra² -ri²)
π · (170² mm² - 15,9 ² mm²)
89997,8 mm²
Diese Fläche muss so einteilt werden, dass Trebble : Double : einfach = 1 : 2 : 3 gilt. Das lässt sich auch als 1/6 : 2/6 : 3/6 schreiben.
14999,6 : 29999,26 : 44998,9
Double liegt außen ==> 29999,26 mm² = π (ra² - rt²) = π (170² mm² - rd²)
Die Gleichung lässt sich leicht nach rd (rd Abstand bis zum Doublering) auflösen. rd = 139,1 m, d.h. der Doublering müsste (170 - 139,1) 30,9 mm breit sein.
Der äußere Ring des Treble hat einen Abstand von 107 mm zum Bull und damit einen ra von 107 mm.
14999,6 mm² = π (ra² - rd²) = π (107² mm² - rt²)
(rt Abstand bis zum Treblelering)
rt = Wurzel ((π107² - 14999,6)/π) = 81, 69 mm ==> der Treblering müsste (107 - 81,7) 25,3 mm breit sein.
Anmerkung: Man kann natürlich auch noch die Bullflächen anpassen, so dass dort die Gerechtigkeit für die 50 bzw. 25 Punkte herrschen.



Aufgabe 9

249. Wertungsaufgabe

Bernd kam nach Hause, griff sich die Zeitung und konnte folgendes im Sportteil lesen.
Der Dartverein "volle Scheibe" sucht einen neuen Trainer. Einer von  den fünf Bewerbern ist der allseits bekannte Ole King. Dann war da  noch von einem Blume die Rede so wie von dem nicht 37-jährigen Bruno Sack, dieses Alter traf auf einen anderen Bewerber zu. Der 42-jährige Vincent hieß nicht Seidel. Dann war noch von Ingo die Rede. Erstaunt musste Bernd  lesen, dass der sich bewerbende Tom schon 36 Jahre alt war, er hatte  ihn jünger geschätzt. Der Held war 2 Jahre älter als Tom, der älteste  Bewerber war gar 43. Für 5 blaue Punkte lassen sich die Vornamen,  Namen und das Alter ermitteln.
Alle Kandidaten kamen an einem Tag, aber zu verschiedenen Zeiten zum Vorstand des Vereins und berichteten von ihrer Aufenthaltsdauer bei ihrem jeweils letzten Verein. Der als Vierter kam war 5 Jahre in  seinem Verein gewesen. Auch der Held kam nicht als Erster. Der  37-jährige war nicht 4 Jahre in seinem Verein gewesen. Der zweite  Kandidat war Vincent. Etwas durcheinander kam unser Bernd als er  hinter den Sinn der letzten Informationen kommen wollte. Entweder  war Ole der Erste und Ingo der Dritte oder umgekehrt. Einer von  denen war 4 Jahre bei seinem Verein, der andere ganze 7 Jahre. 8 Jahre lang war der Herr Sack bei seinem Verein, während die längste  Trainerzeit sogar bei immerhin 10 Jahren lag. Wer kam in welcher  Reihenfolge zum Vorstellungsgespräch und hatte welche  Anstellungszeit zu verbuchen? (6 rote Punkte)

Lösung

Hier die Lösung von Sabine F., danke:
BLAUER TEIL
Zunächst mache ich Bestandsaufnahme:
Bernd ist der Leser der Zeitung. Des Weiteren geht es um 5 Personen, deren Namen und Alter gefunden werden soll.
Bekannt ist:
- Ole King
- Herrn Blume
- Bruno Sack (nicht 37 Jahre alt)
- Vincent (nicht Seidel, 42 Jahre alt)
- Ingo
- Tom (36 Jahre alt)
- Herr Held (2 Jahre älter als Tom)
- der älteste Bewerber ist 43

Folgendes kann man kombinieren:
Herr Held ist 38 Jahre alt. Daher kann er mit Vornamen nicht Vincent (42) oder Tom (36) heißen, sondern muss - da die anderen Kombinationen (Ole King und Bruno Sack) schon feststehen - Ingo heißen. Dann gibt es noch die Nachnamen BLUME und SEIDEL. Wenn Vincent nicht Seidel heißt, dann muss er Blume heißen. Also heißt dann Tom Seidel.
Zum Alter: Wenn Bruno nicht 37 ist, dann muss er 43 sein, denn Vincent ist 42, Ingo 38 und Tom 36 Jahre alt. Somit muss Ole 37 Jahre alt sein. Die Lösung lautet also:
1) Ole King (37)
2) Bruno Sack (43)
3) Vincent Blume (42)
4) Ingo Held (38)
5) Tom Seidel (36)
Alle Aussagen treffen so zu.
ROTER TEIL
Die Informationen aus dem Text trage ich in meine Liste der Platzierungen (numeriert von 1 bis 5) ein:
Platz 4 war 5 Jahre im Verein.
Der Held war nicht Erster - also war Ingo nicht Erster Vincent Blume war Zweiter.
Der 37jährige war nicht 4 Jahre im Verein gewesen - also war Ole nicht 4 Jahre im Verein
Wenn Ingo Erster oder Dritter war, muss er Dritter gewesen sein, denn Ingo Held war ja nicht Erster.
In Folge dessen war Ole Erster.
Wenn Ole nicht 4 Jahre im Verein war, dann war er 7 Jahre im Verein und Ingo dann 4 Jahre.
Also wissen wir bisher
1. Ole King (37), 7 Jahre
2. Vincent Blume (42),
3. Ingo Held (38), 4 Jahre
4. ?? , 5 Jahre
5. ??
Noch nicht verteilt: 10 Jahre und 8 Jahre - Tom Seidel und Bruno Sack.
Da Herr Sack 8 Jahre beim Verein war, muss er Fünfter sein. Folglich muss Vincent 10 Jahre beim Verein sein.
Verbleibt für Platz 4 nur noch Tom Seidel und Platz 5 für Bruno Sack, der 8 Jahre beim Verein ist.
Also abschließend:
1. Ole King (37), 7 Jahre
2. Vincent Blume (42), 10 Jahre
3. Ingo Held (38), 4 Jahre
4. Tom Seidel (36), 5 Jahre
5. Bruno Sack (43), 8 Jahre



Aufgabe 10

250. Wertungsaufgabe

"Dart ist eine Sportart, die in geschlossenen Räumen gespielt wird. Bei dem schönen Wetter aber möchte ich lieber eine Radtour machen". "Das geht mir genauso", gab Bernd Mike Recht. "Wie ist das eigentlich mit der Übersetzung bei so einem Fahrrad?" "Wie meinst du das?", fragte Lisa nach, die sich zu beiden gesellt hatte. "Nun, an meinem Moutainbike habe ich drei Kettenblätter vorn (22; 32 und 44 Zähne) und hinten sind es 9 Ritzel (11; 12; 14; 16; 18; 21; 24; 28 und 32 Zähne). Ich kann also 27 verschiedene Gänge (k;r) nutzen. Die Übersetzung ergibt sich aus k/r - (k - Zähne Kettenblatt geteilt durch r - Zähne am Ritzel)". "Ich glaube, du hast nicht Recht, dass alle Gänge - bezogen auf das Übersetzungsverhältnis - verschieden sind.", meinte Lisa. Wie viele echt verschiedene Gänge gibt es, wenn das Übersetzungsverhältnis auf eine Stelle nach dem Komma gerundet angegeben wird? - 5 blaue Punkte.
"Was hast du konstruiert?", fragte Mike bei Lisa nach. "Das ist mein vereinfachter Fahrradantrieb. Zwei Kreise (15 cm bzw. 8 cm Durchmesser), deren Mittelpunkte 60 cm entfernt sind. Ich frage mich nun, wie lang ist meine straff gespannte Fahrradkette." - 5 rote Punkte (Anmerkung: Länge der "Fahrradkette" - idealisiert als Linie um die Kreise.)

Lösung

Die Lösungen von Felix Karu (danke), ergänzt durch Anmerkungen: 250.pdf



Aufgabe 11

251. Wertungsaufgabe

Baseball"Siebenundzwanzig Gänge hast du also nicht wirklich an deinem Fahrrad," bemerkte Maria. "Ja, ja, du hast ja Recht", antwortete Mike. "Da sich die Sportserie dem Ende nähert, möchte ich mich noch einmal mit einem Spielfeld, dem Baseballfeld beschäftigen." "Na, dann mal los".
"Die Maße des Feldes sind nicht einheitlich und so verwende ich hier gerundete Meterangaben. Das Quadrat hat eine Kantenlänge von 27,5 m. Dieses befindet sich in einem Viertelkreis mit einem Radius von 39 m. Von H nach B sind es 97 m. A; B und I bilden einen Kreisausschnitt mit einem Radius von 121 m."
Wie groß (in m²) ist der grau gezeichnete Teil? - 4 blaue Punkte. Wie groß  (in m²) ist das eigentliche Spielfeld? 5 rote Punkte (Anmerkung: Das Spielfeld ist die Fläche AHB)

Lösung

blau: Der graue Teil ergibt sich aus der der Differenz der Flächeninhalte des Viertelkreises (Radius von 39 m) und des Quadrates (a = 27,5 m).
A = π· r² / 4 - a² = ... = 438,34 m²
Baseball-Lsgrot: Die gesuchte Spielfläche ist gleich der Differenz des Kreisausschnittes IAB und den Flächeninhalten der beiden Dreiecke IHB bzw. IHA.
AB ist die Diagonale des halben Quadrates AHBY (Y ist nicht eingezeichnet, wäre der Bildpunkt von H bei der Spiegelung an AB)
AX = XH = 1/2 · AB (Alles Eigenschaften des Quadrates AHBY)
AX = 1/2 · Wurzel (2) · AH = 1/2 · Wurzel (2) · 97 m
AX = 68,6 m
IX = √AI² - AX² = √121² - 68,6² = 99,68 m ==> HI = 30,1 m
∠ AIB lässt sich mittels Kosinussatz ausrechnen.
cos AIB = 2 · 121² - 137,2² /(2 · 121²) ==> ∠ AIB = 69,1°
Die Fläche des Kreisausschnittes AIB ergibt sich aus 69,1°/360° · π · 121² ==> 8828,69 m²
Das Flächeninhalt des Dreiecks HIB lässt mit 0,5 · HI · BI · ∠ (HIB) berechnen. ==> 1032,76 m²
Der gesuchte Flächeninhalt ist demzufolge 8825,4 m²- 2 · 1032,76 m² = 6759,87 m²
Durch die vielen Rundungen in dem Zwischenrechnungen kann es durchaus zu Abweichungen bei den Ergebnissen kommen. (Packt man alles in eine Formel liegt der Wert bei rund 6692 m² ;-)



Aufgabe 12

252. Wertungsaufgabe

Tennis"Die Baseballaufgabe war ja gar nicht so schwer, wobei der rote Teil es schon in sich hatte", fand Lisa. Mike stimmte ihr zu. "Dann lass uns ein letztes Spielfeld anschauen - das vom Tennis."
Wie lang sind alle Linien des Tennisfeldes zusammen? 4 blaue Punkte. (Die Mittemarkierungen (z.B. zwischen B und C) der Außenlinien zählen nicht.)
Rote Aufgabe. Es ist ein Rundweg von A nach A zu finden, der über alle Linien führt und so kurz wie möglich ist. (Buchstaben und Länge angeben.) Je kürzer desto mehr Punkte gibt es.
Für einen Beweis, dass es nicht mehr kürzer sein kann, gibt es extra Punkte.
Damit die Zahlenangaben leichter sind, können auch gern die Angaben in yard genommen werden (diese sind ja eigentlich die Grundlage).
6,40 m = 7 yd
8,23 m = 9 yd
10,97 m = 12 yd
11,89 m = 13 yd

Lösung

Schaut man sich das Bild genau an, so erkannt 4 senkrechte lange Begrenzungen. 4*2*13 yard und noch eine weitere senkrecht verlaufende Mittelline zu 2 * 9 yard.
Waagerecht sind es 3 Linien zu je 12 yard und noch 2 zu 7 yard.
Zusammen ergiebt das 172 yard.
Eine letztendliche Lösung liegt noch nicht vor, aus Zeitgründen muss die Begründung, ob der kürzeste Weg schon dabei war, noch etwas warten.
Danke schon mal bei den Mitmachenden:
Felix Karu:
A H L S P H I Q R O M I J N O C B E F J I E G K L D A 224 yard
Doreen N:
1. A I M N O R Q M N J K O R S L K G C D L K J F G C B E F J I H P Q M I E B A 2. A B E I H P Q M N O R S L K G C D L K O N J F G C B E F J I M Q R O K J I H A
In beiden Fällen sind es 222 yd.
Noch etwas kürzer die Variante Katrin P.: (von mir nachgerechet, 220 yard)
AH>HI>IQ>QP>PH>HI>IB>BD>DL>LK>KR>RQ>QR>RS>SL>LJ>JN>NO>OM>ME>EG>GF>FJ>JK>KC>CA Die Überlegungen von XXX

Ein idealer Rundweg läuft alle Wege genau einmal, also 172 y.

Die Weggabelungen B, C, E, F, G, H, L, M, N, O, Q, R sind von ungerader Parität, mithin muss jeweils ein zu ihnen führender Weg doppelt gegangen werden.

Wir suchen, anders ausgedrückt, sechs möglichst kurze Wege um immer zwei der 12 Punkte zu verbinden.

Wir suchen 6 kurze Wege zwischen diesen 12 Punkten („von außen rein“)

BE

CG

RO

QM

FN

HL

Summe

6

6

6

6

14

12

60

So haben wir einen Weg der Länge 172y + 60y = 232y:

 

A(BEB)(CGC)D(LHL)KG(FJNJF)EIMNOKLS(ROR)(QMQ)PHA

 

neuer Versuch mit den superkurzen EF usw.

EF

NO

BC

QR

HM

GL

Summe

4,5

4,5

9

9

8,5

8,5

45

Und nun ist der Weg 217y

A(BCB)(EFE)IJF(GKLKG)CDLS(RQR)(ONO)KJN(MIHIM)QPHA

 

im Detail

AB

1,5

BC.CB

18

BE

6

EF.FE

9

EI

7

IJ

4,5

JF

7

FG

4,5

GKL.LKG

17

GC

6

CD

1,5

DL

13

LS

13

SR

1,5

RQ.QR

18

RO

7

ON.NO

9

OK

7

KJ

4,5

JN

7

NM

4,5

MIH.HIM

17

MQ

6

QP

1,5

PH

13

HA

13

Weg

217

 

Einen dritten Versuch lasse ich sein. Ich habe schließlich die kürzesten Strecken (4,5y) zweimal, für H und L einen nahesten Punkt. Der Versuch für die Punkte an der Schmalseite eine bessere Verbindung zu erhalten, verschlechtert die guten Startbedingungen.



Auswertung Serie 21 (rote Liste)

Platz

Name

Ort

Summe

Aufgabe

 

241

242

243

244

245

246

247

248

249

250

251

252

1.

Doreen Naumann

Duisburg

66

6

4

4

2

16

7

3

8

6

3

1

6

2.

Katrin Posselt

Chemnitz

63

6

4

5

1

16

8

3

6

5

2

-

7

3.

Felix Karu

Innsbruck

56

-

-

-

2

16

8

3

6

6

5

5

5

4.

Sabine Fischbach

Hessen

53

6

2

5

1

12

6

3

8

6

1

2

1

5.

Anja Posselt

Chemnitz

41

6

4

5

-

-

8

3

6

-

2

-

7

6.

Torsten Zwingelberg

Pritzwalk

37

6

4

5

2

-

8

-

-

6

3

3

-

7.

Paula

Hartmannsdorf

28

6

-

-

-

12

-

-

-

-

5

5

-

8.

Andree Dammann

München

27

-

4

5

2

-

7

3

6

-

-

-

-

9.

Wadim Michaljow

Königs Wusterhausen

19

6

4

-

2

-

-

3

4

-

-

-

-

10.

Christian Wagner

Bamberg

12

6

-

-

-

-

-

-

-

6

-

-

-

10.

XXX

???

12

-

-

-

-

-

-

-

-

-

-

-

12

11.

Daniel Hufenbach

Chemnitz

6

6

-

-

-

-

-

-

-

-

-

-

-

11.

Kai-Lutz Wagner

Chemnitz

6

-

-

-

-

-

-

-

-

6

-

-

-

11.

Marie Neudert

Chemnitz

6

6

-

-

-

-

-

-

-

-

-

-

-

11.

Themos Käßler

Chemnitz

6

6

-

-

-

-

-

-

-

-

-

-

-

12.

Alice Ludewig

Chemnitz

5

-

4

-

1

-

-

-

-

-

-

-

-

13.

Richard Brinkel

Chemnitz

4

-

4

-

-

-

-

-

-

-

-

-

-

14.

Juliane Hansmann

Kiel

3

-

-

-

-

-

-

3

-

-

-

-

-

14.

Corinna Böhme

Chemnitz

3

3

-

-

-

-

-

-

-

-

-

-

-

15.

Duncan Mahlendorff

Chemnitz

2

-

-

-

-

-

-

2

-

-

-

-

-

16.

Robin Schmidt

Chemnitz

1

-

-

-

1

-

-

-

-

-

-

-

-

16.

Jakob Schreiter

Chemnitz

1

-

-

-

1

-

-

-

-

-

-

-

-

16.

Rafael Seidel

Chemnitz

1

-

-

-

-

-

-

-

-

-

-

1

-

16.

Max Geißler

Chemnitz

1

-

-

-

1

-

-

-

-

-

-

-

-

16.

Sophie Dani

Chemnitz

1

-

-

-

1

-

-

-

-

-

-

-

-

16.

Isabell Wache

Chemnitz

1

-

-

-

1

-

-

-

-

-

-

-

-

16.

Bernhard Richter

Chemnitz

1

-

-

-

1

-

-

-

-

-

-

-

-

17.

Christoph-Stefan Thibaut

Chemnitz

0

-

-

-

0

-

-

-

-

-

-

-

-

 

Auswertung Serie 21 (blaue Liste)

Platz

Name

Ort

Summe

Aufgabe

 

241

242

243

244

245

246

247

248

249

250

251

252

1.

Doreen Naumann

Duisburg

64

6

8

5

3

6

6

4

8

5

5

4

4

1.

Rafael Seidel

Chemnitz

64

6

8

5

3

6

6

4

8

5

5

4

4

2.

Sabine Fischbach

Hessen

62

5

8

5

3

6

6

4

7

5

5

4

4

3.

Katrin Posselt

Chemnitz

56

4

8

5

1

6

6

4

8

5

5

-

4

4.

Torsten Zwingelberg

Pritzwalk

52

6

8

5

3

6

6

-

-

5

5

4

4

5.

Anja Posselt

Chemnitz

50

4

8

5

-

6

6

4

8

-

5

-

4

6.

Felix Karu

Innsbruck

45

-

-

-

3

6

6

4

8

5

5

4

4

7.

Andree Dammann

München

40

-

8

5

3

6

6

4

8

-

-

-

-

8.

Linus-Valentin Lohs

Chemnitz

35

6

7

-

3

6

-

-

-

5

4

4

-

9.

Paula

Hartmannsdorf

21

6

-

-

-

6

-

-

-

-

5

4

-

9.

Wadim Michaljow

Königs Wusterhausen

21

6

8

-

3

-

-

4

-

-

-

-

-

10.

Hermann Thum

Chemnitz

16

-

-

-

3

-

6

-

-

-

-

4

3

11.

Ernesto Uhlmann

Chemnitz

15

-

8

3

-

-

-

-

-

-

-

-

4

12.

Judith Bergmann

Chemnitz

13

-

8

5

-

-

-

-

-

-

-

-

-

13.

Ronja Fischer

Chemnitz

12

-

8

4

-

-

-

-

-

-

-

-

-

14.

Christian Wagner

Bamberg

11

6

-

-

-

-

-

-

-

5

-

-

-

15.

Ellen Richter

Chemnitz

10

-

-

-

-

-

6

-

-

-

-

4

-

16.

Ria Hopke

Chemnitz

9

-

-

-

-

-

5

-

-

-

-

4

-

16.

Felix Taubert

Chemnitz

9

-

-

-

-

6

-

-

-

-

-

3

-

16.

Robin Schmidt

Chemnitz

9

-

-

-

3

6

-

-

-

-

-

-

-

17.

Alice Ludewig

Chemnitz

8

-

-

5

3

-

-

-

-

-

-

-

-

17.

Sophie Dani

Chemnitz

8

-

-

5

3

-

-

-

-

-

-

-

-

17.

Isabell Wache

Chemnitz

8

-

-

-

2

6

-

-

-

-

-

-

-

18.

Jakob Schreiter

Chemnitz

7

-

-

-

3

4

-

-

-

-

-

-

-

19.

Richard Brinkel

Chemnitz

6

6

-

-

-

-

-

-

-

-

-

-

-

19.

Loise Reichmann

Chemnitz

6

-

-

-

-

-

6

-

-

-

-

-

-

19.

Stephanie Dani

Chemnitz

6

-

-

-

-

-

6

-

-

-

-

-

-

19.

Corinna Böhme

Chemnitz

6

6

-

-

-

-

-

-

-

-

-

-

-

19.

Marie Neudert

Chemnitz

6

6

-

-

-

-

-

-

-

-

-

-

-

19.

Daniel Hufenbach

Chemnitz

6

6

-

-

-

-

-

-

-

-

-

-

-

20.

Matthias Keussen

Chemnitz

5

-

-

5

-

-

-

-

-

-

-

-

-

21.

Emil Maibier

Chemnitz

4

-

-

-

-

4

-

-

-

-

-

-

-

21.

Kai-Lutz Wagner

Chemnitz

4

-

-

-

-

-

-

-

-

4

-

-

-

21.

XXX

???

4

-

-

-

-

-

-

-

-

-

-

-

4

21.

Duncan Mahlendorff

Chemnitz

4

-

-

-

-

-

-

4

-

-

-

-

-

21.

Juliane Hansmann

Kiel

4

-

-

-

-

-

-

4

-

-

-

-

-

21.

Themos Käßler

Chemnitz

4

4

-

-

-

-

-

-

-

-

-

-

-

21.

Steffen Scheunpflug

Chemnitz

4

-

-

-

-

4

-

-

-

-

-

-

-

21.

Tim Jechorek

Chemnitz

4

-

-

-

-

-

-

4

-

-

-

-

-

22.

Marie Sophie Roß

Chemnitz

3

-

-

-

-

-

-

-

-

-

-

3

-

22.

Rico Bräutigam

Chemnitz

3

-

-

-

-

3

-

-

-

-

-

-

-

22.

Max Geißler

Chemnitz

3

-

-

-

3

-

-

-

-

-

-

-

-

22.

Benedikt Hastedt

Chemnitz

3

-

-

-

-

-

-

-

-

-

-

-

3

22.

David Kahle

Chemnitz

3

-

-

-

-

-

-

-

-

-

-

-

3

22.

Max Steinert

Chemnitz

3

-

-

-

-

-

-

-

-

-

-

-

3

22.

Ellen Wilde

Chemnitz

3

-

-

-

-

-

-

-

-

-

-

-

3

22.

Lukas Kirchberg

Chemnitz

3

-

-

-

3

-

-

-

-

-

-

-

-

22.

Bernhard Richter

Chemnitz

3

-

-

-

3

-

-

-

-

-

-

-

-

23.

Willy Stöckel

Chemnitz

2

-

-

-

-

-

-

-

-

-

-

-

2

23.

Phillpp Schleupner

Chemnitz

2

-

-

-

-

-

-

-

-

-

-

-

2

23.

Lisa Grassmann

Chemnitz

2

-

-

-

-

-

-

-

-

-

-

2

-

23.

Ingmar Richter

Chemnitz

2

-

-

-

-

-

-

-

-

-

-

-

2

24.

Christoph-Stefan Thibaut

Chemnitz

1

-

-

-

1

-

-

-

-

-

-

-

-

Valid XHTML 1.0 Transitional

Weitere Beiträge ...

  1. Serie-20
  2. Serie-19
  3. Serie-18
  4. Serie-17